ADS

Réussis tes devoirs et examens dès maintenant avec Quizwiz!

"C: ""injection of 200 units onabotulinumtoxinA in two months."" was the correct answer. The urodynamic study is consistent with detrusor overactivity. This patient has failed medical therapy so the next option, if he would like to improve his continence, would be bladder augmentation or onabotulinumtoxinA. Pelvic floor physical therapy would not be helpful in a patient with a complete spinal cord injury. Sacral nerve stimulation does not have an FDA indication for the treatment of neurogenic bladder. Artificial urinary sphincter would be indicated for stress urinary incontinence which would be unlikely with this level of injury and was not demonstrated on the urodynamic study. Doses of onabotulinumtoxinA should not exceed 360 units every 12 weeks. Since this patient was injected for his spasticity last month, he would be best treated in two months and not immediately. In addition, further treatment of both his detrusor overactivity and lower extremity spasticity with onabotulinumtoxinA should be coordinated between his urologist and physiatrist to eliminate the potential for over dosage over a period of time. Regarding the treatment of neurogenic bladder with onabotulinumtoxinA, 200 units is an efficacious starting point with the dose increased to 300 units for refractory cases. Studies comparing injections of onabotylinumtoxinA into the detrusor only, compared to a combined injection, with two-thirds of the volume injected into the detrusor and one-third of the volume injected into the trigonal areas, have found that the combination injection had better relief of symptoms, improved compliance, and no increased risk of vesicoureteral reflux compared to detrusor-only injections. Andersson KE, Wein AJ: Pharmacologic management of lower urinary tract storage and emptying failure, Wein, AJ, Kavoussi LR, Novick AC, Partin AW, Peters CA (eds): CAMPBELL-WALSH UROLOGY, ed 10. Philadelphia, Elsevier Saunders, 2012, vol 3, chap 68, p 2002. http://www.auanet.org/education/guidelines/prostate-cancer.cfm Gulanhusein A, Mangera A: OnabotulinumtoxinA in the treatment of neurogenic bladder. BIOLOGICS 2012;6:299-306. "

"A 24-year-old man with a T4 complete spinal cord injury has urinary incontinence despite CIC and maximal anticholinergic therapy. His urodynamic study is shown. He would like to be dry. He was injected with 300 units of onabotulinumtoxinA for lower extremity spasticity one month ago. The next step is: A. increase frequency of catheterization. B. immediate injection of 200 units onabotulinumtoxinA. C. injection of 200 units onabotulinumtoxinA in two months. D. sacral neuromodulation. E. artificial urinary sphincter."

"A: ""anti-inflammatories."" is correct. This patient is currently in the active phase of Peyronie's disease. The vacuum erection device and the penile traction device have not been adequately studied as primary treatment therapies during the acute phase of this disorder and should not be recommended at this time. The role of ESWL to treat Peyronie's disease has been shown to decrease penile pain but does not significantly improve the penile curvature or resolve the hourglass deformity. Intra-lesional collagenase is recommended to be used in patients in the active phase and with patients with a minimum of 30 degree curvature. Collagenase has not, however, been tested in patients with an hourglass deformity. Incision and grafting or plication should only be performed in patients in the quiescent phase. The best option at this time is anti-inflammatories and observation. Jordan GH, McCammon KA: Peyronie's disease, Wein, AJ, Kavoussi LR, Novick AC, Partin AW, Peters CA (eds): CAMPBELL-WALSH UROLOGY, ed 10. Philadelphia, Elsevier Saunders, 2012, vol 1, chap 28, p 802."

"A 47-year-old man has a dorsal penile nodule for three months. There is a painful hourglass deformity upon erection. The next step is: A. anti-inflammatories. B. vacuum erection device. C. intra-lesional collagenase. D. ESWL of penile plaque. E. plaque incision and grafting."

"D: ""temsirolimus."" was the correct answer. The patient has advanced RCC with predominantly non-clear cell histology. Additionally, she is now presenting with poor prognostic features. Prognostic scoring systems have been developed in advanced RCC and have been shown to have important correlations to response to therapies and to overall survival. The most commonly used prognostic model is from the Memorial Sloan Kettering Cancer Center and includes performance status, serum LDH, serum calcium, hemoglobin, and interval from diagnosis to treatment. Although the majority of clinical trials have focused on patients with clear cell histologies, data exists indicating that targeted therapies may have benefits in patients with non-clear cell histologies. According to the NCCN Kidney Cancer Panel, temsirolimus is a category 1 recommendation for the treatment of patients with non-clear cell histology and poor prognostic features. These recommendations by the panel are based primarily on the retrospective analysis of the global ARCC trial which evaluated temsirolimus in both clear cell and non-clear cell patients. In fact, this is one of the only phase III trials that included patients with non-clear cell histologies. Tyrosine kinase inhibitors (TKIs) (sunitinib, sorafenib) have been shown to have some effect in phase II trials in non-clear cell patients and are both are a category 2A recommendation by the NCCN panel. There are ongoing trials examining the potential benefits of pazopanib and axitinib as first-line therapies in non-clear cell patients, but data are limited and both drugs receive a category 2A recommendation based on extrapolation of results from other TKIs. Bevacizumab has been studied in a small phase II study of papillary renal cell carcinoma patients but the study was closed early due to slow accrual. The NCCN guideline panel has provided a category 2A recommendation for bevacizumab in these patients. The use of cytokine therapies (interferon and IL-2) have not been shown to have significant benefits in non-clear cell patients. In the aforementioned ARCC trial, temsirolimus was compared to interferon-alpha. Overall survival was significantly better in the temsirolimus subgroup, thereby, justifying the recommendation of temsirolimus. Interferon is not recommended by the NCCN panel. Srinivasan R, Linehan WM: Treatment of advanced renal cell carcinoma, Wein, AJ, Kavoussi LR, Novick AC, Partin AW, Peters CA (eds): CAMPBELL-WALSH UROLOGY, ed 10. Philadelphia, Elsevier Saunders, 2012, vol 2, chap 50, p 1490. Motzer RJ, Jonasch E, Agarwal N, et al: NCCN clinical practice guidelines in oncology: Kidney cancer (version 1.2015). http://www.nccn.org/professionals/physician_gls/pdf/kidney.pdf (accesses 10/21/2014). "

"A 65-year-old woman has recurrent metastatic RCC after undergoing radical nephrectomy for a pT4 papillary RCC. She has poor prognostic features by Memorial Sloan Kettering risk criteria. The next step is: A. interleukin-2. B. bevacizumab. C. sorafenib. D. temsirolimus. E. pazopanib."

"B: ""cremaster."" was the correct answer. The ilioinguinal nerve runs in the cremaster layer, and can be effectively spared by opening the cremaster and separating it from the remainder of the cord. The nerve does not have to be individually dissected. Chung BI, Sommer G, Brooks JD: Anatomy of the lower urinary tract and male genitalia, Wein, AJ, Kavoussi LR, Novick AC, Partin AW, Peters CA (eds): CAMPBELL-WALSH UROLOGY, ed 10. Philadelphia, Elsevier Saunders, 2012, vol 1, chap 2, pp 46-47. "

"During an inguinal approach to a pediatric hernia, the ilioinguinal nerve can be separated from the cord structures by entering the: A. external oblique fascia. B. cremaster. C. internal spermatic fascia. D. processus vaginalis. E. Scarpa fascia."

"D: ""may completely disappear after stopping furosemide."" was the correct answer. Nephrocalcinosis of infancy has been found in up to 64% of low birth weight (< 1500 g) or premature infants (gestational age of 32 weeks or less). Specifically, neonates who require oxygen for greater than four weeks of life, undergo dietary supplementation with high caloric formula (increased calcium and phosphate), and infants receiving loop diuretics are at the highest risk. The largest risk factor for nephrocalcinosis is in infants who have received loop diuretics (furosemide) to treat fluid overload, improve pulmonary oxygenation, and increase cardiac output. Indeed, 70% of the patients with nephrocalcinosis will have a history of prolonged loop diuretic exposure. The normal hypercalciuric effect of furosemide found in adults is enhanced in infants by the reduced glomerular filtration rate and the decrease hepatic function associated with infancy. The reduction in GFR and decreased systemic clearance of loop diuretics by the liver will significantly prolong the half-life of this drug, resulting in excessive hypercalciuria. In infants with clinically documented nephrocalcinosis, approximately 15% will develop renal calculi by five years of age. The calculi isolated from these patients are composed exclusively of calcium oxalate. The vast majority of these infants (85%) will completely resolve their nephrocalcinosis following stopping the use of loop diuretics. Although neonatologists will frequently replace the use of loop diuretics by using thiazides for fluid management issues in these critically ill infants, it is noteworthy that resolution of nephrocalcinosis has not been found to be significantly enhanced by the use of thiazides. Treatment of infants diagnosed with nephrocalcinosis should be with stoppage of the loop diuretics and interval follow-up with renal ultrasound until the nephrocalcinosis has resolved or alternatively renal calculi have been documented to develop and necessary treatment has been accomplished. There is no association of nephrocalcinosis of infancy to renal tubular acidosis. Increased dietary calcium and phosphate in the diet routinely found in infant high caloric formula supplements may at times actually be a cause of nephrocalcinosis of infancy and is, therefore, not indicated as treatment. Karlowitz MG, Adelman RD: Renal calcification in the first year of life. PED CLIN N AM 1995;42:1397-1413. Ferrandino MN, Pietrow PK, Preminger GM: Evaluation and medical management of urinary lithiasis, in Wein, AJ, Kavoussi LR, Novick AC, Partin AW, Peters CA (eds): CAMPBELL-WALSH UROLOGY, ed 10. Philadelphia, Elsevier Saunders, 2012, vol 2, chap 46, p 1321. "

"Nephrocalcinosis of infancy: A. should be treated by thiazide diuretics. B. results in a high incidence of renal calculi in childhood. C. is associated with distal renal tubular acidosis. D. may completely disappear after stopping furosemide. E. should be treated by increased calcium and phosphate in the neonatal diet."

"C: ""dietary purine excess."" was the correct answer. The most common cause of hyperuricosuria is excessive dietary purine intake. Gout, metabolic syndrome (obesity, hypertension, hyperlipidemia, and hyperglycemia) and myeloproliferative disorders are significant but less common risk factors. Decreased urinary volume and decreased citrate intake are not typical causes of hyperuricosuria. Excess Vitamin C intake is associated with hyperoxaluria. Pearle MS, Goldfarb DS, Assimos DG, et al: Medical management of kidney stones: AUA Guideline: AUA Guideline. American Urological Association Education and Research, Inc., 2014. http://www.auanet.org/education/guidelines/management-kidney-stones.cfm "

"The most common cause of hyperuricosuria is: A. gout. B. excess Vitamin C intake. C. dietary purine excess. D. decreased urinary volume. E. myeloproliferative disorders."

D New onset hydroureteronephrosis found after sphincter/sling placement in patients with a neurogenic bladder may be caused by bladder decompensation (detrusor noncompliance) that was not identified on pre-operative urodynamic studies. Provocative urodynamic studies (occlusion of the bladder outlet while filling), although once recommended to be performed prior to sphincter or sling placement has not been found to be highly accurate in determining the bladders ability to develop or lose compliance following an increase in outlet resistance, and is therefore of equivocal value. Persistent VUR and ureteral atony would not explain rapid decrease in renal function. Surgical obstruction of the ureters due to the sphincter placement would be seen in an acute setting. Six months following reimplantation, obstruction due to ureteral scarring would be unusual and would not be the most likely reason for the increased dilation seen in the upper tracts. Sphincter malfunction would result in incontinence. The most likely source of new onset hydronephrosis in this setting is either the development of detrusor noncompliance, or patient non-compliance with CIC and timed voiding intervals. Complete urodynamic evaluation and maintenance of a voiding-catheterization calendar will help delineate between the two possibilities. MacLellan DL, Bauer SB: Neuropathic dysfunction of the lower urinary tract, Wein AJ, Kavoussi LR, Novick AC, Partin AW, Peters CA (eds): CAMPBELL-WALSH UROLOGY, ed 10. Philadelphia, Elsevier Saunders, 2012, vol 4, chap 128, pp 3467-3469.

2014 - 108 A 12-year-old girl with myelodysplasia underwent placement of an artificial urinary sphincter and bilateral ureteral reimplantation six months ago. She has new bilateral hydroureteronephrosis and her serum creatinine has increased from 0.8 to 1.4 mg/dl. The likely explanation is: A. persistent VUR. B. chronic ureteral atony due to infection. C. surgical obstruction of the distal ureters. D. loss of bladder compliance. E. sphincter malfunction.

A With regards to prostatic intraepithelial neoplasia (PIN): 1) Low-grade PIN should not be documented in pathology reports due to poor interobserver reproducibility and a relatively low risk of cancer following re-biopsy, 2) The expected incidence of HGPIN on needle biopsy is between 5-8%, 3) Although the diagnosis of HGPIN is subjective, interobserver reproducibility for its diagnosis is fairly high among urological pathologists, 4) The median risk recorded in the literature for finding of cancer sometime in the patient's life following the diagnosis of HGPIN on needle biopsy is approximately 24%, 5) The majority of publications that compared the risk of cancer following a needle biopsy diagnosis of HGPIN to the risk of cancer following a benign diagnosis on needle biopsy show no differences between the two groups, 6) Clinical and pathological parameters do not help stratify which men with HGPIN are at increased risk for a future diagnosis of prostate cancer, 7) A major factor contributing to the decreased incidence of cancer following a diagnosis of HGPIN on needle biopsy in the contemporary era is related to increased needle biopsy core sampling, which detects many associated cancers on initial biopsy, such that re-biopsy following HGPIN, even with good sampling, does not detect many additional cancers, 8) It is recommended that men do not need routine repeat needle biopsy within the first year following the diagnosis of HGPIN. Further studies are needed to confirm whether routine repeat biopsies should be performed several years following a HGPIN diagnosis. In contrast, atypical small acinar proliferation (ASAP) is a small foci of glands that exhibit some features of adenocarcinoma, it is believed this diagnosis is frequently due to insufficient biopsy material to make the diagnosis of prostate cancer and therefore repeat biopsy is recommended. Regarding ASAP: 1) An average of 5% of needle biopsy pathology reports are diagnosed as having ASAP, 2) Cases diagnosed as ASAP have the highest likelihood of being changed to prostate cancer upon expert review and urologists should consider sending such cases for consultation in an attempt to resolve the diagnosis as definitively benign or malignant before subjecting the patient to repeat biopsy, 3) Ancillary pathologic techniques using basal cell markers and AMACR (alpha-methyl-acyl-coenzyme A racemase) can decrease the number of ASAP diagnoses. 4) The average risk of cancer on a subsequent biopsy following a diagnosis of ASAP within a six month time period is approximately 40%, 5) Clinical and pathological parameters do not help predict which men with an atypical diagnosis have cancer on repeat biopsy, 6) Repeat biopsy should include increased sampling of the initial atypical site, and adjacent ipsilateral and contralateral sites with routine sampling of all sextant sites. Therefore, it is critical for urologists to submit needle biopsy specimens in a manner in which the sextant location of each core can be determined, 7) All men with an ASAP diagnosis need re-biopsy within three to six months. It is critical for urologists to distinguish between a diagnosis of HGPIN and that of ASAP on needle biopsy. These two entities indicate different risks of carcinoma on re-biopsy and different recommendations for followup. Epstein JI, Herawi M: Prostate needle biopsies containing prostatic intraepithelial neoplasia or atypical foci suspicious for carcinoma: Implications for patient care. J UROL 2006;175:820-834.

2014 - 113 A repeat prostate biopsy after an initial finding of atypical small acinar proliferation (ASAP), as compared to high-grade prostatic intraepithelial neoplasia (HGPIN), has: A. a significantly higher risk of prostate cancer. B. a significantly higher risk of high-grade prostate cancer only. C. an equivalent risk of prostate cancer. D. a significantly lower risk of prostate cancer. E. a significantly lower risk of high-grade prostate cancer only.

C D-penicillamine is used in the management of patients with cystine stones. D-penicillamine is a chelating agent that produces complexes 50 times more soluble than cystine. Side effects include gastrointestinal disturbances, fever and rash, arthralgia, leukopenia, thrombocytopenia, proteinuria with nephrotic syndrome, and polymyositis. It can lead to pyridoxine (Vitamin B6) deficiency and supplementation is recommended with 50 mg/dl. Shekarriz B: Cystine stone disease. Stoller ML, Meng MV (eds): URINARY STONE DISEASE: THE PRACTICAL GUIDE TO MEDICAL AND SURGICAL MANAGEMENT. Totowa NJ, Humana Press, 2007, chap 18, p 340.

2014 - 118 Patients treated with D-penicillamine should receive supplemental: A. calcium. B. magnesium. C. Vitamin B6. D. Vitamin B12. E. iron.

D The descended testis is not hypertrophied (> 2 cm in length in a prepubertal boy) implying that the undescended testis is present and located in an abdominal position. None of the imaging studies would eliminate the need for diagnostic laparoscopy to exclude an intra-abdominal testicle. Scrotal exploration would be a reasonable option only if the descended testis was hypertrophied, as approximately 90-95% of the cases will have documented gonadal vessels in the inguinal canal. However, it should be noted that hypertrophy of the contralateral testis, if present, is neither perfectly sensitive nor specific for the presence of vanishing testis. Therefore, surgical exploration is indicated in all children with a nonpalpable testis, regardless of the size of the contralateral testis. In 5% of cases with unilateral hypertrophy, the testicle will have torsed intra-abdominally, and laparoscopy or retroperitoneal dissection will be necessary to visualize the blind-ending vessels proximal to the internal ring. There is no role for hormonal therapy to induce testicular descent due to its low efficacy. Barthold, JS: Abnormalities of the Testis and Scrotum and Their Surgical Management, in Wein AJ, Kavoussi, LR, Novick AC, Partin, AW, Peters, CA (eds): CAMPBELL'S UROLOGY, ed 10. Philadelphia, Saunders Elsevier, 2012, vol 4, chap 132, p 3565. Kolon TF, Herndon CDA, Baker LA, et al: Evaluation and Treatment of Cryptorchidism: AUA GUIDELINE. American Urological Association Education and Research, Inc, 2014. <a href="http://www.auanet.org/education/guidelines/cryptorchidism.cfm" target="_new"><u>http://www.auanet.org/education/guidelines/cryptorchidism.cfm</u></a>

2015 - 69 A six-month-old boy has a non-palpable left testis. The contralateral testis is descended and normal in size. The next step is: A. inguinal scrotal ultrasound. B. abdominal CT scan. C. left scrotal exploration. D. diagnostic laparoscopy. E. hCG treatment.

C Inverted papillomas are typically of two types (Type 1 and Type 2). Type 2 may exhibit a malignant behavior whereas Type 1 is benign. Unfortunately, they are histologically indistinguishable. Because of this, conservative treatment followed by surveillance for two years is recommended. Therefore no further treatment or follow-up is incorrect. Since it is possibly a benign lesion and presumably completely ablated by the laser, no additional treatment is warranted including antibiotic therapy. Sagalowsky AI, Jarrett TW, Flanigan RC: Urothelial tumors of the upper urinary tract and ureter, Wein AJ, Kavoussi LR, Novick AC, Partin AW, Peters CA (eds): CAMPBELL-WALSH UROLOGY, ed 10. Philadelphia, Elsevier Saunders, 2012, vol 2, chap 53, p 1521.

2014 - 21 A 50-year-old smoker with gross hematuria has a 1 cm left mid-ureteral filling defect on CT urography. The lesion is biopsied and laser ablated ureteroscopically. Histology reveals an inverted papilloma. The next step is: A. no further treatment or follow-up. B. long-term antibiotics. C. surveillance of the bladder and upper tracts. D. segmental ureterectomy. E. left nephroureterectomy.

E DDAVP, similar to ADH, exerts its effects on the collecting ducts to absorb water. DDAVP is often used for the treatment of nocturnal enuresis in children and polynocturia in adults. Side effects, although rare, do exist and may result in hyponatremia, headaches, and on extremely rare occasions, hyponatremic-induced mental confusion and seizures. If this medication is chronically administered, the urologist should consider obtaining intermittent serum electrolyte evaluations to observe for the development of hyponatremia. DDAVP should not be used to treat nocturnal polyuria in the adult with a history of CHF or renal insufficiency. Shoskes DA, McMahon AW: Renal physiology and pathophysiology, Wein AJ, Kavoussi LR, Novick AC, Partin AW, Peters CA (eds): CAMPBELL-WALSH UROLOGY, ed 10. Philadelphia, Elsevier Saunders, 2012, vol 2, chap 38, p 1031.

2015 - 87 DDAVP causes absorption of water by exerting its effects on: A. juxtaglomerular apparatus. B. distal tubule. C. macula densa. D. loop of Henle. E. collecting ducts.

D ( intracellular shifting of potassium. There is no evidence of adrenal insufficiency. Low magnesium levels can contribute to hypokalemia. There is no evidence that thiazides permanently damage the kidneys ability to handle potassium. The glomerulus does not play an important role in potassium homeostasis. When administering potassium for hypokalemia the rise in serum potassium is blunted because about 80% of the potassium enters the intracellular space.Shoskes DA, McMahon AW: Renal physiology and pathophysiology, Wein AJ, Kavoussi LR, Novick AC, Partin AW, Peters CA (eds): CAMPBELL-WALSH UROLOGY, ed 10. Philadelphia, Elsevier Saunders, 2012, vol 2, chap 38, p 1042. 2013 General Fluid & Electrolyte,Transplantation, Hypertension, Vasc Disease, Nephrology )

A patient taking chronic thiazide diuretics undergoes nephrectomy. After surgery, serum K is 3.0 mEq/l despite adequate fluid replacement. She is normotensive and alert. Attempts to restore her serum K level are hampered by: A) adrenal insufficiency. B) glomerular hyperfiltration. C) normal magnesium levels. D) intracellular shifting of potassium. E) tubular dysfunction.

E ( electrolyte absorption from large bowel. Genitourinary anomalies associated with an imperforate anus are common. The most common GU anomalies are; vesicoureteral reflux found in 40%, renal agenesis 25%, renal ectopy in 25%, rectourethral or rectal vaginal fistula in 20-30%, neurogenic bladder in 15- 25%, and a tethered spinal cord in 2-8%. The latter three, rectourethral fistula, neurogenic bladder, and tethered spinal cord are more frequently found in patients with a high or supra levator imperforate anus. When a rectourethral fistula exists, urine can flow into the colon leading to electrolyte resorption and recurrent UTIs even following placement of a diverting colostomy. If the electrolyte dysfunction is severe, consideration for a vesicostomy may be necessary to correct the acidosis. Intermittent catheterization in children with an active urethral rectal fistula is problematic, with the fistula acting as a false channel.Adams MC, Joseph DB: Urinary tract reconstruction in children, Wein AJ, Kavoussi LR, Novick AC, Partin AW, Peters CA (eds): CAMPBELL-WALSH UROLOGY, ed 10. Philadelphia, Elsevier Saunders, 2012, vol 4, chap 129, p 3479-3487. 2013 Pediatric Urinary Diversion ) $#$

A ten-day-old boy with a transverse colostomy performed for a high imperforate anus has a serum Cl of 115 mEq/l, Na of 145 mEq/l, K of 4.5 mEq/l, and CO2 of 17 mEq/l. The most likely explanation for these findings is: A) sepsis. B) renal dysplasia. C) severe hydronephrosis. D) neurogenic bladder dysfunction. E) electrolyte absorption from large bowel.

B ( objective demonstration of SUI. According to the 2009 Update for the Guideline on Surgical Management of Female Stress Urinary Incontinence, evaluation of the index patient should include the following components: focused history, focused physical examination, objective demonstration of SUI, assessment of PVR urine volume, urinalysis and urine culture if indicated. Additional diagnostic studies are not required but can be performed as needed and include pad testing, voiding diary, urodynamics, cystoscopy, and imaging.Appell RA, Dmochowski RR, Blaivas JM, et al: Guideline for the surgical management of female stress urinary incontinence: 2009 update. INCONTINENCE. American Urological Association Education and Research, Inc, 2009. http://www.auanet.org/content/guidelines-and-quality-care/clinical-guidelines/main-reports/stress2009/page 16 2013 Adult Neurogenic Bladder, Voiding Dysfunction, Incontinence )

According to the 2009 AUA Guidelines on Surgical Management of Female Stress Urinary Incontinence (SUI), evaluation of the index patient with SUI should include a focused history, physical examination, urinalysis, PVR and: A) pad test. B) objective demonstration of SUI. C) cystoscopy. D) urodynamics. E) urodynamics and cystoscopy.

96. A recurrent calcium oxalate stone former has a urinary calcium of 298 mg/24 hr (normal < 250 mg/24 hr). He is interested iin alternatives to traditional medical therapy. The next step is to recommend: A.. fish oil. B.. pyridoxine. C. cranberry. D.. Echinacea. E.. Vitamin E.

question #96 ANSWER=A Fish oil is an effective first-line therapy for mild-moderate hypercalciuria. Fish oils are rich in n-3 fatty acids more specifically eicosapentaenoic acid (EPA) which undergoes the same pathway of eicosanoid metabolism as the n-6 fatty acids found more commonly in Western diets. EPA is an essential dietary fatty acid as humans are unable to synthesize it from its precursor fatty acid linoleic acid. EPA is found mainly in such cold-water seafood as salmon mackerel tuna herring sardines bluefish trout whitefish and striped bass. Fish oils are sold in pill and liquid form as a source of n-3 fatty acids. EPA is thought to have a protective role in preventing nephrolithiasis by decreasing urinary calcium and oxalate excretion through alteration of prostaglandin metabolism. EPA competes with arachidonic acid for cyclooxygenase resulting in the formation of less PGE2. When PGE2 is inhibited urinary calcium excretion is reduced. Decreased PGE2 also leads to an activation of the nephron Na/K/2Ca transporter which results in increased renal calcium reabsorption. Greenland Eskimos a population which has an extraordinarily low incidence of renal stone disease consume approximately 5 to 10 gm of n-3 fatty acids daily. Most human clinical trials using fish oils have given 1200-1800 mg daily to their subjects. None of the other vitamins or supplements listed have any impact on calcium metabolism. Pearle MS Goldfarb DS Assimos DG et al: MEDICAL MANAGEMENT OF KIDNEY STONES: AUA GUIDELINE: AUA GUIDELINE. American Urological Association Education and Research Inc 2014. http://www.auanet.org/education/guidelines/management-kidney-stones.cfm

"D: ""triglycerides."" was the correct answer. The clinical presentation and CT scan is highly consistent with chylous ascites. Patients with postoperative chylous ascites classically have abdominal distention without significant pain or fevers and will have normal bowel habits. This complication results from disruption of the major para-aortic lymphatics channels leading to the cisterna chyli and is predominantly noted after left-sided procedures (i.e., radical and donor nephrectomy) or retroperitoneal lymphadenectomy. Chylous ascites is diagnosed by paracentesis with ascitic fluid found to have classically white and turbid appearance with fluid analysis showing elevated lymphocytes, associated with a high cholesterol and triglyceride content. Initial treatment is to reduce the flow of chyle into the lymphatics by a low-fat medium-chain triglyceride diet. If chylous ascites persist despite dietary management, the next step should involve bowel rest and the institution of total parenteral nutrition with the concurrent use of octreotide, a somatostatin analog. Somatostatin has been documented to significantly decrease postprandial increases in triglyceride levels by inhibiting lymphatic flow. Open or laparoscopic treatment of chylous ascites, using suture ligation and fibrin glue to control the leak, can be pursued if conservative management fails. Intraoperative location of the lymphatic leakage can be challenging and the combined use of preoperative lymphangiography and consumption of ""fatty"" meal immediately before surgery has been documented to be beneficial in helping the surgeon locate the site of the leak. Eichel L, Clayman RV: Fundamentals of laparoscopic and robotic urologic surgery, Wein, AJ, Kavoussi LR, Novick AC, Partin AW, Peters CA (eds): CAMPBELL-WALSH UROLOGY, ed 10. Philadelphia, Elsevier Saunders, 2012, vol 1, chap 9, pp 249-250. "

"A 25-year-old woman has mild abdominal pain and distension two weeks after left laparoscopic donor nephrectomy. She is afebrile with normal vital signs. CT scan is shown. Paracentesis is most likely to show elevated: A. bacteria. B. RBCs. C. creatinine. D. triglycerides. E. amylase."

"D: ""left radical orchiectomy."" was the correct answer. Intratubular germ cell neoplasia (ITGCN) is diagnosed by testicular biopsy performed for the investigation of infertility, by biopsy of the contralateral testis in a patient with germ cell tumor (GCT), or by biopsy within the affected testis in a patient undergoing testis-sparing surgery. The rationale for treatment of ITGCN is based on the high risk of developing invasive germ cell tumor in at least 50% of patients. Treatment options include, orchiectomy, low-dose radiotherapy, and close observation. The choice of therapy should be individualized based on the patient's desire for future paternity, the presence or absence of a normal contralateral testis, and the patient's desire to avoid testosterone replacement therapy. Radical orchiectomy is the most definitive, although low-dose radiotherapy (20 Gy) is associated with similar rates of local control with the prospect of preserving testicular endocrine function owing to the relative radioresistance of Leydig cells compared with germinal epithelium. For patients who do not desire future paternity, XRT is the preferred option, although radical orchiectomy and testosterone replacement therapy is a reasonable alternative. For patients with a normal contralateral testis who desire future paternity (as is the case with this patient who is pursuing evaluation for infertility), radical orchiectomy is preferred because scatter to the contralateral testis from radiotherapy may impair spermatogenesis. For patients with semen parameters that are abnormal but sufficient for assisted reproductive techniques, close surveillance with periodic ultrasound evaluation of the testis is a reasonable strategy with deferred therapy until successful pregnancy. Another option for these patients is exploration of the testis, sperm harvesting, and cryopreservation for assisted reproductive techniques, and radical orchiectomy followed by testosterone replacement therapy. Left varicocele repair alone does not address the concerns for malignant transformation of ITGCN. Single-dose carboplatin chemotherapy has been used in patients with stage I seminoma, but has no defined role in ITGCN. If no intervention is taken, close follow-up, as noted above, is still warranted. Stephenson AJ, Gilligan TD: Neoplasms of the testis, Wein, AJ, Kavoussi LR, Novick AC, Partin AW, Peters CA (eds): CAMPBELL-WALSH UROLOGY, ed 10. Philadelphia, Elsevier Saunders, 2012, vol 1, chap 31, pp 850-851. "

"A 31-year-old man undergoes bilateral testicular biopsy during evaluation for infertility. Left testicular biopsy reveals intratubular germ cell neoplasia (ITGCN). Scrotal ultrasound is normal except for a moderate left varicocele. The next step is: A. in vitro fertilization/intracytoplasmic sperm injection. B. left varicocele repair. C. low dose XRT to the left testis. D. left radical orchiectomy. E. single-dose carboplatin chemotherapy."

"C: ""potassium citrate."" was the correct answer. In this clinical scenario, the use of topiramate creates a chronic intracellular acidosis. This in turn creates a urinary milieu similar to distal renal tubular acidosis with hyperchloremic acidosis, high urine pH, extremely low urinary citrate, and hypercalciuria. Treatment may be potassium citrate or cessation of the medication if possible. Amiloride will reduce urine calcium when used in conjunction with thiazide but is not indicated in this situation. Allopurinol is the preferred treatment for hyperuricosuria. Calcium carbonate has no role in the medical management of kidney stones. Topiramate acts as a carbonic anhydrase inhibitor and addition of a similar agent would be counterproductive. Pearle MS, Goldfarb DS, Assimos DG, et al: Medical management of kidney stones: AUA Guideline: AUA Guideline. American Urological Association Education and Research, Inc., 2014. http://www.auanet.org/education/guidelines/management-kidney-stones.cfm "

"A 37-year-old woman with recurrent stones is taking topiramate for migraine headaches. To minimize the risk of stones, the next step is: A. amiloride. B. allopurinol. C. potassium citrate. D. calcium carbonate. E. a carbonic anhydrase inhibitor."

"D: ""percutaneous cyst drainage."" was the correct answer. Patients with autosomal dominant polycystic kidney disease can develop cyst infections. Symptoms and signs may include fever and flank or abdominal pain with normal urinalysis and sterile urine cultures. Therapy with lipid soluble antibiotics such as a fluoroquinolone agent is indicated if this diagnosis is suspected. Patients should defervesce within 72 hours. Persistent symptoms should prompt an abdominal CT scan to seek for infected cysts. Studies have shown that symptomatic infected cysts are typically larger than neighboring cysts, have thickened walls, are of higher attenuation, and enhance after contrast administration. This patient has not responded to appropriate antibiotic therapy. Therefore, the cyst should be drained and a percutaneous approach is preferred. Partial or simple nephrectomy is not indicated nor are additional antibiotics. A tagged indium-111 WBC scan can help localize the infection in patients without a dominant infected appearing cyst on CT scan. Pope JC IV: Renal dysgenesis and cystic disease of the kidney, Wein, AJ, Kavoussi LR, Novick AC, Partin AW, Peters CA (eds): CAMPBELL-WALSH UROLOGY, ed 10. Philadelphia, Elsevier Saunders, 2012, vol 4, chap 118, pp 3170-3176. Gupta S, Seith A, Sud K, et al: CT in the evaluation of complicated autosomal dominant polycystic kidney disease. ACTA RADIOL 2000;41:280-284. "

"A 43-year-old woman with autosomal dominant polycystic kidney disease has fever and left flank pain. She is treated with ciprofloxacin but after four days, she remains febrile. A CT scan demonstrates that one of the cysts in the left kidney is 7 cm in diameter and has a thickened wall which enhances after contrast administration. Her serum creatinine is 1.2 mg/dl and her current urinalysis is within normal limits. The next step is: A. continue treatment. B. add gentamicin. C. add vancomycin. D. percutaneous cyst drainage. E. partial nephrectomy."

"C: ""addition of metyrosine."" was the correct answer. In patients with a suspected pheochromocytoma, it is critical to achieve adequate alpha-blockade prior to surgery. This is typically accomplished with phenoxybenzamine. In patients in whom blockade with phenoxybenzamine is inadequate, the addition of metyrosine, a tyrosine hydroxylase inhibitor, has been recommended to prepare the patient for anesthesia or surgery. An alternative to metyrosine would be to continue the phenoxybenzamine and add a beta-blocker, but in this setting of inadequate alpha-blockade, the addition of a beta-blocker has on occasion been found to be associated with increased alpha-receptor stimulation and furthering hypertension. Therefore, the addition of metyrosine would be a better choice in patients who have their hypertension poorly-controlled by phenoxybenzamine. Weak alpha-blockers, such as prazosin, have been used, however, a majority of patients still appear to have hypertensive crises prior to or during surgery while on these drugs. Immediate surgical removal would be dangerous without pre-surgical medical stabilization which has not been accomplished in this patient. Clonidine and metyrapone do not play a role in the medical management of pheochromocytoma. Kutikov A, Crispen PL, Uzzo RG: Pathophysiology, evaluation, and medical management of adrenal disorders, Wein AJ, Kavoussi LR, Novick AC, Partin AW, Peters CA (eds): CAMPBELL-WALSH UROLOGY, ed 10. Philadelphia, Elsevier Saunders, 2012, vol 2, chap 57, pp 1703-1711. Michalakis K, Ilias I: Medical management of adrenal disease: A narrative review. ENDOCR REGUL 2009;43:127-135. Pacak K, Eisenhofer G, Ahlman H, et al: Pheochromocytoma: Recommendations for clinical practice from the First International Symposium. October 2005. NAT CLIN PRACT ENDOCRINOL 2007;3:92-102."

"A 50-year-old man with multiple endocrine neoplasia II has hypertension resistant to three antihypertensives. An MRI scan shows a 3 cm adrenal mass. A plasma-free metanephrine is elevated, and he is placed on phenoxybenzamine which is increased to the maximal dose. Despite this, he continues to have episodic hypertensive crises associated with sweating and palpitations. The next step is: A. switch phenoxybenzamine to metyrapone. B. addition of prazosin. C. addition of metyrosine. D. addition of clonidine. E. immediate surgical removal."

"A: ""intravesical BCG."" is correct. Immunocompromised patients are generally considered to be contraindicated to receive intravesical BCG therapy due to the theoretical concerns of loss of efficacy and increase in toxicity. However, there exists little evidence to support or refute such claims. A recent report from Herr and Dalbagni demonstrate the safety and efficacy of intravesical BCG in immunocompromised patients including patients with autoimmune disease, transplant patients, and patients undergoing systemic chemotherapy. The authors demonstrated that intravesical BCG is safe and effective in immunologically compromised patients with bladder cancer with the majority of patients (91%) having an initial complete response. Transplant patients fared worse but still demonstrated a response to BCG with acceptable tolerability. Therefore, an initial trial of BCG therapy can be considered in this patient following appropriate counseling about risks, benefits, and safety issues. Valrubicin was approved by the FDA in 1998 for the treatment of BCG refractory CIS of the bladder in patients who are medically unfit or refuse a cystectomy, with modest efficacy observed in this setting. It is not indicated in this setting and its role in the immunocompromised patient is uncertain. Although thiotepa is the only intravesical chemotherapeutic agent FDA-approved for papillary bladder cancer, it may result in significant myelosuppression which would be of particular concern in the immunocompromised patient. Furthermore, its efficacy in CIS is limited and it has not been studied in this setting. Intravesical gemcitabine has been shown to have activity in non-muscle invasive bladder cancer in high-risk patients. Although early results are promising in the second line or salvage setting, the limited patient population evaluated supports the need for additional phase II and phase III studies. Moreover, its efficacy in the immunocompromised patient is unknown. Radical cystectomy is a reasonable option, but may be too aggressive for this patient as a first line option with untreated CIS. Herr HW, Dalbagni G: Intravesical bacille Calmette-Guérin (BCG) in immunologically compromised patients with bladder cancer. BRI J UROL INT 2013;111:984-987. Hall MG, Chang SS, Dalbagni G, et al: Guideline for the management of nonmuscle invasive bladder cancer: (Stages Ta, T1, and Tis): 2007 Update. American Urological Association Education and Research, Inc., 2007. http://www.auanet.org/education/guidelines/bladder-cancer.cfm Jones JS, Larchian WA: Non-muscle-invasive bladder cancer (Ta, T1, and CIS), Wein, AJ, Kavoussi LR, Novick AC, Partin AW, Peters CA (eds): CAMPBELL-WALSH UROLOGY, ed 10. Philadelphia, Elsevier Saunders, 2012, vol 3, chap 81, pp 2343-2348."

"A 54-year-old woman is diagnosed with CIS of the bladder. She is currently taking methotrexate and prednisone for severe rheumatoid arthritis. The next step is: A. intravesical BCG. B. intravesical valrubicin. C. intravesical gemcitabine. D. taper off methotrexate and prednisone then administer BCG. E. radical cystectomy."

"A: ""hypertension."" is correct. Preoperative hypertension, especially uncontrolled hypertension, and bilateral application of SWL are risk factors for the development of perirenal bleeding. Patient size or weight, or stone size, location, and number do not correlate with this complication. Clinically significant perirenal hematomas occur in approximately 0.2-0.4% of patients undergoing SWL. Most such patients can be managed with surveillance, although a third may require transfusion. Matlaga BR, Lingeman JE: Surgical management of upper urinary tract calculi, Wein, AJ, Kavoussi LR, Novick AC, Partin AW, Peters CA (eds): CAMPBELL-WALSH UROLOGY, ed 10. Philadelphia, Elsevier Saunders, 2012, vol 2, chap 48, p 1395."

"The factor most likely to increase the risk of a perirenal hematoma after SWL is: A. hypertension. B. stone size > 2 cm. C. > 2,000 shocks. D. UTI. E. diabetes mellitus."

" B: ""increase urinary oxalate."" was the correct answer. Approximately 10-20% of dietary ascorbate is metabolized into oxalic acid that is then excreted into the urine. Ascorbic acid in doses of 2 grams daily was shown to increase urinary oxalate by 20-30%, but had no effect on urinary pH, citrate, calcium, or magnesium. Ferrandino MN, Pietrow PK, Preminger GM : Evaluation and medical management of urinary lithiasis, Wein, AJ, Kavoussi LR, Novick AC, Partin AW, Peters CA (eds): CAMPBELL-WALSH UROLOGY, ed 10. Philadelphia, Elsevier Saunders, 2012, vol 2, chap 46, p 1309. Pearle MS, Goldfarb DS, Assimos DG, et al: Medical management of kidney stones: AUA Guideline: AUA Guideline. American Urological Association Education and Research, Inc., 2014. http://www.auanet.org/education/guidelines/management-kidney-stones.cfm "

"Two grams of oral ascorbic acid (Vitamin C) per day will: A. decrease urinary calcium. B. increase urinary oxalate. C. decrease urinary magnesium. D. decrease urinary acidity. E. decrease urinary citrate."

B ( caput epididymis. Seminiferous tubules, organized into 200-300 conical tubules, drain into 20-30 tubuli recti which enter the rete testis, a network of ducts in the testicular mediastinum. At the upper end of the mediastinum, the ductuli efferentes emerge from the testis to enter the head (caput) of the epididymis. The cauda epididymis is the most distal portion of this organ from the testis.Chung BI, Sommer G, Brooks JD: Anatomy of the lower urinary tract and male genitalia, Wein AJ, Kavoussi LR, Novick AC, Partin AW, Peters CA (eds): CAMPBELL-WALSH UROLOGY, ed 10. Philadelphia, Elsevier Saunders, 2012, vol 1, chap 2, pp 67-68. 2013 General Congenital Anomalies, Embryology, Anatomy )

The ductuli efferentes of the testis enter the: A) seminiferous tubules. B) caput epididymis. C) cauda epididymis. D) vas deferens. E) rete testis.

C ( increased peak systolic velocity. Duplex ultrasound of the renal arteries is a useful noninvasive anatomic study for the diagnosis of renal artery stenosis (RAS). Although an altered flow pattern distal to the stenosis, including decreased diastolic flow and a turbulent systolic jet, can be suggestive of RAS, the most important single indicator is a peak systolic velocity (PSV) > 180 cm/sec. The renal aortic ratio (RAR) is the ratio of renal PSV to the aortic PSV. A RAR > 3.5 indicates > 60% stenosis. The renal resistive index does not directly assess renal artery flow.Fergany A, Novick AC: Renovascular hypertension and ischemic nephropathy, Wein AJ, Kavoussi LR, Novick AC, Partin AW, Peters CA (eds): CAMPBELL-WALSH UROLOGY, ed 10. Philadelphia, Elsevier Saunders, 2012, vol 2, chap 39, p 1061. 2013 General Fluid & Electrolyte,Transplantation, Hypertension, Vasc Disease, Nephrology )

The finding most suggestive of renal artery stenosis on duplex ultrasonography is: A) decreased diastolic flow. B) turbulent systolic flow. C) increased peak systolic velocity. D) renal aortic ratio < 3.5. E) resistive index < 0.8.

B ( sperm motility < 5%. Flagellar defects result in motilities of less than 5% - 10%. In these samples most of the non-motile sperm are viable. Most cases of ultrastructural flagellar defects are associated with normal sperm densities. Kallmann syndrome is associated with hypogonadotropic hypogonadism and resultant azoospermia not motility defects. Klinefelter syndrome is associated with azoospermia, not motility defects. Antisperm antibodies may result in low motility but the non-motile sperm are non-viable.Chemes HE, Rawe VY: Sperm pathology: A step beyond descriptive morphology. Origin, characterization and fertility potential of abnormal sperm phenotypes in infertile men. HUMAN REPRODUCTION UPDATE, 2003 vol 9, pp 405-428.Sabanegh E, Agarwal A: Male infertility, Wein AJ, Kavoussi LR, Novick AC, Partin AW, Peters CA (eds): CAMPBELL-WALSH UROLOGY, ed 10. Philadelphia, Elsevier Saunders, 2012, vol 1, chap 21, p 622. 2013 Adult Sexual Dysfunction, Endocrinopathy, Fertility Problems )

The indication for sperm viability testing is: A) sperm density < 5 million sperm per ml. B) sperm motility < 5%. C) Kallmann syndrome. D) Klinefelter syndrome. E) presence of anti-sperm antibodies.

C ( reduce lumen pressure. Early studies of orthotopic reconstruction by Camey and others demonstrated that while continence can be achieved utilizing a tubularized segment of bowel as a urinary reservoir, upper tract deterioration inevitably occurred due to high reservoir pressure. In part, the pressure in such segments is due to the continued organized bowel peristalsis, and the reduced diameter of the lumen. The observation that reservoir pressure is lowered by increasing the internal radius of the reservoir is based upon Lapace's law that mural tension is equivalent to pressure times radius cubed. While the increased radius achieved by detubularization and folding of the bowel also results in increased reservoir capacity, the primary goal of detubularization is reduction in pressure through reduction of organized peristalsis and wall tension. Although this may indirectly lead to reduced ureteral reflux, it is not the primary reason for bowel detubularization.Skinner EC, Skinner DG, Stein JP: Orthotopic urinary diversion, Wein AJ, Kavoussi LR, Novick AC, Partin AW, Peters CA (eds): CAMPBELL-WALSH UROLOGY, ed 10. Philadelphia, Elsevier Saunders, 2012, vol 3, chap 87, p 2479. 2013 Adult Urinary Diversion )

The primary goal of bowel detubularization at the time of neobladder construction is to: A) recreate the shape of the native bladder. B) provide more efficient neobladder emptying. C) reduce lumen pressure. D) reduce vesicoureteral reflux. E) reduce urinary contact time.

109. A 67-year-old woman is on active surveillance for a 3 cm renal mass. Initial percutaneous biopsy demonstrated oncocytoma. Her first follow-up imaging at three months shows no change. The next step is annual history physical exam and: A.. observation. B.. repeat biopsy at one year. C. annual abdominal imaging (ultrasound or CT or MRI scan). D.. annual abdominal imaging (ultrasound or CT or MRI scan) and chest X-ray. E.. repeat biopsy at one year and annual abdominal imaging (ultrasound or CT or MRI scan).

question #109 ANSWER=D Patients with an oncocytoma or small renal tumors with indeterminate histology should be followed with the same imaging protocols used for untreated low risk (cT1 NO Nx) renal cancer patients. This recommendation for benign tumor follow-up is based on two concerns: 1) benign tumors can exhibit substantial growth patterns over time that may threaten destruction of the renal unit by compression/invasion of surrounding parenchyma and vascular structures; 2) although the accuracy of percutaneous biopsy has improved substantially in the past several years the pathologic differentiation between oncocytoma and oncocytic neoplasms (e.g. chromophobe renal cell carcinoma) and renal cell carcinoma can at times be difficult with the true pathology of the mass only coming to attention by rapid tumor growth. The purpose of routine imaging of these benign neoplasms is therefore to capture undue tumor growth and allowing for expedient surgical/ablative intervention and avoidance of radical nephrectomy. AUA Guidelines for the follow-up of renal cancers and untreated low-risk tumors including oncocytoma include: 1) history and physical examination; 2) basic laboratory testing to include blood urea nitrogen (BUN)/creatinine urine analysis (UA) and estimated glomerular filtration rate (eGFR); 3) continued renal imaging (US CT or MRI scan) at least annually and annual chest X-ray (CXR) to assess for pulmonary metastases. Repeat biopsy is not indicated or warranted for the routine follow-up of such patients as therapeutic intervention is based on subsequent neoplasm growth rate. Donat SM Chang SS Bishoff JT et al: FOLLOW-UP FOR CLINICALLY LOCALIZED RENAL NEOPLASMS: AUA GUIDELINE. American Urological Association Education and Research Inc 2013. http://www.auanet.org/ed ucation/g u idel i nes/renal-cancer-fol low-u p.cfm

114. A 52-year-old man has new onset bright red blood in his nephrostomy tube three days after right PCNL. He is hemodynamically stable; however his hemoglobin has decreased from 13 mg/dl to 10 mg/dl. CT scan reveals a right perinephric hematoma. The next step is: A.. bed rest and serial hemoglobin levels. B.. nephrostomy tamponade catheter. C. inject fibrin glue. D.. nephroscopy and fulguration. E.. selective angioembolization.

question #114 ANSWER=E Delayed bleeding following percutaneous nephrolithotomy indicates the presence of an arteriovenous fistula or arterial pseudo aneurysm. New onset bright red blood favors an arterial pseudo aneurysm. The next best step is selective angioembolization. The pseudo aneurysm is at risk to bleed so bed rest is not appropriate in this scenario. A nephrostomy tamponade catheter and/or fulguration is most effective for immediate bleeding following the nephrolithotomy procedure. Renal exploration is reserved for recalcitrant bleeding if angioembolization fails or life threatening bleeding ensues. Wolf JS Jr: Percutaneous approaches to the upper urinary tract collecting system in Wein AJ Kavoussi LR Partin AW Peters CA (eds): CAMPBELL-WALSH UROLOGY ed 11. Philadelphia Elsevier 2015 vol 1 chap 8 p 153.

120. An 81-year-old man has muscle-invasive urothelial carcinoma of the bladder with multifocal CIS. Metastatic evaluation is negative. GFR is 48 mUmin/1.7 m2 • The next step is: A.. chemoradiation therapy. B.. cisplatin-based chemotherapy followed by radical cystectomy. C. carboplatin-based chemotherapy followed by radical cystectomy. D.. radical cystectomy. E.. radical cystectomy followed by adjuvant chemotherapy.

question #120 ANSWER=D The patient presents with muscle-invasive bladder cancer (cT2) with a notable history of renal insufficiency and advanced age. Nevertheless he remains a candidate for radical cystectomy and this should be the primary recommendation. Comorbidities not age should be used when deciding on radical cystectomy and surgery can and should be considered for this patient as multiple series have demonstrated benefits of radical cystectomy in elderly patients with invasive disease. Multifocal CIS is a contraindication for chemoradiation therapy alone. Neoadjuvant chemotherapy is intended for patients with operable clinical stage T2 to T4a muscle-invasive disease. Although the data available supports the use of either M-VAC (Methotrexate Vinblastine Adriamycin and Cisplatin) or CMV (Cisplatin Methotrexate and Vinblastine) as neoadjuvant chemotherapy it has been estimated that more than 50% of patients are ineligible for cisplatin based chemotherapy because of impaired renal function or medical comorbidities. This patient has impaired renal function that will preclude the use of cisplatin-based neoadjuvant chemotherapy. In patients with compromised renal function carboplatin + gemcitabine has been utilized. However the efficacy of carboplatin-based regimens in the neoadjuvant setting is unproven and may contribute to a delay in definitive surgery without a known oncologic benefit. Adjuvant chemotherapy has been advocated for high-risk patients (pT3-4 N+ patients) in an effort to delay recurrence and prolong survival. Unfortunately the ??question of the true benefit of adjuvant chemotherapy in high-risk patients with pT3 pT4 and N+ disease is currently unknown. Although this patient may be a candidate for adjuvant chemotherapy the decision to pursue such an approach will depend on his pathologic staging and is therefore not a foregone conclusion. Guzzo TJ Vaughn DJ: Management of metastatic and invasive bladder cancer in Wein AJ Kavoussi LR Partin AW Peters CA (eds): CAMPBELL-WALSH UROLOGY ed 11. Philadelphia Elsevier 2015 vol 3 chap 94 p 2230. Wosnitzer MS1 Hruby GW Murphy AM et al: A comparison of the outcomes of neoadjuvant and adjuvant chemotherapy for clinical T2-T4aN0-N2M0 bladder cancer. CANCER 2012;118:358-364. Copyright 2017 by The American Urological Association. 43

129. A 19-year-old man has pulmonary function tests showing decreased diffusion capacity for carbon monoxide (DLCO) with a CT scan showing diffuse areas of ground-glass opacity after his second cycle of BEP chemotherapy. Prior to his next cycle of chemotherapy the next step is to: A.. discontinue bleomycin. B.. repeat pulmonary function tests. C. administer prophylactic hydrocortisone. D.. administer granulocyte colony stimulating factor. E.. change to vinblastine ifosfamide and cisplatin (VIP) chemotherapy.

question #129 ANSWER=A The side effects of cytotoxic chemotherapy for metastatic testicular cancer are well-established as BEP chemotherapy has been a mainstay regimen for decades. The major toxicity of cisplatin is neuropathy while the most major side effect of etoposide is bone marrow suppression. Bleomycin has pulmonary toxicity risks that have also been well-documented with an incidence of approximately 10%. Pulmonary toxicity is dose-dependent and thus increasing in severity with more cycles of BEP. The reported fatal pulmonary toxicity after three cycles of BEP is <1 % and after four cycles is 1-2%. In patients with known bleomycin pneumonitis during chemotherapy the typical maneuver would be to discontinue the bleomycin at the next cycle. There is no clear evidence that decreasing the dose of bleomycin affords the same efficacy and additionally still exposes a patient with known bleomycin-induced pneumonitis to the drug. Administration of prophylactic steroids do not counteract the possible bleomycin exposure and thus are not indicated. Additionally repeating pulmonary function tests while possibly giving additional information on pulmonary function are not necessary before the next cycle. Giving granulocyte colony stimulating factor (GCSF) will not affect pulmonary function. Lastly a change of chemotherapy is not indicated except for excluding the bleomycin. Montgomery RB Lin DW: Toxicities of chemotherapy for genitourinary malignancies in Taneja SS (ed): COMPLICATIONS OF UROLOGIC SURGERY ed 4. Philadelphia Elsevier 2010 chap 10 p 119.

130. A 17-year-old boy with cystinuria has a unilateral staghorn calculus. The next step is PCNL and: A.. D-penicillamine. B.. potassium citrate. C. hydration with 1.5 L intake daily. D.. acetohydroxamic acid. E.. Renacidin® irrigation.

question #130 ANSWER=B Cystinuria is an inherited autosomal recessive disorder characterized by excessive urinary excretion of cystine. Average age at first stone diagnosis is 12.2 years. The most important factors in preventing cystine stone formation in the urinary tract are lowering urinary cystine concentration and maintaining an alkaline urinary pH. Adequate hydration with fluid intake of 3-4 L per day with waking at night to imbibe is recommended. Potassium citrate alkalinizes the urine and increases the solubility of cystine in the urine. Renacidin irrigation of the renal collecting system is indicated for apatite (calcium carbonate) or struvite stones. Acetohydroxamic acid enhances efficacy of antibiotic therapy directed toward U TI caused by urea-splitting organisms. Penicillamine and alpha-mercaptopropionlglycine (alpha-MPG are chelating agents that combine with cystine to increase solubility and are used when hydration and alkalinization therapies fail. Penicillamine has a higher rate of adverse reactions compared to alpha-MPG. Lipkin ME Ferrandino MN Preminger GM: Evaluation and medical management of urinary lithiasis in Wein AJ Kavoussi LR Partin AW Peters CA (eds): CAMPBELL-WALSH UROLOGY ed 11. Philadelphia Elsevier 2015 vol 2 chap 52 p 1229. Copyright 2017 by The American Urological Association. 47

147. A 60-year-old man is scheduled for a retroperitoneal laparoscopic radical nephrectomy. Following balloon dilation of the retroperitoneal space a standard 12 mm trocar is inserted and secured to the fascia. During the operation he develops subcutaneous crepitus and the end tidal CO2 gradually climbs. The most likely cause is: 46 A.. B .. C. D .. E .. occult pulmonary bleb disease. dislodgement of the trocar. gas leakage around the trocar and fascia. accidental entry into the peritoneal cavity. entry into a venous sinus.

question #147 ANSWER=C The use of a conventional Hasson trocar following balloon dilation in retroperitoneal laparoscopic procedures is often fraught with unique challenges including leakage of gas around the trocar despite securing fascial sutures to the cone portion of the Hasson trocar. This occurs as the initial fascial incision is often larger than the size of the 12 mm trocar allowing for diffusion of CO2 gas around the trocar and into the subcutaneous space. Subsequent systemic absorption of the subcutaneous gas results in hypercarbia. With the advent of balloon access trocars this problem is less common than with standard 12 mm trocars that may not form as tight a seal against the fascia. The retention doughnut-shaped balloon placed on the inside of the fascia and peritoneum is secured against a foam cuff on the outside of the fascia creating a Copyright 2017 by The American Urological Association. 53 tight seal thus minimizing gas leakage around the trocar. This patient is healthy and has no history of underlying pulmonary disease. A rupture of an occult pulmonary bleb would result in a pneumothorax but not subcutaneous emphysema. Entry into the peritoneal cavity would not necessarily increase the risk of hypercarbia. Lastly there is no mention of bleeding or entry into a large vascular structure that would lead one to suspect venous absorption of gas. Ordon M Eichel L Landman J: Fundamentals of laparoscopic and robotic urologic surgery in Wein AJ Kavoussi LR Partin AW Peters CA (eds): CAMPBELL-WALSH UROLOGY ed 11. Philadelphia Elsevier 2015 vol 1 chap 10 p 201.

30. A 25-year-old man sustains perinea! trauma and a pelvic fracture. A retrograde urethrogram shows contrast in the upper thigh. The initial tissue plane the contrast passed through to reach the thigh is: A.. Buck's fascia. B.. Colles' fascia. C. fascia lata. D.. external spermatic fascia. E.. dartos fascia.

question #30 ANSWER=A It is unusual for extravasation of urine to reach the thigh. This suggests that the normal layers that more commonly contain urinary extravasation have been disrupted. The first layer that has to be disrupted for urine to reach the thigh must be Buck's fascia. The dartos fascia Colles' fascia and insertion of the fascia lata represent a continuation of the same fascial layer. Chung Bl Sommer G Brooks JD: Surgical radiographic and endoscopic anatomy of the male pelvis in Wein AJ Kavoussi LR Partin AW Peters CA (eds): CAMPBELL-WALSH UROLOGY ed 11. Philadelphia Elsevier 2015 vol 3 chap 68 p 1612.

38. A 54-year-old woman with a history of cervical cancer treated with radiation therapy five years ago undergoes a TUR of a 2 cm mass above the left ureteral orifice. Final pathology reveals an inverted papilloma. On the third post-operative day she develops continuous urinary incontinence. CT urogram reveals no evidence of upper tract pathology perivesical abscess or urinoma. Subsequent cystoscopic evaluation reveals a 2 cm vesicovaginal fistula at the site of the resection. The next step is: A.. cauterization of the fistula site and placement of a urethral catheter. B.. immediate transvaginal repair. C. transvaginal repair in three months. D.. immediate transabdominal repair. E.. transabdominal repair in three months.

question #38 ANSWER=D In a woman presenting with the acute onset of chronic incontinence following a surgical procedure she should be evaluated for possible simultaneous upper UTI and ureteral or combined ureteral-vaginal fistulas to rule-out the presence of a perivesical abscess or fluid collection. In the absence of pelvic infection immediate repair is justified. In the presence of a large diameter vesicovaginal fistula in an irradiated field an abdominal approach will concurrently allow an omental pedicle flap to be interposed between the irradiated bladder and vaginal wall tissues. In the presence of an irradiated field obliteration of dead space good Copyright 2017 by The American Urological Association. 13 bladder drainage control of infection and interposition of healthy tissue are critical elements to successful fistula closure. Proximal urinary diversion with bilateral percutaneous nephrostomy tubes with delayed repair should be considered in patients where the initial evaluation suggests the presence of a concurrent pelvic abscess. An endoscopic approach with fulguration of the fistula tract and urethral catheter or suprapubic tube drainage may be considered in vesicovaginal fistulas where the diameter of the fistula is < 5 mm in size and radiographic evaluations (fistulogram) suggests the presence of a long-necked and tortuous fistula. Badlani GH De Ridder DJMK Mettu JR Rovner ES: Urinary tract fistulae in Wein AJ Kavoussi LR Partin AW Peters CA (eds): CAMPBELL-WALSH UROLOGY ed 11. Philadelphia Elsevier 2015 vol 3 chap 89 pp 2111-2112.

68. A 24-year-old man has stage 1 pure seminoma without vascular invasion. He is reluctant to undergo adjuvant XRT. An alternative to observation is one cycle of: 24 A.. B .. C. D .. E .. paclitaxel. etoposide. bleomycin. carboplatin. ifosfamide.

question #68 ANSWER=D The success of chemotherapy for high-stage seminoma has led investigators to examine its use in low-stage disease. Its use has been supported by several non-randomized trials. In addition the Medical Research Council recently completed a randomized clinical trial comparing carboplatin with standard retroperitoneal irradiation in the setting of stage 1 seminoma. The two treatments 24 Copyright 2017 by The American Urological Association. had similar efficacy. Therefore for stage 1 seminoma the correct chemotherapy agent is carboplatin. Stephenson AJ Gilligan TD: Neoplasms of the testis in Wein AJ Kavoussi LR Partin AW Peters CA (eds): CAMPBELL-WALSH UROLOGY ed 11. Philadelphia Elsevier 2015 vol 1 chap 34 pp 808- 809.

77. A SO-year-old man is diagnosed with an asymptomatic 15 mm stone in an anterior upper pole caliceal diverticulum. The best management is: A.. observation. B.. SWL. C. ureteroscopy laser incision of diverticular neck and lithotripsy. D.. PCNL and dilation of the diverticular neck. E.. laparoscopic removal of stone and ablation of diverticulum.

question #77 ANSWER=A If the patient is asymptomatic no immediate treatment is necessary. There is evidence that a 24- hour urine collection will be normal as the stone is most likely secondary to urinary stasis. SWL ureteroscopic and percutaneous approaches are not necessary unless the patient becomes symptomatic. Indications for intervention would include pain and recurrent UTls. In this case if he was symptomatic both ureteroscopic and laparoscopic approaches would be reasonable for a caliceal diverticulum in an anterior location. Copyright 2017 by The American Urological Association. 27 Leavitt DA de la Rosette JJMCH Hoenig DM: Strategies for nonmedical management of upper urinary tract calculi in Wein AJ Kavoussi LR Partin AW Peters CA (eds): CAMPBELL-WALSH UROLOGY ed 11. Philadelphia Elsevier 2015 vol 2 chap 53 p 1243.

95. A 40-year-old man involved in an MVC has mild lower abdominal pain and gross hematuria. Radiologic evaluation reveals a normal urethra a pelvic fracture with a large pelvic hematoma and a small 1-2 cm extraperitoneal extravasation from the bladder. A 26 Fr urethral catheter repetitively clots off. The next step is: A.. place three-way urethral catheter and begin continuous bladder irrigation. B.. percutaneous suprapubic tube placement. C. cystoscopy with clot evacuation. D.. extraperitoneal exploration of bladder repair of laceration and catheter drainage. E.. intraperitoneal exploration of bladder repair of laceration and catheter drainage. Copyright 2017 by The American Urological Association. 33

question #95 ANSWER=E Bladder rupture should be suspected in patients with pelvic fractures. Bladder injury may be intraperitoneal or extraperitoneal. Small extraperitoneal lacerations may be managed nonoperatively with urethral catheter drainage and antibiotics. Contraindications to conservative management include gross hematuria with repetitive clot retention concomitant rectal or vaginal injury bladder neck injury the presence of a foreign body in the bladder (such as a piece of bone or a bullet) or injury due to a gunshot. This patient has clot retention and therefore should be managed by surgical repair. The injury should be approached by an intraperitoneal approach in those with pelvic hematomas. Entering the hematoma through an extraperitoneal approach risks release of a contained pelvic hematoma and significant hemorrhage. Continuous bladder irrigation is contraindicated in the presence of a bladder rupture since the fluid will extravasate increasing the risk of pelvic abscess and pelvic osteomyelitis. The placement of a percutaneous suprapubic tube will not correct the injury and may be difficult and risky in the presence of a pelvic hematoma. Cystoscopy with clot evacuation will not correct the injury. Copyright 2017 by The American Urological Association. 33 Morey AM Brandes S Dugi Ill DD et al: UROTRAUMA: AUA GUIDELINE. American Urological Association Education and Research Inc 2014. http://www.auanet.org/education/guidelines/urotrauma.cfm

97. Ir ·' · · nt whose external genitalia are shown the arrows denote the urethral meatus. The embryologic origin of the structure that may cause difficulty with catheterization is: 34 A.. B .. C. D .. E .. Mullerian duct. mesonephric duct. urorectal septum. urogenital sinus. urethral plate.

question #97 ANSWER=A A prostatic utricle is present in 10-15% of boys with a proximal hypospadias and is the most common cause of difficulty in catheterizing the bladder with proximal hypospadias. The embryologic origin of the utricle is the paramesonephric duct or Mullerian duct. Remnants of the mesonephric duct give rise to Gartner's cyst whereas the mesonephric duct gives rise to the vas deferens and seminal vesicle. The urorectal septum divides the common cloaca into the hindgut and urogenital sinus. The urethral plate gives rise to the urethra and prostate from the urogenital sinus. A narrow urethral plate does not cause difficulty with catheterization. Park JM: Embryology of the genitourinary tract in Wein AJ Kavoussi LR Partin AW Peters CA (eds): CAMPBELL-WALSH UROLOGY ed 11. Philadelphia Elsevier 2015 vol 4 chap 122 p 2839.

"D: ""transurethral incision of PUV."" was the correct answer. The current VCUG image is concerning for the persistence of (residual) PUV tissue with persistence of a dilated posterior urethra and a filling defect noted at the transition point from the prostatic to membranous urethra. Due to these findings, cystourethroscopy and repeat evaluation for possible repeat transurethral incision of PUV is mandatory. He may at some point benefit from biofeedback, oxybutynin, or CIC; however, these treatment modalities should not be employed until after there is documentation of valvular destruction. Radiographic studies do document bilateral bladder diverticula; however, these may or may not be clinically relevant to his voiding dysfunction and their treatment should only be pursued following documentation of valve destruction and failure of conservative treatment modalities. Casale AJ: Posterior urethral valves, Wein, AJ, Kavoussi LR, Novick AC, Partin AW, Peters CA (eds): CAMPBELL-WALSH UROLOGY, ed 10. Philadelphia, Elsevier Saunders, 2012, vol 4, chap 126, p 3397. Frimberger DC, Kropp BP: Bladder anomalies in children, Wein, AJ, Kavoussi LR, Novick AC, Partin AW, Peters CA (eds): CAMPBELL-WALSH UROLOGY, ed 10. Philadelphia, Elsevier Saunders, 2012, vol 4, chap 125, p 3384. "

"A 12-year-old boy has daytime urinary incontinence five years following transurethral incision of PUV. The preoperative VCUG and the current VCUG images are shown. The next step is: A. CIC. B. oxybutynin. C. biofeedback. D. transurethral incision of PUV. E. diverticulectomy."

"A: ""observation."" is correct. Bagged collection of urine for evaluation in infants is notoriously non-specific for true urinary infection. The usual conundrum is that a bagged specimen will include many bacteria which were perineal and not within the urinary tract; this leads to many false positive results. However, when a bagged specimen shows no growth on culture, it can be seen as being a strong indicator with a > 99% probability that no urinary infection is present. Basically, a bagged culture is helpful to rule out infection if negative, but not very helpful to rule in infection; indeed studies have revealed that if the bag specimen is positive for > 100,000 colonies of a single organism, up to 80% of the specimens will be false positive compared to simultaneously obtain catheterized urine specimen. Leukocyte esterase was positive, but this finding only indicates that the urine came into contact with inflamed tissues and is not at all very specific for a true UTI. Urinary nitrite is produced when bacteria that reduce urinary nitrate to nitrite have been in contact with the urine. Classically, this signifies that the urine has come into contact with a member of the bacterial family of Enterobacteriaceae , e.g., Escherichia coli, Klebsiella Proteus, Enterobacter, Serratia, or Citrobacter. It is noteworthy that clinically symptomatic UTIs with streptococcus, enterococcus, pseudomonas, and Candida will turn urine nitrite positive in < 5% of infections. The combined findings in this patient highly suggest that this girl is very unlikely to have a UTI; negative bagged urine culture, negative urine nitrite. She should not be treated for UTI and repeat culture in this circumstance is likely a waste of resources. Another source for the possible febrile illness should be sought. Shortliffe LMD: Infection and inflammation of the pediatric genitourinary tract, Wein, AJ, Kavoussi LR, Novick AC, Partin AW, Peters CA (eds): CAMPBELL-WALSH UROLOGY, ed 10. Philadelphia, Elsevier Saunders, 2012, vol 4, chap 116, pp 3096-3097."

"A 13-month-old girl has a fever of 38.5°C. A bagged urine specimen is positive for leukocyte esterase, negative for nitrites, and has no growth on culture at 24 hours. The next step is: A. observation. B. empiric antibiotics for UTI. C. catheterized urine specimen. D. suprapubic aspiration. E. renal bladder ultrasound."

"B: ""CT cystogram."" was the correct answer. It is noteworthy that up to 50% of cystogram studies in cases of a ruptured augmented bladder may be negative for the injury; therefore, a negative cystogram does not rule-out a bladder rupture. With this patient having clear clinical signs of acute peritoneal inflammation, the differential diagnosis is cystitis with peritoneal irritation (treated by antibiotic therapy and continuous bladder drainage and not requiring surgery), a loculated infected ventriculo-peritoneal shunt (requiring a neurosurgical consultation), or a missed bladder rupture (requiring emergent surgery). Confirmation of suspected perforation of an augmented bladder is best achieved by performing a CT cystogram with a minimum of 300 ml of contrast placed in the bladder with a CT phase taken with the bladder distended and emptied. An abdominal CT without a cystogram may show a significant increase in peritoneal fluid if an infected ventriculoperitoneal shunt is present (false positive for bladder rupture) or, alternatively, a high false negative rate for rupture if a small bladder perforation is present and occluded by either small bowel or omentum. The performance of a CT cystogram with a minimum of 300 ml of contrast has been shown to dislodge the omentum or small bowel that would be plugging the bladder laceration and diagnosis of extravasated contrast is more readily found on the CT evaluation than the plain cystogram. Placement of a large bore suprapubic tube will not sufficiently divert urine or drain infected urine from the peritoneal cavity if a bladder rupture is present. Continuous catheter drainage along with I.V. antibiotics may be of benefit for severe cystitis but should only be pursued after documentation of the absence of a bladder perforation. Exploratory laparotomy would be indicated without a CT cystogram if the patient was clinically unstable and not responsive to fluid resuscitation. Adams MC, Joseph DB: Urinary tract reconstruction in children, Wein AJ, Kavoussi LR, Novick AC, Partin AW, Peters CA (eds): CAMPBELL-WALSH UROLOGY, ed 10. Philadelphia, Elsevier Saunders, 2012, vol 4, chap 129, pp 3484-3485."

"A 14-year-old boy with myelodysplasia and a ventriculo-peritoneal shunt has acute abdominal pain. He underwent a sigmoid bladder augmentation six months ago. His pulse is 100 bpm and blood pressure is 100/70 mmHg. He has diffuse abdominal tenderness. Urine output over the past six hours is 70 ml. Cystogram shows no extravasation. The best management is I.V. hydration, broad spectrum antibiotics, and: A. abdominal CT scan. B. CT cystogram. C. continuous catheter drainage. D. placement of large bore (24F), percutaneous suprapubic tube. E. exploratory laparotomy."

"C: ""inadequate tubular resorption of ornithine and cystine."" was the correct answer. Cystinuria is the result of inadequate renal tubular resorption of cystine, ornithine, lysine, and arginine, and is also associated with inadequate intestinal absorption of these amino acids. Stones result from increased concentrations of cystine in the urine which precipitate in acidic urine. The other amino acids do not crystallize to form stones. Pearle MS, Lotan Y: Urinary lithiasis: Etiology, epidemiology, and pathogenesis, Wein, AJ, Kavoussi LR, Novick AC, Partin AW, Peters CA (eds): CAMPBELL-WALSH UROLOGY, ed 10. Philadelphia, Elsevier Saunders, 2012, vol 2, chap 45, pp 1279-1280. "

"A 16-year-old boy passes a stone composed of 100% cystine. This condition is characterized by: A. excessive jejunal absorption of cystine. B. excessive tubular resorption of arginine and lysine. C. inadequate tubular resorption of ornithine and cystine. D. excessive metabolic production of cystine, ornithine and lysine. E. impaired conversion of cystine to ornithine."

"A: ""seminoma."" is correct. The syndrome of complete androgen insensitivity (CAIS), previously known as testicular feminization syndrome, is due to absence of a functional androgen receptor. All affected individuals will have a 46 XY genotype and approximately 80% will have ""normal"" appearing external female genitalia. These patients currently present by one of five different means: fetal karyotype (46 XY) incongruent with newborn infant's phenotype (5% of patients), relative or family member with CAIS with patient diagnosed due to recommendation for genetic screening (15% of patients), ambiguous genitalia at birth, i.e., female phenotype with palpable gonads or mild to moderate clitorimegaly (20% of patients), primary amenorrhea (30% of patients), and testicle found within a inguinal hernia at the time of surgical repair (30% of patients). Patients with complete androgen insensitivity syndrome will have a substantial increased risk of developing a testicular seminoma and if the testis is left in situ, approximately 20% of the patients will have developed a testicular malignancy by the age of 30. Removal of the testicles are, therefore, strongly recommended either prior to or immediately following pubescence. Gonadoblastoma is a tumor that is associated with disorders of sex development. Specifically, it is found in infants noted to have partial or pure gonadal dysgenesis (46 XY or 46 XY/XO genotypes) and is not associated with complete androgen insensitivity syndrome. Individuals with complete androgen insensitivity syndrome are at no higher risk of Leydig cell tumors, embryonal cell carcinoma, and teratomas than the normal population. Diamond DA, Yu RN: Sexual differentiation: Normal and abnormal, Wein, AJ, Kavoussi LR, Novick AC, Partin AW, Peters CA (eds): CAMPBELL-WALSH UROLOGY, ed 10. Philadelphia, Elsevier Saunders, 2012, vol 4, chap 133, p 3621. Ritchey ML, Shamberger RC: Pediatric urologic oncology, Wein, AJ, Kavoussi LR, Novick AC, Partin AW, Peters CA (eds): CAMPBELL-WALSH UROLOGY, ed 10. Philadelphia, Elsevier Saunders, 2012, vol 4, chap 137, p 3725. "

"A 16-year-old phenotypic girl undergoes evaluation for primary amenorrhea. Testes are found on laparoscopic examination. Karyotype is 46 XY. If the gonads are not removed, the tumor most likely to develop is: A. seminoma. B. Leydig cell tumor. C. gonadoblastoma. D. embryonal cell carcinoma. E. teratoma."

"A: ""ketamine abuse."" is correct. Ketamine (also known as: K, Special K, Vitamin K, green, and jet) is a tranquilizer that will induce a trance-like state while providing pain relief, sedation, and memory loss. Heart rate, respiratory function, and airway reflexes will remain functional. Ketamine is most commonly used for pain relief in emergency rooms and intensive care units and/or sedation at the time of anesthetic induction. Beginning in the late 1990's, it has gained popularity as a street drug and has quickly outgrown heroin and methamphetamine as the drug of choice in many parts of the world due to its low cost and easy accessibility. Ketamine has joined the ranks of PCP, Ecstasy (MDMA), GHB (Gamma Hydroxybutyrate), and Rohypnol as popular club drugs. It is frequently mixed with Rhobypnol or GHB to create the ""date rape"" drug. Ketamine, when used alone, will give a floating out of the body sensation combined with a mild hallucinogenic effect. Within the United States, 1-2% of individuals < 21 years of age admit to having tried ketamine and 10% of these individuals will become habitual users. Ketamine is currently the leading street drug used in Hong Kong and other parts of Asia. Indeed, drug and alcohol rehabilitation clinics within Hong Kong and other select Asian locations have found up to 80% of the individuals admitted for rehabilitation to be abusing the medication. Urologists should be aware that the chronic use of ketamine will induce ketamine cystitis. Symptoms, cystoscopic, and biopsy findings are highly consistent with non-Hunner's interstitial cystitis, and without the proper history, distinction between the two diagnoses is almost impossible. Ketamine cystitis will usually lead to a severely fibrotic end-stage bladder that will result in the need for cystectomy. There are, however, successful case reports where the fibrosis has been halted or reversed by the use of intravesical chondroitin sulphate or hyaluronic acid. Tuberculosis of the bladder occurs secondary to tuberculosis of the kidney, and, therefore, upper tract abnormalities would be expected on CT scan. Malacoplakia typically manifests as mucosal plaques or nodules with bladder biopsy demonstrating Michaelis-Gutman bodies. Herpes simplex virus infection causes painful ulcers of the genitalia typically without bladder involvement. Cytomegalovirus may cause hematuria and urinary symptoms but only in immunocompromised individuals. Wood D, Cottrell A, Baker SC, et al: Recreational ketamine: from pleasure to pain. BRI J UROL INT 2011;107:1881-1884. Schaeffer AJ, Schaeffer EM: Infections of the urinary tract, Wein, AJ, Kavoussi LR, Novick AC, Partin AW, Peters CA (eds): CAMPBELL-WALSH UROLOGY, ed 10. Philadelphia, Elsevier Saunders, 2012, vol 1, chap 10, pp 309-311. Frenkl TL, Potts JM: Sexually transmitted infections, Wein, AJ, Kavoussi LR, Novick AC, Partin AW, Peters CA (eds): CAMPBELL-WALSH UROLOGY, ed 10. Philadelphia, Elsevier Saunders, 2012, vol 1, chap 13, pp 404-405. Ghoneim IA, Rabets JC, Mawhorter SD: Tuberculosis and other opportunistic infections of the genitourinary system, Wein, AJ, Kavoussi LR, Novick AC, Partin AW, Peters CA (eds): CAMPBELL-WALSH UROLOGY, ed 10. Philadelphia, Elsevier Saunders, 2012, vol 1, chap 16, pp 469-474. "

"A 25-year-old man is referred from the drug and alcohol rehabilitation center. He complains of severe diurnal and nocturnal urinary urgency, frequency, and intermittent gross hematuria. Urine culture is negative. CT urogram reveals a small thick-walled bladder but is otherwise unremarkable. Cystoscopy reveals multiple glomerulations and petechial hemorrhages and no Hunner's lesions. Bladder biopsy shows severe inflammation and epithelial denudation. The most likely etiology is: A. ketamine abuse. B. genitourinary tuberculosis. C. malacoplakia. D. herpes simplex virus. E. cytomegalovirus."

"C: ""inguinal exploration with cyst enucleation."" was the correct answer. The differential diagnosis of cystic testicular lesions includes: epidermoid cyst, dermoid cyst, prepubertal teratoma, juvenile granulosa cell tumor, cystic dysplasia of the rete testis, lymphangioma, simple cyst, and cyst degeneration after torsion. The patient's age, physical examination, and ultrasound findings suggest an epidermoid cyst or simple cyst. Since these lesions are clinically indistinguishable from neoplasms, preoperative tumor markers should be drawn. In this age group, if preoperative tumor markers are negative and a benign lesion is suspected on imaging, testicular sparing surgery via an inguinal incision with the use of frozen section histopathology for confirmation, should be attempted when possible. Cyst aspiration for cytopathology is neither diagnostic nor indicated, and worst, would upstage a malignant tumor and potentially seed the needle tract. Placement of a prosthesis is usually delayed until adolescence when an appropriate size prosthesis can be used, as well as involve the adolescent or young adult patient in preoperative discussion as to their desire for a prosthesis. Ritchey ML, Shamberger RC: Pediatric urologic oncology, Wein, AJ, Kavoussi LR, Novick AC, Partin AW, Peters CA (eds): CAMPBELL-WALSH UROLOGY, ed 10. Philadelphia, Elsevier Saunders, 2012, vol 4, chap 137, p 3725. Garrett JE, Cartwright PC, Snow BW, Coffin CM: Cystic testicular lesions in the pediatric population. J UROL 2000,163:928-936. "

"A 17-month-old boy has a firm, smooth testicular mass. Testicular ultrasound shows a 2 cm upper pole intratesticular cyst with preservation of the parenchyma. Tumor markers are negative. The next step is: A. transscrotal enucleation. B. radical inguinal orchiectomy. C. inguinal exploration with cyst enucleation. D. cyst aspiration for cytology. E. simple orchiectomy with placement of prosthesis."

"C: ""bladder augmentation prior to kidney transplant."" was the correct answer. The patient has poor bladder compliance with bilateral grade 2 VUR, elevated pressures of > 40 cm at 100 ml, and end fill detrusor pressure with overflow at system LPP of 85 cm at 280 ml. These urodynamic findings are highly suggestive of high detrusor pressures being related to her loss of renal function. She can undergo kidney transplant, however, to reduce risk of damage to the transplant, she would be best treated with pre-transplant bladder augmentation. This would lower her detrusor storage pressures and probably minimize or resolve her low grade VUR. It is noteworthy that this patient does not have oliguric renal failure. Poor detrusor compliance is frequently found on a urodynamic study in oliguric or anuric pre-transplant patient due to disuse detrusor atrophy. In the latter patients, the bladder will almost invariably develop normal compliance post-transplant and no preoperative intervention is required. If there is concern that the bladder is really noncompliant in a patient with oligouric renal failure, individuals may be started on CIC with increasing volumes of fluid placed in the bladder over a series of weeks and the urodynamic study repeated. This patient does not have any of the criteria necessary to consider pre-transplant nephrectomy. Indications for pre-transplant nephrectomy are: renal lithiasis, renal mass, chronic pyelonephritis, uncontrollable hypertension, and excessive proteinuria. Occasionally in a patient with autosomal dominant polycystic kidney disease, a nephrectomy may be performed due to inadequate space for a renal transplant. Wein AJ, Dmochowski RR: Neuromuscular dysfunction of the lower urinary tract, Wein, AJ, Kavoussi LR, Novick AC, Partin AW, Peters CA (eds): CAMPBELL-WALSH UROLOGY, ed 10. Philadelphia, Elsevier Saunders, 2012, vol 3, chap 65, pp 1929, 1943. "

"A 22-year-old woman with ESRD and spina bifida undergoes pre-op transplant evaluation. Renal bladder ultrasound reveals moderate bilateral hydroureteronephrosis associated with a full bladder. Videourodynamics reveals a total fill poorly-compliant bladder with detrusor pressures of 50 cm H2O at 100 ml, associated with bilateral grade 2 VUR, and end fill detrusor pressure of 85 cm H2O at 280 ml with overflow incontinence. She leaks urine several times a day despite CIC four times daily and obtains 250-300 ml/catheterization. The next step is: A. proceed with kidney transplant, then re-evaluate bladder. B. bladder augmentation after kidney transplant. C. bladder augmentation prior to kidney transplant. D. bilateral native nephrectomies and augment prior to kidney transplant. E. recommend against kidney transplant."

"B: ""MRI scan."" was the correct answer. Penile fracture can usually be diagnosed by history and physical examination and imaging is unnecessary in the majority of patients. However, the clinician should be aware that differential diagnosis includes a false penile fracture and/or rupture of a penile vein with a subsequent penile hematoma. These latter patients may clinically present with the classic history of a typical snap-pop sensation and immediate detumescence as in this case. In cases in which the physical examination is not classic for a penile facture, imaging with a penile-perineal MRI scan is the most accurate test to prevent unnecessary surgical exploration. Both penile Doppler and cavernosography have very high false negative rates and are not recommended in the evaluation of suspected penile fracture. Obtaining a urinalysis to rule-out a concurrent urethral injury, found in up to 10-20% of patients with a penile fracture is absolutely necessary. In patients with a confirmed penile fracture, imaging of the urethra prior to exploration is mandatory and can be performed with either a retrograde urethrogram or cystoscopy at the time of the procedure. If a urethral injury is found, this should be repaired concurrently with the penile fracture. Morey AF, Dugi DD III: Genital and lower urinary tract trauma, Wein AJ, Kavoussi LR, Novick AC, Partin AW, Peters CA (eds): CAMPBELL-WALSH UROLOGY, ed 10. Philadelphia, Elsevier Saunders, 2012, vol 3, chap 88, p 2507. "

"A 23-year-old man has buckling and pain during intercourse. On examination, he has minimal penile tenderness and swelling with no ecchymosis or hematoma. Urinalysis is normal. The next step is: A. Doppler ultrasound. B. MRI scan. C. infusion cavernosography. D. flexible urethroscopy. E. surgical exploration."

"B: ""thiazides."" was the correct answer. The radiograph demonstrates the typical ""bunches of grapes"" or ""bouquet of flowers"" appearance of medullary sponge kidney. The characteristic anatomic feature of medullary sponge kidney is dilation of the distal portion of the collecting ducts associated with numerous cysts and diverticula. If symptomatic, most patients (60%) present with flank pain, followed by UTI (25%) and gross hematuria (15%). For patients with renal lithiasis and hypercalciuria, thiazides should be administered as they arrest stone growth and prevent new stone formation. Parathyroidectomy is not indicated in this patient with normal serum calcium and no measurement of serum parathormone. In the absence of an obstructing stone, PCNL is not indicated. Pearle MS, Lotan Y: Urinary lithiasis: Etiology, epidemiology, and pathogenesis, Wein, AJ, Kavoussi LR, Novick AC, Partin AW, Peters CA (eds): CAMPBELL-WALSH UROLOGY, ed 10. Philadelphia, Elsevier Saunders, 2012, vol 2, chap 45, p 1285."

"A 23-year-old woman has dull, persistent, bilateral flank pain. A scout CT image without contrast is shown. Serum calcium is 9.4 mg/dl and urine calcium on an unrestricted diet is 300 mg/day (normal < 250 mg/day). Urine culture is negative. The next step is: A. observation. B. thiazides. C. potassium citrate. D. percutaneous nephrolithotomy. E. parathyroidectomy."

"B: ""ureteral stent and urethral catheter."" was the correct answer. The patient has a delayed urine leak following high velocity penetrating renal trauma. CT scan and retrograde pyelogram reveal a urine leak. The next step is ureteral stent and urethral catheter drainage to prevent reflux of urine retrograde into the stent which may potentiate the leak and result in infection of the retroperitoneal hematoma. Ureteral stent placement may allow for resolution of the leak without operative intervention. The patient's recent history of shattered kidney along with liver and bowel injury placement of a percutaneous nephrostomy tube would result in increased risk for infection of the perinephric hematoma and maybe difficult to place and maintain in the proper position due to the renal injury. Exploration of the right kidney is not indicated at this time. Santucci RA, Doumanian LR: Upper urinary tract trauma, Wein, AJ, Kavoussi LR, Novick AC, Partin AW, Peters CA (eds): CAMPBELL-WALSH UROLOGY, ed 10. Philadelphia, Elsevier Saunders, 2012, vol 2, chap 42, p 1178. "

"A 24-year-old man is shot in the right flank and sustains a grade 5 renal injury and extensive liver and bowel injury. During exploration, his liver and bowel injuries are stabilized and a nonpulsatile right retroperitoneal hematoma is identified with no intervention. One week after exploration, he develops increased drainage from a perihepatic drain. Creatinine of the drainage fluid is 8 mg/dl. CT scan and retrograde pyelogram are shown. The next step is: A. ureteral stent. B. ureteral stent and urethral catheter. C. percutaneous nephrostomy. D. exploration, debridement, and renal repair. E. nephrectomy."

"C: ""modified RPLND."" was the correct answer. The presence of rete testis involvement is associated with approximately a 20-25% incidence of tumor relapse and lymphovascular invasion with approximately a 50% incidence of relapse of his tumor to his retroperitoneal lymph nodes. Placement of this patient on an observation protocol would suggest, at minimum, a 50% risk for the need for salvage chemotherapy. If he is observed and recurs, he will require chemotherapy which will have an adverse effect on his fertility. Chemotherapy, even if administered in low doses, can result in a permanent adverse effect on fertility in approximately 30% of patients. In contrast, fertility is preserved in 80%-90% of patients treated with a modified RPLND or nerve-sparing dissection, Even if he has retroperitoneal lymph node involvement found on pathologic assessment of his lymph nodes, RPLND is curative in two-third of these patients and chemotherapy can be avoided. The controversy regarding whether or not to proceed with modified RPLND is based upon the fact that approximately 50% of the patients would undergo needless surgery. Radiation therapy and carboplatin are not indicated in NSGCT. Stephenson AJ, Gilligan TD: Neoplasms of the testis, Wein, AJ, Kavoussi LR, Novick AC, Partin AW, Peters CA (eds): CAMPBELL-WALSH UROLOGY, ed 10. Philadelphia, Elsevier Saunders, 2012, vol 1, chap 31, pp 851-854. Sheinfeld J, Bosl GJ: Surgery of testicular tumors, Wein, AJ, Kavoussi LR, Novick AC, Partin AW, Peters CA (eds): CAMPBELL-WALSH UROLOGY, ed 10. Philadelphia, Elsevier Saunders, 2012, vol 1, chap 32, p 876. Lowe BA: Surveillance versus nerve-sparing retroperitoneal lymphadenectomy in Stage 1 nonseminomatous germ-cell tumors. UROL CLIN N AM 1993;20(1):75."

"A 25-year-old man undergoes a radical orchiectomy for a T2, predominantly embryonal cell carcinoma. Pathology reveals a 3 cm embryonal cell carcinoma with involvement of the rete testis and lymphovascular invasion. Serum markers and a chest and abdominal CT scan are normal. His semen analysis was normal. The treatment that offers the best chance for natural fertility on a long-term basis is: A. XRT. B. surveillance. C. modified RPLND. D. single dose carboplatin. E. two cycles BEP."

"D: ""15% risk of retroperitoneal relapse."" was the correct answer. The standard of care for decades in a patient with clinical stage I seminoma (negative CT scan and negative tumor markers) was retroperitoneal XRT. In patients receiving XRT for stage I seminoma, the risk of overall relapse is 5% or less and it would be extremely uncommon for such a patient to relapse with nonseminomatous elements. The major concern regarding the routine use of XRT in these patients was the risk of secondary malignancy, usually leukemias and lymphomas found in approximately 18% of patients 25 years after radiation therapy. To reduce the risk of secondary malignancy, surveillance may be a reasonable option. In a patient with stage one seminoma, there is an approximately 15% risk of relapse of the tumor to retroperitoneal lymph nodes; almost all are salvaged provided they were reliable candidates for serial surveillance and were not lost to follow-up. Visceral and pulmonary relapse of seminoma or alteration to non-seminomatous elements is extremely uncommon in patients with stage I seminoma. Single agent carboplatin is the other option in place of either XRT or observation for clinical stage I seminoma and is associated with a 3-5% relapse rate in reported series. Stephenson AJ, Gilligan TD: Neoplasms of the testis, Wein, AJ, Kavoussi LR, Novick AC, Partin AW, Peters CA (eds): CAMPBELL-WALSH UROLOGY, ed 10. Philadelphia, Elsevier Saunders, 2012, vol 1, chap 31, pp 862-864."

"A 26-year-man undergoes orchiectomy for seminoma. Abdominal CT scan and tumor markers are negative. He elects surveillance. The most accurate statement regarding his outcome over the next two years is: A. similar risk of relapse as patients treated with XRT. B. 5% risk of visceral relapse. C. 5% risk of relapse with non-seminomatous elements. D. 15% risk of retroperitoneal relapse. E. 15% risk of pulmonary relapse."

"C: ""testicular ultrasound."" was the correct answer. Gynecomastia occurs in the male when there is an alteration of the testosterone-estradiol ratio which may be due to either reduced testosterone or elevated estradiol. The differential diagnosis of gynecomastia in an adult includes prolactin-secreting pituitary tumor, estrogenic drugs, exogenous testosterone, testicular tumors, and idiopathic. The elevated testosterone level with low gonadotropin levels in this patient is most suggestive of either exogenous testosterone or a testicular tumor. Exogenous testosterone will usually result in a reduction in the size and softening of the consistency of the testes. The most likely etiology is a testicular tumor despite his normal testicular physical examination. Leydig cell tumors are one of the more common nonpalpable testicular tumors and with their inherent ability to secret testosterone is the most likely diagnosis. If it had been a prolactin secreting tumor, the LH and the testosterone would be low. The MRI scan is not needed since the pituitary is responding normally and prolactinoma is not suspected. Gynecomastia in these patients is caused by the conversion of excess (unregulated) testosterone to estradiol by aromatase. Sabanegh E, Agarwal A: Male infertility, Wein, AJ, Kavoussi LR, Novick AC, Partin AW, Peters CA (eds): CAMPBELL-WALSH UROLOGY, ed 10. Philadelphia, Elsevier Saunders, 2012, vol 1, chap 21, p 639."

"A 26-year-old man has bilateral gynecomastia. Testicular examination is normal. Free testosterone is elevated and LH is decreased. The next step is: A. cranial MRI scan. B. abdominal CT scan. C. testicular ultrasound. D. serum prolactin. E. serum estradiol."

"E: ""ureteral reimplant."" was the correct answer. The patient has suffered a ureteral contusion from a high velocity weapon, and many times the extent of the injury is underestimated at initial exploration. Following a high velocity injury, the patient is at high risk for subsequent ureteral stenosis, or tissue necrosis from delayed microvascular injury to the ureter. The best option for a distal ureteral contusion from a high velocity injury is ureteral reimplantation. A ureteroureterostomy should not be performed in the distal ureter. Observation, ureteral stent, and percutaneous nephrostomy drainage will not alter the eventual tissue damage from the blast effect and, thus, would not be helpful in this situation. Santucci RA, Doumanian LR: Upper urinary tract trauma, Wein, AJ, Kavoussi LR, Novick AC, Partin AW, Peters CA (eds): CAMPBELL-WALSH UROLOGY, ed 10. Philadelphia, Elsevier Saunders, 2012, vol 2, chap 42, pp 1179, 1182-1184."

"A 26-year-old man is shot with a high velocity rifle in the lower abdomen. Exploration reveals a rectal injury which is repaired. Inspection of the left distal ureter reveals a 2 cm contusion. I.V. administration of methylene blue does not reveal a ureteral leak. He is hemodynamically stable. The next step is: A. observation. B. ureteral stent placement. C. percutaneous nephrostomy. D. ureteroureterostomy. E. ureteral reimplant."

"C: ""stent placement."" was the correct answer. The patient presented with symptoms of autonomic dysreflexia that have temporarily subsided but are likely to recur. This represents a life-threatening situation. Stent placement will avert recurrent obstruction and dysreflexia. Alkalinization and medical expulsive therapy are appropriate once the patient has been stented. Surgical intervention should be entertained if dissolution therapy is unsuccessful. Wein AJ, Dmochowski RR: Neuromuscular dysfunction of the lower urinary tract, Wein, AJ, Kavoussi LR, Novick AC, Partin AW, Peters CA (eds): CAMPBELL-WALSH UROLOGY, ed 10. Philadelphia, Elsevier Saunders, 2012, vol 3, chap 65, p 1926. "

"A 26-year-old man with complete C7 spinal cord injury managed by CIC has recurrent episodes of vague abdominal pain, severe headache, and diaphoresis. Imaging reveals a 12 mm obstructing radiolucent UPJ stone and urinalysis shows a pH of 5.0, 25-50 RBC/hpf, 0-5 WBC/hpf, and no bacteriuria. The next step is: A. urinary alkalinization. B. urinary alkalinization and tamsulosin. C. stent placement. D. ureteroscopic intervention. E. percutaneous nephrolithotomy."

"B: ""sperm viability testing."" was the correct answer. Ultrastructural flagellar defects result in motilities of less than 5-10%. In these samples, most of the non-motile sperm are viable. Most cases of ultrastructural flagellar defects are associated with normal sperm densities. Since the semen volume is normal, there is no reason to look for retrograde ejaculation with a post-ejaculate urinalysis. While a partial ejaculatory duct obstruction has been reported to be associated with low motility, this should only be considered if the sperm are viable and thus would not be the next step. Cystic fibrosis mutations are associated with congenital absence of the vas deferens and azoospermia. Semen WBC staining is indicated only if increased numbers of round cells are noted in the semen. Sabanegh E, Agarwal A: Male infertility, Wein, AJ, Kavoussi LR, Novick AC, Partin AW, Peters CA (eds): CAMPBELL-WALSH UROLOGY, ed 10. Philadelphia, Elsevier Saunders, 2012, vol 1, chap 21, pp 621-622. Jarow JP, Sigman M, Kolettis PN, et al: The optimal evaluation of the infertile male: AUA BEST PRACTICE STATEMENT. American Urological Association Education and Research, Inc, 2010. http://www.auanet.org/education/guidelines/male-infertility-d.cfm "

"A 30-year-old man with primary infertility has a semen analysis that shows a 2.5 ml ejaculate volume, 50 million sperm/ml, and 1% motility. The morphology score and round cell concentration are normal. A repeat semen analysis is similar. The next step is: A. post-ejaculate urine analysis. B. sperm viability testing. C. transrectal ultrasound. D. cystic fibrosis mutation screening. E. semen white blood cell stain."

"E: ""testicular biopsy with sperm retrieval."" was the correct answer. This patient has normal seminal volume and normal seminal pH. This rules out ejaculatory duct obstruction or congenital bilateral absence of the vas (CBAVD) which are both associated with low volume, acidic semen specimens. A low normal FSH of 3.6 mIU/ml indicates spermatogenesis is likely intact and that obstructive azoospermia is present. A testicular biopsy is warranted during which sperm retrieval and cryopreservation should be considered. If the testicular biopsy is normal, then a post-testicular obstruction is present and either an epididymovasostomy or a vasovasostomy can be performed at the time of biopsy. Clomiphene will increase serum FSH but will not correct obstructive azoospermia. Seminal fructose is absent in ejaculatory duct obstruction or CBAVD. A transrectal ultrasound is not indicated because seminal volume is normal. Post-ejaculate urine analyses are useful in cases of low volume semen specimens or cases with dry ejaculates to rule-out retrograde ejaculation. Jarow J, Sigman M, Kolettis PN, et al: The evaluation of the azoospermic male: An AUA best practice policy. AZOOSPERMIC MALE BEST PRACTICE STATEMENT. American Urological Association Education and Research, Inc., 2011. http://www.auanet.org/education/guidelines/male-infertility-b.cfm Sabanegh E, Agarwal A: Male infertility, Wein, AJ, Kavoussi LR, Novick AC, Partin AW, Peters CA (eds): CAMPBELL-WALSH UROLOGY, ed 10. Philadelphia, Elsevier Saunders, 2012, vol 1, chap 21, pp 634-635. "

"A 31-year-old man desires a biological child. His physical exam is normal with 30 ml testes. He is azoospermic and has a semen volume of 3.5 ml with a pH of 7.6. His FSH is 3.6 IU/l. The next step is: A. clomiphene citrate. B. seminal fructose level. C. post-ejaculate urine. D. transrectal ultrasound. E. testicular biopsy with sperm retrieval."

"C: ""VCUG."" was the correct answer. The retrograde urethrogram (RGUG) provided in the image is an incomplete study. Two major problems exist with this image. First, the patient is not tipped far enough laterally; this can be identified by the visible obturator ring. If either or both of the obturator rings are visible, a distortion of the urethra is produced due to an improper pelvic angle and the length of the stricture will appear shorter than it actually is. Second, not enough contrast passed through the stricture to delineate its proximal extent. A repeat RGUG would most likely not reveal any additional information. A voiding urethrogram would provide an antegrade study that will allow the physician to determine the proximal extent of the urethral stricture. To perform this study, the urologist may need to pass a glide wire through the stricture followed by a 6 Fr ureteral catheter. This will allow the bladder to be filled with contrast and not dilate or destroy the stricture. The patient is subsequently asked to void and the proximal location of the stricture identified. If you can combine the VCUG with a simultaneous RGUG, a so-called up and downogram can be obtained that will delineate the length of the stricture. Cystoscopy or urethral ultrasound would not add additional information as to the extent of the stricture. Urethral ultrasound has been shown to determine the degree of spongiofibrosis yet would not add additional information over an antegrade study regarding the length of the stricture. Urethroplasty with incomplete staging information should not be performed in order to prevent unnecessary buccal mucosa harvest, and/or added operative time in the lithotomy position while a graft is unexpectedly harvested. Jordan GH, McCammon KA: Surgery of the penis and urethra, Wein, AJ, Kavoussi LR, Novick AC, Partin AW, Peters CA (eds): CAMPBELL-WALSH UROLOGY, ed 10. Philadelphia, Elsevier Saunders, 2012, vol 1, chap 36, p 970."

"A 32-year-old man has a weak urinary stream and recurrent UTIs. He has a known urethral stricture and has undergone three dilations in the past. Retrograde urethrography is shown. The next step is: A. cystoscopy. B. urethral ultrasound. C. VCUG. D. excision and primary anastomosis. E. urethroplasty with graft or flap."

"D: ""low semen volume."" was the correct answer. Congenital bilateral absence of the vas deferens (CBAVD) is associated with cystic fibrosis gene mutations in 85% of patients. Approximately 7% of brothers will have also vasal agenesis. Only 10-15% of men with CBAVD will have unilateral renal agenesis and most of these are in patients with no identifiable cystic fibrosis gene mutation. There is no association with Y microdeletions. Semen is almost always of low volume and acidic due to hypoplasia or absence of the seminal vesicles, which provide alkalinity. Oates RD: The genetic basis of male reproductive failure. UROL CLIN N AM, 2008;35:257-270. Sabanegh E, Agarwal A: Male infertility, Wein, AJ, Kavoussi LR, Novick AC, Partin AW, Peters CA (eds): CAMPBELL-WALSH UROLOGY, ed 10. Philadelphia, Elsevier Saunders, 2012, vol 1, chap 21, pp 641-642."

"A 32-year-old man with bilateral congenital absence of the vas deferens (CBAVD) is most likely to have: A. unilateral renal agenesis. B. a brother with vasal agenesis. C. hypospermatogenesis. D. low semen volume. E. Y chromosome microdeletion."

"A: ""penile Doppler ultrasound and corporal blood gas determination."" was the correct answer. Ischemic priapism presenting in a significantly delayed fashion (> 48-72 hours) may predictably fail to resolve with intracavernous treatment as well as surgical shunting and suggests that erectile dysfunction will be the inevitable outcome. Even if a successful shunt is performed, woody induration of the penis may persist. To confirm the presence of persistent hypoxemia, a penile Doppler ultrasound to assess for vascular flow and a corporal blood gas determination should be performed. If these evaluations confirm poor vascular flow and continued ischemia, experts have advocated immediate placement of a penile prosthesis. The recommendation is based in part on the knowledge that placement of a penile prosthesis at a later time, after significant fibrosis has evolved, is extremely difficult and fraught with higher complication rates. Based on the duration of priapism, it is unlikely that continuous irrigation with an alpha-adrenergic agent or systemic baclofen would be effective. Selective embolization of the cavernosal artery is a consideration for treatment of high flow priapism, not ischemic induced priapism. Broderick GA: Priapism, Wein, AJ, Kavoussi LR, Novick AC, Partin AW, Peters CA (eds): CAMPBELL-WALSH UROLOGY, ed 10. Philadelphia, Elsevier Saunders, 2012, vol 1, chap 25, p 760."

"A 34-year-old man has priapism for more than 48 hours. He continues to have a firm, indurated penis despite a corporal-glanular shunt followed by a proximal corpora spongiosum shunt. The next step is: A. penile Doppler ultrasound and corporal blood gas determination. B. continuous irrigation with an alpha-adrenergic agent. C. systemic baclofen. D. selective embolization of cavernosal artery. E. penile prosthesis."

"C: ""pure embryonal cell carcinoma in the primary tumor."" was the correct answer. Patients with fibrosis/necrosis, only in the post-chemotherapy specimens, have a favorable prognosis with relapse rates of 10% or less in most series. Investigators have sought to identify factors that reliably predict for a high probability of fibrosis/necrosis in a post-chemotherapy residual mass. The absence of teratoma in the primary tumor, the percentage reduction in the retroperitoneal mass with chemotherapy (usually cited as 90% or greater), and the size of the post-chemotherapy residual mass have consistently been identified as predictors of fibrosis/necrosis in the post-chemotherapy specimens. In this question, pure embryonal carcinoma would be the best predictor of fibrosis only in the retroperitoneum. Neither pre-chemotherapy lymph node size nor pre-chemotherapy serum tumor marker levels are predictive for fibrosis and the percent reduction has been cited as 90% or greater to predict fibrosis. The absence of teratoma in the primary would predict for fibrosis, not the presence of teratoma. Stephenson AJ, Gilligan TD: Neoplasms of the testis, Wein, AJ, Kavoussi LR, Novick AC, Partin AW, Peters CA (eds): CAMPBELL-WALSH UROLOGY, ed 10. Philadelphia, Elsevier Saunders, 2012, vol 1, chap 31, pp 857-858. "

"A 35-year-old man has persistent retroperitoneal lymphadenopathy after platin-based chemotherapy for NSGCT. The parameter most predictive of finding only fibrosis in the retroperitoneum is: A. pre-chemotherapy lymph node size. B. 75% reduction in size of the mass on CT scan. C. pure embryonal cell carcinoma in the primary tumor. D. pre-chemotherapy marker levels. E. teratoma in the primary tumor."

"C: ""increase intake of calcium."" was the correct answer. Urinary oxalate is modulated by calcium intake which influences intestinal oxalate absorption. Large cohort studies have demonstrated an increased risk of stone formation with lower calcium diets and have also found that diets higher in calcium are associated with reduced oxalate excretion. Of note, patients with enteric hyperoxaluria and high levels of urinary oxalate, such as those with malabsorptive conditions (e.g., inflammatory bowel disease or Roux-en-Y gastric bypass), may benefit from more restrictive oxalate diets as well as from higher calcium intakes, which may include supplements, specifically timed with meals. In such cases, calcium will serve as an oxalate binder so that a significant proportion will appear in the stool. However, 24-hour urine monitoring can be used to ensure that hypercalciuria does not result. Animal protein restriction is not correct as that recommendation would be most appropriate for those with hyperuricosuria. A role for nutritional supplements, such as, omega-3 fatty acids, pyridoxine, and cholestyramine has been suggested, but at present the evidence is not sufficient to guide recommendations. Pearle MS, Goldfarb DS, Assimos DG, et al: Medical management of kidney stones: AUA Guideline: AUA Guideline. American Urological Association Education and Research, Inc., 2014. http://www.auanet.org/education/guidelines/management-kidney-stones.cfm "

"A 35-year-old man with Crohn's disease and recurrent calcium oxalate stone formation should be counseled to restrict intake of dietary oxalate and: A. restrict intake of animal protein. B. restrict intake of omega-3 fatty acids. C. increase intake of calcium. D. increase intake of pyridoxine. E. initiate cholestyramine therapy."

"A: ""observation."" is correct. The finding of asymptomatic bacteriuria in patients on CIC has been reported to occur in up to 50% of individuals. Treatment in asymptomatic individuals is not required; in fact, treatment will predispose these patients to develop resistant organisms and significantly increases the risk for Clostridium difficile colitis. Changing to antibiotic coated catheters or sterile CIC are not indicated in this setting. Averch TD, Stoffel J, Goldman HB, et al: AUA White Paper on catheter-associated urinary tract infections: Definitions and significance in the urologic patient: 2014. American Urological Association Education and Research, Inc., 2014. http://www.auanet.org/common/pdf/education/clinical-guidance/Catheter-Associated-Urinary-Tract-Infections-WhitePaper.pdf "

"A 35-year-old paraplegic man on CIC is referred for recurrent UTIs. Review of the patient history reveals that these are asymptomatic in nature and diagnosed by the family physician obtaining intermittent urine cultures. All cultures are positive for E. coli. CT urogram and cystoscopy are unremarkable. The next step is: A. observation. B. change to antibiotic-coated catheters. C. sterile CIC. D. suppressive antibiotics. E. gentamicin bladder irrigation."

"C: ""scrotal exploration and testis biopsy."" was the correct answer. Persistent azoospermia following resection of the ejaculatory ducts may be due to persistent obstruction, concomitant epididymal obstruction, or testicular failure. The restoration of normal ejaculate volume rules out persistent ejaculatory duct obstruction, therefore, neither a repeat TUR-ejaculatory duct nor a TRUS is indicated. Scrotal exploration is necessary to differentiate between epididymal obstruction and testicular failure. Donor insemination is not necessary with obstructive azoospermia unless the obstruction is uncorrectable and the couple refuses ICSI. Varicocelectomy may be indicated in some instances of non-obstructive azoospermia but it is not helpful with obstructive azoospermia. Sabanegh E, Agarwal A: Male infertility, Wein, AJ, Kavoussi LR, Novick AC, Partin AW, Peters CA (eds): CAMPBELL-WALSH UROLOGY, ed 10. Philadelphia, Elsevier Saunders, 2012, vol 1, chap 21, pp 629-633, 634. "

"A 36-year-old man with ejaculatory duct obstruction and a left varicocele has persistent azoospermia despite normalization of ejaculatory volume following TUR of the ejaculatory duct. The next step is: A. TRUS. B. repeat TUR of the ejaculatory duct. C. scrotal exploration and testis biopsy. D. varicocelectomy. E. donor insemination."

"C: ""ureteroscopic endoureterotomy."" was the correct answer. The best management for ureteral strictures less than 2 cm with no previous intervention is an endoscopic approach. Balloon dilation has been reported with some short-term success but long-term outcomes are unfavorable. Should balloon dilation fail, a much longer stricture may develop requiring more invasive intervention. Ureteroscopic ureterotomy, such as with laser incision, provides a more durable long-term outcome. Care must be taken to place the endoureterotomy incision away from the iliac vessels medially (anteromedial incision). Cautery wire incision does not offer any advantage over ureteroscopic endoureterotomy, and ureteroureterostomy is reserved for recurrent ureteral strictures or strictures > 2 cm upon presentation. Nakada SY, Hsu THS: Management of upper urinary tract obstruction, Wein, AJ, Kavoussi LR, Novick AC, Partin AW, Peters CA (eds): CAMPBELL-WALSH UROLOGY, ed 10. Philadelphia, Elsevier Saunders, 2012, vol 2, chap 41, p 1154."

"A 38-year-old man develops a 5 mm ureteral stricture overlying the iliac vessels six months following ureteroscopy and stone basketing. The next step is: A. retrograde balloon dilation. B. percutaneous nephrostomy and antegrade balloon dilation. C. ureteroscopic endoureterotomy. D. ureteroureterostomy. E. ureteral reimplantation with psoas hitch."

"E: ""bilateral varicocelectomy."" is correct. Semen parameters improve to a greater extent after bilateral varicocelectomy as compared to just repairing the dominant varicocele in patients with a grade II-III varicocele and a contralateral grade I varicocele. This approach does not extend to patients with a ""subclinical"" varicocele (those only detected by Doppler ultrasound). Transrectal ultrasonography and clomiphene citrate (Clomid™) are not indicated in this setting. Intrauterine insemination is another alternative but varicocelectomy is more cost effective. Intrauterine insemination is not indicated because sperm concentration is too low. Age of the female partner is also an important consideration; if partner is older, in vitro fertilization should be considered as a treatment option. Sabanegh E, Agarwal A: Male infertility, Wein AJ, Kavoussi LR, Novick AC, Partin AW, Peters CA (eds): CAMPBELL-WALSH UROLOGY, ed 10. Philadelphia, Elsevier Saunders, 2012, vol 1, chap 21, pp 636-637. Scherr D, Goldstein M: Comparison of bilateral versus unilateral varicocelectomy in men with palpable bilateral varicoceles. J UROL 1999;162:85-88. "

"A 42-year-old infertile man has a semen volume of 2 ml, sperm density of 5 million/ml, and decreased sperm motility. Physical examination demonstrates a grade 3 left varicocele and grade 1 right varicocele. His partner's evaluation is normal. The next step is: A. transrectal ultrasonography. B. clomiphene citrate (Clomid™). C. intrauterine insemination. D. left varicocelectomy. E. bilateral varicocelectomy."

"C: ""internal fixation of the femur."" was the correct answer. Approximately 50% of patients with multi-organ metastases from RCC exhibit evidence of skeletal involvement. It has been estimated that between 15% and 30% of such skeletal lesions are solitary. Eighty percent of skeletal metastases occur in the axial skeleton, thoracic/lumbar spine, and pelvis. When long bones are involved, only the proximal portions are characteristic targets for metastatic disease. Surgical treatment of bony metastases is indicated for weight-bearing bones with lytic lesions > 3 cm. Internal stabilization or replacement of the destroyed periarticular segment often results in significant pain relief and tremendously improves the patient's quality of life. For a lesion this large, XRT or strontium are likely to be ineffective. This patient needs surgery to prevent fracture as a priority over systematic treatments or pain management alone. Laitinen M, Parry M, Ratasvuori M, et al: Survival and complications of skeletal reconstructions after surgical treatment of bony metastatic renal cell carcinoma. EUR J SUR ONCOL 2015;41:886-892. D'Amico AV, Crook JM, Beard CJ: Radiation therapy for prostate cancer, Wein, AJ, Kavoussi LR, Novick AC, Partin AW, Peters CA (eds): CAMPBELL-WALSH UROLOGY, ed 10. Philadelphia, Elsevier Saunders, 2012, vol 3, chap 104, p 2870. "

"A 42-year-old man had a right radical nephrectomy two years ago for a Stage pT3aN0MX RCC. He now has right hip pain. Bone scan shows intense uptake in the right hip area and plain films of the right femur show a 3.5 cm lytic lesion. The next step is: A. external beam radiation to the femur. B. strontium-89. C. internal fixation of the femur. D. pain management. E. systemic targeted therapy."

"D: ""nephroureterectomy."" was the correct answer. The best answer involves nephrouretectomy in this setting with hemodialysis and possible future renal transplantation. The use of BCG in patients with ileal conduit urinary diversion is associated with up to a 10% risk of sepsis due to absorption. Partial nephrectomy in the setting of multifocality is associated with an unacceptable recurrence rate. Systemic chemotherapy would be poorly tolerated in the setting of a solitary kidney and not effective for CIS. The ileal conduit should be left in-situ and could be used in the future if renal transplantation is planned. Thalmann GN, Markwalder R, Walter B, Studer UE: Long-term experience with bacillus Calmette-Guerin therapy of upper urinary tract transitional cell carcinoma in patients not eligible for surgery. J UROL 2002;168:1381-1385. Sagalowsky AI, Jarrett TW, Flanigan RC: Urothelial tumors of the upper urinary tract and ureter, Wein, AJ, Kavoussi LR, Novick AC, Partin AW, Peters CA (eds): CAMPBELL-WALSH UROLOGY, ed 10. Philadelphia, Elsevier Saunders, 2012, vol 2, chap 53, pp 1525-1528. "

"A 51-year-old man with a solitary kidney has multifocal high grade T1 urothelial carcinoma in the left upper pole and multifocal CIS in the left ureter three years after cystectomy and ileal conduit. The next step is: A. retrograde upper tract BCG. B. percutaneous resection followed by antegrade BCG. C. partial nephrectomy followed by retrograde upper tract BCG. D. nephroureterectomy. E. nephroureterectomy with conduit excision."

"E: ""report as a SERS (serious event reporting system) event."" was the correct answer. Retention of vaginal packs for > 24 hours post-surgery does place the patient at risk for toxic shock syndrome, however, antibiotic therapy is not indicated in an asymptomatic patient. In the clinical situation described, treatment should be with outpatient observation of the individual and with notification of the hospital SERS (Serious Event Reviews System) program. This system will review the process by which this complication occurred and recommend steps to prevent its recurrence. The Joint Commission on the Accreditation of Health Care Organizations (JCAHC) is a national association existing since 1951 that accredits and certifies more than 20,000 health care organizations and programs in the United States. Joint Commission accreditation and certification is performed to verify that the health care organization undergoing review meets or exceeds current national standards of care, and is recognized nationwide as a symbol of quality. Accreditation is necessary for financial reimbursement of the institution by select insurers and other third parties and may be required in select states to care for patients. JACHO recognizes and rewards hospitals for running quality improvement projects, such as SERS, but does not establish or maintain the quality improve project; the latter is a function of the individual hospital or surgical center involved. There is no need to notify your malpractice insurer or the insurer of the outpatient surgical center regarding this finding unless a serious consequence of its action should occur or comments from the patient are a concern. The Joint Commission. Frequently asked questions: Retained foreign object after surgery. http://www.jointcommission.org/assets/1/18/retained_foreign_objects_faqs.pdf "

"A 43-year-old asymptomatic woman returns three days after an uncomplicated outpatient vaginal sling surgery with a vaginal pack still in place. The next step is to remove vaginal pack and: A. administer I.V. antibiotics. B. call your malpractice insurer. C. notify the insurer of the outpatient surgical center. D. notify Joint Commission of Health (JCAHO). E. report as a SERS (serious event reporting system) event."

"D: ""radical cystectomy."" was the correct answer. T1 tumors with aggressive features (tumor size > 3 cm, micropapillary histology, and lymphovascular invasion) have an increased risk of progression and should undergo definitive cystectomy that is highly effective for early stage tumors. While intravesical BCG is an option for the treatment of T1 bladder cancer, the high risk features of this tumor leave the patient at a very high risk of relapse and progression. This is inappropriate given his young age and excellent health. Partial cystectomy is not the best choice for tumor control given the high risk of multifocal recurrences within the remaining bladder. Prophylactic radiotherapy and systemic chemotherapy have not been shown to reduce the risk of recurrence in individuals with T1 disease. Micropapillary urothelial carcinoma is unlikely to respond to intravesical therapies. The use of neoadjuvant chemotherapy in non-muscle invasive disease is unproven, so immediate cystectomy is most appropriate. Wood DP: Urothelial tumors of the bladder, Wein, AJ, Kavoussi LR, Novick AC, Partin AW, Peters CA (eds): CAMPBELL-WALSH UROLOGY, ed 10. Philadelphia, Elsevier Saunders, 2012, vol 3, chap 80, p 2331. "

"A 43-year-old man has a 4 cm micropapillary urothelial carcinoma that extensively invades the lamina propria near the bladder dome. Muscularis propria is present and uninvolved. The next step is: A. restaging TURBT. B. partial cystectomy. C. neoadjuvant chemotherapy followed by radical cystectomy. D. radical cystectomy. E. chemotherapy."

"A: ""proceed with MRI scan."" is correct. Performance of an MRI scan in patients with a sacral neuromodulation device in-situ has been considered to be a potential hazard secondary to the interaction of the MRI magnet with the metal leads or implanted pulse generator (IPG). Specifically, there is concern that this interaction could result in tissue damage arising from motion, dislocation or torquing of the implanted pulse generator (IPG), heating of the leads, and/or permanent damage to the IPG. In April 2012, the FDA approved the use of head (brain) MRI scans in patients implanted with the newer (InterStim II) IPG, the latter stimulation device has been in use since 2007. Since a head MRI scan is needed, this patient can proceed with the MRI scan. The physician should be aware that an FDA warning still exists regarding performing an MRI scan of the spine in the setting of an implanted neurostim device and removal of the sacral neuromodulation device maybe requested by the radiologist prior to performing the study. However, multiple studies currently exist that document that a 1.5 Tesla MRI scan of the spine may be safely performed provided the sacral neuromodulation is out of the isocenter of the MRI scanner (remote site). To date, multiple patients positioned with the IPG outside of the magnet bore have undergone remote site MRI with minimal or no complications reported in 97% of the patients. Complications, when they occurred, were device-related problems, such as malfunction or failure, with minimal to no patient-related problems. Most radiologists now consider remote site MRI scan of the spine to be a safe option following implantation of an InterStim II device. U.S. Food and Drug Administration April 2012 Premarket Supplemental Approvals. http://www.fda.gov/medicaldevices/productsandmedicalprocedures/deviceapprovalsandclearances/pmaapprovals/ucm308481.htm. Accessed February 9, 2015. Medtronic Guidelines for InterStim Therapy neurostimulation systems. http://manuals.medtronic.com/wcm/groups/mdtcom_sg/@emanuals/@era/@neuro/documents/documents/contrib_119885.pdf. Accessed February 9, 2015. Chermansky CJ, Krlin RM, Holley DM: Magnetic resonance imaging following Interstim. An institutional experience with imaging safety and patient satisfaction. NEURO and URODYNAMICS 2011;30;1486-1488. "

"A 44-year-old woman with an InterStim II™, sacral neuromodulation device, needs to undergo an MRI scan of the brain. The next step is: A. proceed with MRI scan. B. reprogram battery to MRI compatible setting. C. remove lead. D. remove battery. E. remove lead and battery."

"B: ""cisplatin-based neoadjuvant chemotherapy."" was the correct answer. A recent secondary analysis of the SWOG neoadjuvant chemotherapy trial testing M-VAC followed by cystectomy vs. cystectomy alone for muscle invasive bladder cancer showed that the survival benefit to chemotherapy in patients with mixed histology including squamous or glandular differentiation was actually superior to that seen in patients with pure urothelial cancer. Partial cystectomy is not appropriate for large urothelial carcinomas regardless of their location. Radical cystoprostatectomy and ileal conduit will be appropriate after chemotherapy or if the patient cannot safely have chemotherapy due to comorbidities or other constraints such as poor renal function. Adriamycin-based chemotherapy is used for pure squamous cell carcinoma of other primary sites but has no role in urothelial carcinoma. Chemotherapy and XRT are not preferred in a patient that is otherwise a surgical candidate. If the patient had pure squamous cell carcinoma, immediate radical cystoprostatectomy would be appropriate. Wood DP: Urothelial tumors of the bladder, Wein AJ, Kavoussi LR, Novick AC, Partin AW, Peters CA (eds): CAMPBELL-WALSH UROLOGY, ed 10. Philadelphia, Elsevier Saunders, 2012, vol 3, chap 80, p 2309. "

"A 45-year-old man with a T6 spinal cord injury managed with an indwelling urethral catheter is diagnosed with a 6 cm T2 high grade urothelial carcinoma with squamous differentiation. The tumor is located near the dome and random biopsies are negative. The metastatic evaluation is negative. The next step is: A. partial cystectomy. B. cisplatin-based neoadjuvant chemotherapy. C. Adriamycin™ based neoadjuvant chemotherapy. D. chemotherapy and XRT. E. radical cystoprostatectomy."

"E: initiate potassium citrate is correct. Thiazides are frequently used to treat hypercalciuric calcium nephrolithiasis. Following initiation of the thiazides, up to 30% of patients will develop hypokalemia and 3-5% will develop new onset of hyperglycemia. The mechanism traditionally associated with this increased risk of thiazide-associated diabetes mellitus is a reduction in serum potassium. A meta-analysis of 59 studies involving thiazide treatment found a significant correlation between the degree of diuretic-induced hypokalemia and an increase in plasma glucose. The increase in plasma glucose levels are significantly increased if the serum potassium levels fall below 3.5 mEq/l. The mechanism of this glucose increase is related to the fact that low plasma potassium levels impair insulin secretion, thereby increasing plasma glucose. In addition to the hypokalemia induced by hyperglycemia, hydrochlorothiazide can also induce hyperuricemia and hyperlipidemia (hypertriglyceridemia). It is noteworthy that if hyperlipidemia develops, the increase in free fatty acids will cause damage to the pancreatic beta-cells secretion of insulin and will further impair glucose metabolism. There is substantial clinical evidence that prevention of hypokalemia with K+ supplementation (potassium citrate), potassium-sparing agents (triamterene), or with dietary supplements such as a banana per day, either prevents or significantly lessens the degree of glucose intolerance, and, therefore, should be initiated at either the onset of thiazide therapy or upon the discovery of glucose intolerance. Pearle M, Goldfarb DS, Assimos DG, et al: Medical management of kidney stones: AUA Guideline: AUA Guideline. American Urological Association Education and Research, Inc., 2014. http://www.auanet.org/education/guidelines/management-kidney-stones.cfm"

"A 46-year-old man with recurrent calcium nephrolithiasis secondary to hypercalciuria is treated with hydrochlorothiazide. Follow-up evaluation at six months reveals a serum potassium of 3.5 mEq/l and new onset of hyperglycemia with a fasting blood sugar of 200 mg/dl. The next step is: A. hemoglobin A1C. B. glucose tolerance test. C. initiate diabetic diet. D. discontinue hydrochlorothiazide. E. initiate potassium citrate."

"C: ""unmeshed split thickness skin graft."" was the correct answer. The highest probability of 100% graft take and best cosmetic results occur with penile skin reconstruction using an unmeshed thick (0.012 - 0.015 inch) split thickness skin (STS) graft. Meshed or thinner STS grafts have more of a tendency to contract which compromises penile functionality and cosmesis. Local skin flaps from the abdomen or thigh generally do not produce the same cosmetic outcomes. Full thickness skin grafts may provide adequate cosmesis but are associated with a higher incidence of failure of the graft to take. Morey AF, Dugi DD III: Genital and lower urinary tract trauma, Wein, AJ, Kavoussi LR, Novick AC, Partin AW, Peters CA (eds): CAMPBELL-WALSH UROLOGY, ed 10. Philadelphia, Elsevier Saunders, 2012, vol 3, chap 88, p 2513. "

"A 48-year-old man has undergone extensive penile skin debridement secondary to Fournier's gangrene. Reconstruction for the penile shaft should include the use of a(n): A. local skin flap. B. meshed split thickness skin graft. C. unmeshed split thickness skin graft. D. meshed full thickness skin graft. E. unmeshed full thickness skin graft."

"C: ""percutaneous drainage of fluid collection."" was the correct answer. Urine leak following partial nephrectomy occurs in up to 15% of cases. Provided a postoperative drain is left in situ, spontaneous closure of the urinary leak occurs within two to four weeks. In the case of an unrecognized or delayed urinary leak, the presence of an adjacent urinoma will prevent fistula closure and predispose the patient to infection/abscess formation. Percutaneous drainage of the urinoma is the preferred method used to control an unrecognized or delayed pyelocutaneous fistula. If the leak does not heal with drainage of the urinoma, consideration should be given to the possibility of either ureteral/bladder obstruction or bladder dysfunction as a cause of the persistent fistula. In these situations, a cystoscopy with a retrograde pyelogram followed by ureteral stent and urethral catheter placement should be pursued. The concomitant urethral catheter is used to aid healing by preventing high pressure reflux up the ureteral stent and/or to treat bladder outlet obstruction or voiding dysfunction as an etiology for the persistent urinary fistula. Kavoussi LR, Schwartz MJ, Gill IS: Laparoscopic surgery of the kidney, Wein, AJ, Kavoussi LR, Novick AC, Partin AW, Peters CA (eds): CAMPBELL-WALSH UROLOGY, ed 10. Philadelphia, Elsevier Saunders, 2012, vol 2, chap 55, p 1668."

"A 48-year-old man undergoes partial nephrectomy for a 3 cm renal mass. His flank drain is removed on the third postoperative day. Seven days later, he has clear fluid dripping from the flank drain site. He is otherwise asymptomatic. CT scan demonstrates a 5 cm by 10 cm fluid collection adjacent to the kidney with extravasation of contrast from the collecting system. The next step is: A. observation. B. urethral catheter. C. percutaneous drainage of fluid collection. D. percutaneous nephrostomy. E. ureteral stent."

"D: ""angiotensin receptor antagonist."" was the correct answer. Hypertension is a very common presenting sign in patients with autosomal dominant polycystic kidney disease. The mechanism appears to be compression of the intrarenal vessels from the cysts causing ischemia and, thus, renin-mediated hypertension. Early recognition and treatment is important to avoid hypertensive-related renal deterioration. Because the mechanism is renin-mediated, the most logical choice in antihypertensive medication is either an ACE inhibitor or angiotensin receptor antagonist. Pope JC IV: Renal dysgenesis and cystic disease of the kidney, Wein, AJ, Kavoussi LR, Novick AC, Partin AW, Peters CA (eds): CAMPBELL-WALSH UROLOGY, ed 10. Philadelphia, Elsevier Saunders, 2012, vol 4, chap 118, pp 3173-3176. "

"A 50-year-old man has autosomal dominant polycystic kidney disease, hypertension, and normal renal function. The best medication for his hypertension is a(n): A. alpha-blocker. B. beta-blocker. C. calcium channel blocker. D. angiotensin receptor antagonist. E. thiazide diuretic."

"D: ""bladder neck closure and suprapubic tube."" was the correct answer. This patient has the classic findings of an eroded urethra secondary to long-term indwelling catheter use. She has essentially lost almost all of her urethral length and has a patulous wide open bladder neck and meatus. Placement of a larger catheter with a larger balloon will only worsen the erosion and would likely not improve the incontinence. A minimum of 1 cm urethral length is believed to be needed to allow for obstruction of the urethra by bulking agents or by sling placement. Although one could attempt urethral lengthening and then proceed with either bulking agents or sling placement, use of these procedures following urethral reconstruction has been found to be fraught with complications. A suprapubic (SP) tube alone would not be sufficient as urine would still leak per her incompetent bladder neck and urethra. This patient would be best served with a bladder neck closure (which could be done via a vaginal or transabdominal approach) and SP tube placement. Other options which could be considered would be bladder neck closure and construction of a continent urinary stoma if she is willing to perform CIC, bladder neck closure and ileovesicostomy, or supra-vesical non-continent urinary diversion. Cespedes RD, Gerboc JL: Other therapies for storage and emptying failure, Wein, AJ, Kavoussi LR, Novick AC, Partin AW, Peters CA (eds): CAMPBELL-WALSH UROLOGY, ed 10. Philadelphia, Elsevier Saunders, 2012, vol 3, chap 75, pp 2191-2193. "

"A 50-year-old woman with a T4 complete spinal cord injury has been managed with an indwelling catheter since her injury ten years ago. She currently complains of constant leakage of urine around her 18 Fr urethral catheter. Examination reveals a patulous urethral meatus with a diameter of two fingers and a urethral length less than 1 cm. The next step is: A. 22 Fr catheter with 30 ml balloon. B. peri-urethral calcium hydroxylapatite (Coaptite™) injection. C. suprapubic tube. D. bladder neck closure and suprapubic tube. E. autologous fascial sling."

"A: ""oral PDE-5 inhibitor."" is correct. The clinical picture is consistent with Peyronie's disease. While the criteria for surgery includes the presence of stable and mature disease of at least a 12 month interval from its onset, surgical correction is not clearly warranted in this situation given the minimal degree (< 30 degrees) of penile angulation that may not preclude sexual intercourse. Penile prosthesis is not indicated in this patient due to the fact that he has adequate penile blood flow and minimal degree of penile curvature. Initial therapy may be best directed to improving erectile function using non-invasive oral pharmacotherapy such as PDE-5 therapy. Intracavernous pharmacotherapy is not recommended since it may exacerbate penile scarring. Jordan GH, McCammon KA: Peyronie's disease, Wein, AJ, Kavoussi LR, Novick AC, Partin AW, Peters CA (eds): CAMPBELL-WALSH UROLOGY, ed 10. Philadelphia, Elsevier Saunders, 2012, vol 1, chap 28, pp 798, 802. "

"A 52-year-old man with erectile dysfunction has a penile curvature less than 30° that developed spontaneously 18 months ago and is unresponsive to oral Vitamin E. Doppler ultrasound reveals normal bilateral cavernous arterial flow, mild venous leak, and a dorsal penile plaque. The next step is: A. oral PDE-5 inhibitor. B. intracavernous pharmacotherapy. C. corporal plication. D. plaque incision and grafting. E. penile prosthesis surgery."

"C: ""remove urethral catheter and repeat stress maneuvers."" was the correct answer. The AUA/SUFU Urodynamics Guidelines suggest that clinicians should repeat stress testing with the urethral catheter removed in patients suspected of having stress incontinence (SUI) who do not demonstrate SUI with the catheter in place. Filling the bladder to a higher volume than maximum capacity, would not be indicated. Changing the prolapse reduction method would not be expected to change the findings. It should be noted the guidelines on urodynamic assessment of stress incontinence states that there is no standardized method with which to reduce prolapse during a urodynamic study. Patients with marked vaginal prolapse are at a high risk for concomitant stress incontinence and presence of a urethral catheter may prevent manifestation of the stress incontinence during urodynamics. Removal of the catheter (with continued reduction of prolapse) allows for complete evaluation and is the next step prior to proceeding with surgical planning. Winter JC, Dmochowski RR, Goldman HB, et al: Adult urodynamics: AUA/SUFU GUIDELINE. American Urological Association Education and Research, Inc., 2012. http://www.auanet.org/education/adult-urodynamics.cfm "

"A 53-year-old woman has POP-Q stage 3 anterior compartment prolapse and urinary incontinence. No stress urinary incontinence is noted during urodynamics with cough or Valsalva maneuver performed at a maximum bladder capacity of 400 ml with or without reduction of pelvic organ prolapse by physical manipulation. The next step is: A. fill the bladder to 100 ml beyond maximal capacity and repeat stress maneuvers. B. repeat urodynamic study with prolapse reduction performed by a vaginal pessary. C. remove urethral catheter and repeat stress maneuvers. D. proceed with prolapse repair with no anti-incontinence procedure. E. proceed with prolapse repair with sling."

"E: ""denosumab."" was the correct answer. Use of denosumab or bisphosphonate therapy in men with prostate cancer has demonstrated two outcomes: prevention of osteoporosis and reduction of skeletal-related events. Denosumab and zoledronic acid both reduce analgesic use and the time to a skeletal event compared to placebo in this setting. However, these changes do not influence survival. None of the other listed agents will prevent bone events. The use of other agents such as enzalutamide or abiraterone have been shown to increase progression-free survival after the development of castrate resistant prostate cancer, but their immediate use in the hormone-sensitive setting is unknown. Estramustine can be associated with thromboembolic events and should not be used in this setting. Antonarakis ES, Carducci MA, Eisenberger MA: Treatment of castration-resistant prostate cancer, Wein, AJ, Kavoussi LR, Novick AC, Partin AW, Peters CA (eds): CAMPBELL-WALSH UROLOGY, ed 10. Philadelphia, Elsevier Saunders, 2012, vol 3, chap 110, p 2964. Saad F, Gleason DM, Murray R, et al: A randomized placebo-controlled trial of zoledronic acid in patients with hormone-refractory metastatic prostate carcinoma. J NAT CANCER INST 2002;94:1458-1468."

"A 54-year-old man with prostate cancer has skeletal metastases. Three months after beginning LH-RH agonist therapy, his PSA is undetectable. The likelihood of developing further bone pathology is best reduced with: A. enzalutamide. B. abiraterone. C. bicalutamide. D. estramustine. E. denosumab."

"C: ""prostatic urethral lift (UroLift™)."" was the correct answer. The urodynamics are consistent with bladder outlet obstruction with a voiding pressure close to 100 cm H2O and a maximal flow close to 9 ml/sec. All of the options address bladder outlet obstruction, however, only the prostatic urethral lift does not cause retrograde ejaculation. TUIP, TUMT, PVP, and HOLEP all place patients at risk for retrograde ejaculation with the risk ranging from 9.2-67%. The prostatic lift obtained FDA approval in 2013 for the treatment of symptomatic BPH. The procedure does not involve cutting or resection of prostate tissue, instead relying on an implant to ""lift"" or pull the prostatic tissue away and open the prostatic urethra. To date, there have been no reports of erectile dysfunction or retrograde ejaculation in patients treated with this modality. Perera M, Roberts MJ, Doi SA, et al: Prostatic urethral lift improves urinary symptoms and flow while preserving sexual function for men with benign prostatic hyperplasia: A systematic review and meta-analysis. EUR UROL 2015;67:704-713. Cornu JN, Ahyai S, Bachmann A, et al: A systematic review and meta-analysis of functional outcomes and complications following transurethral procedures for lower urinary tract symptoms resulting from benign prostatic obstruction: EUR UROL 2015:67:1066-1096. "

"A 55-year-old man has bothersome LUTS poorly responsive to medical therapy. Pressure-flow urodynamics are shown. He refuses therapy that may result in retrograde ejaculation. The next step is: A. TUIP. B. TUMT. C. prostatic urethral lift (UroLift™). D. photoselective vaporization of the prostate (PVP). E. Holmium laser enucleation of the prostate (HOLEP)."

D: ""bedrest, serial monitoring, and transfusions as needed."" was the correct answer. Complications of partial nephrectomy include hemorrhage, urinary fistula formation, ureteral obstruction, renal insufficiency, and infection. Significant intraoperative bleeding can occur in patients who are undergoing partial nephrectomy. The need for early control and ready access to the renal artery is emphasized. Postoperative hemorrhage may be self-limiting if confined to the retroperitoneum with or without associated gross hematuria. The initial management of postoperative hemorrhage is expectant with bed rest, serial hemoglobin and hematocrit determinations, frequent monitoring of vital signs, and blood transfusions as needed. Angiography may be helpful if bleeding persists, to localize actively bleeding segmental arteries, which may be controlled by angioinfarction. Severe intractable hemorrhage may necessitate re-exploration with early control of the renal vessels and ligation of the active bleeding points. Ureteral stent is not indicated. Kenney PA, Wotkowicz C, Libertino JA: Contemporary open surgery of the kidney, Wein, AJ, Kavoussi LR, Novick AC, Partin AW, Peters CA (eds): CAMPBELL-WALSH UROLOGY, ed 10. Philadelphia, Elsevier Saunders, 2012, vol 2, chap 54, p 1624. Weizer A, Wolf JS: Complications of laparoscopic renal surgery. Taneja SS (ed): COMPLICATIONS OF UROLOGIC SURGERY, ed 4. Philadelphia, Elsevier Saunders, 2010, chap 29, p 349. "

"A 55-year-old man undergoes a left lower pole partial nephrectomy for a 3.5 cm solid renal mass. The contralateral kidney is normal. Two days postoperatively, his serum hematocrit drops from 32% to 28% over 24 hours. Urine is dark burgundy in color. Vital signs are stable. The next step is: A. angiography and embolization of bleeding vessels. B. reoperation with suture ligation of bleeding vessels. C. reoperation with nephrectomy. D. bedrest, serial monitoring, and transfusions as needed. E. ureteral stent placement."

"B: ""obtain a PVR."" was the correct answer. Approximately 5% of patients undergoing bladder onabotulinumtoxinA injections may develop urinary retention. Accordingly, patients with worsening urinary incontinence post-injection should undergo a PVR assessment to rule-out overflow incontinence. Antimuscarinics would not be advisable if her residual were high. Urodynamics would be premature at this early stage after injection. Repeating onabotulinumtoxinA injections may be an option at some point if she is not retaining urine. Sacral neuromodulation, while an option, is not recommended in the presence of known neurologic conditions and would only be considered after an elevated residual has been ruled-out. Andersson KE, Wein AJ: Pharmacologic management of lower urinary tract storage and emptying failure, Wein, AJ, Kavoussi LR, Novick AC, Partin AW, Peters CA (eds): CAMPBELL-WALSH UROLOGY, ed 10. Philadelphia, Elsevier Saunders, 2012, vol 3, chap 68, p 1987. "

"A 55-year-old woman with multiple sclerosis has worsening urinary incontinence three weeks after undergoing an onabotulinumtoxinA injection. The next step is: A. start an antimuscarinic. B. obtain a PVR. C. perform a complete urodynamic study. D. repeat onabotulinumtoxinA with higher dose. E. initiate sacral neuromodulation."

"A: ""observation."" is correct. This patient represents index patient 1 in the AUA Guideline for Castration-Resistant Prostate Cancer (CRPC). The guidelines make a primary recommendation for observation for this patient. The guidelines also make the recommendation that clinicians should not offer systemic chemotherapy or immunotherapy to patients with non-metastatic CRPC outside the context of a clinical trial, thus, making the use of sipuleucel-T, abiraterone with prednisone, and enzalutamide incorrect in this patient. As an option, the guidelines state that clinicians may offer treatment with first- generation anti-androgens (flutamide, bicalutamide and nilutamide) or first generation androgen synthesis inhibitors (ketoconazole with prednisone) to select patients with non-metastatic CRPC who are unwilling to accept observation. However, given the lack of data suggesting any clinical benefit and given the potential side effects of therapy, the panel only recommended the use of anti-androgens in this patient population only if patients are unwilling to accept the recommendation of observation. There is no statement that this patient is unwilling to accept observation, and, accordingly, observation should be the initial step and recommendation. Cookson MS, Kibel AS, Dahm P, et al: Castration-Resistant Prostate Cancer: AUA Guideline. American Urological Association Education and Research, Inc., 2015. http://www.auanet.org/education/guidelines/castration-resistant-prostate-cancer.cfm "

"A 58-year-old man has asymptomatic non-metastatic castration-resistant prostate cancer demonstrated by a rising PSA on LH-RH agonist therapy. The next step is: A. observation. B. bicalutamide. C. sipuleucel-T. D. abiraterone and prednisone. E. enzalutamide."

"C: ""benign prostate enlargement."" was the correct answer. The pressure flow urodynamic study reveals detrusor overactivity (DO), coexisting with urodynamic evidence of bladder outlet obstruction. In this study, external sphincter activity is noted with each episode of DO. This is a normal guarding response to an uninhibited bladder contraction and will occur in an attempt to prevent urge incontinence. When given permission to void, EMG activity, which measures the electronic potential of the external sphincter, appropriately quiets and the patient is able to void. Patients with multiple sclerosis (MS) may have DO which is seen in the urodynamic tracing. However, if high-pressure voiding was due to voiding dysfunction secondary to MS then the EMG would be active throughout the voiding phase. Hinman Syndrome (also termed non-neurogenic neurogenic bladder) and dysfunctional voiding are similar entities and would be characterized by simultaneous bladder emptying and voluntary striated sphincter contraction. Detrusor-external sphincter dyssynergia (DESD) is also characterized by simultaneous detrusor and involuntary striated sphincter contraction. The likely etiology of this patient's urodynamic findings is benign prostate enlargement with secondary DO. Bladder outlet obstruction on a urodynamic study may be determined by use of either detrusor pressure flow rate nomograms, or alternatively may be diagnosed when two of the following four urodynamic criteria are met; flow rate < 12 ml/sec, detrusor pressure at peak flow > 50 cm/H2O, elevated urethral resistance (Pdet at Qmax divided by 2 X Qmax > 0.2), or significant residual urine (> 100-150 ml) in the presence of high detrusor voiding pressures, i.e., detrusor pressures > 50 cm of H2O at maximum flow rate. This patient has detrusor pressures at peak flow > 50 cm/H2O, an elevated urethral resistance of approximately 0.3, and a peak uroflow at the cut-off level. Nitti VW: Urodynamic and video-urodynamic evaluation of the lower urinary tract, Wein, AJ, Kavoussi LR, Novick AC, Partin AW, Peters CA (eds): CAMPBELL-WALSH UROLOGY, ed 10. Philadelphia, Elsevier Saunders, 2012, vol 3, chap 62, pp 1861, 1862. Wein AJ, Dmochowski RR: Neuromuscular dysfunction of the lower urinary tract, Wein, AJ, Kavoussi LR, Novick AC, Partin AW, Peters CA (eds): CAMPBELL-WALSH UROLOGY, ed 10. Philadelphia, Elsevier Saunders, 2012, vol 3, chap 65, pp 1918, 1936. Nitti VW: Pressure flow urodynamic studies: The gold standard for diagnosing bladder outlet obstruction. REV UROL 2005;7:S14-S21. "

"A 58-year-old man with bothersome LUTS undergoes urodynamics as shown. His postvoid residual urine is 0 ml. The likely etiology of his LUTS is: A. dysfunctional voiding. B. detrusor-external sphincter dyssynergia. C. benign prostate enlargement. D. multiple sclerosis. E. Hinman syndrome."

"B: ""right inguinal lymphadenectomy."" was the correct answer. Partial penectomy with an adequate surgical margin is the treatment of choice for urethral tumors localized to the distal half of the penis. Palpable inguinal lymph nodes occur in about 20% of cases and almost always represent metastatic disease, in contrast to penile cancer, in which a substantial percentage of palpable nodes may be inflammatory. Lymphatics from the male anterior urethra drain into the superficial and deep inguinal lymph nodes and occasionally into the external iliac nodes. Ilioinguinal lymphadenectomy is indicated in the presence of palpable inguinal lymph nodes without evidence of metastatic disease. Unlike penile cancer, benefit from prophylactic contralateral inguinal node dissection or XRT has not been demonstrated in urethral cancer. Although some instances of tumor control by radiation have been reported, in general, XRT has been reserved for patients with early-stage lesions who refuse surgery. The combination of chemotherapy and radiation has shown some success in a small number of patients, however, squamous cell cancers respond poorly to the M-VAC regimen. Sharp DS, Angermeier KW: Surgery of penile and urethral carcinoma, Wein, AJ, Kavoussi LR, Novick AC, Partin AW, Peters CA (eds): CAMPBELL-WALSH UROLOGY, ed 10. Philadelphia, Elsevier Saunders, 2012, vol 1, chap 35, p 948. "

"A 60-year-old man has a 3 cm squamous cell carcinoma of the penile urethra (3 cm from the meatus) and a palpable 2 cm right inguinal lymph node. Metastatic survey is otherwise negative. The next steps are partial penectomy and: A. right inguinal XRT. B. right inguinal lymphadenectomy. C. bilateral inguinal lymphadenectomy. D. cisplatin-based chemotherapy. E. 5-FU and bilateral inguinal XRT."

"C: ""maintenance BCG with the first of three weekly doses at three months."" was the correct answer. BCG maintenance therapy is indicated for patients with CIS, even if focal, who achieve a complete response. The Southwest Oncology Group (SWOG) reported the most significant impact of maintenance therapy. Patients received a six-week induction course followed by three weekly instillations at three and six months and every six months, thereafter, for three years. This patient is at the three month point and should receive the first set of maintenance treatments as the next step. Estimated median recurrence-free survival was 76.8 months in the maintenance arm and 35.7 months in the control arm (P = .0001). Bladder biopsy is not necessary if the patient's cystoscopy and cytology are negative. Observation alone is not optimal therapy unless the patient has a contraindication or serious complication from BCG. Monthly BCG has been described and is commonly used, but has not been tested against the maintenance schedule proven to be effective in the large published SWOG trial. Jones JS, Larchian WA: Non-muscle-invasive bladder cancer (Ta, T1, and CIS), Wein AJ, Kavoussi LR, Novick AC, Partin AW, Peters CA (eds): CAMPBELL-WALSH UROLOGY, ed 10. Philadelphia, Elsevier Saunders, 2012, vol 3, chap 81, p 2335."

"A 60-year-old man is diagnosed with a single focus of CIS of the bladder. He is treated with six doses of intravesical BCG. A month after his last dose, his cystoscopy is normal and his cytology is negative. The next step is: A. bladder biopsy to confirm complete response. B. observation with regular surveillance cystoscopy and cytology. C. maintenance BCG with the first of three weekly doses at three months. D. maintenance BCG with the first of three weekly doses at six months. E. maintenance BCG single monthly dose for one year."

"D: ""the cause of this patient's LUTS cannot be differentiated between obstruction and underactive bladder."" was the correct answer. Uroflow evaluation with the determination of urinary residual is an important screening tool for patients with lower urinary tract symptoms. Normal values vary depending on age, sex, and volume voided. In men, urine flow declines with age while women will have minimal alterations with age. In general, in pre-pubertal males and females, the average peak flow rates range from 10-15 ml/sec. Post-puberty until age 45 years, the average peak uroflow rate in males is 21 ml/sec. The average peak flow rate for females is 18 ml/sec. Between the ages 46 to 65 years, the average peak uroflow rate for males will decrease to 12 ml/sec. The average peak uroflow rate for females will remain at 18 ml/sec. Between 66 to 80 years of age, the average peak uroflow rate for males will further decrease to 9 ml/sec. The average peak uroflow rate for females remains at 18 ml/sec. In general, provided the patient voids a minimum of 125-150 ml, urologists will find a peak flow rate of < 15 ml/sec in one third of patients evaluated (one standard deviation below the mean) and a peak flow rate of < 12 ml/sec in 5% (two standard deviations below the mean). In using the uroflow to evaluate patients, it is critical to note the following: 1) The uroflow represents the combined dynamics of the outflow tract and detrusor contractility, a decrease in peak uroflow may be due to either the obstruction of the outflow tract, poor detrusor contractility, or both. 2) There is minimal to no correlation of the peak uroflow to prostate symptom scores. Specifically, pharmacological therapy for BPH will frequently document significant improvement in symptom scores with minimal to no increase in peak uroflow. 3) Studies have found that patients with a peak uroflow of > 15 ml/sec have significantly less improvement in prostate symptoms scores following TURP compared to patients with a peak uroflow of < 15 ml/sec. The uroflow, being of prognostic value in this circumstance, enables the surgeon to determine how well surgical intervention will improve the patient's symptoms. McNicholas TA, Kirby RS, Lepor H: Evaluation and nonsurgical management of benign prostatic hyperplasia, Wein, AJ, Kavoussi LR, Novick AC, Partin AW, Peters CA (eds): CAMPBELL-WALSH UROLOGY, ed 10. Philadelphia, Elsevier Saunders, 2012, vol 3, chap 92, pp 2616-2617. Kumar V, Dhabalic J, Nelvigi G, et al: Age, gender and voided volume effect on peak uroflow rate and the uroflow nomogram. INDIAN J UROL 2009;4:461-466."

"A 60-year-old man with bothersome LUTS undergoes a non-invasive uroflow which is shown. He has a PVR of 100 ml. Based on these results: A. the study should be repeated until the total voided volume is greater than 180 ml. B. he would likely be obstructed if he was ten years older with the same uroflow results. C. he has a high likelihood of moderate to significant bother based on his AUA Symptom Score. D. the cause of this patient's LUTS cannot be differentiated between obstruction and underactive bladder. E. he would have a poorer outcome after prostatectomy compared to patients with a peak flow rate greater than 15 ml/sec."

"E: ""biopsy and fulguration."" was the correct answer. Interstitial cystitis (IC) is a clinical diagnosis characterized by diurnal and nocturnal urinary frequency and urgency, lower abdominal or suprapubic pain with bladder distension, and relief or improvement of pain with voiding. Symptoms should be present in the absence of identifiable pathology, e.g., UTI, and persisting for greater than nine months. Urodynamic evaluations will typically show a bladder capacity < 350 ml. The International Continence Society (ICS) prefers the term ""painful bladder syndrome"" (PBS) to describe this symptom complex, and reserves the diagnosis of IC for patients with ""typical cystoscopic and histological features following hydrodistention"". Classic cystoscopic findings are glomerulations and bladder wall petechial hemorrhage found following hydrodistention. The urologist must be aware, however, that these cystoscopic findings can also be seen upon hydrodistention in patients without IC and other diagnostic criteria, as noted above, must be met. In addition to glomerulations, Hunner originally described a characteristic lesion or ""ulcer"" in some patients with IC which has led to the classification of IC into 2 types: ulcerative and non-ulcerative. Approximately 15% of patients with IC will have extremely painful areas of inflammation in the bladder, well known as Hunner's lesions (formerly, Hunner's ulcers). The detection of a Hunner lesion is best possible at cystoscopy following hydrodistension. In patients with PBS, the hydrodistension is best done under general, epidural, or local anesthesia due to the exquisite pain it will often invoke within the patient. A Hunner's lesion or ""ulcer"" is a distinctive inflammatory lesion presenting a characteristic deep rupture through the mucosa and submucosa provoked by bladder distension. Despite the name that has been commonly used, the lesion is not a pathologic ulcer and hence, the term, Hunner's lesion is gradually being adopted worldwide. A Hunner's lesion presents as a circumscript, reddened mucosal area with small vessels radiating towards a central scar. This site ruptures with increasing bladder distension, resulting in petechial oozing of blood from the lesion in a waterfall manner. Biopsy of the lesion classically shows inflammatory cystitis, often with epithelial denudation. AUA Treatment Guidelines outline current treatments for Hunner's lesions to include: fulguration, laser therapy, and/or the injection of triamcinolone into the center of the ulcer. Although long-term outcomes regarding injection of triamcinolone are currently unavailable, fulguration and laser therapy have been found to be associated with slightly > 50% of patients having complete resolution of their symptoms, with another 25% having > 50% improvement of their symptoms following treatment. Unfortunately, long-term follow-up reveals approximately one-third of the patients that symptomatically improved or resolved their symptoms following fulguration will have a clinical recurrence of their disease process. It is noteworthy that polymyxa viruses have recently been found to be present within Hunner's lesions suggestive that antiviral medications or topical agents with an antiviral effect (Clorpactin®) may be of benefit to this class of IC patients. Hanno PM: Bladder pain syndrome (interstitial cystitis) and related disorders, Wein, AJ, Kavoussi LR, Novick AC, Partin AW, Peters CA (eds): CAMPBELL-WALSH UROLOGY, ed 10. Philadelphia, Elsevier Saunders, 2012, vol 1, chap 12, p 357. Hanno PM, Burks DA, Clemens JQ, et al: Guideline on the diagnosis and treatment of interstitial cystitis/bladder pain syndrome (2014). American Urological Association Education and Research, Inc., 2014. http://www.auanet.org/education/guidelines/ic-bladder-pain-syndrome.cfm"

"A 60-year-old woman has chronic dysuria and suprapubic discomfort when her bladder is full and during voiding. She has a normal physical exam and urinalysis. Flow rate reveals a total volume voided of 100 ml, peak of 14 ml/sec, average of 8 ml/sec, and a postvoid residual urine of 0 ml. Cystoscopy reveals a focal ulceration on the right lateral wall. Cytology is normal. The next step is: A. antihistamines (loratadine). B. tricyclic antidepressants (amitriptyline). C. oral sodium pentosan polysulfate (Elmiron). D. intravesical dimethyl sulfoxide (DMSO). E. biopsy and fulguration."

"D: ""assessment of life expectancy."" was the correct answer. This patient has intermediate-risk, clinically localized prostate cancer. According to the AUA Guidelines for the management of clinically localized prostate cancer, as a standard, an assessment of the patient's life expectancy, overall health status, and tumor characteristics should be performed prior to making any treatment decisions. Life expectancy, not patient age, is a major factor to consider in treatment selection. When life expectancy is long, localized prostate cancer can be a cause of morbidity and mortality. On the other hand, when life expectancy is relatively short, competing causes for mortality reduce the chance that a man will experience disease progression or die from prostate cancer. Imaging studies, including CT scan, bone scan, and ProstaScint® scans are generally not indicated in the pretreatment evaluation of patients with clinically localized disease. This is especially true in patients with low and intermediate risk disease because the yield is so low to preclude their usefulness. Urodynamics are not indicated. The role of molecular testing in this setting remains uncertain. Thompson I, Thrasher JB, Aus G, et al: Guideline for the management of clinically localized prostate cancer: 2007 update. MANAGEMENT OF CLINICALLY LOCALIZED PROSTATE CANCER GUIDELINE. American Urological Association Education and Research, Inc., 2007. http://www.auanet.org/education/guidelines/prostate-cancer.cfm"

"A 61-year-old man has a T1c, Gleason 7 prostate cancer with a PSA of 9.1 ng/ml. He has moderate LUTS and prostate volume of 42 ml. Before any treatment decisions are made, he should undergo: A. CT scan of abdomen and pelvis. B. bone scan. C. urodynamics. D. assessment of life expectancy. E. molecular testing."

"C: ""potassium citrate."" was the correct answer. Following ileal or colonic urinary reconstruction, patients are prone to the develop hyperchloremic, metabolic acidosis due to the exchange of ammonium ions (ammonium chloride) for carbonic acid and water. Treatment of chronic acidosis involves reduction of urinary contact time to ensure minimization of ammonium absorption. In the event the acidosis persists despite frequent catheterization, alkalinizing agents or inhibitors of chloride transport may utilized. Inhibitors of chloride transport, such as chlorpromazine or nicotinic acid, are generally not effective alone in the treatment of acidosis, but they may be used in conjunction with alkalinizing agents. In this case, the use of sodium bicarbonate and sodium citrate should be discouraged given the history of hypertension and the desire to limit sodium intake. As such, potassium citrate is the ideal first choice for treatment. Dahl DM, McDougal WS: Use of intestinal segments in urinary diversion, Wein, AJ, Kavoussi LR, Novick AC, Partin AW, Peters CA (eds): CAMPBELL-WALSH UROLOGY, ed 10. Philadelphia, Elsevier Saunders, 2012, vol 3, chap 85, pp 2442-2444."

"A 62-year-old man with an ileal neobladder has a serum chloride of 114 mEq/l, potassium of 3.8 mEq/l, creatinine of 1.8 mg/ml, and bicarbonate of 16 mEq/l, despite low urinary residuals, normal upper tracts, and timed voiding every three hours. He has a history of hypertension and diabetes mellitus. The next step is: A. sodium bicarbonate. B. sodium citrate. C. potassium citrate. D. chlorpromazine. E. nicotinic acid."

"C: ""CT scan with and without contrast."" was the correct answer. Lesion size is the most reliable clinical means to differentiate benign from malignant incidental adrenal tumors. Tumors < 4 cm in size are malignant in only 2% of cases. All of the listed radiographic techniques have been applied to adrenal tumor diagnosis, however, CT scan without and with contrast with measurement of contrast washout is the most reliable indicator for malignant mass. On non-contrast images, lipid-rich adrenal adenomas will be homogenous and will have < 10 Hounsfield units. For lesions with > 10 Hounsfield units, a washout contrasted CT scan is necessary to distinguish benign lipid-poor adenomas versus malignant masses. Malignant masses will have delayed washout of contrast maintaining an average of 30 Hounsfield units higher signal intensity at ten minutes following contrast administration when compared to benign lesions which washout rapidly. Kutikov A, Crispen PL, Uzzo RG: Pathophysiology, evaluation, and medical management of adrenal disorders, Wein AJ, Kavoussi LR, Novick AC, Partin AW, Peters CA (eds): CAMPBELL-WALSH UROLOGY, ed 10. Philadelphia, Elsevier Saunders, 2012, vol 2, chap 57, pp 1726-1728. "

"A 63-year-old man has an incidentally discovered 3.8 cm left adrenal mass with a negative endocrine evaluation. The test most likely to differentiate between malignant and benign histology is: A. MRI scan. B. duplex Doppler ultrasound. C. CT scan with and without contrast. D. MIBG. E. fluorodeoxyglucose PET."

"E: ""bilateral inguinal node dissection."" was the correct answer. This patient has pathologic stage T2 disease and is at high risk for occult inguinal node metastases. Regardless of inguinal lymph node status (palpable or not), these patients should undergo inguinal lymph node dissection. In this setting, antibiotics decrease the risk of inguinal wound infection, but response of lymph nodes to antibiotic treatment should not influence the decision for surgical staging. Whether inguinal nodes are palpable unilaterally or not palpable, bilateral inguinal node dissection is indicated. In this case, superficial inguinal dissection is indicated on the left and superficial and deep dissection on the right. Pettaway CA, Lance RS, Davis JW: Tumors of the penis, Wein, AJ, Kavoussi LR, Novick AC, Partin AW, Peters CA (eds): CAMPBELL-WALSH UROLOGY, ed 10. Philadelphia, Elsevier Saunders, 2012, vol 1, chap 34, pp 920-922. A: ""water restriction."" is correct. This patient has hyponatremia due to SIADH from self-medicating with DDAVP. He has a chronic condition (> 48 hours) with no symptoms and modest hyponatremia that can be corrected slowly over time. Removing the inciting agent (DDAVP) together with water restriction would be the optimal treatment. In the absence of symptoms, the addition of diuretics and saline replacement are not necessary. There is no role for oral sodium bicarbonate in this setting. Shoskes DA, McMahon AW: Renal physiology and pathophysiology, Wein, AJ, Kavoussi LR, Novick AC, Partin AW, Peters CA (eds): CAMPBELL-WALSH UROLOGY, ed 10. Philadelphia, Elsevier Saunders, 2012, vol 2, chap 38, pp 1039-1041."

"A 63-year-old man has partial penectomy for squamous cell carcinoma invading the glans penis (pT2). Pelvic CT scan demonstrates a 1.5 cm right inguinal lymph node that cannot be palpated. The next step is: A. antibiotics and repeat CT scan in six weeks. B. CT-guided aspiration of the suspicious node. C. right sentinel node biopsy. D. right inguinal node dissection. E. bilateral inguinal node dissection."

"A: ""observation."" is correct. In a patient with a low grade pT1 penile cancer without lymphovascular invasion, the rate of inguinal metastases is < 10% and, thus, these patients should be observed. In this low risk patient, there is no indication for sentinel lymph node biopsy, prophylactic radiation, or lymphadenectomy. In general, bilateral inguinal lymphadenectomy is reserved for patients with pT2 or high grade disease. In addition, consideration for immediate bilateral prophylactic node dissection can be given for low grade pT1 tumors that harbor findings that are highly indicative of occult metastasis, such as lymphovascular invasion. It is noteworthy that in the setting of palpable lymph nodes, antibiotics administration and delayed reassessment is not indicated if there are clinical indications for proceeding with a lymph node dissection, namely pT1 high grade or pT2 or greater tumors. Pettaway CA, Lance RS, Davis JW: Tumors of the penis, Wein, AJ, Kavoussi LR, Novick AC, Partin AW, Peters CA (eds): CAMPBELL-WALSH UROLOGY, ed 10. Philadelphia, Elsevier Saunders, 2012, vol 1, chap 34, p 920."

"A 63-year-old man underwent a partial penectomy. Pathologic evaluation of the specimen reveals a grade 1, pT1 penile cancer without lymphovascular invasion. Physical examination reveals no palpable inguinal lymph nodes, the next step is: A. observation. B. bilateral superficial inguinal lymph node dissection. C. bilateral superficial and deep inguinal lymph node dissection. D. diagnostic sentinel node biopsy. E. prophylactic XRT to both groins."

"D: ""subcutaneous heparin."" was the correct answer. This patient is at high risk for venous thromboembolic event (VTE) because of his cancer diagnosis, age, surgery, and prolonged immobility. Thus, guidelines indicate the need for full precautions. His idiopathic thrombocytopenic purpura may put him at increased risk of perioperative bleeding, but does not protect him from the risk of VTE. Thus, subcutaneous heparin is the standard prophylaxis, although it should be used cautiously in patients with severe thrombocytopenia. Enoxaparin should be used only with extreme caution in patients with thrombocytopenia. In either case, close monitoring of the platelet count is indicated to detect any heparin-induced thrombocytopenia. Warfarin does not provide immediate protection, and is, therefore, reserved for long-term anticoagulation. Pentoxifylline has not been shown to prevent VTE. Forrest JB, Clemens JQ, Finamore P, et al: Best practice policy statement for the prevention of deep vein thrombosis in patients undergoing urologic surgery. PREVENTION OF DVT AFTER UROLOGIC SURGERY BEST PRACTICE STATEMENT. American Urological Association Education and Research, Inc., 2008. http://www.auanet.org/content/guidelines-and-quality-care/clinical-guidelines/main-reports/dvt.pdf "

"A 63-year-old man with idiopathic thrombocytopenic purpura (ITP) requires pelvic exenteration. His platelet count prior to surgery was 50,000/cu mm. Venous thromboembolic prophylaxis should consist of: A. oral low dose aspirin. B. oral warfarin. C. oral pentoxifylline (Trental™). D. subcutaneous heparin. E. subcutaneous enoxaparin (Lovenox™)."

"D: ""peritoneal flap."" was the correct answer. Uncomplicated vesicovaginal fistulas can be closed using meticulous technique and a multi-layer closure. Repair of more complex fistulae often requires the use of well-vascularized tissue flaps positioned between the bladder and vaginal repair sites. The use of Martius (fibrofatty tissue), myocutaneous labial (skin and fibrofatty tissue) or, more rarely, gluteal skin flaps can be used to repair low fistulas. Such flaps may not be of sufficient length to reach high fistulae. Peritoneal flaps can be harvested through the vaginal incision and are in close proximity to such fistulas. Use of peritoneum obviates the morbidity of an abdominal incision (necessary for harvesting the omentum) or an incision along the inner thigh (necessary for harvesting the gracilis muscle). Rovner ES: Urinary tract fistulae, Wein, AJ, Kavoussi LR, Novick AC, Partin AW, Peters CA (eds): CAMPBELL-WALSH UROLOGY, ed 10. Philadelphia, Elsevier Saunders, 2012, vol 3, chap 77, p 2240."

"During transvaginal repair of a high vesicovaginal fistula, a Martius flap is harvested, but is of insufficient length to reach the fistula site. The next step is: A. myocutaneous gracilis flap. B. gluteal flap. C. omental flap. D. peritoneal flap. E. myocutaneous labial flap."

"A: ""continue CIC."" is correct. This can be a challenging study to interpret due to the excessive activity from the rectal catheter. This results in artifactual activity in Pdet that is not a true change in this patient's detrusor pressures. In interpreting this study, it is important to look at all of the tracings on the study. Aside from a few pressure spikes due to cough, the Pves tracing is essentially flat to a filled volume of more than 1000 ml. This patient will be best treated with continued CIC. Therapies aimed at decreasing detrusor pressure (antimuscarinics and onabotulinumtoxinA) are not needed. Use of alpha-blockers would not be helpful in this patient who appears to have an areflexic bladder with only a small, short-lived rise in his vesical pressure at capacity. Lumbo-sacral re-routing was initially reported to be an option for spina bifida patients that were unable to void spontaneously; however, subsequent reports have not shown this to be overly beneficial and this procedure has not been evaluated in patients with urinary retention following laminectomy. Wein AJ, Dmochowski RR: Neuromuscular dysfunction of the lower urinary tract, Wein, AJ, Kavoussi LR, Novick AC, Partin AW, Peters CA (eds): CAMPBELL-WALSH UROLOGY, ed 10. Philadelphia, Elsevier Saunders, 2012, vol 3, chap 65, pp 1930-1931."

"A 64-year-old man requires CIC to empty his bladder six months after a lumbar laminectomy. He denies incontinence between CIC. Urodynamics are shown. The next step is: A. continue CIC. B. antimuscarinics. C. alpha-blocker. D. onabotulinumtoxinA. E. lumbar to sacral nerve re-routing (""Xiao procedure"")."

"D: ""calcium and Vitamin D."" was the correct answer. Osteoporosis is a recognized risk of long-term androgen ablation and has been shown to have significant clinical impact. Men started on LH-RH agonists should receive annual bone density scans and regular calcium and Vitamin D supplements. Treatment of osteoporosis, if it develops, may be done with bisphosphonates, but the ideal type of therapy is not proven. Prophylactic monthly zoledronate (Zometa®) or denosumab are cost prohibitive agents that have been shown to prevent bone-related events only in men with documented bony metastases. Nelson JB: Hormone therapy for prostate cancer, Wein, AJ, Kavoussi LR, Novick AC, Partin AW, Peters CA (eds): CAMPBELL-WALSH UROLOGY, ed 10. Philadelphia, Elsevier Saunders, 2012, vol 3, chap 109, p 2942. "

"A 65-year-old man is treated with goserelin acetate and bicalutamide for a rising serum PSA following definitive XRT for Gleason 8 prostate cancer. Bone scan and CT scan are negative. In addition to the hormone therapy, he should have annual bone density scans and: A. observation. B. monthly zoledronate (Zometa). C. weekly alendronate (Fosamax). D. calcium and Vitamin D. E. denosumab."

"B: ""quarterly upper tract surveillance, urine cytology, and cystoscopy."" was the correct answer. The management of upper tract urothelial carcinoma has changed significantly in the past decade as endoscopic technologies have improved. Many of these tumors, which previously would have been managed by open surgical excision, can now be safely managed with endoscopic ablation. Patients with low volume, low grade, and low stage disease can be safely managed with minimally invasive techniques. Other factors that should lead to strong consideration of minimally invasive management include renal insufficiency or other abnormalities. In this case, the patient has a single non-invasive, low grade tumor which favors a minimally invasive nephron-sparing approach. While ureterectomy would spare the involved kidney, it is a major surgical procedure. As such, the likely minimal added cancer control benefit of open surgical extirpation over endoscopic management is outweighed by the risk of peri-operative complications. While ureteral stent placement or percutaneous nephrostomy with BCG instillation might be indicated for higher grade or stage disease managed endoscopically, it is unnecessary in this situation. In this case, endoscopic laser ablation is likely adequate initial treatment of the ureteral tumor. Follow-up for these tumors must include evaluation of both the upper tract and the bladder, as there is increased risk of bladder tumors in these patients. Therefore, quarterly upper tract surveillance with imaging or ureteroscopy, cystoscopy, and urine cytology is the best surveillance strategy in this patient. BCG for high grade upper tract UC is occasionally used in patients with a solitary kidney or similar circumstances, but there is no definitive data to support a benefit. Sagalowsky AI, Jarrett TW, Flanigan RC: Urothelial tumors of the upper urinary tract and ureter, Wein, AJ, Kavoussi LR, Novick AC, Partin AW, Peters CA (eds): CAMPBELL-WALSH UROLOGY, ed 10. Philadelphia, Elsevier Saunders, 2012, vol 2, chap 53, p 1552."

"A 65-year-old woman undergoes ureteroscopic biopsy and laser ablation of a single right mid-ureteral tumor. Histology reveals a low grade Ta urothelial carcinoma. The next step is: A. CT urogram and urine cytology. B. quarterly upper tract surveillance, urine cytology, and cystoscopy. C. ureteral stent placement and BCG instillation. D. nephrostomy and antegrade administration of BCG. E. partial ureterectomy and Boari flap."

"E: ""intraurethral alprostadil suppository."" was the correct answer. An FDA warning exists that recommends against the use of phosphodiesterase type 5 inhibitors in patients with a recent history of an myocardial infarction (less than six months) and in patients with known hereditary degenerative retinal disorders such as retinitis pigmentosa. Thus, the use of tadalafil or sildenafil in this patient would not be advised. Testosterone therapy is not considered first-line therapy for erectile dysfunction. Intracavernous injection is relatively contraindicated in men with a history of coagulopathy(clopidogrel) or unstable cardiovascular disease and thus would not be a good option for this patient. The best option for this patient would be intraurethral alprostadil suppository. Burnett AL: Evaluation and management of erectile dysfunction, Wein, AJ, Kavoussi LR, Novick AC, Partin AW, Peters CA (eds): CAMPBELL-WALSH UROLOGY, ed 10. Philadelphia, Elsevier Saunders, 2012, vol 1, chap 24, p 742. "

"A 67-year-old man has erectile dysfunction after a myocardial infarction four months ago. He has retinitis pigmentosa and takes clopidogrel (Plavix). His testosterone is 195 ng/dl. The next step is: A. sildenafil 100 mg on demand. B. daily tadalafil 5 mg. C. daily testosterone gel. D. intracavernous prostaglandin injection. E. intraurethral alprostadil suppository."

"B: ""CT scan in six months."" was the correct answer. The AUA Guidelines for Follow-up of Renal Masses recommends that patients undergo cross-sectional abdominal scanning (CT or MRI, but not ultrasound) within six months of active surveillance initiation to establish a growth rate. The Panel further recommends continued imaging at least annually thereafter. Data on long-term growth patterns of renal masses are limited, although there is a suggestion that masses show linear growth on axial imaging rather than logarithmic. Thus, serial axial imaging as early as six months after the initiation of surveillance will establish an expected growth rate for subsequent comparison. After this initial scan, subsequent imaging, which then can include ultrasound, can be performed yearly unless the pace of growth is concerning. Fluorodeoxyglucose-PET scans have no utility in staging for RCC, and there is no indication for a functional renogram study. There is no indication for repeat biopsy for surveillance of renal masses. Donat SM, Chang SS, Bishoff JT, et al: Follow-up for clinically localized renal neoplasms: AUA Guideline. American Urological Association Education and Research, Inc., 2013. http://www.auanet.org/education/guidelines/renal-cancer-follow-up.cfm "

"A 69-year-old woman has a 2.2 cm, biopsy-proven, grade 2, clear cell RCC and elects active surveillance. The next step is: A. renal ultrasound in six months. B. CT scan in six months. C. CT scan in one year. D. fluorodeoxyglucose-PET imaging. E. repeat biopsy in one year."

"D: ""prostatic urethral biopsy."" was the correct answer. In patients with high-risk non-muscle-invasive disease, there is a high risk of secondary tumor involvement of the prostatic urethra and ducts. Prostatic urethral involvement may be detected in 20-40% of these patients within ten years of diagnosis. In patients with refractory disease, the incidence of extravesical recurrence in the prostatic fossa may be as high as 33%, and a substantial proportion may be fatal. In this patient, persistently positive cytologies with no clear upper tract or bladder source would warrant biopsy of the prostatic urethra to investigate the source of the abnormal cytologies. Fluorescent in situ hybridization (FISH) or other molecular markers would not provide additional information over clearly positive malignant cytology. Intravesical topical therapies, such as mitomycin or BCG, will not treat the prostatic urethra adequately and are not indicated for cytology alone diagnoses. Ureteroscopy is not indicated with negative upper tract washings. Jones JS, Larchian WA: Non-muscle-invasive bladder cancer (Ta, T1, and CIS), Wein, AJ, Kavoussi LR, Novick AC, Partin AW, Peters CA (eds): CAMPBELL-WALSH UROLOGY, ed 10. Philadelphia, Elsevier Saunders, 2012, vol 3, chap 81, p 2353. "

"A 67-year-old man with a history of multifocal CIS is on maintenance BCG after a six-week induction course 18 months ago. He has positive voided urine cytologies with negative bilateral upper tract imaging and washings, normal appearing bladder, and normal random bladder biopsies. The next step is: A. fluorescent in situ hybridization (FISH) testing. B. intravesical induction mitomycin C. C. repeat induction BCG. D. prostatic urethral biopsy. E. ureteroscopy with biopsy."

"A: ""myocardial infarction and stroke."" is correct. Recently, the FDA has issued a warning and label change with testosterone products for the potential increased risk of cardiovascular disease in men taking testosterone. Patients should be counseled on this potential cardiovascular risk prior to starting therapy. Testosterone therapy has been shown to improve insulin resistance and osteoporosis. Patients on testosterone therapy can develop erythrocytosis, not anemia. There is no data to support that testosterone causes prostate cancer or increases incidence of recurrence. Morales A: Androgen deficiency in the aging male, Wein, AJ, Kavoussi LR, Novick AC, Partin AW, Peters CA (eds): CAMPBELL-WALSH UROLOGY, ed 10. Philadelphia, Elsevier Saunders, 2012, vol 1, chap 29, p 819. AUA Position Statement on Testosterone Therapy https://www.auanet.org/education/testosterone-therapy.cfm"

"A 67-year-old man with a history of radical prostatectomy, cardiovascular disease, osteoporosis, and diabetes is planning to initiate testosterone therapy. Prior to initiating testosterone therapy, he should be counseled about an increased risk of: A. myocardial infarction and stroke. B. prostate cancer recurrence. C. insulin resistance. D. anemia. E. osteoporosis progression."

"C: ""cystoscopy and biopsy."" was the correct answer. Persistent drainage nine weeks following a repaired bladder injury would indicate issues related to poor healing such as foreign body, inadequate drainage from an indwelling catheter or recurrent/persistent malignancy. This injury is unlikely to heal with further catheter drainage following nine weeks of observation and waiting an additional three weeks and repeating a cystogram would be of little to no merit. The next step should be a cystoscopy and biopsy to rule out recurrent/persistent malignancy before proceeding with definitive therapy. Assuming the urethral catheter is working appropriately, suprapubic drainage will not be any more efficient and not resolve the issue. Fulguration of a fistula tract and/or injection of fibrin glue have been shown to be of benefit in fistula < 5 mm in size with definitive repair of fistula pursued in large or persistent fistula. Surgical intervention, however, should not be pursued until persistent or recurrent malignancy has been ruled-out. Rovner ES: Urinary tract fistulae, Wein AJ, Kavoussi LR, Novick AC, Partin AW, Peters CA (eds): CAMPBELL-WALSH UROLOGY, ed 10. Philadelphia, Elsevier Saunders, 2012, vol 3, chap 77, pp 2252-2255."

"A 68-year-old man has a posterior bladder injury during a low anterior resection for locally advanced colon cancer that is repaired primarily with a two layer repair. Postoperatively, a urethral catheter is left indwelling. Serial postoperative cystograms obtained at three, six, and nine weeks following surgery reveal persistent extraperitoneal extravasation from the posterior bladder wall. A CT scan reveals no pelvic masses and no evidence of ureteral injuries. The next step is: A. repeat cystogram in three weeks. B. cystoscopy and fulguration. C. cystoscopy and biopsy. D. primary repair and omental interposition. E. suprapubic cystostomy."

"A: ""observation."" is correct. This asymptomatic patient has developed granulomatous prostatitis. This is a common condition following intravesical BCG therapy that requires no further intervention. If the patient is on maintenance BCG, it can be continued in this scenario. Isoniazid, rifampin, and cycloserine are used for systemic BCG toxicity and are not indicated in this case. The other more common side effect associated with intravesical BCG therapy is bladder irritability. This symptom complex includes dysuria (91%), urinary frequency (90%), hematuria (46%), fever (24%), malaise (18%), nausea (8%), chills (8%), arthralgia (2%), and pruritus (1%). Tareen B, Taneja SS: COMPLICATIONS OF INTRAVESICAL THERAPY. Taneja SS (ed): COMPLICATIONS OF UROLOGIC SURGERY, ed 4. Philadelphia, Elsevier Saunders, 2010, chap 8, pp 97-98. "

"A 68-year-old man has received intravesical BCG therapy for bladder cancer. He is asymptomatic, but a new firm area in the prostate gland is noted on DRE. The serum PSA is 3.1 ng/ml. A TRUS-guided biopsy reveals a caseating granuloma. The next step is: A. observation. B. isoniazid for six months. C. cycloserine for six weeks. D. isoniazid and rifampin for three months. E. isoniazid, rifampin, and cycloserine for six months."

"B: ""ortho-phthalaldehyde (OPA)."" was the correct answer. All of the substances listed may be used to disinfect flexible cystoscopies. Ortho-phthalaldehyde (OPA), however, is the only one associated with anaphylactic reactions following repeated patient exposure. Consequently, it is recommended that OPA not be used to sterilize cystoscopies in patients requiring repetitive cystoscopy such as a patient with a history of bladder cancer. Clemens JQ, Dowling R, Foley F, et al: Joint AUA/SUNA White Paper on Reprocessing of Flexible Cystoscopes. American Urological Association Education and Research, Inc., 2013. http://www.auanet.org/education/guidelines/flexible-cystoscopes.cfm "

"A 68-year-old man with a history of superficial bladder cancer develops a severe anaphylactic reaction while undergoing surveillance flexible cystoscopy. The most likely causative agent is: A. glutaraldehyde. B. ortho-phthalaldehyde (OPA). C. peracetic acid. D. hydrogen peroxide. E. ethylene oxide."

"D: ""autologous fascia sling."" was the correct answer. This patient has minimal urethral mobility with stress maneuvers and a low abdominal LPP consistent with intrinsic sphincter deficiency (LPP < 60 cm H2O). The treatment of choice with persistent incontinence secondary to intrinsic sphincter deficiency following a failed sling procedure is placement of an autologous rectus fascia sling. In and of itself, the postoperative findings of an open bladder neck on a videourodynamic study is not suggestive of a neurologic disorder (such as Shy-Drager syndrome). A neurologic consultation is, therefore, not indicated unless concurrent neurologic findings are present. Kegel exercises in a postoperative patient with these urodynamic findings would be of extremely limited benefit. Radiofrequency bladder neck suspension has not been shown to be of significant benefit in patients with recurrent stress incontinence following surgical repair and/or in patients with intrinsic sphincter deficiency (Valsalva LPP < 60 cm H2O). Artificial urinary sphincter placement may be considered an option but should be considered only after the patient had failed an autologous fascial sling. Dmochowski RR, Padmanabhan P, Scarpero HM: Slings: Autologous, biologic, synthetic, and midurethral, Wein, AJ, Kavoussi LR, Novick AC, Partin AW, Peters CA (eds): CAMPBELL-WALSH UROLOGY, ed 10. Philadelphia, Elsevier Saunders, 2012, vol 3, chap 73, p 2116. "

"A 71-year-old woman has persistent bothersome stress incontinence six months following a transobturator mid-urethral sling. Videourodynamics show a Valsalva LPP of 55 cm H2O with minimal urethral mobility and an open bladder neck at rest. The next step is: A. neurology evaluation. B. Kegel exercises. C. radiofrequency of bladder neck suspension. D. autologous fascia sling. E. artificial urinary sphincter."

"E: ""bladder neck involvement."" was the correct answer. The incidence of tumor involving the urethra in women undergoing cystectomy for bladder urothelial carcinoma is approximately 12%. The urethral recurrence rate in properly selected patients is low at less than 4%. Bladder neck tumor involvement in women, however, may be seen in approximately 22% of patients and is a risk for a urethral tumor in about half (12%). Preoperative involvement of the bladder neck with tumor in women is not an absolute contraindication as long as full-thickness, intraoperative, frozen section analysis reveals no tumor involvement of the proximal urethra (distal surgical margin). Age, tumor grade and pathological stage do not appear to be significant risk factors or contraindications to orthotopic diversion in women. CIS and multifocality also increase the risk of urethral occurrence, but not to the same degree as bladder neck involvement. Stein JP, Penson DF, Wu SD et al: Pathological guidelines for orthotopic urinary diversion in women with bladder cancer: A review of the literature. J UROL 2007;178:756-760. Skinner EC, Skinner DG, Stein JP: Orthotopic urinary diversion, Wein, AJ, Kavoussi LR, Novick AC, Partin AW, Peters CA (eds): CAMPBELL-WALSH UROLOGY, ed 10. Philadelphia, Elsevier Saunders, 2012, vol 3, chap 87, pp 2483-2484. "

"A 73-year-old woman with high-grade muscle invasive bladder cancer in the bladder neck and dome of the bladder as well as CIS desires an orthotopic urinary diversion. The strongest relative contraindication to this type of diversion would be: A. age. B. multifocal disease. C. presence of CIS. D. preoperative unilateral hydronephrosis. E. bladder neck involvement."

"A: ""BCG instillation in the urethra and neobladder."" is incorrect. E: ""urethrectomy and use afferent limb for cutaneous diversion."" was the correct answer. Urethral recurrence after neobladder is an uncommon but troublesome complication best treated with urethrectomy and some form of urinary diversion. The Studer neobladder affords a reasonable solution to this problem since the non-intussuscepted afferent limb may be detached from the neobladder and converted to a standard ileal loop. A transverse colon conduit would require ureteral reimplantation and continent cutaneous diversion and while an option, would have a greater complication rate in an older and previously operated upon patient. Local therapies, such as BCG and laser fulguration, are inadequate for this invasive tumor. Skinner EC, Skinner DG, Stein JP: Orthotopic urinary diversion, Wein, AJ, Kavoussi LR, Novick AC, Partin AW, Peters CA (eds): CAMPBELL-WALSH UROLOGY, ed 10. Philadelphia, Elsevier Saunders, 2012, vol 3, chap 87, pp 2503-2504."

"A 74-year-old man with a Studer ileal neobladder develops invasive urothelial carcinoma of the proximal urethra three years post-cystectomy. A metastatic evaluation is negative. The next step is: A. BCG instillation in the urethra and neobladder. B. urethrectomy and transverse colon loop construction. C. urethral laser fulguration. D. urethrectomy and continent cutaneous diversion. E. urethrectomy and use afferent limb for cutaneous diversion."

"D: ""pelvic radiation exposure."" was the correct answer. Urethral erosion is a devastating complication of artificial urinary sphincter (AUS) placement which requires explantation. Patients classified as high risk for cuff erosion following AUS placement are defined as patients that have undergone: radiation therapy, a prior urethroplasty, multiple endoscopic treatments for bladder neck contracture or urethral stricture, prior urethral stent placement, or have a history of erosion or infection in a previous AUS. There is also a much lower but still significant increased risk of erosion with a prolonged postoperative catheterization interval (> 48 hours), or use of either a 3.5 cm or transcorporal cuff compared to a standard 4 cm cuff. No increased risk of erosion has been found related to age, proximal or distal bulbar urethral placement of the AUS, or prior radical prostatectomy. Brant WO, Erickson BA, Elliott SP, et al: Risk factors for erosion of artificial urinary sphincters: A multicenter prospective study. UROL 2014;84:934-938. Seideman CA, Zhao LC, Hudak SJ, et al: Is prolonged catheterization a risk factor for artificial urinary sphincter cuff erosion? UROL 2013;82:943-946. "

"A 78-year-old man undergoes an artificial urinary sphincter for severe stress urinary incontinence following radical prostatectomy and adjuvant radiation treatment for prostate cancer. He has a 4 cm cuff placed around the distal bulbar urethra. The risk factor which places him at highest risk for urethral erosion is: A. age. B. cuff size. C. cuff location. D. pelvic radiation exposure. E. history of radical prostatectomy."

"A: ""reassurance."" is correct. This is a typical presentation of pediatric benign urinary frequency syndrome and is best managed expectantly. Reassurance is all that is necessary with the expectation that the symptoms will resolve in one to three months. It is important to confirm that both the physical examination and urinalysis are normal with no evidence of neurologic abnormalities such as a spinal dimple or lower extremity weakness or UTIs. In most instances, pharmacological therapy and invasive investigation or therapy have proven to be unsuccessful and may actually worsen symptoms. In almost all cases, these symptoms resolve spontaneously within a few months and the family should be reassured. Canning DA, Lambert SM: Evaluation of the pediatric urology patient, Wein, AJ, Kavoussi LR, Novick AC, Partin AW, Peters CA (eds): CAMPBELL-WALSH UROLOGY, ed 10. Philadelphia, Elsevier Saunders, 2012, vol 4, chap 115, p 3070. Yeung CK, Sihoe JDY: Non-neuropathic dysfunction of the lower urinary tract in children, Wein, AJ, Kavoussi LR, Novick AC, Partin AW, Peters CA (eds): CAMPBELL-WALSH UROLOGY, ed 10. Philadelphia, Elsevier Saunders, 2012, vol 4, chap 127, pp 3412-3414. Zoubeck J, Bloom DA, Sedman AB: Extraordinary urinary frequency. PED 1990;85:1112-1114. "

"A five-year-old boy has a two-week history of severe daytime urinary frequency. There is no history of UTI and urinalysis is negative. Physical and neurological examination are normal. The next step is voiding diary and: A. reassurance. B. oxybutynin. C. tamsulosin. D. ultrasound. E. VCUG."

"A: ""continue current antibiotic regimen."" is correct. In the treatment of pyelonephritis, persistence of fever for 72 hours following initiation of I.V. appropriate antibiotics is within the range of normal and no additional studies should be done at this time. Guidelines from the American Academy of Pediatrics recommend further radiologic testing in this age group only if the initial renal bladder ultrasound is abnormal or for occurrence of a second febrile UTI. Acute DMSA scanning may be helpful in situations where the diagnosis regarding the etiology of the fever is unclear and the physicians desire to document the presence of pyelonephritis. CT or MRI scans may be considered if the renal bladder ultrasound is abnormal or if fever persists beyond 72 hours. Persistent fever > 72 hours is best evaluated with either a CT or MRI scan, with repeat ultrasonography having an unacceptable false negative rate for documenting the presence of a perinephric abscess. A VCUG is not indicated in the acute setting but can be considered once the child is afebrile. Repeat cultures or broadened coverage is not required when initial sensitivities are available but should be considered for a temperature persisting > 72 hours. Circumcision at this age usually requires a general anesthetic and is not needed for a first UTI. Shortliffe LMD: Infection and inflammation of the pediatric genitourinary tract, Wein, AJ, Kavoussi LR, Novick AC, Partin AW, Peters CA (eds): CAMPBELL-WALSH UROLOGY, ed 10. Philadelphia, Elsevier Saunders, 2012, vol 4, chap 116, p 3085. "

"A four-month-old uncircumcised boy is hospitalized for a febrile UTI. He has persistent fevers after 24 hours of appropriate I.V. antibiotic therapy. Renal and bladder ultrasound is normal. The next step is: A. continue current antibiotic regimen. B. repeat urine and blood cultures. C. VCUG. D. CT scan. E. circumcision."

"A: ""unchanged detrusor LPP; unchanged Valsalva LPP."" is correct. The LPP is determined by the resistance of the external sphincter. Antimuscarinic medication, such as oxybutynin, does not affect the external sphincter which is a striated muscle. Therefore, there should be no change in the LPP (detrusor or Valsalva). The effect of the antimuscarinic medication on the detrusor may increase bladder compliance by increasing the volume of urine stored at a given pressure, and thereby, increase the patient's bladder capacity. The closing pressure of the proximal urethra will not be affected. Yeung CK, Sihoe JDY: Non-neuropathic dysfunction of the lower urinary tract in children, Wein AJ, Kavoussi LR, Novick AC, Partin AW, Peters CA (eds): CAMPBELL-WALSH UROLOGY, ed 10. Philadelphia, Elsevier Saunders, 2012, vol 4, chap 127, p 3411. McGuire E, Fitzpatrick C, Wan JW, et al: Clinical assessment of urethral sphincter function. J UROL 1993;150:1452-1454. "

"A four-year-old girl with spina bifida is wet between urethral catheterizations. Her detrusor LPP is 50 cm H2O and her Valsalva LPP is 70 cm H2O. She is started on oxybutynin. A repeat urodynamic study will reveal: A. unchanged detrusor LPP; unchanged Valsalva LPP. B. unchanged detrusor LPP; increased Valsalva LPP. C. decreased detrusor LPP; increased Valsalva LPP. D. decreased detrusor LPP; decreased Valsalva LPP. E. decreased detrusor LPP; unchanged Valsalva LPP."

"C: ""ejaculatory dysfunction."" was the correct answer. Permanent ejaculatory dysfunction may occur in as high as 20% of men following urethroplasty. Complaints are usually related to pooling of semen within the urethra and/or loss of force with ejaculation. The etiology is poorly defined but is presumed to be due to either tortuosity of the neourethra and/or dysfunction of the bulbocavernosal muscle. Temporary erectile dysfunction is found in up to 20% of individuals undergoing an anterior urethroplasty. This incidence is similar between all types of anterior urethroplasties, e.g., excision and primary anastomosis, vascularized or graft urethroplasties. The erectile dysfunction symptoms classically resolve within six months with < 3-4% of patients reporting a permanent alteration in their erectile capabilities New onset of penile curvature may occur usually following an overaggressive attempt at excision and primary anastomosis performed in the distal bulbar region. Loss of libido and anorgasmia are very rare complaints following urethroplasty and are predominately due to a psychological component . Blaschko SD, Sanford MT, Cinman NM, et al: De novo erectile dysfunction after anterior urethroplasty: A systematic review and meta-analysis. BRI J UROL INT 2013;112:655-663. Dubey D, Kumar A, Bansal P, Srivastava A, Kapoor R, Mandhani A, Bhandari M. Substitution urethroplasty for anterior urethral strictures: A critical appraisal of various techniques. BRI J UROL INT 2003;91:215-218. "

"A healthy 26-year-old man undergoes urethroplasty with buccal mucosa for a 5 cm bulbar urethral stricture. The most common form of sexual dysfunction he may experience postoperatively is: A. loss of libido. B. erectile dysfunction. C. ejaculatory dysfunction. D. anorgasmia. E. penile curvature."

"A: ""no antimicrobials."" is correct. The American Heart Association does not recommend the administration of prophylactic antibiotics to prevent bacterial endocarditis associated with urodynamic testing or other genitourinary procedures presuming a negative urinalysis and sterile technique. Infectious endocarditis is more likely to result from random bacteremias associated with daily activities than from those caused by genitourinary procedures. Prophylaxis may prevent only a very small number of cases of infectious endocarditis, if any, in individuals undergoing genitourinary procedures. Overall, the risk of antimicrobial-associated adverse events exceeds the benefit from prophylactic antimicrobial therapy solely to prevent infectious endocarditis in patients undergoing genitourinary procedures. Wilson W, Taubert KA, Gewitz M, et al: Prevention of bacterial endocarditis: Guidelines from the American Heart Association. CIRCULATION 2007;116:1736-1754. Wolf JS, Jr, Bennett CJ, Dmochowski RR, et al: Best Practice Policy Statement on Urological Surgery Antimicrobial Prophylaxis. American Urological Association Education and Research, Inc., 2008. http://www.auanet.org/education/guidelines/antimicrobial-prophylaxis.cfm"

"A healthy 55-year-old woman with rheumatic heart disease and a systolic murmur is about to undergo urodynamic testing. Urinalysis is negative. Appropriate antimicrobial prophylaxis includes: A. no antimicrobials. B. amoxicillin. C. fluoroquinolone. D. trimethoprim-sulfamethoxazole. E. gentamicin."

"D: ""hepatotoxicity."" was the correct answer. Abiraterone acetate is an oral Cyp-17 lyase and hydroxylase inhibitor indicated for the treatment of men with castration-resistant metastatic prostate cancer post-docetaxel treatment. The blockade of Cyp-17 inhibits androgen biosynthesis at all sites as well as inhibiting the biosynthesis of corticosteroids and can cause upregulation of ACTH and resultant hypersecretion of mineralocorticoids. Increased mineralocorticoids can cause hypertension, fluid retention, and hypokalemia. It is co-administered with prednisone. The most serious adverse event associated with its administration is hepatotoxicity, that can be severe and potentially life threatening. Liver enzymes as well as electrolytes must be checked frequently (every two weeks for the first three months) when initiating the medication. There is no associated renal toxicity. Fatigue, hypokalemia, and fluid retention are relatively common side effects but typically do not require dose reduction. de Bono JS, Logothetis CJ, Molina A, et al: HI; COU-AA-301 Investigators. Abiraterone and increased survival in metastatic prostate cancer. NEJM 2011;364:1995-2005. "

"A man with castration-resistant metastatic prostate cancer receives oral abiraterone acetate and prednisone. The toxicity most likely to require dose reduction or discontinuation is: A. hypertension. B. fluid retention. C. hypokalemia. D. hepatotoxicity. E. nephrotoxicity."

"A: ""lansoprazole (Prevacid™)."" is correct. Lansoprazole is a proton pump inhibitor (PPI). It is noteworthy that all PPIs have been found to be significantly associated with an increased risk of C. difficile colonization and infection. This has been attributed to their attenuation of acid secretion by the stomach and elimination of this barrier to bacterial colonization. None of the other medications has been associated with C. difficile colonization or infection. Hookman P, Barkin JS: Clostridium difficile associated infection, diarrhea, and colitis. WORLD J GASTROENTEROL 2009;15:1554-1580."

"A medication associated with an increased risk of C. difficile in hospitalized patients is: A. lansoprazole (Prevacid™). B. metoclopramide (Reglan™). C. dicyclomine (Bentyl™). D. diphenoxylate and atropine (Lomotil™). E. alvimopan (Entereg™)."

"A: ""bilateral needle biopsy of the kidneys, then chemotherapy."" is incorrect. E: ""left nephrectomy and surveillance."" was the correct answer. This disease process is most likely a congenital mesoblastic nephroma (CMN) and should be distinguished from a Wilms' tumor by the patient's age and the fact that CMN is typically infiltrative on imaging, whereas Wilms' tumors displace and compress renal architecture. CMNs are the most common tumor in infants greater than four months of age, with a median age at the time of diagnosis of three months. In contrast, the median age for diagnosis of a Wilms' tumor is 3.5 years. Two types of mesoblastic nephroma exist, the classic type which is far more common, and, provided surgical margins are negative, rarely recur and the cellular variant. The cellular variant consists of atypical spindle cells with frequent mitotic figures (25-30/10 hpf) and necrosis and is considered a variant of a fibrosarcoma. This variant is associated with both local recurrence and widespread metastasis. Surveillance by interval six month abdominal ultrasounds for the first two years is usually recommended for the classic variant and more aggressive follow-up with interval CT or MRI scans of the lungs and abdomen are recommended at three to six month intervals for the first two years for the cellular variant. Normal newborn kidneys are slightly echogenic, hence, the right kidney is normal in the boy presented here. Ritchey ML, Shamberger RC: Pediatric urologic oncology, Wein, AJ, Kavoussi LR, Novick AC, Partin AW, Peters CA (eds): CAMPBELL-WALSH UROLOGY, ed 10. Philadelphia, Elsevier Saunders, 2012, vol 4, chap 137, p 3723."

"A newborn boy has a left-sided abdominal mass. Ultrasound reveals an infiltrative mass replacing the lower third of the left kidney. The right kidney appears to have a slightly increased echogenicity, but is otherwise normal. The next step is: A. bilateral needle biopsy of the kidneys, then chemotherapy. B. exploratory laparotomy and bilateral biopsy. C. left nephrectomy followed by chemotherapy. D. left lower pole partial nephrectomy followed by chemotherapy. E. left nephrectomy and surveillance."

"B: ""VCUG."" was the correct answer. It is not uncommon for children with prenatally recognized hydronephrosis to have normal renal ultrasounds after birth. Many of these represent patients with insignificant prenatal physiologic dilation. Specifically, in patients with a history of mild to moderate fetal hydronephrosis and a normal postnatal ultrasound, the latter obtained one to two months post-partum; additional evaluations are deemed unnecessary. The exception to this rule is the finding of moderate to severe hydroureteronephrosis found on the prenatal evaluation with a normal post-partum ultrasound. The presence of ureteral dilation on the prenatal ultrasound has been found to be associated with a significant increased risk for neonatal UTIs and high grade vesicoureteral reflux despite a normal postpartal renal ultrasound. Documentation of ureteral dilation on the fetal ultrasound, therefore, results in the recommendation that a neonatal VCUG be obtained. Observation alone may miss the opportunity to diagnose high grade VUR and place this infant at significant risk. Repeat ultrasound at such a short interval is not indicated and, if normal, would not obviate the need for a VCUG. MAG-3 renal scan should be considered if the neonatal follow-up ultrasound evaluations demonstrate persistent hydroureteronephrosis and the VCUG reveals absence of reflux. Antibiotic prophylaxis is recommended for infants found on neonatal ultrasound to have a dilated ureter and/or VUR. Unless the renal ultrasound reveals a structural abnormality, i.e., poorly defined duplication anomaly, magnetic resonance urogram would not be indicated. Serial observation without the need for antibiotic prophylaxis would be indicated in the presence of hydronephrosis alone without evidence of prenatal ureteral dilation. In these latter infants, a MAG-3 renal scan with a diuretic washout phase would be indicated at two to three months of age. Khoury AE, Bagli DJ: Vesicoureteral reflux, Wein, AJ, Kavoussi LR, Novick AC, Partin AW, Peters CA (eds): CAMPBELL-WALSH UROLOGY, ed 10. Philadelphia, Elsevier Saunders, 2012, vol 4, chap 122, p 3279."

"A newborn girl had moderate left hydroureteronephrosis on prenatal ultrasound. Ultrasound at one month of age demonstrates normal kidneys bilaterally. Physical examination is normal. The next step is: A. observation. B. VCUG. C. MAG-3 renal scan. D. repeat ultrasound in three months. E. MR urogram."

"D: ""bladder neck hypertrophy."" was the correct answer. Consequences of PUV are detrusor hypertrophy and bladder neck hypertrophy. The outlet obstruction will result in elongation and dilation of the prostatic urethra due to the relative lack of musculature of the prostatic urethra. The hypertrophy of the bladder neck causes an upward course of the prostatic urethra and a high riding bladder neck which can make urethral catheterization difficult. During cystoscopy of such a posterior urethra, the cystoscope must be angled directly anterior in order to gain access to the bladder. All of the other options may also make catheterization difficult, but they are less likely to occur than bladder neck hypertrophy in the setting of a history of posterior urethral valves. Casale AJ: Posterior urethral valves, Wein, AJ, Kavoussi LR, Novick AC, Partin AW, Peters CA (eds): CAMPBELL-WALSH UROLOGY, ed 10. Philadelphia, Elsevier Saunders, 2012, vol 4, chap 126, p 3391. "

"A three-month-old boy with a history of incised PUVs has a febrile illness. There is difficulty passing a catheter per urethra into the bladder to obtain a urine specimen. The most likely cause is: A. urethral stricture. B. residual valve leaflet. C. external sphincter spasm. D. bladder neck hypertrophy. E. false passage from previous catheterization."

"C: ""ureterocele."" was the correct answer. Ureteral duplication is one of the most common renal abnormalities with approximately 1% of the population diagnosed as having a duplication anomaly during their lifetime with 10% of the affected individuals will have a bilateral duplication anomaly. In individuals with a duplication anomaly, 10% are diagnosed following an evaluation for a UTI, another 10% are discovered during an evaluation for prenatal hydronephrosis, and the remainders are incidentally discovered during radiographic evaluations. Severe upper pole hydronephrosis of a duplicated moiety is commonly associated with either a ureterocele or ureteral ectopy, while lower pole hydronephrosis is commonly associated with vesicoureteral reflux or a UPJ obstruction. In this patient, the ultrasound (US) images are classic for obstruction of the upper pole due to a ureterocele. Differentiation of a ureterocele from an ectopic ureter can at times be problematic. Indeed, the term ""pseudoureterocele"" has been coined for enlarged ectopic ureter that protrudes into the bladder. Classically, a tip off that it is an ectopic ureter and not a ureterocele is found in the thickness of the ureterocele membrane. Presence of a thin-wall membrane is almost invariably associated with a ureterocele. In contrast, a thick-walled ""ureterocele"" or cystic structure may indicate a ectopic ureter with the thick wall being attenuated detrusor muscle. Inadvertent endoscopic incision of a pseudoureterocele has led to the creation of detrusor-ureteral-vaginal fistulas that may be highly problematic to repair in a neonate. To differentiate an ectopic ureter (pseudoureterocele) from a ureterocele, a VCUG may be helpful. If the ectopic ureter inserts into the bladder neck or urethra, reflux into the upper system may be found confirming the diagnosis of an ectopic ureter. If no reflux is noted and the physician has a high degree of suspicion that the diagnosis is truly an ectopic ureter, a pelvic MRI or CT scan may be of benefit. Peters CA, Schlussel RN, Mendelsohn C: Ectopic ureter, ureterocele, and ureteral anomalies, Wein AJ, Kavoussi LR, Novick AC, Partin AW, Peters CA (eds): CAMPBELL-WALSH UROLOGY, ed 10. Philadelphia, Elsevier Saunders, 2012, vol 4, chap 121, p 3247."

"A one-month-old girl with prenatal hydronephrosis underwent postnatal imaging as shown. The most likely cause of findings in the bladder and kidney is: A. VUR. B. UPJ obstruction. C. ureterocele. D. bladder diverticulum. E. ectopic ureteral insertion into the vagina."

"B: ""placement of a ureteral stent."" was the correct answer. Identification of a ureteral perforation is an indication for immediate discontinuation of the procedure and passage of a ureteral stent. Persistence of the operation may lead to further shear force injury on the ureter or extravasation of irrigant or urine. If a ureteral stent is placed, percutaneous drainage is not necessary. Ost MC, Schneck FX: Surgical management of pediatric stone disease, Wein, AJ, Kavoussi LR, Novick AC, Partin AW, Peters CA (eds): CAMPBELL-WALSH UROLOGY, ed 10. Philadelphia, Elsevier Saunders, 2012, vol 4, chap 135, p 3667."

"A seven-year-old boy with two 5 mm renal pelvis stones undergoes ureteroscopy and basket extraction of one stone. During reinsertion of the ureteroscope, a small ureteral perforation is identified. The next step is: A. remove ureteroscope and stop operation. B. placement of a ureteral stent. C. basket extraction of remaining stone and stent placement. D. fragment stone into 1 mm fragments and stent placement. E. stent and percutaneous nephrostomy tube placement."

A: ""maintenance of voiding diary and timed voiding."" is correct. Dysfunctional voiding syndrome is frequently manifested by worsening diurnal incontinence, posturing when voiding, with or without primary nocturnal enuresis, recurrent UTIs, and encopresis. Classically, urodynamic studies will reveal poor cerebral appreciation of sudden detrusor contractions. Voluntary sphincter contraction is thought to be a response to sudden detrusor contraction. This can arise from either delayed neurourologic maturation or inappropriately learned voiding behavior. Initial treatment should be with maintenance of a voiding and stooling diary, timed voiding every two hours while awake, and treatment of constipation issues if present. Approximately 80-85% of patients will resolve their symptoms on this treatment. Individuals failing the conservative management outlined above may be treated by either the addition of antimuscarinics, such as oxybutynin or alternatively, behavior modification with biofeedback pelvic floor muscle retraining. Use of a sacral neuromodulator should only be considered after the patient has failed the more conservative treatment modalities outlined above. Prazosin, an alpha-1- receptor blocker, is potentially useful in the setting of bladder neck dysfunction, but not in the patient described above. Yeung CK, Sihoe JDY: Non-neuropathic dysfunction of the lower urinary tract in children, Wein, AJ, Kavoussi LR, Novick AC, Partin AW, Peters CA (eds): CAMPBELL-WALSH UROLOGY, ed 10. Philadelphia, Elsevier Saunders, 2012, vol 4, chap 127, p 3419. "

"A seven-year-old, neurologically normal boy with recurrent UTIs and worsening diurnal incontinence has a normal renal ultrasound and VCUG. His bowel function is normal. Urodynamic evaluation reveals detrusor overactivity and increased sphincter activity during voiding. His initial treatment should be: A. maintenance of voiding diary and timed voiding. B. biofeedback pelvic floor muscle retraining. C. oxybutynin. D. prazosin. E. sacral neuromodulator."

"B: ""renal vein injury."" is correct. CT scan remains the gold standard for radiologic staging of renal trauma. One major limitation of CT scan, however, is the inability to define a renal venous injury adequately. A medial hematoma strongly suggests a venous injury, however, there is no imaging modality which can accurately diagnose a venous injury. A renal contusion is defined as normal urologic imaging with the presence of hematuria. Parenchymal lacerations are clearly defined, and extravasation of contrast-enhanced urine can easily be detected which has led to an enhanced ability to manage many injuries non-operatively. Repeated/delayed scanning of the kidneys ten minutes after injection of contrast identifies parenchymal lacerations and urinary extravasation accurately and reliably. It is important to note that ureteral injuries can be difficult to diagnose on CT scan. If the urinary extravasation from the ureteral injury is contained by Gerota's fascia, the extent of medial leakage can be minimal. It is also known that ureteral injuries do not show contrast in the ureter on delayed images. Tracing both ureters throughout their entire course on a CT scan is mandatory to fully evaluate urogenital injuries. Delayed images should be obtained 5 to 20 minutes after contrast injection to allow contrast to extravasate from the collecting system injury. Santucci RA, Doumanian LR: Upper urinary tract trauma, Wein, AJ, Kavoussi LR, Novick AC, Partin AW, Peters CA (eds): CAMPBELL-WALSH UROLOGY, ed 10. Philadelphia, Elsevier Saunders, 2012, vol 2, chap 42, p 1172. "

"A significant limitation of CT imaging for renal trauma is an inability to fully assess a: A. renal pelvis injury. B. renal vein injury. C. renal artery injury. D. parenchymal laceration < 1 cm. E. renal contusion."

"C: ""I.V. ceftriaxone."" was the correct answer. This child has clinical signs and symptoms of severe UTI (high fever, lethargy, and poor oral intake). Although culture results are not available, the urinalysis findings are suggestive of a UTI. Specifically, if both a positive leukocyte esterase, and positive nitrite are present, 75% of the patients will have a positive culture result, i.e., a single bacterial organism at > 50,000 colonies of bacteria per ml of urine. If bacteria are seen on a catheterized urine specimen, it is 70% sensitive for a UTI for a positive culture result, i.e., a single bacterial organism at > 50,000 colonies of bacteria per ml of urine. The combination of these findings along with the patient's clinical symptoms strongly suggest that treatment should be instituted while awaiting the culture results. Nitrofurantoin achieves a low serum concentration, and is, therefore, inappropriate for severe systemic UTI. Imaging studies, such as ultrasound, CT, or MRI evaluation are indicated to rule out anatomic abnormalities if a child does not respond to treatment after 72 hours of I.V. appropriate antibiotics. A DMSA scan may be used to determine if the febrile infections is due to pyelonephritis, however, it should not be done prior to the institution of antibiotic therapy in this setting. The correct step is to begin a broad-spectrum antibiotic therapy, such as I.V. ceftriaxone. Shortliffe LMD: Infection and inflammation of the pediatric genitourinary tract, Wein AJ, Kavoussi LR, Novick AC, Partin AW, Peters CA (eds): CAMPBELL-WALSH UROLOGY, ed 10. Philadelphia, Elsevier Saunders, 2012, vol 4, chap 116, pp 3097-3105. "

"A six-month-old girl has a fever of 39°C, lethargy, and poor oral intake. Catheterized urine specimen demonstrates 2+ leukocyte esterase, positive nitrite, and few bacteria on microscopic examination. A urine culture is sent. The next step is: A. hold antibiotic treatment until the culture is finalized. B. oral nitrofurantoin. C. I.V. ceftriaxone. D. renal and bladder ultrasound. E. DMSA renal scan."

"E: ""observation, excision at six months if symptoms persist."" was the correct answer. This child has a patent urachus. Fifty percent of radiographically confirmed patent urachus will spontaneously close in the first six months of life and observation is warranted. Topical silver nitrate is a reasonable treatment modality for small urachal cysts, but the physician should be aware of the increased risk of bladder or bowel injury with silver nitrate used in the presence of a patent urachus or omphalomesenteric duct (connection of the umbilicus to small bowel). Treatment of a patent urachus or mesonephric duct (i.e., omphalomesenteric duct) by silver nitrate is contraindicated due to possible injury to the bladder or small bowel. A CT or MRI scan is sometimes obtained on these children to exclude a patent omphalomesenteric duct; however, in this case, the CT scan is not necessary because the diagnosis of patent urachus is confirmed by sinogram. A VCUG is usually not necessary if a sinogram confirms the diagnosis unless history suggests the presence of a bladder outlet obstruction that could be the etiology of urachal patency. Frimberger DC, Kropp BP: Bladder anomalies in children, Wein, AJ, Kavoussi LR, Novick AC, Partin AW, Peters CA (eds): CAMPBELL-WALSH UROLOGY, ed 10. Philadelphia, Elsevier Saunders, 2012, vol 4, chap 125, pp 3381-3384. "

"A six-week-old asymptomatic boy has intermittent drainage from the umbilicus. A fistulogram with contrast injected through the umbilicus is shown. The next step is: A. topical silver nitrate to umbilical stump. B. VCUG. C. CT scan. D. immediate excision with bladder cuff. E. observation, excision at six months if symptoms persist."

"C: ""urodynamics."" was the correct answer. This child has new onset reflux and febrile UTIs after a surgical procedure that increased his bladder outlet resistance. Of concern in this situation is that approximately 15% of patients with a neurogenic bladder will develop a poorly compliant bladder after increasing urinary outlet resistant. New onset of vesicoureteral reflux in this situation may be due to either noncompliance with CIC, thus resulting in an overdistention of the bladder, or the development of detrusor non-compliance. Urodynamics are first required to see if the bladder has deteriorated and to better direct treatment. If compliance and capacity are unchanged, then attention to the frequency of catheterization, with or without the addition of antimuscarinic therapy, should be the primary consideration. Treatment of reflux in a patient with a neurogenic bladder is not appropriate without an understanding of bladder dynamics since high storage pressures doom correction to failure. A spinal MRI scan can be considered if the urodynamics are abnormal, although a change in bladder function is likely related to the sling and not a tethered cord. Adams MC, Joseph DB: Urinary tract reconstruction in children, Wein, AJ, Kavoussi LR, Novick AC, Partin AW, Peters CA (eds): CAMPBELL-WALSH UROLOGY, ed 10. Philadelphia, Elsevier Saunders, 2012, vol 4, chap 129, p 3464. "

"A six-year-old boy with a history of myelomeningocele has two febrile UTI's six months after a bladder neck sling and appendicovesicostomy. An ultrasound shows normal upper tracts, and a VCUG shows new grade II/V bilateral VUR. The next step is: A. increase CIC frequency. B. add antimuscarinic therapy. C. urodynamics. D. MRI scan of spine. E. endoscopic correction of VUR."

"E: ""tapered ureteral reimplantation."" was the correct answer. This child has a primary obstructive megaureter associated with diminished renal function. Although many children with a primary obstructive megaureter will show gradual progressive improvement in the degree of dilation and rapidity of washout with age, the decision to observe without surgical intervention is dependent upon the renal function and the absence of symptoms. In the presence of significantly impaired function (differential function of < 40%), surgical intervention is indicated. Since the child is otherwise healthy and uninfected, primary reconstruction with a tapered ureteral reimplantation is the method of choice. Temporary diversion by a distal cutaneous ureterostomy or alternatively creation of a widely refluxing ureterovesical junction may be considered in the presence of extremely poor renal function and or a premature or young, less than six- to eight-month-old infant, where a tapered reimplantation may be technically challenging. Placement of a percutaneous nephrostomy tube is mainly used in the severely ill infant with a severe urinary infection who is not responding to parenteral antibacterials. Carr MC, Casale P: Anomalies and surgery of the ureter in children, Wein, AJ, Kavoussi LR, Novick AC, Partin AW, Peters CA (eds): CAMPBELL-WALSH UROLOGY, ed 10. Philadelphia, Elsevier Saunders, 2012, vol 4, chap 120, p 3229. Peters CA, Schlussel RN, Mendelsohn C: Ectopic ureter, ureterocele, and ureteral anomalies, Wein, AJ, Kavoussi LR, Novick AC, Partin AW, Peters CA (eds): CAMPBELL-WALSH UROLOGY, ed 10. Philadelphia, Elsevier Saunders, 2012, vol 4, chap 121, p 3236. "

"A ten-month-old boy has persistent right hydroureteronephrosis with a negative VCUG. The right kidney contributes 35% of the total renal function and the T 1/2 is 30 minutes on MAG-3 renal scan after diuretic. The next step is antibiotic prophylaxis and: A. observation with repeat MAG-3 diuretic scan in one year. B. percutaneous nephrostomy. C. creation of a freely refluxing ureterovesical junction. D. distal end cutaneous ureterostomy. E. tapered ureteral reimplantation."

"C: ""24-hour urinary calcium collection."" was the correct answer. This boy has a typical presentation for hypercalciuria. The diagnosis is best made by a 24-hour urine collection with calcium excretion exceeding 4 mg/kg/day being diagnostic. Often, a urinalysis will reveal a high specific gravity > 1.020 and an abnormal spot urine calcium-to-creatinine ratio (> 0.21). Although these findings are helpful to prove dehydration and hypercalciuria as a cause of the dysuria, the best true diagnostic assay is the 24-hour urine collection checking for both 24-hour urinary volume and calcium excretion. Patients with documented hypercalciuria and low urinary volume are at higher risk for urolithiasis. Treatment should be with liberalization of fluids and repeat urine assessment. Identification of calculus disease via plain x-ray or CT scan would not be diagnostic of hypercalciuria and, in view of the normal ultrasound, would place the patient at needless risk of radiation exposure. In the absence of proteinuria, there is no need for renal biopsy. Palmer LS, Trachtman H: Renal functional development and diseases in children, Wein, AJ, Kavoussi LR, Novick AC, Partin AW, Peters CA (eds): CAMPBELL-WALSH UROLOGY, ed 10. Philadelphia, Elsevier Saunders, 2012, vol 4, chap 112, p 3006. "

"A ten-year-old boy has persistent dysuria. Serial urinalysis reveals a pH of 7.2, specific gravity of 1.025, protein is negative, 0-2 WBC/hpf, 20-30 RBC/hpf, and a negative urine culture. A renal bladder ultrasound and a VCUG are normal. The diagnosis is best made by: A. KUB. B. serum calcium-to-creatinine ratio. C. 24-hour urinary calcium collection. D. CT scan. E. renal biopsy."

"E: ""ureteral stent."" was the correct answer. This child has an obstructing lower ureteral calculus. His immune status is depressed due to his recent chemotherapy, including a low platelet and WBC count. He is symptomatic and requires decompression of his kidney before he becomes septic. Considering his hematologic state, the best option is ureteral stent placement. Although observation, antibiotics, and/or alkalinization with a distal uric acid stone may be appropriate in an asymptomatic patient, in this scenario, immediate decompression is necessary. Ureteral stent is a better option than percutaneous nephrostomy due to the thrombocytopenia. SWL is not appropriate for a distal ureteral stone in a child. Ost MC, Schneck FX: Surgical management of pediatric stone disease, Wein AJ, Kavoussi LR, Novick AC, Partin AW, Peters CA (eds): CAMPBELL-WALSH UROLOGY, ed 10. Philadelphia, Elsevier Saunders, 2012, vol 4, chap 135, pp 3669-3676. "

"A ten-year-old boy with acute lymphoblastic leukemia has right flank pain, nausea, and vomiting after chemotherapy. Urinalysis shows pH 5.5, 10-20 RBC/hpf, and 5-10 WBC/hpf, platelet count is 50,000/cu mm, and WBC is 3,500/cu mm. Ultrasound images are shown. The next step is: A. observation and antibiotics. B. urine alkalinization. C. SWL. D. percutaneous nephrostomy. E. ureteral stent."

"A: ""medial and through a posterior calyx."" is correct. Horseshoe kidneys may develop renal calculi, possibly as a result of high insertion of the ureter and relative stasis of urine. Patients with small, nonobstructing calculi in nondependent locations may be treated with SWL. However, obstructing calculi or those that are large or in dependent locations, are best treated with percutaneous techniques. The initial puncture is more medial than that for normally positioned kidneys and should be placed through a posterior calyx. Matlaga BR, Lingeman JE: Surgical management of upper urinary tract calculi, Wein, AJ, Kavoussi LR, Novick AC, Partin AW, Peters CA (eds): CAMPBELL-WALSH UROLOGY, ed 10. Philadelphia, Elsevier Saunders, 2012, vol 2, chap 48, p 1368."

"The initial nephrostomy tube puncture site in a horseshoe kidney compared to that in a normally positioned kidney is more: A. medial and through a posterior calyx. B. lateral and through an anterior calyx. C. medial and through an anterior calyx. D. lateral and through a posterior calyx. E. inferior and through an anterior calyx."

"B: ""E. coli."" was the correct answer. This patient has hemolytic uremic syndrome. This is most frequently due to E. coli infection, 70% due to the O157:H7 serotype. In recent years, many large outbreaks have occurred due to eating uncooked hamburger meat, cheese made from unpasteurized dairy products, and unpasteurized apple cider. With aggressive management of the acute renal failure, the majority of patients will recover normal renal function. The presentation is classic, with a prodrome of enterocolitis with bloody diarrhea, fever, gross hematuria, and oliguric renal failure. Treatment is supportive. Although hemolytic uremic syndrome classically arises following an E. coli enterocolitis, it can develop as a consequence of streptococcus pneumonia, post-bone marrow transplant, and Vitamin B12 metabolic disorders. Palmer LS, Trachtman H: Renal functional development and diseases in children, Wein, AJ, Kavoussi LR, Novick AC, Partin AW, Peters CA (eds): CAMPBELL-WALSH UROLOGY, ed 10. Philadelphia, Elsevier Saunders, 2012, vol 4, chap 112, pp 3012-3013. "

"A three-year-old boy has a 48-hour history of abdominal pain, fever, vomiting, bloody diarrhea, hematuria, and oliguria. Physical examination reveals mild peripheral edema. Laboratory evaluation demonstrates a hemoglobin of 5 g/dl, thrombocytopenia, and serum creatinine of 3.2 mg/dl. The most likely etiology is infection caused by: A. streptococcus. B. E. coli. C. cytomegalovirus. D. clostridium difficile. E. hepatitis C virus."

"C: ""antimuscarinics and CIC."" was the correct answer. This boy has findings consistent with a neurogenic bladder associated with high storage pressures, detrusor overactivity, and tonic sphincter activity. In this scenario, if the physician is seeking to gain both urinary continence and provide safe storage pressures, treatment of the detrusor to improve compliance, decrease overactivity, and CIC to empty the bladder must be the mainstay of treatment. Injection of onabotulinumtoxinA into the external urinary sphincter would decrease detrusor LPP but would not aid in urinary continence. Although the combination of onabotulinumtoxinA into the detrusor and CIC would work, this would require general anesthesia to accomplish in a pediatric-aged patient and should not be pursued unless the patient has failed more conservative measures such as oral antimuscarinics and CIC. Enterocystoplasty could be considered as an option if medical therapy fails. MacLellan DL, Bauer SB: Neuropathic dysfunction of the lower urinary tract, Wein, AJ, Kavoussi LR, Novick AC, Partin AW, Peters CA (eds): CAMPBELL-WALSH UROLOGY, ed 10. Philadelphia, Elsevier Saunders, 2012, vol 4, chap 128, p 3437. "

"A three-year-old boy with thoracolumbar myelomeningocele has diurnal and nocturnal urinary incontinence. Renal ultrasound is normal and VCUG shows no VUR. Urodynamic study reveals total fill noncompliance with end-fill detrusor pressures of 50 cm H2O at 150 ml. In addition to the poor compliance, there is intermittent detrusor overactivity beginning at 50 ml, with intermittent urge incontinence noted to be associated with the overactivity. Tonic external sphincter activity is noted with the leakage. The next step is: A. timed voiding. B. antimuscarinics. C. antimuscarinics and CIC. D. onabotylinumtoxinA to the external urinary sphincter. E. onabotyliniumtoxinA into the detrusor muscle and CIC."

"B: ""transurethral incision of ureterocele."" was the correct answer. The obstruction of the upper urinary tract by a ureterocele, resulting in moderate hydroureteronephrosis associated with bilateral vesicoureteral reflux, is the most concerning element found on diagnostic imaging. The findings of associated bilateral ureteral reflux suggest the ureterocele may be obstructing the bladder neck resulting in high pressure voiding and secondary reflux. Intravenous antibiotics followed by transurethral incision of the ureterocele would be the most prudent course of action in this young infant. All of the other options listed would be associated with increased risk of morbidity in an infant of this age. In less than 10% of cases will incision of the ectopic ureterocele completely resolve the clinical findings on repeat VCUG in three to six months. Frequently a repeat VCUG will reveal decompression of the ureterocele, but will have either persistent or new onset of reflux to the left upper pole. Prior to definitive surgical intervention, the latter usually performed at one year of age, assessment of the function of the left upper pole with a DMSA scan is performed along with repeat renal ultrasound and VCUG. Definitive treatment could include any of the surgical options listed; the type of surgery performed would be dependent upon radiographic findings. Peters CA, Schlussel RN, Mendelsohn C: Ectopic ureter, ureterocele, and ureteral anomalies, Wein, AJ, Kavoussi LR, Novick AC, Partin AW, Peters CA (eds): CAMPBELL-WALSH UROLOGY, ed 10. Philadelphia, Elsevier Saunders, 2012, vol 4, chap 121, p 3249."

"A two-month-old girl with urosepsis has an ectopic ureterocele associated with moderate left upper pole hydroureteronephrosis and bilateral VUR, grade 2 reflux to left lower pole, and grade 3 reflux to a right solitary collecting system. She has defervesced on I.V. antibiotics. The next steps are prophylactic antibiotics and: A. observation with repeat renal ultrasound and VCUG at one year of age. B. transurethral incision of ureterocele. C. left upper pole partial nephrectomy. D. left upper to left lower pole ureteroureterostomy. E. ureterocele excision and left common sheath ureteral reimplant and right ureteral reimplant."

"A: ""initiate chemotherapy."" is correct. There is a high probability that this two-year-old girl has bilateral Wilms' tumor. The current Children's Oncology Group Wilms' Tumor protocol recommends six weeks of initial chemotherapy without the need for renal biopsy in patients with highly probable bilateral Wilms' tumor. The response to the chemotherapy is assessed after six weeks of chemotherapy by CT or MRI scans. Imaging at this time will quantify any reduction in tumor volume and allow the surgeon to assess the feasibility of renal sparing surgery. Tumors responding to chemotherapy may receive a second round of medications in an attempt to further reduce the size of the tumor allowing the surgeon to maximally salvage non-involved renal parenchyma. Tumors not responding to therapy will require bilateral open renal biopsy and lymph node sampling in order to both determine histopathology and guide further treatment. Although right nephrectomy and partial left nephrectomy may eventually be the ultimate outcome for this patient, chemotherapy is the initial step in management with renal preservation the primary goal. Ritchey ML, Shamberger RC: Pediatric urologic oncology, Wein, AJ, Kavoussi LR, Novick AC, Partin AW, Peters CA (eds): CAMPBELL-WALSH UROLOGY, ed 10. Philadelphia, Elsevier Saunders, 2012, vol 4, chap 137, p 3721. "

"A two-year-old girl has a large abdominal mass. Chest and abdominal CT scans show clear lung fields with a 12 cm tumor arising from the center of the right kidney and two 2-3 cm exophytic tumors of the mid- and lower poles of the left kidney. The next step is: A. initiate chemotherapy. B. bilateral percutaneous needle biopsy. C. exploration and bilateral open renal biopsy. D. right nephrectomy and partial left nephrectomy. E. bilateral nephrectomy."

"C: ""46 XY complete androgen insensitivity syndrome."" was the correct answer. Histology in this disorder of sex development patient demonstrates seminiferous tubules with no ovarian stroma. This finding is consistent with testicular differentiation, thus ruling out Turner syndrome (bilateral streak gonads), pure gonadal dysgenesis (bilateral streak gonads), mixed gonadal dysgenesis (one streak gonad, one testis), and congenital adrenal hyperplasia (two ovaries). Based on the histology and the normal external female genitalia, the patient has 46 XY complete androgen insensitivity syndrome (CAIS). It is not uncommon in AIS patients for the seminiferous tubules to demonstrate a paucity of germ cells, indeed many biopsies are found to contain Sertoli cells only. Diamond DA, Yu RN: Sexual differentiation: Normal and abnormal, Wein, AJ, Kavoussi LR, Novick AC, Partin AW, Peters CA (eds): CAMPBELL-WALSH UROLOGY, ed 10. Philadelphia, Elsevier Saunders, 2012, vol 4, chap 133, pp 3607-3623. "

"An 11-year-old girl undergoes bilateral inguinal hernia repairs. Gonads measuring 3.5 cm are found at the internal rings. Biopsies show seminiferous tubules with few germ cells, interstitial fibrosis, and no ovarian follicles. The external genitalia is female, and she has not yet started menses. The most likely diagnosis is: A. 45 X0 Turner syndrome. B. 46 XX congenital adrenal hyperplasia. C. 46 XY complete androgen insensitivity syndrome. D. 45 X/46 XY mixed gonadal dysgenesis. E. 46 XY pure gonadal dysgenesis."

"A: ""water restriction."" is correct. This patient has hyponatremia due to SIADH from self-medicating with DDAVP. He has a chronic condition (> 48 hours) with no symptoms and modest hyponatremia that can be corrected slowly over time. Removing the inciting agent (DDAVP) together with water restriction would be the optimal treatment. In the absence of symptoms, the addition of diuretics and saline replacement are not necessary. There is no role for oral sodium bicarbonate in this setting. Shoskes DA, McMahon AW: Renal physiology and pathophysiology, Wein, AJ, Kavoussi LR, Novick AC, Partin AW, Peters CA (eds): CAMPBELL-WALSH UROLOGY, ed 10. Philadelphia, Elsevier Saunders, 2012, vol 2, chap 38, pp 1039-1041. "

"An 18-year-old man with primary nocturnal enuresis is managed with oral DDAVP 0.6 mg/day. He has worsening nocturnal enuresis after starting college and increases his dose to 1 mg every night. He presents for a follow-up examination requesting another prescription for DDAVP. He is asymptomatic and a routine check of his serum sodium reveals a value of 129 mEq/l. After withdrawing the DDAVP, the next step is: A. water restriction. B. oral furosemide. C. oral sodium bicarbonate. D. I.V. 0.5 NS and furosemide. E. I.V. 3% hypertonic saline and furosemide."

"D: ""chronic prostatitis - pelvic pain failing traditional treatment modalities."" was the correct answer. Sacral nerve stimulation (SNS) has FDA approval for treatment of: non-obstructive urinary retention, urinary urge incontinence, urinary urgency-frequency syndrome, and for the gastrointestinal indication of chronic fecal incontinence. Clinical trials are ongoing for its use in interstitial cystitis, chronic prostatitis (i.e., chronic pelvic pain), and idiopathic constipation failing traditional treatment modalities. It has not been cleared by the FDA to be used in patients with urologic or gastrointestinal symptoms arising from a known neuropathy, such as multiple sclerosis, Parkinson's disease, a congenital neuropathic anomaly, post-traumatic spinal cord injury, and/or pelvic nerve injury arising from pelvic surgery. http://professional.medtronic.com/pt/uro/snm/ind/index.htm#.VmBm6NIrKig "

"An FDA approved clinical trial for placement of a sacral nerve stimulation device, exists for which clinical scenario: A. medication resistant urinary incontinence secondary to multiple sclerosis. B. bladder underactivity following a spinal cord injury. C. bladder underactivity following pelvic surgery such as a low anterior resection or hysterectomy. D. chronic prostatitis - pelvic pain failing traditional treatment modalities. E. medical refractory urinary and fecal incontinence associated with spina bifida."

"A: ""t-tests."" is correct. T-tests as well as analysis of variance, correlation coefficients, and linear multiple regression are used to compare results of a dependent variable that is measured as a continuous variable. These tests can compare groups that have means and standard deviations. When the dependent variable is measured as a binary variable, then statistics such as the frequency are able to be determined, but means and standard deviations or variance are not. In that case, the types of statistical procedures used are chi-square, Fisher's exact test, point-biserial correlation coefficients, and logistic regression. AUA Core Curriculum statistics section https://www.auanet.org/university/core_topic.cfm?coreid=122 "

"An antibiotic is being studied to test its ability to prevent UTI. The binary outcome measurement is the development of a UTI. Analysis of this binary variable would not include: A. t-tests. B. chi-square. C. Fisher's exact test. D. logistic regression. E. point-biserial correlation."

"C: ""ultrasound of bladder and kidneys."" was the correct answer. It is recognized that underlying genitourinary malformations or other pathologies are at least three-fold more common in pediatric patients relative to adults undergoing evaluation for trauma. This is a classic case for raising the concern of an underlying abnormality since the gross hematuria seems out of proportion with the low severity of the trauma. The underlying problems may include hydronephrosis, multicystic kidney, Wilms' tumor, and various renal fusion anomalies. Therefore, it is appropriate to image with ultrasound to look for such potentially significant problems. A patient should not be considered for admission with serial examinations and hematocrit determination unless there is a documented substantial renal injury. Cystoscopy is not indicated in the initial evaluation of gross hematuria in children. CT scan would only be indicated if there is significant injury or abnormality on ultrasound, or if the mechanism of injury was more concerning. Husmann DA: Pediatric genitourinary trauma, Wein, AJ, Kavoussi LR, Novick AC, Partin AW, Peters CA (eds): CAMPBELL-WALSH UROLOGY, ed 10. Philadelphia, Elsevier Saunders, 2012, vol 4, chap 138, p 3742. "

"An asymptomatic five-year-old boy develops gross hematuria two hours after wrestling with his younger brother. Physical examination is normal. The next step is: A. observation. B. serial examinations and hematocrit determination. C. ultrasound of bladder and kidneys. D. CT scan. E. cystoscopy."

"C: ""CIC."" was the correct answer. This patient has the valve bladder syndrome with progressive hydroureteronephrosis developing secondary to nephrogenic diabetes insipidus, obligatory high urine output, and a poorly compliant bladder at capacities of > 500 ml. With approximately 125-150 ml of urine output per hour, he would stay in fairly safe volume/pressure ranges if CIC was instituted every three hours. If hydroureteronephrosis persisted, then overnight catheterization would also be an option. Bladder augmentation would be too aggressive without an aggressive trial of medical management. Oral anticholinergics or onabotulinumtoxinA, if instituted without concurrent use of CIC, would be insufficient therapy due to already large PVR urine. Timed voiding every two hours while awake would also be reasonable option, however, this would not address his large residuals and may or may not address his hydroureteronephrosis and would demand strict follow-up to verify compliance with medical directives. Fluid restriction would not benefit this patient with nephrogenic diabetes insipidus and would only lead to relative dehydration and possibly worsening renal failure. Casale AJ: Posterior urethral valves, Wein, AJ, Kavoussi LR, Novick AC, Partin AW, Peters CA (eds): CAMPBELL-WALSH UROLOGY, ed 10. Philadelphia, Elsevier Saunders, 2012, vol 4, chap 126, pp 3402-3403."

"An eight-year-old boy with a history of incised posterior urethral valves has worsening hydroureteronephrosis. His creatinine is 1.3 mg/dl. He is on a timed void interval, with a voiding calendar revealing he voids 400-500 ml every three hours. He has overnight urine output of 1000 ml and remains continent throughout the day and night. Videourodynamics reveals no urethral obstruction or reflux, with detrusor pressures of 40 cm H2O at volumes 500 ml and end-fill detrusor pressures of 60 cm H2O at volumes 700 ml. He voids with pressures generated from the noncompliant bladder with residual urines of 200 ml. The next step is: A. fluid restriction. B. oral anticholinergics. C. CIC. D. onabotulinumtoxinA. E. bladder augmentation."

"B: ""MAG-3 renal scan."" was the correct answer. In this setting, one must determine if the hydronephrosis is causing significant obstruction and, if so, at what level in the urinary tract. The ultrasound may show ureteral dilation which suggests obstruction at the ureterovesical junction, but the MAG-3 renal scan will determine the differential function and the degree of obstruction. The ideal timing for performing this study is during the second month of life when the GFR has increased several fold. DMSA scan would not provide information regarding degree or level of obstruction. The magnetic resonance urogram may be useful after the first week of life, but does not provide differential renal function. Antegrade studies are not needed at this point and should not be performed prior to the less invasive MAG-3 renal scan. CT urogram does not provide quantitative data and exposes the infant to unnecessary radiation and anesthesia. Lee RS, Borer JG: Perinatal urology, Wein AJ, Kavoussi LR, Novick AC, Partin AW, Peters CA (eds): CAMPBELL-WALSH UROLOGY, ed 10. Philadelphia, Elsevier Saunders, 2012, vol 4, chap 114, p 3064. Palmer LS, Trachtman H: Renal functional development and diseases in children, Wein AJ, Kavoussi LR, Novick AC, Partin AW, Peters CA (eds): CAMPBELL-WALSH UROLOGY, ed 10. Philadelphia, Elsevier Saunders, 2012, vol 4, chap 112, p 3002. "

"Antenatally diagnosed marked bilateral hydronephrosis is confirmed postnatally in a newborn girl. VCUG is normal. The next step is: A. magnetic resonance urogram. B. MAG-3 renal scan. C. DMSA renal scan. D. CT urogram. E. repeat ultrasound in two months."

"A: ""stimulation of beta-3 adrenergic receptors via the hypogastric nerve."" is correct. The accommodation of the bladder to increasing volumes of urine is primarily a passive phenomenon dependent on the intrinsic properties of the vesical smooth muscle and the quiescence of the parasympathetic efferent pathway. Stimulation of alpha-receptors do not initiate bladder relaxation. Stimulation of beta-2 and beta-3 adrenergic receptors in the detrusor muscle results in the direct relaxation of the detrusor smooth muscle. Mirabegron, a beta-3 agonist, takes advantage of this pathway to improve symptoms of overactive bladder. Yoshimura N, Chancellor MB: Physiology and pharmacology of the bladder and urethra, Wein AJ, Kavoussi LR, Novick AC, Partin AW, Peters CA (eds): CAMPBELL-WALSH UROLOGY, ed 10. Philadelphia, Elsevier Saunders, 2012, vol 3, chap 60, pp 1790-1794."

"Bladder relaxation during filling is accomplished by: A. stimulation of beta-3 adrenergic receptors via the hypogastric nerve. B. stimulation of alpha-1 d receptors via the hypogastric nerve. C. stimulation of alpha-1 a receptors via the hypogastric nerve. D. inhibition of sympathetic input from the spinal cord. E. inhibition of nicotinic receptors via the pelvic nerve."

"A: ""abandon the operation."" is correct. Infection transmitted from a cadaver donor to an immunocompromised recipient can be rapidly fatal. Gross bacterial contamination of the operative site gives the surgeon only one single safe option which is to abandon the procedure. Barry JM, Conlin MJ: Renal transplantation, Wein, AJ, Kavoussi LR, Novick AC, Partin AW, Peters CA (eds): CAMPBELL-WALSH UROLOGY, ed 10. Philadelphia, Elsevier Saunders, 2012, vol 2, chap 44, pp 1236-1237. B: ""ureteral stent and urethral catheter."" was the correct answer. The patient has a delayed urine leak following high velocity penetrating renal trauma. CT scan and retrograde pyelogram reveal a urine leak. The next step is ureteral stent and urethral catheter drainage to prevent reflux of urine retrograde into the stent which may potentiate the leak and result in infection of the retroperitoneal hematoma. Ureteral stent placement may allow for resolution of the leak without operative intervention. The patient's recent history of shattered kidney along with liver and bowel injury placement of a percutaneous nephrostomy tube would result in increased risk for infection of the perinephric hematoma and maybe difficult to place and maintain in the proper position due to the renal injury. Exploration of the right kidney is not indicated at this time. Santucci RA, Doumanian LR: Upper urinary tract trauma, Wein, AJ, Kavoussi LR, Novick AC, Partin AW, Peters CA (eds): CAMPBELL-WALSH UROLOGY, ed 10. Philadelphia, Elsevier Saunders, 2012, vol 2, chap 42, p 1178. "

"During a laparoscopic donor nephrectomy, a 2 cm laceration is made in the splenic flexure of the colon. The most appropriate action is to: A. abandon the operation. B. close the laceration, administer broad-spectrum antibiotics to the donor, and continue with the transplantation. C. close the laceration, irrigate wound with neomycin, and proceed with the transplantation. D. close the laceration, administer broad-spectrum intravenous antibiotics to the donor and recipient, and proceed with the transplantation. E. close the laceration, irrigate wound with neomycin, administer broad-spectrum antibiotic to the donor and recipient, and proceed with the transplantation."

"D: ""application of the endovascular stapler onto the main renal vein."" was the correct answer. Excessive placement of clips when managing the gonadal, adrenal, or lumbar branches can severely restrict the working space available for safe placement of the endovascular stapler later in the case when the main renal vein is addressed. Application of the stapler across a clip can lead to intraoperative or postoperative hemorrhage and must be avoided. Use of clips should have no effect on the ability to perform the lymphadenectomy, adrenalectomy, or the ability to achieve negative margins, or ability to control renal artery. Kavoussi LR, Schwartz MJ, Gill IS: Laparoscopic surgery of the kidney, Wein, AJ, Kavoussi LR, Novick AC, Partin AW, Peters CA (eds): CAMPBELL-WALSH UROLOGY, ed 10. Philadelphia, Elsevier Saunders, 2012, vol 2, chap 55, pp 1638-1639. "

"During laparoscopic left radical nephrectomy, judicious placement of clips on the primary branches of the main renal vein is most important to facilitate: A. lymphadenectomy. B. en bloc excision with negative margins. C. adrenalectomy. D. application of the endovascular stapler onto the main renal vein. E. dissection and occlusion of the main renal artery(ies)."

"A: ""reduce insufflation pressure."" is correct. Renal artery vasospasm may occur during renal hilar dissection, leading to reduced renal perfusion. The first step should be to simply reduce the insufflation pressure. This maneuver will allow improved renal perfusion and no additional steps are necessary in the majority of patients. If renal perfusion continued to be suboptimal, topical papaverine applied to the renal hilar vessels would be the next step. A fluid bolus in the presence of increased intraperitoneal pressure from the pneumoperitoneum is usually of little to no benefit. There is no role for systemic heparinization in this patient. Eichel L, Clayman RV: Basics of laparoscopic urologic surgery, in Wein AJ, Kavoussi LR, Novick AC, Partin AW, Peters CA (eds): CAMPBELL'S UROLOGY, ed 10. Philadelphia, Saunders Elsevier, 2011, vol 1, chap 9, pp 231-232. Ratner LE, Montgomery RA, Kavoussi LR: Laparoscopic live donor nephrectomy: A review of the first five years. UROL CLIN N AM 2001;28:709-719."

"During laparoscopic live donor nephrectomy, the kidney becomes visibly pale with loss of turgor after 15 minutes of hilar dissection. The next step is: A. reduce insufflation pressure. B. fluid bolus. C. intraarterial papaverine. D. systemic heparin. E. convert to open nephrectomy."

"D: ""cystoscopy."" was the correct answer. The patient appears to be obstructed following mid-urethral sling placement and should be considered for sling incision. Urodynamic data can be useful in select cases, but if the symptoms began after the sling was placed, she will likely need to have the sling incised regardless of urodynamic findings. Cystoscopy is necessary, however, to evaluate for urethra or bladder mesh perforation and other bladder pathology, as well as hypersuspension, and should be done before planned sling incision. Cystoscopy is of particular importance in the patient with hematuria. Sling incision will likely be necessary but only after cystoscopy has ruled-out overt urethral erosion or other bladder/urethral pathology. Office urethral dilation is not recommended at such a late period of time after sling surgery as the synthetic mesh is likely scarred in at this time. Tamsulosin may help mild functional obstructions but iatrogenic anatomic obstruction is unlikely to improve. Dmochowski RR, Padmanabhan P, Scarpero HM: Slings: Autologous, biologic, synthetic, and midurethral, Wein, AJ, Kavoussi LR, Novick AC, Partin AW, Peters CA (eds): CAMPBELL-WALSH UROLOGY, ed 10. Philadelphia, Elsevier Saunders, 2012, vol 3, chap 73, pp 2148-2150. "

"Eight weeks following placement of a mid-urethral polypropylene sling, a 73-year-old woman has no leakage but reports difficulty voiding. Her urinalysis shows large numbers of RBCs and WBCs, and PVR is 300 ml. She is started on antibiotics. The next step is to teach her CIC and: A. pressure flow urodynamics. B. urethral dilation. C. tamsulosin. D. cystoscopy. E. incise sling."

"A: ""ammonia."" is correct. Primary idiopathic (endemic) calculi form in children are commonly found in children from North Africa, the Middle East, and the Far East. With a large immigrant population in the United States, it is important to be aware of this health problem. These children classically rely on a cereal-based diet that is lacking in animal proteins. The lack of protein leads to a dietary phosphate deficiency, low urinary phosphate, and high levels of urinary ammonia. Due to this, the most common stone found in children from these areas is ammonium acid urate. High urinary sodium, calcium, and oxalate are not characteristic findings with endemic bladder stones. Benway BM, Bhayani SB: Lower urinary tract calculi, Wein, AJ, Kavoussi LR, Novick AC, Partin AW, Peters CA (eds): CAMPBELL-WALSH UROLOGY, ed 10. Philadelphia, Elsevier Saunders, 2012, vol 3, chap 89, p 2522."

"Endemic bladder stones in children form due to high urinary excretion of: A. ammonia. B. phosphate. C. sodium. D. calcium. E. oxalate."

"D: ""hypoxia-inducible factor-1 alpha."" was the correct answer. Production of erythropoietin (EPO) is closely associated with circulating oxygen levels. During conditions of hypoxia, hypoxia-inducible factor-1-alpha (HIF-1-a) is upregulated increasing EPO transcription. HIF-1-a is then rapidly degraded by proteases upon restoration of normal oxygen tension. Renin does not directly influence EPO production. Testosterone and angiotensin II induce erythrocytosis through proposed mechanisms, including stimulation of EPO, however, these are thought to be less significant than HIF-1-a. Mutation of the VHL gene such as in patients with von Hippel-Lindau syndrome results in constitutive production of HIF-1-a, increased EPO with resultant polycythemia. HIF-1-a also upregulates production of vascular endothelial growth factor (VEGF), which is the primary angiogenic factor responsible for the neovascularity associated with RCC. Shoskes DA, McMahon AW: Renal physiology and pathophysiology, Wein, AJ, Kavoussi LR, Novick AC, Partin AW, Peters CA (eds): CAMPBELL-WALSH UROLOGY, ed 10. Philadelphia, Elsevier Saunders, 2012, vol 2, chap 38, pp 1029-1030. "

"Erythropoietin production in the kidney is regulated primarily by: A. angiotensin II. B. high oxygen tension. C. renin. D. hypoxia-inducible factor-1 alpha. E. testosterone."

C: ""revision of conduit length and stoma."" was the correct answer. Despite improvement in surgical techniques, stomal stenosis is still the most likely cause of the elongated dilated conduit with hydronephrosis and hyperchloremic acidosis. Therefore, the most appropriate treatment is revision of the stoma and shortening of the conduit. Administration of electrolytes is symptomatic treatment only and does not address the anatomic cause of the acidosis. Conversion to a non-refluxing colon conduit is not required in this setting where a simpler procedure, shortening of the ileal conduit and stomal revision, will suffice. Treatment of high grade vesicoureteral reflux by a bulking agent in an ileal conduit is not indicated; the high grade reflux is a direct sign of distal obstruction and is usually due to either stomal or mid-ileal loop stenosis. Although this patient may eventually need a renal transplant, a functional loop is necessary for transplantation, therefore, revision of the loop is indicated. No indications are currently present for proceeding with bilateral nephrectomy. Dahl DM, McDougal WS: Use of intestinal segments in urinary diversion, Wein, AJ, Kavoussi LR, Novick AC, Partin AW, Peters CA (eds): CAMPBELL-WALSH UROLOGY, ed 10. Philadelphia, Elsevier Saunders, 2012, vol 3, chap 85, pp 2425-2426. "

"Five years after a radical cystectomy and ileal conduit for bladder cancer, a 65-year-old man has a serum creatinine of 3.0 mg/dl, BUN 40 mg/dl, sodium 146 mEq/l, potassium 4.6 mEq/l, and bicarbonate 16 mEq/l. A loopogram shows a 35 cm long moderately dilated conduit with grade 4 reflux into dilated upper tracts. The most appropriate treatment is: A. sodium citrate. B. injection of bulking agent into the ureteroenteric anastomosis. C. revision of conduit length and stoma. D. excision of ileal conduit and conversion to a non-refluxing colon conduit. E. bilateral nephrectomies and placement on transplant list."

"B: ""ureteral stent and urethral catheter placement."" was the correct answer. The CT image demonstrates a grade 4 renal laceration with extravasation of urine into a perinephric hematoma. The patient's symptoms of a low grade fever and ileus are consistent with peritoneal irritation from the perinephric urinoma and treatment is indicated. Options for management would be retrograde stent placement, with or without concurrent urethral catheter drainage, percutaneous nephrostomy or percutaneous drain placement. Retrograde stent placement provides the patient with a faster return to ambulation and normal activities with less concern for infection of the perinephric hematoma by the percutaneous tract. The disadvantage of placing a ureteral stent in children is the need for anesthesia for placement and removal and the small size of the ureteral stent. Specifically, if there is a large amount of blood clot in the renal pelvis, the clot may occlude the small ureteral stent with persistent extravasation of urine into the urinoma. In this radiograph, there is no evidence of significant blood clot within the renal pelvis and the urinoma is small in size, therefore, percutaneous nephrostomy or drain placement is not indicated. There is no significant hematoma present and, therefore, no need for selective angiographic embolization. The best treatment is with ureteral stent placement with concurrent drainage of the bladder via a urethral catheter to prevent free reflux up the stent from causing persistent symptoms. Once the ileus has resolved, the urethral catheter can be removed. Use of a urethral catheter alone would not be sufficient to treat the urinoma. Husmann DA: Pediatric genitourinary trauma, Wein, AJ, Kavoussi LR, Novick AC, Partin AW, Peters CA (eds): CAMPBELL-WALSH UROLOGY, ed 10. Philadelphia, Elsevier Saunders, 2012, vol 4, chap 138, p 3735."

"Following a snowboarding accident, a 13-year-old girl has gross hematuria and a renal injury. Eight days later, she has a persistent ileus and a low-grade fever. CT scan is shown. The next step is: A. urethral catheter placement. B. ureteral stent and urethral catheter placement. C. percutaneous nephrostomy. D. percutaneous drain placement. E. selective angiographic embolization."

"A: ""planimetry."" is correct. Unfortunately, no prostate is the perfect sphere, ellipse, or prolate sphere, rendering calculations somewhat inaccurate. The most accurate means of volume measurement by ultrasound is planimetry, which allows for variation in shape. In this, the probe is mounted on a stepping device and the signal marched through the gland at defined intervals, usually 3-5 mm. At each interval, the surface area of the prostate image is obtained. Volume is calculated by multiplying the sum of the surface areas by the stepping interval. While a prolate ellipsoid is the calculation typically used in TRUS of the prostate, it is not as accurate as planimetry. Because of its superior accuracy, planimetry is the method of choice for brachytherapy treatment planning. Trabulsi EJ, Halpern EJ, Gomella LG: Ultrasonography and Biopsy of the Prostate, Wein, AJ, Kavoussi LR, Novick AC, Partin AW, Peters CA (eds): CAMPBELL-WALSH UROLOGY, ed 10. Philadelphia, Elsevier Saunders, 2012, vol 3, chap 97, pp 2737-2738. "

"For dosimetry planning for brachytherapy, the most accurate method to estimate prostate volume by ultrasound is based on: A. planimetry. B. a sphere. C. an ellipse. D. a prolate ellipse. E. a prolate sphere."

"D: ""loss of afferent input from the detrusor to the central nervous system."" was the correct answer. Isolated nocturnal incontinence is commonly noted in patients who have undergone radical cystectomy and orthotopic neobladder. This is thought to be related to loss of afferent input from the detrusor to the central nervous system. Normally, the afferent input causes a reflex rise in urethral pressure during reservoir filling. This reflex is maintained following radical prostatectomy which explains why isolated nocturnal incontinence is uncommon after this procedure. If there was damage to the rhabdosphincter, the patient would experience incontinence during the day and night, as opposed to isolated nocturnal incontinence. This would also be the case if the neobladder had poor compliance or had contractions. While some authors have suggested that damage to the pelvic and hypogastric plexus contributes to incontinence after cystectomy, this also would not cause isolated nighttime incontinence. Skinner EC, Skinner DG, Stein JP: Orthotopic urinary diversion, Wein, AJ, Kavoussi LR, Novick AC, Partin AW, Peters CA (eds): CAMPBELL-WALSH UROLOGY, ed 10. Philadelphia, Elsevier Saunders, 2012, vol 3, chap 87, pp 2499-2500."

"In a man who has good daytime continence following radical cystectomy and orthotopic neobladder, nocturnal incontinence is due to: A. damage to the urinary rhabdosphincter. B. neobladder hypercontractility. C. inadequate compliance of the neobladder. D. loss of afferent input from the detrusor to the central nervous system. E. damage to the inferior hypogastric nerve plexus."

"B: ""broad spectrum antibiotic therapy."" was the correct answer. Early recognition of the diagnosis, aggressive surgical debridement and the use of broad-spectrum antibiotics are the most essential interventions in stopping the rapidly progressing infectious process of Fournier's gangrene. The combination of aggressive surgical therapy and appropriate antibiotic coverage has been documented to result in a reduction in mortality. Although high volume centers may have lower mortality, transfer to a tertiary facility may interrupt or delay care and should not be considered until the patient is initially stabilized. While hyperbaric oxygen has been proposed to reduce ongoing necrotizing tissue loss, it has not been found to reduce mortality. Glycemic control while important has also not been shown to influence survival. Diversion of urinary or fecal streams play a major role in reducing postoperative wound complications and length of postoperative care but have not been directly correlated to increased patient survival. Norton KS, Johnson LW, Perry T, et al: Management of Fournier's gangrene: An eleven year retrospective analysis of early recognition, diagnosis, and treatment. AM SUR 2002;68:709-713. Sorensen MD, Krieger JN, Rivara FP, et al: Fournier's gangrene: Management and mortality predictors in a population based study. J UROL 2009;182:2742-2747. "

"In a patient with Fournier's gangrene, the factors found to improve patient survival are early recognition of the diagnosis, aggressive surgical debridement and: A. transfer to a tertiary care center. B. broad spectrum antibiotic therapy. C. hyperbaric oxygen. D. aggressive glycemic control. E. placement of a suprapubic tube."

"E: ""penile exploration."" was the correct answer. A known complication of intra-lesional collagenase is a penile fracture. Thus, patients are advised not to engage in sexual activity for two weeks after the injection in order to mitigate the risk of penile fracture. In this case, it is highly likely that he has experienced a penile fracture and he should be surgically explored. A cystoscopy would be warranted at the time of penile exploration to rule-out a concurrent urethral injury. Delaying the second injection to > 72 hours, using anti-inflammatories or applying a pressure dressing, would be appropriate if the penile pain would have occurred following the injection only, but would not be the best option in treating a patient who developed symptoms during or following coitus. Jordan GH, McCammon KA: Peyronie's disease, Wein, AJ, Kavoussi LR, Novick AC, Partin AW, Peters CA (eds): CAMPBELL-WALSH UROLOGY, ed 10. Philadelphia, Elsevier Saunders, 2012, vol 1, chap 28, p 801. Gelbard M, Goldstein I, Hellstron WJ, et al: Clinical efficacy, safety and tolerability of collagenase clostridium histolyticum for the treatment of Peyronie's disease in two large double-blind, randomized, placebo controlled phase 3 studies. J UROL 2013;190:199-207."

"One day following his first injection of his first cycle of intra-lesional collagenase (Xiaflex™) for Peyronie's disease, a 54-year-old-man experiences penile pain, edema, and ecchymosis after sex. The next step is: A. anti-inflammatory. B. pressure dressing. C. delay the second collagenase injection to greater than 72 hours. D. cystoscopy. E. penile exploration."

"E: ""no better than a basic office evaluation in determining surgical outcomes."" was the correct answer. The VALUE trial assessed the utility of urodynamics versus basic office evaluation in the uncomplicated patient with stress incontinence (SUI). Uncomplicated patients with SUI are those with demonstrable SUI on physical exam, no overactive bladder symptoms, normal bladder emptying, no prior surgery for SUI, and no signs, symptoms, or past medical history consistent with a possible neurologic etiology for their urinary incontinence. This study documented that within the uncomplicated patient population, the addition of urodynamic studies did not improve the surgical outcome following incontinence surgery compared to women who had a basic office evaluation alone. A basic office evaluation includes a pelvic exam with supine stress test with a full bladder, assessment of PVR, and urinalysis. Nager, CW, Brubaker L, Litman HJ, et al: A randomized trial of urodynamics testing before stress incontinence surgery. NEJM 2012;366:1987-1997. Zimmern, P, Litman H, Nagler C, et al: Pre-operative urodynamics in women with stress urinary incontinence increases physician confidence, but does not improve outcomes. NEUROUROL URODYN 2014;33:302-306. "

"Prior to surgery for uncomplicated stress incontinence in women, urodynamics is: A. necessary for all patients having sling surgery. B. useful in determining which type of sling procedure (transobturator versus retropubic) to perform. C. useful in determining prognosis following sling surgery. D. able to determine prognosis following an autologous fascia sling only. E. no better than a basic office evaluation in determining surgical outcomes."

"D: ""decrease in sympathetic activity."" was the correct answer. Endovascular radiofrequency ablation (RFA) of the renal artery results in destruction of the renal sympathetics and is being increasingly utilized for management of pharmacologically resistant hypertension. Sympathetic innervation to the kidneys run in the adventitial wall of the renal arteries. Stimulation of the renal sympathetics results in a decrease in renal blood flow, an increase in renin secretion, retention of sodium and water, and hypertension. Circumferential RFA of the renal sympathetics occurs by introduction of a 6 Fr catheter by the femoral vein; the therapeutic catheter has a steerable tip and is connected to a console that delivers radiofrequency energy. The level of energy delivered is monitored with a sensor at the tip of the catheter so that excessive tissue injury is avoided. Ablation of the renal sympathetics results in dilation of the renal efferent arteries, decreased plasma renin activity, and increased renal blood flow. In addition, it is postulated that renal artery sympathetic ablation results in stimulating an inhibitory regulatory feedback mechanism that decreases the overall sympathetic outflow. Renal artery RFA has no effect on parasympathetic nerve activity. It has been associated with rare intimal dissections of the renal artery and renal artery aneurysms that may require emergent nephrectomy. Symplicity HTN-2 Investigators, Esler MD, Krum H, Sobotka PA, Schlaich MP, Schmieder RE, Böhm M: Renal sympathetic denervation in patients with treatment-resistant hypertension (The Symplicity HTN-2 Trial): A randomised controlled trial. LANCET 2010;376:1903-1909. "

"Radiofrequency renal artery denervation for the treatment of medically refractory hypertension causes a(n): A. increase in plasma renin activity. B. decrease in renal blood flow. C. increase in renal efferent arterial tone. D. decrease in sympathetic activity. E. increase in parasympathetic activity."

"C: ""pons."" was the correct answer. The center for integration and coordination of bladder and urethral activity is in the pons (pontine micturition center). Suprasacral spinal cord injury disrupts the necessary communication between the pontine micturition center and the sacral cord. This results in detrusor overactivity and detrusor external sphincter dyssynergia. Patients with this issue may have urinary incontinence, incomplete bladder emptying, and their upper tracts may be at risk. Yoshimura N, Chancellor MB: Physiology and pharmacology of the bladder and urethra, Wein AJ, Kavoussi LR, Novick AC, Partin AW, Peters CA (eds): CAMPBELL-WALSH UROLOGY, ed 10. Philadelphia, Elsevier Saunders, 2012, vol 3, chap 60, p 1810. "

"Reflex bladder and urethral activity are coordinated by a reflex center located in the: A. sacral cord. B. thoracic and sacral cord. C. pons. D. medulla. E. basal ganglia."

"E: ""metastatic disease at presentation."" was the correct answer. Renal medullary carcinoma was first described in 1995 and, to date, less than 100 cases have been reported in the literature. It is an extremely aggressive disease with nearly all patients presenting with metastatic disease. Mean survival is less than six months and only two long-term survivors are reported in the literature. It has not been shown to respond to standard chemotherapy and/or immunotherapy regimens. Interestingly, approximately three-fourths of cases have been reported to involve the right kidney. Recent studies have reported amplification of the ABL gene and increased expression of ABL protein in the absence of BCR-ABL translocation characteristic of chronic myeloid leukemia. Medullary carcinoma is more common in patients with sickle cell trait. It tends to be central and infiltrative. Campbell SC, Lane BR: Malignant renal tumors, Wein, AJ, Kavoussi LR, Novick AC, Partin AW, Peters CA (eds): CAMPBELL-WALSH UROLOGY, ed 10. Philadelphia, Elsevier Saunders, 2012, vol 2, chap 49, p 1436. Simpson L, He X, Pins M, et al: Renal medullary carcinoma and ABL gene amplification. J UROL 2005;173:1883-1888."

"Renal medullary carcinoma is characterized by: A. occurrence predominately in the left kidney. B. relatively indolent clinical course. C. good response to cisplatin-based chemotherapy. D. typically amenable to partial nephrectomy. E. metastatic disease at presentation."

" B: ""renin."" is correct. ACTH and serum potassium may increase aldosterone secretion but the effect is much less pronounced than that achieved by renin. The primary mechanism for control of aldosterone production resides in a feedback system involving the kidney and its juxtaglomerular apparatus. In the presence of appropriate stimuli (i.e., decreased renal perfusion pressure), juxtaglomerular cells release renin which results in the increased production of angiotensin II. Angiotensin II is a potent stimulator of aldosterone output from the zona glomerulosa of the adrenal cortex. Sodium concentration in the thick ascending Loop of Henle (not the proximal tubule or collecting duct) can also be sensed by the macula densa which can stimulate the release of renin. Kutikov A, Crispen PL, Uzzo RG: Pathophysiology, evaluation, and medical management of adrenal disorders, Wein, AJ, Kavoussi LR, Novick AC, Partin AW, Peters CA (eds): CAMPBELL-WALSH UROLOGY, ed 10. Philadelphia, Elsevier Saunders, 2012, vol 2, chap 57, pp 1697-1698. "

"Secretion of aldosterone is primarily mediated by: A. ACTH. B. renin. C. serum potassium level. D. sodium concentration in the proximal tubule. E. sodium concentration in the collecting duct. "

"B: ""vaginal estrogen and repeat examination in three months."" was the correct answer. Even though mesh is not directly palpable on exam, the patient should be presumed to have an extrusion based on the bloody vaginal discharge six months out from surgery. Observation may be reasonable early on, but is unlikely to lead to resolution of the extrusion at this point. Vaginal estrogen is reasonable for a small exposure, such as this one, and failing that, partial sling excision with reapproximation of the vaginal edges may be necessary. Alternatively, one may wish to go straight to partial excision with reapproximation. Complete sling excision would not be required in the case of small extrusion such as this one. If the patient had not had prior hysterectomy, then evaluation for vaginal bleeding (particularly uterine pathology) should be considered. If this patient had blood in the urine or severe new onset urinary symptoms, then cystoscopy would be warranted. Dmochowski RR, Padmanabhan P, Scarpero HM: Slings: Autologous, biologic, synthetic, and midurethral, Wein AJ, Kavoussi LR, Novick AC, Partin AW, Peters CA (eds): CAMPBELL-WALSH UROLOGY, ed 10. Philadelphia, Elsevier Saunders, 2012, vol 3, chap 73, p 2115. "

"Six months following an uneventful transobturator mid-urethral sling, a 63-year-old woman with previous hysterectomy has a slightly bloody vaginal discharge, but no mesh is visible. Urinalysis is normal. The next step is: A. repeat examination in three months. B. vaginal estrogen and repeat examination in three months. C. cephalexin and reevaluate in one month. D. cystoscopy. E. complete sling excision."

"A: ""ampicillin."" is correct. Neonates and young infants should be covered for Enterococcus species when choosing empiric antibiotics since the incidence of infections with this uropathogen is higher in early infancy than at a later age. Enterococcus is frequently sensitive to ampicillin and first-generation cephalosporins. Ceftriaxone and trimethoprim-sulfamethoxazole are specifically contraindicated in neonates with jaundice. Nitrofurantoin would not provide systemic treatment and is both contraindicated in infants with jaundice and in infants less than one month of age due to an increased risk for hemolytic anemia arising from the interactions of an immature liver and the medication. Ciprofloxacin typically would not cover the most common infecting organism, enterococcus, and is contraindicated in this age group. Shortliffe LMD: Infection and inflammation of the pediatric genitourinary tract, Wein, AJ, Kavoussi LR, Novick AC, Partin AW, Peters CA (eds): CAMPBELL-WALSH UROLOGY, ed 10. Philadelphia, Elsevier Saunders, 2012, vol 4, chap 116, p 3085. "

"The antibiotic of choice for a three-week-old girl with a febrile UTI and mild jaundice is: A. ampicillin. B. ceftriaxone. C. trimethoprim-sulfamethoxazole. D. nitrofurantoin. E. ciprofloxacin."

"A: ""reflex erection, no emission, and no ejaculation."" is correct. Three neural pathways are necessary for complete sexual function. They are: sympathetic (T10-L2), parasympathetic (S2-S4), and motor function to the perineal muscles from the somatic motor neurons (S2-S4). Erections may be either cerebrally or directly induced by penile stimulation, i.e., reflexic erections. Cerebral induced erections occur in response to visual or sensory erotic stimulation to the cerebral cortex. Erections by this method occur via central innervation of the parasympathetic pathway, through the S2-S4 nerve routes and require an intact spinal cord. The parasympathetic pathway can also be stimulated by direct penile stimulation through S2-S4 sensory fiber stimulation to cause erections, i.e., reflexic induced erections. The three phases necessary for semen production are: emission, closure of the bladder neck, and ejaculation. In response to cerebral stimulation, signals are conducted down the thoracolumbar sympathetic nerves, resulting in sympathetic induce contraction of prostatic smooth muscle, seminal vesicles, and the vas deferens and allowing pre-ejaculatory fluid to be deposited into the prostatic urethra, a process called emission. Bladder neck closure occurs concurrently with emission in response to alpha-sympathetic stimulation by the sympathetic innervations. Emission will result in swelling of the bulbar urethra, the distension of the bulbar urethra stimulating rhythmic contractions of the bulbocavernosus, and pelvic floor muscles under somatic control through S2-S4 nerve roots (Onuf's nucleus). Semen is then projected in an antegrade fashion. Disruption of the sympathetic nerve fibers that travel through these plexuses with a complete spinal cord injury between T12-L2 can, therefore, cause loss of seminal vesicle emission and/or failure of bladder neck closure. Distention of the prostatic urethra will not occur, i.e., failure of emission, and the somatic controlled periurethral and pelvic muscles will not be stimulated to contract resulting in anejaculation. Reflexic erections, however, will be maintained since the parasympathetic and penile sensory S2-S4 nerve plexuses are intact. Anderson JK, Cadeddu JA: Surgical anatomy of the retroperitoneum, adrenals, kidneys, and ureters, Wein AJ, Kavoussi LR, Novick AC, Partin AW, Peters CA (eds): CAMPBELL-WALSH UROLOGY, ed 10. Philadelphia, Elsevier Saunders, 2012, vol 1, chap 1, pp 14-18. Lue TF: Physiology of penile erection and pathophysiology of erectile dysfunction, Wein AJ, Kavoussi LR, Novick AC, Partin AW, Peters CA (eds): CAMPBELL-WALSH UROLOGY, ed 10. Philadelphia, Elsevier Saunders, 2012, vol 1, chap 23, pp 693-695. Sabanegh E, Agarwal A: Male infertility, Wein AJ, Kavoussi LR, Novick AC, Partin AW, Peters CA (eds): CAMPBELL-WALSH UROLOGY, ed 10. Philadelphia, Elsevier Saunders, 2012, vol 1, chap 21, pp 642-643."

"The baseline sexual and ejaculatory function of a man with a complete L1 spinal cord injury is: A. reflex erection, no emission, and no ejaculation. B. no erection, normal emission, and retrograde ejaculation. C. no erection, no emission, and no ejaculation. D. reflex erection, normal emission, and normal ejaculation. E. reflex erection, normal emission, and retrograde ejaculation."

"C: ""defective renal ammonia excretion."" was the correct answer. Uric acid stones are found in 34% of stone-forming patients with diabetes mellitus compared to 6% of non-diabetic stone formers. In essence, diabetic stone formers are approximately six fold more likely to develop a uric acid stone. Accordingly, diabetic stone formers have a lower urine pH compared with non-diabetic stone formers. The etiology of low urine pH in diabetic uric acid stone formers is believed to be due to insulin resistance. In normal individuals, insulin stimulates ammoniagenesis in renal tubule cells by promoting gluconeogenesis from glutamine and by stimulating ammonium excretion by the proximal tubular sodium/hydrogen exchanger. In obese patients with insulin resistance as seen in metabolic syndrome, failure of the renal tubule cells to respond to insulin results in defective ammonia production and/or excretion, thereby leading to a reduction in urinary pH and uric acid stone formation. Pearle MS, Lotan Y: Urinary lithiasis: Etiology, epidemiology, and pathogenesis, Wein, AJ, Kavoussi LR, Novick AC, Partin AW, Peters CA (eds): CAMPBELL-WALSH UROLOGY, ed 10. Philadelphia, Elsevier Saunders, 2012, vol 2, chap 45, p 1278."

"The cause of low urine pH in uric acid stone formers with Type II diabetes mellitus is: A. glucosuria. B. type IV RTA. C. defective renal ammonia excretion. D. ketone body excretion. E. recurrent UTIs."

"A: ""hypotension."" is incorrect. C: ""cutaneous hyperpigmentation."" was the correct answer. Since the amino acids of ACTH are identical to the terminal amino acids of melanocyte stimulating hormone (MSH), over-production of ACTH results in cutaneous hyperpigmentation. Adrenal loss results in lack of negative feedback and over-production of ACTH. Pituitary failure, on the other hand, results in a lack of ACTH. Vitiligo may also be seen in these patients. Primary hypoadrenalism (or Addison's disease) is notable for hyponatremia (not hypernatremia) and hyperkalemia (not hypokalemia). Hypotension can be present in primary or secondary hypoadrenalism. Kutikov A, Crispen PL, Uzzo RG: Pathophysiology, evaluation, and medical management of adrenal disorders, Wein, AJ, Kavoussi LR, Novick AC, Partin AW, Peters CA (eds): CAMPBELL-WALSH UROLOGY, ed 10. Philadelphia, Elsevier Saunders, 2012, vol 2, chap 57, pp 1711-1713. "

"The characteristic that distinguishes primary hypoadrenalism from secondary (pituitary) hypoadrenalism is: A. hypotension. B. metabolic alkalosis. C. cutaneous hyperpigmentation. D. hypernatremia. E. hypokalemia."

"E: ""intramuscular injection."" was the correct answer. Testosterone stimulates erythropoiesis with a direct correlation between hemoglobin and hematocrit levels and testosterone levels. Injection of testosterone is associated with higher potential for secondary erythrocytosis than topical or oral preparations. High levels of testosterone are associated with polycythemia (hematocrit over 51%). Due to the increased risk of clotting complications with an elevated hematocrit, it is recommended that the testosterone dose is adjusted and/or periodic phlebotomy be performed to keep the hematocrit below 52-55%. All patients placed on testosterone should have periodic CBC assessments with the initial evaluation for erythrocytosis occurring three months following initiation of treatment. Bassil N, Alkaade S, Morley JE: The benefits and risks of testosterone replacement therapy: A review. THERAPEUTICS AND CLINICAL RISK MANAGEMENT 2009;5:438. Bhasin S, Cunningham GR, Hayes FJ: Testosterone therapy in men with androgen deficiency syndromes: An Endocrine Society Clinical Practice Guideline. J CLIN ENDOCRINOL METAB 2010;95:2536-2559. "

"The form of testosterone most commonly associated with erythrocytosis and an increased risk of thrombotic events is: A. topical gel. B. transdermal patch. C. oral. D. sublingual. E. intramuscular injection."

"C: ""intraoperative bleeding."" was the correct answer. Laparoscopic retroperitoneal lymphadenectomy (RPLND) has become a feasible technique compared to the open approach. The laparoscopic complications seem to compare favorably to the historic open RPLND series with the exception that an increased blood loss with the laparoscopic technique over the open technique has been reported. This complication appears to be related to the surgeon's experience and is more common during the initial learning curve. That stated, this complication is still the most commonly reported complication of the laparoscopic procedure. The adequacy of the laparoscopic lymph node dissection in an experienced surgeon's hands is similar to open surgery with no increase in tumor recurrence noted. The observed frequencies in retrograde ejaculation, chylous ascites, and prolonged ileus are no different compared to the open approach. Allaf JE, Kavoussi LR: Laparoscopic retroperitoneal lymphadenectomy for testicular tumors, Wein, AJ, Kavoussi LR, Novick AC, Partin AW, Peters CA (eds): CAMPBELL-WALSH UROLOGY, ed 10. Philadelphia, Elsevier Saunders, 2012, vol 1, chap 33, p 898. "

"The most common complication associated with laparoscopic RPLND is: A. retrograde ejaculation. B. bowel injury. C. intraoperative bleeding. D. chylous ascites. E. prolonged ileus."

"A: ""decrease utilization of intravesical therapies."" is incorrect. D: ""increase detection of CIS."" was the correct answer. Hexaminolevulinate hydrochloride was approved in the U.S. by the FDA in 2010 as an optical imaging agent indicated for use in the cystoscopic detection of non-muscle invasive bladder cancers. It is used with a cystoscopic photodynamic detection system to perform so-called fluorescent, or ""blue light"" cystoscopy to aide in the detection of bladder tumors. According to the pivotal phase III North American multicenter trial, fluorescence cystoscopy improved the detection of both CIS and of papillary tumors as compared to conventional white light cystoscopy. As of yet, these pivotal trials have not demonstrated a reduction in progression rates, cystectomy rates, or use of intravesical therapies. Of note, it is contraindicated in patients who have received BCG within 90 days of treatment and, therefore, is not commonly utilized at this time for the tumor detection post-BCG. Jones JS, Larchian WA: Non-muscle-invasive bladder cancer (Ta, T1, and CIS), Wein, AJ, Kavoussi LR, Novick AC, Partin AW, Peters CA (eds): CAMPBELL-WALSH UROLOGY, ed 10. Philadelphia, Elsevier Saunders, 2012, vol 3, chap 81, p 2342. Burger M, Grossman HB, Droller M, et al: Photodynamic diagnosis of non-muscle-invasive bladder cancer with hexaminolevulinate cystoscopy: A meta-analysis of detection and recurrence based on raw data. EUR UROL 2013;64:846-854. Fradet Y, Grossman HB, Gomella L, et al: PC B302/01 Study Group. A comparison of hexaminolevulinate fluorescence cystoscopy and white light cystoscopy for the detection of carcinoma in situ in patients with bladder cancer: A phase III, multicenter study. J UROL 2007;178:68-73. Grossman HB, Gomella L, Fradet Y, et al; PC B302/01 Study Group. A phase III, multicenter comparison of hexaminolevulinate fluorescence cystoscopy and white light cystoscopy for the detection of superficial papillary lesions in patients with bladder cancer. J UROL 2007;178:62-67. "

"The use of fluorescence (""blue light"") cystoscopy has been shown to: A. decrease utilization of intravesical therapies. B. decrease progression rates. C. decrease cystectomy rates. D. increase detection of CIS. E. increase tumor detection after BCG."

"E: ""cholestyramine."" was the correct answer. The etiology of diarrhea following urinary diversion is related to the type and amount of bowel used in the diversion. Many patients initially suffer from diarrhea after the procedure, but this usually resolves during the postoperative period. A small number of patients suffer chronic and bothersome diarrhea that necessitates treatment. In the case of diversions that require ileal resection of > 40 cm but < 100 cm, the diarrhea is thought to be due to decreased ileal bile salt absorption that leads to increased bile salt delivery to the colon. This in turn leads to colonic irritation and increased bicarbonate and water secretion. This is effectively a secretory diarrhea. The initial treatment should be decreased fat intake and initiation of cholestyramine. In contrast, diversions in which the ileocecal valve or colon is resected result in decreased bowel transit time and an osmotic diarrhea. This is best treated with oral bulking agents and loperamide. Vitamin B12 replacement is usually indicated in patients who have had significant portions of terminal ileum removed, but this will not impact the diarrhea. Fluid restriction is not advisable in the setting of a urinary diversion. Dahl DM, McDougal WS: Use of intestinal segments in urinary diversion, Wein, AJ, Kavoussi LR, Novick AC, Partin AW, Peters CA (eds): CAMPBELL-WALSH UROLOGY, ed 10. Philadelphia, Elsevier Saunders, 2012, vol 3, chap 85, pp 2446-2447. "

"Three years after radical cystectomy and orthotopic ileal neobladder, a 61-year-old man has chronic and severe diarrhea. The next step is: A. fluid restriction. B. Metamucil. C. loperamide. D. Vitamin B12 replacement. E. cholestyramine."

"B: ""two months."" was the correct answer. Regarding operative risks for major urologic procedures, epidemiologic studies have shown that postoperative morbidity is fourfold and mortality is up to tenfold higher in smokers compared to nonsmokers. To decrease the risk of pulmonary complications to the level of a non-smoker, the smoking must be discontinued for at least two months preoperatively. Vira MA, Steckel J: Core principles of perioperative care, Wein AJ, Kavoussi LR, Novick AC, Partin AW, Peters CA (eds): CAMPBELL-WALSH UROLOGY, ed 10. Philadelphia, Elsevier Saunders, 2012, vol 1, chap 6, p 161."

"To reduce post-operative pulmonary morbidity in smokers to the same level as non-smokers, patients should discontinue smoking for a minimum of: A. one month. B. two months. C. three months. D. six months. E. one year."

"C: ""IPSS and IIEF score."" was the correct answer. Daily tadalafil is FDA approved for the signs and symptoms of BPH and erectile dysfunction. Improvements may be seen with both erectile dysfunction and urinary symptom scores such as the International Prostate Symptom Score (IPSS). Use of once daily tadalafil (or other phosphodiesterase inhibitors) does not improve urinary flow rates. There is no evidence that a certain degree of baseline bother, as measured by the IPSS or International Index of Erectile Function (IIEF), are necessary to predict success with this therapy. In addition, there is evidence that suggests that combination therapy with phosphodiesterase inhibitors and alpha-blockers work together synergistically; however, this combination may also result in symptomatic hypotension. Phosphodiesterase inhibitors are not presently part of the AUA Guidelines for the Management of BPH. McNicholas TA, Kirby RS, Lepor H: Evaluation and nonsurgical management of benign prostatic hyperplasia, Wein, AJ, Kavoussi LR, Novick AC, Partin AW, Peters CA (eds): CAMPBELL-WALSH UROLOGY, ed 10. Philadelphia, Elsevier Saunders, 2012, vol 3, chap 92, p 2649. Gacci M, Corona G, Salvi M, et al: A systematic review and meta-analysis on the use of phosphodiesterase 5 inhibitors alone or in combination with alpha-blockers for lower urinary tract symptoms due to benign prostatic hyperplasia. EUR UROL 2012;61:994-1003. "

"Treatment with tadalafil 5 mg daily for the signs and symptoms of BPH and erectile dysfunction may result in improvements in: A. urinary flow rates and International Prostate Symptom Score (IPSS). B. urinary flow rates and International Index of Erectile Function (IIEF). C. IPSS and IIEF score. D. IPSS if baseline evaluation score is > 18. E. IIEF if baseline evaluation score is < 12."

"E: ""neurologic complications."" was the correct answer. The TOMUS trial compared the outcomes of retropubic sling to transobturator mid-urethral slings for treatment of stress urinary incontinence. In the original trial, success rates were statistically equivalent between the two groups. However, in long-term analysis (one versus five years), several differences were noted. Specifically, transobturator sling patients were more likely to manifest neurologic complications. It is believed these complications arose due to injury of small nerve branches from either the pudendal or obturator nerves occurring either at the time of trocar passage or alternatively due to long-term compression of the nerves from the obturator sling. Transobturator slings were also statistically less effective in preventing long-term urinary incontinence compared to retropubic urethral slings. Patients undergoing a retropubic urethral sling, however, had more bladder dysfunction, including higher postvoid residual urines, increased incidence of recurrent UTIs, and increased risk of delayed sling erosion into the bladder compared to patients undergoing a transobturator sling. Richter, H, Albo ME, Zyczynski HM, et al: Retropubic vs transobturator mid-urethral slings for stress urinary incontinence. NEJM 2010;362:2066-2076. "

"When compared to women undergoing a retropubic mid-urethral sling, women undergoing transobturator slings are more likely to experience: A. improved continence. B. UTIs. C. urinary retention. D. bladder perforations. E. neurologic complications."

"C: ""repair diaphragm laparoscopically, aspirate pleural cavity, and continue operation."" was the correct answer. Diaphragmatic injury is a rare complication of laparoscopic upper tract urinary surgery. It is important to note that this injury has occurred early in the operation in a stable patient. The injury is large enough that it must be repaired and placement of a chest tube to vent the pleural cavity alone would be inadequate treatment and could result in persistent loss of the pneumoperitoneum and/or inadequate pulmonary re-expansion. There is no need for open conversion; the injury can be repaired laparoscopically with aspiration of air from the pleural cavity and continuation of the operation. Placement of a chest tube is not routinely necessary following surgical repair. Packing the diaphragmatic injury or repairing the injury at the end of the operation risks continued filling of the pleural cavity with carbon dioxide leading to poor ventilation and high peak airway pressures. Kavoussi LR, Schwartz MJ, Gill IS: Laparoscopic surgery of the kidney, Wein, AJ, Kavoussi LR, Novick AC, Partin AW, Peters CA (eds): CAMPBELL-WALSH UROLOGY, ed 10. Philadelphia, Elsevier Saunders, 2012, vol 2, chap 55, p 1668. "

"While mobilizing the spleen during a left laparoscopic adrenalectomy, a 1.5 cm diaphragmatic injury is noted with entry into the pleural cavity. The patient remains hemodynamically stable. The next step is to: A. pack defect, decrease insufflation pressure, and continue operation. B. place chest tube in the second anterior intercostal space and continue operation. C. repair diaphragm laparoscopically, aspirate pleural cavity, and continue operation. D. continue procedure and repair diaphragm through specimen extraction site. E. convert to open surgery, repair diaphragm, and complete adrenalectomy."

"E: ""release pneumoperitoneum."" was the correct answer. During use of the argon beam electrocoagulator in a laparoscopic setting, it is important to ""vent"" from one of the trocar side arm ports during its use in order to avoid over-pressurizing the abdomen with the infused argon gas. If unrecognized, this can cause an abrupt increase in intra-abdominal pressure and eventual compromise of ventilation. All other listed maneuvers and ventilatory changes would not address the primary problem of increased intraperitoneal pressure. Eichel L, Clayman RV: Fundamentals of laparoscopic and robotic urologic surgery, Wein, AJ, Kavoussi LR, Novick AC, Partin AW, Peters CA (eds): CAMPBELL-WALSH UROLOGY, ed 10. Philadelphia, Elsevier Saunders, 2012, vol 1, chap 9, p 222. "

"While using an argon beam electrocoagulator during a laparoscopic partial nephrectomy, poor tidal volumes are noted. The next step is: A. obtain deeper sedation. B. increase ventilation rate. C. add positive end-expiratory pressure (PEEP). D. decrease CO2 insufflation flow rate. E. release pneumoperitoneum."

A Conservative measures, including observation, are reasonable in the properly-selected patient who has a small area of exposure of mesh in the vagina. If the patient is not symptomatic, she does not necessarily need to abstain from intercourse, though it would be advisable to counsel her on the possibility that intercourse could exacerbate mesh exposure. Addition of local hormone therapy would be very reasonable in a postmenopausal woman, however, in a premenopausal woman, her vaginal tissue is likely to be adequately estrogenized; supplementation would therefore be of little to no benefit. Vaginal excision of the exposed mesh could be considered if she or her partner are symptomatic, but does not need to be performed in an asymptomatic patient. Removal of the entire sling is not indicated in this asymptomatic patient. Dmochowski RR, Padmanabhan P, Scarpero HM: Slings: Autologous, biologic, synthetic, and midurethral, Wein AJ, Kavoussi LR, Novick AC, Partin AW, Peters CA (eds): CAMPBELL-WALSH UROLOGY, ed 10. Philadelphia, Elsevier Saunders, 2012, vol 3, chap 73, pp 2147-2148.

2014 - 1 A 32-year-old woman has a 0.5 cm vaginal mesh exposure one year following sling placement. There is no urinary tract involvement. Although she has occasional spotting, she is continent and neither she nor her sexual partner are bothered by the mesh. The next step is: A. observation. B. abstain from sexual intercourse until the mesh is epithelialized over. C. local hormone therapy. D. vaginal excision of exposed mesh. E. remove the entire sling and perform a urethral bulking injection.

D The patient is classified as moderate risk for DVT based on her age (> 40) and absence of additional risk factors, therefore prophylaxis is indicated. A pneumatic compression device would be recommended if the risk of intraoperative bleeding were high. Otherwise, heparin 5,000 units every 12 hours is recommended. Heparin 5,000 every eight hours dosing is recommended for those at high risk for DVT. Forrest JB, Clemens JQ, Finamore P, et al: Best practice policy statement for the prevention of deep vein thrombosis in patients undergoing urologic surgery. PREVENTION OF DVT AFTER UROLOGIC SURGERY BEST PRACTICE STATEMENT. American Urological Association Education and Research, Inc, 2008. http://www.auanet.org/education/guidelines/deep-vein-thrombosis.cfm

2014 - 10 A 47-year-old woman undergoes an abdominal sacrocolpopexy and a suburethral sling procedure. She is a nonsmoker and does not use estrogen replacement therapy. She is not obese. Recommended DVT prophylaxis is: A. early ambulation only. B. pneumatic compression device only. C. heparin 5000 units subcutaneous every eight hours starting after surgery. D. heparin 5000 units subcutaneous every 12 hours starting after surgery. E. heparin 5000 units subcutaneous every 24 hours starting after surgery.

B If voiding dysfunction persists after stroke, the most common findings include detrusor overactivity, intact sensation, and a synergistic sphincter. However, it is also important to note that non-neurogenic voiding dysfunction may occur as well, especially in this patient population which is often older. The urodynamic findings in this patient are consistent with bladder outlet obstruction so tamsulosin should be started. There is no urodynamic evidence of detrusor overactivity, so therapies directed at that such as mirabegron (beta-3 agonist), SNS, PTNS, and onabotulinumtoxinA would not be the next appropriate step. Wein AJ, Dmochowski RR: Neuromuscular dysfunction of the lower urinary tract, Wein AJ, Kavoussi LR, Novick AC, Partin AW, Peters CA (eds): CAMPBELL-WALSH UROLOGY, ed 10. Philadelphia, Elsevier Saunders, 2012, vol 3, chap 65, p 1913.

2014 - 100 A 66-year-old man with a history of a CVA five years ago complains of bothersome lower urinary tract symptoms. No improvement was noted after a trial of oxybutynin. A pressure flow urodynamic study reveals no evidence of detrusor overactivity and end fill detrusor pressure of 3 cm H2O at a maximum bladder capacity of 325 ml. He voids with a detrusor pressure of 95 cm after a trial of oxybutynin. A pressure flow urodynamic study reveals no evidence of detrusor overactivity and an end fill detrusor pressure of 3 cm H2O at a peak uroflow of 3 ml/sec and an average of 2 ml/sec. His total volume voided is 200 ml and his PVR is 125 ml. The next step is: A. mirabegron. B. tamsulosin. C. sacral nerve stimulation. D. posterior tibial nerve stimulation. E. onabotulinumtoxinA.

D Due to the high association of subsequent development of overt carcinoma, testicular intratubular germ cell neoplasia may be treated by orchiectomy, radiation therapy, or chemotherapy. Radiation therapy is effective and may maintain Leydig cell function. In cases of bilateral disease, bilateral orchiectomy would be a drastic recommendation. Given the potential toxicities of chemotherapy compared to radiation, this approach would be reserved for contralateral CIS in the setting of metastasis from the primary tumor. Prior to treatment, testicular sperm extraction should be considered as some men with maturation arrest on diagnostic biopsy will have sperm found with a more extensive biopsy. Stephenson AJ, Gilligan TD: Neoplasms of the testis, Wein AJ, Kavoussi LR, Novick AC, Partin AW, Peters CA (eds): CAMPBELL-WALSH UROLOGY, ed 10. Philadelphia, Elsevier Saunders, 2012, vol 1, chap 31, pp 850-851.

2014 - 101 A 33-year-old man with infertility has azoospermia. He has normal testes bilaterally and a serum FSH of 7 IU/l. Bilateral testis biopsies reveal maturation arrest and bilateral intratubular germ cell neoplasia. The next step is: A. serial ultrasound and tumor markers. B. repeat biopsy in six months. C. bilateral inguinal orchiectomy. D. XRT to both testes. E. systemic chemotherapy.

C The patient has the urologic finding of incontinence combined with fecal soiling and a foot abnormality. A tethered cord due to occult spinal dysraphism needs to be ruled-out as the cause of his incontinence. Tethered cord syndrome is a stretch-induced functional disorder of the spinal cord with the most caudal part of the cord anchored by inelastic structures. Specific treatment of his bladder instability (timed voids, medication, catheterization, treatment of constipation) is appropriate after a neurologic cause is excluded. Wein AJ, Dmochowski RR: Neuromuscular dysfunction of the lower urinary tract, Wein AJ, Kavoussi LR, Novick AC, Partin AW, Peters CA (eds): CAMPBELL-WALSH UROLOGY, ed 10. Philadelphia, Elsevier Saunders, 2012, vol 3, chap 65, p 1909.

2014 - 102 A five-year-old boy has day and night wetting, constipation, and fecal soiling. Physical examination is normal except for a high-arched right foot. Urodynamics show detrusor overactivity and normal sphincter function. The next step is: A. CIC. B. timed voiding schedule. C. spinal MRI scan. D. antimuscarinics. E. bowel program.

A This man likely has chronic bacterial prostatitis, which is usually manifested by recurrent UTIs with the same organism. While lower urinary tract bacterial localization is helpful in identifying the prostate as a nidus of infection, localization is useless in the presence of bacteruria, with all isolation specimens having bacterial growth due to contamination. This man is currently symptomatic with documented bacteruria and should not, therefore, undergo localization testing. In this situation, preferred treatment is to obtain a midstream urine culture and treat preferably with nitrofurantoin. Nitrofurantoin will clear the urine of bacteriuria and have little to no effect on intraprostatic bacteria. Once the urine has been documented to be sterile, usually in three to five days, bacterial localization studies with initial voided, mid-stream, expressed prostatic secretions and post-prostatic massage urinary specimens should be obtained. These tests will allow confirmation of chronic bacterial prostatitis. For chronic prostatitis caused by E. coli, four to six week treatment with the fluoroquinolones is superior to the alternative three month therapy with trimethoprim-sulfamethoxazole. Approximately 20% of the patients will fail the initial therapy and a rescue treatment with a second cycle of therapy with an alternative quinolone has been found to rescue the majority of the relapsing patients. Nickel JC: Prostatitis and Related Conditions, Orchitis, and Epididymitis, in Wein AJ, Kavoussi LR, Novick AC, Partin AW, Peters CA (eds): CAMPBELL'S UROLOGY, ed 10. Philadelphia, Saunders Elsevier, 2012, vol 1, chap 11, pp 333-338.

2014 - 103 A 45-year-old man has two days of perineal discomfort, dysuria, and urinary frequency. Urinalysis reveals bacteruria and pyuria. Physical examination reveals an enlarged prostate. He has a history of four symptomatic E. coli UTIs over the past year. He finished his last course of antibiotics one week ago. Before antibiotic treatment is restarted, which culture should be obtained: A. midstream urine. B. urine by suprapubic aspiration. C. expressed prostatic fluid. D. pre- and post-prostatic massage voided urine and prostatic fluid. E. initial voided urine.

B Careful selection of the optimal site for a stoma is a critically important part of the preoperative evaluation of the patient. The stomal site should avoid the umbilicus, bony prominences, scars, and folds. It should be placed in a region where the flat abdominal skin measures 5-7 cm in both directions. The stoma for an ileal conduit is usually located in the right lower quadrant. Paraplegics, often with abdominal prolapse and wheel-chair existence pose a special problem for stomal site selection. A right lower quadrant site often results in a stoma that is difficult for the patient to see, with the stoma and appliance squeezed between the abdomen and thigh. For this reason, in the paraplegic patient, the stoma should always, if possible, be placed in the upper abdomen. In addition, in all patients the stoma should be brought through the rectus muscle to minimize the risk of peristomal herniation. All patients should see an enterostomal therapist prior to stomal placement. Mattos RM, Libertino JA: Complications associated with urinary diversion, continent reservoirs, and neobladders, Libertino JA (ed): RECONSTRUCTIVE UROLOGIC SURGERY, ed 3. St. Louis, Mosby-Year Book Inc, 1998, chap 43, pp 423-432. Kanofsky JA, Godoy G, Taneja SS: Complications of conduit urinary diversion, Taneja SS (ed): COMPLICATIONS OF UROLOGIC SURGERY, ed 4. Philadelphia, Elsevier Saunders, 2010, chap 46, p 536.

2014 - 104 A 36-year-old obese paraplegic man undergoes evaluation for an ileal conduit diversion. The preferred site for urostomy placement is: A. right upper quadrant lateral to rectus. B. right upper quadrant through rectus. C. right lower quadrant through rectus. D. right lower quadrant lateral to rectus. E. at umbilicus.

E Bladder calculi develop in 18-30% of children following augmentation cystoplasty. Various factors may contribute to stone formation, e.g., bacteriuria, urinary stasis, hypocitraturia, mucus, etc. The most effective preventive measure for bladder stone formation is regular irrigation to clear the mucus that may serve as a nidus. No studies have demonstrated efficacy of bladder stone prevention using oral potassium citrate, prophylactic antibiotics, urinary acidification, and thiazide. Adams MC, Joseph DB: Urinary tract reconstruction in children, Wein AJ, Kavoussi LR, Novick AC, Partin AW, Peters CA (eds): CAMPBELL-WALSH UROLOGY, ed 10. Philadelphia, Elsevier Saunders, 2012, vol 4, chap 129, pp 3483-3484.

2014 - 105 The most effective intervention to prevent the formation of bladder calculi in children after augmentation cystoplasty is: A. oral potassium citrate. B. prophylactic antibiotics. C. urinary acidification. D. oral thiazide. E. regular bladder irrigation.

D When a metal trocar is placed directly through the skin, the charge surrounding an activated monopolar instrument is conducted back through the metal trocar and dispersed by the abdominal wall. However, placing a metal trocar within a plastic sheath may result in insulation of an electric field from the abdominal wall. The resultant buildup of the electrical field of an activated electrosurgical instrument can then conduct to the exposed metal trocar extending beyond the plastic sheath inside the abdomen resulting in direct conduction to nearby organs such as the bowel. This condition is called capacitance coupling and is one potential cause of electrosurgical injury to the bowel or other internal organs during laparoscopy. As the 8 mm metal trocar is placed within the 12 mm plastic trocar, this would not increase the risk of incisional hernia or trocar site discomfort. As the robotic instrument pivots within a trocar about a fixed remote center point, this avoids any excess inward-outward motion of the trocar thus reducing any risk of trocar dislodgement. Lastly, this unique hybrid trocar arrangement would not itself increase the risk of vascular injury. Eichel L, Clayman RV: Basics of laparoscopic urologic surgery, Wein AJ, Kavoussi LR, Novick AC, Partin AW, Peters CA (eds): CAMPBELL-WALSH UROLOGY, ed 10. Philadelphia, Elsevier Saunders, 2012, vol 1, chap 9, pp 242-244.

2014 - 106 During robotic cystectomy, the assistant places an extra-long 8 mm metal robotic trocar within a previously placed standard 12 mm plastic trocar to facilitate use of an additional robotic arm for dissection. The use of monopolar scissors within this hybrid trocar creates a higher risk of: A. trocar dislodgement. B. trocar site pain. C. incisional hernia. D. thermal injury to bowel. E. vascular injury.

A A number of biochemical tests are available to exclude or confirm the diagnosis of pheochromocytoma. Plasma-free metanephrine testing is the most sensitive test (99%). Because this conversion of catecholamines to metanephrines is an uninterrupted process within pheochromocytomas, the measurement of plasma concentration of metanephrines is a much more sensitive means of tumor detection than the measurement of rises in plasma catecholamines, which may be paroxysmal. Other clinical tests with lower sensitivities include urinary fractionated metanephrines, plasma catecholamines, urinary catecholamines, urinary total metanephrines, and urinary vanillymandelic acid. Kutikov A, Crispen PL, Uzzo RG: Pathophysiology, evaluation, and medical management of adrenal disorders, Wein AJ, Kavoussi LR, Novick AC, Partin AW, Peters CA (eds): CAMPBELL-WALSH UROLOGY, ed 10. Philadelphia, Elsevier Saunders, 2012, vol 2, chap 57, pp 1703-1711. Lenders JWM, Pacak K, Walther MM, et al: Biochemical diagnosis of pheochromocytoma: Which test is best? JAMA 2002;287:1427-1434.

2014 - 107 The most sensitive biochemical test for confirming the diagnosis of pheochromocytoma is: A. plasma free metanephrines. B. plasma catecholamines. C. urinary metanephrines. D. urinary vanillylmandelic acid. E. urinary catecholamines.

D Abiraterone irreversibly inhibits the products of the CYP17 gene including both 17,20-lyase and 17-alpha-hydroylase, resulting in a comprehensive block of androgen synthesis within the tumor, testes, and adrenal glands. As a result of inhibition of 17-alpha-hydroxylase, there is a decrease in cortisol synthesis with compensatory rise in ACTH, resulting in increased adrenal mineralocorticoid production. This can lead to hypertension, hypokalemia, and fluid retention if concomitant glucocorticoids (such as prednisone) are not administered with abiraterone to attenuate these effects. Therefore, glucocorticoid should be given concurrently with abiraterone. Clinical adrenal insufficiency does not generally occur, likely due to mineralocorticoid precursors which have some glucocorticoid activity. Spironolactone may temporarily address the hypokalemia and hypertension respectively, but are not treating the source of the problem. Fludrocortisone is a mineralocorticoid and would not be advisable as the patient is already suffering from mineralocorticoid excess. There is no need to discontinue abiraterone as the patient is responding to the therapy from a cancer perspective. Docetaxel chemotherapy would be reserved for failure of abiraterone therapy, which there are no signs of in this case. Kutikov A, Crispen PL and Uzzo RG: Pathophysiology, evaluation and medical management of adrenal disorders, Wein AJ, Kavoussi LR, Novick AC, Partin AW, Peters CA (eds): CAMPBELL-WALSH UROLOGY, ed 10. Philadelphia, Saunders Elsevier, 2012, vol 2, chapter 573, pp 1711-1714. Sonpavde G, Attard G, Bellmunt J, et al: The role of abiraterone acetate in the management of prostate cancer: A critical analysis of the literature. EUR UROL 2011;60:270-278.

2014 - 109 Three months after starting abiraterone for metastatic castrate-resistant prostate cancer, a 75-year-old man is responding to the therapy but develops worsening hypertension, hypokalemia, edema, and fatigue. The next step is: A. discontinue abiraterone. B. start spironolactone. C. start fludrocortisone. D. start prednisone. E. start docetaxel.

E If urethral perforation occurs during dilation, it is best to abandon the procedure, divert the urine with a urethral catheter, and return at a later date. If the contralateral cylinder has already been placed and there is no septal perforation, then it may be left in place. The urine should be diverted. Urethral repair would be difficult and is unnecessary. Should this occur with an inflatable prosthesis, the entire device should be removed and the urethra allowed to heal. Montague DK: Prosthetic surgery for erectile dysfunction, Wein AJ, Kavoussi LR, Novick AC, Partin AW, Peters CA (eds): CAMPBELL-WALSH UROLOGY, ed 10. Philadelphia, Elsevier Saunders, 2012, vol 1, chap 27, p 790.

2014 - 13 A distal urethral perforation occurs during insertion of a malleable penile prosthesis. The contralateral cylinder has not been placed. The next step is: A. place urethral catheter and complete the implantation. B. repair urethra and implant the contralateral prosthesis. C. repair urethra and place a suprapubic tube only. D. repair urethra, proceed with implantation, and place a suprapubic tube. E. place urethral catheter and terminate the procedure.

D The proper management is to divide the two arteries separately during kidney removal and perform ex vivo end-to-side anastomosis of the small artery to the larger one. Unlike in cadaveric kidney donation, use of an aortic patch is contraindicated in living renal donors due to potential risk to the donor. Ligation of the lower pole vessel may lead to lower pole ischemia and potentially a urinary fistula. Transplantation with two arterial anastomoses in the recipient will lengthen the revascularization time and increase the risk of ischemic renal damage. Use of a Dacron patch is rarely indicated unless the recipient has significant atherosclerosis. Barry JM, Conlin MJ: Renal transplantation, Wein AJ, Kavoussi LR, Novick AC, Partin AW, Peters CA (eds): CAMPBELL-WALSH UROLOGY, ed 10. Philadelphia, Elsevier Saunders, 2012, vol 2, chap 44, pp 1241-1242.

2014 - 11 A 25-year-old man is evaluated as a potential living renal donor to his sister. Arteriography shows a single artery to each kidney. A left donor nephrectomy is undertaken and at surgery, a second unsuspected 1.5 mm diameter artery to the lower pole is found 5 mm inferior to the main renal artery. The best management is: A. ligation of the polar artery. B. use of a donor aortic patch encompassing both arteries. C. use of a donor aortic patch encompassing the polar artery. D. ex vivo anastomosis of the polar to the main renal artery. E. ex vivo anastomosis of both arteries to a Dacron patch.

E Among patients with invasive bladder cancer, M-VAC chemotherapy will result in a complete pathologic response in 20% of patients prior to cystectomy. However, between 20-30% of patients with a clinical response will harbor residual invasive disease at the time of cystectomy. Accordingly, following chemotherapy this patient should undergo a cystoprostatectomy. There is no role for additional intravesical therapy in this patient. In addition, it would be inappropriate to place him on a surveillance protocol. HB Grossman, RB Natale, CM Tangen, et al: Neoadjuvant chemotherapy plus cystectomy compared with cystectomy alone for locally advanced bladder cancer. NEJM 2003;349:859-866. Lerner SP, Sternberg CN: Management of metastatic and invasive bladder cancer, Wein AJ, Kavoussi LR, Novick AC, Partin AW, Peters CA (eds): CAMPBELL-WALSH UROLOGY, ed 10. Philadelphia, Elsevier Saunders, 2012, vol 3, chap 82, p 2361.

2014 - 110 A 67-year-old man with a history of six weekly BCG treatments undergoes four cycles of M-VAC for a muscle invasive urothelial cell carcinoma (T3bNXMO) at the bladder neck. Following chemotherapy, there is no visible tumor although the urine cytology is positive. The next step is: A. cystoscopy in three months. B. repeat six weeks of BCG. C. maintenance BCG. D. XRT. E. radical cystoprostatectomy.

D The body has three primary mechanisms to handle physiologic and pathophysiologic acid loads including buffers within the blood, CO2 excretion in the lungs, and intracellular pH. Chronic acidosis is characterized by respiratory compensation of the underlying metabolic process. This leads to an increased respiratory clearance of CO2 and a decreased pCO2 over time. The pH is not as low as in the acute condition, and plasma bicarbonate is less depressed due to the compensation. Compensation includes HCO2 reabsorption within the proximal convoluted tubule and increased hydrogen ion secretion. The anion gap is often normal in the acidosis of chronic renal failure. Shoskes DA, McMahon AW: Renal physiology and pathophysiology, Wein AJ, Kavoussi LR, Novick AC, Partin AW, Peters CA (eds): CAMPBELL-WALSH UROLOGY, ed 10. Philadelphia, Elsevier Saunders, 2012, vol 2, chap 38, p 1042.

2014 - 111 The acidosis of chronic renal failure may be distinguished from acute acidosis by: A. increased anion gap. B. decreased plasma HCO2. C. increased pCO2 and HCO2. D. decreased pCO2. E. decreased pH.

D This patient has congenital penile curvature which is often noticed in the postpubescent time period when the man becomes sexually active. Patients with congenital curvature of the penis can have ventral, lateral (most often to the left), or unusually, dorsal curvature. Photographs of the erect penis demonstrate a smooth curvature of the penile shaft. Patients usually are healthy young men classically presenting between the ages of 18 and 30 years old. Many of these patients have noticed curvature before passing through puberty but have presumed it to be normal. Evaluation is sought by the patient only when the abnormal curvature is brought to their attention from their sexual partner. There is usually no association with penile trauma which may lead to curvature from acute or subacute penile fracture and can cause acquired penile curvature. Peyronie's disease generally occurs in older men and is associated with pain on erection and gradual change in the degree of curvature. Another name for congenital penile curvature is chordee without hypospadias. Surgical repair, when necessary, is usually performed using a plication technique. Jordan GH, McCammon KA: Surgery of the penis and urethra, Wein AJ, Kavoussi LR, Novick AC, Partin AW, Peters CA (eds): CAMPBELL-WALSH UROLOGY, ed 10. Philadelphia, Elsevier Saunders, 2012, vol 1, chap 36, p 992.

2014 - 112 A 19-year-old man complains that his partner has noticed significant left lateral deviation of his penis on erection. He denies any penile trauma and has not previously noticed the curvature. The most likely diagnosis is: A. Peyronie's disease. B. subacute penile fracture. C. acquired penile curvature. D. congenital penile curvature. E. hypospadias with chordee.

B Magnesium depletion occurs with diuretic use, diabetic ketoacidosis, alcoholism, and prolonged nasogastric suction. The latter is associated with hyperchloremic metabolic acidosis. For reasons that are not clear, this eventually results in hypocalcemia and hypomagnesemia. When hypokalemia is associated with hypomagnesemia, potassium replacement cannot be accomplished until magnesium is replenished. Hypomagnesemia is one of the chief causes of inability to replenish body stores of potassium associated with prolonged nasogastric suction. If not corrected, hypomagnesemia will lead to an alteration in mental status and seizures. Shoskes DA, McMahon AW: Renal physiology and pathophysiology, Wein AJ, Kavoussi LR, Novick AC, Partin AW, Peters CA (eds): CAMPBELL-WALSH UROLOGY, ed 10. Philadelphia, Elsevier Saunders, 2012, vol 2, chap 38, p 1025.

2014 - 114 A 56-year-old man has a prolonged ileus requiring nasogastric suction following radical cystectomy and ileal conduit. On the tenth postoperative day, he has a seizure. The most likely cause of the seizure is decreased serum: A. calcium. B. magnesium. C. potassium. D. phosphate. E. sodium.

C Several sodas are acidified by citric acid and contain an amount of citrate equal to or greater than that of lemonade, including Diet Sunkist Orange, Diet 7Up, Sprite Zero, Diet Canada Dry Ginger Ale, Sierra Mist Free, Diet Orange Crush, Fresca, and Diet Mountain Dew. All of the aforementioned sodas have the potential to decrease the risk of kidney stones similar to or greater than lemonade. In contrast, colas, including Caffeine Free Diet Coke, Diet Coke, Diet Coke with Lime, Coke Zero, Caffeine Free Diet Pepsi and Pepsi, are acidified by phosphoric acid, not by citric acid and contain low citrate levels. One randomized study of recurrent stone formers with baseline soda consumption > 160 ml per day, found that over a three-year period those who abstained from any soft drink consumption had a lower risk of symptomatic stone events (34%) compared to those who continued to drink sodas acidified by phosphoric acid (41%; RR, 0.83). Eisner BH, Asplin JR, Goldfarb DS, et al: Citrate, malate and alkali content in commonly consumed diet sodas: Implications for nephrolithiasis treatment. J UROL 2010;183:2419-2423. Shuster J, Jenkins A, Logan C, et al: Soft drink consumption and urinary stone recurrence: A randomized prevention trial. J CLIN EPID 1992;45:911-916.

2014 - 115 A recurrent stone former has low urinary citrate. The patient inquires which soda he can drink. He should be advised to drink: A. Caffeine Free Diet PepsiTM. B. Diet Coke with LimeTM. C. Sprite ZeroTM. D. Coke ZeroTM. E. no soda.

A Local recurrence after radical nephrectomy is rare with an incidence of 2.9%. In addition, it is associated with a poor prognosis. Patients with local recurrence treated nonsurgically have a low survival rate. In addition, the presence of synchronous metastasis is associated with a significantly lower survival rate. Best outcomes are reported in those patients treated with surgical resection for isolated local recurrence only. Other factors, including time to local recurrence, location in the ipsilateral adrenal or renal fossa, and size of local recurrence are not independently associated with a worse outcome. Overall, five year survival rates for patients with surgically resected isolated local recurrences are approximately 30%. Bruno JJ, Snyder ME, Motzer RJ, et al: Renal cell carcinoma local recurrences: Impact of surgical treatment and concomitant metastasis on survival. UROL ONCOL 2006;97:933-938. Campbell SC, Lane BR: Malignant renal tumors, Wein AJ, Kavoussi LR, Novick AC, Partin AW, Peters CA (eds): CAMPBELL-WALSH UROLOGY, ed 10. Philadelphia, Elsevier Saunders, 2012, vol 2, chap 49, p 1467.

2014 - 116 An independent factor associated with poor prognosis in patients with local recurrence of RCC following radical nephrectomy is: A. pulmonary metastasis. B. age > 65 years. C. ipsilateral adrenal location. D. time to local recurrence. E. size of local recurrence.

D The best choice for management is a tetanus immunization if he is not up to date, antibiotics, debridement, and primary closure with drainage. Skin grafts and placement of the testicles in the thigh are seldom required when half of the scrotal skin remains. Secondary closure for such a recent injury is unnecessary. If grafting is required, a meshed split thickness graft is preferable because the meshing allows exudate to escape and gives improved cosmesis. Thigh pouches are rarely required as wet to dry dressings of the exposed gonads can be effective in critically ill patients until reconstruction is feasible. Husmann DA: Pediatric genitourinary trauma, Wein AJ, Kavoussi LR, Novick AC, Partin AW, Peters CA (eds): CAMPBELL-WALSH UROLOGY, ed 10. Philadelphia, Elsevier Saunders, 2012, vol 4, chap 138, pp 3752-3753.

2014 - 117 A three-year-old boy lost one-half of his scrotal skin after a dog attack two hours ago. His testicles, penis, and urethra are spared. Best management includes antibiotics, debridement, and: A. split-thickness skin graft. B. full-thickness skin graft. C. placement of testicles in the thigh. D. scrotal closure with drainage. E. secondary scrotal closure.

A Recent studies on the outcome of radial nephrectomy for localized RCC have demonstrated that the risk of recurrence is stage dependent with the risk approximately 7%, 25%, and 40% for T1, T2, and T3 N0 M0 patients, respectively. The risk of recurrence is also greatest during the first three years. The AUA Guidelines for follow-up of clinically localized renal neoplasms has recently been published. Evidence based recommendations for follow-up should be tailored according to pathologic stage. This patient has stage T1aNo disease. For patients with stage pT1 disease, routine abdominal CT scans are not necessary, however, a yearly chest x-ray for three years is now recommended per AUA Guidelines. For stage pT2 and pT3 patients, it is recommended to obtain chest x-ray and CT scan every six months for three years, then annually to year five. Donat SM, Chang SS, Bishoff JT, et al: Follow-up for clinically localized renal neoplasms: AUA Guideline. FOLLOW-UP CARE FOR RENAL CANCER. American Urological Association Education and Research, Inc, 2013. http://www.auanet.org/education/guidelines/renal-cancer-follow-up.cfm

2014 - 119 A 49-year-old man had a hand-assisted laparoscopic radical nephrectomy for a 3.8 cm renal mass. Pathologic analysis reveals a grade 3 clear cell RCC confined to the kidney and three regional lymph nodes are negative. The most appropriate follow-up for this patient in addition to routine history and physical exam is: A. labs and chest x-ray yearly. B. labs, chest x-ray, and CT scan yearly. C. labs, chest x-ray, CT scan, and bone scan yearly. D. labs and chest x-ray every six months for three years, then yearly. E. labs, chest x-ray, and CT scan every six months for three years, then yearly.

C While this patient has depression, this is not specific for testosterone deficiency, and low testosterone should be demonstrated before initiating therapy. The patient has a normal total testosterone but a low free testosterone measured by an analog assay. The gold standard techniques for measurement of free testosterone are ultracentrifugation or equilibrium dialysis. Immunoassays, while convenient and widely available, utilize a testosterone analog and do not give accurate results. Measurement of bioavailable testosterone or free testosterone measured by ultracentrifugation or dialysis should be utilized. Calculated free testosterone which utilizes the total testosterone, sex hormone binding globulin with or without albumin levels, may also be used if the prior tests are not available. Determination of LH and prolactin may be indicated with low total testosterone levels to differentiate between primary and secondary hypogonadism. Since the total testosterone is normal, there is no role for LH or prolactin determination in this patient. While estradiol does increase with obesity, there is no indication for measurement of estradiol in this patient. Aromatase inhibitors are often used off label in patients with elevated estradiol levels and symptomatic gynecomastia. This is second line therapy and should not be initial therapy in most patients since long term safety has not been assessed. Long term use of aromatase inhibitors may result in elevation of liver function tests and affect bone health. Testosterone replacement therapy should not be instituted without demonstrating a low testosterone level. Morales A: Androgen deficiency in the aging male, Wein AJ, Kavoussi LR, Novick AC, Partin AW, Peters CA (eds): CAMPBELL-WALSH UROLOGY, ed 10. Philadelphia, Elsevier Saunders, 2012, vol 1, chap 29, pp 813-816.

2014 - 130 A 56-year-old obese man with a history of depression has a total testosterone of 450 ng/dl. Free testosterone level measured by an analog assay (immunoassay) is low. Physical exam is unremarkable. The next step is: A. measure estradiol. B. measure LH and prolactin. C. measure bioavailable testosterone. D. testosterone replacement therapy. E. aromatase inhibitor therapy.

B Conservative measures such as hydration, hygiene, and voiding after intercourse have been documented to be ineffective in preventing post-coital UTIs. The most likely cause for her recurrent infections is persistence of uropathogenic bacteria in the vaginal flora, which were ineffectively treated with the urinary concentrated antibiotic, nitrofurantoin. Treatment with three days of trimethoprim-sulfamethoxazole will eliminate vaginal colonization. Upper tract imaging and cystoscopy are indicated if recurrence of the same bacteria occurs rapidly despite treatment with an antibiotic that eradicates vaginal colonization. Localization cultures may be indicated if cystoscopy and upper tract imaging are unrevealing. Post-coital antibiotic prophylaxis is another treatment option. Nicolle LE: Uncomplicated urinary tract infection in adults including uncomplicated pyelonephritis. UROL CLIN N AM 2008;35:5. Schaeffer AJ, Schaeffer EM: Infections of the urinary tract, Wein AJ, Kavoussi LR, Novick AC, Partin AW, Peters CA (eds): CAMPBELL-WALSH UROLOGY, ed 10. Philadelphia, Elsevier Saunders, 2012, vol 1, chap 10, p 291.

2014 - 12 A 25-year-old woman experiences recurrent UTIs following sexual activity. Cultures with each episode reveal pan-sensitive E. coli. Each symptomatic episode has been treated for 14 days with nitrofurantoin. Five days after completing her most recent treatment, catheterized urine is sterile, PVR is negligible, and pelvic exam is normal. If she experiences another UTI, the next step is: A. retreat with nitrofurantoin and counsel the patient to drink more fluids, improve hygiene, and void after intercourse. B. treat with three days of trimethoprim-sulfamethoxazole. C. renal bladder ultrasound. D. renal bladder ultrasound and cystoscopy. E. renal bladder ultrasound, cystoscopy, and localization cultures.

E Bladder dysfunction in PUV is common. Three distinct forms of bladder dysfunction have been characterized including detrusor overactivity, decreased capacity/compliance, and myogenic failure. In infants, decreased capacity/compliance predominates which then transitions to detrusor overactivity as the child grows older. In adolescents, myogenic failure is the most common pattern seen which results in a large capacity bladder with decreased sensation and a variable degree of bladder wall compliance. Patients may have incontinence from poor emptying and overflow patterns. Management options include timed voiding, double voiding, alpha-blockers, and CIC. While sphincteric injury can occur, it is uncommon, as is the finding of an occult neurogenic bladder in children with PUV. Casale AJ: Posterior urethral valves, Wein AJ, Kavoussi LR, Novick AC, Partin AW, Peters CA (eds): CAMPBELL-WALSH UROLOGY, ed 10. Philadelphia, Elsevier Saunders, 2012, vol 4, chap 126, pp 3402-3403.

2014 - 120 A 14-year-old boy with incontinence had PUV resected as a neonate. The most likely etiology for his incontinence is: A. occult neurogenic bladder. B. sphincteric injury. C. detrusor overactivity. D. poor bladder compliance. E. myogenic failure.

A Due to routine bacterial colonization of struvite calculi, any stone fragments left behind following destruction or removal of struvite calculi places the patient at a high risk of both recurrent UTI and stone disease. Significant efforts should be made to render all patients with struvite stones, stone-free. In patients where this is not possible, Acetohydroxamic acid (AHA), a urease inhibitor may be of benefit. AHA has been shown in randomized controlled studies to decrease growth of residual struvite stone fragments, but does not decrease stone recurrence in patients who were made stone free at the time of their surgical procedure. Adverse reactions have occurred in up to 30% of the patients receiving AHA. Most of these reactions are mild in nature and do not require cessation of treatment. Specifically, about 25% of patients will complain of mild headaches and gastrointestinal symptoms within the first 72 hours of starting the treatment. These symptoms are mild, transitory, and should not result in interruption of treatment. It is noteworthy that numerous patients have noted the development of a nonpruritic macular skin rash of the face and upper extremities when concomitantly consuming alcohol. The rash commonly appears 30-45 minutes after ingestion of alcoholic beverages. It characteristically disappears spontaneously in 30-60 minutes. Patients should therefore be counseled to either avoid using alcohol or use smaller quantities of it. Three clinically significant side effects may result in either temporary cessation of treatment or complete withdrawal: 1) Superficial phlebitis involving the lower extremities has been seen in a number of patients on AHA, rarely will these patients advance to deep vein thrombosis. If this side effect should occur, the medication can be temporarily withdrawn and then restarted almost invariably without ill effects. 2) The most serious side effect of AHA is hemolytic anemia, found in up to 15% of the patients placed on this medication and is more prevalent in patients with renal insufficiency. AHA is a known inhibitor of DNA synthesis and chelates iron. Its bone marrow suppressant effect is probably related to its ability to inhibit DNA synthesis, but anemia could also be related to depletion of iron stores. Subsequently, a decrease in the hemoglobin and hematocrit may arise while on chronic use of this medication. Abnormalities in platelet or WBC have not been noted. Systemic symptoms may be associated with the anemia and are classically described as concurrent malaise, lethargy, and fatigue, and gastrointestinal symptoms (3% of patients with GI symptoms will have concurrent hemolytic anemia). Symptoms and laboratory findings have invariably improved following cessation of treatment. Due to the frequency of this side effect, monitoring patients with quarterly CBC analysis is recommended. 3) Severe depression and or anxiety will arise in 5% of patients. These symptoms are poorly treatable by pharmacologic means and when they develop, interruption or discontinuation of treatment is recommended. AHA is not associated with alterations in liver function, calcium, or magnesium metabolism. Johnson DB, Pearle M: Struvite Stones. Stoller ML, Meng MV (eds): URINARY STONE DISEASE: THE PRACTICAL GUIDE TO MEDICAL AND SURGICAL MANAGEMENT. Totowa NJ, Humana Press, 2007, chap 17, p 317.

2014 - 121 Patients treated with acetohydroxamic acid should be monitored with the following blood test: A. CBC. B. LFT. C. calcium level. D. magnesium level. E. ammonia level.

C The administration of a single perioperative dose of intravesical chemotherapy after complete TURBT for Ta-T1 TCC of the bladder significantly reduces the rate of tumor recurrence. This has also been summarized in a large meta-analysis in which patients receiving perioperative chemotherapy had a recurrence rate of 36.7%, compared to 48.4% with TUR alone. In all studies where benefit was noted, the instillation was given within 24 hours, generally either immediately or within six hours following the TURBT. Several randomized trials have demonstrated significant reductions in recurrence when the intravesical chemotherapy was administered within 24 hours as compared to starting it seven days later. Thus, there is evidence that the instillation should be given on the same day as the TURBT and not later. Oosterlinck W, Sylvester RJ: Perioperative instillation of chemotherapeutic drugs, Lerner SP, Schoenberg MP, Sterberg CN (eds): TEXTBOOK OF BLADDER CANCER. Abingdon, Oxon, Taylor and Francis, 2006, chap 29, pp 330-331.

2014 - 122 Instillation of perioperative intravesical chemotherapy after a complete TURBT for urothelial cell carcinoma of the bladder is effective only if given: A. immediately following the resection. B. within six hours of the resection. C. within 24 hours of the resection. D. within seven days of the resection. E. within two weeks of the resection.

B Preservation of the sacral nerve arcs, as indicated by an intact bulbocavernosus reflex, suggests the potential for detrusor-sphincter dyssynergia. While other parameters such as lower extremity movement, spontaneous voiding, and normal anal sphincter tone may also suggest the presence of intact sacral arcs, the bulbocavernosus reflex is the most clinically specific. Unlike spinal cord injury, in lumbosacral spina bifida neural function and urodynamic findings cannot be predicted by the level of the lesion. MacLellan DL, Bauer SB: Neuropathic dysfunction of the lower urinary tract, Wein AJ, Kavoussi LR, Novick AC, Partin AW, Peters CA (eds): CAMPBELL-WALSH UROLOGY, ed 10. Philadelphia, Elsevier Saunders, 2012, vol 4, chap 128, pp 3433-3438. Pontari MA, Keating M, Kelly M, et al: Retained sacral function in children with high level myelodysplasia. J UROL 1995;154:775-777.

2014 - 123 A three-month-old boy had a lumbar myelomeningocele closed at birth. The finding associated with the potential for detrusor-sphincter dyssynergia is: A. level of lesion. B. intact bulbocavernosus reflex. C. lower extremity movement. D. spontaneous voiding. E. decreased anal sphincter tone.

D The patient has suffered a significant burn to the penile shaft. Most genital burns are full thickness with significant tissue sloughing of the penis and urethra in the days following injury. Conservative management, including suprapubic tube placement in the bladder, should be maintained until the full extent of injury is known. Many times, prolonged bladder drainage is required in these patients due to the extent of their injury. Suprapubic cystostomy is preferred over urethral catheterization in order to avoid further urethral damage with catheterization, instrumentation, contrast studies, or endoscopy. Penile debridement is not indicated at this early stage of injury and retrograde urethrogram will not provide useful information four hours after injury. Medendorp AR, Albrecht MC, Morey AF: Natural history of full-thickness electrical burns involving the penis. J UROL 2007;70:588-589. Jordan GH: Lower genitourinary tract trauma and male external genital trauma (avulsion injuries, burn injuries, penile laceration with membranous urethral injuries) Part 3. AUA UPDATE SERIES 2000;19:12. Waguespack RL, Thompson IM, McManus WF, et al: Genital and perineal burns. AUA UPDATE SERIES 1995;14:30-35.

2014 - 124 A 52-year-old man sustains an electrical burn to the penis while repairing a high voltage power line. Four hours after the injury, the penile shaft and glans are erythematous with superficial skin sloughing and blistering. The next step is: A. observation. B. retrograde urethrogram. C. urethral catheter. D. suprapubic cystostomy. E. penile debridement.

E Currently available 5-alpha-reductase inhibitors (finasteride and dutasteride) lower serum PSA in patients with and without prostate cancer. The doubling rule has been used to compensate for PSA reductions in patients on these medications. With growing experience and data from long-term clinical trials using these drugs (e.g. PCPT and PLESS), the effect on PSA and prostate cancer detection has been more clearly defined. Although the doubling rule does accurately reflect the population as a whole, given the wide intra and interindividual variability of serum PSA changes while on these medications, it is relatively inaccurate in the individual patient. While treatment of patients with either finasteride or dutasteride results in a -50% median change in the group as a whole, the 5th to 95th percentile range is -81-20%. Recent analysis of long-term data regarding the change in PSA in men receiving 5-alpha-reductase inhibitors, suggests that in the first year the doubling rule overestimates PSA and can systematically lead to an increased likelihood of prostate biopsy. Beyond 24 months (with further decline in PSA), the doubling rule may underestimate PSA and result in fewer biopsies. As most patients receiving these drugs attain a nadir serum PSA, a sustained increase from nadir can be used to prompt biopsy the same way as it would in untreated patients. Using a 0.3 ng/ml increase from nadir as a trigger for biopsy maintains a sensitivity and specificity similar to an absolute value of 4.0 ng/ml in untreated patients. Free and total PSA ratio should remain unaffected by 5-alpha-reductase inhibitor use. In the absence of infection, there is no indication for use of antibiotics. PCA3 is only FDA-approved for patients with a prior history of a negative biopsy. Marks LS, Andriole GL, Fitzpatrick JM, et al: The interpretation of serum prostate specific antigen in men receiving 5-alpha-reductase inhibitors: A review and clinical recommendations. J UROL 2006;176:868-874.

2014 - 125 A 61-year-old man has an International Prostate Symptom Score (IPSS) score of 18, a 40 gm benign feeling prostate on DRE, and a PSA of 3.2 ng/ml. He was started on dutasteride and the LUTS improved markedly during the first year of treatment. PSA data on follow-up is: 12 months, 1.2 ng/ml; 18 months, 0.9ng/ml; 24 months, 1.4 ng/ml; and 36 months, 1.9 ng/ml. The next step is: A. PCA3. B. free and total PSA in six months. C. PSA in one year. D. quinolone antibiotic followed by repeat PSA in six weeks. E. TRUS and prostate biopsy.

C After brain tumors, neuroblastoma is the most common malignant solid tumor of childhood. More than half of these originate in the abdomen and a palpable abdominal mass is the most common finding. Periorbital metastatic disease is common, causing periorbital edema, proptosis, or raccoon-like ecchymoses. Tuberous sclerosis is associated with facial lesions of adenoma sebaceum in 75% of cases. These lesions are firm, discrete, red or brown telangiectatic papules (facial angiofibromata), and are typically located on the nasolabial folds, chin, and cheeks. Renal angiomyolipomas occur in 40-80% of patients with tuberous sclerosis. Wilms' tumor typically presents as an abdominal mass or with gross hematuria. Hodgkin's disease and leukemia often present with fever, night sweats, or fatigue. Neither Wilms' tumor, Hodgkin's disease, nor leukemia are associated with facial or periorbital lesions. Pope JC IV: Renal dysgenesis and cystic disease of the kidney, Wein AJ, Kavoussi LR, Novick AC, Partin AW, Peters CA (eds): CAMPBELL-WALSH UROLOGY, ed 10. Philadelphia, Elsevier Saunders, 2012, vol 4, chap 118, p 3177.

2014 - 126 A three-year-old girl has an abdominal mass and periorbital ecchymoses. These findings are most suggestive of: A. tuberous sclerosis. B. Wilms' tumor. C. neuroblastoma. D. Hodgkin's disease. E. leukemia.

B The patientÆs presentation is characteristic for scabies. Scabies infestation is associated with overcrowding and poor health conditions. Severe pruritis is the hallmark of scabies and involvement of the external genitalia is very common. The characteristic findings on exam are small erythematous papules and thin grey or white burrows. Treatment of choice for scabies is 5% permethrin cream applied to the entire body overnight with a second application one week later. Clotrimazole is useful for treating candida but the examination is not consistent with candidal infection. Herpes simplex virus does not produce pruritis and there is no need for biopsy. Link RE: Cutaneous diseases of the external genitalia, Wein AJ, Kavoussi LR, Novick AC, Partin AW, Peters CA (eds): CAMPBELL-WALSH UROLOGY, ed 10. Philadelphia, Elsevier Saunders, 2012, vol 1, chap 15, pp 456-457.

2014 - 127 A 58-year-old man experiences severe pruritus involving his genitals and buttocks several weeks after returning from a trip to Southeast Asia. Examination reveals small erythematous papules, excoriations, and small raised lines in the skin. The next step is: A. topical clotrimazole. B. permethrin. C. prednisone. D. serum herpes simplex virus titer. E. biopsy.

B Perioperative mitomycin C is effective in decreasing the chance of recurrence in short term (one to two years), however there is no effect on progression or long term recurrence rate, and no significant differences in complications has been reported when compared to other intravesical therapies. Hall MC, Chang SS, Dalbagni G, et al: Guideline for the management of nonmuscle invasive bladder cancer (stages Ta, T1, and Tis): 2007 update. BLADDER CANCER. American Urological Association Education and Research, Inc, 2007. http://www.auanet.org/education/guidelines/bladder-cancer.cfm

2014 - 128 A 62-year-old man undergoes TURBT for a solitary low-grade 1 cm papillary Ta bladder tumor. He receives a single perioperative dose of mitomycin C. This therapy will most likely decrease his risk of: A. postoperative side effects. B. short term recurrence (< 2 years). C. long term recurrence (> 2 years). D. progression. E. cancer-specific mortality.

B Mesoblastic nephromas are infiltrative tumors not amenable to partial nephrectomy. Rhabdoid tumors are highly malignant and similarly are best treated by radical nephrectomy. Lymphomas are optimally treated with systemic chemotherapy and neuroblastomas rarely involve the kidney. If the neuroblastoma involves the renal hilum, radical nephrectomy is often necessary. In the treatment of Stage V (bilateral) Wilms' tumor, renal preservation is important and hence partial nephrectomy is preferred when appropriate (mass amenable to partial nephrectomy) following chemotherapy. Ritchey ML, Shamberger RC: Pediatric urologic oncology, Wein AJ, Kavoussi LR, Novick AC, Partin AW, Peters CA (eds): CAMPBELL-WALSH UROLOGY, ed 10. Philadelphia, Elsevier Saunders, 2012, vol 4, chap 137, p 3721.

2014 - 129 In a three-year-old boy, a partial nephrectomy is the most appropriate treatment for: A. congenital mesoblastic nephroma. B. stage V Wilms' tumor. C. stage I rhabdoid tumor. D. stage III lymphoma. E. stage III neuroblastoma.

A Low volume azoospermic acidic semen suggests lack of seminal vesicle contribution to the semen. The differential diagnosis of this finding is: bilateral ejaculatory duct obstruction and congenital bilateral absence of the vas deferens (CBAVD). Transrectal ultrasound can differentiate between the two entities with CBAVD patients having either seminal vesicle agenesis or seminal vesicle hypoplasia. Bilateral ejaculatory ductal obstruction is associated with midline urethral cysts, bilateral seminal vesicle cysts, or the bilateral seminal vesical cysts with dilation of the ejaculatory ducts. CBAVD variants, as in this patient, one vas may be non-palpable while the other is present in the scrotum but absent in the pelvis. All CBAVD patients (including variants) should have genetic testing for cystic fibrosis. Up to 30% of patients will have no identifiable cystic fibrosis mutation. Some of these patients will have unilateral renal agenesis. It is thought that these patients have CBAVD due to mesonephric ductal-ureteral bud abnormalities. Up to 5% of these patients will be found to have renal agenesis, a finding not associated with CBAVD due to cystic fibrosis. It is therefore recommended that patients with CBAVD with a negative genetic test for cystic fibrosis, have a renal ultrasound performed. Scrotal exploration and vasography are not indicated in CBAVD patients, diagnosis is made by physical exam plus transrectal ultrasound for CBAVD variants. Scrotal ultrasound will not help with the diagnosis. The sweat test is not indicated in this patient with normal genetic testing and no clinical symptoms of cystic fibrosis. Testis biopsy to evaluate infertility is not indicated because the patient has CBAVD, normal FSH, and normal-sized testes. Treatment options for infertility in CBAVD patients is ICSI with sperm retrieval by TESE and implantation by IVF. Note in patients with CBAVD negative for cystic fibrosis, offspring may carry the trait and children with unilateral and bilateral renal agenesis have been reported. Jarow JP, Sigman M, Kolettis PN, et al: The optimal evaluation of the infertile male: AUA best practice statement revised, 2010. OPTIMAL EVALUATION OF THE INFERTILE MALE. American Urological Association Education and Research, Inc, 2010. http://www.auanet.org/education/guidelines/male-infertility-d.cfm Schwarzer JU, Schwarz M: Significance of CFTR gene mutations in patients with congenital aplasia of vas deferens with special regard to renal aplasia. ANDROLOGIA 2012;44:305-307. McCallum TJ, Milunsky JM, Munarriz R, et al: Unilateral renal agenesis associated with congenital bilateral absence of the vas defens: Phenotypic and genetic considerations. HUM REPROD 2001;16:282-299.

2014 - 20 A 32-year-old man with infertility has unilateral absence of the vas deferens and 28 ml testes. Semen analysis reveals a volume of 0.5 ml, azoospermia, and pH of 6.4. FSH is 4.9 IU/l. Transrectal ultrasound reveals ipsilateral seminal vesicle agenesis and contralateral seminal vesical hypoplasia. Genetic testing is normal. The next step is: A. renal ultrasonography. B. scrotal exploration with vasography. C. scrotal ultrasound. D. sweat test. E. testis biopsy.

D A randomized, placebo-controlled trial Selenium and Vitamin E Cancer Prevention Trial [SELECT] of 35,533 men randomly assigned to four groups (selenium, Vitamin E, selenium/Vitamin E, and placebo) was performed in a double-blind fashion. Baseline eligibility included age 50 years or older (African American men) or 55 years or older (all other men), a serum prostate-specific antigen level of 4 ng/ml or less, and a digital rectal examination not suspicious for prostate cancer. Patients received oral selenium (200 _g/d from L-selenomethionine) and matched Vitamin E placebo, Vitamin E (400 IU/d of all race-alpha-tocopheryl acetate) and matched selenium placebo, selenium/Vitamin E, or placebo/placebo for a planned follow-up of minimum of seven years and a maximum of 12 years. There were no differences in prostate cancer risk between the four groups. However, there were statistically nonsignificant increased risks of prostate cancer in the Vitamin E group (P=.06) and type 2 diabetes mellitus in the selenium group (relative risk, 1.07; 99% CI, 0.94-1.22; P=.16) but not in the selenium/Vitamin E group. The conclusion of the study was that selenium or Vitamin E, alone or in combination at the doses and formulations used, did not prevent prostate cancer in this population of relatively healthy men. Non-genitourinary cancers were unchanged. Lippman SM, Klein EA, Goodman PJ, et al: Effect of selenium and Vitamin E on risk of prostate cancer and other cancers: The selenium and Vitamin E cancer prevention trial (SELECT). JAMA 2009;301:39-51.

2014 - 131 A 62-year-old man takes selenium and Vitamin E. He should be informed that his risk of: A. high-grade PIN will decrease. B. prostate cancer will decrease. C. type 2 diabetes will decrease. D. prostate cancer is unchanged. E. non-genitourinary cancers are increased.

B Combination chemotherapy with or without extirpative surgery has improved the prognosis for patients with rhabdomyosarcoma. The prostate has the worst prognosis as pertains to organ of origin, and radical surgery may be required if there is lack of response to non-operative treatment. The best prognosis in rhabdomyosarcoma has been associated with vaginal origin and embryonal histology. Ritchey ML, Shamberger RC: Pediatric urologic oncology, Wein AJ, Kavoussi LR, Novick AC, Partin AW, Peters CA (eds): CAMPBELL-WALSH UROLOGY, ed 10. Philadelphia, Elsevier Saunders, 2012, vol 4, chap 137, p 3704.

2014 - 132 The site of origin associated with the worst prognosis in pediatric rhabdomyosarcoma is: A. uterus. B. prostate. C. vagina. D. spermatic cord. E. bladder.

D The need for an imaging study to confirm stone/fragment clearance after ureteroscopy with lithotripsy, although controversial, is widely accepted. These imaging studies are performed to document: 1) clearance of the stone/fragments. 2) resolution of preoperative obstructive hydronephrosis, and/or, 3) rule-out the development of obstruction from a ureteral stricture. The need for these follow-up studies in an asymptomatic patient is, however, subject to debate. At the foundation of this controversy is the low incidence of obstruction following ureteroscopic stone extraction, the ability of the patientÆs symptoms to predict ureteral obstruction, the unknown time of when a ureteral stricture occurs post-instrumentation, the renal salvage rate following relief of the obstruction, and the cost of a lost kidney versus the need to the need to contain medical costs. Obstruction after ureteroscopic stone extraction has a reported incidence of 1-4%, with the majority of obstructions due to a persistent stone fragment, a ureteral stricture is found to be the cause of the obstruction < 30% of the time. It is noteworthy, that when obstruction is present the vast majority of patients are symptomatic with the incidence of asymptomatic obstruction reported to range from 0.25-2% of patients undergoing ureteroscopic stone extraction. Stated another way, if obstruction develops post-ureteroscopic stone extraction, the majority of patients will be symptomatic. Indeed, a number of investigators have reported on the safety of selective imaging of only symptomatic patients, as opposed to the use of routine imaging after ureteroscopy. These authors center their recommendation for selective renal imaging based on the facts outlined above and believe that routine imaging to detect the rare case of silent obstruction is not cost-effective. The authors of the AUA Guidelines for imaging in patients with ureteral calculus acknowledge this controversy and note that the Level of Evidence for their recommendation for routine post-operative imaging is low (Grade C). In patients with a preoperative radiopaque calculi who have undergone ureteroscopy and lithotripsy, the Guideline Panel believes that the combination of a KUB and sonogram provides the best means to identify the patient who will require either additional imaging or interventional treatment. (Remember, retained stone fragments are the most common etiology overall for obstruction and may be missed byultrasound alone.) If the patient is asymptomatic and KUB/sonogram show no stones or hydronephrosis, no further imaging is required. If KUB/sonogram demonstrates hydronephrosis and/or residual fragments, further observation with repeat imaging or treatment is indicated. Patients with radiolucent stones or individuals who have undergone ureteroscopy and removal of stone without lithotripsy should undergo a renal ultrasound alone. CT scan is utilized for evaluation if the patient is symptomatic or if hydronephrosis is found on the screening ultrasound. It should be noted that the timing of the follow-up imaging studies is left to the discretion of the treating physician and generally ranges from one to three months post-ureteroscopy. The lack of a specific recommendation regarding a time interval on when to obtain the post-ureteroscopic image is based on inconsistent and limited data regarding rapidity of ureteral stricture development after ureteroscopic stone extraction. Whether the obstruction is incomplete or complete, the impact of pyelolymphatic and pyelovenous back flow on renal preservation and the unknown length of time obstruction needs to be present to cause irreversible renal loss. PF Fulgham, DG Assimos, MS Pearle, et al: Clinical effectiveness protocols for imaging in the management of ureteral calculous disease: AUA technology assessment. IMAGING FOR URETERAL CALCULOUS DIESEASE. American Urological Association Education and Research, Inc, 2012. http://www.auanet.org/education/imaging-for-ureteral-calculous-disease.cfm Adiyat KT, Meuleners R, Monga M: Selective postoperative imaging after ureteroscopy. J UROL 2009;73:490-493. Sutherland, TN, Pearle, MS. Lotan, Y: How much is a kidney worth? Cost effectiveness of routine imaging after ureteroscopy to prevent silent obstruction. J UROL 2013;189:2136-2141.

2014 - 133 A 23-year-old woman undergoes ureteroscopy with holmium laser lithotripsy for a 5 mm distal radiopaque ureteral calculus. Endoscopically, there was no residual stone. She is now asymptomatic. The next step is: A. observation. B. KUB. C. ultrasonography. D. KUB and ultrasonography. E. non-contrast CT scan.

C This lesion likely represents an adenomatoid tumor, however, one cannot rule-out sarcomas. Adenomatoid tumors are the most common tumor of the paratesticular tissues, accounting for approximately 30% of all paratesticular tumors. There has never been a documented case of metastasis. Most occur in individuals in their twenties or thirties. Tumors present as small, solid, asymptomatic masses generally found on routine examination, ranging from 0.5-4 cm in size. They are typically located in the anatomic distribution of the epididymis, testicular tunicae, and rarely, the spermatic cord. Most tumors will arise in or adjacent to the upper or lower pole of the epididymis, with a slightly higher incidence in the lower pole. The definitive treatment is surgical excision through an inguinal incision. Transscrotal incision is contraindicated because malignancy has not been ruled out. Fertility status, and/or semen analysis, and/or fertility preservation should be discussed with the patient prior to the procedure. Stephenson AJ, Gilligan TD: Neoplasms of the testis, Wein AJ, Kavoussi LR, Novick AC, Partin AW, Peters CA (eds): CAMPBELL-WALSH UROLOGY, ed 10. Philadelphia, Elsevier Saunders, 2012, vol 1, chap 31, p 869.

2014 - 134 During a routine sports physical, a 25-year-old man has a 2 cm, well-circumscribed, solid mass discovered in the lower pole of the left epididymis confirmed by ultrasound. The testis is palpably normal. The next step is: A. CT scan of the chest, abdomen, and pelvis. B. transscrotal excision of the mass. C. inguinal excision of the mass. D. left epididymectomy. E. left radical orchiectomy.

D Exteriorization of the dome of the bladder by the Blocksom technique will prevent prolapse of the bladder. Placement of the vesicostomy cephalad to the urachus immobilizes the peritonealized portion of the bladder dome. None of the other techniques have been shown to adequately immobilize the bladder and prevent the posterior wall of the bladder from extruding through the stoma. Caldamone AA, Woodard JR: Prune-belly syndrome, Wein AJ, Kavoussi LR, Novick AC, Partin AW, Peters CA (eds): CAMPBELL-WALSH UROLOGY, ed 10. Philadelphia, Elsevier Saunders, 2012, vol 4, chap 123, p 3318. Casale AJ: Posterior urethral valves, Wein AJ, Kavoussi LR, Novick AC, Partin AW, Peters CA (eds): CAMPBELL-WALSH UROLOGY, ed 10. Philadelphia, Elsevier Saunders, 2012, vol 4, chap 126, pp 3398-3399.

2014 - 135 A newborn boy with prune belly syndrome has urinary retention and progressive hydroureteronephrosis. The best method to reduce the risk of postoperative bladder wall prolapse after cutaneous vesicostomy is to: A. perform reduction cystoplasty. B. create an 8 Fr stoma. C. place the stoma in the umbilicus. D. exteriorize the bladder dome. E. suture bladder muscle to the fascia.

A The presentation is very strongly consistent with either torsion of an appendix or noninfectious epididymitis. Obtaining a VCUG or uroflow-EMG are unlikely to add any useful information. Neither STD treatment, nor antibiotics should be started without the findings of either a urethral discharge, positive urinalysis, and/or admission of sexual activity. The most appropriate treatment includes NSAIDs as well as rest and scrotal support. Shortliffe LMD: Infection and inflammation of the pediatric genitourinary tract, Wein AJ, Kavoussi LR, Novick AC, Partin AW, Peters CA (eds): CAMPBELL-WALSH UROLOGY, ed 10. Philadelphia, Elsevier Saunders, 2012, vol 4, chap 116, pp 3117-3118.

2014 - 136 A 13-year-old boy with dysuria, left scrotal pain, and swelling has a tender indurated epididymis and normal testes. Urinalysis is normal. An ultrasound shows normal testes with blood flow and an enlarged hypervascular left epididymis. The next step is: A. NSAIDS. B. ciprofloxacin. C. ceftriaxone and doxycycline. D. uroflow-EMG. E. VCUG.

D Several series have demonstrated that administration of hormonal deprivation therapy prior to radical prostatectomy does not lower the risk of biochemical relapse. Although tumor volume, stage, nodal status, and margin status appear improved in men receiving neoadjuvant hormonal ablation, the risk of biochemical relapse is equivalent to those men who do not receive any neoadjuvant therapy. Gleason scoring can be quite difficult in men who have received hormonal deprivation due to the noted treatment effect in the tissues. Among these men, Gleason score can appear artificially higher and it is recommended that Gleason score not be assigned. Epstein JI: Pathology of prostatic neoplasia, Wein AJ, Kavoussi LR, Novick AC, Partin AW, Peters CA (eds): CAMPBELL-WALSH UROLOGY, ed 10. Philadelphia, Elsevier Saunders, 2012, vol 3, chap 96, p 2732.

2014 - 137 A 67-year-old man underwent radical prostatectomy with pelvic lymph node dissection following three months of LH-RH agonist therapy. The pathologist will not be able to accurately describe: A. tumor volume. B. tumor stage. C. nodal status. D. Gleason score. E. surgical margins.

D Most renal cystic conditions whether congenital, acquired, or sporadic arise from the nephrons and collecting ducts after they have formed. Multicystic dysplastic kidney is an exception in that it arises prior to formation of the nephron. It results from the abnormal differentiation of the metanephric parenchyma during early embryologic development of the kidney. Autosomal recessive polycystic kidney disease occurs during the formation of the nephron and results from mutations in the PKHD1 gene that encodes for a membrane associated receptor like protein known as fibrocystin. Abnormalities in fibrocystin secretion result in dilation of the epithelial line tubules predominantly the renal collecting ducts and the bile ducts. Abnormal fibrocystin secretion, in addition to causing abnormal ductal development, stimulates fibrous connective tissue development resulting in congenital hepatic fibrosis. Autosomal dominant polycystic kidney disease is due to either a defect in the PKD-1 or PKD-2 gene, the former being more common. These genes encode for either polycystin-1 or polycystin-2 respectively. The polycystins function within the calcium channels and disruption of their activity will lead to cyst formation and fibrotic renal stroma and usually begin to become manifest in the second to third decade of life. Medullary sponge kidney is characterized by tubular dilation of the collecting ducts with associated cysts and diverticula. Juvenile nephronophthisis, although rare, is the most common genetic cause of childhood kidney failure and is characterized by fibrosis and cystic dysplasia of renal tubules. Pope JC IV: Renal dysgenesis and cystic disease of the kidney, Wein AJ, Kavoussi LR, Novick AC, Partin AW, Peters CA (eds): CAMPBELL-WALSH UROLOGY, ed 10. Philadelphia, Elsevier Saunders, 2012, vol 4, chap 118, p 3182.

2014 - 138 The renal cystic condition that arises prior to formation of the nephron is: A. autosomal recessive polycystic kidney disease. B. autosomal dominant polycystic kidney disease. C. medullary sponge kidney. D. multicystic dysplastic kidney. E. juvenile nephronophthisis.

D In this patient, the best long-term treatment option is urethroplasty with buccal mucosa. It is felt that the use of extragenital tissue, namely buccal mucosa, reduces the risk of lichen sclerosis (LS) induced stricture recurrence as there is a theoretical field change effect of LS involvement of the entire genital skin. Meatotomy is an option, yet this may create a distal hypospadias appearance for this young man along with potential voiding issues if the meatus is cut too proximal on the glans. Meatal dilation is not effective in this setting and the outcomes of CO2 laser use for LS induced urethral meatal stricture disease is unknown. Jordan GH, McCammon KA: Surgery of the penis and urethra, Wein AJ, Kavoussi LR, Novick AC, Partin AW, Peters CA (eds): CAMPBELL-WALSH UROLOGY, ed 10. Philadelphia, Elsevier Saunders, 2012, vol 1, chap 36, p 962.

2014 - 139 A 24-year-old man has a split urinary stream with significant bother. Examination reveals a circumcised penis with a grayish white plaque surrounding a constricted urethral meatus. Retrograde urethrogram reveals a 1.5 cm meatal stricture and the rest of the urethra is unremarkable. Biopsy of the meatus confirms lichen sclerosis. The next step is: A. meatal dilation. B. CO2 laser ablation and urethrotomy. C. meatotomy. D. distal urethroplasty with buccal mucosa. E. distal urethroplasty with penile skin flap.

A Simple measures such as catheter irrigations and placement of a larger diameter suprapubic tube may temporize but not completely address the underlying problem of recurrent catheter encrustation with sediment. Catheter encrustation is attributed to bacterial biofilm, particularly biofilms made by urease producing bacteria such as proteus mirabilis. Urine culture is the appropriate next step. If a urease producing organism is identified, both treatment of the offending organism and evaluation for the presence of bladder or upper tract stones that could be serving as a nidus for bacterial infection is necessary. Wyndaele JJ: The encrustation and blockage of longterm indwelling catheters. SPINAL CORD 2010;48:783.

2014 - 14 A 27-year-old man with a C5 spinal cord injury has recurrent problems with sediment and clogging of his indwelling urethral catheter despite frequent catheter changes. The next step is: A. urine culture to identify urease producing organism. B. daily acetic acid irrigation. C. placement of a large lumen suprapubic tube. D. non-contrast CT scan. E. cystoscopy.

C In decreasing order of frequency, the most common sites of origin of extragonadal germ cell tumors are the mediastinum, retroperitoneum, sacrococcygeal region, and pineal gland, although many unusual sites have also been reported. Stephenson AJ, Gilligan TD: Neoplasms of the testis, Wein AJ, Kavoussi LR, Novick AC, Partin AW, Peters CA (eds): CAMPBELL-WALSH UROLOGY, ed 10. Philadelphia, Elsevier Saunders, 2012, vol 1, chap 31, p 845.

2014 - 140 The most common site of origin of extragonadal germ cell tumors is the: A. lung. B. pelvis. C. mediastinum. D. pineal gland. E. retroperitoneum.

E Hematuria after prolonged strenuous exercise is relatively common in individuals who are otherwise free of congenital urinary tract disease. There may be mild proteinuria with the RBCs. Erythrocyte casts are usually not present and the supernatant urine specimen is clear, ruling out significant hemoglobinuria. Hypercalciuria is not related to exercise, and therefore, the urinary calcium to creatinine ratio will be normal. The urinalysis in this benign condition is generally normal 48-72 hours after the exercise. Myoglobinuria is characteristically brown without RBCs. Cystoscopy and CT urogram will be normal. Polito C, Andreoli S: Sport hematuria in boys: A provocative test. PED NEPHR 2005;20:1171-1173. Holmes FC, Hunt JJ, Sevier TL: Renal injury in sport. CURR SPORTS MED REP 2003;2:103-109.

2014 - 141 A 17-year-old boy has bright red urine shortly after prolonged exercise. He has no history of genitourinary disease and is otherwise asymptomatic. Urinalysis reveals 1+ proteinuria and > 50 RBC/hpf. There are no casts. Renal and bladder ultrasound is normal. The next step is: A. cystoscopy. B. CT urogram. C. urinary myoglobin. D. urinary calcium to creatinine ratio. E. urinalysis 72 hours later.

E The incidence of Clostridium difficile infection (CDI) is increasing with a preponderance of the NAP1 hypervirulent strain of C. difficile found in recent epidemics. It is believed the NAP1 strain arose due to the widespread use of fluoroquinolone antibiotics. The NAP1 strain is more likely to cause severe and fulminant colitis, characterized by marked leukocytosis, renal failure, hemodynamic instability, and toxic megacolon. While oral vancomycin and oral metronidazole are both considered standard therapies for CDI, recent studies suggest that vancomycin is more effective. Neither I.V. vancomycin nor I.V. metronidazole have been found to be more effective than the oral form of the medications for treatment of CDI. There is no role for fluoroquinolone antibiotics in treatment of CDI. This patient with development of worsening diarrhea, fever, and leukocytosis, demonstrates failure to improve following implementation of standard treatment and is an absolute indication for surgical consultation. Subtotal colectomy with end ileostomy is the procedure of choice for fulminant CD colitis nonresponsive to medications and has been documented to result in improved survival. Pant C, Sferra TJ, Deshpande A, et al: Clinical approach to severe Clostridium difficile infection: Update for the hospital practitioner. EUR J INT MED 2011;22:561-568. Moudgal V, Sobel JD: Clostridium difficile colitis: A review. HOSP PRACT 2012;40:139-148. Nassour I, Carchman EH, Simmons RL, et al: Novel management strategies in the treatment of severe Clostridium difficile infection. ADV SUR 2012;46:111-135.

2014 - 142 A 72-year-old woman with pseudomembranous colitis is treated with oral vancomycin. After 48 hours, her diarrhea worsens and she develops fever and leukocytosis. The next step is: A. oral metronidazole. B. I.V. vancomycin. C. I.V. metronidazole. D. I.V. fluoroquinolone. E. surgical consultation.

E Patients with high-grade T2 cancer are at a significant risk of occult regional disease or of development of this disease. Studies have shown that patients who have an immediate bilateral inguinal lymphadenectomy have an increased survival than if they develop palpable disease during follow-up. Although this patient currently has no evidence of adenopathy, he has a 59% risk of developing involved lymph nodes. Sentinel lymph node biopsy is gaining popularity, but still lacks sufficient sensitivity and specificity. Pettaway CA, Lance RS, Davis JW: Tumors of the penis, Wein, AJ, Kavoussi LR, Novick AC, Partin AW, Peters CA (eds): CAMPBELL-WALSH UROLOGY, ed 10. Philadelphia, Elsevier Saunders, 2012, vol 1, chap 34, p 916.

2014 - 143 A 65-year-old man has a partial penectomy for a high-grade pT2 lesion. He has no palpable adenopathy in the inguinal region. Metastatic workup is negative. The next step in management is: A. serial physical examination of the inguinal nodes. B. serial imaging with CT scan. C. sentinel inguinal node biopsy. D. delayed inguinal lymphadenectomy if nodes become palpable. E. immediate bilateral inguinal lymphadenectomy.

C Radionuclide imaging using cortical scanning agents (DMSA, as opposed to MAG-3 which is filtered and excreted) will demonstrate an area of activity when the suspicious lesion is cortical hyperplasia (column of Bertin). MRI scanning will not differentiate this from other solid lesions. Although arteriography and CT scanning will both help in differentiating malignancy from pseudotumor, they are more invasive and expensive than renal scan and also have higher radiation exposure. They should be utilized only if the radionuclide study does not provide a definitive diagnosis. Shapiro E, Bauer SB, Chow JS: Anomalies of the upper urinary tract, Wein AJ, Kavoussi LR, Novick AC, Partin AW, Peters CA (eds): CAMPBELL-WALSH UROLOGY, ed 10. Philadelphia, Elsevier Saunders, 2012, vol 4, chap 117, p 3158.

2014 - 144 The diagnosis of a renal pseudotumor (column of Bertin, focal cortical hyperplasia) is best established by: A. MRI scan. B. MAG-3 scan. C. DMSA scan. D. arteriogram. E. CT scan.

E Candida species are capable of causing severe infection and should be treated as a uropathogen. Suspected UTI should be treated prior to ureteroscopic stone treatment with surgery performed only after culture documented clearance of funguria. Neither irrigation nor intravenous administration of anti-fungals just prior to surgery is sufficient to reduce the risk of postoperative candidemia. Schaeffer AJ, Schaeffer EM: Infections of the urinary tract, Wein AJ, Kavoussi LR, Novick AC, Partin AW, Peters CA (eds): CAMPBELL-WALSH UROLOGY, ed 10. Philadelphia, Elsevier Saunders, 2012, vol 1, chap 10 p 298. Gautam G, Singh AK, Kumar R, et al: Beware! Fungal urosepsis may follow endoscopic intervention for prolonged indwelling ureteral stent. J ENDO 2006;20:522-524. Preminger GM, Tiselius HG, Assimos DG, et al: 2007 Guideline for the management of ureteral calculi. URETERAL CALCULI, American Urological Association Education and Research, Inc, 2007. http://www.auanet.org/education/guidelines/ureteral-calculi.cfm

2014 - 145 A 55-year-old diabetic woman stented three months earlier for an obstructing ureteral calculus is scheduled for ureteroscopy and laser lithotripsy. A preoperative urinalysis reveals yeast that is confirmed with a catheterized specimen. The next step is: A. administer preoperative amphotericin I.V. and proceed with planned surgery. B. administer preoperative fluconazole I.V. and proceed with planned surgery. C. administer preoperative caspofungin I.V. and proceed with planned surgery. D. instill amphotericin solution into the bladder 30 minutes preoperatively and proceed with planned surgery. E. reschedule surgery, proceed with surgery only after culture-documented clearance of funguria.

C If this 6 cm tumor were contained within GerotaÆs fascia, the pathologic (p)T stage would be pT1b. However, there is renal vein involvement that is now classified as pT3a (previously pT3b). Tumors with isolated renal vein thrombus are known to have a relatively favorable prognosis, and as a result are now staged as T3a rather than T3b. The margin status is designated as R1, but does not affect primary pathologic tumor stage. Edge SB, Byrd D, Compton C, Fritz A: AJCC Staging Manual, ed 7. New York, Springer-Verlag, 2010, chap 43, pp 479-490.

2014 - 146 A 44-year-old woman undergoes a left radical nephrectomy for a 6 cm RCC with renal vein involvement. The posterior surgical margin is positive. Pathologic tumor stage is: A. T1b. B. T2b. C. T3a. D. T3b. E. T4.

C Three percent of girls with inguinal hernias have complete androgen insensitivity syndrome. A testicle is found in the hernia sac and a chromosome analysis will reveal a 46 XY karyotype. They should be raised as females. 45 X0/46 XY chromosomal patterns are accompanied by a streak gonad most commonly. Elevated 17-hydroxyprogesterone is seen with CAH, but in this case the gonads would be ovaries. Patients with 5-alpha-reductase deficiency present with either proximal hypospadias or more ambiguous genitalia. Low testosterone with elevated LH and FSH occurs with abnormal testosterone production. Diamond DA, Yu RN: Sexual differentiation: Normal and abnormal, Wein AJ, Kavoussi LR, Novick AC, Partin AW, Peters CA (eds): CAMPBELL-WALSH UROLOGY, ed 10. Philadelphia, Elsevier Saunders, 2012, vol 4, chap 133, pp 3621-3622.

2014 - 147 A phenotypically normal three-year-old girl has testicles found in the hernia sacs at herniorrhaphy. She will also have: A. elevated 17-hydroxyprogesterone. B. absent 5-alpha-reductase. C. abnormal androgen receptor activity. D. low testosterone, and elevated LH and FSH. E. 45 X0/46 XY karyotype.

E This man presents with symptoms of pyelonephritis. While uncomplicated cystitis in a young sexually active male may not require investigation beyond a follow-up urine culture, a complicated UTI in an older male warrants urologic evaluation such as CT urogram and cystoscopy due to the high incidence of associated urologic abnormalities such as obstruction from either urethral or ureteral strictures, tumor, or stones. Observation is incorrect. Localization cultures might be considered as part of a urologic evaluation but are not sufficient as isolated tests in the presence of a complicated febrile UTI. There is no justification for prophylaxis or complete urodynamic studies. Schaeffer AJ, Schaeffer EM: Infections of the urinary tract, Wein AJ, Kavoussi LR, Novick AC, Partin AW, Peters CA (eds): CAMPBELL-WALSH UROLOGY, ed 10. Philadelphia, Elsevier Saunders, 2012, vol 1, chap 10, pp 272-273.

2014 - 148 A 65-year-old man has symptoms of cystitis, left flank pain, and a fever to 39_ C. Urinalysis reveals pyuria and culture shows a pan-sensitive E. coli. One month after appropriate antibiotic therapy, he is asymptomatic and repeat urinalysis and midstream culture are negative. PVR is 45 ml. The next step is: A. observation. B. complete urodynamic studies. C. prostatic localization cultures. D. trimethoprim sulfamethoxazole prophylaxis. E. CT urogram and cystoscopy.

E Progression while on docetaxel-chemotherapy defines failure of the docetaxel. Cabazitaxel is a novel tubulin-binding taxane drug with anti-tumor activity in docetaxel-resistant cancers. In a large Phase 3 trial, the efficacy and safety of cabazitaxel plus prednisone was compared to that of mitoxantrone plus prednisone. There was a 30% reduction in the risk of death with the cabazitaxel (15.1 months versus 12.7 months). None of the other agents listed have been shown to improve survival in men with castration-resistant prostate cancer. Two additional agents recently approved by the FDA which are abiraterone and enzalutamide, have been shown to also improve survival in this patient population. None of the other regimens have demonstrated a survival difference in this population of patients. de Bono JS, Oudard S, Ozguroglu M, et al: Prednisone plus cabazitaxel or mitoxantrone for metastatic castration-resistant prostate cancer progressing after docetaxel treatment: A randomised open-label trial. LANCET 2010;376:1147-1154. Cookson MS, Kibel AS, Dahm P, et al: Castration-resistant prostate cancer: AUA guideline. CASTRATION-RESISTANT PROSTATE CANCER. American Urological Association Education and Research, Inc, 2013. http://www.auanet.org/education/guidelines/castration-resistant-prostate-cancer.cfm

2014 - 149 A 68-year-old active man with metastatic castration-resistant prostate cancer receives six cycles of docetaxel plus prednisone. Bone scan reveals several new rib lesions. To improve the likelihood of overall survival, the next treatment step is prednisone and: A. continued docetaxel. B. zoledronic acid. C. ketoconazole. D. mitoxantrone. E. cabazitaxel.

C Acute adrenal insufficiency, or adrenal crisis, is an acute condition often preceded by hypotension unresponsive to fluid resuscitation. Patients are often misdiagnosed with an acute abdomen, whereas abdominal pain, nausea, vomiting, and fever frequently accompany hypovolemia. Adrenal insufficiency following adrenalectomy in the setting of a normally functioning contralateral adrenal gland is unlikely, but possible. This is especially true for patients who are undergoing adrenalectomy for a cortisol-secreting lesion, because functionality of the contralateral gland can be suppressed as in this patient. The diagnosis of primary adrenal insufficiency is primarily made on clinical grounds, with a high index of suspicion given a patient's history, exam, and laboratory evaluation. Because adrenal crisis can be an acute and potentially life-threatening condition, consideration for treatment (i.e. repletion) should be made at the outset. If desired, the diagnosis is ultimately secured by measurements of morning serum cortisol and ACTH, but unnecessary delay in treatment should not be made simply to secure this diagnosis. Urinary cortisol and metanephrine levels are not indicated or necessary in this case. Furthermore, the initiation of broad-spectrum antibiotics will not address the underlying problem of adrenal insufficiency. The treatment of adrenal insufficiency involves adrenal hormonal repletion. Cortisol is replaced with hydrocortisone or with cortisone acetate. Kutikov A, Crispen PL, Uzzo RG: Pathophysiology, evaluation, and medical management of adrenal disorders, Wein AJ, Kavoussi LR, Novick AC, Partin AW, Peters CA (eds): CAMPBELL-WALSH UROLOGY, ed 10. Philadelphia, Elsevier Saunders, 2012, vol 2, chap 57, p 1685.

2014 - 15 A 58-year-old woman undergoes an uncomplicated laparoscopic right adrenalectomy for a 6 cm cortisol hypersecreting right adrenal mass. On postoperative day two, she has a low-grade fever, nausea, vomiting, hypotension, and abdominal pain. The next step is: A. 24-hour urine-free cortisol measurement. B. measurement of plasma metanephrine levels. C. hydrocortisone therapy. D. broad-spectrum antibiotics. E. exploratory laparotomy.

E This patient is likely suffering from hypokalemia. Patients following vigorous bowel cleansing, even with Go-Lytely or other electrolyte solutions, will often have low potassium. The characteristic signs include muscular weakness with poor respiratory effort causing decreased ventilation in the presence of a normal chest x-ray. Latex allergy may present with itching, erythema, wheezing and coughing, or with anaphylactic shock and hypotension, and severe respiratory distress with or without loss of consciousness. Hypercalcemia may present with excessive thirst, nausea and vomiting, abdominal and joint pain, and lethargy. Muscle cramps, weakness and tremors, and cardiac arrhythmia, may develop in the patient with hypomagnesemia. Common signs and symptoms of pulmonary embolism include shortness of breath, chest pain similar to that experienced with a myocardial infarction, excessive sweating, rapid or irregular heart rate, and in some cases, loss of consciousness. Shoskes DA, McMahon AW: Renal physiology and pathophysiology, Wein AJ, Kavoussi LR, Novick AC, Partin AW, Peters CA (eds): CAMPBELL-WALSH UROLOGY, ed 10. Philadelphia, Elsevier Saunders, 2012, vol 2, chap 38, p 1042.

2014 - 150 A 17-year-old girl with spina bifida undergoes a two-day bowel preparation followed by bladder neck reconstruction and ileocystoplasty. Intraoperatively, she receives five liters of D5 NS with two units of packed RBCs. Two hours postoperatively, she has decreased respiratory effort and generalized muscle weakness. Arterial blood gas shows a respiratory acidosis. Her chest x-ray is normal. Her symptoms are most likely due to: A. latex allergy. B. hypercalcemia. C. hypomagnesemia. D. pulmonary embolus. E. hypokalemia.

D In boys with proximal hypospadias, the prostatic utricle is often enlarged. In a female, this would represent the distal one-third of the vagina. The utricle is of urogenital sinus origin. While an ectopic ureter or bladder diverticulum could have a similar appearance on ultrasound, they generally are not midline in location. Ectopic ureter or bladder diverticulum are not seen commonly with hypospadias. A cecoureterocele would have a bladder deformity in addition to a suburethral extension. A Cowper's duct cyst or syringocele should be confined to the bulbous or prostatic urethra where Cowper's ducts drain. Snodgrass WT: Hypospadias, Wein AJ, Kavoussi LR, Novick AC, Partin AW, Peters CA (eds): CAMPBELL-WALSH UROLOGY, ed 10. Philadelphia, Elsevier Saunders, 2012, vol 4, chap 130, p 3506.

2014 - 39 A seven-year-old boy has had multiple repairs for penoscrotal hypospadias. He has recurrent lower UTIs and post-void dribbling. A renal ultrasound is normal. A pelvic ultrasound is shown. The most likely diagnosis is: A. cecoureterocele. B. ectopic ureter. C. Cowper's duct cyst. D. prostatic utricle. E. bladder diverticulum.

E The urodynamic tracing indicates detrusor overactivity associated with incontinence and bladder outlet obstruction during emptying. The tracing does show increased EMG activity during the detrusor overactivity but this is due to volitional suppression, not true striated dyssynergia. The image demonstrates narrowing at the proximal urethra consistent with bladder neck obstruction with voiding. At the time, the voiding image is taken there is no increased EMG activity excluding striated sphincter dyssynergia. There is no bulbar urethral narrowing to suggest urethral stricture disease. High-pressure voiding excludes impaired detrusor contractility. Nitti VW: Urodynamic and video-urodynamic evaluation of the lower urinary tract, Wein AJ, Kavoussi LR, Novick AC, Partin AW, Peters CA (eds): CAMPBELL-WALSH UROLOGY, ed 10. Philadelphia, Elsevier Saunders, 2012, vol 3, chap 62, pp 1863-1864.

2014 - 16 A 45-year-old neurologically normal man has worsening urinary incontinence for several years. Videourodynamic study is shown with the voiding image. The diagnosis is: A. striated sphincter dyssynergia. B. stress urinary incontinence. C. bulbar urethral stricture disease. D. detrusor overactivity with impaired contractility. E. bladder neck obstruction.

A Intra-urethral prostaglandin administration is a reasonable alternative to intracavernosal injection therapy. Significant decrease in blood pressure occurs in approximately 2% of men. Penile pain is estimated to occur in 18-33% of men. Only 40% of men will consistently attain an erection adequate for penetration with intra-urethral alprostadil. Urethral bleeding is reported in about 5% of men and headache is rare. Burnett AL: Evaluation and management of erectile dysfunction, Wein AJ, Kavoussi LR, Novick AC, Partin AW, Peters CA (eds): CAMPBELL-WALSH UROLOGY, ed 10. Philadelphia, Elsevier Saunders, 2012, vol 1, chap 24, p 745.

2014 - 17 Three months following a bilateral nerve sparing radical prostatectomy, a 65-year-old man has erectile dysfunction. He has failed oral therapy and wishes not to pursue intracavernosal injection therapy. He attempts intraurethral alprostadil 1000 mcg. The most likely occurrence is: A. inadequate erection. B. penile pain. C. headache. D. hypotension. E. urethral bleeding.

C The natural history of RCC is highly unpredictable. For instance, approximately 5% of patients with what are usually small indolent tumors (< 4 cm in size), will have metastatic disease at presentation and subsequently an elevated risk of disease-specific mortality. In contrast, up to 40% of patients with lymph node metastases diagnosed at the time of nephrectomy are alive five years after surgery. The Motzer criteria is a validated risk measurement tool that the physician may employ to aid in the discussion regarding the patient's prognosis and is a useful guide in the formulation of treatment options. RCC risk groups are determined by the number of existing adverse features. The adverse features included in the Motzer criteria are Karnosky performance status < 80%, elevated LDH, anemia, hypercalcemia, and prior partial or total nephrectomy. Thrombocytopenia is not part of the criteria. If no risk factors are present, the patient is considered at low risk for recurrence. The presence of one to two features indicates intermediate risk and the presence of three to five adverse features poor/high risk for tumor recurrence. Srinivasan R, Linehan WM: Treatment of advanced renal cell carcinoma, Wein AJ, Kavoussi LR, Novick AC, Partin AW, Peters CA (eds): CAMPBELL-WALSH UROLOGY, ed 10. Philadelphia, Elsevier Saunders, 2012, vol 2, chap 50, p 1476. Motzer RJ, Mazumdar M, Bacik J, et al: Survival and prognostic stratification of 670 patients with advanced renal cell carcinoma. J CLIN ONCOL 1999;17:2530-2540.

2014 - 18 An adverse prognostic feature not included in the Motzer Criteria for patients with metastatic RCC is: A. Karnofsky performance status (KPS) < 80%. B. elevated LDH. C. thrombocytopenia. D. prior nephrectomy. E. hypercalcemia.

D In 2009, the Division of Cancer Treatment and Detection of the National Institutes of Health (NIH) conducted a review concerning 18F-PET imaging and its utility for assessing cancer metastases to bone, and concluded that 18F-PET provides the best sensitivity and specificity for the detection of bony metastases in prostate cancer. This review and other studies have demonstrated the superiority of 18F-PET to conventional (99mTc-MDP) bone scan with regard to specificity and sensitivity. Plain film tomography, CT scan with bone windows, and SPECT/CT have been used to evaluate suspicious or suspected areas of bony metastasis, but are not utilized for the initial survey of metastases in the high-risk patient. Each of these studies have more limited performance characteristics than 18F-PET. It remains to be seen whether this imaging modality will become the standard of care. Jadvar H: Molecular imaging of prostate cancer: PET radiotracers. AM J ROENTGENOL 2012;199:278-291. Rioja J, Rodr_guez-Fraile M, Lima-Favaretto R, et al: Role of positron emission tomography in urological oncology. BJU INT 2010;106:1578-1593. Even-Sapir E, Metser U, Mishani E, et al: The detection of bone metastases in patients with high-risk prostate cancer: 99mTc-MDP planar bone scintigraphy, single- and multi-field-of-view SPECT, 18F-fluoride PET, and 18F-fluoride PET/CT. J NUCL MED 2006;47:287-297. Bauman G, Belhocine T, Kovacs M, et al: 18F-fluorocholine for prostate cancer imaging: A systematic review of the literature. PROSTATE CNCR PROSTATIC DIS 2012;15:45-55.

2014 - 19 The imaging study providing the best sensitivity and specificity for assessing bony metastatic disease in men with high-risk prostate cancer is: A. plain film tomography. B. CT scan with bone windows. C. 99mTc-MDP bone scan. D. 18F-fluoride PET scan. E. single-photon emission computed tomography (SPECT) scintigraphy.

B The patient has suffered a major straddle injury resulting in complete bulbar urethral disruption. The next best step is suprapubic tube placement and delayed reconstruction. Perineal exploration is not indicated in the setting of acute blunt trauma and complete disruption of the bulbar urethra. Catheter placement is not indicated with complete disruption. Antegrade or retrograde urethral realignment is not indicated with complete bulbar urethral disruption, yet may be possible with an anterior urethral contusion or incomplete disruption. Morey AF, Dugi DD III: Genital and lower urinary tract trauma, Wein AJ, Kavoussi LR, Novick AC, Partin AW, Peters CA (eds): CAMPBELL-WALSH UROLOGY, ed 10. Philadelphia, Elsevier Saunders, 2012, vol 3, chap 88, p 2520.

2014 - 2 A 42-year-old man is unable to void following a straddle injury. Physical examination reveals blood at the meatus and a large perineal hematoma. Retrograde urethrography reveals a complete bulbar urethral disruption with contrast extravasation. The next step is: A. urethral catheter placement. B. suprapubic tube placement. C. flexible cystoscopy with urethral realignment. D. open cystotomy and antegrade urethral realignment. E. perineal exploration and repair.

C This patient has a large intraperitoneal bladder perforation and urinary ascites from an unrecognized bladder injury during hysterectomy. The best choice for management is immediate transperitoneal exploration with repair of the bladder injury. This will allow drainage of the urinary ascites, washing out of the peritoneal cavity, and significantly reduce the risk of peritonitis and vesicovaginal fistula formation. The use of a pedicalized omental flap to place between the bladder repair and the vaginal cuff suture lines should also be attempted to avoid overlapping suture lines and further minimize risk of post-operative fistula. Prolonged catheter drainage is the incorrect management of an intraperitoneal bladder injury and may result in prolonged urinary ascites with resultant persistent ileus and peritonitis. Bilateral nephrostomy tubes often do not result in complete urinary diversion and would likely result in a prolonged clinical course and delay recovery. Transvaginal repair is the wrong approach to repair an intraperitoneal bladder injury. Morey AF, Dugi DD III: Genital and lower urinary tract trauma, Wein AJ, Kavoussi LR, Novick AC, Partin AW, Peters CA (eds): CAMPBELL-WALSH UROLOGY, ed 10. Philadelphia, Elsevier Saunders, 2012, vol 3, chap 88, p 2516.

2014 - 22 A 40-year-old woman has urine draining from a port site three days following laparoscopic assisted vaginal hysterectomy. Cystogram is shown and bilateral retrograde pyelograms are normal. The next step is: A. prolonged catheter drainage. B. bilateral percutaneous nephrostomies. C. exploratory laparotomy. D. transvaginal repair. E. place pelvic drain.

D In an unstable patient, ureteral injuries are best managed by ureteral ligation, percutaneous nephrostomy drainage, and delayed repair. In these instances, other choices are inappropriate because of the time needed for completion of the repair. In stable patients, short upper ureteral injuries may be managed by ureteroureterostomy with excision of the injured segment. A transureteroureterostomy may be appropriate with a long mid-ureteral injury but not an upper ureteral injury. A nephrectomy is not indicated in the absence of renal trauma especially if delayed salvage of the ureter is possible. Elliott SP, McAninch JW: Ureteral injuries from external violence. AUA UPDATE SERIES 2004, vol 23, lesson 1, pp 4-6.

2014 - 23 A 27-year-old gunshot victim has a short upper ureteral injury and a splenic injury. During exploratory laparotomy, his vital signs are unstable with significant hypotension despite management of the splenic injury. No other acute injuries are present. The next step in management of his ureteral injury is: A. retrograde ureterogram and placement of a ureteral stent. B. excision of injured segment and ureteroureterostomy. C. transureteroureterostomy. D. ligation of ureter and percutaneous nephrostomy. E. nephrectomy.

C In cases of unilateral upper tract cytologic abnormalities (with normal cystoscopy, pyelography, and bladder biopsies), ureteropyeloscopy is indicated as the next step. Ureteropyeloscopy allows for direct visualization of small lesions and is superior to retrograde pyelography in the detection of small tumors. Biopsy at the time of ureteropyeloscopy should be attempted, if feasible. A persistently abnormal cytology without any visualized lesions may signify CIS. In the past, nephroureterectomy was performed for a unilateral cytologic abnormality of the upper tract to eliminate presumed CIS, but this is no longer considered an appropriate initial approach. Observation is also not appropriate without further evaluation given the repeated abnormal cytologies. Current approaches for presumed upper tract CIS include topical immunotherapy or chemotherapy, delivered retrograde intravesically with ureteral stents in place to assist with reflux or antegrade via a nephrostomy tube under careful pressure control. Novel urinary markers (e.g., FISH) have been reported for upper tract tumor surveillance and may aid in the detection of such tumors. To date, none of these markers have a high enough sensitivity or specificity to make decisions for or against therapeutic intervention. In this case, repeat procedure under anesthesia to obtain a selective urinary sample for FISH is not warranted. Sagalowsky AI, Jarrett TW, Flanigan RC: Urothelial tumors of the upper urinary tract and ureter, Wein AJ, Kavoussi LR, Novick AC, Partin AW, Peters CA (eds): CAMPBELL-WALSH UROLOGY, ed 10. Philadelphia, Elsevier Saunders, 2012, vol 2, chap 53, pp 1549-1550. Wood DP: Urothelial tumors of the bladder, Wein AJ, Kavoussi LR, Novick AC, Partin AW, Peters CA (eds): CAMPBELL-WALSH UROLOGY, ed 10. Philadelphia, Elsevier Saunders, 2012, vol 3, chap 80, pp 2328-2329.

2014 - 24 A 60-year-old smoker has a highly suspicious voided urinary cytology. CT urogram is normal. Cystoscopy, bladder biopsy, and bilateral retrograde pyelograms are normal. Selective left upper tract cytologies are highly suspicious for malignancy. The next step is: A. repeat cystoscopy, biopsy, retrograde pyelography, and selective cytologies in three months. B. repeat left ureteral washings for fluorescent in-situ hybridization (FISH). C. left ureteropyeloscopy. D. left ureteral stent and intravesical BCG. E. left percutaneous nephrostomy and antegrade BCG therapy.

D This 52-year-old man is at risk for renovascular hypertension. Of the two captopril modulated tests: plasma renin activity (PRA) and captopril renogram, the renogram is a better test than peripheral PRA. Critical to the performance of these tests is appropriate patient preparation. Ideally, patients should be off all medications for two weeks. This is usually not possible clinically. In patients on medications it is critically important to recognize that the use of ACE inhibitors will significantly reduce the sensitivity of captopril stimulated testing and should be discontinued two weeks prior to the test. However, other antihypertensive medications can be used up to the morning of testing. In this clinical setting, duplex ultrasound will give anatomic information on the renal arteries sufficient to determine the need for angiography. Diuretic renography is not generally used as a test for renal artery stenosis. Fergany A, Novick AC: Renovascular hypertension and ischemic nephropathy, Wein AJ, Kavoussi LR, Novick AC, Partin AW, Peters CA (eds): CAMPBELL-WALSH UROLOGY, ed 10. Philadelphia, Elsevier Saunders, 2012, vol 2, chap 39, pp 1061-1062.

2014 - 25 A 52-year-old man develops abrupt and severe hypertension. He is poorly controlled with an ACE inhibitor, calcium channel blocker, diuretic, and minoxidil. None of these medications can be safely withheld. Serum creatinine is 1.3 mg/dl. The best way to evaluate for renovascular hypertension is: A. captopril plasma renin activity test. B. unstimulated plasma renin activity test. C. captopril renography. D. duplex ultrasound. E. diuretic renography.

D This patient has chancroid. According to the CDC, the drug of choice is a single dose of azithromycin (1 gram p.o.). Alternatively, erythromycin 500 mg p.o. q.i.d. for seven days or ceftriaxone 250 mg I.M. once can be effective. The other antibiotics will not be effective to eliminate the organism which is Haemophilus ducreyi. Acyclovir is effective against the herpes virus. Frenkl T, Potts J: Sexually transmitted diseases, Wein AJ, Kavoussi LR, Novick AC, Partin AW, Peters CA (eds): CAMPBELL-WALSH UROLOGY, ed 10. Philadelphia, Elsevier Saunders, 2012, vol 1, chap 13, pp 405-406.

2014 - 74 Five days after intercourse, a 40-year-old man develops a painful necrotic penile ulcer and painful inguinal lymphadenopathy. Gram stain of an exudate from the lesion reveals gram-negative coccobacilli. Dark-field examination and Tzanck smears are negative. The treatment of choice is: A. penicillin. B. tetracycline. C. acyclovir. D. azithromycin. E. sulfisoxazole.

E The AUA Ureteral Calculi Guideline states that for newly diagnosed ureteral stone < 10 mm and whose symptoms are controlled, observation with periodic evaluation is an initial option. Furthermore, such patients may be offered an appropriate medical therapy to facilitate stone passage during this observation period. Metanalysis has shown tamsulosin to be superior to nifedipine in medical expulsive therapy. However, tamsulosin has been described to cause intraoperative floppy iris syndrome (IFIS) and complicates cataract surgery. Therefore, in a patient who is about to undergo cataract surgery, it would be best to avoid tamsulosin. Therefore, the options for this particular patient are observation, SWL, ureteroscopic extraction, or ureteral stent. A 6 mm stone is best treated by ureteroscopic extraction, because more often than not (53% of the time), these stones will not spontaneously pass. An opaque stone implies that the stone is not uric acid and therefore alkalization with sodium bicarbonate would be an incorrect choice. Preminger GM, Tiselius HG, Assimos DG, et al: 2007 guideline for the management of ureteral calculi. URETERAL CALCULI, American Urological Association Education and Research, Inc, 2007. http://www.auanet.org/education/guidelines/ureteral-calculi.cfm Cantrell MA, Bream-Rouwenhorst HR, Steffensmeier A, et al: Intraoperative floppy iris syndrome associated with alpha1-adrenergic receptor antagonists. ANN PHARM 2008;42:558-563.

2014 - 26 A 65-year-old woman with controlled flank pain has an opaque 6 mm distal right ureteral stone. Urine pH is 6.0. She has no pyuria, fevers, or chills. She is scheduled to undergo cataract surgery in four weeks. The next step is: A. corticosteroids. B. tamsulosin. C. sodium bicarbonate. D. ureteral stent. E. ureteroscopic extraction.

D The relatively low incidence of RCC in the general population and the potential risk of identifying clinically insignificant lesions argue against widespread screening. However, several well-defined target populations who are at increased risk of RCC may be suitable for screening efforts. The relative risk of RCC in patients with ESRD has been estimated to be 5-20 times higher than that in the general population. The majority of patients with ESRD will develop acquired renal cystic disease (ARCD) and some of these patients will develop RCC. Cystic disease is associated with time of maintenance dialysis; increasing with duration of therapy. In renal failure patients, a reasonable approach to screening for patients with ESRD without other major comorbidities, is to delay screening until the third year on dialysis. Those with significant comorbidities, and thereby limited life expectancies, are not felt to benefit from screening and thereby should not be screened. Patients undergoing maintenance hemodialysis were previously thought to have a higher incidence of ARCD and thereby a higher risk of renal carcinoma than patients on peritoneal dialysis. However, recent studies have demonstrated similar rates of ARCD in both dialysis subgroups, and thereby suggests both forms of maintenance dialysis would increase the risk of developing renal carcinoma. Obesity and tobacco abuse have been shown to be risk factors for RCC, but these conditions themselves do not necessarily warrant screening efforts in patients with renal dysfunction. Screening for RCC in autosomal dominant polycystic kidney disease (ADPKD) was previously recommended. However, recent studies suggest no significantly increased risk of RCC in ADPKD and imaging is extremely difficult in this population related to the altered intrarenal architecture. Taken together, these considerations suggest that screening for RCC in patients with ADPKD should not be pursued. Campbell SC, Lane BR: Malignant renal tumors, Wein AJ, Kavoussi LR, Novick AC, Partin AW, Peters CA (eds): CAMPBELL-WALSH UROLOGY, ed 10. Philadelphia, Elsevier Saunders, 2012, vol 2, chap 49, pp 1438-1439.

2014 - 27 Screening for RCC in patients with ESRD should be reserved for patients: A. with significant comorbidities. B. initiating hemodialysis. C. with a history of obesity and tobacco abuse. D. who have undergone at least three years of dialysis. E. with autosomal dominant polycystic kidney disease.

C A metanalysis of patients treated with alpha-blockers for stent discomfort identified 12 randomized controlled trials with 946 patients. It concluded that alpha-blockers were associated with a significant decrease in urinary symptoms and pain, and significant improvement in general health. In contrast, randomized controlled studies failed to demonstrate benefits with oxybutynin, phenazopyridine, or a ketorolac eluting ureteral stent. Periureteral botulinum toxin injection at the time of stent placement has been demonstrated to decrease stent morbidity, however, this would require a secondary cystoscopic procedure, and has not been studied as a rescue procedure after stent placement. Nifedipine and prednisone have not been studied for use in stent pain. Yakoubi R, Lemdani M, Monga M, et al: Is there a role for alpha-blockers in ureteral stent related symptoms? A systematic review and meta-analysis. J UROL 2011;186:928-934. Norris RD, Sur RL, Springhart WP, et al: A prospective, randomized, double-blinded placebo-controlled comparison of extended release oxybutynin versus phenazopyridine for the management of postoperative ureteral stent discomfort. J UROL 2008;71:792-795. Gupta M, Patel T, Xavier K, et al: Prospective randomized evaluation of periureteral botulinum toxin type A injection for ureteral stent pain reduction. J UROL 2010;183:598-602. Krambeck AE, Walsh RS, Denstedt JD, et al: A novel drug eluting ureteral stent: A prospective, randomized, multicenter clinical trial to evaluate the safety and effectiveness of a ketorolac loaded ureteral stent. J UROL 2010;183:1037-1042.

2014 - 28 A 55-year-old man has flank and bladder pain with a ureteral stent after uncomplicated ureteroscopy. The next step is analgesics and: A. oxybutynin. B. nifedipine. C. tamsulosin. D. prednisone. E. phenazopyridine.

B This patient has an infected IPG. The bacterial infection of the artificial IPG will result in a bacterial biofilm that will also contaminate the lead. Both the IPG and lead should be explanted. Their risk of infection of a new device or new lead placement at the time of explantation is too high and should not be pursued. The proper management is explantation of all prosthetic material, treatment of the infection, and repeat test stimulation in the future when the patient is completely recovered. Vasavada SP, Rackley RR: Electrical stimulation and neuromodulation in storage and emptying failure, Wein AJ, Kavoussi LR, Novick AC, Partin AW, Peters CA (eds): CAMPBELL-WALSH UROLOGY, ed 10. Philadelphia, Elsevier Saunders, 2012, vol 3, chap 70, p 2039.

2014 - 29 A 58-year-old woman returns to the office two months following sacral neuromodulation with a low grade fever and incisional drainage associated with pain and erythema over the implantable pulse generator (IPG) site. The next step is explantation of the: A. IPG only. B. IPG and lead. C. IPG, wound irrigation and cleansing with antibiotic solution, and reimplantation of the IPG. D. IPG and lead with simultaneous test stimulation of a new lead. E. IPG with simultaneous placement of an IPG on the contralateral side.

C Autonomic dysreflexia (AD) is most commonly seen in patients with spinal cord injury levels above T8. It is an exaggerated sympathetic nervous system response to afferent visceral stimulation, manifest with sweating, headache, hypertension, reflex bradycardia, and flushing above the level of the spinal cord lesion. The normal BP in para and quadriplegics is low, usually 90-110 mmHg systolic. Elevation with autonomic dysreflexia symptoms classically begin with a 20 mmHg rise above baseline, well within normal range for a neurologic intact individual. If BP is > 120 mmHg and patient is symptomatic, presumed autonomic dysreflexia is present. Initial therapy should focus on the removal of inciting factors. In this case, that would include emptying of the bladder and removal of all urodynamic catheters. If the symptoms persist and systolic pressure remains elevated but lower than 150 mmHg, then evaluation for and treatment of fecal impaction, the second most common cause of AD after the bladder, is recommended. However, if the systolic pressure remains above 150 mmHg after bladder emptying and catheter removal, then use of a rapid-onset, short-acting antihypertensive is recommended while the cause of AD is investigated. Nitropaste 2%, applied .5-1 inch above the level of the lesion (vasoconstriction occurs below the level of the lesion and may interfere with the drugs absorption) is preferred due to its ability to be wiped free if rebound hypotension occurs. Alternatively, Nitroglycerin 0.4 mg sublingually, are the two first line drugs of choice in the outpatient setting. The physician must make sure the patient has not used a PDE-5 inhibitor for erectile dysfunction in the past 24 hours, due to concern for rebound hypotension. If a sildenafil agent has been used within 24 hours, Captopril 25 mg chewed or given sublingually becomes the drug of choice. Nifedipine used to be recommended as primary treatment or prophylactic agent for AD, however, because of several adverse, rebound hypotensive crisis resulting in stroke or MI after its use, the Joint commission for treatment of High Blood Pressure and National Spinal Cord Injury committees have discouraged its use and it has been banned for treatment or prevention of autonomic dysreflexia in some hospitals. If the blood pressure remains elevated and does not respond to oral therapy I.V., hydralazine is an option, however, BP may be quite labile after its use with both hypotension and/or rebound hypertension, the patient will require hospital admission with further monitoring. In the outpatient setting, when autonomic dysreflexia is triggered and successfully treated, it is recommended that the patient should be monitored for resumption of hypertension for a minimum of two hours. If AD recurs, hospitalization with monitoring for 24 hours is recommended, if not, the patient can be discharged from the outpatient setting. Recommendations to prevent autonomic dysreflexia preceding cystoscopy or urodynamic evaluations are to use: terazosin 5 mg, prazosin 1 mg, or tamsulosin 0.8 mg the night before the exam, or alternatively at the time of the exam place Nitropaste 2% .5 inch (if not on sildenafil) or Captopril 25 mg sublingually 10-15 minutes prior to exam. Gunduz H, Binak DF: Autonomic dysreflexia: An important cardiovascular complication in spinal cord injury patients. CARD J 2012;19:215-219. Wein AJ, Dmochowski RR: Neuromuscular dysfunction of the lower urinary tract, Wein AJ, Kavoussi LR, Novick AC, Partin AW, Peters CA (eds): CAMPBELL-WALSH UROLOGY, ed 10. Philadelphia, Elsevier Saunders, 2012, vol 3, chap 65, p 1926.

2014 - 75 A 32-year-old man with a T5 spinal cord injury develops profuse sweating, hypertension, and bradycardia during urodynamic evaluation. The bladder is emptied but his symptoms persist with a blood pressure of 170/100 mmHg. The next step is to administer: A. oral nifedipine. B. sublingual nifedipine. C. transdermal nitroglycerin, above the level of injury. D. I.V. atropine. E. I.V. hydralazine.

C Suboptimal approaches to testicular neoplasms, including scrotal orchiectomy, transscrotal biopsy, or fine-needle aspiration are reported from 4-17% of the time. A recent meta-analysis of 206 cases of scrotal violation reported a local recurrence rate of 2.9% compared with 0.4% of patients treated by inguinal orchiectomy, but no difference in systemic relapse or survival rates. There did not appear to be any advantage to adjuvant therapy. Others have reported an increased local recurrence rate in patients with scrotal contamination and an 11% presence of tumor in hemiscrotectomy specimens of patients with scrotal violation. Therefore, for patients with scrotal violation with low-stage seminoma, the radiation portals should be extended to include the ipsilateral groin and scrotum. This may result in an increased risk of azoospermia. Observation is not appropriate in the setting of scrotal violation. Platinum, etoposide, and bleomycin (BEP) is appropriate for treatment of non-seminomatous germ cell tumors and advanced seminoma, not low-stage seminoma. Sheinfeld J, Bosl GJ: Surgery of testicular tumors, Wein AJ, Kavoussi LR, Novick AC, Partin AW, Peters CA (eds): CAMPBELL-WALSH UROLOGY, ed 10. Philadelphia, Elsevier Saunders, 2012, vol 1, chap 32, p 873.

2014 - 3 A patient has a transscrotal orchiectomy for a 3 cm testicular mass. At the time of exploration, there is violation of the tunica vaginalis. Pathology is pure seminoma. Chest and abdominal CT scan and serum markers are negative. The next step is: A. surveillance. B. retroperitoneal XRT. C. retroperitoneal XRT including the groin and hemiscrotum. D. excision of scrotal scar and retroperitoneal XRT. E. three cycles of BEP.

C The incidental adrenal lesion, or incidentaloma, is an adrenal mass 1 cm or larger in size discovered during radiographic imaging performed for indications other than adrenal disease. They are found in approximately four to six percent of the imaged population. Almost all of incidental adrenal lesions < 4 cm in size will prove to be benign in a patient without a known history of cancer. In patients with a known history of cancer, up to 50% will represent metastatic malignancy. This patient has an incidentally discovered adrenal adenoma that is probably benign based on its size and the HU of 5. Specifically, a non-contrast CT scan revealing a < 4 cm adrenal mass with HU < 10 carries a 98% specificity that the diagnosis is a benign adrenal adenoma. In masses with HU > 10, a contrast CT scan with early phase wash-out could be obtained to rule-out malignancy; it is, however, not necessary in this instance. Similarly, a gadolinium-enhanced MRI scan is not indicated and is not as good as CT scan when looking for adrenal hypervascularity. The current recommendation for the management of an incidentally discovered adrenal mass < 4 cm in size with an HU value of < 10 is a biochemical work-up to include cortisol and catecholamines. In patients with a history of hypertension, a hyperaldosteronemia should also be ruled out. Approximately 7% of these incidentalomas are metabolically active, 6% cortisol, and 1% sex or aldosterone secreting. Surgical resection is recommended for all metabolically active adrenal tumors or adrenal tumors > 4 cm. Of note, in a seven year follow-up study of 231 patients with incidental adrenal tumors < 4 cm in size and HU < 10, less than 2% of the metabolically inactive adrenal tumors will either substantially grow or become metabolically active. Therefore, neither observation nor repeat CT scan in three months is appropriate. If the patient had a history of a prior malignancy, higher HU, or larger mass, then biopsy might be indicated. Kutikov A, Crispen PL, Uzzo RG: Pathophysiology, evaluation, and medical management of adrenal disorders, Wein AJ, Kavoussi LR, Novick AC, Partin AW, Peters CA (eds): CAMPBELL-WALSH UROLOGY, ed 10. Philadelphia, Elsevier Saunders, 2012, vol 2, chap 57, p 1731.

2014 - 30 A 35-year-old woman with no significant findings on medical history has a non-contrast CT scan performed for right flank pain. A 2 cm left adrenal mass (5 HU) is identified. The next step is: A. observation. B. repeat CT scan in three months. C. biochemical work-up. D. percutaneous biopsy. E. gadolinium enhanced MRI scan.

A The major clotting protein in semen has been termed semenogelin, which has been shown to be the seminal vesicle-specific antigen. These clotted proteins serve as the substrate for PSA which liquefies the semen. Calcium-binding substances, such as sodium citrate and heparin do not inhibit the coagulation. Blood clotting proteins such as prothrombin, fibrinogen, and factor XII are not present in semen. Berman DM, Rodriguez R, Veltri RW: Development, molecular biology, and physiology of the prostate, Wein AJ, Kavoussi LR, Novick AC, Partin AW, Peters CA (eds): CAMPBELL-WALSH UROLOGY, ed 10. Philadelphia, Elsevier Saunders, 2012, vol 3, chap 90, p 2568.

2014 - 31 The coagulation of human semen is dependent on: A. seminal vesicle-specific antigen. B. PSA. C. calcium. D. fibrinogen. E. factor XII.

A The clinical picture is consistent with benign urethrorrhagia based on terminal gross hematuria and normal physical examination. Urinalysis shows microscopic hematuria only, and the culture is negative. This condition is caused by transient inflammation of the bulbar urethral epithelium. The cause is not known. No further work-up such as calcium to creatinine ratio, non-contrast CT scan, VCUG, or cystoscopy is necessary since this is self-limiting. Palmer LS, Trachtman H: Renal functional development and diseases in children, Wein AJ, Kavoussi LR, Novick AC, Partin AW, Peters CA (eds): CAMPBELL-WALSH UROLOGY, ed 10. Philadelphia, Elsevier Saunders, 2012, vol 4, chap 112, p 3005.

2014 - 32 A 12-year-old boy has painless terminal gross hematuria. Physical examination is normal. Urinalysis shows 3-5 RBC/hpf. Urine culture is negative. The next step is: A. observation. B. urine calcium to creatinine ratio. C. non-contrast CT scan. D. VCUG. E. cystoscopy.

D The radiograph reveals a staghorn stone in the right kidney and several large stones in the Indiana Pouch. SWL or percutaneous extraction of the pouch stones does not address the renal calculi. The staghorn stone needs to be treated as well, an untreated staghorn stone risks urosepsis. Since the right kidney is contributing little to the patientÆs adequate renal function, nephrectomy is recommended over stone removal procedures that leave the kidney in place. Though percutaneous extraction of pouch stones is recommended, open surgical extraction is a good alternative with this large stone burden. In a urinary diversion with good continence, pouch revision is not necessary. The patient should be encouraged to catheterize and irrigate the pouch more frequently. L'Esperance JO, Sung J, Marguet C, et al: The surgical management of stones in patients with urinary diversions. CURR OPIN UROL 2004;14:129-134. Preminger GM, Assimos DG, Lingeman JE, et al: AUA guideline on management of staghorn calculi: Diagnosis and treatment recommendations. STAGHORN CALCULI, American Urological Association Education and Research, Inc, 2005. http://www.auanet.org/education/guidelines/staghorn-calculi.cfm

2014 - 33 A 49-year-old man with a continent cutaneous reservoir has a KUB as shown. He has good continence. Serum creatinine is 1.4 mg/dl. Split renal function by renal scintigraphy is 10% right, 90% left. The next step is: A. open extraction of reservoir stones. B. percutaneous extraction of reservoir stones. C. percutaneous extraction of reservoir and renal stones. D. open extraction of reservoir stones and right nephrectomy. E. revision of reservoir and right nephrectomy.

C Some hydronephrosis is expected after ileal conduit urinary diversion. Without a history of malignancy, computed tomography with contrast is not necessary. Loopography is the recommended next step with the new onset of hydronephrosis. It can assess for reflux into the left kidney. If there is free reflux, then obstruction is unlikely. If reflux is absent, then diuretic renal scintigraphy can be used to assess for obstruction. If obstruction is present, percutaneous nephrostomy can drain the kidney and allow the system to be studied. Looposcopy with ureteral catheterization is unlikely to succeed if the system is obstructed, and is unnecessary if it is not obstructed. Fulgham PF, Bishoff JT: Urinary tract imaging: Basic principles, Wein AJ, Kavoussi LR, Novick AC, Partin AW, Peters CA (eds): CAMPBELL-WALSH UROLOGY, ed 10. Philadelphia, Elsevier Saunders, 2012, vol 1, chap 4, pp 108-109.

2014 - 34 A 55-year-old woman has new onset moderate left hydronephrosis on non-contrast CT scan. Twenty years earlier, she underwent cystectomy and ileal conduit for a neurogenic bladder. Her serum creatinine is 1.4 mg/dl. The next step is: A. observation. B. urine cytology. C. loopogram. D. percutaneous nephrostomy. E. looposcopy with catheterization of the left ureter.

C The patient's evaluation reveals oligoasthenospermia with low ejaculate volume without any obvious cause. Post-ejaculatory urinalysis (PEU) is the best next step in order to identify a correctable cause for this patient's semen abnormalities. Hamster egg penetration test and antisperm antibody testing are useful for prognostic purposes but will not identify a correctable abnormality. Post-coital test is helpful in patients with isolated volume problems but would not be helpful in patients with oligoasthenospermia. Scrotal ultrasonography for subclinical varicocele is not indicated. TRUS may be considered if the PEU is negative but should not be considered first since it is more invasive than a PEU. Jarow J, Sigman M, Kolettis PN, et al: The optimal evaluation of the infertile male: AUA best practice statement revised, 2010. OPTIMAL EVALUATION OF THE INFERTILE MALE, American Urological Association Education and Research, Inc, 2010. http://www.auanet.org/education/guidelines/male-infertility-d.cfm

2014 - 35 A 26-year-old infertile man has an ejaculate volume of 0.9 ml, sperm count of 10 million/ml, and 20% motility. Physical examination and hormonal evaluation are normal. The next step is: A. antisperm antibody testing. B. semen culture. C. post-ejaculatory urinalysis. D. TRUS. E. scrotal Doppler ultrasonography.

E At the time of radical cystectomy, suspicious lymph nodes can be encountered. Approximately 25% of patients will ultimately have positive lymph nodes at the time of radical cystectomy. If the suspicious node(s) can be safely resected and the volume of suspicious lymph nodes is limited, it is reasonable to continue the cystectomy and continent urinary diversion. The patient will benefit from the local control of the lymphadenectomy and cystectomy. There is no evidence that stopping the surgery to treat with neoadjuvant chemotherapy or chemotherapy and radiation therapy is superior to completing the cystectomy. Positive lymph nodes are not a contraindication to continent urinary diversion. Lerner SP, Sternberg CN: Management of metastatic and invasive bladder cancer, Wein AJ, Kavoussi LR, Novick AC, Partin AW, Peters CA (eds): CAMPBELL-WALSH UROLOGY, ed 10. Philadelphia, Elsevier Saunders, 2012, vol 3, chap 82, p 2360.

2014 - 36 A 70-year-old man is undergoing radical cystectomy and continent orthotopic urinary diversion for muscle-invasive high-grade urothelial carcinoma. During lymphadenectomy, a suspicious, firm 1.5 cm positive external iliac lymph node is confirmed to be positive for metastasis. The next step is: A. abort surgery and treat with chemotherapy and XRT. B. abort surgery and treat with chemotherapy followed by cystectomy. C. perform lymphadenectomy and treat with chemotherapy and XRT. D. complete surgery but perform an ileal conduit urinary diversion. E. complete surgery as planned.

A This patient complains of pain only when the device is inflated during intercourse which may indicate that the cylinders are too large and are buckling, causing pain. MRI scan with the prosthesis inflated is the best imaging modality to prove the possible size discrepancy and buckling is the next step. If the diagnosis is confirmed, revision of the penile prosthesis with placement of smaller cylinders will usually resolve the problem. Penile infection following prosthetic placement usually has associated physical signs of infection (warmth, tenderness, erythema, etc.) that exist in both the flaccid or erect state. An SST deformity (floppy glans) is usually well-described by a patient and can be confirmed by physical examination. It can be repaired by moving the glans onto the distal portion of the cylinders with a glansplasty. Cystourethroscopy is indicated in the presence of voiding symptoms and/or possible infection of the penile prosthesis to rule out urethral erosion, but is not indicated in patients with pain only on inflation. Revision with a lock-out valve would only be useful in cases of autoinflation. Kirkemo A: Complications of urologic surgery, COMPLICATIONS OF PENILE SURGERY, ed 3. Smith and Ehrlich, 1999, chap 39, p 541. Montague DK: Prosthetic surgery for erectile dysfunction, Wein AJ, Kavoussi LR, Novick AC, Partin AW, Peters CA (eds): CAMPBELL-WALSH UROLOGY, ed 10. Philadelphia, Elsevier Saunders, 2012, vol 1, chap 27, p 787.

2014 - 37 Three months after placement of a three-piece inflatable penile prosthesis, a patient has persistent penile shaft pain with inflation. Physical examination of the penis and scrotum is normal. The next step is: A. pelvic and scrotal MRI scan. B. cystourethroscopy. C. 500 mg cephalexin BID for 30 days. D. glansplasty. E. revision of reservoir with a lock out valve.

C Although hyperuricosuria, low urine pH, and low urine volume are all important contributors to uric acid stone formation, a persistently acid urine (pH < 5.5) is the most important determining factor for uric acid stones. Although hyperuricemia is the hallmark of primary gout, not all patients with uric acid stones have either hyperuricemia or gout. Renal tubular acidosis is associated with high rather than low urine pH, hyperchloremia, and hypokalemia. Knudsen BE, Beiko DT, Denstedt JD: Uric acid urolithiasis, Stoller ML, Meng MV (eds): URINARY STONE DISEASE, Totowa NJ, Humana Press, 2007, chap 16, pp 299-301.

2014 - 38 In patients with uric acid stones, the primary underlying metabolic defect is: A. hyperuricosuria. B. hyperuricemia. C. low urine pH. D. low urine volume. E. RTA.

C When the testis is more than 2 cm above the internal ring with no vascular redundancy, it is unlikely that the testis will be able to reach the scrotum without dividing the spermatic vessels. In this situation, staged Fowler-Stephens procedure with division of the gonadal vessels at the first stage has a significantly higher success rate than primary Fowler-Stephens procedure. With the high abdominal position of the testis and short spermatic vessels, the testis is unlikely to reach the scrotum with either a laparoscopic or open transabdominal orchiopexy, which would both be limited by the ability to adequately mobilize the vessels. Although hCG has been used to increase testicular vascularity, there are no documented studies to prove that administration of hCG pre-Fowler Stephen orchiopexy improves testicular viability. Indeed, there is concern that acute hCG stimulation could cause damage to the seminiferous tubules. Barthold JS: Abnormalities of the testes and scrotum and their surgical management, Wein AJ, Kavoussi LR, Novick AC, Partin AW, Peters CA (eds): CAMPBELL-WALSH UROLOGY, ed 10. Philadelphia, Elsevier Saunders, 2012, vol 4, chap 132, pp 3570-3572. Leon MD, Chiazzi VA, Calvo JC, et al: Acute hCG administration induces seminiferous tubule damage in the adult rat. ACTA PHYSIOL PHARM LATINOAM 1987;37:277-288. Hjertkvist M, Bergh A, Damber JE: hCG treatment increases intratesticular pressure in the abdominal testis of unilaterally cryptorchid rats. J ANDROL 1988;9:116-120.

2014 - 93 Diagnostic laparoscopy is performed on a one-year-old boy with a nonpalpable left testicle. The testis is found 3 cm above the internal ring, with a long looping vas deferens and short spermatic vessels. The procedure most likely to result in successful orchiopexy is: A. laparoscopic orchiopexy with intact spermatic vessels. B. laparoscopic single stage Fowler-Stephens orchiopexy. C. clip spermatic vessels and return in six months. D. open transabdominal orchiopexy. E. remove laparoscope, administer intramuscular hCG for four weeks, then re-explore.

A There are two isoenzymes of 5-alpha-reductase (Type I and Type II). Finasteride blocks only the Type II isoenzyme. The prostate contains the Type II enzyme. Therefore, dihydrotestosterone (DHT) levels are decreased in the prostate. There is an upregulation of serum and intraprostatic testosterone. The serum DHT is typically decreased by about 85%, but not to castrate levels as circulating testosterone is converted to DHT by Type I isoenzyme in the skin and liver. The decreased DHT production leads to reduction in negative feedback which leads to increased LH production, hence increase serum testosterone. McNicholas TA, Kirby RS, Lepor H: Evaluation and nonsurgical management of benign prostatic hyperplasia, Wein AJ, Kavoussi LR, Novick AC, Partin AW, Peters CA (eds): CAMPBELL-WALSH UROLOGY, ed 10. Philadelphia, Elsevier Saunders, 2012, vol 3, chap 92, p 2633.

2014 - 4 The effect of finasteride on serum and intraprostatic testosterone is: A. Serum Testosterone: <UP>; Intraprostatic Testosterone: <UP> B. Serum Testosterone: <UNCH>; Intraprostatic Testosterone: <UNCH> C. Serum Testosterone: <DOWN>; Intraprostatic Testosterone: <DOWN> D. Serum Testosterone: <UP>; Intraprostatic Testosterone: <DOWN> E. Serum Testosterone: <UNCH>; Intraprostatic Testosterone: <UP>

E Neoadjuvant platinum-based chemotherapy is indicated for patients with difficult to resect matted or fixed inguinal lymph nodes from metastatic penile cancer. The neoadjuvant chemotherapy should reduce the size of the nodes and enhance their resectability. Following neoadjuvant chemotherapy, the patient should have a bilateral inguinal and pelvic lymphadenectomy. A percutaneous biopsy is not necessary in this setting. Pettaway CA, Lance RS, Davis JW: Tumors of the penis, Wein AJ, Kavoussi LR, Novick AC, Partin AW, Peters CA (eds): CAMPBELL-WALSH UROLOGY, ed 10. Philadelphia, Elsevier Saunders, 2012, vol 1, chap 34, p 926.

2014 - 40 A 45-year-old man undergoes a partial penectomy for a 3 cm, grade 3, squamous cell carcinoma of the penis. Pathology reveals invasion of the corpus cavernosum and negative margins. On exam, he has matted firm 6 cm right inguinal lymph nodes and a CT scan of the chest, abdomen, and pelvis reveals right sided inguinal adenopathy but no other metastases. The next step is: A. percutaneous biopsy of the inguinal lymph nodes. B. right inguinal lymph node dissection. C. bilateral inguinal lymph node dissection. D. bilateral inguinal and pelvic lymph node dissection. E. neoadjuvant chemotherapy.

D Leukocytes are often present in the semen of infertile men. They are seldom due to documentable infection or antisperm antibodies. They do often indicate functional damage from DNA fragmentation to sperm membrane lipid peroxidation from reactive oxygen species released from the leukocytes. Leukocytes can occur with UTIs, but unless urine is in the semen, this is an unlikely source. Sexual transmitted infections can also lead to leukocytes in semen and this does need to be ruled-out, however, even PCR DNA testing for sexual transmitted pathogens are often negative verifying the situation is most commonly idiopathic. Sabanegh E, Agarwal A: Male infertility, Wein AJ, Kavoussi LR, Novick AC, Partin AW, Peters CA (eds): CAMPBELL-WALSH UROLOGY, ed 10. Philadelphia, Elsevier Saunders, 2012, vol 1, chap 21, p 622. Lackner JE, Herwig R, Schmidbauer J, et al. Correlation of leukocytospermia with clinical infection and the positive effect of anti-inflammatory treatment on semen quality. FERTIL STERIL 2006;86:601-605. Jarow J, Sigman M, Kolettis PN, et al: The optimal evaluation of the infertile male. AUA BEST PRACTICE STATEMENT. American Urological Association Education and Research, Inc, 2010. http://www.auanet.org/education/guidelines/male-infertility-d.cfm

2014 - 41 Pyospermia in an infertile man commonly suggests: A. a sexually transmitted infection. B. UTI. C. antisperm antibodies. D. functional sperm damage. E. failure to retract foreskin.

B The symptoms suggest either a urinoma or vascular injury to the lower pole segment, and is best evaluated with Doppler ultrasound. Pain, fever, and hematuria are common early, but should improve in the first 48-72 hours. Retrograde pyelogram and ureteral stent would be indicated for documented unresolving urinoma. CT angiogram has the disadvantage of radiation exposure without added clinical information that Doppler ultrasound would provide. Peters CA, Schlussel RN, Mendelsohn C: Ectopic ureter, ureterocele, and ureteral anomalies, Wein AJ, Kavoussi LR, Novick AC, Partin AW, Peters CA (eds): CAMPBELL-WALSH UROLOGY, ed 10. Philadelphia, Elsevier Saunders, 2012, vol 4, chap 121, p 3257.

2014 - 42 A three-year-old girl undergoes an upper pole heminephrectomy for an ectopic ureter. Three days later, she has worsening pain, fever, and hematuria. The next step is: A. observation. B. Doppler ultrasound. C. retrograde pyelogram. D. CT angiogram. E. ureteral stent.

A During placement of an obturator sling, regardless of approach, the trocar should pass through the ischiorectal fossa. It passes through the obturator foramen but not the obturator canal. It does not normally pass through any of the other listed structures. An outside-in transobturator passage traverses the gracilis muscle, adductor longus and brevis muscles, obturator externus muscle, obturator membrane, and obturator internus muscle. The gracilis muscle adducts and flexes the hip. The adductor longus and adductor brevis muscles adduct the thigh. The obturator externus and obturator internus muscles aid with thigh adduction and lateral rotation. Structures that affect medial thigh rotation are not traversed with a transobturator approach. Karram MM, Dmochowski RR, Reynolds WS: Transobturator synthetic midurethral slings, Karram MM (ed): SURGERY FOR URINARY INCONTINENCE: FEMALE PELVIC SURGERY VIDEO ATLAS SERIES, Philadelphia, Elsevier Saunders, 2013, chap 7, p 83. Dmochowski RR, Padmanabhan P, Scarpero HM: Slings: Autologous, biologic, synthetic, and midurethral, Wein, AJ, Kavoussi LR, Novick AC, Partin AW, Peters CA (eds): CAMPBELL-WALSH UROLOGY, ed 10. Philadelphia, Elsevier Saunders, 2012, vol 3, chap 73, p 2115.

2014 - 43 During performance of a transobturator midurethral polypropylene sling, the trocar normally passes through the: A. ischiorectal fossa. B. obturator canal. C. sartorius muscle. D. bulbospongiosus muscle. E. ischiocavernous muscle.

A Parenteral testosterone injection therapy (testosterone enanthate or cypionate) will cause significant peaks and valleys in serum testosterone levels which can cause mood swings and variations in libido and potency ("roller-coaster" effect). Oral, subcutaneous, and transdermal preparations do not have this "roller-coaster" effect. The alkylated oral androgens, e.g., fluoxymesterone, methyltestosterone, have serious liver toxicity and adverse effects on serum lipids (increased LDL, decreased HDL) and should not be used. Morales A: Androgen deficiency in the aging male, Wein, AJ, Kavoussi LR, Novick AC, Partin AW, Peters CA (eds): CAMPBELL-WALSH UROLOGY, ed 10. Philadelphia, Elsevier Saunders, 2012, vol 1, chap 29, pp 812, 817-819.

2015 - 47 During testosterone replacement therapy for androgen deficiency, significant mood swings, and variations in libido are most likely to develop when using: A. testosterone enanthate. B. testosterone gel. C. transdermal testosterone patch. D. methyltestosterone. E. subcutaneous testosterone pellets (Testopel^TM).

E In patients with an acute uncomplicated pyleonephritis, hospitalization is not required if they are able to maintain oral hydration. These patients should have a urine culture and sensitivity obtained and empirical antimicrobial therapy should be initiated immediately. Fluoroquinolones and TMP-SMX are excellent choices. Nitrofurantoin is effective in urine, but does not obtain therapeutic levels in most body tissues and is therefore not preferred for UTIs. Upper tract imaging is not required when uncomplicated pyelonephritis is suspected. However, imaging should be considered if the patient does not respond to antibiotic therapy within 72 hours, or in the presence of specific indications suggestive of a complicated UTI such as diabetes mellitus, immunosuppression, and history of nephrolithiasis or symptoms suggestive of urinary tract obstruction. Schaeffer AJ, Schaeffer EM: Infections of the urinary tract, Wein AJ, Kavoussi LR, Novick AC, Partin AW, Peters CA (eds): CAMPBELL-WALSH UROLOGY, ed 10. Philadelphia, Elsevier Saunders, 2012, vol 1, chap 10, p 292.

2014 - 44 A 45-year-old, otherwise healthy woman, has mild left CVA tenderness, bacteruria, and a temperature of 38.8_ C. She is able to take oral fluids. The next step is urine culture and: A. inpatient I.V. antibiotic therapy. B. abdominal and pelvic CT scan. C. renal ultrasonography. D. outpatient therapy with oral nitrofurantoin. E. outpatient therapy with an oral fluoroquinolone.

D The Clavien-Dindo classification of surgical complications is largely becoming the standard system for reporting adverse events after surgery that deviate from the normal postoperative course. As the patient required dialysis, due to single organ dysfunction, he would be considered grade IVa. The classification is: Grade I: Any deviation from the normal postoperative course without the need for pharmacological treatment or surgical, endoscopic, and radiological interventions. Grade II: Requiring pharmacological treatment with drug other than such allowed for grade 1 complications. (Includes blood transfusion and TPN) Grade III: Requiring surgical, endoscopic or radiological intervention. Grade IIIa: Intervention not under general anesthesia. Grade IIIb: Intervention under general anesthesia. Grade IV: Life-threatening complication. Grade IVa: Single organ dysfunction. Grade IVb: Multiorgan dysfunction. Dindo D, Demartines N, Clavien P: Classification of surgical complications. ANN SUR 2004;240:205-213.

2014 - 45 A 70-year-old man undergoes a difficult radical cystectomy for high-risk bladder cancer. His perioperative course is complicated by hypotension managed with crystalloid, two units of packed RBC, vasopressors, and intensive care monitoring. He later develops severe renal insufficiency requiring dialysis. According to the Clavien-Dindo Classification, his surgical complication grade is: A. II. B. IIIa. C. IIIb. D. IVa. E. V.

E Mid-urethral sling is warranted due to visualization of leak with Valsalva. Additionally, POP-Q exam reveals both posterior (Bp +2 notes most distal portion of posterior vaginal wall is 2 cm beyond the hymen) and apical prolapse (C at 0 notes vaginal cuff is at the hymen). The anterior vaginal wall is well supported (Aa and Ba -3). A posterior repair and procedure for apical prolapse is indicated. Kobashi KC: Evaluation of patients with urinary incontinence and pelvic prolapse, Wein AJ, Kavoussi LR, Novick AC, Partin AW, Peters CA (eds): CAMPBELL-WALSH UROLOGY, ed 10. Philadelphia, Elsevier Saunders, 2012, vol 3, chap 64, p 1900.

2014 - 46 A 57-year-old woman complains of stress urinary incontinence and pelvic pressure. On exam, she is noted to leak with Valsalva. Relevant pelvic organ prolapse quantification (POP-Q) exam points reveal Aa and Ba at -3, Ap at +1, Bp at +2, and C at 0. The correct surgical procedure is a mid-urethral sling: A. only. B. with anterior repair. C. with posterior repair. D. with anterior and apical vault repair. E. with posterior and apical vault repair.

B Bacterial vaginosis results from an alteration of normal vaginal flora with a predominance of Lactobacillus species and high concentrations of anaerobic bacteria. Diagnosis can be confirmed with identification of clue cells, a homogenous vaginal discharge, a vaginal pH > 4.5, and a malodorous fishy vaginal discharge. Risk factors may include multiple sexual partners, a new sexual partner, use of an intrauterine device, and douching. Treating the patient is as effective as treating the patient and partner. Symptoms may recur in 1/3 of patients after treatment. Metronidazole is the treatment of choice. Colli E, Landoni M, Parazzini F: Treatment of male partners and recurrence of bacterial vaginosis: A randomized trial. GENITOURIN MED 1997;73:267-270. Frenkl T, Potts J: Sexually transmitted diseases, Wein AJ, Kavoussi LR, Novick AC, Partin AW, Peters CA (eds): CAMPBELL-WALSH UROLOGY, ed 10. Philadelphia, Elsevier Saunders, 2012, vol 1, chap 13, p 415.

2014 - 47 A 32-year-old woman complains of a malodorous fishy vaginal discharge. She has a single male partner and uses an intrauterine device for contraception. The next step is: A. remove intrauterine device. B. metronidazole for patient. C. metronidazole for patient and partner. D. ciprofloxacin for patient. E. ciprofloxacin for patient and partner.

B When the semirigid ureteroscope will not pass easily over a guidewire in a prepubertal child, the next step is to perform a ureteric orifice dilation using either coaxial dilators or balloon dilator. Studies have demonstrated that ureteric dilation at the time of ureteroscopy is safe in children. After dilation, ureteral access sheath may also be used to minimize trauma. If the dilators do not pass easily, then placing a ureteral stent for passive dilation for several weeks is appropriate, but dilation should be tried first to avoid second anesthesia. Flexible ureteroscope is comparable in size (6.9 Fr) to the 6.5 Fr semirigid scope and may not offer any additional advantage in passing it up the ureter. Furthermore, a smaller working channel may make the stone fragmentation and extraction more difficult. SWL, percutaneous stone extraction, and laparoscopic ureterolithotomy are inappropriate for a distal ureteral stone. Ost MC, Schneck FX: Surgical management of pediatric stone disease, Wein AJ, Kavoussi LR, Novick AC, Partin AW, Peters CA (eds): CAMPBELL-WALSH UROLOGY, ed 10. Philadelphia, Elsevier Saunders, 2012, vol 4, chap 135, pp 3671-3678.

2014 - 48 A ten-year-old boy undergoes ureteroscopic stone extraction for a 1 cm right distal ureteral stone. After placing a guidewire, a 6.5 Fr semirigid ureteroscope will not pass easily at the ureteric orifice. The next step is: A. place ureteral stent and retry three days later. B. dilate the ureteric orifice. C. SWL. D. percutaneous stone extraction. E. laparoscopic ureterolithotomy.

B Many think that the clinical incidence of Peyronie's disease is increasing. The increase however may be associated and coincide with the increased use of erection-enhancing medications. Phosphodiesterase type 5 (PDE5) inhibitor medications are not contraindicated in the treatment of Peyronie's disease, while intracavernous injection therapy has been linked to the development of penile fibrosis. There has never been any suggestion that PDE5 inhibitors are in any way directly causally related to the development of Peyronie's disease, nor is there suggestion that their use would worsen the course of Peyronie's disease. Data is emerging to suggest that certain endothelial impairment may be reversed with the initiation of PDE5 inhibitor therapy. This patient has erectile dysfunction (ED) and it is reasonable to treat the ED with sildenafil. Reassurance will not improve his ED and surgical correction is not indicated early in the disease course. Jordan GH, McCammon KA: Peyronie's disease, Wein AJ, Kavoussi LR, Novick AC, Partin AW, Peters CA (eds): CAMPBELL-WALSH UROLOGY, ed 10. Philadelphia, Elsevier Saunders, 2012, vol 1, chap 28, p 801.

2014 - 49 A 53-year-old man complains of four months of pain with erections, poor tumescence, and a 30 degree dorsal penile curvature. Physical examination reveals a 1 cm dorsal plaque. The next step is: A. reassurance and observation. B. treatment with sildenafil. C. intracavernous injection therapy. D. tunical plication. E. plaque excision and grafting.

E This patient has several residual masses after salvage chemotherapy. Given that he has received salvage chemotherapy and that his markers are normal, his best option is RPLND. The chance of viable disease is approximately 50%, teratoma 40%, and necrosis/fibrosis only 10%, thus, observation is a poor choice. A PET scan would not tell about teratoma which would need to be resected. A percutaneous biopsy could have sampling errors and would not obviate the need for additional treatment. Since the markers are normal, there is no indication at this time for additional chemotherapy especially in the salvage setting. Sheinfeld J, Bosl GJ: Surgery of testicular tumors, Wein AJ, Kavoussi LR, Novick AC, Partin AW, Peters CA (eds): CAMPBELL-WALSH UROLOGY, ed 10. Philadelphia, Elsevier Saunders, 2012, vol 1, chap 32, p 887.

2014 - 5 A 21-year-old man receives a stem cell transplant with high-dose salvage chemotherapy for a Stage III NSGCT refractory to primary chemotherapy. After his salvage chemotherapy, his markers normalize and his CT scan is shown. The next step is: A. observation. B. PET scan. C. percutaneous biopsy. D. two additional cycles of chemotherapy. E. RPLND.

C The image intensifier should be positioned above the patient and the x-ray tube below the patient to minimize radiation exposure (Images A, B, C). The x-ray tube should be positioned as far from the patient as feasible. (Image C) Angulation of the C-arm to a lateral or oblique position (Images B, D) increase the dose rate due to the increased body mass thickness that must be penetrated, and also brings the x-ray tube closer to the patient. Wagner LK, Archer BR: Minimizing risks from fluoroscopic x rays: Bioeffects, instrumentation, and examination, PARTNERS IN RADIATION MANAGEMENT, ed 4. Houston, TX, 2004.

2014 - 50 A patient is undergoing fluoroscopy for a ureteroscopic procedure. The fluoroscopic set-up which will result in the least amount of scatter radiation to the operating room personnel is illustrated in the diagram labeled: A. A. B. B. C. C. D. D. E. E.

E Symptomatic lower calyceal stones clear poorly after fragmentation with SWL, especially in horseshoe kidneys. Ureteroscopic fragmentation and stone extraction in these malrotated kidneys can be difficult. Residual stone fragments are unlikely to pass spontaneously and the likelihood of rendering the patient stone-free is limited. Percutaneous renal access can be performed safely through a superior and posterior calyceal puncture. The entry site of the puncture needle is more medial, just lateral to the paraspinous musculature, in comparison to inferior pole punctures in normally positioned kidneys. An inferior calyceal puncture in a horseshoe kidney could be dangerous due to its medial, more anterior and inferior location. Even in thin patients, one may need to utilize longer instruments including flexible cystoscopes, flexible or rigid ureteroscopes, or extra-long nephroscopes. Percutaneous renal access through a superior pole puncture can be performed in a safe fashion and is the most optimal route to render the patient stone free. The only variable identified that decreases the efficacy of PCNL in a horseshoe kidney is the presence of a Staghorn calculus. Stone-free rates are 84% with a primary PCNL and rise to 93% after a second-look nephroscopy. Skolarikos A, Binbay M, Bisas A, et al: Percutaneous nephrolithotomy in horseshoe kidneys: Factors affecting stone-free rate. J UROL 2011;186:1894-1898. Miller NL, Matlaga BR, Handa SE, et al: The presence of horseshoe kidney does not affect the outcome of percutaneous nephrolithotomy. J ENDOUROL 2008;22:1219-1225.

2014 - 51 A 36-year-old woman with a horseshoe kidney has a symptomatic 1.8 cm left lower calyceal stone. The next step is: A. SWL. B. open nephrolithotomy. C. ureteroscopic stone extraction. D. ureteral stent and SWL. E. percutaneous nephrolithotomy.

B Poor support of the glans penis by cylinder or rod tips leads to a drooping appearance of the glans, in which it appears to flop over the prosthesis. This deformity may result from inadequate distal dilation, too short cylinders, or in the case of minor deformity, variations in anatomy where the corpora cavernosum does not extend completely under the glans. For a severe deformity, definitive correction involves removing both cylinders, perforating the distal corpora with scissors, resizing, and then inserting longer cylinders or the same cylinders with longer rear tip extenders. For mild defects, like in this patient, dorsal plication of the glans back onto the shaft of the penis (glansplasty) is preferable when there are minor but otherwise bothersome degrees of poor glanular support. Changing to a three-piece prosthesis is not needed and adding a larger cylinder (upsizing) could result in pain and buckling. Montague DK: Prosthetic surgery for erectile dysfunction, Wein AJ, Kavoussi LR, Novick AC, Partin AW, Peters CA (eds): CAMPBELL-WALSH UROLOGY, ed 10. Philadelphia, Elsevier Saunders, 2012, vol 1, chap 27,

2014 - 52 During placement of a two-piece inflatable penile prosthesis, the glans penis is noted to have poor support and minimal movement over the top of the prosthetic with a mild SST deformity. The best treatment is: A. observation and cycling. B. glansplasty. C. placement of an additional 1 cm rear tip extender. D. upsize the prosthesis. E. convert to a three-piece inflatable prosthesis.

D This patient's persistent, post-obstructive diuresis is most likely iatrogenic. The brisk diuresis that accompanies release of obstruction may represent a physiologic response to the expansion of extracellular fluid volume occurring during the period of obstruction in many patients. This may be perpetuated by overzealous administration of intravenous fluids after relief of the obstruction. It would be safe in this clinically stable, neurologically intact patient to decrease intravenous fluids, allow her free access to fluid, and observe her closely for signs of hypovolemia and electrolyte imbalance. Hydrochlorothiazide is a diuretic and may worsen her diuresis. Vasopressin (ADH) acts to increase water reabsorption at the collecting duct and is not indicated in this scenario. Singh I, Strandhoy JW, Assimos DG: Pathophysiology of urinary tract obstruction, Wein AJ, Kavoussi LR, Novick AC, Partin AW, Peters CA (eds): CAMPBELL-WALSH UROLOGY, ed 10. Philadelphia, Elsevier Saunders, 2012, vol 2, chap 40, pp 1107-1108.

2014 - 53 A 50-year-old woman has azotemia and an obstructed solitary kidney. A percutaneous nephrostomy is placed and urine output is initially 200 ml/hr. She receives I.V. fluids. Her renal function improves over the next four days but the diuresis is now more pronounced. Vital signs and neurocognitive exam are normal. The next step is: A. oral vasopressin. B. I.V. vasopressin. C. hydrochlorothiazide. D. decrease I.V. fluids. E. increase I.V. fluids.

E The clinical presentation is that of weakness and lethargy following the course of chemotherapy accompanied by significant hypokalemia and hyperchloremic metabolic acidosis. Despite the acidosis, his urinary pH is alkaline, indicating inability to acidify urine. The most likely diagnosis is a drug-induced renal tubular acidosis (RTA) type I (distal), in which there is a failure of ammonium secretion in the distal tubule. The type II (proximal) RTA represents a defective reabsorption of bicarbonate in the proximal tubule. Although distal RTA can be genetic, this most likely represents an acquired condition due to ifosfamide chemotherapy. The treatment is oral potassium and bicarbonate supplementation. I.V. fluid hydration is unnecessary with no evidence of dehydration, and normal creatinine with no evidence of contraction alkalosis. The use of steroids or diuretics would be detrimental to this patient. Pearle MS, Lotan Y: Urinary lithiasis: Etiology, epidemiology, and pathogenesis, Wein AJ, Kavoussi LR, Novick AC, Partin AW, Peters CA (eds): CAMPBELL-WALSH UROLOGY, ed 10. Philadelphia, Elsevier Saunders, 2012, vol 2, chap 45, p 1275.

2014 - 54 A 22-year-old man with history of NSGCT completed chemotherapy that included ifosfamide two weeks ago. He has weakness and lethargy. Serum labs reveal Na 137 mEq/l, Cl 135 mEq/l, CO2 12 mEq/l, K 2.7 mEq/l, and creatinine 0.9 mg/dl. Blood gas reveals serum pH 7.3. Urinalysis reveals pH 7.6. The next step is: A. I.V. Ringer's lactate. B. I.V. D5W with 20 mEq/l KCL. C. I.V. hydrocortisone. D. oral spironolactone. E. oral potassium citrate.

B This patient has developed a high output (> 500 ml per day) enterocutaneous (EC) fistula. Management of high output EC fistula when they develop is extremely challenging and complex due to significant fluid and electrolyte loss and coexisting malnutrition. Emergent surgical repair in this patient would be highly technically demanding due to significant perisurgical reactive fibrosis with healing greatly impaired by concurrent malnutrition. Indeed up to 70% of patients with high output fistulae have concurrent malnutrition with documented hypoalbumemia, the latter of which is a significant prognostic factor. The loss of large amounts of fluid in the immediate postoperative period can lead to severe dehydration with oral intake frequently worsening the high output. The immediate step is to discontinue oral intake and initiate TPN. Agents that can decrease bowel motility (loperamide, diphenoxylate/atropine, etc.) can be helpful if the high output continues after the initial steps are taken. Although octreotide can be used to decrease pancreatic secretions, prospective double blinded placebo controlled trials demonstrated no significant reduction in postoperative EC fistula fluid losses nor did they find an increased rate of spontaneous EC closure. It should therefore only be used if high EC fistula outputs continue after initial measures fail. Pritts TA, Fischer DR, Fischer JE: Postoperative enterocutaneous fistula. SURGICAL TREATMENT: EVIDENCE BASED AND PROBLEM ORIENTED. Munich Zuckschwerdt, 2001. Dahl DM, McDougal WS: Use of intestinal segments in urinary diversion, Wein, AJ, Kavoussi LR, Novick AC, Partin AW, Peters CA (eds): CAMPBELL-WALSH UROLOGY, ed 10. Philadelphia, Elsevier Saunders, 2012, vol 3, chap 85, p 2411. Schecter WP: Management of enterocutaneous fistulas. SUR CLIN N AM 2001;91:481-491. Scott NA, Finnegan S, Irving, MH: Octreotide and postoperative enterocutaneous fistulae: A controlled prospective study. ACTA GASTRO-ENEROLOGIC BELICA 1993;56:266-270.

2014 - 55 A 64-year-old man has a radical cystectomy and ileal neobladder. Ten days after surgery, 2.5 liters of bilious fluid drains from his incision over 24 hours. He is otherwise stable. The next step is: A. observation. B. discontinue diet and start TPN. C. oral loperamide. D. oral octreotide. E. surgical repair.

C The non-medical use of anabolic steroids by athletes is potentially a significant cause of infertility in male adolescents and adults. It has been estimated that 3-12% of male athletes of high school age in the United States have used steroids. Infertility associated with anabolic steroid use commonly presents as oligospermia or azoospermia along with abnormalities of sperm motility and morphology. As with many of the other steroid side effects, the semen parameter deficits are thought to be reversible and consequently, the discontinuation of all steroids is the first course of therapy. Unfortunately, some steroid abusers develop a chronic anabolic-induced hypogonadism syndrome/state. Successful gonadotropin (hCG) replacement is the best therapy when this occurs. This patient has already been off steroids with no return of sperm after one year, therefore, continued observation is not likely to work at this point and a trial of hCG therapy is indicated. Testosterone supplementation will only continue to suppress the central axis (decrease FSH and LH release) and perpetuate the problem. Clomiphene citrate might work by blocking testosterone and estradiol feedback on the central axis and stimulate LH/FSH release, but will not be as effective as hCG. Testis biopsy is not needed at this time, but might be needed if the patient does not respond to hormonal manipulation and maturation arrest is suspected. Fronczak CM, Kim ED, Barqawi AB: The insults of illicit drug use on male fertility. J ANDROL 2012;33:515-528. de Souza GL, Hallak J: Anabolic steroids and male infertility: A comprehensive review. BJU INT 2011;108:1860-1865. Sabanegh E, Agarwal A: Male infertility, Wein, AJ, Kavoussi LR, Novick AC, Partin AW, Peters CA (eds): CAMPBELL-WALSH UROLOGY, ed 10. Philadelphia, Elsevier Saunders, 2012, vol 1, chap 21, p 639.

2014 - 56 A 26-year-old infertile body builder with a five year history of heavy anabolic steroid use is azoospermic 12 months after discontinuing all steroids. His serum testosterone is 150 ng/dl and FSH is normal. The next step is: A. observation. B. testosterone replacement. C. hCG. D. testicular biopsy. E. clomiphene citrate.

B Renal venous thrombosis is suggested in the neonate with enlarged kidneys, hematuria, anemia, and thrombocytopenia, often with a history of prolonged delivery and prematurity. Thrombosis is peripheral and initial therapy should be aimed at support and I.V. fluid administration in order to treat the underlying dehydration. Observation would be inadequate with the thrombotic phenomenon likely to escalate in severity with extension of the thrombus. Anticoagulation is indicated if the process extends beyond the renal vein(s). The clinical scenario presented and the imaging reported of bilaterally enlarged kidneys on ultrasound are sufficient to make the diagnosis and no further imaging is necessary. Lee RS, Borer JG: Perinatal urology, Wein AJ, Kavoussi LR, Novick AC, Partin AW, Peters CA (eds): CAMPBELL-WALSH UROLOGY, ed 10. Philadelphia, Elsevier Saunders, 2012, vol 4, chap 114, p 3060.

2014 - 57 A two-day-old boy has gross hematuria. He was born at 38 weeks gestation via Caesarian section for failure to progress. Ultrasound shows bilaterally enlarged kidneys without cysts or hydronephrosis and a normal bladder. Hemoglobin and platelet count are low. The next step is: A. observation. B. I.V. hydration. C. anticoagulation. D. VCUG. E. CT angiogram.

D This patient has failed two previous attempts at fistula closure following a complicated surgery. One would expect significant scarring in her pelvis and anticipate a difficult dissection for exposure of the fistula. Although cystoscopy and fulguration of the fistula tract is appealing for its surgical ease and is associated with a high rate of success (> 66%) for fistula < 7 mm in size, when used as either a primary or secondary treatment modality, it has no role in the management of large > 7 mm vesicovaginal fistula. Retrograde endoscopic injection of fibrin glue is a minimally invasive approach that may avoid the morbidity of an open surgery. It has been used to close complex fistulas up to 15 mm in size. Although short term outcomes less than one year are encouraging, approximately 75% successful long-term follow-ups reveal the success of this technique breaks down with time. Whether late break downs can be successfully retreated with fibrin glue injection remains to be clarified. Clearly, with each subsequent surgery, the likelihood of successful repair decreases. Optimal success is achieved with a tension-free primary bladder closure with an interpositional graft between the bladder and vaginal vault. Of the choices, the abdominal approach with rectus muscle graft interposition would provide the best technical option and be considered the gold standard in a patient who has undergone repeated failed attempts at vesicovaginal fistula closure. An ileal conduit is not indicated in this otherwise healthy young woman. Rovner ES: Urinary tract fistulae, Wein AJ, Kavoussi LR, Novick AC, Partin AW, Peters CA (eds): CAMPBELL-WALSH UROLOGY, ed 10. Philadelphia, Elsevier Saunders, 2012, vol 3, chap 77, p 2241. Stovsky MD, Ignatoff JM, Blum MD, et al: Use of electrocoagulation in the treatment of vesicovaginal fistulas. J UROL 1994;152:1443-1444. Shah SJ: Role of day care vesicovaginal fistula fulguration in small vesicovaginal fistula. J ENDOUROL 2010;10:1659-1660. Shivan MK, Alamdari DH, Ghorefi A: A novel method for iatrogenic vesicovaginal fistula treatment. J UROL 2013;189:2125-2129. Safan A, Shaker H, Abdelaal A, et al: Fibrin glue versus martius flap interposition in the repair of complicated obstetric vesicovaginal fistula: A prospective multi-institutional randomized trial. NEUROUROL URODYN 2009;28:438-441.

2014 - 58 A 38-year-old woman developed a 10 mm vesicovaginal fistula following a complicated hysterectomy. She has failed two prior attempts at repair, one transvaginally using a Martius flap and the other transabdominally using an omental flap. Upper tracts are normal. The next step is: A. cystoscopy, fulguration of the fistula tract, and suprapubic tube drainage. B. cystoscopy with fibrin glue application to the fistula. C. repeat vaginal repair with contralateral Martius fat pad graft. D. abdominal repair with rectus muscle interposition. E. ileal conduit.

E The history and physical examination of this patient is highly suspicious for a testis fracture. Immediate exploration is indicated without the need for scrotal imaging for a definitive diagnosis. Indeed, ultrasonography should not deny or delay surgical exploration if the physical examination findings dictate a possible fracture. The overall accuracy of ultrasonography for detection of traumatic testis fracture has been found to be highly operator dependent with both significant false positive and false negative rates found in the published literature. MRI scan has not been extensively studied, is costly, and could delay definitive treatment. Nuclear medicine scan will delay diagnosis and will not add useful information. Morey AF, Dugi DD III: Genital and lower urinary tract trauma, Wein AJ, Kavoussi LR, Novick AC, Partin AW, Peters CA (eds): CAMPBELL-WALSH UROLOGY, ed 10. Philadelphia, Elsevier Saunders, 2012, vol 3, chap 88, p 2510.

2014 - 59 A 19-year-old man has painful severe left scrotal swelling after being kicked. The right testicle is palpably normal, but the left testicle and epididymis are nonpalpable secondary to a left hemiscrotal hematoma. The next step is: A. observation. B. nuclear medicine scan. C. MRI scan. D. testicular ultrasound. E. surgical exploration.

A Reservoirs made of detubularized ileum appear to have the greatest compliance and lowest likelihood of generating intermittent high-pressure contractions. Several clinical studies have demonstrated that the urodynamic characteristics of the ileum appear to be superior to those of the colon and is therefore the preferred segment of bowel used for a neobladder. Larger bowel lengths with increased intra-operative volumes are not necessary as all bowel segments effectively stretch over time if there is adequate outflow resistance. In fact, commonly utilized techniques (e.g. Studer, T-pouch) utilize 40-44 cm of ileum with resultant intra-operative volumes of 200 ml or less. For ileal neobladders, it has been shown that the capacity increases sevenfold after one year. The true benefits of anti-refluxing anastomosis remain uncertain. It does not appear that conduit pressures are transmitted to the renal pelvis. Also, there is no difference in conduits between those with versus without reflux, with regard to renal function measured two to five years postoperatively. Furthermore, the successful construction of an anti-refluxing anastomosis does not prevent bacterial colonization of the renal pelvis. Many of these patients have no untoward effects and seem to do well with chronic bacteriuria. Deterioration of the upper tracts is more likely when the culture becomes dominant for Proteus or Pseudomonas, and should therefore be treated, whereas those with mixed cultures may generally be observed, provided they are not symptomatic. Dahl DM, McDougal WS: Use of intestinal segments in urinary diversion, Wein AJ, Kavoussi LR, Novick AC, Partin AW, Peters CA (eds): CAMPBELL-WALSH UROLOGY, ed 10. Philadelphia, Elsevier Saunders, 2012, vol 3, chap 85, p 2411. Skinner EC, Skinner DG, Stein JP: Orthotopic urinary diversion, Wein AJ, Kavoussi LR, Novick AC, Partin AW, Peters CA (eds): CAMPBELL-WALSH UROLOGY, ed 10. Philadelphia, Elsevier Saunders, 2012, vol 3, chap 87, p 2479.

2014 - 6 A 56-year-old man undergoes a radical cystoprostatectomy and orthotopic neobladder. Long-term preservation of renal function is most dependent on: A. preferential use of ileum over colon. B. use of > 60 cm of detubularized bowel. C. performance of an antirefluxing ureteroileal anastomosis. D. intraoperative neobladder capacity of >= 500 ml. E. postoperative avoidance of any bacteriuria.

B According to the most recently published AUA Guideline for the Management of VUR, the required step after an open-robotic surgical intervention for VUR is renal ultrasound. Ureteral obstruction can be clinically silent, and therefore, the absence of ureteral obstruction must be documented. Based on the rigorous meta-analysis performed on the available literature, the use of postoperative VCUG following an open robotic ureteral reimplantation is considered optional. Renal ultrasonography obtained postoperatively will reveal Society of Fetal Urology (SFU) Grade 2 or higher hydronephrosis in approximately 25% of patients one month after a ureteral reimplantation and in 15% of patients three months after a ureteral reimplantation. The exact percentage of patients with postoperative hydronephrosis is dependent upon the percentage of patients with preoperative hydronephrosis, and the percentage of patents with preoperative grade of 4-5/5 VUR. If SFU Grade 2 or higher, hydronephrosis was noted preoperatively, approximately 60% resolve their hydronephrosis by three months, 30% improve, and the remainder are unchanged to slightly worsened. MAG-3 renal scan and VCUG can be obtained in individuals with persistent or worsening hydronephrosis. Evaluation of these patients will reveal persistent VUR in 4-8%, ureteral obstruction and/or bladder dysfunction (noncompliance) noted to be the cause of the hydronephrosis in approximately 1-2%. It is noteworthy that following injection of a bulking agent for treatment of VUR both a VCUG along with a renal ultrasound is recommended. DMSA scan is optional if additional febrile UTI's should occur. Peters CA, Skoog SJ, Arant BS Jr, et al: Management and screening of primary vesicoureteral reflux in children: AUA guideline. VESICOURETERAL REFLUX. American Urological Association Education and Research, Inc, 2010. http://www.auanet.org/education/guidelines/vesicoureteral-reflux-a.cfm Aboutaleb H, Bolduc S, Bagli DJ, et al: Correlation of vesicoureteral reflux with degree of hydronephrosis and the impact of antireflux surgery. J UROL 2003,170:1560-1562. Ellsworth PI, Freilich DA, Lahey S: Cohen cross-trigonal ureteral reimplantation: Is a one-year postoperative renal ultrasound scan necessary after normal initial postoperative ultrasound findings? UROL 2008;71:1055-1058. Charbonneau SG, Tackett LD, Gray EH, et al: Is long-term sonographic followup necessary after uncomplicated ureteral reimplantation in children? J UROL 2005;174:1429-1431.

2014 - 60 A four-year-old girl with bilateral grade 3 VUR has recurrent breakthrough UTIs despite continuous antibiotic prophylaxis. She undergoes a bilateral ureteroneocystostomy. According to the 2010 AUA Guideline on vesicoureteral reflux, the required step in postoperative management is: A. observation, postoperative imaging only required if additional UTIs occur. B. renal ultrasound. C. VCUG. D. DMSA renal scan. E. MAG-3 diuretic enhanced renal scan.

B Following hysterectomy, a high-riding vesicovaginal fistula (VVF) can often be repaired through a vaginal approach. A simple procedure to augment the repair is harvest of the posterior peritoneum which is often present around the cervical cuff. This can be carried up over the two-layer fistula closure and attached to cover the fistula. Omentum can easily be harvested through a trans-abdominal approach but is difficult with a vaginal incision. Myocutaneous and Martius flaps are used with extensive fistulas and are more challenging to harvest and transpose. Rovner ES: Urinary tract fistulae, Wein AJ, Kavoussi LR, Novick AC, Partin AW, Peters CA (eds): CAMPBELL-WALSH UROLOGY, ed 10. Philadelphia, Elsevier Saunders, 2012, vol 3, chap 77, p 2240.

2014 - 61 During vaginal repair of a high-riding post-hysterectomy vesicovaginal fistula, the easiest tissue to interpose as a flap is: A. anterior peritoneum. B. posterior peritoneum. C. omentum. D. Martius fat pad graft. E. gracilis.

D Colonic perforation is a rare complication of nephrostomy tube placement, occurring in < 1% of patients. It most commonly occurs with lower pole nephrostomy tube placement in patients with malrotated kidneys and/or distended colons. Laparotomy is not necessary in the absence of clinical signs of peritonitis. Separate drainage of the colon and urinary system should resolve this problem. Gerspach JM, Bellman GC, Stoller ML, et al: Conservative management of colon injury following percutaneous renal surgery. J UROL 1997;49:831-836. El-Nahas AR, Shokeir AA, El-Assmy AM, et al: Colonic perforation during percutaneous nephrolithotomy: Study of risk factors. J UROL 2006;67:937-941.

2014 - 62 A 42-year-old man has persistent ileus and fever after left PCNL. Nephrostogram demonstrates that the nephrostomy tube has traversed the colon in its passage into the renal pelvis. The next step is I.V. antibiotics and: A. colostomy and drainage of the area. B. colonic repair and drainage of the area. C. removal of the nephrostomy tube. D. withdraw nephrostomy into colon and place ureteral stent. E. replace nephrostomy tube in a more posterior position.

E Bladder neck injuries in the setting of a pelvic fracture are more common in children than in adults and are frequently associated with injuries to the vagina and rectum. Immediate repair of the bladder neck, urethra, and vagina, and diverting colostomy with rectal repair is recommended. Attempts at management with a temporary suprapubic tube, urethral catheters, and delayed repair is associated with a high incidence of persistent bladder neck incompetence, urinary fistula, pelvic abscess, and osteomyelitis. Thus, in a hemodynamically stable patient, immediate repair of all injuries through a combined suprapubic and perineal approach is recommended. Primary diversion with either percutaneous nephrostomy tubes or externalized ureteral stents and urinary catheters is appropriate when the patient is hemodynamically unstable and blood loss is life threatening. Husmann DA: Pediatric genitourinary trauma, Wein AJ, Kavoussi LR, Novick AC, Partin AW, Peters CA (eds): CAMPBELL-WALSH UROLOGY, ed 10. Philadelphia, Elsevier Saunders, 2012, vol 4, chap 138, pp 3746-3749.

2014 - 63 A six-year-old girl has gross hematuria after an MVC. CT scan of the abdomen shows a pelvic fracture and no evidence of renal or intra-abdominal injury. She is hemodynamically stable. Attempts to pass a urethral catheter are unsuccessful. Endoscopy reveals a tear through the bladder neck, vaginal laceration, and anterior rectal wall injury. The next steps are colostomy, suprapubic tube, and: A. observation. B. urethral catheter. C. urethral catheter and extraperitoneal drain. D. urethral catheter, repair of vagina and rectum. E. repair of bladder neck, vagina, and rectum.

A Dysfunctional voiding is characterized by involuntary contraction of the pelvic floor during voiding in a neurologically intact individual. This is clearly demonstrated by the increased EMG activity during voiding seen on this pressure flow urodynamic tracing. Treatment generally starts with conservative measures including pelvic floor physical therapy with biofeedback. Other theoretical options to consider include pharmacologic therapy (Valium or Baclofen) to relax the external sphincter. Some authors have reported success with the use of alpha-blockade, but theoretically, one would not expect an improvement in voiding with a medication that would more likely affect the internal sphincter/bladder neck than the pelvic floor. Urethral dilation is not widely advocated. OnabotulinumtoxinA injection and sphincterotomy, while targeting the proper area, would be very aggressive and certainly not recommended in a neurologically healthy patient as a first line treatment. Bladder neck incision would not treat pelvic floor dysfunction. Wein AJ, Dmochowski RR: Neuromuscular dysfunction of the lower urinary tract, Wein AJ, Kavoussi LR, Novick AC, Partin AW, Peters CA (eds): CAMPBELL-WALSH UROLOGY, ed 10. Philadelphia, Elsevier Saunders, 2012, vol 3, chap 65, pp 1938-1939.

2014 - 64 A healthy 23-year-old woman has urinary hesitancy, urgency, a slow stream, and incomplete bladder emptying. A pressure flow urodynamic study reveals a compliant bladder with end fill detrusor pressures of 2 cm H2O at 350 ml capacity. There is pelvic floor activity noted on the EMG during voiding, with peak detrusor pressures of 50 cm H2O at the time of peak flow. Total volume voided was 150 ml, peak uroflow of 4 ml/min, average uroflow of 2 ml/min, and postvoid residual of 200 ml. The next step is: A. physical therapy with biofeedback. B. urethral dilation. C. onabotulinumtoxinA injection of sphincter. D. bladder neck incision. E. sphincterotomy.

B Calcium homeostasis involves four key components: serum calcium, serum phosphate (binds calcium thereby blocking calcium bioavailability), 1,25-dihydroxyvitamin D3, and parathyroid hormone (PTH). The major target organs for PTH action are the kidneys, skeletal system, and intestine. The primary effect of PTH is in the kidney, where it will act to block phosphate reabsorption in the proximal tubule (i.e., increase phosphate excretion) and promote calcium reabsorption in the ascending loop of Henle, distal tubule, and collecting tubule. PTH promotes bone resorption with subsequent calcium release. The final function of parathyroid hormone (PTH) is by activation of the enzyme 1-hydroxylase in the proximal tubules of the kidney. The enzyme 1-hydroxylase is a key component in the metabolism of vitamin D3 (cholecalciferol). Vitamin D3 (cholecalciferol) is formed in the skin when a cholesterol precursor, 7-dehydroxycholesterol, is exposed to ultraviolet light. Activation of vitamin D3 occurs when the substance undergoes 25-hydroxylation in the liver and 1-hydroxylation in the kidney. The enzyme 1-hydroxylase converts 25-hydroxyvitamin D3 to its most active metabolite, 1,25-dihydroxyvitamin D3. The primary action of 1,25-dihydroxyvitamin D3 is to promote gut absorption of calcium by stimulating formation of calcium-binding protein within the intestinal epithelial cells. Shoskes DA, McMahon AW: Renal physiology and pathophysiology, Wein AJ, Kavoussi LR, Novick AC, Partin AW, Peters CA (eds): CAMPBELL-WALSH UROLOGY, ed 10. Philadelphia, Elsevier Saunders, 2012, vol 2, chap 38, pp 1030-1031.

2014 - 65 The primary effect of parathyroid hormone (PTH) is to facilitate: A. 1,25-dihydroxyvitamin D3 activity on the enteric epithelium. B. calcium reabsorption in the renal tubule. C. enzymatic activity of 25-hydroxylase. D. conversion of 7-dehydroxycholesterol to Vitamin D3 (cholecalciferol). E. 1,25-dihydroxyvitamin D3 activity on the renal tubule.

C In smaller patients with little perivesical fat and less pelvic vascular development, finding the ureter extravesically is fairly straight forward. For older patients and unusual cases, this can be challenging. The ureter courses dorsal to the obliterated umbilical artery, so searching its ventral surface will not be effective. Finding the vas deferens is difficult and may result in damaging it, so this is also not advisable. The ureter may occasionally be adherent to the peritoneum as it is dissected, making this a low yield place to look initially. Extensive dissection of the bladder should be avoided as it may increase the risk of postoperative voiding dysfunction and urinary retention, especially if bilateral reimplant is performed. Dissecting slightly higher up to the level of the iliac vessels, one should be able to find the ureter as it courses on the ventral surface of both the artery and vein. It can then be followed distally to complete the procedure. Khoury AE, Bagli DJ: Vesicoureteral reflux, Wein AJ, Kavoussi LR, Novick AC, Partin AW, Peters CA (eds): CAMPBELL-WALSH UROLOGY, ed 10. Philadelphia, Elsevier Saunders, 2012, vol 4, chap 122, p 3301.

2014 - 66 The best way to identify a ureter that proves difficult to locate during the course of an extravesical reimplant is to: A. identify the vas deferens and follow this laterally. B. search ventral to the obliterated umbilical artery. C. dissect along the ventral surface of the iliac vessels. D. search the dorsal surface of the mobilized peritoneum. E. dissect the lateral surface of the bladder.

B Rapid short pelvic contractions (termed, quick flicks ) have been shown to decrease bladder overactivity and urinary urgency. This treatment is combined with teaching the individual to delay voiding in response to the urge to void. The principles behind these bladder drills are an attempt to retrain the bladder to overcome the voiding urgency. Patients are taught not to void just in case or when the urge hits, but rather to try to delay the void. Slow sustained pelvic floor contractions are utilized in the treatment of stress incontinence and are not as helpful in patients with urinary urgency. With a single negative urinalysis, a repeat urinalysis is reasonable, but a routine culture is not necessary at this point if the urinalysis remains negative. For persistent irritative symptoms, consideration of mycoplasma/ureaplasma cultures, and/or cystoscopy if the patient would have a history of tobacco usage may be reasonable. An anti-incontinence pessary would not be helpful for urgency incontinence and might actually exacerbate the symptoms. Payne CK: Conservative management of urinary incontinence: Behavioral and pelvic floor therapy, urethral and pelvic devices, Wein AJ, Kavoussi LR, Novick AC, Partin AW, Peters CA (eds): CAMPBELL-WALSH UROLOGY, ed 10. Philadelphia, Elsevier Saunders, 2012, vol 3, chap 69, p 2033.

2014 - 67 A 37-year-old woman has urinary urgency that interrupts her work day. Urinalysis is normal. She does not wish to take medications. In addition to avoiding dietary bladder irritants, she should: A. void as soon as she feels the urge. B. perform repetitive rapid pelvic contractions (quick flicks) when she has a sense of urgency. C. perform slow, sustained pelvic floor contractions multiple times a day. D. obtain a urine culture. E. use an anti-incontinence pessary.

C Alpha-mercaptopropionylglycine (alpha-MPG) is an oral agent, first introduced in 1968, which reduces cystine stone formation. Approximately 25% of the oral dose appears in the urine and like penicillamine, participates in thiol-disulfide exchange with cystine, thus increasing the latter's solubility. Alpha-MPG appears to be as effective as penicillamine in reducing cystine excretion and significantly reduces the individual stone formation rate, while entailing less significant toxicity. N-acetylcysteine is not effective orally. Tris-hydroxymethyl amino methane is only used for percutaneous dissolution. Acetohydroxamic acid inhibits bacterial urease and is not indicated for cystine stones. Moe OW, Pearle MS, Sakhaee K: Pharmacotherapy of urolithiasis: Evidence from clinical trials. KIDNEY INT 2011;79:385-392. Claes DJ, Jackson E: Cystinuria: Mechanisms and management. PED NEPHROL 2012;27:2031-2038.

2014 - 68 Patients with cystine stones unresponsive to hydration and alkalization are best treated with: A. N-acetylcysteine. B. tris-hydroxymethyl amino methane (THAM). C. alpha-mercaptopropionylglycine. D. penicillamine. E. acetohydroxamic acid.

B Newborns may have a buried penis in which there is poor fixation of the overlying skin at the base of the penis. When they are circumcised with a newborn clamp technique, a trapped penis can develop in which the healing skin edges migrate distally over the glans, which in turn can cause an obstructing cicatrix. Resultant retention of voided urine can cause of ballooning of the penile skin. In severe cases, this can also give rise to significant infections and possible urinary retention. Conservative management with a topical steroid ointment (0.05-0.1% betamethasone) for six weeks will result in resolution of the cicatrix and spontaneous retraction of the overlying skin in up to two thirds of patients. In unresponsive cases or patients with more acute problems such as infection or retention, one can consider use of antibiotics and/or surgical intervention with a dorsal slit. If possible, one should avoid performing a revision circumcision in the acute setting because of the poor results that can result from reconstruction of the inflamed skin. Forced manual retraction should be avoided since there will be a significant risk for increased scarring and recurrence. Palmer JS, Elder JS, Palmer LS: The use of betamethasone to manage the trapped penis following neonatal circumcision. J UROL 2005;174:1577-1578. Palmer JS: Abnormalities of the external genitalia in boys, Wein AJ, Kavoussi LR, Novick AC, Partin AW, Peters CA (eds): CAMPBELL-WALSH UROLOGY, ed 10. Philadelphia, Elsevier Saunders, 2012, vol 4, chap 131, pp 3540-3541.

2014 - 69 Two weeks following a circumcision, a newborn has progressive swelling and ballooning of the penile skin with voiding. Physical exam reveals a trapped penis in which the glans is not visible and the overlying skin cannot be reduced. The next step is: A. observation and parental reassurance. B. topical steroid cream. C. topical anesthetic and manual retraction of penile skin. D. dorsal slit. E. revision circumcision.

E The clinical scenario described is ischemic acute tubular necrosis (ATN). This is characterized by tubular cell injury which may be sublethal or lethal. During normal renal function, the medulla operates at the brink of hypoxia due to countercurrent diffusion of oxygen in the vasa rectae. During prolonged ischemia, medullary hypoxia is intensified and high metabolic requirements of the structures located in the outer medulla are most sensitive to injury. The medullary thick ascending limb of Henle is rich in the energy requiring Na+-K+ ATPase and is most sensitive to ischemic damage. The concept of ATN is important in partial nephrectomies and renal transplantation. The other structures are not located in the medulla. Management of ATN would include optimizing perfusion and oxygenation of the kidney, and minimizing nephrotoxic agents. Goldfarb DA, Poggio ED: Etiology, pathogenesis, and management of renal failure, Wein AJ, Kavoussi LR, Novick AC, Partin AW, Peters CA (eds): CAMPBELL-WALSH UROLOGY, ed 10. Philadelphia, Elsevier Saunders, 2012, vol 2, chap 43, p 1200.

2014 - 7 A 63-year-old man undergoes partial nephrectomy in a solitary kidney. The renal artery is clamped for 40 minutes without the use of surface hypothermia. Postoperatively, the creatinine rises from 1.5 mg/dl to 2.5 mg/dl. The renal structure most likely to have been injured is: A. cortical collecting duct. B. distal convoluted tubule. C. juxtaglomerular apparatus. D. proximal convoluted tubule. E. medullary thick ascending loop of Henle.

E The possibility of the presence of intrinsic sphincter deficiency (ISD) should be considered in patients with a history of recurrent stress urinary incontinence following anti-incontinence surgery. In the face of urethral hypermobility, a retropubic suspension (e.g. Burch) may be an option, but a pubovaginal sling is more effective and durable for ISD. Injection therapy would also be reasonable, but will likely not be as durable as a sling procedure in this relatively young woman. Pelvic floor muscle therapy is a consideration but is known to be less effective than surgery and is best used in patients with mild leakage. Imipramine is off-label and has the potential to cause cardiac effects. Although it may improve her continence, it is also unlikely to result in complete resolution of her symptoms. Dmochowski RR, Padmanabhan P, Scarpero HM: Slings: Autologous, biologic, synthetic, and midurethral, Wein, AJ, Kavoussi LR, Novick AC, Partin AW, Peters CA (eds): CAMPBELL-WALSH UROLOGY, ed 10. Philadelphia, Elsevier Saunders, 2012, vol 3, chap 73, pp 2115-2116.

2014 - 70 A 48-year-old woman develops recurrent stress urinary incontinence one year following a midurethral sling. Pelvic examination reveals mild urethral hypermobility and urodynamics shows a Valsalva LPP of 40 cm H2O. The next step is: A. pelvic floor muscle therapy. B. off-label imipramine. C. urethral bulking injection therapy. D. retropubic bladder neck suspension (Burch). E. pubovaginal sling.

C Hidradenitis suppurativa is an inflammatory process involving the apocrine glands, most commonly in the groin, axillae, and buttock or below the breasts. They are sterile abscesses. Old scars and sinus tracts are common. Fistulae tracts and perineal abscesses are rare findings in inflammatory bowel disease. Syphilis and chancroid both present with superficial ulcerations. Granuloma inguinale presents with painful lymphadenopathy without abscesses. Link RE: Cutaneous diseases of the external genitalia, Wein AJ, Kavoussi LR, Novick AC, Partin AW, Peters CA (eds): CAMPBELL-WALSH UROLOGY, ed 10. Philadelphia, Elsevier Saunders, 2012, vol 1, chap 15, pp 453-454.

2014 - 71 A 26-year-old man has recurrent groin abscesses. Rectal examination is normal. He has a good urinary stream. The most likely diagnosis is: A. syphilis. B. inflammatory bowel disease. C. hidradenitis suppurativa. D. chancroid. E. granuloma inguinale.

D The femoral nerve is of L2-L4 origin and has both motor and sensory components, providing sensation to the anterior thigh and motor innervation to the extensors of the knee. The femoral nerve travels within the belly and exits through the lateral aspect of the psoas muscle to pass under the ilioinguinal ligament. There are two potential injuries to the femoral nerve during a psoas hitch: the nerve can become entrapped from a suture placed at right angle to the muscle fibers and the direction of the nerve, or if performed by an open approach, the blades of a retractor can compress the psoas muscle and can result in a femoral nerve palsy. The ilioinguinal and genitofemoral nerves are easily identified during exposure for the psoas hitch and are readily mobilized away from the operative field. Chung BI, Sommer G, Brooks JD: Anatomy of the lower urinary tract and male genitalia, Wein AJ, Kavoussi LR, Novick AC, Partin AW, Peters CA (eds): CAMPBELL-WALSH UROLOGY, ed 10. Philadelphia, Elsevier Saunders, 2012, vol 1, chap 2, p 47.

2014 - 72 The nerve that is most likely to be injured during a psoas hitch for a tapered ureteral reimplantation is the: A. obturator. B. ilioinguinal. C. genitofemoral. D. femoral. E. iliohypogastric.

D The primary target of onabotulinumtoxinA treatment is inhibition of acetylcholine release from postsynaptic parasympathetic efferent nerves to the detrusor muscle, resulting in transient paralysis of the detrusor muscle. The efficacy of the treatment, however, exceeds that of pure detrusor paralysis and is associated with a significant reduction in bladder sensation. The decrease in the afferent sensation (urgency) is due to localized inhibition of detrusor muscle ATP, substance P production along with a reduction in activity of afferent axonal capsaicin and purinergic receptors. The overall effect is a profound inhibition of both efferent acetylcholine release (primary) and interference with afferent neuronal mediators and receptors (secondary). Yoshimura N, Chancellor MB: Physiology and pharmacology of the bladder and urethra, Wein AJ, Kavoussi LR, Novick AC, Partin AW, Peters CA (eds): CAMPBELL-WALSH UROLOGY, ed 10. Philadelphia, Elsevier Saunders, 2012, vol 3, chap 60, p 1825. Apostolidis A, Dasgupta P, Fowler CJ: Proposed mechanism for the efficacy of injected botulinum toxin in the treatment of human detrusor overactivity. EUR UROL 2006;49:644-650.

2014 - 73 The primary effect of intradetrusor onabotulinumtoxinA is to inhibit release of acetylcholine from: A. presynaptic afferent nerves. B. postsynaptic afferent nerves. C. presynaptic efferent nerves. D. postsynaptic efferent nerves. E. sympathetic efferent nerves.

B The patientÆs motor and sensory deficits are classic presentations for femoral neurapraxia, thought to be due to entrapment by the inguinal ligament in patients whose hips are hyperextended during positioning for robotic pelvic procedures. The femoral nerve has both sensory (anteromedial thigh) and motor components essential for hip flexion. The obturator nerve governs adduction of the thigh (motor) and sensation to medial aspect of thigh (sensory). The lateral femoral cutaneous nerve is purely a sensory nerve innervating the lateral thigh. The sciatic nerve has sensory (posterior calf) and motor (posterior thigh muscle groups) branches. The genitofemoral nerve supplies sensation to the scrotal skin and cremaster muscle (motor). Koc G, Tazeh NN, Joudi FN, et al: Lower extremity neuropathies after robot-assisted laparoscopic prostatectomy on a split-leg table. J ENDOUROL 2012;26:1026-1029.

2014 - 76 A 70-year-old obese man has difficulty walking one day following robotic cystectomy and pelvic lymphadenectomy. On examination, his extremities appear normal with slight decreased sensation of the anteromedial thigh and weakness bilaterally with hip flexion. The most likely cause for his symptoms is injury to which nerve: A. obturator. B. femoral. C. lateral femoral cutaneous. D. sciatic. E. genitofemoral.

B If a urethral catheter balloon will not deflate, the first step should be to attempt to place another 2-5 ml of fluid into the balloon and reattempt deflation of the balloon. If that fails, the inflation valve should be cut. This will allow access to the inflation channel. A guide or glidewire passed down the inflation channel will almost always clear the passage and let the water out without breaking the balloon. The other choices listed will result in breaking the balloon, often leaving a floating fragment that cannot be voided or irrigated out and can possibly lead to bladder stone formation. If rupture of the balloon is performed, cystoscopy is recommended to evaluate and retrieve any fragment left behind. Mendez CE, Razvi H, Denstedt JD: Fundamentals of instrumentation and urinary tract drainage, Wein AJ, Kavoussi LR, Novick AC, Partin AW, Peters CA (eds): CAMPBELL-WALSH UROLOGY, ed 10. Philadelphia, Elsevier Saunders, 2012, vol 1, chap 7, p 182.

2014 - 77 A 55-year-old man has a 16 Fr urethral catheter that cannot be removed because the balloon will not deflate. The best way to deflate it is to: A. inject ether into the inflation port. B. pass a guidewire down the inflation port. C. puncture the balloon percutaneously with ultrasound guidance. D. inject water into the inflation port until the balloon bursts. E. pass a resectoscope sheath over the cut-off catheter to guide cystoscopic scissors.

D This scenario is concerning for UTI with an urea-splitting gram-negative organism in a urinary tract with significant urinary stasis. This can occur in prune belly syndrome children or any other patient with urinary stasis or obstruction. Urea in the urine is split into large amounts of ammonia which is absorbed systemically and may result in encephalopathy or even coma at high levels. Treatment is support, urinary drainage, and antibiotic administration. The vital signs, urine output, and normal hematocrit in the current patient argue for relative normovolemia and are not consistent with sepsis or other illness. Although perturbations of other electrolytes, bicarbonate, or lactate may occur, significant abnormality of one or more of these should lead to hemodynamic instability if severe enough to lead to an obtunded state. DeJonghe B: Urinary tract infection and coma. LANCET 2002;360:996. Shortliffe LMD: Infection and inflammation of the pediatric genitourinary tract, Wein AJ, Kavoussi LR, Novick AC, Partin AW, Peters CA (eds): CAMPBELL-WALSH UROLOGY, ed 10. Philadelphia, Elsevier Saunders, 2012, vol 4, chap 116, p 3114.

2014 - 78 A 13-year-old boy with prune belly syndrome and chronic hydronephrosis is obtunded. He has a normal blood pressure and pulse, normal hematocrit, and appropriate urine output. Urinalysis is suspicious for UTI with gram negative rods on Gram stain. The lab value most likely to reveal the source of his obtundation is serum: A. sodium. B. magnesium. C. bicarbonate. D. ammonia. E. lactate.

E Androgen resistance or testicular feminization syndrome presents with a wide range of phenotypes of complete female external genitalia to men with normal genitalia and abnormalities of spermatogenesis. This X linked disorder is due to an abnormality of the androgen receptor. Testosterone production and secretion by the Leydig cells is normal, but the target organs including the pituitary gland do not recognize it due to faulty receptors. Therefore, LH secretion is increased because of the apparent lack of testosterone while FSH is unaffected since it is controlled by inhibin. Puberty would have normal testosterone, LH, and FSH. Klinefelter Syndrome and Sertoli Cell Only Syndrome would have elevated FSH, LH, and low testosterone. Secondary testicular failure can have a multitude of etiologies with the end result being low testosterone. The FSH and LH may be high or low. Diamond DA, Yu RN: Sexual differentiation: Normal and abnormal, Wein, AJ, Kavoussi LR, Novick AC, Partin AW, Peters CA (eds): CAMPBELL-WALSH UROLOGY, ed 10. Philadelphia, Elsevier Saunders, 2012, vol 4, chap 133, p 3621.

2014 - 79 Elevation of the serum testosterone and LH levels associated with a normal FSH level is most frequently associated with: A. puberty. B. Klinefelter syndrome. C. secondary testicular failure. D. Sertoli-cell-only syndrome. E. androgen resistance.

A This patient has a less than 5% motility. This raises the prospect of an ultrastructural abnormality in the sperm tails such as primary ciliary dyskinesia. This is characterized by extremely low motility but relatively high sperm viability on a sperm viability assay. Since the sperm concentration is normal, there is no indication for determination of testosterone, FSH, karyotype, or Y chromosome microdeletion testing. Scrotal ultrasonography will not add any useful information. Testicular sperm extraction is indicated for azoospermia. Since this patient has motile sperm in the semen, there is no reason for sperm extraction. Sperm from patients with ciliary dyskinesia may be used for intra-cytoplasmic sperm injection (ICSI), but pregnancy rates are low. Jarow J, Sigman M, Kolettis PN, et al: The optimal evaluation of the infertile male: AUA best practice statement revised, 2010. OPTIMAL EVALUATION OF THE INFERTILE MALE. American Urological Association Education and Research, Inc, 2010. http://www.auanet.org/education/guidelines/male-infertility-d.cfm Sabanegh E, Agarwal A: Male infertility, Wein AJ, Kavoussi LR, Novick AC, Partin AW, Peters CA (eds): CAMPBELL-WALSH UROLOGY, ed 10. Philadelphia, Elsevier Saunders, 2012, vol 1, chap 21, p 621.

2014 - 8 A 34-year-old man and his 29-year-old wife have a two year history of infertility. His physical exam is normal. Semen analysis reveals a volume of 2 ml, sperm 23 mil/ml, 2% motility, and 12% normal morphology. Repeat analysis is similar. The next step is: A. sperm viability assay. B. testosterone and FSH levels. C. karyotype and Y chromosome microdeletion testing. D. scrotal ultrasonography. E. testicular sperm extraction.

A Following chemotherapy for retroperitoneal metastases from non-seminomatous germ cell tumors, teratoma will be found in approximately 40% of patients. When this is found, complete resection is mandatory for a number of reasons. Although the teratoma may be benign, continued growth of the tumor can lead to significant morbidity and the tumor can become unresectable. Additionally, complete retroperitoneal nodal evaluation will detect foci of residual cancer in up to 20% of patients. Thus, complete RPLND is necessary. Sheinfeld J, Bosl GJ: Surgery of testicular tumors, Wein AJ, Kavoussi LR, Novick AC, Partin AW, Peters CA (eds): CAMPBELL-WALSH UROLOGY, ed 10. Philadelphia, Elsevier Saunders, 2012, vol 1, chap 32, p 875.

2014 - 80 A 23-year-old man has a persistent 5 cm interaortocaval mass after three cycles of BEP for left NSGCT. Serum tumor markers are normal. During RPLND, the mass is densely adherent to the aorta and vena cava. An incisional biopsy of the mass reveals fibrosis. The next step is: A. complete RPLND. B. sample para-aortic and paracaval nodes. C. completely resect residual mass. D. terminate RPLND and follow tumor markers. E. terminate RPLND and give XRT.

B The interaortocaval nodes primarily drain the right kidney. The left kidney drains to the interaortocaval nodes only in advanced disease. Anderson JK, Cadeddu JA: Surgical anatomy of the retroperitoneum, adrenals, kidneys, and ureters, Wein AJ, Kavoussi LR, Novick AC, Partin AW, Peters CA (eds): CAMPBELL-WALSH UROLOGY, ed 10. Philadelphia, Elsevier Saunders, 2012, vol 1, chap 1, p 3.

2014 - 81 A 66-year-old woman is undergoing abdominal surgery for diverticulitis when the surgeon notices a solitary enlarged interaortocaval lymph node in the upper abdomen and removes it for biopsy. The biopsy reveals RCC. The most likely source of this disease is: A. the left kidney. B. the right kidney. C. either kidney. D. pelvic kidney. E. horseshoe kidney.

E The use of over-the-counter phytotherapy continues to be a popular therapy for the treatment of male LUTS due to BPH. However, there are a limited number of well done placebo-controlled trials evaluating its efficacy. The vast majority of these trials suggest that Serenoa repens is no more effective than placebo. No differences are seen in regards to AUA-SI or flow rates. There does not appear to be any decrease in prostate size or evidence that suggests that this drug is able to minimize the risk of progression to acute urinary retention. Initial prostate size does not appear to impact the outcome when Serenoa repens is used. Adverse events are mild and appear to be comparable to placebo. McNicholas TA, Kirby RS, Lepor H: Evaluation and nonsurgical management of benign prostatic hyperplasia, Wein AJ, Kavoussi LR, Novick AC, Partin AW, Peters CA (eds): CAMPBELL-WALSH UROLOGY, ed 10. Philadelphia, Elsevier Saunders, 2012, vol 3, chap 92, p 2651. MacDonald R, Tacklind JW, Rutks I, et al: Serenoa repens monotherapy for benign prostatic hyperplasia (BPH): An updated Cochrane systematic review. BJU INT 2012;109:1756. Barry MJ, Neleth S, Lee JY, et al: Effect of increasing doses of saw palmetto extract on lower urinary tract symptoms. JAMA 2011;306:1344-1351.

2014 - 82 Compared to placebo, monotherapy with Serenoa repens (saw palmetto) for men with bothersome LUTS due to BPH is likely to result in: A. improved AUA symptom index. B. improved maximal flow rate. C. decreased prostate size. D. lowered risk of acute urinary retention. E. similar rate of adverse events.

E Small-cell carcinoma of the prostate carries a very poor prognosis. In most patients, the course is rapidly fatal. Most patients present with rapid onset of symptoms indistinguishable from other causes of bladder outlet obstruction. Systemic constitutional symptoms are reported in about 10% of cases. Chemotherapy in combination with surgery or radiation appears to be the most important component of management. Radiation is ineffective as is surgery alone, since most patients have metastatic disease. These tumors are generally not hormonally sensitive. Abbas F, Civantos F, Benedett P, et al: Small cell carcinoma of the bladder and prostate. J UROL 1995;46:30. Antonarakis ES, Carducci MA, Eisenberger MA: Treatment of castration-resistant prostate cancer, Wein AJ, Kavoussi LR, Novick AC, Partin AW, Peters CA (eds): CAMPBELL-WALSH UROLOGY, ed 10. Philadelphia, Elsevier Saunders, 2012, vol 3, chap 110, pp 2961-2962.

2014 - 83 A 55-year-old man has a transrectal biopsy which reveals small cell carcinoma. Metastatic workup is negative. The next step is: A. radical prostatectomy. B. LH-RH agonist. C. cystoprostatectomy. D. external beam radiation therapy (EBRT). E. chemotherapy.

E Of all the potential complications of a large ureterocele, bladder outlet obstruction is the most worrisome because it affects both kidneys, leads to recurrent infection, and progressive renal damage. A key indicator of bladder outlet obstruction in this patient is the presence of contralateral hydroureteronephrosis. Non-function of the upper pole system does not require early intervention unless the patient presents with febrile UTI unresponsive to antibiotic therapy. Initial therapy for reflux either into the ipsilateral or contralateral kidney would be managed with continuous antibiotic prophylaxis as first line therapy. Incontinence occurs in a small percentage of children with ureteroceles, usually due to a congenital defect in the bladder neck due to distortion from the ureterocele, and is best managed in the older child. Peters CA, Schlussel RN, Mendelsohn C: Ectopic ureter, ureterocele, and ureteral anomalies, Wein AJ, Kavoussi LR, Novick AC, Partin AW, Peters CA (eds): CAMPBELL-WALSH UROLOGY, ed 10. Philadelphia, Elsevier Saunders, 2012, vol 4, chap 121, pp 3240-3241.

2014 - 84 A newborn girl has a febrile UTI. Her abdominal ultrasound reveals a left duplicated system with a large ureterocele and severe bilateral hydroureteronephrosis. Early intervention is required because of: A. non-function of the upper pole segment. B. reflux to the lower pole segment. C. reflux to the contralateral kidney. D. increased risk of incontinence. E. bladder outlet obstruction.

B Emission is defined as the deposition of seminal fluid into the posterior urethra by the vasa deferentia and the seminal vesicles. Ejaculation is the forceful expulsion of seminal fluid out the urethral meatus by contraction of the bulbospongiosus and ischiocavernosus muscles. Since the vasa and the seminal vesicles are innervated primarily by the sympathetic nervous system, emission is under control of the sympathetic nervous system. Alpha-adrenergic nerve stimulation causes not only contraction of the seminal vesicles and vasa deferentia but also closure of the bladder neck. Ejaculation is the result of somatic nerve stimulation of the periurethral striated musculature. The parasympathetic nervous system is not directly involved with either emission or ejaculation. Turek PJ: Male reproductive physiology, Wein, AJ, Kavoussi LR, Novick AC, Partin AW, Peters CA (eds): CAMPBELL-WALSH UROLOGY, ed 10. Philadelphia, Elsevier Saunders, 2012, vol 1, chap 20, p 610.

2015 - 1 Seminal emission depends on an intact: A. parasympathetic and somatic nervous system. B. sympathetic nervous system. C. parasympathetic nervous system. D. sympathetic and parasympathetic nervous system. E. sympathetic and somatic nervous systems.

B Patients who are taking PDE5 inhibitors cannot concomitantly use nitrates due to the potential risk of severe hypotension. For the short acting PDE5 inhibitors (vardenafil, sildenafil, avanafil), medical personnel should not use nitrates for at least 24 hours and for 48 hours with the long acting PDE5 inhibitor, tadalafil. Men who have taken PDE5 inhibitors and develop angina during sexual activity (which has a greater risk of bringing on a non-fatal MI) should stop sexual activity and seek emergency medical therapy immediately. Burnett AL: Evaluation and management of erectile dysfunction, Wein AJ, Kavoussi LR, Novick AC, Partin AW, Peters CA (eds): CAMPBELL-WALSH UROLOGY, ed 10. Philadelphia, Elsevier Saunders, 2012, vol 1, chap 24, p 742. Montague DK, Jarow J, Broderick GA, et al: The management of erectile dysfunction: An update. ERECTILE DYSFUNCTION. American Urological Association Education and Research, Inc, 2005. http://www.auanet.org/education/guidelines/erectile-dysfunction.cfm

2014 - 85 A 64-year-old man with erectile dysfunction and a history of coronary disease is taking tadalafil 5 mg daily. During sexual activity, he develops angina. The next step is: A. stop sexual activity and rest. B. stop sexual activity and seek emergency treatment. C. take nitroglycerin only if it has been at least 24 hours since his last tadalafil dose. D. take nitroglycerin only if it has been at least 48 hours since his last tadalafil dose. E. take nitroglycerin only in a medically monitored setting.

E Early after continent urinary diversion, many patients misinterpret distension of the pouch due to urine and mucus as diffuse abdominal pain. This patient's KUB, normal WBC, normal bowel sounds, and lack of fever do not suggest obstruction or leakage from the bowel reanastomosis or the urinary pouch. The first and simplest measure is to catheterize and drain the pouch to determine if the symptoms are relieved. Only if the pain persists are some or all of the other measures necessary to exclude other postoperative complications such as ileus, pouch leak or rupture, or urinary tract obstruction. Patient education on symptoms is paramount to prevent unnecessary interventions. McKiernan JM, DeCastro GJ, Benson MC: Cutaneous continent urinary diversion, Wein AJ, Kavoussi LR, Novick AC, Partin AW, Peters CA (eds): CAMPBELL-WALSH UROLOGY, ed 10. Philadelphia, Elsevier Saunders, 2012, vol 3, chap 86, p 2458.

2014 - 86 A 42-year-old woman has crampy abdominal pain three months after undergoing a continent cutaneous urinary diversion. She is afebrile and has a WBC of 7,500/cu mm. Her abdomen is mildly distended, diffusely tender, and bowel sounds are present. KUB reveals nonspecific small and large bowel gas patterns. The next step is: A. I.V. fluids and nasogastric suction. B. exploratory laparotomy. C. pouchogram. D. CT urogram. E. pouch catheterization.

A Although ureteral injuries are uncommon during transvaginal suspension procedures, the ureter occasionally can be injured or obstructed by the surgical dissection or suspension suture. If recognized intraoperatively, the suspension suture should be removed and a ureteral stent placed. If this is recognized postoperatively, a retrograde pyelogram will define the extent of the injury and retrograde stent placement attempted. If stenting is not successful, antegrade stenting is advised. Open repair is advised when retrograde or antegrade ureteral stent placement cannot be performed. Ureteroneocystostomy is performed only when significant devitalization of the ureter occurs. Adams MC, Joseph DB: Urinary tract reconstruction in children, Wein AJ, Kavoussi LR, Novick AC, Partin AW, Peters CA (eds): CAMPBELL-WALSH UROLOGY, ed 10. Philadelphia, Elsevier Saunders, 2012, vol 4, chap 129, pp 3465-3466. Ficazzola M, Nitti VW: Complications of incontinence procedures in women, Taneja SS, Smith RB, Ehrlich RM (eds): COMPLICATIONS OF UROLOGIC SURGERY: PREVENTION AND MANAGEMENT, ed 3. Philadelphia, Elsevier Saunders, 2001, chap 42, pp 489-491.

2014 - 87 A 17-year-old girl with a history of an augmentation cystoplasty undergoes a pubo-vaginal sling procedure for persistent stress incontinence. On postoperative day three, she has left flank pain and increased drainage from the incision (creatinine of drainage is 9 mg/dl). CT scan reveals new onset of left hydroureteronephrosis and left perivesical fluid collection, and CT cystogram reveals no extravasation of contrast from bladder. The next step is: A. cystoscopy, retrograde pyelogram, and stent placement. B. percutaneous nephrostomy. C. removal of sling. D. open ureteral repair and removal of sling. E. ureteroneocystostomy.

A Over 85% of patients and 70% of partners rate satisfaction following placement of penile prosthesis as very high . That being said, there are still patients who have significant problems with establishing intimacy following the diagnosis and treatment of prostate cancer. There are also patients who note significant decrease in penile size following radical prostatectomy. In this patient with no problems noted with his implant, evaluation and treatment by a sex therapist would be the best treatment. Combination therapy with a PDE5 inhibitor or alprostadil can increase glanular engorgement but are not FDA-approved and would not necessarily help with partner issues. There is no need to change out the prosthesis for a different device if it has been properly sized and implanted. Montague DK: Prosthetic surgery for erectile dysfunction, Wein AJ, Kavoussi LR, Novick AC, Partin AW, Peters CA (eds): CAMPBELL-WALSH UROLOGY, ed 10. Philadelphia, Elsevier Saunders, 2012, vol 1, chap 27, p 791. Montague DK, Jarow J, Broderick GA, et al: The management of erectile dysfunction: An update. ERECTILE DYSFUNCTION GUIDELINE. American Urological Association Education and Research, Inc, 2005. http://www.auanet.org/education/guidelines/erectile-dysfunction.cfm

2014 - 88 A 61-year-old man develops complete erectile dysfunction following radical prostatectomy and has placement of a two-piece inflatable penile prosthesis. Three months after implantation, he complains of decreased penile size and inability to satisfy his partner. Examination demonstrates a functional prosthesis. The best treatment is: A. referral to a sex therapist. B. combination therapy with sildenafil. C. combination therapy with intraurethral alprostadil. D. upsize to a larger two-pieced inflatable prosthesis. E. remove and implant a three-piece inflatable prosthesis.

B Penetrance for all of the manifestations of VHL is incomplete. Pheochromocytoma is found only in certain families with the VHL syndrome, primarily those with a missense mutation of the VHL gene. All of the other manifestations of VHL are found in most families with the syndrome. A careful family history and thorough review of preoperative CT scans for potential associated tumors is important in all patients with familial RCC. Pheochromocytomas in particular are a critical entity to recognize prior to any surgical intervention. CAMPBELL SC, Lane BR: Malignant renal tumors, Wein, AJ, Kavoussi LR, Novick AC, Partin AW, Peters CA (eds): CAMPBELL-WALSH UROLOGY, ed 10. Philadelphia, Elsevier Saunders, 2012, vol 2, chap 49, p 1440.

2015 - 104 The manifestation of the von Hippel-Lindau syndrome that tends to be clustered only within a subset of affected families is: A. RCC. B. pheochromocytoma. C. retinal angioma. D. cerebellar hemangioblastoma. E. epididymal papillary cystadenoma.

D Under usual circumstances, a peripherally located tumor in a solitary kidney is best managed by partial nephrectomy, both for potential cure and to prevent the need for dialysis. However, the presence of a venous thrombus makes complete tumor excision less likely and is associated with a high likelihood of tumor recurrence and poor prognosis after partial nephrectomy. Given the patient's good health, long life expectancy, and episode of gross hematuria, observation is likely to result in further tumor progression, more bleeding or other local symptoms which may require intervention, and potentially metastatic disease. Laparoscopic cryoablation is best used for smaller tumors and will not control the tumor thrombus. Systemic immunotherapy rarely results in a response in the primary tumor and is unlikely to be curative. Thus, the best choice is radical nephrectomy which will result in the need for dialysis. A prolonged disease-free interval would make the patient eligible for subsequent renal transplantation. Selective angioinfarction of the tumor is another alternative which could be used for palliation of the bleeding, but will not be curative. Angermeier KW, Novick AC, Streem SB, et al: Nephron-sparing surgery for renal cell carcinoma with venous involvement. J UROL 1990;144:1352-1355. Campbell SC, Lane BR: Malignant renal tumors, Wein AJ, Kavoussi LR, Novick AC, Partin AW, Peters CA (eds): CAMPBELL-WALSH UROLOGY, ed 10. Philadelphia, Elsevier Saunders, 2012, vol 2, chap 49, p 1445.

2014 - 89 A healthy 50-year-old man with gross hematuria has a peripherally located 4 cm solid mass in a solitary kidney. The serum creatinine is 1.2 mg/dl. CT scan demonstrates a 1 cm renal vein tumor thrombus. There is no evidence of metastasis. The next step is: A. angiographic embolization. B. laparoscopic cryoablation. C. partial nephrectomy. D. radical nephrectomy. E. interleukin-2.

C The patient has multiple risk factors which increase his risk for complications after surgery for cystectomy and orthotopic diversion, however, only urethral stricture disease would be a contraindication for the diversion. CIS does not appear by itself to significantly increase the risk of urethral recurrence after orthotopic diversion. Obese patients may actually do better with an orthotopic diversion as it avoids stomal issues that are common in the morbidly obese. Studies have demonstrated the safety and feasibility of performing orthotopic diversions in previously radiated patients. Similarly, age is not a contraindication. Patients with urethral stricture disease should not undergo orthotopic diversion as they may be unable to catheterize should the need arise and may cause overdistension of the neobladder which could lead to rupture. Skinner EC, Skinner DG, Stein JP: Orthotopic urinary diversion, Wein AJ, Kavoussi LR, Novick AC, Partin AW, Peters CA (eds): CAMPBELL-WALSH UROLOGY, ed 10. Philadelphia, Elsevier Saunders, 2012, vol 3, chap 87, pp 2484-2485.

2014 - 9 A morbidly obese 72-year-old man undergoes XRT for prostate cancer complicated by a urethral stricture requiring multiple direct visual internal urethrotomies. He subsequently develops BCG refractory CIS of the bladder cancer and chooses to undergo cystectomy. The risk factor that makes him an unacceptable candidate for orthotopic diversion is: A. prior XRT. B. morbid obesity. C. urethral stricture disease. D. age. E. presence of CIS.

D The incidence of Candidal nosocomia UTIs occurring within neonatal intensive care units is increasing and directly related to the use of parenteral antibiotics. In this select patient population, aggressive treatment of asymptomatic candiduria is required due to a high incidence of subsequent candidemia. Indeed, in some published series, failure to treat asymptomatic candiduria in premature neonates resulted in systemic candidemia in up to 80% of patients. Isolating treatment to the bladder with topical irrigation will not effectively treat the risk of candidemia and parenteral treatment is required. Fluconazole is the treatment of choice in a premature infant when compared to amphotericin because of significantly diminished systemic side effects. Circumcision will not decrease the risk of candidemia. Shortliffe LMD: Infection and inflammation of the pediatric genitourinary tract, Wein AJ, Kavoussi LR, Novick AC, Partin AW, Peters CA (eds): CAMPBELL-WALSH UROLOGY, ed 10. Philadelphia, Elsevier Saunders, 2012, vol 4, chap 116, p 3119.

2014 - 90 A 32-week-gestation neonate maintained in the neonatal ICU for respiratory difficulties, is found to have candiduria on two successive catheterized urine cultures. He is voiding spontaneously, and his renal and bladder ultrasound is normal. The most appropriate therapy is: A. observation with repeat urine culture in one week. B. circumcision. C. intravesical amphotericin. D. parenteral fluconazole. E. parenteral amphotericin.

D T cell activation requires three signals. First, T cell receptor engages specific antigens presented by MHC molecules on APCs. Without the second signal, the T cell remains functionally inactive. Second, stabilization and costimulation between APC glycoproteins (e.g., CD 40 and CD 80) with their T cell ligands are required. Lastly, cytokine (especially interleukins) stimulation is necessary to complete T cell activation. Although intracellular signal transduction and cell proliferation are integral parts of T cell activation, these steps are not possible without T cell costimulation. Flechner SM, Finke JH, Fairchild RL: Basic principles of immunology in urology, Wein AJ, Kavoussi LR, Novick AC, Partin AW, Peters CA (eds): CAMPBELL-WALSH UROLOGY, ed 10. Philadelphia, Elsevier Saunders, 2012, vol 1, chapter 17, p 503.

2014 - 91 T cell activation requires T cell receptor engagement of antigen presented by major histocompatibility complex (MHC) molecule on the surface of antigen presenting cells (APCs) and: A. intracellular signal transduction. B. cell division and proliferation. C. direct cell-cell interaction with B cells. D. APC glycoprotein and T cell ligand interaction. E. interferon-gamma stimulation.

C An ileal conduit is not a contraindication to having a successful, normal pregnancy. Unless a specific indication exists, the routine obstetrical care of a pregnant woman with an ileal conduit is the same as that for a woman with no previous urinary tract surgery. Antibiotics are only required if the patient experiences a symptomatic urinary tract infection. There is no required need for Cesarean section and amniocentesis should be reserved for fetal indications. Serial renal ultrasounds would likely show progressive physiologic hydroureteronephrosis of pregnancy, and are not indicated in asymptomatic patients. Loughlin KR: Management of urologic problems during pregnancy. J UROL 1994;44:159. Adams MC, Joseph DB: Urinary tract reconstruction in children, Wein AJ, Kavoussi LR, Novick AC, Partin AW, Peters CA (eds): CAMPBELL-WALSH UROLOGY, ed 10. Philadelphia, Elsevier Saunders, 2012, vol 4, chap 129, pp 3485-3486.

2014 - 92 A 31-year-old woman had an ileal conduit urinary diversion at age ten years for a neurogenic bladder. She is now ten weeks pregnant. In addition to appropriate obstetrical care, she should have: A. amniocentesis. B. a Cesarean section at term. C. no additional urologic treatment. D. prophylactic antibiotics. E. serial renal ultrasounds.

E The TOMUS (Trial of Mid-Urethral Slings) study is a multicenter, randomized equivalence trial sponsored by the Urinary Incontinence Treatment Network (UITN) which evaluated 597 women with SUI who were assigned to undergo treatment either with a retropubic or transobturator midurethral sling. At 12 months, the patients in both groups met the prespecified criteria for equivalence with regard to the objective definition of success (negative stress test, negative pad test, and no retreatment) with 80.8% in the retropubic group and 77.7% in the transobturator group having a successful outcome. Subjective definition of success was defined as a self-reported absence of symptoms, no leakage episodes recorded on diary, and no retreatment. Subjectively, the outcomes between the two groups were similar; 62.2% in the retropubic group and 55.8% in the transobturator. Rates of postoperative voiding dysfunction were higher (2.7%) in the retropubic group compared to the transobturator group (0%). Richter HE, Albo ME, Zyczynski HM, et al: Retropubic versus transobturator midurethral slings for stress incontinence. NEJM 2010;362:2066-2076.

2014 - 94 At twelve months, results of the Trial of Mid-Urethral Slings (TOMUS) showed that retropubic slings compared to transobturator slings have: A. superior efficacy and greater risk of post-operative voiding dysfunction. B. superior efficacy and lesser risk of post-operative voiding dysfunction. C. inferior efficacy and greater risk of post-operative voiding dysfunction. D. inferior efficacy and lesser risk of post-operative voiding dysfunction. E. equivalent efficacy and greater risk of post-operative voiding dysfunction.

E Further chemotherapy is not indicated without histologic evidence of residual disease. Although assessment of the retroperitoneal nodes must be considered, removal of the lung mass by thoracotomy is indicated first. Needle biopsy would be an insufficient sampling method and if this is teratoma, excision of the lung mass would be therapeutic. Bosl GS, Motzer RJ: Testicular germ cell cancer. NEJM 1997;337:242-253.

2014 - 95 Left inguinal orchiectomy is performed on a 24-year-old man for an embryonal cell carcinoma. AFP is elevated but beta-hCG is normal. On chest x-ray, there is a 5 cm mass in the right lung and an abdominal CT scan shows a 2 cm periaortic adenopathy. After four cycles of platinum-based chemotherapy, AFP has returned to normal, and an abdominal CT scan shows resolution of the retroperitoneal adenopathy. On chest x-ray, the lung mass is still present but has decreased to 3 cm in size. The next step is: A. retroperitoneal node dissection. B. salvage chemotherapy with ifosfamide. C. continue platinum chemotherapy for two more cycles. D. needle biopsy of the lung mass. E. pulmonary wedge resection.

D Imperforate anus is accompanied by a high incidence of bony sacral and neurological anomalies that may lead to a neurogenic bladder. On the other hand, diurnal and nocturnal enuresis are common in six year olds. This boy can completely empty the bladder volitionally; i.e., no postvoid residual. There is no evidence of spinal cord tethering with the conus medullaris in the normal position upper border of L-2. A vesico-colonic fistula would be unlikely. The urodynamic study is not suggestive of detrusor-sphincter dyssynergia, and the detrusor overactivity at high volume is unlikely to suggest any serious underlying pathology. MacLellan DL, Bauer SB: Neuropathic dysfunction of the lower urinary tract, Wein AJ, Kavoussi LR, Novick AC, Partin AW, Peters CA (eds): CAMPBELL-WALSH UROLOGY, ed 10. Philadelphia, Elsevier Saunders, 2012, vol 4, chap 128, pp 3450-3452.

2014 - 96 A six-year-old boy with a history of imperforate anus has persistent diurnal urinary incontinence. Evaluation includes an MRI scan showing the conus medullaris at the L-2 level. Urodynamics show no postvoid residual, a flat filling curve with high volume detrusor overactivity, and normal voiding pressures. These findings suggest: A. tethering of the spinal cord. B. vesico-colonic fistula. C. detrusor-sphincteric dyssynergia. D. no significant uropathology. E. intrinsic sphincteric deficiency.

E The urodynamics are consistent with bladder outlet obstruction. The patient has normal bladder compliance, no evidence of detrusor overactivity, and during the pressure-flow study, elevated detrusor pressures with a low flow. Therefore, treatments aimed at overactive bladder (solifenacin and onabotulintumtoxinA) would not be indicated. This patient does not have a large prostate on exam; thus, dutasteride is not indicated. A TUIP would not be appropriate in light of the enlarged median lobe; therefore, TURP would be the appropriate therapy for this patientÆs LUTS. Fitzpatrick JM: Minimally invasive and endoscopic management of benign prostatic hyperplasia, Wein AJ, Kavoussi LR, Novick AC, Partin AW, Peters CA (eds): CAMPBELL-WALSH UROLOGY, ed 10. Philadelphia, Elsevier Saunders, 2012, vol 3, chap 93, p 2679.

2014 - 97 A 65-year-old man complains of residual LUTS after prior TUNA. His symptoms are unchanged after the initiation of tamsulosin. He has a 30 gram benign prostate and cystoscopy reveals median lobe enlargement. The pressure-flow urodynamic study reveals no evidence of detrusor overactivity, with an end fill detrusor pressure of 3 cm H2O at a maximum capacity of 450 ml. He voids with a synergistic curve pattern with detrusor pressure at the time of peak flow of 120 cm H2O. Peak flow is 8 ml/sec, average is 4 ml/sec, and total volume voided is 400 ml PVR of 50 ml. The next step is: A. dutasteride. B. solifenacin. C. onabotulinumtoxinA injection into the detrusor. D. TUIP. E. TURP.

E Deletions of the short arm of chromosome 3 are characteristic of clear cell (conventional type) RCCs, which are seen in sporadic cases of renal cell carcinoma and those associated with von Hippel-Lindau disease. The short arm of chromosome 3 is the location of the VHL gene. Type 1 papillary renal cancers are associated with mutations of the c-met proto-oncogene located on chromosome 7. Type II papillary renal cancers are associated with mutations of the fumarate hydratase gene located on chromosome 1. Oncocytoma can be seen in mutations of the BHD1 gene located on the short arm of chromosome 17. Campbell SC, Lane BR: Malignant renal tumors, Wein AJ, Kavoussi LR, Novick AC, Partin AW, Peters CA (eds): CAMPBELL-WALSH UROLOGY, ed 10. Philadelphia, Elsevier Saunders, 2012, vol 2, chap 49, p 1425.

2014 - 98 Deletions of the short arm of chromosome 3 occur most frequently in association with: A. papillary RCC. B. angiomyolipoma. C. renal medullary carcinoma. D. oncocytoma. E. clear cell RCC.

D The distal prostatic ducts are lined by transitional (urothelial) epithelium as in the prostatic urethra. Primary urothelial carcinoma of the prostate arising from these glands account for 1.5-4% of all male urethral cancers. Chung BI, Sommer G, Brooks JD: Anatomy of the lower urinary tract and male genitalia, Wein AJ, Kavoussi LR, Novick AC, Partin AW, Peters CA (eds): CAMPBELL-WALSH UROLOGY, ed 10. Philadelphia, Elsevier Saunders, 2012, vol 1, chap 2, p 56.

2014 - 99 The epithelial lining of the distal periurethral prostatic ducts is: A. stratified. B. pseudostratified. C. squamous. D. transitional (urothelial). E. cuboidal.

E TRUS and biopsy is one of the most common urologic procedures. Antibiotic prophylaxis is well-established as reducing infection after the procedure. The AUA Best Practice Statement on Antimicrobial Prophylaxis states that the only oral agent approved for TRUS and biopsy prophylaxis is an oral fluoroquinolone. Alternatives are an I.V. 1st, 2nd or 3d generation cephalosporin or aminoglycoside plus metronidazole or clindamycin. Septra and oral cefuroxime are incorrect because of the oral route of administration. Levofloxacin is incorrect since the patient had a severe ciprofloxacin allergy, so other fluoroquinolones should be avoided unless tolerance testing is performed. Gentamicin without metronidazole or clindamycin is also incorrect. Wolf JS Jr, Bennett CJ, Dmochowski RR, et al: Best practice policy statement on urological surgery antimicrobial prophylaxis. UROLOGICAL SURGERY ANTIMICROBIAL PROPHYLAXIS BEST PRACTICE STATEMENT. American Urological Association Education and Research, Inc, 2014. <a href="http://www.auanet.org/education/guidelines/antimicrobial-prophylaxis.cfm" target="_new"><u>http://www.auanet.org/education/guidelines/antimicrobial-prophylaxis.cfm</u></a>

2015 - 10 A 55-year-old man is scheduled to undergo TRUS-guided prostate biopsies. He has a severe allergy to ciprofloxacin. The best antibiotic regimen is: A. trimethoprim and sulfamethoxazole orally twice daily for three days. B. cefuroxime 500 mg orally twice daily for three days. C. levofloxacin 500 mg orally once daily for three days. D. gentamicin 5 mg/kg I.V. 30 minutes prior to the biopsy. E. ceftriaxone 1 gm I.V. 30 minutes prior to the biopsy.

C All exogenous testosterone products serve as a natural contraceptive, and thus, should not be given to men who are trying to achieve a pregnancy. Human chorionic gonadotropin is an LH analog and will increase endogenous testosterone, as well as potentially improve semen parameters. A scrotal ultrasound may be indicated if there was a suspicion for varicoceles although this patient's exam was normal. A testis biopsy would not be indicated in this situation. Burnett AL: Evaluation and management of erectile dysfunction, Wein, AJ, Kavoussi LR, Novick AC, Partin AW, Peters CA (eds): CAMPBELL-WALSH UROLOGY, ed 10. Philadelphia, Elsevier Saunders, 2012, vol 1, chap 24, pp 739-740. Sabanegh E, Agarwal A: Male infertility, Wein, AJ, Kavoussi LR, Novick AC, Partin AW, Peters CA (eds): CAMPBELL-WALSH UROLOGY, ed 10. Philadelphia, Elsevier Saunders, 2012, vol 1, chap 21, pp 618, 639.

2015 - 100 A 32-year-old man with a testosterone of 223 ng/dl has low energy, low libido and increased fatigue. He and his female partner are currently trying to conceive. Semen analysis demonstrates oligoasthenoteratospermia. Physical exam is normal. The next step is: A. topical testosterone. B. subcutaneous testosterone. C. human chorionic gonadotropin. D. scrotal ultrasound. E. testis biopsy.

D Breast tenderness and enlargement during hormonal therapy for prostate cancer is due to an altered ratio of serum estrogens and testosterone. Leuprolide inhibits pituitary LH release so there is no rise in serum estrogens. Ketoconazole inhibits androgen and estrogen synthesis. Bicalutamide inhibits both end organ and central androgen receptors so that both LH and androgen levels increase with loss of negative feedback inhibition. Increased androgen is converted to estradiol, which can cause painful gynecomastia. Although finasteride can cause gynecomastia, the most likely cause is bicalutamide monotherapy. Nelson JB: Hormone therapy for prostate cancer, Wein, AJ, Kavoussi LR, Novick AC, Partin AW, Peters CA (eds): CAMPBELL-WALSH UROLOGY, ed 10. Philadelphia, Elsevier Saunders, 2012, vol 3, chap 109, p 2939.

2015 - 101 The treatment most likely to cause painful gynecomastia is: A. leuprolide. B. leuprolide and bicalutamide. C. ketoconazole. D. bicalutamide. E. finasteride.

A This situation describes retractile testes. The cremasteric reflex is responsible for the superior retraction of the testes, and this reflex becomes more pronounced in many boys between two and eight years of age. The principal diagnostic characteristic is that the testis will remain without tension in a scrotal position after fatiguing (holding the testis manually in the scrotum for approximately 30 seconds and releasing the testis) until the reflex is once again stimulated. Some testes may retract into a higher position than others and require more persistent exam to evaluate and diagnose. Retractile testes have been shown to grow normally over time and be associated with normal paternity rates, thus requiring no immediate intervention. There is an associated phenomenon of secondary "ascent" in 2-4% of boys that may be associated with deterioration of germ cells. For this reason, it is appropriate to examine boys with retractile testes annually until either puberty arrives or the testis becomes non-retractile. hCG injection will consistently cause retractile testis to become intrascrotal and may be used in some cases where the diagnosis of retractile vs. undescended is uncertain. The testicles will, however, frequently retract once the hormonal stimulation is complete. In the case described, retractility is the clear diagnosis; therefore, neither further hormonal studies nor surgery are required. Barthold JS: Abnormalities of the testes and scrotum and their surgical management, Wein, AJ, Kavoussi LR, Novick AC, Partin AW, Peters CA (eds): CAMPBELL-WALSH UROLOGY, ed 10. Philadelphia, Elsevier Saunders, 2012, vol 4, chap 132, pp 3560-3562. Kolon TF, Herndon CDA, Baker LA, et al: Evaluation and treatment of Cryptorchidism: AUA GUIDELINE. American Urological Association Education and Research, Inc, 2014. <a href="http://www.auanet.org/education/guidelines/cryptorchidism.cfm" target="_new"><u>http://www.auanet.org/education/guidelines/cryptorchidism.cfm</u></a>

2015 - 102 A three-year-old boy is referred for bilateral undescended testes. On examination, both testes can be manipulated into a dependent scrotal position where they remain transiently upon fatigue of the cremasteric muscle. The next step is: A. follow-up in one year. B. LH and FSH measurement. C. ultrasound and measurement of testis volume. D. hCG injections. E. bilateral scrotal orchiopexies.

C The use of quick connectors for AUS allows for a secure and watertight connection between the reservoir, cuff, and pump in a freshly placed device. However, studies show they are associated with suboptimal results when they are used for revision surgery. Presumably, the decrease in success is due to the formation of the biofilm on pre-existing devices. Studies suggest that the biofilm interferes with the tubing connection using the quick connectors, and thus leads to a suboptimal connection that is at greater risk of leaking. Hand-tie connectors should be used in all AUS revision procedures. If the entire device is removed and replaced, the quick connectors may be used. Wessells H, Peterson AC: Surgical procedures for sphincteric incontinence in the male: The artificial genitourinary sphincter and perineal sling procedures, Wein, AJ, Kavoussi LR, Novick AC, Partin AW, Peters CA (eds): CAMPBELL-WALSH UROLOGY, ed 10. Philadelphia, Elsevier Saunders, 2012, vol 3, chap 79, p 2298.

2015 - 103 The use of quick connectors when performing an artificial urinary sphincter (AUS) revision: A. allows for the best, watertight connection. B. minimizes the risk of device infection peri-operatively. C. should not be used due to biofilm formation. D. should not be used due to tubing size mismatch. E. are equivalent to hand-tie connectors.

D Plastic bag specimens are unreliable and unacceptable for diagnosis of UTI in high-risk populations and infants. Generally, if a UTI is suspected in a child who is not yet toilet trained, only a catheterized or needle-aspirated specimen is acceptable for diagnosis because bagged urinary specimens have an unacceptably high false-positive rate. Under special collection circumstances when the perineum is cleaned well and the bag removed and processed promptly after voiding, a bagged specimen or even a diaper specimen that shows no growth is useful in eliminating bacteriuria as a diagnosis. In this infant, the urinalysis is equivocal and the risk of UTI is increased. Thus, a more definitive diagnosis with a catheterized urine specimen is needed prior to the initiation of any type of antibiotic therapy. She would likely be treated with oral antibiotic therapy as an outpatient (not observed or managed with I.V. antibiotic) only after a catheterized specimen has been obtained. Repeat ultrasound might be indicated if she has not responded to antibiotic therapy. Shortliffe LMD: Infection and inflammation of the pediatric genitourinary tract, Wein AJ, Kavoussi LR, Novick AC, Partin AW, Peters CA (eds): CAMPBELL-WALSH UROLOGY, ed 10. Philadelphia, Elsevier Saunders, 2012, vol 4, chap 116, pp 3096-3098.

2015 - 105 A four-month-old girl has a two-day history of fever. She is being followed for bilateral grade 2 hydronephrosis that was diagnosed antenatally. She is hemodynamically stable and no localizing signs are found on physical exam. A bagged urine specimen reveals 8-10 WBC/hpf and is nitrite positive. The next step is: A. observation. B. oral antibiotics. C. I.V. antibiotics. D. catheterized urine culture. E. repeat ultrasound.

E OnabotulinumtoxinA is indicated for neurogenic detrusor overactivity. However, the concerning urodynamic finding that can be seen in many patients with spina bifida, including this one, is loss of compliance and an elevated detrusor LPP. Elevated detrusor LPP specifically refers to a loss of compliance. A detrusor LPP > 40 cm H2O places patients at a greater risk for subsequent upper urinary tract damage. While this patient is now continent, that does not mean that his detrusor storage pressure are in a safe range and he should, therefore, undergo a repeat urodynamic study. If the detrusor pressures are in a safe range at appropriate volumes, then increasing the interval between catheterizations can be considered, and he will likely require repeat injection in six to nine months when his symptoms return. If the detrusor pressures are not appropriately lowered with onabotulinumtoxinA, then bladder augmentation should be considered. Chapple CR, Milsom I: Urinary incontinence and pelvic prolapse: Epidemiology and pathophysiology, Wein, AJ, Kavoussi LR, Novick AC, Partin AW, Peters CA (eds): CAMPBELL-WALSH UROLOGY, ed 10. Philadelphia, Elsevier Saunders, 2012, vol 3, chap 63, p 1877. Andersson KE, Wein AJ: Pharmacologic management of lower urinary tract storage and emptying failure, Wein, AJ, Kavoussi LR, Novick AC, Partin AW, Peters CA (eds): CAMPBELL-WALSH UROLOGY, ed 10. Philadelphia, Elsevier Saunders, 2012, vol 3, chap 68, pp 1986-1987.

2015 - 106 A 35-year-old man with spina bifida has urinary incontinence despite CIC every three hours. Urodynamics show detrusor overactivity and detrusor LPP of 60 cm H2O at 200 ml. Continence is achieved two weeks after intradetrusor injection of 200 units of onabotulinumtoxinA. The next step is: A. increase time between catheterizations. B. repeat onabotulinumtoxinA in six months. C. repeat onabotulinumtoxinA when incontinence returns. D. repeat onabotulinumtoxinA when urodynamic evidence of detrusor overactivity returns. E. repeat urodynamics now.

A Kaposi's sarcoma is the 2nd most common malignancy of the penis (after squamous cell carcinoma) as a result of the prevalence of HIV infection. It also occurs in patients on immunosuppression for organ transplantation or other indications. In this setting, Kaposi's sarcoma often regresses with modification of the immunosuppressive regimen and this should be the initial approach. If the tumor fails to respond to these efforts, local excision, laser ablation, or radiation should be considered. Partial penectomy is not indicated for this tumor type. 5-FU is typically utilized in squamous cell carcinoma of the penis. Link RE: Cutaneous diseases of the external genitalia, Wein, AJ, Kavoussi LR, Novick AC, Partin AW, Peters CA (eds): CAMPBELL-WALSH UROLOGY, ed 10. Philadelphia, Elsevier Saunders, 2012, vol 1, chap 15, pp 461-462.

2015 - 107 A 59-year-old man with a history of liver transplantation has a 1 cm raised, tender, penile lesion at the coronal sulcus dorsally. Incisional biopsy reveals Kaposi's sarcoma. The next step is: A. decrease immunosuppression. B. topical 5-FU. C. local excision. D. CO2 laser ablation. E. partial penectomy.

B Following valve ablation, upper tract changes may take months or even years to resolve. However, in most cases, a trend towards improvement is noted in the first three to six months after valve ablation. When there are persistent upper tract changes, the first step is to perform a VCUG to determine if the valves are adequately ablated and whether or not reflux is present. Endoscopic valve ablation can be a technically challenging procedure in infants, and it is not always clear following initial surgery as to whether adequate ablation has occurred. In cases where the follow-up VCUG is unclear as to whether adequate valve ablation has occurred, repeat endoscopy is warranted. If there is no evidence of residual valves on the VCUG, then a careful assessment of bladder function is needed to determine if there is bladder dysfunction that is impairing upper tract drainage. Urodynamics can be performed to help tailor bladder management. In some patients CIC and/or pharmacotherapy may be needed. It is uncommon for upper tract dilation to be secondary to a true ureterovesical obstruction in this setting. Vesicostomy is usually reserved for patients in which initial valve resection cannot be performed due to urethral anatomy or in cases where CIC is needed and cannot be performed reliably. Although the role of supravesical diversion (bilateral cutaneous ureterostomy) is controversial, it is generally limited to those patients that continue to do poorly despite bladder level drainage. Observation is warranted only after the possibility of residual valves and bladder dysfunction has been adequately assessed. Changing prophylactic antibiotics will not lead to clinical improvement if there is persistent valve obstruction. MR urogram would simply confirm upper tract dilation, but would not be helpful in evaluating for persistent bladder outlet obstruction and is contraindicated in the setting of compromised renal function. Casale AJ: Posterior urethral valves, Wein, AJ, Kavoussi LR, Novick AC, Partin AW, Peters CA (eds): CAMPBELL-WALSH UROLOGY, ed 10. Philadelphia, Elsevier Saunders, 2012, vol 4, chap 126, p 3397.

2015 - 108 A newborn has endoscopic ablation of PUV. Three months later, he has a febrile UTI despite being maintained on prophylactic antibiotics. Follow-up ultrasound demonstrates persistent bilateral grade 4 hydroureteronephrosis. Creatinine is 1.0 mg/dl. The next step is: A. MR urogram. B. VCUG. C. urodynamics. D. change prophylactic antibiotics. E. vesicostomy.

B Semen pH is alkaline (pH > 7.2). This is due to the larger amount of alkaline fluid from the seminal vesicles (SVs) over the smaller volume of acidic fluid from the prostate. Very little alkaline fluid is provided by the vas, testes, or epididymides. Alkaline pH of semen is needed for correct function of sperm. The knowledge of semen volume and pH is very important because it is diagnostic, and can determine ejaculatory duct obstruction (EJDO) and congenital bilateral absence of the vas deferens (CBAVD) from other obstructive causes of azoospermia. Both EJDO and CBAVD have low volume acid pH semen due to obstruction and absence of the SVs respectively. Bilateral vasectomy or epididymal obstruction would result in normal semen volume and pH with azoospermia since the SVs are not obstructed. Turek PJ: Male reproductive physiology, Wein, AJ, Kavoussi LR, Novick AC, Partin AW, Peters CA (eds): CAMPBELL-WALSH UROLOGY, ed 10. Philadelphia, Elsevier Saunders, 2012, vol 1, chap 20, p 612.

2015 - 118 Normal semen pH (alkaline) is derived mainly from the: A. prostate. B. seminal vesicles. C. testes. D. vas deferens. E. epididymis.

E The first communication from the FDA regarding vaginal mesh occurred in October 2008 and focused on the risk of complications of surgically placed vaginal mesh to treat both stress urinary incontinence and pelvic organ prolapse (POP). In July 2011, the FDA published an updated safety communication regarding the use of transvaginal mesh placement for POP. These communications do not state that these techniques should not be done. In addition, while the communication does recommend that each surgeon obtain specialized training for each technique they use and be aware of the risks involved with that technique, it does not state that these procedures should only be done by physicians who are board certified in the specialty of Female Pelvic Medicine and Reconstructive Surgery. The statement does note that serious complications associated with transvaginally placed mesh for POP are not rare. The FDA summary of the literature also notes that the use of mesh in transvaginal POP repair introduces possible risks that would not be seen in a repair without mesh, lower mesh-related complication rates are seen with transabdominal mesh use compared to transvaginal repairs and that mesh augmented transvaginal POP repairs may result in a better objective outcome that does not translate into a better symptomatic outcome. Update on serious complications associated with transvaginal placement of surgical mesh for pelvic organ prolapse: FDA Safety Communication. <a href="http://www.fda.gov/MedicalDevices/Safety/AlertsandNotices/ucm262435.htm" target="_new"><u>http://www.fda.gov/MedicalDevices/Safety/AlertsandNotices/ucm262435.htm</u></a>

2015 - 109 According to the 2011 FDA notification, the use of vaginal mesh in pelvic organ prolapse surgery: A. should never be done. B. should only be done by physicians board-certified in Female Pelvic Medicine and Reconstructive Surgery. C. has equivalent complication rates compared to mesh placed via an abdominal approach. D. may result in rare complications if performed by an experienced vaginal surgeon. E. may result in improved anatomic but equivalent symptomatic outcomes compared to transvaginal repairs without mesh.

B Aminoglycosides remain a mainstay of treatment for life-threatening gram negative infections. The risk of nephrotoxicity is increased in the elderly, diabetics, and in patients with pre-existing renal insufficiency. However, the acuity of this patient's pyelonephritis makes those considerations secondary. Cephalosporins and beta-lactam antibiotics (imipenem) are generally contraindicated with a history of anaphylactic reaction to penicillin, even though the absolute risk of severe reaction appears to be quite low. There is no evidence that pre-treatment with diphenhydramine and hydrocortisone would further reduce this risk. Ciprofloxacin is not an ideal choice because the organism exhibits only intermediate sensitivity and antibiotic concentrations in the urine are lower in a kidney with markedly diminished function. Nitrofurantoin is only active in the urine and is not appropriate for the treatment of tissue infections. Sullivan JW, Bueschen AJ, Schlegel JU: Nitrofurantoin, sulfamethizole and cephalexin urinary concentration in unequally functioning pyelonephritic kidneys. J UROL 1975;114):343-347. M|ouml|rike K, Schwab M, Klotz U: Use of aminoglycosides in elderly patients. Pharmacokinetic and clinical considerations. DRUGS AGING 1997;10:259-277. Apter AJ, Kinman JL, Bilker WB, et al: Is there cross-reactivity between penicillins and cephalosporins? AM J MED 2006;119:354. Schaeffer AJ, Schaeffer EM: Infections of the urinary tract, Wein, AJ, Kavoussi LR, Novick AC, Partin AW, Peters CA (eds): CAMPBELL-WALSH UROLOGY, ed 10. Philadelphia, Elsevier Saunders, 2012, vol 1, chap 10, pp 276-278.

2015 - 11 A 78-year-old woman with history of anaphylactic reaction to penicillin, renal insufficiency (Cr 2.3) has right-sided flank pain and high fever. Recent culture revealed E. coli with sensitivity to nitrofurantoin, gentamicin, ceftriaxone, and intermediate sensitivity to ciprofloxacin. The next step is to admit her to the hospital and start: A. ciprofloxacin. B. gentamicin. C. imipenem. D. ceftriaxone with diphenhydramine and hydrocortisone. E. ciprofloxacin and nitrofurantoin.

C The finding of a papillary lesion at the ureteral margin needs further evaluation to fully ascertain the extent of tumors in the system prior to planning the correct therapy. An on-the-table flexible ureteroscopy would thus be useful in detecting more proximal tumors which if present may necessitate more extensive surgery. If only CIS or dysplasia is present, intraoperative endoscopy is not indicated, because visual identification of CIS is unlikely. Presuming that on-the-table ureteroscopy shows no additional tumors, you would then resect until negative margins are obtained. Nephroureterectomy and extensive ureteral resection would only be performed if the ureteroscopy demonstrated tumors at more proximal location. BCG is not indicated for a low grade ureteral tumor. Lerner SP, Sternberg CN: Management of metastatic and invasive bladder cancer, Wein, AJ, Kavoussi LR, Novick AC, Partin AW, Peters CA (eds): CAMPBELL-WALSH UROLOGY, ed 10. Philadelphia, Elsevier Saunders, 2012, vol 3, chap 82, p 2360.

2015 - 110 During a radical cystectomy a small 3 mm papillary lesion is noted at the proximal margin of the transected ureter. Frozen section reveals a low grade Ta urothelial cell carcinoma. The next step is: A. re-resect ureter until margin negative. B. treat with BCG via diversion stent post-op. C. flexible ureteropyeloscopy. D. resect ureter and ileal ureter interposition. E. nephroureterectomy.

C The flow rate is abnormal but there is no anatomic obstruction. This boy has primary bladder neck dysfunction since the bladder contraction precedes the onset of urine flow by a significant period of time. The treatment of choice would be alpha-1 receptor blockade for a period of several weeks to months followed by re-evaluation clinically, and repeat uroflow with EMG. External sphincter dyssynergia can be treated with botulinum toxin injection or pelvic floor retraining (biofeedback). Double voiding may decrease his PVR and improve his continence, but this will not address the primary problem. CIC would 'bypass' the blockage but would not be the preferred primary treatment. Van Batavia JP, Combs AJ, Horowitz M, Glassberg KI. Primary bladder neck dysfunction in children: results of long-term alpha blocker therapy. J UROL 2010;183:724-730. Wein AJ, Dmochowski RR: Neuromuscular dysfunction of the lower urinary tract, Wein, AJ, Kavoussi LR, Novick AC, Partin AW, Peters CA (eds): CAMPBELL-WALSH UROLOGY, ed 10. Philadelphia, Elsevier Saunders, 2012, vol 3, chap 65, p 1938.

2015 - 111 A ten-year-old boy has day and night urinary incontinence. Uroflow with simultaneous EMG reveals a maximum flow rate of 7 ml/sec with good relaxation of the pelvic musculature and an eight second lag between the onset of EMG quiescence and initiation of flow. VCUG shows a normal urethra with a PVR of 100 ml. The next step is: A. double voiding. B. pelvic floor retraining. C. alpha-blockers. D. CIC. E. external sphincteric injection with onabotulinumtoxinA.

A Classically, cryoablation is used to treat tumors < 4 cm in size, with the laparoscopic approach used being dependent on the location of the tumor. Posterior and lateral tumors are usually approached retroperitoneally, and anterior or anterolateral tumors were approached transperitoneally. Theoretically, hilar tumors are more difficult to treat than peripherally located tumors due to the heat-sink effect caused by the renal blood flow within the renal hilar vessels. As such, target temperatures may be more difficult to achieve consistently throughout the entire tumor located near the renal hilum. Specific measures required to optimize tumor destruction during laparoscopic renal cryoablation include: 1) use of real-time intraoperative ultrasound to guide and confirm probe placement during laparoscopic insertion, 2) use of the ultrasound probe to monitor progression of the cryo lesion, 3) placement of the cryo probe tip at or just beyond the deepest or inner margin of the tumor, 4) obtaining target temperatures below -40 degrees Celsius during treatment, 5) extending the cryo lesion approximately 1 cm beyond the margin of the tumor, 6) performing an active double freeze-thaw cycle, as opposed to single cycle. Kavoussi LR, Schwartz MJ, Gill IS: Laparoscopic surgery of the kidney, Wein, AJ, Kavoussi LR, Novick AC, Partin AW, Peters CA (eds): CAMPBELL-WALSH UROLOGY, ed 10. Philadelphia, Elsevier Saunders, 2012, vol 2, chap 55, pp 1662-1664. White WM, Kaouk JH: Ablative therapy for renal tumors, Wein, AJ, Kavoussi LR, Novick AC, Partin AW, Peters CA (eds): CAMPBELL-WALSH UROLOGY, ed 10. Philadelphia, Elsevier Saunders, 2012, vol 2, chap 56, pp 1672-1673.

2015 - 112 Optimal tumor destruction during laparoscopic renal cryoablation is achieved by: A. intraoperative ultrasound-guided cryo probe placement. B. placing the cryo probe tip at the center of the tumor. C. obtaining a target temperature of -10∞C. D. extending the cryo lesion 2 cm beyond the tumor margin. E. performing an active single freeze-thaw cycle.

A Every reasonable effort should be made to obtain a negative proximal margin before re-implantation when a frank tumor is encountered at the margin. However, the findings of CIS at the ureteral margin (either at the time of frozen section or on final pathology) is more uncertain. The group at Memorial Sloan Kettering has questioned the value of achieving a negative margin because this did not alter the risk of development of subsequent upper tract tumor and CIS of the ureter is not independently associated with a worse outcome following cystectomy. Cancer recurrence at the anastomosis is rare even with a positive margin showing CIS, but a positive margin is a risk factor for developing a second primary tumor of the ureter or renal pelvis. Schumacher and colleagues demonstrated that upper tract recurrences occur in 3% to 5% of patients, and they are usually at sites distant from the anastomosis. However, they found no correlation between frozen and permanent section findings in their cohort. Accordingly, the data would suggest that patients with CIS at the ureteral margin may have a mildly increased risk of an upper tract recurrence (often remote from the margin, either on the ipsilateral or contralateral side). As a result, such patients (like all patients with invasive bladder cancer) require close follow-up with upper tract surveillance. Although the most commonly performed method of upper tract surveillance is with imaging (e.g., CT urogram), the most sensitive means involves surveillance ureteroscopy, and this can be used in patients with a very high degree of suspicion for upper tract recurrence. The median time to occurrence in one recent series was 53 months. Pre-emptive antegrade brush biopsy is not indicated at this time in the absence of obstruction or other abnormalities in imaging. BCG is also not indicated, as the finding of CIS at the margin only suggests a slightly increased incidence of recurrence, and often this recurrence is at a location remote from the margin site. Similarly, pre-emptive re-implantation or ipsilateral nephroureterectomy are not indicated or warranted as most patients will not have a local recurrence or ipsilateral upper tract recurrence. Lerner SP, Sternberg CN: Management of metastatic and invasive bladder cancer, Wein, AJ, Kavoussi LR, Novick AC, Partin AW, Peters CA (eds): CAMPBELL-WALSH UROLOGY, ed 10. Philadelphia, Elsevier Saunders, 2012, vol 3, chap 82, p. 2360.

2015 - 33 A 66-year-old man undergoes a radical cystectomy and ileal conduit for pT2N0 urothelial carcinoma of the bladder. Final pathology demonstrates CIS at the right ureteral margin. The next step is: A. surveillance. B. brush biopsy of ureteral anastomosis. C. BCG via nephrostomy tube. D. distal ureterectomy and reimplantation. E. nephroureterectomy.

D During laparoscopic surgery, electrosurgically induced thermal injury may occur via one of four mechanisms: inappropriate direct activation, coupling to another instrument, capacitive coupling, and insulation failure. Intraoperatively, thermal injuries of the bowel may present as whitish spots on the serosal lining. In severe cases, the muscularis mucosae or the intestinal lumen may be seen. However, in many patients, thermal injury of the bowel is not realized at the time of the procedure. Postoperatively, the patient with unrecognized bowel trauma may not develop fever, nausea, or signs of peritonitis for three to seven days; the full extent of the bowel necrosis may take up to 18 days to fully develop. Therefore, the problem often does not become manifest until the patient has been discharged. Accordingly, bowel injury must be ruled-out for any patient who develops a fever beyond postoperative day one or who complains of increasing abdominal discomfort. Abdominal radiographs are notoriously inaccurate because the carbon dioxide from the laparoscopy may remain as "free air" for more than two weeks after the procedure; however, an ileus pattern is usually present. An abdominal ultrasound will similarly be nonspecific and may detect loops of bowel or free fluid. The more sensitive test is an abdominal CT scan with oral contrast and delayed films. Minor postoperative thermal injuries of the bowel may be managed conservatively, aided by administration of antibiotics and an elemental diet. However, if the patient does not respond rapidly or develops worsening peritonitis, open surgical exploration is mandatory. Thermal injury caused by monopolar cautery often results in tissue damage that extends beyond the visible area of necrosis. With this in mind, the surgeon should perform a bowel resection with a safety margin of 6 cm on either side before completing an end-to-end anastomosis. Thermal injury caused by bipolar electrosurgery is more confined to the visible area of damage; thus, if the injury is small, it can be managed by simple excision of the defect and closure of the bowel wall. Injuries that involve more than half of the circumference of the bowel should be treated by excision of the affected bowel segment and end-to-end anastomosis. Eichel L, Clayman RV: Fundamentals of laparoscopic and robotic urologic surgery, Wein, AJ, Kavoussi LR, Novick AC, Partin AW, Peters CA (eds): CAMPBELL-WALSH UROLOGY, ed 10. Philadelphia, Elsevier Saunders, 2012, vol 1, chap 9, pp 242-244.

2015 - 34 A 57-year-old man develops fever, nausea, and increasing abdominal pain seven days following a laparoscopic nephrectomy. Despite bowel rest and antibiotics, he develops worsening symptoms. A KUB reveals free air in the abdominal cavity with dilated loops of small bowel. The next step is: A. abdominal ultrasound. B. barium enema. C. CT scan of the abdomen with I.V. contrast. D. CT scan of the abdomen with oral contrast. E. immediate surgical exploration.

A Urethral recurrence following radical cystectomy and orthotopic neobladder is a rare event with a reported incidence of 2-3%. While invasive tumors are an indication for total urethrectomy and ileal conduit, noninvasive tumors can be managed initially with conservative therapy. For papillary tumors, TUR alone may be curative for low grade recurrences. Intraurethral chemotherapy and immunotherapy have been used for high grade noninvasive recurrences. In this patient with CIS, an initial attempt at preservation of the urethra and neobladder is warranted. For this reason, intraurethral BCG is the most appropriate choice. Huguet J, Palou J, Serrallach M, et al: Management of urethral recurrence in patients with Studer ileal neobladder. EUR UROL 2003;43:495-498. Sharp DS, Angermeier KW: Surgery of penile and urethral carcinoma, Wein, AJ, Kavoussi LR, Novick AC, Partin AW, Peters CA (eds): CAMPBELL-WALSH UROLOGY, ed 10. Philadelphia, Elsevier Saunders, 2012, vol 1, chap 35, p 950.

2015 - 113 A 70-year-old man develops hematuria 18 months following a radical cystectomy and orthotopic neobladder. A urine cytology is positive for malignant cells. On cystoscopy, a focal area of erythema in the bulbar urethra reveals CIS. The next step is: A. intraurethral BCG. B. segmental urethrectomy. C. segmental urethrectomy with perineal urethrostomy. D. total urethrectomy. E. XRT.

B The cremasteric reflex implies an intact reflex arc in the genital branch of the genitofemoral nerve. Both the iliohypogastric and ilioinguinal nerve supply motor function to the internal oblique and transversus abdominis. The obturator nerve is the motor nerve for muscles in the medial thigh compartment. The femoral nerve supplies motor function for the anterior thigh compartment. The lateral femoral cutaneous nerve does not have a motor function and supplies sensation to the lateral aspect of the thigh. Chung BI, Sommer G, Brooks JD: Anatomy of the lower urinary tract and male genitalia, Wein, AJ, Kavoussi LR, Novick AC, Partin AW, Peters CA (eds): CAMPBELL-WALSH UROLOGY, ed 10. Philadelphia, Elsevier Saunders, 2012, vol 1, chap 2, p 47.

2015 - 114 The nerve that must be intact for the cremasteric reflex is: A. iliohypogastric. B. genitofemoral. C. lateral femoral cutaneous. D. ilioinguinal. E. femoral.

C For patients with chronic bicarbonate losses due to diarrhea, citrate should be employed to correct the resulting acidosis. It is recommended that a liquid preparation of potassium citrate be used rather than the slow-release tablet preparation because the slow-release medication may be poorly absorbed due to trapid intestinal transit time. Lemonade therapy is unlikely to normalize citrate levels, and the volume of fluid required will exacerbate the patient's diarrhea. Sodium citrate is not preferred, due to the sodium load which can promote hypercalciuria. Magnesium oxide and pyridoxine are useful treatments for hyperoxaluria, but not for hypocitraturia. Ferrandino MN, Pietrow PK, Preminger GM : Evaluation and medical management of urinary lithiasis, Wein, AJ, Kavoussi LR, Novick AC, Partin AW, Peters CA (eds): CAMPBELL-WALSH UROLOGY, ed 10. Philadelphia, Elsevier Saunders, 2012, vol 2, chap 46, p 1316.

2015 - 115 A 32-year-old man with chronic diarrhea is diagnosed with hypocitraturia. The best therapy is: A. citrate repletion with lemonade. B. potassium citrate tablet. C. potassium citrate liquid. D. sodium citrate liquid. E. magnesium oxide tablet and pyridoxine hydrochloride tablet.

C In a randomized clinical try, observation alone after successful BCG induction has been associated with a high recurrence rate when compared to three weekly maintenance BCG treatments at three months, six months and then every six months for two years. Prior randomized trials using other maintenance schedules have not been associated with a significant decrease in recurrence rates and, therefore, intravesical BCG on a monthly basis is not correct and there is no indication to initiate repeat induction BCG therapy. Monthly mitomycin maintenance has not been evaluated following BCG response in the setting of CIS. Lamm DL, Blumenstein BA, Crissman JD, et al: Maintenance bacillus Calmette-Guerin immunotherapy for recurrent TA, T1 and carcinoma in situ transitional cell carcinoma of the bladder: A randomized Southwest Oncology Group Study. J UROL 2000,163:1124-1129. Hall MG, Chang SS, Dalbagni G, et al: Management of nonmuscle invasive bladder cancer: (Stages Ta, T1, and Tis): AUA GUIDELINE. American Urological Association Education and Research, Inc, 2007. <a href="http://www.auanet.org/education/guidelines/bladder-cancer.cfm" target="_new"><u>http://www.auanet.org/education/guidelines/bladder-cancer.cfm</u></a>

2015 - 116 A 60-year-old man with CIS of the bladder received an induction course of BCG. Follow-up biopsy shows no residual tumor, and cytology shows mild atypia. The next step is: A. cystoscopy and cytology in three months. B. monthly BCG for one year. C. three weeks of BCG at three and six months, then every six months for two years. D. intravesical mitomycin C for six weeks. E. an additional six weeks of BCG.

D Congenital abnormalities of the urachus include a urachal sinus, urachal cyst, patent urachus, vesicourachal diverticulum, and omphalomesenteric cyst/duct. Urachal abnormalities occur equally between boys and girls and present more often in adulthood with umbilical drainage followed by an umbilical mass and periumbilical pain. Staphylococcus aureus is the most common bacteria cultured from an infected sinus tract or cyst. An important aspect of treatment is initial antibiotic therapy. Incision and drainage or percutaneous drainage should be considered for a loculated abscess, particularly when it is non-responsive to antibiotic therapy. Omphalomesenteric ducts may connect the midgut to the umbilicus. Ultrasound will demonstrate a periumbilical mass and a sinogram will reveal communication of the umbilical sinus to the ileum. Definitive treatment centers on excision that should be delayed for a few weeks following the initial antibiotic therapy in order to decrease the inflammatory response. This will help limit the amount of bladder that may need to be resected and will reduce the risk of injury to adjacent intra-peritoneal structures. Frimberger DC, Kropp BP: Bladder anomalies in children, Wein, AJ, Kavoussi LR, Novick AC, Partin AW, Peters CA (eds): CAMPBELL-WALSH UROLOGY, ed 10. Philadelphia, Elsevier Saunders, 2012, vol 4, chap 125, pp 3381-3384.

2015 - 117 A three-year-old boy has a fever of 100∞F, WBC count of 17,000, and purulent umbilical drainage. Pelvic ultrasound reveals a 2 cm complex mass between the umbilicus and dome of the bladder. The next step is antibiotics and: A. cystoscopy. B. incision, drainage and packing. C. excision of urachus and partial cystectomy. D. delayed excision of urachus. E. chemical sclerosis of the sinus tract.

C The size of residual masses is an important predictor of viable malignancy; 27% to 38% of discrete residual masses larger than 3 cm contain viable malignancy compared with 0% to 4% for masses < 3 cm. Moreover, surgical resection is technically difficult due to severe fibrosis, and often incomplete. The recommended management for this situation (small tumor < 3 cm) is observation with serial physical exam, serum markers, and CT scans. Recently, FDG-PET has been found to be a useful adjunct to CT to select patients for post-chemotherapy surgery. Thus, patients with discrete residual masses > 3 cm should be evaluated further with FDG-PET and those who are PET positive should undergo surgery. PET-negative residual masses > 3 cm and masses < 3 cm (as above) should be observed. This management should be distinguished from individuals with mixed germ cell tumors or non-seminomatous germ cell tumors in whom RPLND is indicated for the vast majority of residual masses within the retroperitoneum. Stephenson AJ, Gilligan TD: Neoplasms of the testis, Wein, AJ, Kavoussi LR, Novick AC, Partin AW, Peters CA (eds): CAMPBELL-WALSH UROLOGY, ed 10. Philadelphia, Elsevier Saunders, 2012, vol 1, chap 31, p 865.

2015 - 119 A 21-year-old man who underwent inguinal orchiectomy for a pure seminoma of the right testis has an 11 cm retroperitoneal mass. Serum beta-hCG and AFP are normal. Following three cycles of BEP, repeat CT scan demonstrates a residual 2 cm mass in the inter-aortocaval region. Chest CT scan is negative, and tumor markers remain normal. The next step is: A. local excision of the mass. B. RPLND. C. observation. D. salvage chemotherapy. E. retroperitoneal XRT.

D This patient has T2N2M0S2 NSGCT, also categorized as clinical stage 2B. The standard treatment should be primary chemotherapy. The selection of chemotherapy regimen depends on the International Germ Cell Cancer Collaborative Group Risk Classification for Advanced Germ Cell Tumor (IGCCG) that includes location of primary tumor, metastases and tumor marker levels. This patient is considered intermediate risk based on the post orchiectomy AFP over 1,000 IU/ml, and all intermediate and high risk patients should receive four cycles of BEP. Stephenson AJ, Gilligan TD: Neoplasms of the testis, Wein AJ, Kavoussi LR, Novick AC, Partin AW, Peters CA (eds): CAMPBELL-WALSH UROLOGY, ed 10. Philadelphia, Elsevier Saunders, 2012, vol 1, chap 31, p 855.

2015 - 12 A 17-year-old boy has a left radical orchiectomy for a pathologic T2 5 cm tumor, which is 70% embryonal cancer and 30% teratoma. He has a 2 cm para-aortic adenopathy and no other visible metastases. His initial markers show an AFP of 7,000 IU/ml and a normal beta-hCG. Two weeks later, his beta-hCG is normal and his AFP is 5,000 IU/ml. The next step is: A. repeat tumor markers in two weeks. B. three cycles BEP. C. four cycles etoposide and cisplatin. D. four cycles of BEP. E. RPLND.

A Children who initially fail an alarm system in the treatment of nocturnal enuresis will benefit from combination therapy e.g., enuretic alarm with pharmacological treatment. Comparison of estimated bladder capacity (age in years x 30 ml + 60 ml = bladder capacity) divided by the mean of the voided diurnal urine volumes obtained on a voiding diary can guide combination therapy. Patients with values of > 70% (estimated bladder capacity/mean diurnal voided volume) will have a better response to the combination of a enuretic alarm and DDAVP. Response to enuretic alarm and anticholinergic medication is better in patients with < 70% (estimated bladder capacity/ mean diurnal voided volume). Yeung CK, Sihoe JDY: Non-neuropathic dysfunction of the lower urinary tract in children, Wein, AJ, Kavoussi LR, Novick AC, Partin AW, Peters CA (eds): CAMPBELL-WALSH UROLOGY, ed 10. Philadelphia, Elsevier Saunders, 2012, vol 4, chap 127, p 3427. Andersson KE, Wein AJ: Pharmacologic management of lower urinary tract storage and emptying failure, Wein, AJ, Kavoussi LR, Novick AC, Partin AW, Peters CA (eds): CAMPBELL-WALSH UROLOGY, ed 10. Philadelphia, Elsevier Saunders, 2012, vol 3, chap 68, p 1994.

2015 - 120 A nine-year-old boy has primary nocturnal enuresis. He has daytime urgency and frequency, but remains dry. He has used an alarm for four months without improvement. Urinalysis is normal. The next step is: A. voiding diary. B. urodynamics. C. VCUG. D. DDAVP. E. oxybutynin.

A Cystic fibrosis (CF) is a progressive, lifelong condition in which the glands that produce mucus, sweat, and intestinal secretions produce abnormally high amounts of NaCl. The abnormal functioning of the epithelial cells result in thick mucus accumulation in the lungs, leading to breathing difficulty and repeated respiratory infections, impaired digestion due to thick mucus blocking the pancreatic ducts with failure of pancreatic enzymes secretion, and infertility (> 98% of affected men are infertile) due to vasal atrophy and dissolution. Vasal dissolution presumably occurs secondary to mucus build-up within the vasal lumen and pressure induced atrophy. Due to the thick build-up of mucous within the lungs and recurrent pulmonary infections, many of the patients with CF are exposed to prolonged antibiotic use resulting in the loss of the intestinal bacteria, Oxalobacter formigenes. This bacteria normally metabolizes intestinal oxalate, reducing the amount of oxalate available for systemic absorption. Due to the lack of digestive pancreatic enzymes, cystic fibrosis patients will frequently have intestinal malabsorption resulting in diarrhea, caloric deficiencies and deficiencies in the absorption of Vitamins A, D, E, and K and the minerals of calcium, magnesium, iron, sodium chloride, and zinc. To combat these deficiencies, CF patients are encouraged to eat a diet high in calories, fat, and salt. The elevation in salt intake is critical in CF patients due to the high salt content (> 10 fold normal) found in the sweat of affected patients. Individuals with cystic fibrosis will, therefore, not have absorptive hypercalciuria; indeed, they have a failure to absorb calcium from their GI tract due to saponification of calcium by the high fat content of their stools. They will have elevated levels of urinary sodium due to increased sodium intake. CF patients do have diarrhea with resultant hypocitraturia and hypomagnesuria; however, replacement of citrate or magnesium have not been found to reduce stone formation within this patient population. The chief cause of stones in patients with CF is the excess oxalate absorbed due to absence of Oxalobacter formigenes, the saponification of calcium within the intestine, and low urinary volume secondary to fluid depletion from diarrhea. Renal tubular acidosis is not part of the cystic fibrosis disease complex. Pearle MS, Lotan Y: Urinary lithiasis: Etiology, epidemiology, and pathogenesis, Wein, AJ, Kavoussi LR, Novick AC, Partin AW, Peters CA (eds): CAMPBELL-WALSH UROLOGY, ed 10. Philadelphia, Elsevier Saunders, 2012, vol 2, chap 45, p 1273.

2015 - 121 A 24-year-old woman with cystic fibrosis undergoes ureteroscopic treatment of a calcium oxalate stone. The most likely etiology of stone formation is: A. absent Oxalobacter formigenes. B. absorptive hypercalciuria. C. reduced urinary magnesium. D. reduced urinary sodium. E. renal tubular acidosis.

A Primary hyperaldosteronism (Conn's Syndrome) due to adrenal hypersecretion of aldosterone causes hypertension and hypokalemia in non-edematous patients. Conn's syndrome accounts for about 1% of hypertensive patients, usually women between the ages of 30-50 years. Patients typically have hypertension, muscle weakness, polyuria, hypokalemia, and mild metabolic acidosis. However, essential hypertension treated with diuretics is the most common cause of hypokalemia in patients with hypertension. Secondary hyperaldosteronism can be due to renal artery stenosis, cardiac failure, hepatic cirrhosis, pregnancy and Bartter's Syndrome (due to juxtaglomerular cell hyperplasia). Kutikov A, Crispen PL, Uzzo RG: Pathophysiology, evaluation, and medical management of adrenal disorders, Wein AJ, Kavoussi LR, Novick AC, Partin AW, Peters CA (eds): CAMPBELL-WALSH UROLOGY, ed 10. Philadelphia, Elsevier Saunders, 2012, vol 2, chap 57, pp 1697-1703.

2015 - 122 A 32-year-old woman has a two year history of hypertension treated with a diuretic. BP is 150/95, sodium 135 mEq/l, creatinine 0.8 mg/dl and potassium 2.7 mEq/l. The most likely diagnosis is: A. essential hypertension. B. aldosteronoma. C. adrenal hyperplasia. D. fibromuscular dysplasia. E. Bartter's syndrome.

C Stranguria and gross hematuria in a child with a normal ultrasound suggests a possible urethral abnormality, such as congenital urethral polyp, or urethral stricture. A VCUG will likely be diagnostic. CT scan and MRI are unlikely to add useful information in a patient with a normal renal and bladder ultrasound and urine cytology is not indicated. Cystoscopy may be indicated based on the findings of the VCUG. Casale AJ: Posterior urethral valves, Wein, AJ, Kavoussi LR, Novick AC, Partin AW, Peters CA (eds): CAMPBELL-WALSH UROLOGY, ed 10. Philadelphia, Elsevier Saunders, 2012, vol 4, chap 126, p 3407.

2015 - 123 A two-year-old boy with stranguria develops gross hematuria. Urine culture is negative. Ultrasound shows normal kidneys and bladder. The next step is: A. CT urogram. B. urine cytology. C. VCUG. D. MRI scan with gadolinium. E. cystoscopy.

E Asymptomatic bacteriuria has not been demonstrated to cause harm in most adults. While these patients are at risk for developing symptomatic UTIs, treatment of the asymptomatic bacteriuria does not result in fewer symptomatic UTIs. In contrast, treatment of asymptomatic bacteriuria is indicated in pregnant women and patients undergoing urologic procedures because they are at increased risk of developing adverse outcomes if left untreated. The presence of diabetes is not an indication to screen for or treat asymptomatic bacteriuria. Neither imaging studies nor cystoscopy is indicated if the bacteriuria is asymptomatic. In this setting, a culture should not be sent because it will not affect management. Schaeffer AJ, Schaeffer EM: Infections of the urinary tract, Wein, AJ, Kavoussi LR, Novick AC, Partin AW, Peters CA (eds): CAMPBELL-WALSH UROLOGY, ed 10. Philadelphia, Elsevier Saunders, 2012, vol 1, chap 10, p 288.

2015 - 124 A 35-year-old woman with diabetes has asymptomatic bacteriuria on a routine urine analysis. The next step is: A. antibiotics. B. antibiotics followed by regular screening urinalyses. C. imaging of the urinary tract. D. antibiotics followed by cystoscopy. E. no treatment.

A Cytoreductive nephrectomy as part of a multidisciplinary approach to metastatic RCC has been shown to prolong survival in two randomized clinical trials. However, its use is controversial and proper patient selection is crucial. The following inclusion criteria have been suggested for selecting patients for cytoreductive nephrectomy: 1) ability to resect more than 75% of the tumor volume, 2) no brain metastases, 3) adequate pulmonary and cardiac reserve, 4) ECOG performance status of 0 or 1, and 5) predominantly clear cell histology. Although sarcomatoid cell histology has been associated with poor prognosis it is not a contraindication to cytoreductive nephrectomy. Patients with brain metastasis should be treated with stereotactic radiation or CNS surgery prior to consideration for cytoreductive nephrectomy. Srinivasan R, Linehan WM: Treatment of advanced renal cell carcinoma, Wein, AJ, Kavoussi LR, Novick AC, Partin AW, Peters CA (eds): CAMPBELL-WALSH UROLOGY, ed 10. Philadelphia, Elsevier Saunders, 2012, vol 2, chap 50, p 1478.

2015 - 125 A contraindication to cytoreductive nephrectomy prior to systemic therapy in an asymptomatic patient with metastatic RCC is: A. brain metastasis. B. vena caval thrombus. C. involvement of the contralateral adrenal gland. D. sarcomatoid cell histology. E. pulmonary metastases.

A Minor degrees of separation of circumcision edges are common. Complete separation as described is uncommon. This incision should be considered contaminated in the baby's diaper. Therefore, immediate closure is not recommended. Skin grafts are not indicated because of the contaminated bed and would have a high risk of infection. Since the length of the skin was adequate at the time of circumcision, observation is the best choice. Usually this complication rapidly heals well and nothing further will be necessary. If an undesirable scar develops, it can be revised or grafted electively at a later time. Palmer JS: Abnormalities of the external genitalia in boys, Wein, AJ, Kavoussi LR, Novick AC, Partin AW, Peters CA (eds): CAMPBELL-WALSH UROLOGY, ed 10. Philadelphia, Elsevier Saunders, 2012, vol 4, chap 131, pp 3540-3541.

2015 - 126 Twenty-four hours after a newborn circumcision, a circumferential skin separation with a 1.5 cm defect is noted. The best management is: A. observation with local wound care. B. immediate repair. C. delayed repair. D. split thickness skin graft. E. full thickness skin graft.

C The main purpose of the one shot IVP is to assess for the presence of a functioning contralateral kidney and to assess the degree of damage to the ipsilateral kidney and ureter when a non-expanding, non-pulsatile retroperitoneal hematoma is encountered during abdominal exploration for a GSW in the absence of preoperative imaging. Ultrasound cannot delineate parenchymal lacerations, vascular injury, or collecting system or ureteral injuries in the acute setting. Renal arteriography is not indicated in the acute, intra-operative setting. Expectant management has been advocated as long as there is not an expanding or pulsating hematoma and renal pelvic or ureteral injury can be excluded by pre-operative CT scan or pre- or intra-operative one-shot IVP. Because of the risk of renal loss with exploration, a conservative approach is advocated and has been associated with a lower nephrectomy rate. In the presence of an expanding or pulsatile retroperitoneal hematoma, renal exploration is indicated. Santucci RA, Doumanian LR: Upper urinary tract trauma, Wein, AJ, Kavoussi LR, Novick AC, Partin AW, Peters CA (eds): CAMPBELL-WALSH UROLOGY, ed 10. Philadelphia, Elsevier Saunders, 2012, vol 2, chap 42, pp 1172-1174.

2015 - 127 A 19-year-old man sustained a gunshot wound to the abdomen. Preoperative imaging was not performed. During emergent exploratory laparotomy, a non-expanding, non-pulsatile, left retroperitoneal hematoma is encountered. The next step is: A. observation. B. intraoperative sonography. C. one shot IVP. D. renal arteriography. E. left renal exploration.

B This patient has low risk clinically localized prostate cancer (normal DRE, PSA <10, Gleason score < 7, and minimal core involvement). Although there is no clinical or biochemical evidence at this point of disease progression, a repeat prostate biopsy within 18 months is an essential component of active surveillance and can provide important information useful in management. This is especially true for men electing active surveillance who have a life-expectancy > 10 to 15 years and are otherwise candidates for definitive therapy. A repeat biopsy may provide information that would lead one to consider local therapy. This includes grade progression and an increase in the percentage of the biopsy specimen involved with cancer. MRI remains investigational as a tool for monitoring patients on active surveillance. Bone scan and ProstaScintÆ scan are not indicated at this point in time. Eastham JA, Scardino PT: Expectant management of prostate cancer, Wein, AJ, Kavoussi LR, Novick AC, Partin AW, Peters CA (eds): CAMPBELL-WALSH UROLOGY, ed 10. Philadelphia, Elsevier Saunders, 2012, vol 3, chap 101, p 2799. Thompson I, Thrasher JB, Aus G, et al: Management of clinically localized prostate cancer: AUA GUIDELINE. American Urological Association Education and Research, Inc, 2007. <a href="http://www.auanet.org/education/guidelines/prostate-cancer.cfm" target="_new"><u>http://www.auanet.org/education/guidelines/prostate-cancer.cfm</u></a>

2015 - 128 An otherwise healthy 65-year-old man elects active surveillance for T1c, PSA 8.2 ng/dl, Gleason score 6 (1 of 12 cores positive) prostate cancer. His DRE remains normal and has subsequent serial PSA values of 7.1, 9.2, and 7.9 ng/dl at 6, 12, and 18 months respectively. The next step is: A. endorectal MRI scan. B. TRUS and prostate biopsy. C. bone scan. D. ProstaScint^TM scan. E. definitive local therapy.

C This baby has a clinically and functionally significant right distal ureterovesical junction obstruction associated with decreased renal function. The diagnostic parameters indicate that treatment is needed. Observation is inappropriate and may result in further loss of renal function and/or severe infection in this obstructed system. Definitive treatment would entail ureteral reimplantation with tapering. However, this is technically challenging in this age group and better deferred for a child older than one year of age. A percutaneous nephrostomy tube requires external drainage which is not practical when required longer than several weeks. The most appropriate temporizing procedure is a distal cutaneous ureterostomy. Another option to be considered is an anastomosis of the distal ureter to the bladder in a refluxing manner. Carr MC, Casale P: Anomalies and surgery of the ureter in children, Wein, AJ, Kavoussi LR, Novick AC, Partin AW, Peters CA (eds): CAMPBELL-WALSH UROLOGY, ed 10. Philadelphia, Elsevier Saunders, 2012, vol 4, chap 120, p 3215.

2015 - 129 A four-week-old boy has a febrile UTI. Renal ultrasound shows right hydroureteronephrosis with the distal ureter measuring 2.0 cm in diameter and a normal left kidney. VCUG is normal and a MAG-3 scan shows 80% function on the left and 20% on the right. T Ω on the right is > 30 minutes. The next step is antibiotics and: A. ultrasound and MAG-3 scan in three months. B. percutaneous nephrostomy. C. distal cutaneous ureterostomy. D. primary ureteral reimplant with tapering. E. nephrectomy.

B A particularly distressing postoperative complication following radical nephrectomy is the development of a pancreatic fistula because of an unrecognized intraoperative injury to the pancreas. This is usually manifested in the immediate postoperative period with signs and symptoms of acute pancreatitis and drainage of alkaline fluid from the incision. A CT scan of the abdomen demonstrates a fluid collection in the retroperitoneum. Fluid draining from the incision should be analyzed for pH and the presence of amylase. Treatment involves percutaneous drainage of the pseudocyst or abscess. The majority of fistulae close spontaneously with the establishment of adequate drainage. Because the healing of a pancreatic fistula is usually a slow process associated with significant nutritional loss, the patient is also supported with hyperalimentation. Surgical treatment with resection of the distal pancreas is necessary if nonoperative management fails. Open surgical drainage or ligation of the fistula would not be indicated and/or considered the treatment of choice. A low triglyceride diet would be indicated for a lymphatic leak. Kenney PA, Wotkowicz C, Libertino JA: Contemporary open surgery of the kidney, Wein, AJ, Kavoussi LR, Novick AC, Partin AW, Peters CA (eds): CAMPBELL-WALSH UROLOGY, ed 10. Philadelphia, Elsevier Saunders, 2012, vol 2, chap 54, p 1623.

2015 - 13 A 58-year-old man develops abdominal pain and fever to 101F three days after left radical nephrectomy. He is treated with I.V. antibiotics. The next day, the previously dry incision leaks 100 ml of cloudy fluid (pH 9.5, amylase 8,000 U/l). CT scan shows a 5 cm fluid collection in the left renal fossa. The next step is a naso-gastric tube and: A. low triglyceride diet. B. percutaneous drainage and TPN. C. open surgical drainage. D. open ligation of fistula site and drainage. E. distal pancreatectomy and drainage.

B Urethral "rest" involves a period of several weeks where the urethra is not instrumented. This "rest period" allows inflammation to decrease within the lumen and facilitates accurate identification of severely fibrotic segments of the stricture which will eventually require focal or complete excision. Suprapubic tube urinary diversion, removal of a urethral catheter after recent instrumentation, or cessation of CIC all accomplish urethral rest. The median duration of urethral rest is reported as three months in this study; however, repeat urethral staging can be repeated at four to six weeks. Urethroplasty following recent dilation, CIC, or DVIU may not accurately assess the location and severity of fibrotic urethral segments thus understaging the disease and placing the patient at a higher risk for recurrence following urethroplasty. DVIU is not indicated in any situation for a urethral stricture > 1.5 cm. The urethroplasty technique will depend on the degree and length of fibrotic urethra following urethral rest. Terlecki RP, Steele MC, Valadez C, Morey AF: Urethral rest: Role and rationale in preparation for anterior urethroplasty. UROL 2011;77:1477-1481.

2015 - 130 A 24-year-old man on CIC for a neurogenic bladder is unable to catheterize. Retrograde urethrogram reveals a 2 cm proximal bulbar urethral stricture. The next step is: A. direct vision internal urethrotomy and resume CIC. B. suprapubic tube placement. C. excision and primary anastomosis. D. bulbar urethroplasty with graft. E. bulbar urethroplasty with flap.

B Similar to retroperitoneal lymph node dissection for testicular cancer, chylous leak can occur after regional lymphadenectomy at the time of renal surgery. Care must be taken to ligate small lymphatic vessels, particularly those close to the cisterna chyli below the right crus. In a setting where a drain is left in place, the fluid will have a milky white appearance and will have elevated fat and total protein levels. While pancreatic injury and fistula needs to be considered in the setting of a resection of a large left-sided upper pole tumor, this is not as likely on the right side. In addition, the clinical presentation is not consistent with a pancreatic leak, as pancreatic fluid is usually clear. Similarly, a urine leak would present with clear fluid. Finally, an unrecognized bowel injury resulting in enterocutaneous fistula is unlikely given the appearance of the fluid. Therefore, an activated charcoal test would not be helpful. Eichel L, Clayman RV: Fundamentals of laparoscopic and robotic urologic surgery, Wein, AJ, Kavoussi LR, Novick AC, Partin AW, Peters CA (eds): CAMPBELL-WALSH UROLOGY, ed 10. Philadelphia, Elsevier Saunders, 2012, vol 1, chap 9, p 249.

2015 - 131 A 52-year-old man undergoes right partial nephrectomy and regional lymphadenectomy for a large upper pole tumor in a solitary kidney. A drain is placed in the renal fossa. The drain output increases precipitously when the patient begins a regular diet. The drain output is a milky white fluid. The test which will confirm the diagnosis is: A. drain fluid amylase and lipase level. B. drain fluid fat and total protein level. C. drain fluid creatinine level. D. activated charcoal test. E. abdominal CT scan with oral and I.V. contrast.

B Maintenance fluids are based on body weight. One formula is: 4 ml/kg/hr for the first 10 kg and 2 ml/kg/hr for the second 10 kg, then 1 ml/kg/hr for the remainder. Hence, for a 14 kg child, 48 ml/hr is appropriate (or approximately 50 ml/hr). Infants under six months are generally given 1/4 NS because of their high water needs per kg, but older than 6 months should receive 1/2 NS. Estrada CR Jr, Ferrari LR: Core principles of perioperative management in children, Wein, AJ, Kavoussi LR, Novick AC, Partin AW, Peters CA (eds): CAMPBELL-WALSH UROLOGY, ed 10. Philadelphia, Elsevier Saunders, 2012, vol 4, chap 119, p 3200.

2015 - 132 The appropriate maintenance I.V. fluid for a 14 kg child is: A. 25 ml/hr of 1/2 NS. B. 50 ml/hr of 1/2 NS. C. 50 ml/hr of Lactated Ringers Solution. D. 75 ml/hr of 1/4 NS. E. 75 ml/hr of 1/2 NS.

D The patient has suffered a significant blunt bulbar urethral injury. Dilation and DVIU would have a much lower success rate in this situation as compared to non-traumatic strictures, as traumatic strictures have significant spongiofibrosis. The best treatment is excision of the strictured urethral segment and primary anastomosis of healthy urethral mucosa and spongiosum. A patch graft could be considered, but the outcomes with a nearly obliterated urethra and significant spongiofibrosis would be inferior to excision and primary anastomosis. Mitomycin injection may be indicated in recurrent bladder neck contractures but is not indicated in this situation. Morey AF, Dugi DD III: Genital and lower urinary tract trauma, Wein, AJ, Kavoussi LR, Novick AC, Partin AW, Peters CA (eds): CAMPBELL-WALSH UROLOGY, ed 10. Philadelphia, Elsevier Saunders, 2012, vol 3, chap 88, p 2520.

2015 - 133 A 28-year-old man sustains a straddle injury with bulbar urethral disruption. A suprapubic tube is placed for three months. Cystoscopy and retrograde urethrogram reveal a 1 cm mid bulbar urethral stricture with a 4 Fr lumen. The next step is: A. dilation. B. direct visual internal urethrotomy with mitomycin injection. C. laser urethrotomy. D. excision and primary anastomosis. E. urethroplasty with graft.

D Bladder preservation with chemoradiation can be effective in well-selected patients. The use of radical TURBT and concurrent radiosensitizing chemotherapy has achieved ~50% five year survival. The most well-studied and effective chemotherapeutic agent used in this setting is cisplatin. Methotrexate, vinblastine, and doxorubicin are all important systemic agents with efficacy against bladder cancer; however, they do not have significant roles as concurrent treatments along with radiation therapy. Paclitaxel may have a selected role as a radiosensitizer, but to a much lesser degree than cisplatin. Lerner SP, Sternberg CN: Management of metastatic and invasive bladder cancer, Wein AJ, Kavoussi LR, Novick AC, Partin AW, Peters CA (eds): CAMPBELL-WALSH UROLOGY, ed 10. Philadelphia, Elsevier Saunders, 2012, vol 3, chap 82, pp 2365-2366.

2015 - 134 A 57-year-old man has a 2 cm cT2 urothelial carcinoma of the bladder. He opts for chemoradiation. The most effective concurrent chemotherapeutic agent is: A. methotrexate. B. vinblastine. C. doxorubicin. D. cisplatin. E. paclitaxel.

C Approximately 1% of children with unilateral Wilms' tumor develop contralateral disease. Children younger than 12 months diagnosed with Wilms' tumor who also have nephrogenic rests, in particular perilobar nephrogenic rests, have a markedly increased risk of developing contralateral disease. These children require frequent and regular surveillance for several years. They are not at higher risk for renal disease (other than that usually associated with nephrectomy, chemotherapy, radiation therapy), nor is she at greater risk for local or distant recurrence. Coppes MJ, Arnold M, Beckwith JB, Ritchey ML, et al: Factors affecting the risk of contralateral Wilms' tumor development: A report from the National Wilms' Tumor Study Group. CANCER 1999;85:1616-1625. Ritchey ML, Shamberger RC: Pediatric urologic oncology, Wein, AJ, Kavoussi LR, Novick AC, Partin AW, Peters CA (eds): CAMPBELL-WALSH UROLOGY, ed 10. Philadelphia, Elsevier Saunders, 2012, vol 4, chap 137, p 3711.

2015 - 135 A ten-month-old girl undergoes left nephrectomy for Wilms' tumor. The right kidney is normal at the time of surgery. Pathology reveals the presence of perilobar nephrogenic rests. She is at increased risk for: A. nephrotic syndrome. B. renal failure. C. metachronous Wilms' tumor. D. local recurrence. E. metastatic disease.

E Proximal and large ureteral defects are difficult to manage. The surgeon must be ready to use a variety of approaches depending on the intraoperative findings. Due to poor long term outcomes, endopyelotomy is contraindicated in strictures over 2 cm. Ureteroureterostomy is ideal for short upper or mid-ureteral strictures but is not possible with large defects. Transureteroureterostomy is contraindicated in those with nephrolithiasis. Ureterocalicostomy is reserved for individuals with UPJ and proximal 2-3 cm stricture with a dilated lower pole calyx, this patient has a length ureteral stricture to amenable to this procedure. Ileal ureter should be considered for long, > 4 cm upper ureteral defects, in patients with serum creatinines of < 2.0 mg/dl. Fallopian tube and appendiceal substitutions although described, are not reliable reconstructive techniques and are associated with recurrent stricture on long-term follow-up. Other options for this patient include autotransplantation or nephrectomy. Nephrectomy is usually considered as a last resort and is reserved for patients with a normal serum creatinine and poor function (classically < 20% on renal flow scan). Nakada SY, Hsu THS: Management of upper urinary tract obstruction, Wein, AJ, Kavoussi LR, Novick AC, Partin AW, Peters CA (eds): CAMPBELL-WALSH UROLOGY, ed 10. Philadelphia, Elsevier Saunders, 2012, vol 2, chap 41, p 1122.

2015 - 136 A 22-year-old woman with a prior history of right nephrolithiasis and ureteroscopic stone extraction has right flank pain. Serum creatinine is 1.0 mg/dl. A CT urogram demonstrates left hydronephrosis, no calculi and a 6 cm ureteral stricture extending from the UPJ to mid-ureteral region. Diuretic renal scan shows left renal function of 35% and T1/2 of 40 minutes after Lasix administration. The next step is: A. endopyelotomy. B. transureteroureterostomy. C. ureterocalicostomy. D. appendiceal ureteral substitution. E. ileal ureter.

C Approximately three quarters of urine specimens from ileal conduits are culture positive. Most adult patients show no untoward effects when exposed to chronic bacteriuria. Deterioration of the upper tracts is more likely when the culture becomes dominant for Proteus or Pseudomonas, and thus, these patients should receive antibiotic therapy to reduce the incidence of stone formation. Those patients with mixed cultures may generally be observed, provided they are not symptomatic. Further imaging in this asymptomatic patient is not indicated. Dahl DM, McDougal WS: Use of intestinal segments in urinary diversion, Wein, AJ, Kavoussi LR, Novick AC, Partin AW, Peters CA (eds): CAMPBELL-WALSH UROLOGY, ed 10. Philadelphia, Elsevier Saunders, 2012, vol 3, chap 85, p 2446.

2015 - 137 A 70-year-old asymptomatic woman has bladder cancer surveillance after cystectomy and ileal conduit diversion. Her family physician recently checked a random urine culture which revealed > 100,000 Proteus species. The next step is: A. observation. B. repeat urine culture. C. antibiotic therapy. D. loopogram. E. non-contrast CT scan.

A Post-traumatic hypertension can occur in the setting of renal ischemia, development of an arteriovenous (AV) malformation, or kidney compression from a hematoma. The ultrasound rules out infarction, venous thrombosis and global scarring. The nuclear scan rules out infarction and segmental and global scarring. In this case of delayed development of hypertension, the most likely cause is an AV fistula secondary to penetrating trauma. Diagnosis can be made with an MRI or CT angiogram. Successful treatment is usually achieved with angiographic embolization. In contrast to blunt or penetrating traumatic AV fistula, those occurring after a needle biopsy will often close spontaneously. Husmann DA: Pediatric genitourinary trauma, Wein, AJ, Kavoussi LR, Novick AC, Partin AW, Peters CA (eds): CAMPBELL-WALSH UROLOGY, ed 10. Philadelphia, Elsevier Saunders, 2012, vol 4, chap 138, p 3741.

2015 - 138 An 11-year-old boy is shot by a .22 caliber rifle and suffers a penetrating injury to the left renal cortex. The injury did not require surgery. Six months later, he develops headaches and is found to be severely hypertensive. Both kidneys measure 10 cm on ultrasound. Renal nuclear scan shows equal function of both kidneys with no photopenic areas. The most likely cause of his hypertension is: A. arteriovenous fistula. B. global renal scarring. C. renal infarction. D. renal vein thrombosis. E. segmental renal scarring.

B This patient has sustained a high velocity urethral injury. The best treatment is suprapubic tube placement followed by staged urethroplasty. The blast effect of the high velocity missile will cause significant collateral tissue damage which will evolve over the next several days. Suprapubic urinary diversion is the best course of action as compared to a urethral catheter which may compound the injury, followed by restaging the injury and eventual staged urethroplasty. Penile urethroplasty with or without graft immediately following a high velocity injury is not indicated as the full extent of the urethral injury is not known. The patient will eventually require a urethroplasty; however, this should be performed following suprapubic urinary diversion and restaging of the injury. Morey AF, Dugi DD III: Genital and lower urinary tract trauma, Wein, AJ, Kavoussi LR, Novick AC, Partin AW, Peters CA (eds): CAMPBELL-WALSH UROLOGY, ed 10. Philadelphia, Elsevier Saunders, 2012, vol 3, chap 88, p 2509.

2015 - 139 A 24-year-old man is shot with a high velocity hand gun in the mid-penile shaft. CT scan of the abdomen and pelvis reveals no other injuries. Retrograde urethrogram shows contrast extravasation from a 2 cm segment of the mid-penile urethra. The next step is: A. cystoscopy and primary realignment. B. suprapubic tube placement. C. debridement and primary closure. D. debridement and urethroplasty with graft. E. debridement and urethroplasty with flap.

A The sodium nitroprusside spot test will turn urine purple in the presence of cystine. This test is used for screening purposes to identify patients with cystine stone disease who are undergoing a 24 hour urine collection for evaluation. Phenolphthalein is a urinary marker for laxative abuse and may be helpful in the diagnosis of ammonium acid urate stones. A thiazide challenge may be helpful in the diagnosis of hyperparathyroidism. Serum pH and serum chloride may be helpful in the diagnosis of RTA type I. Ferrandino MN, Pietrow PK, Preminger GM: Evaluation and medical management of urinary lithiasis, Wein AJ, Kavoussi LR, Novick AC, Partin AW, Peters CA (eds): CAMPBELL-WALSH UROLOGY, ed 10. Philadelphia, Elsevier Saunders, 2012, vol 2, chap 46, p 1300.

2015 - 14 Cystine calculi can be diagnosed with the following test: A. sodium nitroprusside. B. phenolphthalein. C. thiazide challenge. D. serum pH. E. serum chloride.

D Failure to empty occurs more commonly in female than in male patients. Uniquely, females may develop hypercontinence that is likely due to a mechanical issue in which the orthotopic diversion falls back into a wide pelvic cavity. This in turn results in an acute angulation of the posterior pouch-urethral junction. Various maneuvers including omental packing behind (posterior) to the reservoir, suspending the dome of the reservoir to the rectus muscle and suspension of the vaginal stump to the preserved round ligaments have been described to prevent this angulation. Nerve-sparing techniques and increased intestinal length have been described as risk factors for retention. Suspension of the pouch to the back of the rectus muscle at the time of surgery is the most effective way to prevent angulation and hypercontinence. Skinner EC, Skinner DG, Stein JP: Orthotopic urinary diversion, Wein AJ, Kavoussi LR, Novick AC, Partin AW, Peters CA (eds): CAMPBELL-WALSH UROLOGY, ed 10. Philadelphia, Elsevier Saunders, 2012, vol 3, chap 87, p 2479.

2015 - 140 A 52-year-old woman undergoes radical cystectomy with orthotopic urinary diversion. A maneuver to prevent hypercontinence (retention) is: A. nerve-sparing techniques. B. the use of > 60 cm of intestine for the reservoir. C. omental packing anterior to the reservoir. D. suspension of the pouch to the back of the rectus muscle. E. suspension of the vagina to the endopelvic fascia.

A Boys with urethral valves often have problems with bladder training because of a combination of factors. Often these patients will have persistent tubular concentrating defects resulting in polydipsia and polyuria combined with small bladders capacities, detrusor overactivity with or without poor detrusor compliance. The low capacity bladder associated with large urine volume production results in significant urinary frequency, urgency, urge incontinence and primary nocturnal enuresis. The most useful studies to define the anomalies are a 24-hour collection for volume and urodynamics. Diuretic renography and antegrade pressure perfusion study are not indicated in this patient with stable hydronephrosis and normal creatinine. Cystoscopy would not be indicated with normal voiding phase of the VCUG. Antimuscarinics are not indicated unless bladder overactivity is documented on urodynamic study. Palmer LS, Trachtman H: Renal functional development and diseases in children, Wein, AJ, Kavoussi LR, Novick AC, Partin AW, Peters CA (eds): CAMPBELL-WALSH UROLOGY, ed 10. Philadelphia, Elsevier Saunders, 2012, vol 4, chap 112, p 3002. Casale AJ: Posterior urethral valves, Wein, AJ, Kavoussi LR, Novick AC, Partin AW, Peters CA (eds): CAMPBELL-WALSH UROLOGY, ed 10. Philadelphia, Elsevier Saunders, 2012, vol 4, chap 126, p 3391.

2015 - 141 A five-year-old boy who had PUV fulgurated as a newborn is referred because of daytime urinary incontinence and nocturnal enuresis. An ultrasound shows stable bilateral hydronephrosis and VCUG shows a trabeculated bladder, no VUR and no residual valve tissue. Serum creatinine is normal. The next step is: A. 24-hour urine volume and urodynamics. B. diuretic renography. C. antegrade pressure perfusion study. D. antimuscarinics. E. cystoscopy.

C Mutations in the CFTR are responsible for a phenotypic spectrum from overt cystic fibrosis to congenital bilateral absence of the vas deferens. Over 1,000 CFTR mutations have been identified and only a portion are detected by routine testing. The man should be assumed to be a CFTR carrier despite a negative CFTR gene test. Therefore, his female partner needs to be tested prior to any assisted reproductive techniques to determine the risk of cystic fibrosis in their offspring. Jarow JP, Sigman M, Kolettis PN, et al: The evaluation of the azoospermic male: AUA BEST PRACTICE STATEMENT. American Urological Association Education and Research, Inc, 2010. <a href="http://www.auanet.org/education/guidelines/male-infertility-b.cfm" target="_new"><u>http://www.auanet.org/education/guidelines/male-infertility-b.cfm</u></a> Jarow JP, Sigman M, Kolettis PN, et al: The optimal evaluation of the infertile male: AUA BEST PRACTICE STATEMENT. Revised 2010. American Urological Association Education and Research, Inc, 2010. <a href="http://www.auanet.org/education/guidelines/male-infertility-d.cfm" target="_new"><u>http://www.auanet.org/education/guidelines/male-infertility-d.cfm</u></a>

2015 - 142 A 32-year-old man with congenital bilateral absence of the vas deferens (CBAVD) desires a biological child. Testing for the cystic fibrosis transmembrane conductance regulator (CFTR) gene is negative. The next step is: A. test the man's parents for the CFTR gene. B. proceed with testicular sperm extraction and ICSI. C. CFTR testing of his female partner. D. proceed with testicular sperm extraction and ICSI and do pre-implantation genetic diagnosis. E. CFTR testing of the man's sperm.

B Multiple series have documented the safety and efficacy of endoscopic management of upper tract urothelial carcinoma (TCC). This elderly patient has significant comorbidities and a low grade distal ureteral TCC. Low grade tumors at ureteroscopic biopsy have a strong correlation with noninvasive stage at the time of nephroureterectomy. Similarly, high grade disease identified on ureteroscopic biopsy is very likely to represent invasive disease at the time of final pathologic staging. In a younger, healthier patient, distal ureterectomy and reimplantation would be another option; however, this older patient would be better served with endoscopic management. Although upper tract tumors can be ablated with electrocautery delivered through a small Bugbee electrode, the variable depth of penetration and risk of stricture formation have made the use of laser energy for ablation more popular. Sagalowsky AI, Jarrett TW, Flanigan RC: Urothelial tumors of the upper urinary tract and ureter, Wein, AJ, Kavoussi LR, Novick AC, Partin AW, Peters CA (eds): CAMPBELL-WALSH UROLOGY, ed 10. Philadelphia, Elsevier Saunders, 2012, vol 2, chap 53, pp 1542-1546.

2015 - 143 A 77-year-old man with hypertension, coronary artery disease, and a creatinine of 1.3 mg/dl has gross hematuria. Cystoscopy reveals a normal bladder, and bilateral retrogrades show a small right distal ureteral filling defect. On ureteroscopy, there is a 5 mm solitary papillary tumor and biopsy demonstrates a low grade urothelial carcinoma. The next step is: A. intravesical BCG with ureteral stent in place. B. ureteroscopic laser ablation of tumor. C. segmental resection and ureteroureterostomy. D. distal ureterectomy with reimplant. E. laparoscopic nephroureterectomy.

A The half-life of AFP is approximately five days and the fall in AFP levels described in this patient would be considered abnormal and suspicious of metastatic disease in any patient > 18 months of age. However, in infants (< 18 months of age) with a yolk sac tumor the physician should see a precipitous drop in the AFP levels post-orchiectomy but persistent elevations in the AFP (> 10 ng/dl) are a normal finding. The reason behind the persistent elevation in AFP is its production in the fetal and neonatal liver. Indeed, AFP is the chief serum protein (instead of albumin) for the first six months of life. The persistent elevation in AFP in a neonate is a normal finding with the AFP levels reaching normal adult values (< 10 ng/dl) at approximately eighteen months of age. In infants undergoing resection of a yolk sac tumor serial AFP levels are obtained with evidence of persistent or recurrent disease noted by serial elevations of the AFP level above the nadir value. Brewe JA, Tank ES: Yolk sac tumors and alpha-fetoprotein in first year of life. UROL 1993;42:79-80. Ritchey ML, Shamberger RC: Pediatric urologic oncology, Wein, AJ, Kavoussi LR, Novick AC, Partin AW, Peters CA (eds): CAMPBELL-WALSH UROLOGY, ed 10. Philadelphia, Elsevier Saunders, 2012, vol 4, chap 137, p 3696.

2015 - 144 A one-month-old boy undergoes left radical orchiectomy for a yolk sac tumor. Chest and abdominal CT scans are normal. Preoperative AFP is 2600 ng/dl, and postoperative levels at one and two months are 330 and 90 ng/dl. The next step is: A. repeat AFP in one month. B. ultrasound of right testis. C. bone scan. D. RPLND. E. multiagent chemotherapy.

C All five drug classes have been used for the pain relief of acute renal colic. NSAIDS and narcotics are most commonly used; however, meta-analysis and comparative studies show that NSAIDS are superior to narcotics for the acute relief of renal colic. NSAIDS treat the inflammatory basis of pain and reduce collecting system pressure as compared to narcotics. Narcotics are also associated with more side effects as compared to NSAIDS. Alpha-blockers, calcium channel blockers, and steroids are used more commonly for medical expulsive therapy. Singh I, Strandhoy JW, Assimos DG: Pathophysiology of urinary tract obstruction, Wein, AJ, Kavoussi LR, Novick AC, Partin AW, Peters CA (eds): CAMPBELL-WALSH UROLOGY, ed 10. Philadelphia, Elsevier Saunders, 2012, vol 2, chap 40, p 1106.

2015 - 145 The most effective medication class for the relief of pain associated with acute renal colic is: A. alpha-blockers. B. calcium channel blockers. C. NSAIDS. D. narcotics. E. corticosteroids.

D Ureterointestinal stricture can occur early or late after urinary diversion. The first step in management is to rule-out disease recurrence. Once it has been established that the stricture is not due to recurrent cancer, the most effective approach to management is open surgical repair. Although various minimally invasive approaches have been employed, including balloon dilation, cold knife or laser incision and stent placement, open surgical repair has been consistently shown to be the most effective way of addressing this problem, particularly in the setting of a longer stricture and decreased renal function. Minimally invasive approaches may be effective for shorter strictures (1-2 cm or less). Early ureteroenteric strictures (<1 yr postoperative) are usually due to devascularization of the distal ureter or extreme angulation of the left ureter at the level of the inferior mesenteric artery. Late strictures are usually due to a fibrotic response at the ureteroenteric junction. Nephrectomy is not indicated in this kidney with 25% shared function. In younger patients, a chronic ureteral stent is not indicated. Dahl DM, McDougal WS: Use of intestinal segments in urinary diversion, Wein, AJ, Kavoussi LR, Novick AC, Partin AW, Peters CA (eds): CAMPBELL-WALSH UROLOGY, ed 10. Philadelphia, Elsevier Saunders, 2012, vol 3, chap 85, p 2434.

2015 - 146 A 56-year-old woman develops a 3.0 cm benign ureteral stricture at the left ureteroileal anastomosis six months after a radical cystectomy and ileal neobladder. Left renal function is 25%. The next step is: A. endoscopic balloon dilation. B. endoscopic laser incision. C. metallic stent placement. D. open surgical repair. E. nephrectomy.

E Approximately 70% of arteriovenous fistulas occurring after needle biopsy of the kidney close spontaneously within 18 months. In the absence of significant related symptoms, expectant management is appropriate initially. The clinical manifestations of a renal arteriovenous fistula depend upon the size of the fistula. Congestive heart failure, cardiomegaly, and diastolic hypertension are observed in 50% of symptomatic patients. Hematuria is present in about a third, and tachycardia is occasionally found. Flank pain, embolic events, and hypertension are less common. In contrast, AVF following blunt renal trauma is much less likely to resolve spontaneously. Fergany A, Novick AC: Renovascular hypertension and ischemic nephropathy, Wein, AJ, Kavoussi LR, Novick AC, Partin AW, Peters CA (eds): CAMPBELL-WALSH UROLOGY, ed 10. Philadelphia, Elsevier Saunders, 2012, vol 2, chap 39, p 1081.

2015 - 147 The most common long-term outcome for a renal arteriovenous fistula occurring after needle biopsy of the kidney is: A. hematuria. B. embolic events. C. diastolic hypertension. D. high output cardiac failure. E. spontaneous closure.

D The genitofemoral, ilioinguinal, and lateral femoral cutaneous nerve can be seen during the performance of a psoas hitch. Although this patient has suffered an injury to the femoral branch of the genitofemoral nerve, visualization of three nerves will usually result in sparing injury to these structures. Indeed, some studies have revealed the most common nerve injured during the performance of a psoas hitch is the femoral nerve running in the undersurface or belly of the psoas muscle. Sutures placed horizontally in the psoas muscle increase the risk of injury to this nerve. Sutures for a psoas hitch should be placed longitudinally, e.g., in the direction that the muscle is running and superficially to prevent this complication. As noted, this patient has suffered an injury to the femoral branch of L1, L2 as a result of the psoas hitch. This nerve branch is responsible for sensation to the anterior thigh. The genital branch of L1, L2 is motor to the cremasteric muscles and scrotum. The lateral femoral cutaneous nerve is responsible for sensation to the lateral aspect of the thigh, and the ilioinguinal nerve provides sensation to the anterior pubis and scrotum. The obturator nerve provides sensation to the medial thigh. Chung BI, Sommer G, Brooks JD: Anatomy of the lower urinary tract and male genitalia, Wein, AJ, Kavoussi LR, Novick AC, Partin AW, Peters CA (eds): CAMPBELL-WALSH UROLOGY, ed 10. Philadelphia, Elsevier Saunders, 2012, vol 1, chap 2, p 47.

2015 - 148 A 55-year-old man develops anterior thigh numbness following a ureteral reimplant and psoas hitch for a distal ureteral stricture. The nerve most likely injured is the: A. lateral femoral cutaneous nerve. B. ilioinguinal nerve. C. genital branch of the genitofemoral nerve. D. femoral branch of the genitofemoral nerve. E. obturator.

B The presence of even a small amount of fat within a renal lesion on CT scan (confirmed by a value of -20 HU or lower) virtually excludes the diagnosis of RCC and is diagnostic of angiomyolipoma (AML). There are very rare cases of fat containing kidney cancer, but most of those will have calcified areas. Approximately 20% to 30% of AMLs are found in patients with tuberous sclerosis syndrome (TS), an autosomal dominant disorder characterized by mental retardation, epilepsy, and adenoma sebaceum, a distinctive skin lesion. Most patients with acute or potentially life-threatening hemorrhage require total nephrectomy if it is explored; and if a patient has TS, bilateral disease, preexisting renal insufficiency, or other medical or urologic disease that could affect renal function in the future, selective embolization should be considered. In such circumstances, selective embolization can temporize and in many cases prove definitive. This patient is hemodynamically stable and may not require transfusion if selective embolization is successful. There is no role for radio frequency ablation or cryotherapy for a large AML that is acutely bleeding. A partial nephrectomy may be possible or desirable in the future; however, in the setting of acute bleeding, most explorations will result in nephrectomy, which would be undesirable for this woman with renal insufficiency. Margulis V, Matin SF, Wood CG: Benign renal tumors, Wein, AJ, Kavoussi LR, Novick AC, Partin AW, Peters CA (eds): CAMPBELL-WALSH UROLOGY, ed 10. Philadelphia, Elsevier Saunders, 2012, vol 2, chap 51, p 1499.

2015 - 149 A 52-year-old woman is admitted to the hospital with acute flank pain. She is hemodynamically stable. A non-contrast CT scan demonstrates an 8 cm lower pole renal mass measuring -20 Hounsfield units (HU) and evidence of perinephric bleeding. Serum creatinine is 1.6 mg/dl. Hematocrit is 25%. The next step is: A. transfuse two units PRBC and bedrest. B. selective angiographic embolization. C. percutaneous thermal ablation. D. partial nephrectomy. E. radical nephrectomy.

C This patient most likely has T3 or T4 disease based on this CT scan. For T2 to T4 disease, large prospective randomized trials and meta-analyses have demonstrated that outcomes are better in patients who receive neoadjuvant chemotherapy prior to surgery rather than surgery alone. There is no evidence that MRI is significantly better at determining whether there is organ confined disease than a CT scan. In addition, with a CT scan that is fairly unequivocal there is no benefit from additional local imaging. Grossman HB, Natale RB, Tangen CM, et al: Neoadjuvant chemotherapy plus cystectomy compared with cystectomy alone for locally advanced bladder cancer. NEJM 2003;349:859-866. Lerner SP, Sternberg CN: Management of metastatic and invasive bladder cancer, Wein, AJ, Kavoussi LR, Novick AC, Partin AW, Peters CA (eds): CAMPBELL-WALSH UROLOGY, ed 10. Philadelphia, Elsevier Saunders, 2012, vol 3, chap 82, pp 2361-2363.

2015 - 15 A 54-year-old man has a muscle invasive urothelial carcinoma on TURBT. The preoperative CT scan shows loss of the fat plane on the right side of the bladder. The next step is: A. PET scan. B. MRI scan. C. neoadjuvant chemotherapy. D. XRT. E. cystectomy.

B When infradiaphragmatic urinary extravasation extends through Buck's fascia, it is limited only by Colles' fascia which attaches posteriorly at the triangular ligament and laterally at the fascia lata of the thigh. Colles' fascia is continuous anteriorly with Scarpa's fascia which extends superiorly to the thoracoclavicular fascia. Therefore, both Colles' and Scarpa's fascia limit such an extravasation. Such an extravasation, particularly when associated with infection (periurethral phlegmon), can result in edema and necrosis of the skin of the penis, scrotum, and anterior body wall. Denonvillier's fascia does not connect with Colles', Scarpa's, or dartos fascias. Colles' is contiguous with dartos fascia.

2015 - 150 The spread of urinary extravasation secondary to urethral injury below the urogenital diaphragm, when associated with a tear in Buck's fascia, is limited by the following fascial layers A. Denonvilliers' and Colles' B. Colles' and Scarpa's C. Scarpa's and Denonvilliers D. dartos and Colles' E. dartos and Denonvilliers'

D Contrast media accounts for 10% of all causes of hospital-acquired acute renal injury. Three key risk factors that may provoke this injury are: pre-existing renal dysfunction (serum creatinine > 1.6 mg/dl or eGFR < 60 ml/min/BSA), pre-existing diabetes, and reduced intravascular blood volume. Contrast agents evoke renal injury by two mechanisms: first, by acting as an intrarenal vasoconstricting agent resulting in decreased intrarenal blood flow and hypoxemia; second, by a direct toxic effect of the contrast agent on tubular epithelial cells. The combination of renal medullary ischemia and direct cellular toxicity leads to increased renal epithelial cell apoptosis and acute tubular necrosis. The osmolality of the contrast agent once believed to be of paramount importance in the induction of contrast-induced nephropathy has been shown to play a minimal role in contrast-induced nephropathy. Indeed, recent studies have found that viscosity of the contrast agent is more important than osmolality. These findings resulted in the recommendation that periprocedural hydration along with limiting the amount of contrast agent are the key to prevent contrast-induced renal failure. A recent meta-analysis to evaluate the various interventions employed for prevention of this complication, assessing sodium bicarbonate solutions, adenosine antagonists (theophylline), N-acetylcysteine and ascorbic acid noted mixed results with no definitive proof that these agents could prevent the complication. Randomized control studies have, however, shown that in patients with a creatinine of > 3.5 mg/dl prophylactic hemodialysis prior to and following the study can reduce the risk of this complication. Fulgham PF, Bishoff JT: Urinary tract imaging: Basic principles, Wein AJ, Kavoussi LR, Novick AC, Partin AW, Peters CA (eds): CAMPBELL-WALSH UROLOGY, ed 10. Philadelphia, Elsevier Saunders, 2012, vol 1, chap 4, pp 101-103.

2015 - 16 The renal toxicity of intravenous contrast material is due to: A. glomerular injury. B. afferent arteriolar constriction. C. efferent arteriolar constriction. D. intrarenal vasoconstriction and tubular necrosis. E. efferent arteriolar dilation and tubular necrosis.

D The retrograde and antegrade studies show a complete obstruction of the left distal ureter at the level of the uterine vessels. Most likely, the ureter was divided during clamping of the left uterine vascular pedicle or a thermal injury was sustained. The high grade obstruction (no contrast goes through the obstruction with both retro and antegrade injections) demonstrated makes the success of an endoscopic approach unlikely. Ureteroureterostomy is not a good option in the distal ureter, and should be reserved for short mid- to upper ureteral defects. The best repair for this patient is a ureteral reimplant with a psoas hitch. A Boari flap is not necessary in this patient and is reserved for lengthy distal ureteral defects up to 15 cm long. Nakada SY, Hsu THS: Management of upper urinary tract obstruction, Wein, AJ, Kavoussi LR, Novick AC, Partin AW, Peters CA (eds): CAMPBELL-WALSH UROLOGY, ed 10. Philadelphia, Elsevier Saunders, 2012, vol 2, chap 41, pp 1156-1157.

2015 - 17 A 46-year-old woman sustained a ureteral injury during an abdominal hysterectomy for fibroids six weeks ago. A left percutaneous nephrostomy tube was placed. A retrograde ureterogram and an antegrade pyeloureterogram are shown. The next step is: A. balloon dilation. B. endoureterotomy. C. ureteroureterostomy. D. ureteral reimplant with psoas hitch. E. ureteral reimplant with Boari flap.

E Decreasing the rate of shock wave administration from 120 to 60 shocks per minute results in improved stone-free rates. A slower treatment rate of proximal ureteral stones reduces the need for additional SWL or more invasive treatments to render patients stone-free without any increase in morbidity and with an acceptable increase in treatment time. The hypothesized mechanism of this effect is due to the formation of cavitation bubble cloud around the stone, which may shield the stone from subsequent shock waves. This effect is most pronounced at higher shock wave frequency. Matlaga BR, Lingeman JE: Surgical management of upper urinary tract calculi, Wein, AJ, Kavoussi LR, Novick AC, Partin AW, Peters CA (eds): CAMPBELL-WALSH UROLOGY, ed 10. Philadelphia, Elsevier Saunders, 2012, vol 2, chap 48, p 1398.

2015 - 35 The most important benefit of using 60 versus 120 shocks per minute for SWL of a 9 mm proximal ureteral stone is: A. reduced number of shocks. B. reduced renal damage. C. reduced anesthetic requirement. D. reduced steinstrasse rate. E. reduced retreatment rate.

A According to the EARLY DETECTION OF PROSTATE CANCER: AUA GUIDELINES, guideline statement number 1 states that the panel recommends against screening in all men under age 40. In this age group, there is a low prevalence of clinically detectable prostate cancer. There is no evidence to demonstrate a benefit of screening, and there are likely the same harms of screening as in other age groups. This recommendation holds even for African-Americans or those with a family history of prostate cancer. The panel does state that to reduce the harms of screening, a routine screening interval of two years (biennial screening) or more may be preferred over annual screening in those men who have participated in shared decision-making and decided on screening (Guideline statement 4). As compared to annual screening, it is expected that screening intervals of two years preserve the majority of the benefits and reduce over-diagnosis and false positives. However, in this patient population, no screening is recommended. Some authors have put forth the strategy of initial screening and then follow-up in five years. Although such strategies may help reduce over-diagnoses and better select men who are likely to be true positives, this approach has not been well-validated and accepted by the AUA Guidelines. Carter BH, Albertsen PC, Barry MJ, et al: Early detection of prostate cancer: AUA GUIDELINE. American Urological Association Education and Research, Inc, 2013. <a href="http://www.auanet.org/education/guidelines/prostate-cancer-detection.cfm" target="_new"><u>http://www.auanet.org/education/guidelines/prostate-cancer-detection.cfm</u></a>

2015 - 18 A 38-year old man is referred for prostate cancer screening. According to the AUA Guidelines, the next step is: A. advise against screening. B. initiate yearly screening. C. initiate yearly screening if positive family history or African American. D. initiate biennial screening. E. screen now and repeat in five years.

E A rectourethral fistula is relatively low in the pelvis and is best managed by the posterior-transanal repair (York-Mason) approach, in which the posterior anal sphincter is split to provide good exposure of the anterior rectal wall. The fistula site can then be excised with a multilayer closure. A transabdominal repair is difficult in this setting due the location deep within the pelvis, and this type of fistula is better repaired through the posterior-transanal approach. A urinary diversion may need to be considered, but only if attempts for primary repair have failed. This patient's PSA is <0.5 and he is likely to stay cancer free, so salvage prostatectomy or pelvic exoneration should not be considered. Rovner ES: Urinary tract fistulae, Wein, AJ, Kavoussi LR, Novick AC, Partin AW, Peters CA (eds): CAMPBELL-WALSH UROLOGY, ed 10. Philadelphia, Elsevier Saunders, 2012, vol 3, chap 77, p 2257.

2015 - 19 A 67-year-old man has a rectourethral fistula one year after cryotherapy for localized prostate cancer. An initial fulguration failed and six months ago, he underwent proximal colostomy and suprapubic tube placement. He continues to have urine leakage per rectum and recurrent UTIs. Biopsy of the prostate shows no cancer and serum PSA is 0.3 ng/ml. The best therapy is: A. pelvic exoneration. B. transabdominal repair. C. urinary diversion. D. salvage prostatectomy. E. York Mason transrectal, transsphincteric repair.

A The urine usually becomes sterile within a few hours of starting antibiotics even though fever, chills, and flank pain may continue for several days. A delay in clearance of bacteria may occur with obstruction, stone disease, anatomic abnormalities or impaired renal function. Symptoms of pyelonephritis continuing for 72 hours after initiation of culture appropriate antibiotics should result in the physician considering the need for imaging studies and repeat cultures to rule-out anatomic abnormalities or the emergence of antibiotic resistant bacteria. Schaeffer AJ, Schaeffer EM: Infections of the urinary tract, Wein, AJ, Kavoussi LR, Novick AC, Partin AW, Peters CA (eds): CAMPBELL-WALSH UROLOGY, ed 10. Philadelphia, Elsevier Saunders, 2012, vol 1, chap 10, pp 298-299.

2015 - 2 After starting antimicrobials in healthy individuals with uncomplicated acute pyelonephritis, the urine is typically sterile within: A. a few hours. B. twenty-four hours. C. forty-eight hours. D. three days. E. seven days.

E For late-onset urinary retention found in patients with an AUS in situ, endoscopic and urodynamic evaluation is required to identify urethral erosion, proximal obstruction, or the development of detrusor failure. In this case, obstructive voiding symptoms, an abnormal physical examination with a normal endoscopic and complete urodynamic evaluation are highly consistent with periprosthetic infection without urethral erosion, secondary pericuff edema resulting in the obstructive symptoms. Late infections of AUS are usually due to gram positive cocci (S. aureus or S. epidermidis). Treatment with ciprofloxacin will not clear the infection due to bacterial adherence to the biofilm of the AUS. In addition, in this patient with diabetes, the local infection could quickly escalate resulting in widespread cellulitis and Fournier's gangrene. Removal of the AUS with appropriate cultures and, if indicated, salvage AUS replacement should be considered. Cuff deactivation will not prevent retention unless the patient is incapable of using the device. CIC will not treat the underlying problem, which remains undiagnosed. A pelvic CT scan may define inflammation around the device; however, a negative CT scan does not indicate absence of infection and, therefore, cannot be relied upon. Cuff size is not likely to influence voiding status except in the immediate postoperative period when, if retention occurs, cuff upsizing may be necessary. In this patient with a long history of an AUS, sub cuff atrophy is more likely. Wessells H, Peterson AC: Surgical procedures for sphincteric incontinence in the male: The artificial genitourinary sphincter and perineal sling procedures, Wein, AJ, Kavoussi LR, Novick AC, Partin AW, Peters CA (eds): CAMPBELL-WALSH UROLOGY, ed 10. Philadelphia, Elsevier Saunders, 2012, vol 3, chap 79, p 2302.

2015 - 20 A 76-year-old man with insulin dependent diabetes returns six years after artificial urinary sphincter (AUS) placement with difficulty emptying his bladder despite appropriate action of his control pump. Examination reveals perineal induration without fluctuance or tenderness. Urinalysis is normal and PVR is 250 cc. Urodynamics reveals low pressure voiding with incomplete emptying. Urethroscopy shows no evidence of erosion. The next step is: A. ciprofloxacin. B. deactivate cuff. C. initiate CIC. D. pelvic CT scan. E. remove AUS.

D Malignant GCT accumulates fluorodeoxyglucose (FDG), and several studies have investigated FDG-labeled positron emission tomography (FDG-PET) in the staging of GCT at diagnosis and assessing response after chemotherapy. The high sensitivity is likely due to the high turnover and increased metabolic rate of GCTs. Due to limited sensitivity at the time of initial diagnosis, there is currently no role for FDG-PET in the routine evaluation of NSGCT and seminoma at the time of diagnosis. However, there may be a role for detection of recurrent disease and the assessment of residual masses after chemotherapy. For example, PET does appear to be a useful tool in seminoma patients when evaluating post-chemotherapy residual masses. In a series of seminoma patients who were evaluated post-chemotherapy for residual retroperitoneal masses, PET was accurate in 14/14 patients with tumors > 3 cm and in 22/23 patients with lesions < 3 cm. Overall, the sensitivity and specificity was 89% and 100%, respectively. The utility of FDG-PET in the prediction of retroperitoneal histology in NSGCT (particularly in the post-chemotherapy setting) is limited by the fact that teratoma is not FDG avid (likely due to the relatively low metabolic rate of teratomas). This likely accounts for the high false negative rates observed. Similarly, the utility of PET scanning in the immediate post-chemotherapy period appears to be limited. This is likely due to decreased metabolism and increased macrophage activity at that time, which compromises the accuracy of PET scanning. It is recommended that PET/CT be delayed for four to 12 weeks following completion of chemotherapy. There is no difference between abdominal and thoracic imaging using a PET scan in this setting. Stephenson AJ, Gilligan TD: Neoplasms of the testis, Wein, AJ, Kavoussi LR, Novick AC, Partin AW, Peters CA (eds): CAMPBELL-WALSH UROLOGY, ed 10. Philadelphia, Elsevier Saunders, 2012, vol 1, chap 31, pp 847; 858. Fulgham PF, Bishoff JT: Urinary tract imaging: Basic principles, Wein, AJ, Kavoussi LR, Novick AC, Partin AW, Peters CA (eds): CAMPBELL-WALSH UROLOGY, ed 10. Philadelphia, Elsevier Saunders, 2012, vol 1, chap 4, pp 127-128. De Giorgi U, Pupi A, Fiorentini G, et al: FDG-PET in the management of germ cell tumor. ANN ONCOL 2005;16:90-94.

2015 - 21 In a testicular cancer patient, positron emission tomography (PET): A. has decreased sensitivity due to high cell turnover of germ cell tumors. B. is most useful at the time of initial diagnosis. C. is most useful in patients with lung nodules. D. has good sensitivity for post-chemotherapy seminomas. E. can distinguish teratoma versus fibrosis.

B Thiazide diuretics will lose their effectiveness in the treatment of hypercalciuria in up to 25% of patients on long-term management. The loss of effectiveness is due to increased serum calcium levels which stimulate the C cells in the thyroid to produce more calcitonin. Increased calcitonin leads to increased urinary calcium excretion. Increased dietary calcium, decreased patient compliance, increased GI absorption or increased PTH could all lead to hypercalciuria, but are not the proposed mechanisms for tachyphylaxis with thiazides. Ferrandino MN, Pietrow PK, Preminger GM: Evaluation and medical management of urinary lithiasis, Wein, AJ, Kavoussi LR, Novick AC, Partin AW, Peters CA (eds): CAMPBELL-WALSH UROLOGY, ed 10. Philadelphia, Elsevier Saunders, 2012, vol 2, chap 46, p 1310.

2015 - 40 The diminished long term effectiveness of thiazides in the treatment of hypercalciuria is mediated by: A. increased dietary sodium. B. increased serum calcitonin. C. increased parathyroid hormone. D. decreased urinary magnesium. E. increased gastrointestinal absorption of calcium.

D Prosthetic infections occur in 1-3% of patients following inflatable penile implants with antibiotic coating, and usually occur within the first three months of implantation. The most common organism is staphylococcus, and infection occurs at the time of implantation. Pain without WBC count elevation or increase in erythrocyte sedimentation rate is common. The increasing nature of the pain is not consistent with post-operative pain or pain from a traumatic event. Prosthetic erosion would be apparent on physical exam. Corporal fibrosis is an uncommon late complication of penile prosthesis. An oversized cylinder is associated with buckling and pain with prosthetic inflation. Montague DK: Prosthetic surgery for erectile dysfunction, Wein, AJ, Kavoussi LR, Novick AC, Partin AW, Peters CA (eds): CAMPBELL-WALSH UROLOGY, ed 10. Philadelphia, Elsevier Saunders, 2012, vol 1, chap 27, pp 785-787.

2015 - 22 A 62-year-old man develops penile pain three months after implantation of an inflatable penile prosthesis. He denies fever or chills. The prosthesis is functional and in excellent position. Tenderness is localized to the left corpus. WBC count and urinalysis are normal. The most likely cause of the penile pain is: A. oversized cylinder. B. prosthetic erosion. C. corporal fibrosis. D. staphylococcal infection. E. psychogenic.

B Observation places the patient at risk of recurrent urosepsis. This can be prevented by continued nitrofurantoin prophylaxis which will prevent recurrent cystitis and symptomatic infection. I.V. tobramycin achieves poor penetration of the prostate and is unlikely to eradicate infection. Tobramycin instillations would be effective but are more invasive than oral prophylaxis. TURP would be inappropriate in this patient. Hua VN, Schaeffer AJ: Acute and chronic prostatitis. MED CLIN N AM 2004;88:483-494. Nickel JC: Prostatitis and related conditions, orchitis, and epididymitis, Wein, AJ, Kavoussi LR, Novick AC, Partin AW, Peters CA (eds): CAMPBELL-WALSH UROLOGY, ed 10. Philadelphia, Elsevier Saunders, 2012, vol 1, chap 11, pp 342-344.

2015 - 23 A 55-year-old man with a history of chronic bacterial prostatitis experiences urosepsis during induction chemotherapy for small cell lung cancer. Urine culture is positive for E. coli resistant to trimethoprim/sulfamethoxazole and ciprofloxacin; sensitive to nitrofurantoin, tobramycin, amikacin, and meropenem. Thorough urologic evaluation is normal except for documented persistence of the bacteria in the expressed prostatic secretions following a ten day course of I.V. meropenem. The next step is: A. observation. B. nightly prophylaxis with oral nitrofurantoin. C. daily intravesical tobramycin instillation. D. I.V. tobramycin for six to eight weeks. E. TURP.

D There is little long-term data on the cancer control of ablative procedures. Additionally, there is a well-recognized slow natural history of RCC in terms of growth rate. Thus, if imaging findings reveal increasing size, new nodularity, satellite lesions or failure of the treated lesion to regress over time even in the absence of enhancement, then the next step should be lesion biopsy. These findings would be concerning enough to warrant an intervention rather than routine imaging in 6-12 months. There is no data to support the routine use of PET scanning in the evaluation or follow-up of patients with small renal neoplasms, although ongoing studies with newer imaging agents are underway. Repeat ablation with no biopsy is also not indicated. Donat SM, Chang SS, Bishoff JT, et al: Follow-up for clinically localized renal neoplasms: AUA GUIDELINE. American Urological Association Education and Research, Inc, 2013. <a href="http://www.auanet.org/education/guidelines/renal-cancer-follow-up.cfm" target="_new"><u>http://www.auanet.org/education/guidelines/renal-cancer-follow-up.cfm</u></a>

2015 - 24 A 61-year-old woman underwent percutaneous cryoablation of a 2.4 cm renal mass one year ago. On follow-up imaging, the mass now measures 3 cm with some nodularity within the treatment zone. According to the AUA Guidelines, the next step is: A. repeat imaging in six months. B. repeat imaging in one year. C. PET scan. D. percutaneous biopsy. E. repeat cryoablation.

D When faced with a patient with a persistent urinary fistula, the acronym FETID will aid the physician in determining its etiology and hence management plans: F- Foreign Body, E- Epithelization of the fistula tract, T- Tumor or chronic trauma causing persistence, I-Infection or chronic inflammation arising from inflammatory bowel disease, radiation therapy, etc. D-Distal obstruction. In this young patient with a history of persistent fistula, following closure of a bladder rupture after a pelvic fracture, persistent drainage from a suprapubic tube site is most likely from either a foreign body within the bladder, i.e., bony spicule or bladder calculi formed as a nidus from the prior indwelling suprapubic tube or bladder outlet obstruction arising from either a bladder neck contracture or urethral stricture. The single best diagnostic study is cystourethroscopy. Pressure flow urodynamic studies could demonstrate findings consistent with high pressure voiding and outlet obstruction. But the source of the obstruction, which is likely a urethral stricture or a bladder neck contracture, would not be able to be determined by this test and this test does not rule-out the possibility of a foreign body within the bladder. A CT scan may allow one to visualize either a foreign body or bladder calculi to be present, but would not be able to assess the urinary outlet. Similarly, a fistulogram or pelvic MRI scan are unlikely to yield adequate diagnostic information in this situation to result in definitive operative plans. Rovner ES: Urinary tract fistulae, Wein, AJ, Kavoussi LR, Novick AC, Partin AW, Peters CA (eds): CAMPBELL-WALSH UROLOGY, ed 10. Philadelphia, Elsevier Saunders, 2012, vol 3, chap 77, pp 2260-2261.

2015 - 25 Two months following closure of a traumatic bladder rupture associated with a pelvic fracture, a 20-year-old man has persistent urinary leakage through the suprapubic cystostomy site despite voiding. The diagnostic test most likely to diagnose the etiology of the problem is: A. CT urogram. B. pelvic MRI scan. C. fistulogram. D. cystourethroscopy. E. urodynamics.

E The patient has peritoneal signs, which dictate exploration and intestinal diversion. If the patient did not have peritoneal signs, then repositioning the tube in the colonic lumen and placing a ureteral stent would be an optimal approach to prevent a nephrocolonic fistula. Matlaga BR, Lingeman JE: Surgical management of upper urinary tract calculi, Wein, AJ, Kavoussi LR, Novick AC, Partin AW, Peters CA (eds): CAMPBELL-WALSH UROLOGY, ed 10. Philadelphia, Elsevier Saunders, 2012, vol 2, chap 48, p 1405.

2015 - 26 Two days after PCNL, a patient is febrile with abdominal pain, rebound, and guarding. A nephrostogram via the nephrostomy tube opacifies the colon and the renal pelvis. The next step is antibiotics and: A. withdraw the nephrostomy tube into the colon. B. remove nephrostomy tube and place ureteral stent. C. withdraw the nephrostomy tube into the colon and place a ureteral stent. D. withdraw the nephrostomy tube into the colon and place another nephrostomy tube into the kidney. E. abdominal exploration, diverting colostomy, nephrostomy tube.

C Alvimopan, a peripherally acting µ-opioid receptor antagonist, is indicated to accelerate upper and lower GI recovery following surgeries that include a bowel resection. In October 2013, the U.S. Food and Drug Administration (FDA) authorized an expanded indication on the basis of a Phase 4 randomized multicenter clinical trial. In this trial, patients receiving alvimopan experienced more rapid bowel recovery and had a shorter hospital stay compared with those who received placebo. There were no differences with regard to early (< 7 day) post-op ileus (POI) or 30-day all cause readmission rates between the two groups. Alvimopan has been associated with the potential for increased cardiac toxicity in patients with chronic narcotic use, and is therefore contraindicated in this patient population. In the aforementioned trial, such patients were excluded, and there were no differences in cardiac adverse events between the alvimopan and placebo groups. In a preplanned economic analysis of this study, alvimopan use decreased hospitalization costs by reducing health care services associated with POI and decreasing hospital length of stay; total costs were $2,640 lower per patient for alvimopan compared with placebo. The study did not address opiate consumption, but it is unlikely that a µ-opioid receptor antagonist should affect opioid intake. Instead, it would be expected to affect the peripheral effects of opioids on bowel motility. Lee CT, Chang SS, Kamat AM, et al: Alvimopan accelerates gastrointestinal recovery after radical cystectomy: A multicenter randomized placebo-controlled trial. EUR UROL 2014;66:265-272. Kauf TL, Svatek RS, Amiel G, et al: Alvimopan, a peripherally acting µ-opioid receptor antagonist, is associated with reduced costs after radical cystectomy: Economic analysis of a phase 4 randomized, controlled trial. J UROL 2014;191:1721-1727.

2015 - 27 The use of µ-opioid receptor antagonists after radical cystectomy and urinary diversion is associated with: A. increased cardiac events. B. reduced opioid consumption. C. reduced length of stay. D. increased hospitalization costs. E. reduced early readmission for post-operative ileus.

C There is no consensus as to whether transplanted ureters should be reimplanted into a recipient's native bladder with an antirefluxing technique. However, there is certainly a concern that reflux of infected urine and/or reflux associated with elevated detrusor pressures can be damaging to the transplanted kidney. This patient does not have any lower urinary tract symptoms and there is no evidence of elevated storage pressures on her urodynamic study (i.e., no detrusor overactivity and normal compliance); thus, there is no reason to initiate therapy for OAB with either oxybutynin or mirabegron. She does abdominally recruit with some degree of Valsalva voiding at the end of her micturition, but that is unlikely related to bladder infections or pyelonephritis because she empties effectively. Decreasing her immunosuppression would not address the issue of her infections, would not minimize risk of future episodes of pyelonephritis, and would only place the kidney at possible risk for rejection. Low dose suppressive antibiotic therapy would be the appropriate next step to minimize future episodes of pyelonephritis. If this is not effective then revision of her ureteral reimplant with a non-refluxing neocystostomy should be considered. Barry JM, Conlin MJ: Renal transplantation, Wein, AJ, Kavoussi LR, Novick AC, Partin AW, Peters CA (eds): CAMPBELL-WALSH UROLOGY, ed 10. Philadelphia, Elsevier Saunders, 2012, vol 2, chap 44, p 1251. Veale JL, Gritsch HA: Complications of renal transplantation, Taneja SS (ed): COMPLICATIONS OF UROLOGIC SURGERY, ed 4. Philadelphia, Elsevier Saunders, 2010, chap 37, pp 436-437.

2015 - 28 A 45-year-old woman has recurrent episodes of graft pyelonephritis following a kidney transplantation two years previously. She denies voiding symptoms when she is infection-free. Her renal function is normal and cystogram reveals reflux into the transplanted kidney. Urodynamics are shown. The next step is: A. oxybutynin. B. mirabegron. C. suppressive antibiotics. D. non-refluxing ureteral reimplant. E. decrease immunosuppression dose.

D The option with the best stone-free rate for larger ureteral stones is ureteroscopy. While SWL is acceptable, the best option is ureteroscopy for stones > 1 cm according to the data extracted in the AUA/EAU Ureteral Stone Guidelines. A stone this large would likely not pass. Percutaneous stone removal would be definitive, but should only be considered when the patient already has a pre-existing nephrostomy tube or has failed a retrograde ureteroscopic approach or SWL. Preminger GM, Tiselius HG, Assimos DG, et al: Management of ureteral calculi: EAU/AUA NEPHROLITHIASIS PANEL: AUA GUIDELINE. American Urological Association Education and Research, Inc, 2007. <a href="http://www.auanet.org/education/guidelines/ureteral-calculi.cfm" target="_new"><u>http://www.auanet.org/education/guidelines/ureteral-calculi.cfm</u></a>

2015 - 29 The best treatment for a symptomatic 1.5 cm proximal ureteral stone is: A. medical expulsive therapy. B. in situ SWL. C. stent placement and SWL. D. ureteroscopy and laser lithotripsy. E. percutaneous stone removal.

B In patients with a history of cancer found to have a > 2 cm adrenal mass on a CT scan, approximately 50% of the lesions will be due to a metastasis from the primary tumor. Through the use of CT and MRI manipulations, the indeterminate adrenal mass (classified as a mass 2-5 cm in size) can usually be accurately characterized without biopsy. Benign adrenal tumors, such as a myelolipoma or lipid rich adenoma will usually have non-contrast CT Hounsfield units of < 10. In an adrenal lesion with a CT Hounsfield value of > 10, differentiation of lipid poor adenomas from malignant lesions will require a CT study with contrast and washout, as well as chemical shift MRI scans for differentiation of a benign from a malignant mass. The common features of lipid poor benign adenoma are > 60% washout on CT scan with I.V. contrast while a malignant lesion will usually have a < 60% washout on CT scan. MRI findings consistent with a lipid poor benign adenoma is an adrenal to spleen ratio (ASR) of < 70% and signal loss of > 20% on out of phase imaging. Malignant lesions will display an ASR of > 70% and signal loss of < 20% on out of phase imaging. Sahdev A, Reznek RH: The indeterminate adrenal mass in patients with cancer. CANCER IMAGING 2007;1:7.

2015 - 3 A 55-year-old woman with breast cancer has a 3.5 cm right adrenal nodule. The nodule has an attenuation of 25 Hounsfield units on non-contrast CT scan, 80% washout on contrast enhanced CT scan, and signal loss of 40% on chemical shift MRI scan. The lesion is a: A. lipid rich adenoma. B. lipid poor adenoma. C. myelolipoma. D. breast cancer metastasis. E. primary adrenal cancer.

D Prostatic duct adenocarcinomas arise in the periurethral prostatic ducts, and usually grow as an exophytic lesion in the urethra. They can give rise to either hematuria or obstructive symptoms, and often both are present. These tumors are often underestimated clinically because serum PSA levels and DRE are often normal. Consequently, many ductal adenocarcinomas are at an advanced stage at presentation and have an aggressive course. They are graded as 4+4=8 because of their cribriform morphologic features. These tumors should be treated aggressively and approached surgically. There is no indication that ductal adenocarcinomas are more sensitive to radiation, and similarly, chemotherapy is not indicated in this situation. Epstein JI: Pathology of prostatic neoplasia, Wein, AJ, Kavoussi LR, Novick AC, Partin AW, Peters CA (eds): CAMPBELL-WALSH UROLOGY, ed 10. Philadelphia, Elsevier Saunders, 2012, vol 3, chap 96, p 2733.

2015 - 30 Compared to typical prostate adenocarcinoma, prostatic ductal adenocarcinoma often exhibits: A. less aggressiveness and lower PSA. B. abnormal DRE and higher PSA. C. increased sensitivity to radiation. D. more aggressiveness and more frequent obstructive symptoms. E. should be treated with neoadjuvant systemic chemotherapy.

C Arterioureteral fistula (AUF) is a rare but acute condition that predominantly affects women (> 70%) with a wide time range between the initial placement of the ureteral stent to fistulization, ranging from 2 to 25 years. Factors that should raise suspicion of an AUF include history of hematuria in a patient with indwelling ureteral stents especially in patients with a past medical history of prior abdominal or pelvic irradiation, pelvic surgery or aortoiliac or aortofemoral grafts. Although 55% of patients will present with a history of persistent gross hematuria plus or minus shock, 45% of patients present with herald bleeding (gross hematuria occurring from the ureteral orifice during a ureteral stent exchange.) When herald bleeding occurs as in the patient in this question, treatment should be pursued to prevent a possible exsanguinating emergent fistula complication. The diagnostic goal in these patients is to identify the specific location of the fistula. In patients with bilateral indwelling stents, the side in which the gross hematuria is found is helpful for locating the side involved but is nonspecific in nature. Computed tomographic angiography will document the location in < 40% of patients. However, it may be useful because it can identify a concurrent periarterial abscess, aneurysmal enteric communication, aneurysmal dilation, significant arterial calcification, and concomitant thrombus. Diagnostic provocative angiography with selective iliac views remains the gold standard and is 90% diagnostic. Provocative angiography will require exchange of the double-J ureteral stent for a straight ureteral catheter. This will allow the interventionalist the ability to manipulate the ureteral stent at time of angiography to induce bleeding and identify the site and location of the fistula. While exchanging the stent, it is imperative not to lose access to the ureter; this procedure is best done in an endovascular suite with an operative team immediately available. It should be noted that a minority of patients, 10% will be empirically treated for an AUF without identification of the exact AUF location. These individuals will have a history of herald hematuria with predisposing factors for AUF and no other identifiable source of urinary bleeding. Prior studies have documented that once the AUF has developed, simple removal of the stent is inadequate for fistula closure. Previously, open repair of the fistula (often with vascular bypass procedure of the affected vessel and percutaneous nephrostomy tube placement) was the standard therapy. However, recent advances in endovascular stents have made this the least morbid procedure and the best initial therapeutic option. It is noteworthy that after placement of an endovascular graft, greater than 60% of the patients are still treated by chronic ureteral stent drainage along with chronic antibiotic prophylaxis, while the remaining 40% are managed by either permeant ipsilateral nephrostomy tube drainage or nephrectomy. Rovner ES: Urinary tract fistulae, Wein, AJ, Kavoussi LR, Novick AC, Partin AW, Peters CA (eds): CAMPBELL-WALSH UROLOGY, ed 10. Philadelphia, Elsevier Saunders, 2012, vol 3, chap 77, pp 2259-2260.

2015 - 31 A 68-year-old woman with a history of a lengthy ureteral stricture developing following pelvic surgery and radiation therapy is managed with a chronic indwelling ureteral stent. At the time of stent exchange, she develops profuse bright red blood per ureteral orifice that stops within five minutes of stent placement, the next step is: A. observation and stent exchange in three months. B. placement of nephrostomy tube and removal of the ureteral stent. C. radiologic placement of an endovascular stent. D. oversewing of arterial fistula, ureteroureterostomy, omental wrap around the ureter, and extraperitoneal lateralization of the ureter. E. vascular bypass procedure and nephrostomy tube placement.

D Drug-induced renal calculi represent 1-2% of all renal calculi. They include two categories: those resulting from the urinary crystallization of a highly excreted, poorly soluble drug or drug metabolite, and those due to the metabolic effects of a drug. Four drugs that can induce calculi through precipitation of the medication or its metabolite include: 1) Indinavir, a protease inhibitor used to treat HIV infections, 2) Magnesium Trisilicate, an antacid used to treat gastroesophageal reflux, 3) Triamterene, a potassium sparing diuretic used to treat edema and hypertension, 4) Ephedrine used in a variety of nutritional or energy supplements for its stimulant properties, can when used alone or in combination with guaifenesin, the combination used as an expectorant, induce calculi containing either ephedrine, or both ephedrine and guaifenesin. Five commonly used medications may induce physiologic changes that can lead to metabolic abnormalities that facilitate the formation of calculi these include: 1) Loop diuretics (Furosemide, Lasix), it is noteworthy that up to two thirds of low-birth-weight infants who have received furosemide therapy will develop precipitation of calcium crystals, 2) Carbonic anhydrase inhibitors, drugs such as acetazolamide (Diamox), used to treat glaucoma, altitude sickness, and epilepsy, and 3) topiramate (Topamax), an anticonvulsant medication used to treat refractory seizures, can produce severe hypocitraturia and high urinary pH, and will induce calcium phosphate calculi in up to 2% of patients on long-term therapy, 4) Zonisamide (Zonegran), a sulfonamide anticonvulsant will result in the formation of calcium phosphate calculi in 4% of the patients on this mediation, 5) potential laxative abuse should be considered when ammonium acid urate calculi are found in the absence of UTI or bowl disease. Carbamazepine (Tegretol) used to treat seizure disorders, nerve pain and bipolar disorder is not known to be associated with urolithiasis. Pearle MS, Lotan Y: Urinary lithiasis: Etiology, epidemiology, and pathogenesis, Wein, AJ, Kavoussi LR, Novick AC, Partin AW, Peters CA (eds): CAMPBELL-WALSH UROLOGY, ed 10. Philadelphia, Elsevier Saunders, 2012, vol 2, chap 45, p 1283.

2015 - 32 A 32-year-old anorexic woman with a history of seizures has recurrent urolithiasis. On a 24 hour urine, pH is 7.0 and urinary citrate is 45 (normal > 450 mg/day) The medication responsible for her stone disease is: A. indinavir. B. guaifenesin. C. carbamazepine. D. topiramate. E. ephedrine.

A Bosniak classifications of renal cysts are: 1) Simple hairline thin cyst wall, Hounsfield units < 10. 2) Simple hairline thin cyst wall, few hairline thin septa within cyst, short, thin areas of calcification maybe present, septa and wall do not enhance, Hounsfield units < 10. The patient described in the question has a Bosniak 2 cyst. No follow-up evaluation is indicated for a class 1 or 2 Bosniak cyst. 2F) Thickened cyst wall, multiple septa that may be thickened or contain calcium, Hounsfield units of 10-15 no significant enhancement with contrast. Follow-up is indicated due to an increased risk of malignancy (5-10%). These cysts should therefore undergo periodic surveillance with no set time limit; evaluations every 6-12 months have been purposed. Biopsy is not indicated due to poor reliability in sampling areas of concern. 3) Cystic mass with thickened wall, thick irregular septum, cyst wall or septa enhance with contrast, Hounsfield units >15. 4) Cystic mass with thickened wall, thick irregular septum, cyst wall, septa, and areas within cyst, not associated with the wall or septa enhance, Hounsfield units >15. Both Bosniak 3 and 4 cysts should at a minimum be considered for a biopsy or alternatively surgical excision. CAMPBELL SC, Lane BR: Malignant renal tumors, Wein, AJ, Kavoussi LR, Novick AC, Partin AW, Peters CA (eds): CAMPBELL-WALSH UROLOGY, ed 10. Philadelphia, Elsevier Saunders, 2012, vol 2, chap 49, pp 1418-1420.

2015 - 36 A 52-year-old woman has an incidentally-detected right renal lesion on triphasic CT scan. The lesion is classified as complex due to a few hairline septae with fine calcifications noted within the wall, Hounsfield units of 8 is noted. The next step is: A. no follow-up necessary. B. ultrasound in six months. C. CT scan in six months. D. CT scan in one year. E. biopsy or fine needle aspiration of lesion.

E BPH is a histological diagnosis. This patient has not had a biopsy. Benign prostatic obstruction is a urodynamic diagnosis made on the basis of the relationship between pressure and flow. The poor flow rate in this case may be due to either detrusor underactivity or bladder outlet obstruction and is not diagnostic of either entity. Detrusor overactivity and detrusor underactivity are urodynamic diagnoses that cannot be made in the absence of a urodynamic study. LUTS is a generic term describing lower urinary tract symptoms and does not imply an underlying pathology or pathophysiology. McVary KT, Roehrborn CG, Avins AL, et al: Management of benign prostatic hyperplasia (BPH): AUA GUIDELINE. American Urological Association Education and Research, Inc, 2010. <a href="http://www.auanet.org/education/guidelines/benign-prostatic-hyperplasia.cfm" target="_new"><u>http://www.auanet.org/education/guidelines/benign-prostatic-hyperplasia.cfm</u></a>

2015 - 37 A 68-year-old man with bothersome voiding dysfunction completes a voiding diary revealing 12 voids in 24 hours with volumes ranging from 30 ml to 150 ml, nocturia x 3, and one episode of incontinence. PVR is 50 ml. Uroflowmetry reveals a flattened pattern with a peak flow of 6 ml/sec. His condition is best described as: A. BPH. B. benign prostatic obstruction. C. detrusor overactivity. D. detrusor underactivity. E. LUTS.

B The patient presented here is Index Patient 2 of the AUA Guidelines and should be considered for sipuleucel-T immunotherapy. This patient is only minimally symptomatic (not requiring narcotics) and thus is a candidate for sipuleucel-T, which has demonstrated a survival advantage in this patient population. Abiraterone acetate is also an option but is not listed here. The other treatment options are not appropriate for Index Patient 2. Cabazitaxel is indicated for patients who have failed prior docetaxel chemotherapy. Radium-223 in general is reserved for patients with symptomatic bone metastases. Mitoxantrone has not been shown to provide a survival advantage and in general has been used for palliative purposes in symptomatic patients. Cookson MS, Kibel AS, Dahm P, et al: Castration-resistant prostate cancer: AUA GUIDELINE. American Urological Association Education and Research, Inc, 2013. <a href="http://www.auanet.org/education/guidelines/castration-resistant-prostate-cancer.cfm" target="_new"><u>http://www.auanet.org/education/guidelines/castration-resistant-prostate-cancer.cfm</u></a>

2015 - 38 According to the AUA Guidelines, a patient with progressive metastatic castrate resistant prostate cancer having pain controlled with acetaminophen should be offered treatment with: A. observation. B. sipuleucel-T. C. cabazitaxel. D. radium-223 E. mitoxantrone.

A This patient has a persistent mass following hormonal therapy for endometriosis invading the bladder. She had an adequate trial of GnRH agonist therapy. The next step is upper tract imaging; this test should be obtained in all patients with pelvic endometriosis prior to and following hormonal therapy, and again prior to surgical intervention due to the potential for silent upper urinary tract obstruction which can occur in 10-20% of these women. Repeat biopsy of the bladder mass, endoscopically or percutaneously, is unlikely to be helpful as it will show either fibrosis or persistent endometriosis. Partial or radical cystectomy is overly aggressive and certainly not indicated until the upper tracts have been evaluated. An anatomic study with CT urogram will provide more information than nuclear renography and will complete the hematuria workup. Singh I, Strandhoy JW, Assimos DG: Pathophysiology of urinary tract obstruction, Wein, AJ, Kavoussi LR, Novick AC, Partin AW, Peters CA (eds): CAMPBELL-WALSH UROLOGY, ed 10. Philadelphia, Elsevier Saunders, 2012, vol 2, chap 40, pp 1115-1116.

2015 - 39 A 35-year-old woman with urinary urgency and frequency has a pelvic mass and gross hematuria. Cystoscopy and biopsy of the mass reveals endometriosis. A CT cystogram after four months of a GnRH agonist is shown. The next step is: A. CT urogram. B. repeat biopsy of mass. C. transurethral resection of mass. D. partial cystectomy. E. radical cystectomy with urinary diversion.

C Up to 10% of patients with hypertension may have an element of renal vascular disease as the etiology of their rise in blood pressure. In patients with bilateral renal artery disease, hypertension is largely a volume-dependent phenomenon with excess fluid volume protecting renal function. When diuretics are given to these patients, volume depletion occurs, with renal perfusion subsequently becoming angiotensin-dependent. The combination of diuretics and ACE inhibitors in a patient with bilateral renal artery stenosis will therefore result in the onset of renal insufficiency. Based on this knowledge, the use of the combination of ACE inhibitors with diuretics may be used as a provocative test to identify patients with bilateral ischemic (renal vascular) nephropathy. In essence, a finding of an elevation in serum creatinine within two to four weeks of starting the combination of a diuretic and an ACE inhibitor is highly suggestive of the presence of bilateral renal artery stenosis. Screening for renal artery stenosis in this clinical scenario is mandatory. Captopril enhanced testing is less accurate in the setting of renal insufficiency and is not the test of choice in patients associated with an elevation in serum creatinine. Renin-based testing is mainly utilized to determine the possible presence of renovascular-induced hypertension, and is not indicated once bilateral ischemic nephropathy has been suspected to be present by a provocative test using a diuretic and an ACE inhibitor. The key evaluation in this patient is the anatomical assessment of the renal arteries to determine the possibility for vascular intervention. Imaging studies used to diagnose renal artery stenosis include ultrasound, contrast-enhanced CT angiography, and contrast-enhanced or nonenhanced magnetic resonance (MR) angiography. Although ultrasound is an effective screening tool, visualization of the entire renal artery to assess for interventional repair can be problematic. Contrast-enhanced CT and MR angiography can provide exquisite details of the renal arterial anatomy, and are highly accurate for determining both the diagnosis and extent of renal artery stenosis. However, the use of iodinated contrast for CT or the gadolinium-based contrast for MR angiography may be problematic for patients with renal dysfunction, eGFR < 30 ml/min/BSA. In these patients, the iodinated CT contrast may potentially cause further kidney injury, and the use of gadolinium-based contrast can lead to a condition called nephrogenic systemic fibrosis (fibrosis of the skin, joints, and internal organs) that will lead to significant morbidity or death. The preferred test of choice in a patient with an eGFR of < 30 ml/min/BSA under consideration for surgical intervention is the use of nonenhanced MR angiography. Fergany A, Novick AC: Renovascular hypertension and ischemic nephropathy, Wein AJ, Kavoussi LR, Novick AC, Partin AW, Peters CA (eds): CAMPBELL-WALSH UROLOGY, ed 10. Philadelphia, Elsevier Saunders, 2012, vol 2, chap 39, pp 1062-1063.

2015 - 4 A 54-year-old man with hypertension and a creatinine of 1.7 mg/dl is started on an ACE inhibitor. After two weeks, the creatinine is unchanged, but hypertension persists and a diuretic is added. One week later, the creatinine is 2.5 mg/dl (eGFR of 27 ml/min/1.73 m^2). The next test is: A. split renal vein renin measurements. B. contrast-enhanced MR angiography. C. nonenhanced MR angiography. D. contrast-enhanced CT angiography. E. captopril renography.

B In women with recurrent symptomatic UTI, continuous low-dose antibiotic prophylaxis or if the recurrent UTI can be related to intercourse post-coital antibiotics are indicated. Appropriate antibiotics include: trimethoprim-sulfamethoxazole, nitrofurantoin, and cephalexin. Fluoroquinolones should be reserved for instances of bacterial resistance or allergy. Therapy is usually continued for six months followed by a trial period off prophylaxis. Other strategies such as post-coital voiding, changing to cotton underwear, wiping away from the urethra and avoidance of hot tubs have not been shown to decrease the rate of infections. Cystoscopy is not indicated for recurrent simple cystitis in women. Schaeffer AJ, Schaeffer EM: Infections of the urinary tract, Wein, AJ, Kavoussi LR, Novick AC, Partin AW, Peters CA (eds): CAMPBELL-WALSH UROLOGY, ed 10. Philadelphia, Elsevier Saunders, 2012, vol 1, chap 10, p 260.

2015 - 41 A 25-year-old woman has recurrent pan-sensitive E. coli UTIs with urgency and frequency but no fever. The next step is: A. post-coital voiding. B. nightly trimethoprim-sulfamethoxazole. C. nightly fluoroquinolone. D. abdominal ultrasound. E. cystoscopy.

A ANOVA (analysis of variance) is used when comparison is being made between the mean of more than two groups. A t-test is used to make comparison between the mean of two groups. Chi-square test is used to compare differences in proportions. Pearson r test is used to evaluate the strength and direction of an association. Spearman rank order correlation is used to compare ordinal data. Glaser AN: HIGH YIELD BIOSTATISTICS, ed 3. Philadelphia, Lippincott, Williams, & Wilkins, 2005, p 41. <a href="http://www.auanet.org/education/modules/core/topics/bus-comm-research/basic-research-stats/index.cfm#BIOSTATISTICS" target="_new"><u>http://www.auanet.org/education/modules/core/topics/bus-comm-research/basic-research-stats/index.cfm#BIOSTATISTICS</u></a>

2015 - 42 A novel medication is being studied to determine efficacy in reducing urinary frequency in patients with overactive bladder. The best statistical method to compare the mean number of voiding episodes per day in three groups of subjects receiving either one of two doses of the medication or placebo is: A. ANOVA (analysis of variance). B. chi-square test. C. Pearson r test. D. t-test. E. Spearman rank order.

C It is very unlikely that this patient, who is in complete urinary retention one month after a retropubic mid-urethral sling, will resume normal voiding. If the patient had undergone an autologous sling, it would be appropriate to wait three months before intervening as spontaneous sling loosening may occur with resorption of the sling. In this patient with a synthetic sling, if she desires to void spontaneously, she will need to have the sling loosened or cut. Sling loosening may be attempted within the first week to ten days after surgery, but this must be done surgically by exposing the sling and attempting to loosen it and not by urethral dilation. If loosening is not selected or is ineffective, the sling will need to be cut. Incision of the synthetic sling will restore voiding in approximately 90% of patients; however, recurrent stress incontinence may occur in 15-20% of patients. Transvaginal urethrolysis is indicated when an incision does not work or if the urethra is felt to be fixed to the underside of the pubic symphysis. Suprameatal urethrolysis is unnecessary following mid-urethral sling procedures, as there is very little scarring immediately anterior to the urethra, and the obstruction is presumably due to excessive obstruction from the suburethral sling. Classically, suprameatal urethrolysis is indicated for obstruction following a Marshall-Marchetti-Krantz procedure, or for persistent obstruction following sling incision and/or transvaginal (submeatal) urethrolysis. Dmochowski RR, Padmanabhan P, Scarpero HM: Slings: Autologous, biologic, synthetic, and midurethral, Wein AJ, Kavoussi LR, Novick AC, Partin AW, Peters CA (eds): CAMPBELL-WALSH UROLOGY, ed 10. Philadelphia, Elsevier Saunders, 2012, vol 3, chap 73, pp 2133-2134.

2015 - 43 A 65-year-old woman undergoes a retropubic midurethral synthetic sling and is unable to void after surgery. At one month, she is still catheterizing herself and is unable to void on her own. The next step is: A. continue CIC and reassess at three months. B. urethral dilation. C. sling incision. D. transvaginal urethrolysis. E. suprameatal urethrolysis.

D The need for ventriculo-peritoneal shunting was found in approximately 70% of the infants in the prenatal-surgery group and 98% of those in the postnatal-surgery group. Prenatal surgery also resulted in improvement in the composite score for mental development and motor function at 30 months (p=0.007) and an improvement in several secondary outcomes, including ambulation by 30 months. However, prenatal surgery was associated with an increased risk of preterm delivery and uterine dehiscence at delivery. There has been no documentation of a positive effect on the incidence of spinal cord tethering, urinary continence or bowel function with in utero myelomeningocele repair. In non-randomized, controlled studies, prenatal surgery has not been shown to result in improved bladder dynamics nor function compared to historical controls. Adzick NS, Thom EA, Spong CY, et al. A randomized trial of prenatal versus postnatal repair of myelomeningocele. NEJM 2011;364:993-1004. Clayton DB, Tanaka ST, Trusler L, et al: Long-term urological impact of fetal myelomeningocele closure. J UROL 2011;186:1581-1585.

2015 - 44 In utero myelomeningocele closure has a favorable impact on: A. incidence of spinal cord tethering. B. bladder continence. C. bowel function. D. the need for ventriculo-peritoneal shunting. E. complications at delivery.

C Urologists frequently obtain imaging studies utilizing I.V. contrast agents that are often performed in the office under their direct supervision. Since use of these agents is associated with adverse events including potentially life-threatening anaphylactic-like reactions, an appreciation of the risk factors predisposing to these adverse events is essential. Adverse reactions to radiocontrast media are classified as either systemic allergic-type reactions or chemotoxic-type reactions. Chemotoxic events arise as a result of the physiochemical properties of radiocontrast agents, and include contrast-induced renal failure and seizures. Individuals with poor renal function, diabetes and intravascular volume depletion are predisposed to these events. Obesity, due to its association with metabolic syndrome and diabetes, is a known risk that increases the patient's susceptibility to a chemotoxic event. System allergic-type reactions occur due to the release of active cellular mediators that can result in urticaria, bronchospasm, laryngeal edema, hypotension, and anaphylaxis-like reactions. Patients who have a history of multiple systemic allergies (drug or nutritional allergies) or a history of asthma account for an inordinately large percentage of patient with allergic reactions. In point of fact a history of asthma results in a 3-5x increased risk of a systemic allergic type of reaction to radiocontrast media. "Allergy" to Betadine is a type of contact dermatitis and not associated with increased risk. There does not appear to be any racial differences in risk to either type of event. Bush WH Jr, Lasser EC. Adverse reactions to intravascular contrast material, in Pollack HM, Mc Clennan BL (eds), Clinical Urography, ed 2. Philadelphia, WB Saunders Co, 2000, vol 1, chap 4, pp 43-66. Fulgham PF, Bishoff JT: Urinary tract imaging: Basic principles, Wein, AJ, Kavoussi LR, Novick AC, Partin AW, Peters CA (eds): CAMPBELL-WALSH UROLOGY, ed 10. Philadelphia, Elsevier Saunders, 2012, vol 1, chap 4, p 102.

2015 - 45 A risk factor for systemic allergic-type reactions to radiocontrast media is: A. povidone-iodine (Betadine) allergy. B. African-American ethnicity. C. asthma. D. obesity. E. diabetes.

E Autonomic dysreflexia (AD), a syndrome of unopposed sympathetic discharge classically occurs in patients with a complete spinal cord injury (SCI) at or above T-6 (above the T10-L2 sympathetic outflow. This dysreflexic response will typically occur secondary to, visceral distension (bladder or bowel), or pain stimulation below the level of the lesion. Symptoms classically are sweating and diaphoresis above the level of the lesion, a blood pressure rise of > 20 mm Hg over baseline levels, headache and bradycardia. In the treatment of AD, it should always be assumed that the bladder is distended and/or the urethral catheter malpositioned. Drainage of the bladder, placement of a urethral catheter, or verification that an indwelling catheter is functional and in the correct position, should always be the first step in management. All clothing should subsequently be loosened and the patient's upper torso should be elevated. If these maneuvers do not result in a decrease in blood pressure, topical or oral nitroglycerin is the recommended first line medical therapy. Topical nitroglycerin is preferred due to the ability to wipe off the medication from the skin if rebound hypotension should occur. Prior to nitroglycerin use, it must be verified that the patient has not taken a PDE-5 inhibitor within the prior 24 hours, the combination of NTG and a PDE-5 inhibitor increases the risk of severe rebound hypotension. If the patient has used a PDE-5 inhibitor, captopril 25 mg given sublinguinally or chewed is the drug of choice. Sublingual nifedipine once routinely recommended for this complication is no longer the drug of choice due to variable absorption, and episodic rebound hypotension that has resulted in strokes or myocardial infarction. If the blood pressure does not improve rapidly or rebound, hypertension develops the patient should be examined for other causes of AD including fecal impaction, renal or bladder calculi, decubitus ulcers and asymptomatic broken bones. Wein AJ, Dmochowski RR: Neuromuscular dysfunction of the lower urinary tract, Wein, AJ, Kavoussi LR, Novick AC, Partin AW, Peters CA (eds): CAMPBELL-WALSH UROLOGY, ed 10. Philadelphia, Elsevier Saunders, 2012, vol 3, chap 65, p 1926.

2015 - 46 A 64-year-old, T4 paraplegic man on CIC is admitted for treatment of pneumonia. He suddenly develops a severe headache and has a heart rate of 42 bpm, and blood pressure of 210/130 mmHg. The next step is: A. sublingual nifedipine. B. sublingual terazosin. C. oral nitroglycerine. D. nitroglycerine paste. E. place a urethral catheter.

A A presumptive diagnosis of testicular teratoma can often be made based on testicular ultrasound findings. When a prepubertal testicular teratoma is expected it should be approached via an inguinal incision with vascular control of the spermatic cord. Juvenile testicular teratomas may be treated by either partial orchiectomy or orchiectomy depending upon the size of the mass. This prepubertal boy had a unilateral mature testicular teratoma that has been completely resected with radical orchiectomy. In a prepubertal boy, this is a benign lesion and can be followed with serial annual examinations if an orchiectomy was performed, or serial annual testicular ultrasound evaluations if a partial orchiectomy or enucleation was performed. Follow-up should be through puberty to verify adequate hormonal function of the contralateral testis. In a patient with a prepubertal testicular teratoma there is no need for further CT scans, tumor markers, surgery, or chemotherapy. However, mature teratoma in the pubertal child or postpubertal adolescent has a clinical behavior similar to adults and should be managed with a standard post-orchiectomy protocol for NSGCT. Ritchey ML, Shamberger RC: Pediatric urologic oncology, Wein, AJ, Kavoussi LR, Novick AC, Partin AW, Peters CA (eds): CAMPBELL-WALSH UROLOGY, ed 10. Philadelphia, Elsevier Saunders, 2012, vol 4, chap 137, p 3727.

2015 - 48 An eight-year-old boy with a large expanding cystic right testicular mass has normal tumor markers and organ confined mature testicular teratoma on radical orchiectomy. The next step is: A. serial examination. B. abdomen and pelvis CT scan. C. serial tumor markers. D. RPLND. E. platinum-based chemotherapy.

A The overall success rate for obtaining an erection with intraurethral alprostadil is approximately 55%. If it is successful in producing an erection, the most common side effect is penile pain that can include the scrotum and extremities. No treatment is needed for this pain, but it can be dose limiting in some patients. Terbutaline, methylene blue and Neo-Synephrine are useful for the treatment of priapism, which this patient does not have. No data is available for the use of ibuprofen with PGE-1 induced pain and it is unlikely to work based on ibuprofen's mechanism of action. Burnett AL: Evaluation and management of erectile dysfunction, Wein, AJ, Kavoussi LR, Novick AC, Partin AW, Peters CA (eds): CAMPBELL-WALSH UROLOGY, ed 10. Philadelphia, Elsevier Saunders, 2012, vol 1, chap 24, p 745.

2015 - 49 A man with erectile dysfunction is given a test dose of intraurethral alprostadil 1000 mcg, and achieves complete rigidity. He complains of penile, scrotal, and leg pain during the erection. The next step is: A. reassurance. B. oral terbutaline. C. intraurethral lidocaine. D. methylene blue intracavernosal injection. E. phenylephrine intracavernosal injection.

E The patient has a 5 cm hematocele following blunt scrotal trauma with an indeterminate ultrasound examination. Significant hematoceles (5 cm or greater) should be explored, regardless of imaging studies, as up to 80% will be associated with a testicular rupture. The increased area of echogenicity does not infer tumor, and thus, tumor markers are not indicated. MRI will not add useful information. Morey AF, Dugi DD III: Genital and lower urinary tract trauma, Wein, AJ, Kavoussi LR, Novick AC, Partin AW, Peters CA (eds): CAMPBELL-WALSH UROLOGY, ed 10. Philadelphia, Elsevier Saunders, 2012, vol 3, chap 88, pp 2510-2512.

2015 - 5 A 25-year-old man has left scrotal pain after sustaining an injury playing soccer. The left testis is tender and enlarged on exam. Scrotal ultrasound reveals a 5cm hematocele, normal intraparenchymal blood flow, and a focal area of increased left testis echogenicity. The tunica albuginea cannot be fully visualized. The next step is: A. observation. B. MRI scan of the scrotum. C. repeat scrotal ultrasound in 48 hours. D. obtain tumor markers. E. scrotal exploration.

B The horseshoe kidney is positioned more inferior, anterior and medial than a normal kidney. The upper pole is typically subcostal and superficial, making it the best option for percutaneous access. The medial position of the kidney often requires a percutaneous tract that passes through the paraspinous musculature. Yap WW, Wah T, Joyce AD: Horseshoe kidney, in Smith AD, Badlani GH, Preminger GM, Kavoussi LR (eds): SMITH'S TEXTBOOK OF ENDOUROLOGY, ed 3. Oxford UK, Blackwell Publishing, 2012, vol 1, chap 61, pp 702-706.

2015 - 50 Compared to a normal kidney, the percutaneous access for nephrolithotomy in the kidney shown will be more: A. superior and medial. B. inferior and medial. C. superior and lateral. D. inferior and lateral. E. posterior and medial.

D The right adrenal vein enters the IVC directly on its posterolateral aspect. It does not enter other veins between the adrenal gland and the IVC as occurs on the left side. The left adrenal vein joins with the left phrenic vein and enters the cranial aspect of the left renal vein. The lumbar vein and left gonadal vein enter the left renal vein but do not receive the adrenal vein. Anderson JK, Cadeddu JA: Surgical anatomy of the retroperitoneum, adrenals, kidneys, and ureters, Wein, AJ, Kavoussi LR, Novick AC, Partin AW, Peters CA (eds): CAMPBELL-WALSH UROLOGY, ed 10. Philadelphia, Elsevier Saunders, 2012, vol 1, chap 1, p 19.

2015 - 51 The right adrenal vein enters: A. right renal vein. B. right inferior phrenic vein. C. right gonadal vein. D. IVC. E. ascending lumbar vein.

E This patient has progressive urologic symptomatology that is refractory to a therapeutic modality that was once effective. When the physician encounters a patient with progression of the severity of symptoms, particularly in conjunction with new neurologic symptoms, a neurologic diagnosis such as multiple sclerosis should be considered. Performance of MRI (below the neck) is important in establishing the diagnosis of a neuropathic disease, and is contraindicated in patients with a sacroneuromodulation device in place. Therefore, it must be removed to allow this patient to proceed with the diagnostic MRI scan. Although evaluation of the position of the lead, consideration of reprogramming of the device, surgical revision of the device (pending the effects of reprogramming), or change in treatment modality might all be reasonable, in a patient without progressive neurologic deterioration they are not reasonable alternatives in this clinical scenario. Betts CD, D'Mellow MT, Fowler CJ: Urinary symptoms and the neurological features of bladder dysfunction in multiple sclerosis. J NEUROL NEUROSUR PSYCH 1993;56:245-250. Awad SA, Gajewski JB, Soghein SK, et al: Relationship between neurological and urological status in multiple sclerosis. J UROL 1984;132:499-502. Vasavada SP, Rackley RR: Electrical stimulation and neuromodulation in storage and emptying failure, Wein, AJ, Kavoussi LR, Novick AC, Partin AW, Peters CA (eds): CAMPBELL-WALSH UROLOGY, ed 10. Philadelphia, Elsevier Saunders, 2012, vol 3, chap 70, p 2034.

2015 - 52 Three years after placement of a sacral neuromodulator for refractory urinary urgency and urgency incontinence, a 45-year-old woman develops new symptoms of blurred vision, numbness in her lower extremities, and significant exacerbation of her urinary symptoms. The next step is: A. anterior, posterior and lateral radiograph of the sacrum. B. reprogramming of the device. C. MRI scan of the brain and spine. D. surgical revision of the impulse generator. E. removal of the device.

E Hyperparathyroidism should be suspected in patients with renal calculi and serum calcium levels over 10.1 mg/dl. In patients with suspected hyperparathyroidism, a thiazide challenge may unmask subtle primary hyperparathyroidism by increasing proximal tubular resorption of calcium resulting in a significant rise in serum calcium. The treatment of patients with primary hyperparathyroidism and renal calculi is parathyroidectomy, with over 90% improvement in calculus recurrence. In patients who present with symptomatic or obstructive renal calculi and who are not in hypercalcemic crisis, the calculi should be treated prior to the parathyroid gland. Pearle MS, Lotan Y: Urinary lithiasis: Etiology, epidemiology, and pathogenesis, Wein, AJ, Kavoussi LR, Novick AC, Partin AW, Peters CA (eds): CAMPBELL-WALSH UROLOGY, ed 10. Philadelphia, Elsevier Saunders, 2012, vol 2, chap 45, p 1270.

2015 - 53 A 38-year-old woman with recurrent nephrolithiasis has a serum calcium of 10.8 mg/dl and serum parathyroid hormone level of 85 pg/ml. After administration of thiazide, serum calcium is 11.8 mg/dl. She is currently stone free. The treatment that will best reduce her risk of nephrolithiasis is: A. sodium restriction. B. potassium citrate. C. low calcium diet. D. orthophosphates. E. parathyroidectomy.

D The t-test is the most commonly used method for comparison of means between two groups. Chi-square analysis is the most important nonparametric test and is used to compare proportions. ANOVA is the appropriate test when more than two groups are being compared. Pearson's r test is used to evaluate strength and direction of the relationship between two interval variables. Spearman's rank order test is used to test for an association between ordinal positions in rankings. Glaser AN: HIGH YIELD BIOSTATISTICS, ed 3. Philadelphia; Lippincott, Williams, & Wilkins, 2005, pp 41-53. <a href="http://www.auanet.org/education/modules/core/topics/bus-comm-research/basic-research-stats/index.cfm#BIOSTATISTICS" target="_new"><u>http://www.auanet.org/education/modules/core/topics/bus-comm-research/basic-research-stats/index.cfm#BIOSTATISTICS</u></a>

2015 - 54 A novel medication is being studied for the treatment of urinary frequency. The best statistical method to compare the mean number of voiding episodes per day in subjects receiving the medication versus those receiving placebo is: A. chi-square test. B. ANOVA (analysis of variance). C. Pearson r test. D. t-test. E. Spearman rank order test.

E Indications for autologous sling include a severely dysfunctional urethra, as indicated by low LPP (0-60 cm H2O), loss of urethral tissue (e.g., following synthetic mesh erosion into the urethra, urethral diverticulectomy, or urethrovaginal fistula repair), and multiple previous anti-incontinence procedures. While the other options listed are reasonable options to discuss with patients, this patient's low LPP and her history of previous surgery make autologous sling the best option of those listed. Dmochowski RR, Padmanabhan P, Scarpero HM: Slings: Autologous, biologic, synthetic, and midurethral, Wein, AJ, Kavoussi LR, Novick AC, Partin AW, Peters CA (eds): CAMPBELL-WALSH UROLOGY, ed 10. Philadelphia, Elsevier Saunders, 2012, vol 3, chap 73, p 2116.

2015 - 55 Three years following placement of a retropubic midurethral sling, a 58-year-old woman has recurrent stress urinary incontinence. Valsalva LPP is 32 cm H2O with a stable bladder and a capacity of 400 ml. The urethra is well-supported. The best option is: A. pelvic floor muscle training. B. imipramine. C. Burch urethropexy. D. transobturator sling. E. autologous fascial sling.

D Pyocystitis may be a complication following supravesical diversion in individuals with a neurogenic bladder when a cystectomy is not performed. The failure of irrigation therapy to permanently suppress recurrent pyocystitis is an indication for surgical intervention. Vesicovaginostomy allows the bladder to drain and usually results in symptomatic improvement in patients who have pyocystitis unresponsive to standard treatment. In this woman, vesicovaginostomy would be a significantly less morbid option than simple cystectomy. Spence HM, Allen TD: Vaginal vesicostomy for empyema of the defunctionalized bladder. J UROL 1971;106:862-864. Khoudary KP, Green DH, Koudary ML, Wilkerson JE, Summers JL: Vaginovesicostomy using absorbable staples. BR J UROL 1997;79:127-128. Fazili T, Bhat TR, Masood S, et al: Fate of the leftover bladder after supravesical urinary diversion for benign disease. J UROL 2006;176:620-621.

2015 - 56 A 60-year-old paraplegic woman with multiple medical problems has an ileal conduit because she was unable to perform intermittent catheterization. She develops pyocystis unresponsive to three weeks of oral ciprofloxacin. Pyocystis recurs one week following three days of intravesical bladder irrigation with neomycin. The next step is: A. formalin bladder irrigation. B. suprapubic cystotomy. C. broad spectrum I.V. antibiotics. D. vesicovaginostomy. E. convert to ileovesicostomy.

B The incidence of renal calculi in patients with UPJ obstruction is nearly 20%. Husmann and colleagues reported a 70-fold increased risk of stone formation in the pediatric population with UPJ obstruction. Although the obstruction plays a role in stone formation, several studies have demonstrated that patients with UPJ obstruction and concurrent renal calculi carry the same metabolic risks as other stone formers. Correction of UPJ obstruction did not prevent recurrent stones in most patients, and thus metabolic evaluation, rather than annual urinalysis alone, is the correct next step. Based on the findings of the metabolic evaluation, treatments such as dietary changes, potassium citrate, or hydrochlorothiazide may be appropriate, but not until the work-up is completed. Despite the fact that this was discovered during the evaluation of a UTI, antibiotic prophylaxis is not indicated. Pearle MS, Lotan Y: Urinary lithiasis: Etiology, epidemiology, and pathogenesis, Wein AJ, Kavoussi LR, Novick AC, Partin AW, Peters CA (eds): CAMPBELL-WALSH UROLOGY, ed 10. Philadelphia, Elsevier Saunders, 2012, vol 2, chap 45, p 1284.

2015 - 57 A six-year-old boy undergoes right pyeloplasty and pyelolithotomy for UPJ obstruction and 1 cm renal pelvic stone. The stone is composed of calcium oxalate. Three months post-op, ultrasound shows improved hydronephrosis and diuretic renography shows no obstruction. The next step is: A. observation. B. metabolic stone evaluation. C. low oxalate diet. D. hydrochlorothiazide. E. potassium citrate.

C Repair of vesicovaginal fistula, whether via an open or minimally invasive technique, requires adherence to basic principles of fistula repair. These include: adequate excision of the fistula, use of healthy tissues for repair, performance of a tension free anastomosis with multi-layered closure, interposition with omentum, and adequate bladder drainage. However, the challenge of a minimally invasive approach is losing pneumoperitoneum after the fistula is excised. Techniques such as packing gauze in the vagina and clamping the urethral catheter are helpful but they do not seal the opening adequately to maintain pneumoperitoneum and thus can make suturing more difficult. If pneumoperitoneum is maintained, exposure and ease of suturing should be no different. Risk of ureteral injury and port site complications should not differ between the procedures. Richstone L, Scherr DS: Robotic and Laparoscopic Bladder Surgery, in Wein AJ, Kavoussi LR, Novick AC, Partin AW, Peters CA (eds): CAMPBELL'S UROLOGY, ed 10. Philadelphia, Saunders Elsevier, 2012, vol 4, chap 84, p 2398.

2015 - 58 A unique challenge of robotic vesicovaginal fistula repair as compared to robotic sacrocolpopexy is: A. adequate exposure. B. ease of suturing. C. maintaining pneumoperitoneum. D. avoiding ureteral injury. E. port site complications.

B Streptococcal and Clostridial wound infections are characteristically invasive, painful, and occur within 24 hours after surgery. A thin, watery purulent discharge without frank abscess formation or foul smell is characteristic for Streptococcal infections. Clostridial infections are usually associated with intraoperative fecal contamination; the discharge is gray or reddish brown and foul smelling, and associated with wound crepitus and necrosis. Treatment should include systemic high dose penicillin. Opening of the surgical wound with debridement and drainage is necessary only if there are signs of crepitus or wound fluctuance or wound margin necrosis. Staphylococcal infections usually occur > 24 hours postoperatively, and are characterized by a localized indurated area of cellulitis with associated abscess formation with a thick yellow or cream-colored pus. Postoperative wound infections caused by enteric bacilli have a longer incubation period than those caused by staphylococcus. Mandell J: Cellulitis, necrotizing fasciitis, and subcutaneous tissue infections, in MANDELL, DOUGLAS AND BENNETT'S PRINCIPLES AND PRACTICE OF INFECTIOUS DISEASES, ed 7. Churchill, Livingston and Elsevier, London, 2009, chap 990, p 1295.

2015 - 59 Eighteen hours after a radical nephrectomy, a 35-year-old man has a high grade fever, pain, and impressive erythema at the operative site associated with a thin, watery discharge from the incision. The infection is most likely caused by: A. Clostridium perfringens. B. beta-hemolytic streptococci. C. Staphylococcus aureus. D. Pseudomonas aeruginosa. E. Candida albicans.

E This patient has vaginal vault prolapse. The image from the videourodynamics study does not demonstrate a cystocele. The majority of physicians would recommend that this patient should undergo repair of the vault prolapse with a concurrent anti-incontinence procedure. The concurrent anti-incontinence procedure is performed due to the increased risk of de novo stress incontinence following vault suspension. In the context of a robotic sacrocolpopexy, a midurethral sling would be the most appropriate approach. Urodynamics, with or without prolapse reduction, are not predictive of which patients will develop de novo SUI following vault suspension. While acceptable to proceed with robotic sacrocolpopexy and no sling, the patient should be informed of the risk of postoperative stress incontinence. Some patients may prefer this approach due to the inherent risks of sling procedure, however rare they may be. Winters JC, Togami JM, Chermansky CJ: Vaginal and abdominal reconstructive surgery for pelvic organ prolapse, Wein AJ, Kavoussi LR, Novick AC, Partin AW, Peters CA (eds): CAMPBELL-WALSH UROLOGY, ed 10. Philadelphia, Elsevier Saunders, 2012, vol 3, chap 72, p 2090.

2015 - 6 A 55-year-old active woman desires surgical repair of a vaginal bulge. She has urinary frequency but no urinary or fecal incontinence. The physical examination with a cystoscope in the urethra is shown followed by a cystogram at maximal Valsalva taken during a videourodynamic study. The next step is: A. anterior (cystocele) repair with sling. B. transvaginal vault suspension and anterior (cystocele) repair. C. uterosacral vault suspension and rectocele repair. D. robotic sacrocolpopexy. E. robotic sacrocolpopexy and midurethral sling.

A Primary idiopathic (endemic) calculi form in children, most commonly from North Africa, the Middle East and Far East. With a large immigrant population in the United States, it is important to be aware of this health problem. These children rely on a cereal-based diet that is lacking in animal proteins. This leads to a dietary phosphate deficiency, low urinary phosphate and high peaks of ammonia. Due to this, the most common stone is ammonium acid urate. Though chronic dehydration can lead to calcium oxalate and uric acid stones, high urinary sodium, calcium and oxalate are not characteristic findings with endemic bladder stones. Benway BM, Bhayani SB: Lower urinary tract calculi, Wein, AJ, Kavoussi LR, Novick AC, Partin AW, Peters CA (eds): CAMPBELL-WALSH UROLOGY, ed 10. Philadelphia, Elsevier Saunders, 2012, vol 3, chap 89, p 2522. Ost MC, Schneck FX: Surgical management of pediatric stone disease, Wein, AJ, Kavoussi LR, Novick AC, Partin AW, Peters CA (eds): CAMPBELL-WALSH UROLOGY, ed 10. Philadelphia, Elsevier Saunders, 2012, vol 4, chap 135, p 3682.

2015 - 60 A four-year-old boy who recently emigrated from Ethiopia has gross hematuria. There is no history of UTI. KUB demonstrates a 2 cm bladder stone. The most likely stone composition is: A. ammonium acid urate. B. calcium oxalate. C. calcium phosphate. D. cystine. E. struvite.

C This patient would benefit from mesh excision and closure of the defect. Patients with asymptomatic vaginal mesh exposure can be observed. However, if symptomatic or desiring definitive treatment, the exposed mesh should be removed. Factors responsible for mesh exposure include poor vaginal tissue quality, excessive sling tension, infection, and type of graft material. Multifilament products appear to have greater risk for exposure compared to monofilament products and larger pore sizes in the graft decrease the risk of exposure. It is unlikely an exposure of this size would heal secondarily or with the use of topical estrogens. The majority of patients undergoing removal of exposed mesh do not have recurrent stress urinary incontinence postoperatively; thus, a concomitant sling is unnecessary. Rovner ES: Complications of female incontinence surgery, Taneja SS (ed): COMPLICATIONS OF UROLOGIC SURGERY, ed 4. Philadelphia, Elsevier Saunders, 2010, chap 50, p 590.

2015 - 61 A 55-year-old woman has dyspareunia several months after a mid-urethral sling. She has no residual urinary symptoms. Examination reveals 2 cm of mesh exposure along the anterior vaginal wall and urinalysis is normal. The next step is: A. observation with secondary healing. B. topical estrogen cream. C. excision of exposed mesh and closure of vaginal wall defect. D. excision of exposed mesh and repeat synthetic midurethral sling. E. excision of exposed mesh and autologous fascial sling.

A Cushing's syndrome may be caused by an adrenal tumor, ectopic ACTH production, and by excessive pituitary ACTH secretion (Cushing's disease). The basis of the high dose dexamethasone suppression test is that ACTH secretion in patients with Cushing's disease is not completely, but only partially, resistant to glucocorticoid feedback inhibition. Therefore, by increasing the dose of dexamethasone, pituitary secretion of ACTH is suppressed in patients with Cushing's disease and glucocorticoid production is reduced. In contrast, dexamethasone has no effect in patients with adrenal tumors and ectopic ACTH production, since their pituitary glands are already suppressed. Serum cortisol may be used instead of urinary parameters. Kutikov A, Crispen PL, Uzzo RG: Pathophysiology, evaluation, and medical management of adrenal disorders, Wein, AJ, Kavoussi LR, Novick AC, Partin AW, Peters CA (eds): CAMPBELL-WALSH UROLOGY, ed 10. Philadelphia, Elsevier Saunders, 2012, vol 2, chap 57, pp 1691-1696.

2015 - 62 The finding on a high dose dexamethasone suppression test (2 mg every six hours) that establishes the diagnosis of Cushing's disease is: A. suppression of urinary 17-hydroxycorticosteroids. B. no change in urinary 17-hydroxycorticosteroids. C. elevation of urinary 17-hydroxycorticosteroids. D. suppression of urinary 17-ketosteroids. E. elevation of serum ACTH.

A At this stage of gestation, 90% of the amniotic fluid volume is derived from the fetal urine production. Oligohydramnios early in pregnancy nearly always predicts a poor renal outcome. Echogenicity is more subjective and may be indicative of dysplasia, but is not always predictive of a poor renal outcome. Although concerning, a kidney with thinned renal cortex may have normal function. The thinned cortex may simply be an anatomical or physical reflection of the severity of the obstructive process. A distended bladder may be normal or may only be indicative of VUR, and the cycling of refluxed urine that over time will cause stretching and expansion of the bladder. The magnitude of renal pelvic dilation may predict the degree of obstruction but does not correlate with renal function. Lee RS, Borer JG: Perinatal urology, Wein, AJ, Kavoussi LR, Novick AC, Partin AW, Peters CA (eds): CAMPBELL-WALSH UROLOGY, ed 10. Philadelphia, Elsevier Saunders, 2012, vol 4, chap 114, p 3050.

2015 - 63 Prenatal ultrasonography in a 22-week fetus shows bilateral hydroureteronephrosis. The parameter most predictive of a poor postnatal renal outcome is: A. oligohydramnios. B. a persistently distended bladder. C. diameters of the renal pelves. D. renal cortical thinning. E. echogenic kidneys.

D The most frequently selected insufflation pressure for laparoscopy is 15 mmHg. If insufflation pressures of > 20 mmHg develop, three major systems are affected: 1) a decreased venous return and cardiac output occurs results in tachycardia, 2) an increase in pressure on the renal parenchyma results in a decrease in GFR and oliguria, 3) expanded abdominal pressure results in a decrease in diaphragmatic movement, decreased pulmonary insufflation, hypercarbia, and respiratory acidosis. When faced with the classic triad of tachycardia, oliguria and hypercarbia, the next step is to increase the respiratory rate and decrease the insufflation pressure. Although increasing the PEEP is advisable in patients with lung disease, this would not address the multiple systemic problems seen with elevated intraabdominal pressure. The steep Trendelenburg can cause a decrease in heart rate and systemic vascular resistance and a rise in mean arterial pressure and cardiac output. I.V. fluid bolus would not alter the physiologic condition and is not necessary to address the mild oliguria associated with pneumoperitoneum. Eichel L, Clayman RV: Fundamentals of laparoscopic and robotic urologic surgery, Wein AJ, Kavoussi LR, Novick AC, Partin AW, Peters CA (eds): CAMPBELL-WALSH UROLOGY, ed 10. Philadelphia, Elsevier Saunders, 2012, vol 1, chap 9, pp 231-233.

2015 - 64 During a robotic sacrocolpopexy, a patient has tachycardia and hypercarbia. In addition to increasing respiratory rate, the next step is: A. increase positive end expiratory pressure (PEEP). B. I.V. fluid bolus. C. check arterial blood gas. D. decrease the insufflation pressure. E. take the patient out of steep Trendelenburg.

B The key abnormalities in this evaluation are the two semen analyses demonstrating all normal parameters except for a markedly decreased volume. The history of hypertension is incomplete and should include medications since many anti-hypertensives interfere with bladder neck closure and ejaculation. TRUS is not required because the semen pH is normal implying that ejaculatory duct obstruction is unlikely. The next most useful test would be examination of a post-ejaculate urine specimen. The correct diagnosis can be made by finding large numbers of sperm (10-15/hpf) in the urine. Jarow JP, Sigman M, Kolettis PN, et al: The optimal evaluation of the infertile male: AUA BEST PRACTICE STATEMENT. American Urological Association Education and Research, Inc, 2010. <a href="http://www.auanet.org/education/guidelines/male-infertility-d.cfm" target="_new"><u>http://www.auanet.org/education/guidelines/male-infertility-d.cfm</u></a>

2015 - 65 A 27-year-old man on an alpha-blocker for hypertension undergoing an infertility evaluation has a normal physical examination. Two semen analyses demonstrate volumes of less than 1 ml, pH of 7.4, normal viscosity, and sperm counts in the range of 60 million/ml with 80% motility and 8% normal forms. The following test provides the most useful information: A. TRUS. B. postejaculatory urinalysis. C. serum testosterone. D. serum FSH and LH. E. serum prolactin.

E The incidence of urethral injuries following a pelvic facture are directly related to the number pubic rami-factures, the degree of the separation of the pubic symphysis, and the presence of diastasis of the sacroiliac joint. When any of these injuries are found to occur in conjunction with blood at the penile meatus or blood at the vaginal introitus, a urethral injury must be ruled-out. While a retrograde urethrogram is the preferred modality to assess urethral integrity in males, cystoscopy and vaginoscopy under general anesthesia is preferred in pre- and post-adolescent females. It is important to note that in this patient population concurrent rectal injuries must also be ruled-out. DRE once the mainstay to rule out a concurrent rectal injury has been found to have a large false negative rate and currently rectal endoscopy or other imaging modalities must also concurrently be performed to rule out a concomitant rectal injury. Husmann DA: Pediatric genitourinary trauma, Wein AJ, Kavoussi LR, Novick AC, Partin AW, Peters CA (eds): CAMPBELL-WALSH UROLOGY, ed 10. Philadelphia, Elsevier Saunders, 2012, vol 4, chap 138, p 3748.

2015 - 66 A 15-year-old girl involved in a car versus pedestrian accident sustains a fracture of her left ischiopubic ramus with diastasis of the sacroiliac joint. Urinalysis has 5-10 RBC's and there is blood at the introitus. CT scan with contrast shows no renal fracture. The next step is: A. observation. B. DRE. C. VCUG. D. flexible cystoscopy. E. cystoscopy and vaginoscopy.

D VCUG is recommended if there is evidence of renal scarring on ultrasound or if there is a history of UTI in the sibling who has not been tested. VCUG is recommended for children with high-grade (Society of Fetal Urology grade 3 and 4) hydronephrosis (not just pelviectasis), hydroureter or an abnormal bladder on ultrasound (late term prenatal or postnatal), or who develop a UTI on observation. VCUG is not routinely recommended for all siblings of a child with VUR, or for the offspring of a former patient with VUR. Peters CA, Skoog SJ, Arant BS Jr, et al: Management and screening of primary vesicoureteral reflux in children: AUA GUIDELINE. American Urological Association Education and Research, Inc, 2010. <a href="http://www.auanet.org/education/guidelines/vesicoureteral-reflux-a.cfm" target="_new"><u>http://www.auanet.org/education/guidelines/vesicoureteral-reflux-a.cfm</u></a>

2015 - 72 Regarding family members of a child with VUR, the 2012 AUA Reflux Guidelines recommend VCUG screening in: A. all siblings. B. all offspring. C. any non-toilet-trained sibling. D. any sibling with prenatal hydroureteronephrosis. E. any sibling with prenatal bilateral pelviectasis.

A Subcutaneous emphysema can develop intraoperatively from CO2 gas leakage around trocars and diffusion into the subcutaneous space. This is more common in cases where the trocar sites are made too large, lengthy cases, or use of high intra-abdominal insufflation pressures. Trocars should be directed toward the organ of interest so as to minimize forceful redirection of the trocar and instruments, resulting in enlargement of the trocar tract and gas leakage into the subcutaneous tissues. In obese patients, trocar length limits may result in the trocar pulling back into the subcutaneous tissues and causing gas leakage. In such cases, an extra-long trocar may be necessary. In assessing this patient's particular case, the first step is to ensure that all trocars are properly positioned within the intraperitoneal cavity, and that widening of the tracts has not occurred. Simply relocating the gas insufflation without ensuring proper trocar placement would not correct the problem. Although adjusting ventilator settings may help compensate for the hypercarbia, the initial problem has not been solved. Lastly, converting to an open operation just because of subcutaneous emphysema would be too extreme a measure. Eichel L, Clayman RV: Fundamentals of laparoscopic and robotic urologic surgery, Wein, AJ, Kavoussi LR, Novick AC, Partin AW, Peters CA (eds): CAMPBELL-WALSH UROLOGY, ed 10. Philadelphia, Elsevier Saunders, 2012, vol 1, chap 9, pp 236-237

2015 - 67 A morbidly obese woman has crepitus along the abdomen and thorax and mild hypercarbia four hours into a laparoscopic radical nephrectomy. The next step is: A. confirm trocars are in intraperitoneal location. B. relocate gas insufflation to a different trocar. C. increased respiratory rate. D. increase tidal volume. E. convert to open nephrectomy.

A While studies of empirical treatment of idiopathic oligospermia have shown improvement in sperm parameters, this occurs in a minority of men. Oral agents such as clomiphene citrate, as well as gonadotropins, have been used. Treatment with human chorionic gonadotropin does not generally achieve consistent improvement in sperm concentration, sperm motility, or pregnancy rate. Seminal volume does not increase in men with normal testosterone levels. Testosterone levels do rise because of the stimulation of testosterone production by Leydig cells. Some testosterone is converted to estradiol by the enzyme aromatase. Sabanegh E, Agarwal A: Male infertility, Wein AJ, Kavoussi LR, Novick AC, Partin AW, Peters CA (eds): CAMPBELL-WALSH UROLOGY, ed 10. Philadelphia, Elsevier Saunders, 2012, vol 1, chap 21, pp 639, 645.

2015 - 68 The treatment of idiopathic oligospermia with human chorionic gonadotropin is most likely to result in: A. elevation of serum testosterone. B. increased sperm concentration. C. improved sperm motility. D. increased pregnancy rate. E. increased seminal volume.

C In pregnancy, it has been proposed that the increase in cardiac output leads to increase in glomerular filtration rate (GFR) and renal plasma flow. GFR increases between 30-50% as full term approaches. This increase in GFR leads to a decrease in the serum BUN and creatinine. Therefore, the normal values for BUN and creatinine are lower in pregnant women than they are in non-pregnant women. Schaeffer AJ, Schaeffer EM: Infections of the urinary tract, Wein, AJ, Kavoussi LR, Novick AC, Partin AW, Peters CA (eds): CAMPBELL-WALSH UROLOGY, ed 10. Philadelphia, Elsevier Saunders, 2012, vol 1, chap 10, p 316.

2015 - 7 During the third trimester of pregnancy, the most common changes in renal function tests are: A. elevated BUN; decreased creatinine. B. elevated BUN; elevated creatinine. C. decreased BUN; decreased creatinine. D. decreased BUN; elevated creatinine. E. unchanged BUN and creatinine.

E Any issues of obstruction should be resolved before proceeding with AUS placement. Opening the bladder neck contracture with either dilation or resection and simultaneous AUS placement would not be recommended. It is unknown if the bladder neck contracture will remain open with either therapy, and this issue needs to be resolved prior to AUS placement, as treatment of a bladder neck contracture with an AUS in place can be very challenging. Steroid injection into the resection site is not indicated and would not alter the treatment strategy. This patient should have his bladder neck contracture opened and then undergo repeat cystoscopy in the office. If the bladder neck remains patent over time (at least three months) then AUS placement can proceed. Simultaneous sphincter placement in the setting of a less severe bladder neck contracture has been advocated. Wessells H, Peterson AC: Surgical procedures for sphincteric incontinence in the male: The artificial genitourinary sphincter and perineal sling procedures, Wein, AJ, Kavoussi LR, Novick AC, Partin AW, Peters CA (eds): CAMPBELL-WALSH UROLOGY, ed 10. Philadelphia, Elsevier Saunders, 2012, vol 3, chap 79, p 2293.

2015 - 70 While starting a planned placement of an artificial urinary sphincter (AUS), an 8 Fr bladder neck contracture is noted cystoscopically after a 14 Fr catheter is unable to be placed. The next step is: A. place an 8 Fr urethral catheter and proceed with AUS placement. B. dilate the bladder neck contracture, place a larger catheter and proceed with AUS placement. C. transurethrally resect the bladder neck, place a larger catheter and proceed with AUS placement. D. transurethrally resect the bladder neck, inject the resected bladder neck area with steroids and proceed with AUS placement. E. transurethrally resect the bladder neck and proceed with AUS placement three months later once assured the bladder neck remains open.

A The major advantages of fluoroscopy for stone location are: a short learning curve, a wide range of indications for in situ treatment, and multifunctional use of x-ray. The localizing problems for fluoroscopy consist chiefly of stones close to the vertebral column and radiolucent calculi. The advantages of ultrasound for stone location are: low purchase costs and maintenance, no x-ray exposures, the possibility of continuous real-time monitoring, and location of radiolucent calculi. Calculi in the middle ureter are almost impossible to localize with ultrasound. Multiple calculi may be problematic for ultrasonic stone localization. Matlaga BR, Lingeman JE: Surgical management of upper urinary tract calculi, Wein, AJ, Kavoussi LR, Novick AC, Partin AW, Peters CA (eds): CAMPBELL-WALSH UROLOGY, ed 10. Philadelphia, Elsevier Saunders, 2012, vol 2, chap 48, p 1390.

2015 - 71 The primary advantage of ultrasound for SWL stone localization is: A. capability for continuous real-time monitoring. B. short learning curve. C. multifunctional use for diagnosis and endourological treatment. D. ability to identify ureteral calculi. E. efficacy in clinical situations with multiple calculi.

B UTI in patients with spinal cord injury on CIC is commonly seen but can be a challenge to diagnose. Almost all urine collections will show bacteriuria, and pyuria may occur solely due to the irritative effects of catheterizations, and may not always be related to the presence of infection. The usual symptoms of UTI such as urinary frequency, urgency, and dysuria will not be noted in patients with a complete neurologic injury who have no bladder sensation. Typical UTI symptoms in a patient with a spinal cord injury may include urinary incontinence between catheterizations, increased spasticity (as seen in this patient), malaise, lethargy, persistent cloudy or malodorous urine, and discomfort at the level of the flank, back, or abdomen. Bacteria levels are problematic to interpret and are classically only treated if they are greater than or equal to 100 cfu/ml and the patient is symptomatic. Due to the symptomatic complaint of increased spasticity and positive urine culture, this patient should be given antibiotics. If the spasticity is not resolved after treatment, the patient should be carefully examined for any physical injury below the level of his lesion, such as obstipation, decubitus ulcer, ingrown toe nail, developing syrinx, etc. Baclofen is a commonly used treatment for spasticity in spinal cord injury (SCI) patients, and should be considered for use if the spasticity is not resolved after treatment of the UTI, and additional patient evaluation fails to reveal an underlying cause. Elevated storage pressures do place a patient at increased risk of symptomatic UTI, and urodynamics should be considered if this patient continues to experience recurrent symptomatic infections. Schaeffer AJ, Schaeffer EM: Infections of the urinary tract, Wein, AJ, Kavoussi LR, Novick AC, Partin AW, Peters CA (eds): CAMPBELL-WALSH UROLOGY, ed 10. Philadelphia, Elsevier Saunders, 2012, vol 1, chap 10, pp 322-324.

2015 - 73 A 24-year-old man with a T4 complete spinal cord injury who manages his bladder with CIC every six hours complains of increased lower extremity spasms during the past week. Urine culture reveals 100 cfu/ml E. coli. He denies urinary urgency or incontinence. The next step is: A. observation. B. antibiotics. C. antibiotics if pyuria present. D. baclofen. E. urodynamics.

C It must be decided whether this woman has recurrent E. coli infections or a persistent source of infection. One of the causes of surgically correctable bacterial persistence in the urinary tract is a unilateral infected atrophic kidney. To prove that this woman has bacterial persistence, however, upper tract bacterial localization cultures with bilateral ureteral catheterization need to be performed before nephrectomy is considered to treat her infections. If the kidney is not infected, these infections are the result of frequent E. coli reinfections, in which case a right nephrectomy would not be helpful in treating her. An indium-labeled WBC scan may not be helpful if the kidney is inflamed but not the source of recurrent E. coli infections. Schaeffer AJ, Schaeffer EM: Infections of the urinary tract, in Wein AJ, Kavoussi LR, Novick AC, Partin AW, Peters CA (eds): CAMPBELL'S UROLOGY, ed 10. Philadelphia, Saunders Elsevier, 2012, vol 1, chap 10, pp 290-291.

2015 - 74 A 22-year-old woman with recurrent febrile E. coli UTIs wants to become pregnant. CT scan shows a normal left kidney and an atrophic, scarred right kidney with less than 5% function on a DMSA scan. VCUG shows no reflux. The next step is: A. indium-labeled WBC scan. B. preventive antimicrobial therapy. C. bilateral ureteral catheterization with cultures. D. fluorescent bacterial antibody testing. E. right nephrectomy.

C Neoadjuvant chemotherapy is indicated: in children at risk for tumor recurrence (syndromes such as Beckwith-Wiedemann), in very large tumors or with tumor in the IVC above the hepatic veins making primary resection difficult, and in synchronous bilateral tumors. In bilateral tumors, nephron-sparing surgery is the goal, and repeat CT scan is indicated after chemotherapy to assess tumor shrinkage for timing of partial nephrectomies. Percutaneous or open renal mass biopsy will upstage the tumor and are not required for initiation of therapy. Ritchey ML, Shamberger RC: Pediatric urologic oncology, Wein, AJ, Kavoussi LR, Novick AC, Partin AW, Peters CA (eds): CAMPBELL-WALSH UROLOGY, ed 10. Philadelphia, Elsevier Saunders, 2012, vol 4, chap 137, pp 3721-3722.

2015 - 75 A three-year-old girl has a right 9 cm lower pole renal mass and a left 2 cm upper pole renal mass without lymphadenopathy on CT scan. The next step is: A. percutaneous renal mass biopsies. B. open renal mass biopsies. C. neoadjuvant chemotherapy and repeat CT scan at six weeks. D. right nephrectomy and left partial nephrectomy. E. bilateral partial nephrectomies.

E Clinical studies reveal that that up to half of the patients with the combination of LUTS and oliguria will have significantly improved symptoms after transplantation and the resolution of the oliguria, thereby rendering surgical treatment unnecessary. It is noteworthy that oliguric men undergoing surgical intervention for BPH have a significantly higher risk of developing either a bladder neck contracture, a prostatic fossa obliteration, or a urethral stricture if surgical treatment is performed before transplantation. To avoid these complications, it is recommended that treatment of bladder outlet obstruction in patients with oliguria be delayed until after the transplant is performed. If the physician elects to perform a TURP or TUIP in an oliguric or anuric patient the establishment of "normal voiding" cycles for a period of six weeks post-surgery has been shown to decrease the postoperative complications of urethral obstruction. To allow for "normal voiding", either a SP tube is placed or CIC is done, and water or saline is instilled into the bladder four to five times daily with the patient instructed to void per urethra. These maneuvers prevent the "dry urethra" and aid in prevention of bladder neck or obliterative urethral strictures. Finasteride would be indicated for a patient with a larger prostate with an elevation in residual urine. OnabotulinumtoxinA would be indicated for detrusor overactivity unresponsive to medical therapy with anticholinergics. Treatment of this patient now with onabotulinumtoxinA may improve his bladder capacity, but could lead to urinary retention and the need for CIC. Barry JM, Conlin MJ: Renal transplantation, Wein, AJ, Kavoussi LR, Novick AC, Partin AW, Peters CA (eds): CAMPBELL-WALSH UROLOGY, ed 10. Philadelphia, Elsevier Saunders, 2012, vol 2, chap 44, p 1231.

2015 - 76 A 60-year-old man being evaluated for renal transplantation has urinary frequency and decreased force of stream. He is on tamsulosin. DRE reveals a benign 30 gm prostate. Voiding diary reveals voided volumes of 50-75 ml/void and a total voided volume of 400 ml/24 hours. Maximum flow rate is 8 ml/second and Pdet Qmax is 80 cm H2O on pressure flow urodynamics. Postvoid residual urine is 0 ml. The next step is: A. finasteride. B. onabotulinumtoxinA. C. TUIP. D. TURP. E. evaluate bladder symptoms after transplant.

C Residual urine volumes have a large intraindividual variability, are not associated with renal or bladder damage, are not well-correlated with symptoms, and are not predictive of surgical outcome. In addition, an elevated PVR has not been associated with recurrent pyelonephritis. Therefore, the test is of limited clinical value and, in fact, the AUA Guidelines classify the test as optional in the initial diagnostic evaluation and subsequent assessment of men with bothersome LUTS. McVary KT, Roehrborn CG, Avins AL, et al: Management of benign prostatic hyperplasia (BPH): AUA GUIDELINE. American Urological Association Education and Research, Inc, 2010. <a href="http://www.auanet.org/education/guidelines/benign-prostatic-hyperplasia.cfm" target="_new"><u>http://www.auanet.org/education/guidelines/benign-prostatic-hyperplasia.cfm.</u></a> McNicholas TA, Kirby RS, Lepor H: Evaluation and nonsurgical management of benign prostatic hyperplasia, Wein, AJ, Kavoussi LR, Novick AC, Partin AW, Peters CA (eds): CAMPBELL-WALSH UROLOGY, ed 10. Philadelphia, Elsevier Saunders, 2012, vol 3, chap 92, p 2620.

2015 - 82 A 60-year-old man with LUTS and an AUA Symptom Score of 18 has a PVR of 200 ml. This residual volume is: A. associated with recurrent pyelonephritis. B. an indication for surgical therapy. C. of limited clinical utility. D. highly correlated with urinary symptoms. E. predictive of outcome after surgery.

D Improved patency and pregnancy after vasectomy reversal are correlated with a short time interval between vasectomy and reversal. It has been demonstrated that a proximal vas segment in excess of 2.7 cm predicts the presence of vasal fluid with whole sperm. It is generally believed that the presence of a granuloma and a smaller luminal diameter are favorable, as pressure below the level of the vasectomy may be less. Testicular specimens obtained after vasectomy reveal increased thickness of the seminiferous tubules, reduction in the number of Sertoli cells and spermatids, and an increase in the cross-sectional tubular area. The presence of interstitial fibrosis has an adverse effect on post-vasovasostomy fertility. The type of microscopic anastomosis (multi or one-layer) does not correlate with patency rates. Witt MA, Heron S, Lipshultz LI: The post-vasectomy length of the testicular vasal remnant: A predictor of surgical outcome in microscopic vasectomy reversal. J UROL 1994;151:892-894. Belker AM, Thomas AJ, Fuchs EF, et al: Results of 1,469 microsurgical vasectomy reversals by the vasovasostomy study group. J UROL 1991;145:505-511. Goldstein M: Surgical management of male infertility, Wein, AJ, Kavoussi LR, Novick AC, Partin AW, Peters CA (eds): CAMPBELL-WALSH UROLOGY, ed 10. Philadelphia, Elsevier Saunders, 2012, vol 1, chap 22, pp 656-659.

2015 - 77 The following correlates with improved patency rates after vasectomy reversal: A. absence of sperm granuloma. B. increased diameter of proximal vas. C. multi-layered anastomosis. D. length of the proximal vas. E. increase in cross-sectional tubular area of the testis.

B This child has balanoposthitis. In mild cases, simple removal of irritating agent (such as soaps and detergents) can lead to improvement, but this child has had significant symptoms for over 24 hours and is having difficulty voiding. The correct next step is topical antibiotic rather than observation. If the topical antibiotic is ineffective, then topical or oral antifungal agents may be tried next. Unlike with balanitis xerotica obliterans (BXO), topical testosterone is inappropriate. In cases of severe swelling, topical corticosteroid may be tried. Performance of a dorsal slit is too aggressive for this benign condition. Link RE: Cutaneous diseases of the external genitalia, Wein, AJ, Kavoussi LR, Novick AC, Partin AW, Peters CA (eds): CAMPBELL-WALSH UROLOGY, ed 10. Philadelphia, Elsevier Saunders, 2012, vol 1, chap 15, p 451.

2015 - 78 A three-year-old boy has a 1 cm area of erythema and swelling of the foreskin for 24 hours. He last voided with severe dysuria six hours ago. He has no fever. The next step is: A. observation. B. topical antibiotic. C. topical testosterone. D. oral antifungal. E. dorsal preputial slit.

E It is unlikely that a man with ED not using any erectogenic medications will develop ischemic priapism. Moreover, his ABG does not suggest ischemic, nor non-ischemic priapism. Cavernosal blood gas in men with ischemic priapism typically has a PO2 of < 30 mmHg, a PCO2 of > 60 mmHg, and a pH < 7.25. Cavernous blood gases in men with nonischemic priapism are similar to the blood gases of arterial blood. Normal flaccid penis cavernous blood gas levels are approximately equal to those in normal mixed venous blood. The findings in this patient, blood gas values equal to normal mixed venous blood are most consistent with priapism secondary to metastatic prostate cancer to the corpora and corpora biopsy is indicated. In view of the blood gas finding the use of oral terbutaline, sympathomimetic cavernosal injections or shunting would be inappropriate. Montague DK, Jarow J, Broderick GA, et al: Guideline on the management of priapism. American Urological Association Education and Research, Inc, 2003. <a href="http://www.auanet.org/education/guidelines/priapism.cfm" target="_new"><u>http://www.auanet.org/education/guidelines/priapism.cfm</u></a> Lin YH, Kim JJ, Stein NB, Khera M: Malignant priapism secondary to metastatic prostate cancer: a case report and review of the literature. REV UROL 2011;13:90-94.

2015 - 79 A 62-year-old man with a history of metastatic prostate cancer and erectile dysfunction develops priapism for the past five days. He denies using any erectogenic medications and complains of persistent penile pain. Corporal blood gas reveals pH 7.35, pCO2 50 mmHg, and pO2 40 mmHg. The next step is: A. oral terbutaline. B. sympathomimetic cavernosal injection. C. proximal bulbo-cavernosal shunt. D. selective embolization. E. corporal biopsy.

D Proteinuria of 1+ or greater on repetitive dipstick urinalyses should prompt a 24-hour collection to quantitate the degree of proteinuria. In the absence of significant bleeding, > 1 g/24 hour should then prompt a more extensive evaluation for renal parenchymal disease and possible nephrology referral. This patient in fact does not meet criteria of microhematuria because the number of RBCs/hpf is < 3 thus further hematuria evaluation is not warranted. Mild proteinuria would be unlikely to affect serum albumin levels. Davis R, Jones JS, Barocas DA, et al: Diagnosis, evaluation and follow-up of asymptomatic microhematuria (AMH) in adults: AUA GUIDELINE. American Urological Association Education and Research, Inc, 2012. <a href="http://www.auanet.org/education/asymptomatic-microhematuria.cfm" target="_new"><u>http://www.auanet.org/education/asymptomatic-microhematuria.cfm</u></a>

2015 - 8 A 28-year-old man has 1+ proteinuria and moderate blood on two dipstick analyses. Two microscopic urinalyses each reveals 0-2 RBC/hpf. According to the AUA Guidelines, the next step is: A. reassurance and no further evaluation. B. serum albumin level. C. urine cytology. D. 24-hour urine collection for protein. E. cystoscopy and upper tract imaging.

C Cushing's syndrome implies glucocorticoid excess, while Cushing's disease is glucocorticoid excess specifically due to a pituitary adenoma. Approximately 80% of patients with Cushing's syndrome have hypertension at the time of presentation. Glucocorticoids have weak mineralocorticoid effects resulting in retention of salt and water. With excessive glucocorticoid production these weak mineralocorticoid effects can cause hypertension. Plasma catecholamines are typically not elevated. Kutikov A, Crispen PL, Uzzo RG: Pathophysiology, evaluation, and medical management of adrenal disorders, Wein, AJ, Kavoussi LR, Novick AC, Partin AW, Peters CA (eds): CAMPBELL-WALSH UROLOGY, ed 10. Philadelphia, Elsevier Saunders, 2012, vol 2, chap 57, pp 1691-1694.

2015 - 80 Hypertension in Cushing's syndrome is primarily related to: A. elevated plasma catecholamines. B. elevated plasma aldosterone. C. retention of water and salt. D. ACTH-stimulated renin. E. elevated angiotensin II.

D Patients who undergo bladder augmentation with bowel should be counseled on the possible long-term risk of carcinoma formation, renal and bladder calculi and metabolic abnormalities. The earliest report of tumor formation is four years after bladder augmentation with bowel. Yearly cystoscopic surveillance had been recommended in the past, though the timing of when to start the surveillance was not well-defined. Recent studies have shown that routine yearly endoscopy is not indicated due to the low incidence of malignancy following a bladder augmentation (approximately 1.5-2.5% of patients per decade), lack of proven benefit, and high cost. In the absence of other risk factors, the current recommendation is for annual visits with renal and bladder ultrasound (rule-out stones or the development of hydronephrosis secondary to noncompliance with CIC), electrolytes (rule out metabolic abnormalities), creatinine, serum B12 (rule-out nutritional deficiencies), and urinalysis (assess for hematuria). Endoscopy is reserved for individuals with a past medical history of gross hematuria, microscopic hematuria (> 50 RBC/hpf), new onset of hydronephrosis (rule-out tumor obstructing the ureteral orifice), bladder calculi, chronic bladder/perineal pain or a history of four or more symptomatic UTI per year. Using this screening criteria, > 90% of tumors arising in a bladder augment can be discovered without the use of annual endoscopy. Higuchi TT, Fox JA, Husmann DA. Annual endoscopy and urine cytology for the surveillance of bladder tumors after enterocystoplasty for congenital bladder anomalies. J UROL 2011;186:1791-1795.

2015 - 81 Monitoring for adenocarcinoma of the bladder after a sigmoid bladder augmentation cystoplasty is best performed by yearly: A. urine cytology. B. urine fluorescence in-situ hybridization (FISH) analysis. C. serum carcinoembryonic antigen (CEA) level. D. renal and bladder ultrasound. E. cystoscopy.

A Prostatic granulomas are recognized with increasing frequency in men following intravesical BCG therapy. Although long-term outcome is uncertain, the granulomas are generally asymptomatic and no therapy is recommended. This patient had a prior normal digital rectal exam of the prostate and his serum PSA is normal. Thus, it is unlikely he has prostate cancer and further biopsies are no more indicated in him than in any other age matched male patient. Beltrami P, Ruggera L, Cazzoletti L, et al: Are prostate biopsies mandatory in patients with prostate-specific antigen increase during intravesical immuno- or chemotherapy for superficial bladder cancer? PROSTATE 2008;68:1241-1247. Tareen B, Taneja SS: Complications of intravesical therapy, in Taneja SS (ed): COMPLICATIONS OF UROLOGIC SURGERY, ed 4. Philadelphia, Elsevier Saunders, 2010, chap 8, pp 97-98.

2015 - 83 A 68-year-old asymptomatic man receives two six-week courses of intravesical BCG for recurrent bladder tumors. A new firm area in the prostate is noted on DRE. Serum PSA is 3.0 ng/ml. Ultrasound-directed needle biopsy of the lesion reveals a caseating granuloma. The next step is: A. observation. B. repeat prostate biopsies in three months. C. cycloserine for six months. D. isoniazid for six months. E. isoniazid and rifampin for six months.

E The horseshoe kidney involves anteriorly oriented renal pelves and a low-lying kidney mass with the isthmus sitting just below the inferior mesenteric artery takeoff from the aorta. This makes a transperitoneal, laparoscopic approach to this large stone very attractive. With moderate hydronephrosis, a simple transmesenteric exposure of the renal pelvis followed by pyelotomy can allow intact removal of the large stone. Simple lap suturing can close the pyelotomy incision. The option of percutaneous lithotripsy is appropriate to consider; however, access should be through the upper pole because of calyceal anatomy. The lower pole of a horseshoe kidney will be too medial and anterior for optimal access. Ureteroscopic treatment using a holmium laser is not a good option with a stone of this size as the duration of procedure and failure rate will both be high. ESWL will also carry a high failure rate due to stone size. A double J stent does not offer an advantage in treating the stone in this scenario. Ost MC, Schneck FX: Surgical management of pediatric stone disease, Wein AJ, Kavoussi LR, Novick AC, Partin AW, Peters CA (eds): CAMPBELL-WALSH UROLOGY, ed 10. Philadelphia, Elsevier Saunders, 2012, vol 4, chap 135, p 3682.

2015 - 84 A five-year-old girl with a horseshoe kidney has moderate left hydronephrosis and a 2.1 cm renal pelvis stone. The next step is: A. ureteral stent. B. SWL. C. ureteroscopic laser lithotripsy. D. PCNL via lower pole access. E. laparoscopic pyelolithotomy.

A This patient is likely experiencing worsening of symptoms due to incomplete bladder emptying. Approximately 5% of patients with idiopathic overactive bladder will have issues of incomplete bladder emptying/urinary retention requiring CIC after injection of 100 units of onabotulinumtoxinA. Determination of her PVR would allow for proper evaluation for incomplete bladder emptying and guide the initiation of CIC as needed. Mirabegron is a beta-3-adrenergic receptor agonist and is indicated for OAB; this would only be considered once the possibility of incomplete bladder emptying has been ruled-out. Urodynamics is unnecessary to rule out urinary retention. However, if this patient is emptying adequately, urodynamics may then be considered for diagnostic purposes. If a patient requires reinjection, this should be done at least three months after the initial injection. In addition, studies suggest that the 100 unit dose is adequate for OAB, and a higher dose is not beneficial and maybe associated with an increased incidence of urinary retention. Andersson KE, Wein AJ: Pharmacologic management of lower urinary tract storage and emptying failure, Wein AJ, Kavoussi LR, Novick AC, Partin AW, Peters CA (eds): CAMPBELL-WALSH UROLOGY, ed 10. Philadelphia, Elsevier Saunders, 2012, vol 3, chap 68, p 1987. Nitti VW, Dmochowski R, Herschorn S, et al: OnabotulinumtoxinA for the treatment of patients with overactive bladder and urinary incontinence: results of a phase 3, randomized, placebo controlled trial. J UROL 2013:189:2186-2193.

2015 - 85 A 54-year-old woman undergoes intradetrusor injection of 100 units of onabotulinumtoxinA for symptoms of urgency, frequency, and urgency urinary incontinence. Two weeks later, she complains of worsening frequency, urgency, and urinary incontinence. The next step is urinalysis and: A. PVR. B. urodynamics. C. mirabegron. D. immediate reinjection of an additional 100 units of onabotulinumtoxinA. E. reinject an additional 100 units of onabotulinumtoxinA in three months.

C Chemotherapy-naive relapses occur in men with clinical stage (CS) I NSGCT managed with either surveillance or RPLND, and in the men with CS IIA-B NSGCT treated with RPLND alone. In general these patients are treated with induction chemotherapy, with the specific regimen and duration of therapy determined by risk, and cure rates exceed 95%. This patient has an abdominal relapse after observation. Correct management is three cycles of BEP for disseminated germ cell tumor in good risk patients. Biopsy of the mass could miss elements of residual cancer and should not be performed. XRT is not indicated for mixed germ cell tumors of the testis. Retroperitoneal lymphadenectomy is usually not performed prior to chemotherapy for masses > 3 cm on CT scan, avoiding double therapy in these patients. Stephenson AJ, Gilligan TD: Neoplasms of the testis, Wein, AJ, Kavoussi LR, Novick AC, Partin AW, Peters CA (eds): CAMPBELL-WALSH UROLOGY, ed 10. Philadelphia, Elsevier Saunders, 2012, vol 1, chap 31, p 859.

2015 - 86 A 21-year-old man had a right inguinal orchiectomy for a clinical Stage I mixed germ cell tumor. He was observed and seven months later, a 4 cm mass is seen on the abdominal CT scan in the interaorto-caval region. The chest x-ray, beta-hCG, and AFP are normal. The next step is: A. retroperitoneal XRT. B. percutaneous biopsy of retroperitoneal mass. C. cisplatin-based chemotherapy. D. right modified template RPLND. E. full bilateral RPLND.

E Persistent or recurrent LUTS may occur after TURP. Since < 20% of these men have any evidence of recurrent or persistent bladder outlet obstruction, assessment with pressure-flow studies are particularly useful to make determinations regarding the appropriateness of further surgical intervention. Many of these patients' symptoms are due to poor bladder contractility or detrusor overactivity. Uroflowmetry alone may be used for screening purposes, although it may be misinterpreted in the presence of high-flow, high-pressure voiding. Cystoscopy will rule-out a stricture or bladder neck contracture but the presence of visually obstructing prostatic tissue does not correlate with bladder outlet obstruction. Neither residual urine nor creatinine (even if abnormal) sheds light on the etiology of the emptying disorder. McNicholas TA, Kirby RS, Lepor H: Evaluation and nonsurgical management of benign prostatic hyperplasia, Wein, AJ, Kavoussi LR, Novick AC, Partin AW, Peters CA (eds): CAMPBELL-WALSH UROLOGY, ed 10. Philadelphia, Elsevier Saunders, 2012, vol 3, chap 92, p 2618.

2015 - 88 An 85-year-old man has bothersome LUTS. He underwent TURP 12 years ago. Urinalysis and urine cytology are negative. The most important test before considering repeat TURP is: A. serum creatinine. B. residual urine. C. uroflowmetry. D. cystoscopy. E. pressure-flow study.

C Cystine stones occur in patients homozygous for a complex autosomal recessive disorder of amino acid transport involving cystine, ornithine, lysine, and arginine. Onset is usually in the first or second decades. Cystine crystals are hexagonal and cystine stones are very dense making them difficult to treat by lithotripsy. Medical therapy, by increasing the solubility of cystine, is the mainstay of treatment. This disorder is noted to be of increased incidence in the Japanese population. Ferrandino MN, Pietrow PK, Preminger GM: Evaluation and medical management of urinary lithiasis, Wein, AJ, Kavoussi LR, Novick AC, Partin AW, Peters CA (eds): CAMPBELL-WALSH UROLOGY, ed 10. Philadelphia, Elsevier Saunders, 2012, vol 2, chap 46, pp 1290-1291.

2015 - 99 A nine-year-old girl who recently emigrated from Japan has recurrent right flank pain and urolithiasis with SWL being unsuccessful in the past. Renal ultrasound shows two right 8 mm renal calculi and moderate hydronephrosis. Urinalysis shows hexagonal-shaped crystals. The most likely cause of her urolithiasis is: A. hypercalciuria. B. hyperuricosuria. C. cystinuria. D. low urinary volume. E. UPJ obstruction.

E It was previously believed that scrotal surgery markedly increased the risk of local recurrence and inguinal node metastases. Further evidence now suggests that this risk has been overestimated and a formal hemiscrotectomy or prophylactic inguinal lymph node dissection is rarely indicated. A meta-analysis of all evaluable reported series (1182 total cases, 206 with scrotal violation) found that the risk of local recurrence increased from 0.4% to 2.9% with scrotal violation, but there was no difference in the distant recurrence or survival rates. In patients with low-stage NSGCT, the scrotal scar should be widely excised with the spermatic cord remnant at the time of RPLND. The inguinal nodes should be preserved. Sheinfeld J, Bosl GJ: Surgery of testicular tumors, Wein, AJ, Kavoussi LR, Novick AC, Partin AW, Peters CA (eds): CAMPBELL-WALSH UROLOGY, ed 10. Philadelphia, Elsevier Saunders, 2012, vol 1, chap 32, pp 872-873.

2015 - 89 A 23-year-old man is referred for treatment after a left trans-scrotal orchiectomy. Pathology shows an embryonal cell carcinoma mixed with elements of seminoma. AFP, beta-hCG, and chest x-ray are normal. No inguinal lymph nodes are palpable. In addition to excision of the left spermatic cord, treatment should include: A. hemiscrotectomy. B. left superficial inguinal lymph node dissection. C. biopsy of left sentinel inguinal node. D. radiation of left inguinal nodes. E. observation of inguinal nodes.

D The overall prevalence of RCC in patients with ESRD is 1%. This risk is increased three-four fold in individuals with acquired renal cystic disease of dialysis (ARCD). The onset of ARCD is directly related to the severity of azotemia and the length of time the individual has been on dialysis. RCC in patients with ESRD generally occurs within ten years of the initiation of dialysis. They are multicentric, bilateral, less aggressive than sporadic RCC, and have a male predominance. Both hemodialysis and peritoneal dialysis have been associated with an equivalent incidence of ARCD, and there is no evidence that conversion from one form of dialysis to another influences this disease. For this reason, periodic ultrasound is recommended every six months for patients on chronic dialysis for > 3 years. In this patient population it is appropriate to consider CT, MRI scan, or proceed directly to surgical intervention when the ultrasound suggests a complex cyst or a solid mass > 3 cm. Pope JC IV: Renal dysgenesis and cystic disease of the kidney, Wein, AJ, Kavoussi LR, Novick AC, Partin AW, Peters CA (eds): CAMPBELL-WALSH UROLOGY, ed 10. Philadelphia, Elsevier Saunders, 2012, vol 4, chap 118, pp 3192-3194.

2015 - 9 A 68-year-old man with ESRD has been on peritoneal dialysis for four years. He is anuric and asymptomatic. Ultrasound reveals several non-echogenic cysts involving the left kidney. The next step is: A. left nephrectomy. B. CT scan. C. renal arteriography. D. repeat ultrasound in six months. E. conversion to hemodialysis.

C According to the American Society of Anesthesia guidelines, NPO after midnight is no longer advisable or safe for children. Recommendations for fasting prior to anesthesia include: clear liquids - 2 hours, breast milk - 4 hours, formula or solid food - 6 hours, and fatty foods - 8 hours. Estrada CR Jr, Ferrari LR: Core principles of perioperative management in children, Wein AJ, Kavoussi LR, Novick AC, Partin AW, Peters CA (eds): CAMPBELL-WALSH UROLOGY, ed 10. Philadelphia, Elsevier Saunders, 2012, vol 4, chap 119, p 3202.

2015 - 90 A six-month-old boy is scheduled for elective hypospadias repair. Instructions given to his parents regarding his NPO pre-op fluid status include: A. NPO after midnight. B. clear fluids until one hour prior to anesthesia. C. breast milk until four hours prior to anesthesia. D. formula until four hours prior to anesthesia. E. any fluids until four hours prior to anesthesia.

E Spinal shock is defined as the loss of motor, sensory, reflex, and autonomic neurologic function below the level of spinal cord injury. It is a temporary physiologic disorganization of spinal cord that classically starts within one hour after the neurologic injury. Resolution of spinal cord shock classically begins with the initial return of the bulbocavernosus reflex followed by the eventual restoration of the deep tendon reflexes (DTR) below the level of spinal cord injury. Resolution of spinal cord shock may be defined by either the recurrence of the bulbocavernous reflex or by return of the DTR, the latter of which usually occurs at four to six weeks post injury. Return of the DTR, will be followed by a progressive increase in muscle spasticity and the development of detrusor over activity. The severity of muscle spasticity and/or detrusor dysfunction increases during the first year post injury due to a combination of neuronal sprouting, done by the neurons in an attempt to make neural connections below the spinal cord injury, combined with the up regulation of neural receptors within the target organs. The full extent of the neurogenic bladder dysfunction may take one to two years to become completely manifested. Significant surgical intervention, for example, formation of continent catheterizable stoma or determining if the patient would need a bladder augmentation should therefore be delayed for at least one year following injury to allow the severity of the neurourologic injury to be revealed. The timing for the initial urodynamic study should not arbitrarily be based on the time since the injury, but rather on the clinical findings of the patient. Specifically, the return of DTR associated with the strength of these reflexes following minimal stimulation should dictate when to perform the initial urodynamic evaluation. Since the initial in-patient rehabilitation stay is usually no more than two to three weeks, a study at this time only reflect the finding of detrusor underactivity due to the manifestations of spinal cord shock. The initial findings of urinary incontinence are often a consequence of a UTI due to the initiation of CIC and are not necessarily an indication to perform the initial urodynamic evaluation. On the other hand, new onset of urinary incontinence in a patient with neurogenic bladder who has been stable on their bladder management for greater than six months and is not actively infected would be a reason to evaluate the patient with urodynamic studies. Wein AJ, Dmochowski RR: Neuromuscular dysfunction of the lower urinary tract, Wein AJ, Kavoussi LR, Novick AC, Partin AW, Peters CA (eds): CAMPBELL-WALSH UROLOGY, ed 10. Philadelphia, Elsevier Saunders, 2012, vol 3, chap 65, p 1920.

2015 - 91 The best time to perform urodynamics on a patient after a complete T4 spinal cord injury is: A. during the initial in-patient rehabilitation stay. B. at the initial sign of urinary incontinence. C. after the patient has learned to perform CIC. D. three months after the initial injury. E. after the return of deep tendon reflexes.

C Patients with a NSGCT who have a > 90% reduction in the size of the retroperitoneal mass with chemotherapy and have no teratomatous elements in their tumor (such as this patient who had a pure embryonal tumor) uncommonly have either viable cancer or teratoma in the residual mass at the time of retroperitoneal lymph node resection. In an early study, Donohue and colleagues (1987) reported that none of 15 patients without teratoma in the primary tumor and who achieved a 90% or greater reduction in the size of the residual mass with chemotherapy had any evidence of viable malignancy or teratoma at postchemotherapy surgery. In contrast, seven of nine patients (78%) with teratoma in the primary tumor experiencing a similar reduction in the size of the metastasis with chemotherapy had evidence of viable malignancy and/or teratoma. Predictors of necrosis in post-chemotherapy surgery specimens include the absence of teratoma in the primary tumor, significant percentage reduction (e.g. 90%) in the retroperitoneal mass with chemotherapy, and the size of the residual mass. However, despite statistical modeling using these and other factors, a consistent false-negative rate for necrosis (of up to 20% in some studies) has been reported, largely due to the presence of teratoma. Thus, the presence of necrosis only in the retroperitoneum cannot be predicted with sufficient accuracy to safely obviate the need for surgery in patients with residual masses. Normalization of the serum markers does not predict the presence of fibrosis. Stephenson AJ, Gilligan TD: Neoplasms of the testis, Wein, AJ, Kavoussi LR, Novick AC, Partin AW, Peters CA (eds): CAMPBELL-WALSH UROLOGY, ed 10. Philadelphia, Elsevier Saunders, 2012, vol 1, chap 31, pp 857-858. Donohue JP, Rowland RB, Kopecky K, Steidle CP, et al: Correlation of computerized tomographic changes and histological findings in 80 patients having radical retroperitoneal lymph node dissection after chemotherapy for testis cancer. J UROL 1987;137:1176-1179.

2015 - 92 A 35-year-old man has persistent retroperitoneal lymphadenopathy after cisplatin-based chemotherapy for NSGCT. The parameter most predictive of finding only fibrosis in the retroperitoneum is: A. normalization of serum hCG. B. 50% reduction in size of the mass on CT scan. C. pure embryonal cell carcinoma in the primary tumor. D. normalization of serum alpha-fetoprotein. E. teratoma in the primary tumor.

E Patients with a concentrating defect due to obstruction such as PUV may present with worsening incontinence along with upper urinary tract deterioration due to excessive urine production. The urine volume will not decrease significantly with either salt or water restriction. Furthermore, water restriction is dangerous and often counterproductive, as it may lead to dehydration and worsening renal function. Patients with a renal concentrating defect typically do not respond to DDAVP. Unless there is evidence of myogenic failure and incomplete bladder emptying, daytime CIC to further eliminate post void residual is unlikely to help this patient. It has been suggested that continuous nighttime drainage can improve the fluid dynamics, thus restoring the upper urinary tract (decreasing bilateral hydroureteronephrosis), as well as improving daytime urinary incontinence. Casale AJ: Posterior urethral valves, Wein, AJ, Kavoussi LR, Novick AC, Partin AW, Peters CA (eds): CAMPBELL-WALSH UROLOGY, ed 10. Philadelphia, Elsevier Saunders, 2012, vol 4, chap 126, p 3402.

2015 - 93 A six-year-old boy with the history of a neonatally ablated PUV has worsening bilateral hydroureteronephrosis. He is incontinent at night and occasionally wet during the day. A 24-hour urine collection shows a urine volume of two liters. Videourodynamics shows no detrusor overactivity, no residual valves, and no VUR. Along with more frequent daytime voiding, the next step is: A. decreased fluid intake. B. dietary salt restriction. C. DDAVP. D. CIC. E. continuous night time catheterization.

C Preoperative bladder volume may be the most important predictor of success. If a significant increase in bladder volume capacity is needed, autoaugmentation is not likely to be as useful as other reconstructive techniques. Autoaugmentation usually increases bladder compliance, and operative time is shorter than for bowel augments without a high risk of perforation. Studies have shown no increased complication rate from subsequent enterocystoplasty, if needed. Adams MC, Joseph DB: Urinary tract reconstruction in children, Wein, AJ, Kavoussi LR, Novick AC, Partin AW, Peters CA (eds): CAMPBELL-WALSH UROLOGY, ed 10. Philadelphia, Elsevier Saunders, 2012, vol 4, chap 129, pp 3488-3489.

2015 - 94 The main disadvantage of bladder autoaugmentation is: A. increased operative time. B. decreased bladder compliance. C. limited increase in bladder capacity. D. increased risk of perforation. E. increased complication rate of subsequent enterocystoplasty.

D The patient has a Stage II penile cancer with invasion of the corpora that is associated with a much higher incidence of positive lymph nodes. Although the lymph node has decreased in size, it is still palpable after six weeks and deserves excision. Since this patient is at high risk for nodal disease, neither a negative needle aspiration nor a negative sentinel node biopsy should dissuade one from lymphadenectomy. Among patients found to have unilateral positive groin nodes, a bilateral lymphadenectomy is indicated due to the high rate of bilateral disease. By comparison, patients who present with unilateral adenopathy beyond one year are treated with ipsilateral lymphadenectomy. Pettaway CA, Lance RS, Davis JW: Tumors of the penis, Wein, AJ, Kavoussi LR, Novick AC, Partin AW, Peters CA (eds): CAMPBELL-WALSH UROLOGY, ed 10. Philadelphia, Elsevier Saunders, 2012, vol 1, chap 34, pp 912-922.

2015 - 95 A 60-year-old man with squamous cell carcinoma of the penis invading the right corpus cavernosum undergoes partial penectomy. After six weeks of cephalexin, a 3.5 cm right inguinal lymph node has decreased in size to 2.0 cm. Pelvic CT scan is normal. The next step is: A. reevaluation in three months. B. needle aspiration of the suspicious node. C. sentinel node biopsy. D. bilateral inguinal node dissection. E. right inguinal node dissection.

D Fowler's syndrome was first described in 1985. It is a cause of urinary retention in young women that is associated with abnormally increased EMG activity that results in impaired external sphincter relaxation. No neurologic or anatomic abnormality is associated with the condition, though it has been noted that women with Fowler's syndrome often have polycystic ovaries raising the possibility of a focal hormonal role. That said, neither hormone therapy nor onabotulinumtoxinA injection or alpha-blockade have been found to be successful therapies. Interestingly, however, the condition has been found by several groups to be highly responsive to neuromodulation. Wein AJ, Dmochowski RR: Neuromuscular dysfunction of the lower urinary tract, Wein, AJ, Kavoussi LR, Novick AC, Partin AW, Peters CA (eds): CAMPBELL-WALSH UROLOGY, ed 10. Philadelphia, Elsevier Saunders, 2012, vol 3, chap 65, p 1940.

2015 - 96 A 25-year-old woman has a chronic history of intermittent urinary stream and a lower abdominal discomfort without significant urinary urgency. Physical exam is normal and PVR is 1 liter. MRI scan of the brain and spine is normal, as is cystoscopy. Pressure flow analysis shows an active EMG during voiding. She has failed treatment with biofeedback and prefers not to do CIC. The next step is: A. vaginal estrogen therapy. B. alpha-blocker therapy. C. onabotulinumtoxinA of the sphincter. D. sacral neuromodulation. E. sphincterotomy.

B The deep dorsal vein is occasionally ligated, dissected, and excised during a dorsal penile plication and the plication sutures are then placed in the venous bed. Thus, there is no need to salvage, convert, or abort the procedure. Excision and grafting is becoming less favorable due to the possible development of erectile dysfunction as a result of veno-occlusive dysfunction. Jordan GH, McCammon KA: Peyronie's disease, Wein, AJ, Kavoussi LR, Novick AC, Partin AW, Peters CA (eds): CAMPBELL-WALSH UROLOGY, ed 10. Philadelphia, Elsevier Saunders, 2012, vol 1, chap 28, p 804.

2015 - 97 During a penile plication for a 50 degree ventral penile curvature with palpable plaque, the deep dorsal vein is inadvertently transected. The next step is: A. abort procedure and reattempt three months later. B. ligate deep dorsal vein and continue with plication. C. primary repair of deep dorsal vein and continue with plication. D. anastomose deep dorsal vein to inferior epigastric vein and continue with plication. E. convert to excision and grafting procedure.

E Cycloserine inhibits BCG growth within 24 hours. The other drugs listed require two to seven days to inhibit BCG growth. Because of its relatively rapid action, cycloserine can be lifesaving in patients with BCG sepsis. Isoniazid, rifampin, and ethambutol all have slower onset of action and are less useful in the immediate management of life threatening BCG sepsis. Wood DP: Urothelial tumors of the bladder, Wein AJ, Kavoussi LR, Novick AC, Partin AW, Peters CA (eds): CAMPBELL-WALSH UROLOGY, ed 10. Philadelphia, Elsevier Saunders, 2012, vol 3, chap 80, p 2345.

2015 - 98 The drug with the most rapid onset of action in treating BCG sepsis is: A. isoniazid. B. rifampin. C. ethambutol. D. para-aminosalicylic acid. E. cycloserine.

B ( respiratory alkalosis. Respiratory alkalosis is a very early sign of septic shock and is caused by the initial tachypnea stimulated by the sepsis. As hypoperfusion occurs, metabolic acidosis develops. Tachycardia, oliguria, increased cardiac output, and increased plasma norepinephrine occur in early septic shock.Schaeffer AJ, Schaeffer EM: Infections of the urinary tract, Wein AJ, Kavoussi LR, Novick AC, Partin AW, Peters CA (eds): CAMPBELL-WALSH UROLOGY, ed 10. Philadelphia, Elsevier Saunders, 2012, vol 1, chap 10, p 314. 2013 General Infection & Inflammatory Disease )

An early manifestation of septic shock is: A) bradycardia. B) respiratory alkalosis. C) high output renal failure. D) decreased cardiac output. E) decrease in plasma norepinephrine.

E ( spermicide use. Studies of risk factors for recurrent UTI demonstrate that spermicide use increases urinary tract infections by decreasing normal vaginal flora and decreasing vaginal pH. Several studies have demonstrated the other four choices are not common risk factors. Toileting after intercourse, increasing fluid intake and wiping front to back have not been shown to decrease occurrence of UTIs in adult women. Schaeffer AJ, Schaeffer EM: Infections of the urinary tract, Wein, AJ, Kavoussi LR, Novick AC, Partin AW, Peters CA (eds): CAMPBELL-WALSH UROLOGY, ed 10. Philadelphia, Elsevier Saunders, 2012, vol 1, chap 10, p 267. 2013 Adult Infection & Inflammatory Disease )

A 25-year-old woman reports eight afebrile UTIs in the past year. The factor most likely to increase her risk of UTI is: A) tampon use. B) parity. C) douche use. D) daily bicycle riding. E) spermicide use.

B ( corticosteroids. This patient has eosinophilic granulomatous cystitis. It usually presents as a mass-like lesion with irritative symptoms and hematuria. It may be mistaken for a neoplastic process, but is benign and self-limited. If the lesion is focal with minimal symptomatology, observation is appropriate. Patients with focal lesions may also be managed with laser fulguration or TUR. In diffuse lesions, resection is not indicated. Both antihistamines and steroids may aid in relieving symptomatology. Steroids have been reported to relieve symptoms faster although direct statistical comparisons are not available. Unless infection is also present, there is no need to treat with antibiotics. The process is non-specific and often idiopathic, although consideration for a causative condition is necessary; e.g., schistosomiasis in a patient at risk. Frimberger DC, Kropp BP: Bladder anomalies in children, Wein AJ, Kavoussi LR, Novick AC, Partin AW, Peters CA (eds): CAMPBELL-WALSH UROLOGY, ed 10. Philadelphia, Elsevier Saunders, 2012, vol 4, chap 125, p 3387. 2013 Pediatric Infection & Inflammatory Disease )

A 12-year-old boy has severe dysuria and hematuria. Ultrasound shows a bladder mass and mild right hydronephrosis. Urine culture is sterile. Cystoscopy shows a diffuse, erythematous, bullous mass at the bladder base. Biopsy shows intense inflammation, granulomatous reactions and an eosinophilic infiltrate. The best management is: A) laser fulguration. B) corticosteroids. C) long-term antibiotics. D) TUR of lesion. E) cystectomy and diversion.

D ( ipsilateral RPLND. There is a significant rate of false negative imaging of the retroperitoneum on CT scan and microscopic disease may be present in patients with paratesticular rhabdomyosarcoma. Boys ten years of age or younger can be treated with chemotherapy alone. Boys over ten years of age should undergo ipsilateral RPLND since 50% will have microscopic disease. Bilateral RPLND is more morbid and does not improve survival over unilateral surgery. If the lymph nodes are positive for metastasis, the patient will require radiation therapy to the retroperitoneum in addition to chemotherapy. Ritchey ML, Shamberger RC: Pediatric urologic oncology, Wein AJ, Kavoussi LR, Novick AC, Partin AW, Peters CA (eds): CAMPBELL-WALSH UROLOGY, ed 10. Philadelphia, Elsevier Saunders, 2012, vol 4, chap 137, p 3709. 2013 Pediatric Neoplasm )

A 14-year-old boy has a painless mass above the right testis. Radical orchiectomy is performed. Pathology reveals a completely resected paratesticular rhabdomyosarcoma. His abdominal CT scan is normal. The next step is: A) repeat CT scan in three months. B) adjuvant chemotherapy with doxorubicin and alkylating agents. C) retroperitoneal radiation. D) ipsilateral RPLND. E) bilateral RPLND.

D ( percutaneous nephrostomy drainage. An intussuscepted ureteral segment noted after ureteroscopic stone extraction is a dramatic complication. Retrograde realignment for such a large segment is unrealistic. Percutaneous nephrostomy drainage preserves renal function and minimizes urinary extravasation. Nephrectomy or ileal ureteral replacement may be appropriate actions only after a discussion with the patient and their family. Transuretero-ureterostomy reconstruction places both kidneys at risk especially with a history of urinary stone disease.Matlaga BR, Lingeman JE: Surgical management of upper urinary tract calculi, Wein AJ, Kavoussi LR, Novick AC, Partin AW, Peters CA (eds): CAMPBELL-WALSH UROLOGY, ed 10. Philadelphia, Elsevier Saunders, 2012, vol 2, chap 48, p 1407. 2013 Adult Trauma & Fistulae )

A 16 cm intussuscepted ureteral segment is noted in the bladder after a difficult ureteroscopic stone extraction. The next step is: A) nephrectomy. B) transuretero-ureterostomy. C) ureteral reimplantation. D) percutaneous nephrostomy drainage. E) primary uretero-ureterostomy.

C ( overnight fluid restriction. This clinical scenario suggests diabetes insipidus. Infants should undergo overnight fluid restriction in a hospital setting. Body weight and vital signs are monitored until three percent of body weight is lost or the urine osmolality is > 600 mOsm/kg. If this does not produce a concentrated urine, fluid is restricted on another day and desmopressin is administered. If the renal response to desmopressin is normal, central DI (head CT scan) should be investigated. Salt restriction is not helpful in patients with diabetes insipidus as the primary defect involves free water handling. Renal ultrasound yields no useful information for acute management in this scenario. Shoskes DA, McMahon AW: Renal physiology and pathophysiology, Wein AJ, Kavoussi LR, Novick AC, Partin AW, Peters CA (eds): CAMPBELL-WALSH UROLOGY, ed 10. Philadelphia, Elsevier Saunders, 2012, vol 2, chap 38, pp 1040-1041. 2013 Pediatric Fluid & Electrolyte,Transplantation, Hypertension, Vasc Disease, Nephrology )

A 16-month-old boy is treated for dehydration. He has had no diarrhea or vomiting. He has been drinking excessively over the past several months. The serum sodium is 152 mEq/l. The urine specific gravity is 1.003 and the urine osmolality is 500 mOsm/kg. He is treated with parenteral fluids. The next step is: A) head CT scan. B) renal ultrasound. C) overnight fluid restriction. D) desmopressin (DDAVP). E) salt-restricted diet.

D ( remove urethral catheter and insert suprapubic tube. Multiple debridements of necrotic skin followed by skin graft coverage may eventually be needed with burn injuries to the genitalia. Early suprapubic urinary diversion simplifies wound care and prevents complications related to prolonged urethral catheterization. If a urethral catheter is used in a genitalia burn, it should be removed after 72 hours to prevent urethral slough and fistula formation. Hyperbaric oxygen therapy has been used to promote overall wound healing however there is no evidence that it prevents urethral complications. Split or full thickness skin grafting is used once granulation tissue is present and all non-viable tissue has been removed. Morey AF, Dugi DD III: Genital and lower urinary tract trauma, Wein AJ, Kavoussi LR, Novick AC, Partin AW, Peters CA (eds): CAMPBELL-WALSH UROLOGY, ed 10. Philadelphia, Elsevier Saunders, 2012, vol 3, chap 88, p 2512. 2013 Adult Trauma & Fistulae )

A 22-year-old man sustains a severe burn of his genitalia. There is marked bullous edema and eschar formation of the entire penis and much of the scrotum. He has had a catheter in his urethra for three days to monitor urine output. The next step is: A) radical eschar debridement. B) split thickness skin grafts. C) hyperbaric oxygen therapy. D) remove urethral catheter and insert suprapubic tube. E) observe for wound granulation.

E ( oral acyclovir. This patient has genital herpes (herpes simplex virus, HSV), of which 85-90% are caused by HSV-2 and 10-15% are caused by HSV-1. Initial genital herpes infection is often associated with constitutional flu-like symptoms. While vesicular eruptions can be found on physical exam, women especially may present with atypical lesions such as abrasions, fissures or itching. Empiric treatment may be initiated. Diagnosis can be helped by serology tests for antibodies to HSV-2 and HSV-1. Antiviral creams are not helpful for genital herpes. Oral acyclovir has been shown to prevent recurrence of genital herpes and associated symptoms. Hydrocortisone cream is not recommended for the treatment of genital herpes, however, recent studies suggest that a combination of topical acyclovir and hydrocortisone cream may reduce the recurrence of herpes labialis. Ceftriaxone is an appropriate treatment for chancroid but not genital herpes. Topical imiquimod is not recommended for treatment of routine genital herpes but is being used to treat recalcitrant cases of acyclovir-resistant herpes in immunocompromised hosts. Frenkl TL, Potts JM: Sexually transmitted infections, Wein AJ, Kavoussi LR, Novick AC, Partin AW, Peters CA (eds): CAMPBELL-WALSH UROLOGY, ed 10. Philadelphia, Elsevier Saunders, 2012, vol 1, chap 13, p 405.Perkins N, Nisbet M, Thomas M: Topical imiquimod treatment of aciclovir-resistant herpes simplex disease: Case series and literature review. SEX TRANSM INFECT 2011;87:292-295. 2013 Adult Infection & Inflammatory Disease )

A 22-year-old sexually active woman complains of vulvovaginal itching and flu-like symptoms. On physical exam, she is afebrile and the only finding is a fissure in the left labia majora with no vaginal discharge. Urinalysis is negative. The treatment that can prevent recurrence of her symptoms is: A) hydrocortisone cream. B) diphenhydramine cream. C) ceftriaxone IM. D) imiquimod cream. E) oral acyclovir.

B ( add somatostatin. After making the patient NPO and starting TPN, the next step is the administration of subcutaneous or I.V. somatostatin. Recent reports have shown a beneficial effect leading to drying up of lymphatic fistulas in this setting. Somatostatin works by decreasing the absorption of fats, inhibiting gastric, intestinal, and pancreatic secretions, and inhibiting motor activity of the intestines. The net effect is reduced flow within the major lymphatic channels, and reduced leakage from the fistula site. Somatostatin is thus recommended prior to proceeding with more invasive measures. Lievovitch I, Mor Y, Golomb J, Ramon J: The diagnosis and management of postoperative chylous ascites. J UROL 2002;167:449-457. 2013 Adult Neoplasm )

A 23-year-old man develops chylous ascites after RPLND. He is initially managed with a medium chain triglyceride diet, and for the last two weeks, with TPN. He still requires periodic paracentesis four weeks postoperatively. The next step is: A) continue current management. B) add somatostatin. C) laparoscopic ligation of the source. D) open surgical ligation of the source. E) peritoneovenous shunt.

D ( genitofemoral nerve entrapment. Nerve injury can be seen after transplantation or a psoas hitch. The genitofemoral nerve arises from the L1-2 ventral primary rami and at the level of L3-4, the nerve pierces the anterior surface of the psoas major muscle and descends past the ureter. It then splits into the genital and femoral branches near the inguinal ligament. The surgeon must not mistake this nerve for a lymphatic vessel as it sometimes crosses the external iliac artery. This nerve supplies the cremaster muscle, spermatic cord, scrotum, and thigh in males. Incisional pain can be common after kidney transplantation but after 2 weeks, postoperative pain usually subsides. Although pain to very light touch can suggests addictive behavior, the entrapped genitofemoral nerve can indeed present in this manner. More often, the pain may be worse with internal or external rotation of the hip and prolonged walking. Lateral cutaneous nerve injury presents with anterior and lateral thigh paraesthesia symptoms of burning and tingling that increase with standing, walking, or hip extension. Patients with femoral nerve damage complain of difficulty in walking and sometimes knee buckling depending on the severity of the nerve injury.Barry JM, Conlin MJ: Renal transplantation, Wein AJ, Kavoussi LR, Novick AC, Partin AW, Peters CA (eds): CAMPBELL-WALSH UROLOGY, ed 10. Philadelphia, Elsevier Saunders, 2012, vol 2, chap 44, p 1241. 2013 Adult Fluid & Electrolyte,Transplantation, Hypertension, Vasc Disease, Nephrology )

A 23-year-old man had a successful kidney transplant two weeks ago. He has significant pain below his incision without any leg weakness. The most likely diagnosis is: A) postoperative pain. B) addictive behavior. C) lateral cutaneous nerve injury. D) genitofemoral nerve entrapment. E) femoral nerve damage.

C ( CIC. This patient has classic symptoms of spinal shock which usually lasts 6-12 weeks in complete suprasacral spinal cord lesions. Lower urinary tract function with spinal shock is usually a combination of an acontractile and areflexic bladder and a competent bladder neck. This patient likely has suprapubic fullness due to an elevated residual with leakage secondary to overflow incontinence. This could be managed with either an indwelling catheter or the initiation of regular CIC. As this patient should have normal upper extremity function with this level of injury, and therefore should be able to perform CIC, there is no reason to place a suprapubic catheter. Urodynamic studies are indicated once spinal shock has resolved and bladder function has stabilized. The resolution of spinal shock is often heralded by the return of deep tendon reflexes.Wein AJ, Dmochowski RR: Neuromuscular dysfunction of the lower urinary tract, Wein AJ, Kavoussi LR, Novick AC, Partin AW, Peters CA (eds): CAMPBELL-WALSH UROLOGY, ed 10. Philadelphia, Elsevier Saunders, 2012, vol 3, chap 65, p 1920. 2013 Adult Neurogenic Bladder, Voiding Dysfunction, Incontinence )

A 24-year-old man had a complete T4 spinal cord injury three weeks ago. He is wearing a condom catheter for continuous leakage and examination is significant for suprapubic fullness as well as flaccid muscle paralysis and absent somatic reflexes below the level of injury. The next step is: A) suprapubic catheter. B) suprapubic catheter and urodynamics. C) CIC. D) CIC and urodynamics. E) CIC, urodynamics and renal ultrasound.

A ( observation. PET imaging is useful to assess post-chemotherapy residual masses after treatment of seminoma. Lesions that are less than 3 cm or non-enhancing can be safely observed as over 90% of seminoma postchemotherapy masses are fibrosis. Percutaneous biopsy is not reliable since the masses can be heterogeneous. Resection of non-enhancing masses is not necessary and resection of seminoma post-chemotherapy masses can be technically difficult or impossible. Resection of the mass and bilateral RPLND would be appropriate for NSGCT post-chemo masses but are not necessary in the post-chemo seminoma setting because of the low risk of cancer or teratoma in the remainder of the retroperitoneum. Salvage chemotherapy is not necessary and is highly toxic.Stephenson AJ, Gilligan TD: Neoplasms of the testis, Wein AJ, Kavoussi LR, Novick AC, Partin AW, Peters CA (eds): CAMPBELL-WALSH UROLOGY, ed 10. Philadelphia, Elsevier Saunders, 2012, vol 1, chap 31, p 837. 2013 Adult Neoplasm )

A 25-year-old man has a solid testes mass. His tumor markers are negative. He has an 8 cm retroperitoneal mass and multiple 1-2 cm. pulmonary metastases. His radical orchiectomy reveals pure seminoma. After chemotherapy his retroperitoneal mass is 2.8 cm and his pulmonary masses have resolved. PET/CT reveals no enhancement of his retroperitoneal mass. The next step is: A) observation. B) percutaneous biopsy of retroperitoneal mass. C) resection of retroperitoneal mass. D) bilateral RPLND. E) salvage chemotherapy.

C ( bulbocavernosus muscle. The bulbocavernosus muscle is most responsible for antegrade ejaculation through rhythmic contractions that compress the bulb to expel semen from the urethra. Significant injury or damage to this structure is thought to cause ejaculatory dysfunction. The corpus spongiosum pressurizes and constricts the urethra during ejaculation. The bulbospongiosus, and transverse perineum do not contribute to ejaculation. Injury to the corpus cavernosum could result in erectile dysfunction.Lue TF: Physiology of penile erection and pathophysiology of erectile dysfunction, Wein AJ, Kavoussi LR, Novick AC, Partin AW, Peters CA (eds): CAMPBELL-WALSH UROLOGY, ed 10. Philadelphia, Elsevier Saunders, 2012, vol 6, chap 23, p 690. 2013 Adult Trauma & Fistulae )

A 25-year-old man undergoes excision and primary anastomosis for a 2 cm bulbar urethral stricture. Postoperatively the patient experiences normal erectile function, but difficulty with antegrade ejaculation. The structure most likely injured is the: A) bulbospongiosus. B) transverse perineum. C) bulbocavernosus muscle. D) corpus cavernosum. E) corpus spongiosum.

B ( endopyelotomy. Pyeloplasty may result in early or delayed failure. Failure is most likely secondary to an anastomotic stricture. Transmural endopyelotomy whether performed in a retrograde or antegrade approach is the treatment of choice. Ureterocalycostomy may be considered, but only after less invasive treatments. It should be reserved for patients with intrarenal pelvis, dilated lower calyces or a lengthy proximal ureteral stricture. Nephrectomy should be reserved for kidneys with minimal renal function. Balloon dilation is suboptimal therapy. Pyeloplasty, whether open or laparoscopic should be reserved for endoscopic failures. Nakada SY, Hsu THS: Management of upper urinary tract obstruction, Wein AJ, Kavoussi LR, Novick AC, Partin AW, Peters CA (eds): CAMPBELL-WALSH UROLOGY, ed 10. Philadelphia, Elsevier Saunders, 2012, vol 2, chap 41, p 1127. 2013 Adult Obstructive Uropathy, Laparoscopy/Robotics )

A 26-year-old man has progressive left flank pain 17 years after an open pyeloplasty. Diuretic renography reveals 28% function from his left kidney with delayed renal pelvic drainage (T1/2 = 40 minutes). Retrograde ureterogram shows a 1 cm narrowing at the UPJ. The best treatment is: A) balloon dilation. B) endopyelotomy. C) nephrectomy. D) pyeloplasty. E) ureterocalycostomy.

E ( surgical repair. Pregnancy is a risk factor for renal artery aneurysm rupture, regardless of size or calcification, therefore observation and serial imaging are not recommended. If this was not a woman of child-bearing age, the aneurysm could be followed, as it is not large and completely calcified. Lisinopril will not reduce the likelihood of rupture or ischemic damage. An endovascular stent is not recommended for someone in this age group, due to the risk of lifelong anticoagulation therapy. She should be counseled to undergo surgical treatment of her aneurysm prior to becoming pregnant. Fergany A, Novick AC: Renovascular hypertension and ischemic nephropathy, Wein AJ, Kavoussi LR, Novick AC, Partin AW, Peters CA (eds): CAMPBELL-WALSH UROLOGY, ed 10. Philadelphia, Elsevier Saunders, 2012, vol 2, chap 39, pp 1078-1080. 2013 Adult Fluid & Electrolyte,Transplantation, Hypertension, Vasc Disease, Nephrology )

A 26-year-old woman has a 2 cm, circumferentially calcified saccular aneurysm on renal arteriography. Her blood pressure is 126/82 mm Hg and her creatinine is 1.1 mg/dl. She is newly married and considering pregnancy. The next step is: A) observation. B) serial imaging. C) lisinopril. D) endovascular stent. E) surgical repair.

A ( hydration and analgesics. Between 66% and 85% of women with ureteral colic will spontaneously pass their calculi with hydration and analgesic therapy. If the calculus fails to pass with conservative therapy, a ureteral stent should be placed cystoscopically with sonography or minimal radiographic imaging, as the first trimester presents the period of greatest risk of teratogenicity and spontaneous abortion. Ureteroscopy is an acceptable alternative. Fluoroscopy should be avoided. Pregnancy is an absolute contraindication for the use of SWL. Ferrandino MN, Pietrow PK, Preminger GM : Evaluation and medical management of urinary lithiasis, Wein AJ, Kavoussi LR, Novick AC, Partin AW, Peters CA (eds): CAMPBELL-WALSH UROLOGY, ed 10. Philadelphia, Elsevier Saunders, 2012, vol 2, chap 46, p 1322. 2013 Adult Calculous Disease )

A 27-year-old woman in the seventh week of pregnancy has right flank pain. Ultrasonography demonstrates a 5 mm calculus at the right UPJ. Urine culture is negative. The next step is: A) hydration and analgesics. B) stone protocol CT scan. C) ureteral stent. D) percutaneous nephrostomy. E) SWL.

D ( administer hCG and recombinant FSH. Kallmann syndrome, anosmia or hyposmia associated with hypogonadotropic hypogonadism is commonly diagnosed due to a delayed onset of puberty. Most patients are treated with exogenous testosterone at the time of their diagnosis for virilization. Testosterone is easy and cost effective to administer compared to daily injections of alternative hormones. Azoospermia in these patients results from the combination of inadequate levels of intratesticular testosterone, and the patient's natural absence of stimulatory pituitary hormones. When the patient desires to father children, spermatogenesis can be brought about by discontinuing parenteral testosterone and beginning daily IM or SQ injections of hCG and recombinant FSH. If the response is insufficient, GnRH administration may be considered but is expensive and requires I.V. administration. In patients with low ejaculate volume (< 1.5 ml), a post-ejaculate urine is useful to diagnose retrograde ejaculation, this patient's ejaculate volume is normal. Assay of testosterone, LH and FSH is not needed in this patient in whom a diagnosis of Kallmann syndrome has already been made. It would be inappropriate to proceed with testicular sperm extraction without first giving the hormonal treatment necessary to stimulate spermatogenesis. Sabanegh E, Agarwal A: Male infertility, Wein AJ, Kavoussi LR, Novick AC, Partin AW, Peters CA (eds): CAMPBELL-WALSH UROLOGY, ed 10. Philadelphia, Elsevier Saunders, 2012, vol 1, chap 21, p 639. 2013 Adult Sexual Dysfunction, Endocrinopathy, Fertility Problems )

A 28-year-old man with Kallmann syndrome is treated with exogenous testosterone. He desires a biological child. Semen analysis reveals a volume of 2.2 ml and azoospermia. The next step is: A) post-ejaculate urinalysis. B) assay testosterone, LH, and FSH. C) administer GnRH. D) administer hCG and recombinant FSH. E) testicular sperm extraction for IVF.

E ( ureteroscopic laser lithotripsy. A 9 mm stone in the proximal ureter has little chance of spontaneous passage; as such, observation is futile. Although SWL and ureteroscopy are both acceptable treatment options for management of a proximal ureteral stone, uncorrected bleeding diathesis frequently found in patients with liver dysfunction is a contraindication to SWL. Although it is optimal to correct bleeding diatheses prior to surgical intervention for stones, full correction of coagulation parameters often requires administration of multiple blood products and lengthy hospital stays. Ureteroscopy and Holmium:YAG laser lithotripsy has been shown to be safe and effective in patients with uncorrected bleeding disorders. Watterson JD, Girvan AR, Cook AJ, et al: Safety and efficacy of holmium: YAG laser lithotripsy in patients with bleeding diatheses. J UROL 2002;168:442-445.Turna B, Stein RJ, Smaldone MC, et al: Safety and efficacy of flexible ureterorenoscopy and holmium:YAG lithotripsy for intrarenal stones in anticoagulated cases. J UROL 2008;179:1415-1419. 2013 Adult Calculous Disease )

A 28-year-old woman awaiting a liver transplant because of primary biliary cirrhosis is symptomatic from a 9 mm proximal ureteral stone. Management should be: A) observation and hydration. B) ureteral stent. C) SWL. D) stent placement and SWL. E) ureteroscopic laser lithotripsy.

C ( resection of lung masses. There is about 75% concordance between retroperitoneal pathology and pulmonary mass pathology, however, approximately 25% of cases will have discordant pathology (i.e., retroperitoneal fibrosis and active tumor or teratoma in the lung field). Therefore, post-chemotherapy thoracotomy yields important prognostic information and is curative in patients with resected teratoma and a subset of patents with viable tumor.PET scanning is a valuable decision making tool for retroperitoneal post-chemotherapy seminoma for residual masses greater than or equal to 3 cm. In this patient population, provided the PET scans are performed six weeks after the last chemotherapy cycle (decreased false positives), PET scans have a negative predictive value of 96% and a positive predictive value of 78% for active seminoma. This helps identify patients who merit additional treatment for post-chemotherapy seminoma retroperitoneal masses. PET scans usefulness, however, for the evaluation of supra-diaphragmatic, residual pulmonary nodules or mediastinal masses has not been extensively studied and recommendations for its use in this clinical situation have yet to be determined.Sheinfeld J, Bosl GJ: Surgery of testicular tumors, Wein AJ, Kavoussi LR, Novick AC, Partin AW, Peters CA (eds): CAMPBELL-WALSH UROLOGY, ed 10. Philadelphia, Elsevier Saunders, 2012, vol 1, chap 32, p 871. 2013 Adult Neoplasm )

A 30-year-old man is diagnosed with stage 3 NSGCT. He undergoes radical orchiectomy and four cycles of BEP chemotherapy. His tumor markers have normalized. However, he has a 10 cm retroperitoneal mass and three 1 cm pulmonary masses (50% size reduction after chemotherapy). After his RPLND, the next step is: A) observation with serial imaging. B) PET scan with resection of lung masses if positive. C) resection of lung masses. D) resection of lung masses if retroperitoneum has active tumor. E) resection of lung masses if retroperitoneum has teratoma.

A ( initiate highly active antiretroviral therapy (HAART). The first step in treatment of Kaposi's sarcoma in patients with HIV is to initiate HAART or to optimize the HAART regimen, which generally results in remission of Kaposi's sarcoma. Local treatment can include laser therapy, cryotherapy, surgical excision, application of topical retinoids. Disseminated or visceral Kaposi's sarcoma is treated with combination chemotherapy. The gold standard combination therapy of doxorubicin, bleomycin and vincristine has been replaced in recent years with liposomal anthracyclines, such as doxorubicin. Kaposi's sarcoma is also often seen in immunosuppressed patients, such as renal transplant patients, and in this setting the treatment is a reduction of the immunosuppressive regimen. In the current era of immunosuppression, the frequency of this is diminished.Heyns CF, Groeneveld AE, Sigarroa NB: Urologic complications of HIV and AIDS. NAT CLIN PRACT UROL 2009;6:32-43. 2013 Adult Neoplasm )

A 40-year-old newly-diagnosed HIV positive man has a 2.0 cm. painless red nodule on his glans penis. A biopsy confirms Kaposi's sarcoma. The next step is: A) initiate highly active antiretroviral therapy (HAART). B) systemic chemotherapy. C) laser ablation. D) excise the lesion. E) partial penectomy.

D ( oxybutynin. Detrusor compliance may deteriorate in patients with spinal cord injury and detrusor areflexia. The development of incontinence suggests this occurrence. A detrusor LPP greater than 15 cm H2O indicates that compliance is impaired. In the absence of intervention, renal deterioration may occur. The best treatment is an antimuscarinic agent, such as oxybutynin. Ephedrine would be contraindicated as it may increase sphincter tone and increase detrusor LPP. An alpha-1-blocker may lower the detrusor LPP, but increase the incontinence. Bethanechol would also be contraindicated since it may increase detrusor pressure. Dantrolene is used to treat detrusor external sphincter dyssynergia via relaxation of skeletal muscle. Wein AJ, Dmochowski RR: Neuromuscular dysfunction of the lower urinary tract, Wein AJ, Kavoussi LR, Novick AC, Partin AW, Peters CA (eds): CAMPBELL-WALSH UROLOGY, ed 10. Philadelphia, Elsevier Saunders, 2012, vol 3, chap 65, p 1923.Andersson KE, Wein AJ: Pharmacologic management of lower urinary tract storage and emptying failure, Wein AJ, Kavoussi LR, Novick AC, Partin AW, Peters CA (eds): CAMPBELL-WALSH UROLOGY, ed 10. Philadelphia, Elsevier Saunders, 2012, vol 3, chap 68, pp 1978-1979. 2013 Adult Neurogenic Bladder, Voiding Dysfunction, Incontinence )

A 30-year-old man on CIC develops urinary incontinence three years after a spinal cord injury. Urodynamic testing demonstrates detrusor areflexia, and a detrusor LPP of 60 cm H2O at 200 ml. The next step is: A) tamsulosin. B) ephedrine. C) bethanechol. D) oxybutynin. E) dantrolene.

A ( adoption. The long arm of the Y chromosome harbors genes intrinsic to spermatogenesis, including the azoospermia factor (AZF). While males with AZFc deletions may or may not have sperm in the seminiferous epithelium, an AZFa and/or AZFb deletion in combination with an AZFc deletion uniformly results in a Sertoli cell only phenotype. Biopsy is unnecessary, and microsurgical testicular sperm extraction will not yield sperm. Endocrine therapy with clomiphene citrate to stimulate Leydig cell production of testosterone, or with recombinant follicle stimulating hormone to stimulate Sertoli cell function, will not yield sperm as no germ cells are present.Jarow J, Sigman M, Kolettis PN, et al: The optimal evaluation of the infertile male. OPTIMAL EVALUATION OF THE INFERTILE MALE BEST PRACTICE STATEMENT. American Urological Association Education and Research, Inc, 2010, p 24. http://www.auanet.org/content/media/optimalevaluation2010.pdfSabanegh E, Agarwal A: Male infertility, Wein AJ, Kavoussi LR, Novick AC, Partin AW, Peters CA (eds): CAMPBELL-WALSH UROLOGY, ed 10. Philadelphia, Elsevier Saunders, 2012, vol 1, chap 21, p 641. 2013 Adult Sexual Dysfunction, Endocrinopathy, Fertility Problems )

A 32-year-old man has azoospermia. Y chromosomal microdeletion assay reveals azoospermia factor b (AZFb) and azoospermia factor c (AZFc) deletions. The next step is: A) adoption. B) clomiphene citrate. C) recombinant FSH. D) testis biopsy. E) microsurgical testicular sperm extraction.

C ( placement of a indwelling urethral catheter. The first step necessary in assessing the renal transplant patient with new onset of hydronephrosis is the placement of a urethral catheter and obtaining a cystogram, to assess for the presence of reflux. In the absence of reflux and if urine output is diminished or if the creatinine does not decrease with urethral catheter drainage (drop in creatinine following placement of catheter indicative of bladder dysfunction), assessment for ureteral stricture would be necessary. If the serum creatinine is above 2 mg/dl or twice normal for age, false positive MAG 3 Lasix washout renal scans for obstruction are highly probable due to the decreased renal function. In patients with an elevated creatinine, percutaneous nephrostomy placement is both effective for treatment and can be an excellent way to diagnosis ureteric stenosis. The risks for complications of the ureter in renal transplantation is < 3% overall. The most common complications are those related to the vascular viability of the ureter and result in either urinary leakage or ureteral stenosis. Urinary leakage often occurs early after transplantation but ureteral stenosis can be insidious and late in the course. Risk factors for ureteral complications include advance donor age, delayed graft function, severe graft rejection and kidneys with two or more arteries. Cystoscopy and retrograde pyelography or placement of ureteric stents may be problematic due to the abnormal location of the ureteroneocystostomy following a renal transplant and should be considered only after confirmation that no vesicoureteral reflux is present and following the diagnosis of a ureteral stricture. A CT scan although confirmative for the diagnosis of hydronephrosis does not delineate the etiology for the radiographic finding.Shoskes DS, Cranston D: Urological Complications after kidney transplantation, in Morris PJ, Knechtle SJ (eds): KIDNEY TRANSPLANTATION, PRINCIPLES AND PRACTICE, ed 6. Philadelphia, Elsevier Saunders, 2008, pp 465-466. 2013 Adult Fluid & Electrolyte,Transplantation, Hypertension, Vasc Disease, Nephrology )

A 32-year-old man underwent a deceased donor kidney transplant six months ago with a donor kidney with two renal arteries. The lower pole renal artery was anastomosed separately to the distal external iliac artery. He now has a creatinine of 2.1 mg/dl and ultrasound shows hydroureteronephrosis of the transplanted kidney. The next step is: A) CT scan. B) diuretic renogram. C) placement of a indwelling urethral catheter. D) cystoscopy and retrograde stent insertion. E) percutaneous nephrostomy.

A ( intrauterine insemination. The couple is infertile due to idiopathic male factor. Testicular biopsy is unlikely to be helpful in a patient with oligospermia. Any intervention used in this setting is considered empirical. The main choice is between attempts to improve the husband's fertility or assisted reproductive techniques to improve the chances of conception without altering sperm quality. It is controversial whether clomiphene citrate is occasionally effective in men with idiopathic oligospermia but it is definitely not effective in an individual with an elevated serum FSH. This leaves either intrauterine insemination (IUI) or in vitro fertilization (IVF). It is most reasonable to start with IUI in this couple since the wife is under 30 and the husband's total motile sperm count is well above 10 million. IVF would be the first therapeutic option if there were a significant female factor or his sperm count were extremely low. The patient's motility is above the reference value for motility and therefore there is no indication for antisperm antibody testing.Demir B, Dilbaz B, Cinar O, et al: Factors affecting pregnancy outcome of intrauterine insemination cycles in couples with favorable female characteristics. J OBSTET GYN 2011;31:420-423.Dorjpurev U, Kuwahara A, Yano Y, et al: Effect of semen characteristics on pregnancy rate following intrauterine insemination. J MED INVEST 2011;58:127-133. 2013 Adult Sexual Dysfunction, Endocrinopathy, Fertility Problems )

A 32-year-old man with infertility has an ejaculate volume of 3 ml, sperm count of 13 million/ml, 50% motility, and an elevated serum FSH. Physical examination reveals bilaterally small testes, normal vasa, and no evidence of a varicocele. His 28-year-old wife has a normal evaluation. The most appropriate next step is: A) intrauterine insemination. B) testicular biopsy. C) in vitro fertilization. D) clomiphene citrate. E) antisperm antibody testing.

C ( erectile function. Microvascular reconstruction involves reanastomosis of the dorsal arteries, dorsal vein, and nerves. Macroscopic replantation is the simple anastomosis of the corpora cavernosum and urethra. Erectile function after either microvascular reconstruction or macroscopic replantation of the penile shaft is roughly 50%. Penile skin loss, urethral stricture formation, and loss of penile sensation are all greater with macroscopic replantation as compared to microvascular reconstruction of the penile shaft. Infection of the penile shaft after either technique has not been studied to date.Morey AF, Dugi DD III: Genital and lower urinary tract trauma, Wein AJ, Kavoussi LR, Novick AC, Partin AW, Peters CA (eds): CAMPBELL-WALSH UROLOGY, ed 10. Philadelphia, Elsevier Saunders, 2012, vol 3, chap 88, p 2509. 2013 Adult Trauma & Fistulae )

A 34-year-old man amputates his penis during a psychotic episode. He is brought to the emergency department in stable condition with cold ischemia time of the amputated penis of six hours. Microvascular reconstruction or macroscopic replantation of the penile shaft provides an equivalent outcome for: A) penile skin preservation. B) urethral stricture formation. C) erectile function. D) penile sensation. E) infection.

A ( PVR. This patient is likely experiencing worsening of symptoms due to incomplete bladder emptying. It appears that approximately 25% of patients with a neurogenic bladder who are not catheterizing at baseline will require CIC after injection of 200 units of onabotulinumtoxinA. UTI could cause her worsening symptoms; however, this would be more likely if the patient is already performing catheterization. Urodynamics and cystoscopy would not be helpful in the immediate postoperative period. If a patient requires reinjection this should be done at least two to three months after the initial injection. Recent studies suggest that the 200 U dose is adequate and a higher dose is not beneficial.Andersson KE, Wein AJ: Pharmacologic management of lower urinary tract storage and emptying failure, Wein AJ, Kavoussi LR, Novick AC, Partin AW, Peters CA (eds): CAMPBELL-WALSH UROLOGY, ed 10. Philadelphia, Elsevier Saunders, 2012, vol 3, chap 68, p 1987.Cruz F, Herschorn S, Aliotta P, et al: Efficacy and safety of onabotulinumtoxinA in patients with urinary incontinence due to neurogenic detrusor overactivity: A randomized, double-blind, placebo-controlled trial. EUR UROL 2011;60:742-750. 2013 Adult Neurogenic Bladder, Voiding Dysfunction, Incontinence )

A 34-year-old woman with multiple sclerosis continues to have significant urinary incontinence despite maximal antimuscarinics. Two weeks after intradetrusor injection of 200 units of onabotulinumtoxinA, she complains of worsening frequency, urgency and urinary incontinence. The next step is: A) PVR. B) cystoscopy. C) urodynamics. D) empiric antibiotic therapy. E) reinject 100 units of onabotulinumtoxinA.

E ( microsurgical testicular sperm extraction. With FSH greater than 7.6 IU/l and testis axis less than 4.6 cm, the probability of non-obstructive azoospermia is 89%. Neither clomiphene citrate nor hCG is effective in a patient with highly elevated LH, as clomiphene acts to increase LH secretion and hCG is an LH surrogate. Scrotal ultrasound would not reveal more than that identified on physical examination. As the pituitary is responding appropriately to low serum testosterone, MRI is not indicated. The only chance of a biological child for this patient with non-obstructive azoospermia would be testicular sperm extraction for intracytoplasmic sperm injection.Sabanegh E, Agarwal A: Male infertility, Wein AJ, Kavoussi LR, Novick AC, Partin AW, Peters CA (eds): CAMPBELL-WALSH UROLOGY, ed 10. Philadelphia, Elsevier Saunders, 2012, vol 1, chap 21, p 634.Schoor RA, Elhanbly S, Niederberger CS, Ross LS: The role of testicular biopsy in the modern management of male infertility. J UROL 2002;167:197-200. 2013 Adult Sexual Dysfunction, Endocrinopathy, Fertility Problems )

A 35-year-old man with azoospermia and normal genetic testing desires a biological child. Testosterone is 275 ng/dl, LH is 28 IU/l and FSH is 15 IU/l. Both testes are 3 cm in length and soft. The next step is: A) clomiphene citrate. B) hCG. C) scrotal ultrasound. D) cranial MRI scan. E) microsurgical testicular sperm extraction.

B ( acetazolamide. Acetazolamide is effective in increasing the urinary pH in patients with uric acid and cystine stone formation who are already taking potassium citrate. However, 50% of patients may discontinue the medication due to adverse effects. Acetazolamide, a carbonic anhydrase inhibitor, leads to an increase in urinary bicarbonate and increased H+ reabsorption. It has been shown to increase overnight urine pH when given at bedtime. Allopurinol is effective for uric acid stones but does not increase pH, and might be considered but only if urinary uric acid levels were high. Sodium citrate has been shown to be less effective than potassium citrate therapy. Hydrochlorothiazide may increase urine uric acid.Sterrett SP, Penniston KL, Wolf JS Jr, Nakada SY: Acetazolamide is an effective adjunct for urinary alkalization in patients with uric acid and cystine stone formation recalcitrant to potassium citrate. UROL 2008;72:278-281. 2013 Adult Calculous Disease )

A 35-year-old man with uric acid calculi has a nighttime urinary pH of 5.5, despite potassium citrate 40 mEq TID. The next step is adding: A) allopurinol. B) acetazolamide. C) ascorbic acid. D) sodium citrate. E) hydrochlorothiazide.

A ( lactulose. Urinary ammonium excreted by the kidneys is reabsorbed by the intestinal segment, and then returned to the liver via the portal circulation. The liver metabolizes ammonium to urea via the ornithine cycle. The liver usually adapts to the excess ammonia in the portal circulation without difficulty and rapidly metabolizes it. In the setting of hepatic dysfunction, the hepatic reserve for ammonium metabolism may be exceeded, resulting in the complication of an ammoniagenic coma. The syndrome, however, also has been described in patients with normal hepatic function. Systemic bacteremia, with endotoxin production, inhibits hepatic function and may precipitate this clinical entity. Urinary tract infections with urea-splitting organisms may also overload the ability of the liver to clear the ammonia. If this syndrome occurs in a patient suspected of having near normal hepatic function, systemic bacteremia or urinary obstruction should be suspected. Prompt urinary drainage with treatment of the offending urinary pathogens along with systemic antibiotics and the administration of oral neomycin or lactulose to reduce absorption of ammonia in the gastrointestinal tract are the key components to patient management. There is no indication for the use of Vitamin B12, sodium bicarbonate, nicotinic acid, thiamine and folic acid in this clinical setting. Demarco RT, Koch MO: Metabolic complications of continent urinary diversion. AUA UPDATE SERIES 2003, vol 22, lesson 15, pp 114-119. 2013 Adult Urinary Diversion )

A 37-year-old woman with a continent cutaneous urinary diversion becomes febrile and develops mental status changes and marked hepatic dysfunction. Previously, her hepatic function had been normal. In addition to prompt urinary drainage and systemic antibiotics, the next step is: A) lactulose. B) Vitamin B12. C) sodium bicarbonate. D) nicotinic acid. E) thiamine and folic acid.

B ( cystoscopy and stent placement. The patient has a ureteral contusion to the distal ureter following a gunshot wound. There is no evidence of devitalized tissue or a urine leak. Cystoscopy and ureteral stent placement is the best option for a minor contusion. Ureteral reimplant is the best treatment option if a large contusion or devitalized tissue is identified. Ureteroureterostomy of the distal ureter is never indicated. Percutaneous nephrostomy would divert the urine yet would not stent the ureter which may lead to stricture formation. Observation is not indicated as the ureter may become obstructed from the contusion.Santucci RA, Doumanian LR: Upper urinary tract trauma, Wein AJ, Kavoussi LR, Novick AC, Partin AW, Peters CA (eds): CAMPBELL-WALSH UROLOGY, ed 10. Philadelphia, Elsevier Saunders, 2012, vol 2, chap 42, p 1184. 2013 Adult Trauma & Fistulae )

A 42-year-old man sustains a high velocity pelvic gunshot wound with no obvious ureteral injury at exploration. Two days later during a second look operation for bleeding, a minor distal ureteral contusion is identified. Intravenous indigo carmine does not reveal a urine leak. The next step is: A) observation. B) cystoscopy and stent placement. C) percutaneous nephrostomy. D) ureteroneocystostomy. E) debridement and ureteroureterostomy.

D ( ureterovaginal fistula and urethral leakage. The in-office dye test begins with prescribing oral phenazopyridine several days prior to office visit. As expected, this process will turn the urine orange. At time of office visit, the bladder is filled with dilute methylene blue via urethral catheter infusion. Thus, bladder fluid will have blue coloration, and ureteral urine is expected to have orange discoloration. Three gauze pads are placed in the vagina, the upper pad is near the cuff, and the middle pad is within the vagina, usually above the bladder neck. The lower pad is usually below the bladder neck and urethra. In this pattern of staining, orange urine in the upper pad is concerning for the presence of a ureteral source of drainage into the vagina, most commonly a ureterovaginal fistula. The appearance of blue staining on the lower pad is most consistent with urethral leakage. Thus this patient has results suggestive of a ureterovaginal fistula and urethral leakage. The presence of blue on the upper or middle pad without leakage on the lower pad would suggest a vesicovaginal fistula.Rovner ES: Urinary tract fistulae, Wein AJ, Kavoussi LR, Novick AC, Partin AW, Peters CA (eds): CAMPBELL-WALSH UROLOGY, ed 10. Philadelphia, Elsevier Saunders, 2012, vol 3, chap 77, p 2243. 2013 Adult Trauma & Fistulae )

A 37-year-old woman with a suspected urinary fistula undergoes an in-office double dye test. There is orange staining of the upper gauze pad. The middle gauze pad is dry, and the lower gauze pad is slightly stained blue. The most likely diagnosis is: A) ureterovaginal fistula. B) vesicovaginal fistula. C) ureterovaginal and vesicovaginal fistula. D) ureterovaginal fistula and urethral leakage. E) vesicovaginal fistula and urethral leakage.

E ( transabdominal repair with hysterectomy. Vesicouterine fistula (also known as, Youssef syndrome) is found to be the etiology for 2-4% of all genitourinary fistula. A vesicouterine fistula is a rare complication that classically develops following a Cesarean section. The mechanism of the injury occurs as a consequence of incorporating the bladder wall with the sutures used to close the Cesarean section site, hence, it is frequently a delayed complication noted only as the sutures dissolve. It is also occasionally seen to arise following a dilation and curettage (D and C) procedure or as a consequence of a vaginal delivery following a prior Cesarean section.Due to the sphincteric like activity of the uterine cervix, these patients may not present with urinary incontinence. Indeed, 60% of the patients will present with intermittent or cyclical gross hematuria (menouria; menstrual tissue passed through the urine when voiding) as their only symptom, 20% with chronic urinary incontinence (incompetent uterine sphincter), and 20% with the classic Youssef triad: menouria, amenorrhea (all menstrual tissue passed into the urine) and chronic urinary incontinence.Since radiographic and endoscopic studies may frequently be inconclusive, the diagnoses of this complication requires a high degree of clinical suspicion. Tests used will consist of cystoscopy in an attempt to visualize the fistula tract, ultrasonography, CT cystogram, MRI scan, or hysterosalpingogram, the latter of which is presumably the most certain diagnostic technique available. Once the diagnosis is made, additional tests to rule-out concurrent ureteral injuries must be performed.If the diagnosis of a vesicouterine fistula is made within three to six months of the surgery, conservative treatment with a indwelling urethral catheter and endocrine suppression of menstrual flow has been successful in > 50% of patients. Once the tract has matured and epithelized as in this case, treatment is based on the future fertility desires of the patient. If the individual does not desire to have further children, the most definitive treatment is by transabdominal hysterectomy and repair of the bladder. If additional child bearing is requested, either transvaginal, or transabdominal approaches may be used dependent upon the location of fistula. Juxtaposition of adjacent tissue such as omentum or a labial fat pad (Martius flap) greatly reduces the likelihood of recurrence. These dissections are frequently complicated and placement of bilateral ureteral stents prior to the surgery to help identify the ureters is recommended. Rovner ES: Urinary tract fistulae, Wein AJ, Kavoussi LR, Novick AC, Partin AW, Peters CA (eds): CAMPBELL-WALSH UROLOGY, ed 10. Philadelphia, Elsevier Saunders, 2012, vol 3, chap 77, pp 2246-2268. 2013 Adult Trauma & Fistulae )

A 38-year-old woman develops monthly cyclical episodes of hematuria and menouria one year following Caesarean section. She does not desire more children. CT urogram demonstrates contrast in the bladder and uterine cavity; there is no hydronephrosis. The best treatment is: A) cystoscopy, fulguration and catheter drainage. B) LH-RH agonist. C) transvaginal repair with Martius flap. D) transvaginal repair with omental flap. E) transabdominal repair with hysterectomy.

C ( ureteroscopy, incision of the diverticular neck and laser lithotripsy. Observation is unacceptable in this symptomatic patient with a diverticular stone. Although SWL may be used successfully to treat a subset of patients with stones in calyceal diverticula that have a broad infundibular neck, the overall stone free rates with SWL are unacceptably low. Percutaneous nephrostolithotomy is not only associated with the highest stone-free rate, but also the procedure results in resolution of the diverticulum. However, anteriorly-located diverticula necessitate percutaneous access through the renal parenchyma with a high risk of bleeding complications. The ureteroscopic approach is ideal for upper pole calyceal diverticula with < 2 cm stones. Laparoscopic ablation would be indicated for an anterior calyceal diverticulum > 2 cm. Canales B, Monga M: Surgical management of the calyceal diverticulum. CURR OPIN UROL 2003;13:255-260.Chong TW, Bui MHT, Fuchs GJ: Calyceal diverticula. Ureteroscopic management. UROL CLIN N AM 2000;27:647-653. 2013 Adult Calculous Disease )

A 38-year-old woman has intermittent right flank pain. CT scan shows delayed filling of a right upper pole anterior calyceal diverticulum containing an 8 mm stone. The best treatment is: A) observation. B) SWL. C) ureteroscopy, incision of the diverticular neck and laser lithotripsy. D) percutaneous nephrostolithotomy and dilation of the infundibular neck. E) laparoscopic ablation of calyceal diverticulum and stone removal.

D ( antibiotic therapy. The standard definition of significant bacteriuria for a clean voided urine is > 105 CFU/ml) of a uropathogen. This criterion has stood the test of time for screening and epidemiological studies and for entering patients in clinical trials. However there are several important exceptions to its rigid use in clinical practice and one is in patients with a pyuria/dysuria syndrome. In these patients, a lower colony count may represent significant bacteriuria.Certain bacterial species such as coagulase negative Staphylococci grow slowly in urine and significant infections may only have counts of 103 CFU/ml. Since the patient has a symptomatic, culture-proven UTI, treatment with phenazopyridine alone would be inappropriate. Repeat urine culture (midstream or catheterized) is not indicated. Mycobacteria culture is indicated only in sterile pyuria. Schaeffer AJ, Schaeffer EM: Infections of the urinary tract, Wein AJ, Kavoussi LR, Novick AC, Partin AW, Peters CA (eds): CAMPBELL-WALSH UROLOGY, ed 10. Philadelphia, Elsevier Saunders, 2012, vol 1, chap 10, p 271-272. 2013 Adult Infection & Inflammatory Disease )

A 39-year-old woman has dysuria and frequency. Urinalysis shows 30 WBC/hpf and a few cocci. The midstream urine culture shows 103 coagulase negative Staphylococci/ml. The next step is: A) phenazopyridine. B) repeat midstream culture. C) urine culture for mycobacteria. D) antibiotic therapy. E) catheterized urine culture.

D ( repeat bladder neck closure with rectus flap interposition. Persistent vesicourethral fistula occurs frequently with bladder neck closure where vascularized tissue is not interposed between the bladder neck and urethra. Omentum is the most commonly used tissue for interposition but occasionally is not available or cannot be brought down to the level of the bladder neck closure. When this is not possible, or in high risk cases (radiated patients, persistent vesicourethral fistulae, etc.) a rectus abdominus pedicle flap can be used for interposition. Tube vesicostomy will not help this patient as he will continue to be incontinent and it has been a lengthy interval since his surgery. Permanent nephrostomy tubes are undesirable and the patient may well continue to be incontinent. An ileal conduit can be considered but would be significantly more extensive than repeating the bladder neck closure.Rovner ES: Urinary tract fistulae, Wein AJ, Kavoussi LR, Novick AC, Partin AW, Peters CA (eds): CAMPBELL-WALSH UROLOGY, ed 10. Philadelphia, Elsevier Saunders, 2012, vol 3, chap 77, p 2238. 2013 Adult Trauma & Fistulae )

A 40-year-old man with spina bifida undergoes ileovesicostomy and bladder neck closure with omental flap interposition for severe incontinence. Three months later he develops recurrent incontinence from a vesicourethral fistula. The next step is: A) tube vesicostomy. B) permanent nephrostomy tubes. C) repeat bladder neck closure with omental interposition. D) repeat bladder neck closure with rectus flap interposition. E) ileal conduit.

D ( angiography and placement of endovascular graft. Most cases of ureteroarterial fistulas are reported in patients with a prior history of vascular disease, radiation therapy, or pelvic surgery, especially in the setting of indwelling ureteral stents. In fact, ureteroarterial fistulas are highly associated with indwelling stents. The routine urologic and radiologic evaluation of hematuria will not generally provide evidence of ureterovascular fistula. Even in suspected or proven cases, preoperative radiologic investigations including nonselective arteriography and pyelography are often nondiagnostic. This is especially true in patients with intermittent hematuria in whom there is no active bleeding at the time of the radiographic investigation, presumably because of thrombus over the site of the fistula. Selective or subselective arteriography of the iliac vessels may be more revealing in suspected cases, and provocative maneuvers such as stent removal or mechanical friction of the ureteral lumen by manipulation of the stent may be necessary to demonstrate the fistulous connection in patients without active bleeding undergoing angiography. These adjuvant maneuvers should be performed only with extreme caution in an appropriate setting where immediate angiographic or surgical intervention is possible. In a review, retrograde pyelography was diagnostic in only 6 of 10 patients in whom it was performed, and arteriography diagnosed a ureterovascular fistula in only 4 of 14 cases. Indirect evidence of a ureteroarterial fistula can be found on CT, but findings are usually nonspecific and suggestive only in retrospect after a confirmed diagnosis by other means. Nevertheless, in a stable patient with a suspected ureterovascular fistula, a full radiographic evaluation may be pursued, not only for diagnostic purposes but also to evaluate potential reconstructive options and in select cases to perform therapeutic angiographic embolization procedures. As these patients may present in extremis with hypotension and severe hemorrhage, surgical intervention must be considered early, especially since radiographic evaluation may be nondiagnostic. In this stable patient, an attempt at angiography and placement of an endovascular graft is warranted. Open exploration may be needed if hematuria persists. Replacement of the ureteral stent or percutaneous nephrostomy will not stop the hemorrhage. Ureteroscopy with fulguration will be unsuccessful with an arterial-ureteral fistula.Rovner ES: Urinary tract fistulae, Wein AJ, Kavoussi LR, Novick AC, Partin AW, Peters CA (eds): CAMPBELL-WALSH UROLOGY, ed 10. Philadelphia, Elsevier Saunders, 2012, vol 3, chap 77, pp 2259-2260. 2013 Adult Obstructive Uropathy, Laparoscopy/Robotics )

A 45-year-old woman with prior pelvic radiation for cervical cancer develops severe hemorrhage from the right ureteral orifice during routine exchange of a chronic indwelling 6 Fr ureteral stent. Over the next six hours, she continues to bleed but remains stable. The next step is: A) replace stent with 8 Fr stent. B) right percutaneous nephrostomy. C) ureteroscopy with fulguration. D) angiography and placement of endovascular graft. E) open exploration.

C ( evaluation of the wife. The likelihood of obstructive azoospermia is 96% with testis longitudinal axis greater than 4.6 cm and FSH less than 7.6 IU/l. However, the most significant predictor of any form of reproductive intervention is maternal age, with female fecundity declining precipitously after age 37. The decision to perform microsurgical scrotal ductal reconstruction or to obtain sperm from the testis for IVF and intracytoplasmic sperm injection rests on evaluation of the female partner, especially after age 37. Transrectal ultrasound is not necessary if semen volumes are normal (> 1.5 ml) as ejaculatory ductal obstruction is unlikely.Sabanegh E, Agarwal A: Male infertility, Wein AJ, Kavoussi LR, Novick AC, Partin AW, Peters CA (eds): CAMPBELL-WALSH UROLOGY, ed 10. Philadelphia, Elsevier Saunders, 2012, vol 1, chap 21, pp 619, 635. 2013 Adult Sexual Dysfunction, Endocrinopathy, Fertility Problems )

A 43-year-old man desires a biological child with his 38-year-old wife. Both testes are 5 cm in longitudinal axis and firm on physical examination. Two semen analyses show azoospermia with volumes of 2.1 and 2.3 ml. FSH is 2.8 IU/l. The next step is: A) adoption. B) TRUS. C) evaluation of the wife. D) testicular sperm extraction with ICSI. E) microsurgical scrotal ductal reconstruction.

A ( surveillance with ureteroscopy. This patient is an ideal candidate for endourologic management of an upper tract tumor. The patulous ureteral orifices will allow easy passage of the ureteroscope for surveillance. The surgical options listed are unnecessary for a completely resected tumor. Intravesical BCG is not indicated for a first-time low grade tumor. Martinez-Pineiro JA, Matres MJG, Martinez-Pineiro L: Endourologic treatment of upper tract urothelial carcinomas: analysis of a series of 59 tumors. J UROL 1996;156:377-385.Sagalowsky AI, Jarrett TW, Flanigan RC: Urothelial tumors of the upper urinary tract and ureter, Wein AJ, Kavoussi LR, Novick AC, Partin AW, Peters CA (eds): CAMPBELL-WALSH UROLOGY, ed 10. Philadelphia, Elsevier Saunders, 2012, vol 2, chap 53, p 1516. 2013 Adult Neoplasm )

A 43-year-old paraplegic man with a neurogenic bladder has gross hematuria. A cystogram shows bilateral grade 2 VUR and a 1 cm papillary filling defect in the mid-right ureter. Cystoscopy shows patulous ureteral orifices, but no other abnormalities. Complete ureteroscopic resection of the tumor reveals a low grade non-invasive urothelial carcinoma. The serum creatinine is 1.0 mg/dl. The best management is: A) surveillance with ureteroscopy. B) nephroureterectomy. C) partial ureterectomy. D) ureterectomy and ileal ureter. E) intravesical BCG.

D ( osteitis pubis. The patient presents with the symptoms of osteitis pubis in the athlete. It is estimated that osteitis pubis accounts for 6-14% of groin pain in athletes and is thought to be a form of overuse injury. The onset of unilateral or bilateral groin pain is gradual and may be accompanied by pain in the lower abdomen, hip, thigh, or testicle. Acutely painful infectious vasitis without associated epididymitis is exceedingly rare. Femoral hernia may cause groin pain but is not typically associated with pain of the symphysis pubis. Pubic stress fractures do occur but are less common than osteitis pubis in athletes. Osteomyelitis of the pubis has been reported in athletes and must be considered in the differential diagnosis. However, it is much less common than osteitis pubis. Pelvic MRI scan is used to distinguish between osteomyelitis and osteitis pubis. Treatment for osteitis pubis is anti-inflammatory medications and rest.Johnson R: Osteitis pubis. CURR SPORTS MED REP 2003;2:98-102.Karpos PA, Spindler KP, Pierce MA, et al: Osteomyelitis of the pubic symphysis in athletes: A case report and literature review. MED SCI SPORTS EXCERCISE 1995;27:473-479.Eddy K, Piercy GB, Eddy R: Vasitis: Clinical and ultrasound confusion with inguinal hernia clarified by computed tomography. CAN UROL ASSOC J 2011;5:E74-76. 2013 Adult Infection & Inflammatory Disease )

A 44-year-old runner experiences the gradual onset of left groin pain. Examination reveals normal genitalia and tenderness over the symphysis pubis. His urinalysis is clear. The most likely cause of his pain is: A) vasitis. B) femoral hernia. C) pubic bone stress fracture. D) osteitis pubis. E) osteomyelitis of the pubis.

B ( increased secretion of atrial natriuretic peptide. Sleep apnea is a recognized cause of nocturia and secondary, or adult-onset, nocturnal enuresis. It causes nocturnal diuresis by a cascade of events which are precipitated by hypoxia which occurs during the intermittent occlusion that occurs with obstructive sleep apnea. The hypoxia-induced increase in right atrial transmural pressure leads to elevated atrial natriuretic peptide, resulting in increased nocturnal urinary output. Atrial natriuretic peptide secretion is induced by elevated intrathoracic pressures due to diaphragmatic contraction against a closed upper airway. Drake M: Nocturia, Wein AJ, Kavoussi LR, Novick AC, Partin AW, Peters CA (eds): CAMPBELL-WALSH UROLOGY, ed 10. Philadelphia, Elsevier Saunders, 2012, vol 3, chap 67, p 1961. 2013 Adult Neurogenic Bladder, Voiding Dysfunction, Incontinence )

A 45-year-old obese man with untreated sleep apnea develops nocturnal enuresis. He has no daytime incontinence. Physical examination is unremarkable except for mild lower extremity edema. Urinalysis is negative, and his PVR is 30 ml. The most likely etiology of the enuresis is: A) decreased secretion of ADH. B) increased secretion of atrial natriuretic peptide. C) detrusor overactivity. D) mobilization of lower extremity edema. E) hypercarbia induced drowsiness.

B ( plasma free metanephrine levels. Repeat metabolic testing should be performed after adrenalectomy to document normalization of chromaffin cell function. One of the pitfalls of diagnosing and treating pheochromocytoma is failure to recognize multiple lesions at the time of treatment. It has been estimated that between 10-20% of pheochromocytomas are malignant or multifocal. In a patient who undergoes surgical resection of an adrenal pheochromocytoma but still has persistent unexplained hypertension two to three months after the procedure, residual tumor somewhere else in the body must be considered. While MIBG scan may be helpful in identifying the location of this lesion, the patient should initially have plasma free metanephrine levels measured. If this is normal MIBG scan is not indicated. Similarly, MRI should not be ordered until pheochromocytoma is ruled in or out using less invasive diagnostic testing. Clinical suspicion of primary hyperaldosteronism or Cushing's syndrome is low in this case, so plasma renin activity and 24-hour urinary cortisol levels will not be helpful.Kutikov A, Crispen PL, Uzzo RG: Pathophysiology, evaluation, and medical management of adrenal disorders, Wein AJ, Kavoussi LR, Novick AC, Partin AW, Peters CA (eds): CAMPBELL-WALSH UROLOGY, ed 10. Philadelphia, Elsevier Saunders, 2012, vol 2, chap 57, pp 1710-1711. 2013 Adult Physiology, Immunology, & Adrenal )

A 45-year-old woman undergoes a left adrenalectomy for pheochromocytoma. Three months later, she still has sustained hypertension. The next step is: A) plasma renin activity. B) plasma free metanephrine levels. C) 24-hour urinary cortisol levels. D) abdominal MRI scan. E) MIBG scan.

E ( immediate prosthesis implantation. Priapism lasting longer than 36 hours recalcitrant to multiple attempts at irrigation as well as shunting procedures, is best treated with immediate penile prosthesis implantation. While immediate implantation carries a greater risk of infection and erosion, it preserves penile length and makes prosthesis implantation easier. In this severe case of ischemic priapism, oral therapies are not indicated and he has already failed a proximal shunting procedure. A T-shunt is another form of a distal corporal-glanular shunt. A repeat proximal shunt would not be unreasonable in this individual, but would have a low chance of success and will likely be associated with significant penile shortening and fibrosis.Broderick GA: Priapism, Wein AJ, Kavoussi LR, Novick AC, Partin AW, Peters CA (eds): CAMPBELL-WALSH UROLOGY, ed 10. Philadelphia, Elsevier Saunders, 2012, vol 1, chap 25, pp 764-766. 2013 Adult Sexual Dysfunction, Endocrinopathy, Fertility Problems )

A 46-year-old-man has a sustained erection for 72 hours. His erection has persisted despite irrigation with dilute phenylephrine solution through multiple glanular punctures and a subsequent corporal-cavernosal shunt. One day later, he continues to have a rigid penis. The best treatment is: A) oral terbutaline. B) oral bicalutamide. C) corporal-glanular shunt. D) bilateral T-shunt. E) immediate prosthesis implantation.

D ( coagulopathy and hemorrhage. Hypothermia and circulatory arrest is the treatment of choice for a renal tumor with this level of cephalad extension. This technique has several potential complications such as CNS or hepatic damage yet the most common difficulty associated with this technique is hemorrhage associated with platelet and clotting factor dysfunction. Tumor emboli can occur but are relatively uncommon. Utilization of cardiopulmonary bypass limits the possibility of embolic events.Marshall FF, Dietrick DD, Baumgartner WA, Reitz BA: Surgical management of renal cell carcinoma with intracaval neoplasm extension above the hepatic veins. J UROL 1998;139:1166-1172.Kenney PA, Wotkowicz C, Libertino JA: Contemporary open surgery of the kidney, Wein AJ, Kavoussi LR, Novick AC, Partin AW, Peters CA (eds): CAMPBELL-WALSH UROLOGY, ed 10. Philadelphia, Elsevier Saunders, 2012, vol 2, chap 54, p 1624. 2013 Adult Neoplasm )

A 50-year-old man has a large right renal mass with tumor thrombus extending into the atrium. Under hypothermia and circulatory arrest, he undergoes nephrectomy with removal of the tumor thrombus. The most frequent significant complication is: A) hepatic dysfunction. B) pulmonary air embolus. C) central nervous system deficit. D) coagulopathy and hemorrhage. E) tumor emboli.

D ( alpha-blocker therapy. This patient has urodynamic findings consistent with functional bladder neck obstruction. Approximately 50% of these patients respond to alpha-blocker therapy. The urodynamic test showing a high pressure void of 45 cm H2O coupled with increased PVR is most consistent with bladder outlet obstruction. There are four common causes for anatomic outlet obstruction in a woman, a cystocele (bladder prolapse), a prior bladder neck sling, external striated sphincter dyssynergia, and bladder neck obstruction. Another cause of bladder outlet obstruction is an increased tone of the striated muscle pelvic floor (including external urethral sphincter). Given that this patient has not had previous anti-incontinence surgery and that the external sphincter is silent and the bladder neck minimally opens during voiding, the level of obstruction is at the bladder neck. Antimuscarinic therapy is contraindicated in patients with increased PVR. Biofeedback is useful in patients with pelvic floor spasticity, which this patient does not have. Urethral dilation has no effect on the bladder neck. A trial of medical therapy with an alpha-blocker is indicated. If not successful in relieving symptoms and/or decreasing PVR, incision of bladder neck is indicated.Andersson KE, Wein AJ: Pharmacologic management of lower urinary tract storage and emptying failure, Wein AJ, Kavoussi LR, Novick AC, Partin AW, Peters CA (eds): CAMPBELL-WALSH UROLOGY, ed 10. Philadelphia, Elsevier Saunders, 2012, vol 3, chap 68, pp 1998-2000.Cespedes RD, Gerboc JL: Other therapies for storage and emptying failure, Wein AJ, Kavoussi LR, Novick AC, Partin AW, Peters CA (eds): CAMPBELL-WALSH UROLOGY, ed 10. Philadelphia, Elsevier Saunders, 2012, vol 3, chap 75, p 2199. 2013 Adult Obstructive Uropathy, Laparoscopy/Robotics )

A 50-year-old woman has irritative, obstructive voiding symptoms, and urge incontinence. Videourodynamics demonstrates normal bladder capacity, peak flow of 6 ml/sec, peak detrusor voiding pressure of 45 cm H2O, a silent external sphincter, and minimal opening of the bladder neck during voiding. PVR is 150 ml. Examination reveals no cystocele. Cystoscopy is normal. The next step is: A) antimuscarinic therapy. B) biofeedback. C) urethral dilation. D) alpha-blocker therapy. E) transurethral incision of bladder neck.

E ( open bilateral surgical revision. Although short-term success rates (within the first year post treatment) are high with endoscopic treatments, ranging from >60% to 100%, the long term success rates are quite low regardless of approach or endoscopic treatment. The most definitive treatment is open surgical repair.Tal R, Sivan B, Kedar D, et al: Management of benign ureteral strictures following radical cystectomy and urinary diversion for bladder cancer. J UROL 2007;178:538-542.Msezane L, Reynolds WS, Mhapsekar R, et al: Open surgical repair of ureteral strictures and fistulas following radical cystectomy and urinary diversion. J UROL 2008;179:1428-1431. 2013 Adult Urinary Diversion )

A 51-year-old woman develops bilateral hydronephrosis from benign uretero-enteric strictures, two years after undergoing a radical cystectomy with ileal conduit urinary diversion. The most successful treatment is: A) antegrade bilateral endoscopic balloon dilatation. B) antegrade bilateral endoscopic laser incision. C) retrograde bilateral balloon dilatation. D) retrograde bilateral endoscopic laser incision. E) open bilateral surgical revision.

D ( plaque incision and graft. This patient has demonstrated a stable plaque in Peyronie's disease and is ready for surgical correction. Medical therapy with colchicine may be useful in the acute phase and will decrease penile pain, however it has little to no proven benefit over placebo in preventing curvature and is associated with significant GI side effects. It would not be effective in this clinical situation. Topical verapamil has not been adequately evaluated and would most likely not be effective with this degree of curvature. Plication would require multiple plicating sutures and would severely shorten the penis with this degree of curvature. Plication procedures are usually not recommended if the penile curvature is > 60 degrees due to significant shortening of the penis. As he gets excellent erections, a prosthesis is not yet indicated.Jordan GH, McCammon KA: Peyronie's disease, Wein AJ, Kavoussi LR, Novick AC, Partin AW, Peters CA (eds): CAMPBELL-WALSH UROLOGY, ed 10. Philadelphia, Elsevier Saunders, 2012, vol 1, chap 28, pp 802-808. 2013 Adult Sexual Dysfunction, Endocrinopathy, Fertility Problems )

A 52-year-old man in good health has a three year history of dorsal penile curvature. His angulation is 80 degrees and has been stable without pain for over a year. He has good erections with sildenafil but cannot penetrate due to the curvature. The best treatment is: A) oral colchicine. B) topical verapamil. C) corporal plication. D) plaque incision and graft. E) modeling and placement of inflatable penile prosthesis.

B ( selective arterial embolization. Delayed bleeding after percutaneous procedures is almost always secondary to pseudoaneurysms or arteriovenous fistulas. Both can present with delayed and intermittent bleeding. Arteriovenous fistula bleeding is more likely to be continuous compared with pseudoaneurysms. Management is renal angiography during active bleeding (with the aid of an arterial vasodilator such as papaverine if necessary) and highly selective angiographic embolization. Continued conservative therapy would be incorrect in the face of hemodynamic instability after appropriate resuscitative efforts. A tamponade catheter may be used as a temporizing measure if the nephrostomy tract is still present. The image presented demonstrates absence of a nephrostomy tube. An indwelling ureteral stent will not address the ongoing hemorrhage. Emergent exploration may lead to need for nephrectomy and a conservative approach is more appropriate.Rastinehad AR, Andonian S, Siegel DN: Hemorrhagic complications associated with renal surgery, in Smith AD, Badlani GH, Preminger GM, Kavoussi LR (eds): SMITH'S TEXTBOOK OF ENDOUROLOGY, ed 3. Oxford, Blackwell Publishing Ltd, 2012, vol 1, chap 30, p 337. 2013 Adult Calculous Disease )

A 52-year-old man requires six units of packed red blood cells over a 48-hour period five days after a percutaneous nephrolithotomy. He remains hypotensive at 85/50 mmHg. Imaging study is shown. The next step is: A) emergent exploration and repair of kidney. B) selective arterial embolization. C) bedrest, blood transfusion, and close monitoring. D) insertion of a large-bore percutaneous tamponade catheter in nephrostomy tract. E) insertion of an indwelling ureteral stent and urethral catheter decompression.

E ( ophthalmology consult. Elevated prolactin levels can cause infertility and sexual dysfunction by decreasing the production of testosterone. Mildly elevated levels (< 50ng/ml) can be seen with stress and renal insufficiency. Persistently high levels of serum prolactin are suggestive of a pituitary adenoma and need to be evaluated with a careful measurement of visual fields and an MRI of the pituitary. This patient has levels consistent with a pituitary adenoma and serum testosterone, FSH and LH will not help make the diagnosis. While he may benefit from the care of a nephrologist, it will not direct the diagnosis and treatment of a pituitary adenoma. CT is not specific enough for an evaluation of the pituitary and the best choice is an ophthalmology consult for careful visual field testing and an MRI of the pituitary.Sabanegh E, Agarwal A: Male infertility, Wein AJ, Kavoussi LR, Novick AC, Partin AW, Peters CA (eds): CAMPBELL-WALSH UROLOGY, ed 10. Philadelphia, Elsevier Saunders, 2012, vol 1, chap 21, p 639. 2013 Adult Sexual Dysfunction, Endocrinopathy, Fertility Problems )

A 52-year-old man with renal insufficiency has decreased libido and progressive erectile dysfunction. He has a spot serum prolactin level of 220 ng/ml (normal 5-20 ng/ml). The next step is: A) measure serum testosterone. B) measure FSH and LH. C) brain CT. D) nephrology consult. E) ophthalmology consult.

B ( secure exit tubing of the ipsilateral cylinder. A common intraoperative complication with penile prosthesis surgery is crural perforation. If this occurs with insertion of an inflatable device with attached tubing, placing a tunica albuginea closure suture on either side of the exit tubing to keep the cylinder in place has worked sufficiently without requiring a more extensive repair. A more significant perforation injury, including damage to the urethra, would require termination of the procedure. Placement of a malleable prosthesis is not advised as it cannot be secured and will be more likely to erode.Montague DK: Prosthetic surgery for erectile dysfunction, Wein AJ, Kavoussi LR, Novick AC, Partin AW, Peters CA (eds): CAMPBELL-WALSH UROLOGY, ed 10. Philadelphia, Elsevier Saunders, 2012, vol 1, chap 27, p 787. 2013 Adult Sexual Dysfunction, Endocrinopathy, Fertility Problems )

A 53-year-old diabetic man sustains a minor proximal crural perforation during primary implantation of a three-piece inflatable penile prosthesis via a penoscrotal approach. The best management is: A) abort the procedure. B) secure exit tubing of the ipsilateral cylinder. C) extend corporotomy for primary repair. D) place a malleable implant. E) direct closure via perineal approach.

D ( pyeloureterostomy to native ureter. Ureteral obstruction is difficult to manage in transplant kidneys. Long-term nephrostomy tube drainage increases infectious risks especially in immunocompromised patients and requires frequent tube changes. Double-J stents are difficult to place and replace in a retrograde fashion since the common ureteral reimplantation site for a transplant ureter is on the dome of the bladder. Antegrade balloon dilation of all ureteral strictures has poor long-term success. Open exploration is difficult due to the marked fibrous reactive tissue surrounding transplant kidneys. A pyelovesicostomy will freely reflux and may increase the risk of recurrent infections. Drainage through a native ureter anastomosed to the proximal renal pelvis (transplant kidney) is the best option for long-term success. Barry JM, Conlin MJ: Renal transplantation, Wein AJ, Kavoussi LR, Novick AC, Partin AW, Peters CA (eds): CAMPBELL-WALSH UROLOGY, ed 10. Philadelphia, Elsevier Saunders, 2012, vol 2, chap 44, p 1250. 2013 General Obstructive Uropathy, Laparoscopy/Robotics )

A 54-year-old man develops new onset of renal transplant hydronephrosis secondary to a 2 cm proximal ureteral stricture. Ultrasound reveals normal bilateral atrophic kidneys to be in situ. The next step is: A) double-J stent placement. B) chronic nephrostomy drainage. C) antegrade ureteral balloon dilation. D) pyeloureterostomy to native ureter. E) pyelovesicostomy.

A ( thyroid stimulating hormone and T4. Hypothyroidism has been reported in 36% to 46% of patients who took sunitinib in prospective studies. A higher incidence (53% to 85%) has been reported in studies containing both retrospective and prospective data. The mean time to onset of hypothyroidism after initiation of sunitinib therapy ranged from 12 to 50 weeks. The risk of development of hypothyroidism appears to increase with the increasing duration of sunitinib therapy, and the condition is likely reversible once therapy has been discontinued. Baseline thyroid function tests should be performed before the initiation of sunitinib treatment. Because hypothyroidism can develop early in the course of therapy, thyroid function tests should be monitored frequently throughout the duration of treatment. Possible mechanisms for thyroid dysfunction include impaired thyroid hormone synthesis, a destructive thyroiditis preceding the development of hypothyroidism, and increased thyroid hormone clearance. If hypothyroidism is identified, levothyroxine therapy should be promptly initiated. Sunitinib does not affect testosterone, cholesterol, cortisol or liver function.Vetter ML, Kaul S, Iqbal N: Tyrosine kinase inhibitors and the thyroid as both an unintended and an intended target. ENDOCR PRACT 2008;14:618-624. 2013 Adult Neoplasm )

A 54-year-old man with metastatic clear cell RCC is currently receiving sunitinib. During therapy, he should have monitoring of his serum: A) thyroid stimulating hormone and T4. B) testosterone. C) cholesterol. D) cortisol. E) transaminases.

B ( cT1c. Clinical staging is determined by DRE and TRUS, and is not influenced by biopsy location information. This is a common clinical mistake by practicing urologists. The correct clinical stage is cT1c since it is a PSA diagnosed lesion with a normal physical examination and a normal ultrasound. If he had a small nodule on physical examination or consistently present on ultrasound (or other imaging) he would be cT2a. If the nodule was bilateral his clinical stage would be cT2c. If he has a radical prostatectomy, his most likely pathologic stage will be pT2c since his disease is bilateral on biopsy. Loeb S, Carter HB: Early detection, diagnosis, and staging of prostate cancer, Wein AJ, Kavoussi LR, Novick AC, Partin AW, Peters CA (eds): CAMPBELL-WALSH UROLOGY, ed 10. Philadelphia, Elsevier Saunders, 2012, vol 3, chap 99, pp 2767-2768. 2013 Adult Neoplasm )

A 55-year-old asymptomatic man with a serum PSA of 5.0 ng/ml has a normal DRE. TRUS is normal and prostate biopsy reveals Gleason 3+3 adenocarcinoma in two biopsy cores from the left base and two biopsy cores from the right apex. The remaining biopsies show benign prostate tissue. According to the 2010 AJCC TNM classification, the clinical T stage is: A) cT1b. B) cT1c. C) cT2a. D) cT2b. E) cT2c.

C ( urine culture. Pneumaturia, the passage of gas in the urine, may be due to a fistula between the intestine and bladder or due to gas-forming UTI. Common causes of fistula are diverticulitis, carcinoma of the sigmoid colon, and regional enteritis (Crohn disease). Patients with diabetes mellitus may have gas-forming infections, with carbon dioxide formation from the fermentation of high concentrations of sugar in the urine. In the latter situation, the microorganism most commonly responsible for cystitis is E. coli. Approximately 60% of cases of emphysematous cystitis occur in diabetics. In the current case, a urinalysis and urine culture should be performed first. Additional tests can be performed selectively based on the results of urinalysis and urine culture. Culture results showing multiple organisms is suggestive of a colovesical fistula.Gerber GS, Brendler CB: Evaluation of the urologic patient: History, physical examination, and urinalysis, Wein AJ, Kavoussi LR, Novick AC, Partin AW, Peters CA (eds): CAMPBELL-WALSH UROLOGY, ed 10. Philadelphia, Elsevier Saunders, 2012, vol 1, chap 3, p 79. 2013 Adult Infection & Inflammatory Disease )

A 55-year-old diabetic woman has new onset pneumaturia. The next step is: A) pelvic CT scan. B) cystoscopy. C) urine culture. D) cystogram. E) barium enema.

C ( I.V. hypertonic saline at 1 ml/kg/hr. SIADH is caused primarily by release of ADH from brain infections, neoplasms, drugs, or surgery. The treatment of symptomatic hyponatremia from SIADH is fluid restriction and replenishment of sodium. Fluid restriction alone is not adequate in the face of a seizure. The indications for hypertonic saline should be those patients who have neurologic sequelae as noted in this scenario. Correction should raise the sodium no more than 2 mEq/l/hr or 25 mEq/l/48 hours to minimize the risk of central pontine myelinolysis. Lithium and demeclocycline are antagonists of ADH and used for more chronic conditions without concurrent neurologic symptoms. Correction of the sodium deficit is the appropriate next step.Shoskes DA, McMahon AW: Renal physiology and pathophysiology, Wein AJ, Kavoussi LR, Novick AC, Partin AW, Peters CA (eds): CAMPBELL-WALSH UROLOGY, ed 10. Philadelphia, Elsevier Saunders, 2012, vol 2, chap 38, p 1025. 2013 General Physiology, Immunology, & Adrenal )

A 55-year-old man with metastatic RCC to the brain is confused and has one short generalized seizure. His serum sodium is 110 mEq/l. The next step is: A) total fluid restriction to 2 l/day. B) I.V. normal saline at 1 ml/kg/hr. C) I.V. hypertonic saline at 1 ml/kg/hr. D) lithium 1200 mg daily. E) demeclocycline 600 mg daily.

D ( urodynamics. Urethral atrophy results from chronic compression of the corpus spongiosum by the cuff and is the leading cause of urinary incontinence in this setting. However, urodynamic evaluation may reveal involuntary detrusor contractions or decreased bladder compliance. Deactivation will not permit improved sphincter function. Surgical exploration is not indicated if the cause of the incontinence is unrelated to the device (e.g., detrusor overactivity or impaired compliance). Alpha-blockers would not be expected to have any effect on urinary incontinence in this case regardless of the underlying cause. Antimuscarinics would not treat causes of incontinence related to device malfunction. Treatment options for this patient would include downsizing the cuff movement to a more proximal or distal location or the addition of a second cuff placed in tandem.Wessells H, Peterson AC: Surgical procedures for sphincteric incontinence in the male: The artificial genitourinary sphincter and perineal sling procedures, Wein AJ, Kavoussi LR, Novick AC, Partin AW, Peters CA (eds): CAMPBELL-WALSH UROLOGY, ed 10. Philadelphia, Elsevier Saunders, 2012, vol 3, chap 79, pp 2294-2295. 2013 Adult Neurogenic Bladder, Voiding Dysfunction, Incontinence )

A 55-year-old man with post-prostatectomy incontinence underwent placement of an artificial urinary sphincter with good results for three years. He now complains of recurrent incontinence. Examination of the device and cystoscopy suggests normal cycling and no cuff erosion. The next step is: A) deactivate device for a two month trial period. B) alpha-blocker therapy. C) antimuscarinics. D) urodynamics. E) surgical exploration for repair or replacement.

A ( tumor stage. Important prognostic factors for RCC include specific clinical signs or symptoms such as anemia, hematuria and weight loss; tumor-related factors such as grade and histology; and various laboratory findings such as hypercalcemia. Although an integrative approach utilizing nomograms and risk tables combining a variety of factors have proven to be powerful analyses tools, the local tumor stage remains the most important single prognostic factor for RCC. The cephalad extent of tumor thrombus has been associated with outcome, since a thrombus above the diaphragm increases the stage. However, the absolute size of thrombus does not correlate to outcome. Presence or absence of p53 mutation is not prognostic in RCC.Campbell SC, Lane BR: Malignant renal tumors, Wein AJ, Kavoussi LR, Novick AC, Partin AW, Peters CA (eds): CAMPBELL-WALSH UROLOGY, ed 10. Philadelphia, Elsevier Saunders, 2012, vol 2, chap 49, pp 1443-1444.Glazer AA, Novick AC: Long-term followup after surgical treatment for renal cell carcinoma extending into the right atrium. J UROL 1996;155:448.Reissigl A, Janetschek G, Eberle, et al: Renal cell carcinoma extending into the vena cava: Surgical approach, technique and results. BR J UROL 1995;75:138.Swierzewski DJ, Swierzewski MJ, Libertino JA: Radical nephrectomy in patients with renal cell carcinoma with venous, venal caval, and atrial extension. AM J SUR 1994;168:205. 2013 Adult Neoplasm )

A 55-year-old woman undergoes right radical nephrectomy and inferior vena cava thrombectomy for RCC. There is no evidence of metastatic disease. The prognostic factor most predictive of cancer-free survival is: A) tumor stage. B) tumor grade. C) size of caval thrombus. D) tumor size. E) mutant p53 suppressor gene.

C ( ileocolic pouch. Metabolic complications may result from interposition of intestine in the urinary tract. These generally result from altered urinary solute reabsorption by the intestine. This patient has a hyperchloremic metabolic acidosis with associated mild hypokalemia. This is typical of diversion options that utilize ileum and colon. The mechanism is due to the ionized transport of ammonium. The exchange of the weak acid, NH4, for a proton is coupled with the exchange of bicarbonate for chloride. Ammonium chloride is absorbed across the intestinal lumen into the blood in exchange for carbonic acid (i.e., CO2 and water). This is most likely to occur when urine is in prolonged contact with the bowel or if a long bowel segment is used. Electrolyte disorders resulting from jejunal interposition (particularly, proximal segments) in the urinary tract include hyponatremia, hypochloremia, hyperkalemia, azotemia, and acidosis. When stomach is used, a hypochloremic, hypokalemic metabolic alkalosis may ensue. Patients undergoing ureterosigmoidostomy may also experience metabolic acidosis, but this is generally associated with a profound hypokalemia. Dahl DM, McDougal WS: Use of intestinal segments in urinary diversion, Wein AJ, Kavoussi LR, Novick AC, Partin AW, Peters CA (eds): CAMPBELL-WALSH UROLOGY, ed 10. Philadelphia, Elsevier Saunders, 2012, vol 3, chap 85, p 2411. 2013 Adult Urinary Diversion )

A 55-year-old woman underwent cystectomy and urinary diversion three years ago. Her serum electrolytes are: Na 140 mEq/l, K 3.4 mEq/l, Cl 140 mEq/l, CO2 15 mEq/l. Her urinary diversion is most likely a(n): A) ileal conduit. B) jejunal conduit. C) ileocolic pouch. D) cutaneous ureterostomy. E) ureterosigmoidostomy.

A ( repeat prolactin assay. Elevated serum prolactin from a pituitary tumor that causes clinical symptoms such as low libido, infertility and gynecomastia is usually accompanied by a low serum testosterone. A mildly elevated prolactin, especially accompanying a serum testosterone in the normal range, is rarely clinically significant. Because prolactin has high interassay variability, an elevated prolactin should first be verified by repeat testing. With a normal testosterone, LH assay is unhelpful and exogenous testosterone is not indicated. Likewise, a man with mildly elevated prolactin and normal testosterone is unlikely to benefit from bromocriptine, and MRI is unlikely to reveal a clinically significant anatomic pituitary lesion. The most common cause of low libido in a man with a normal physical exam and adequate testosterone is psychological.Sabanegh E, Agarwal A: Male infertility, Wein AJ, Kavoussi LR, Novick AC, Partin AW, Peters CA (eds): CAMPBELL-WALSH UROLOGY, ed 10. Philadelphia, Elsevier Saunders, 2012, vol 1, chap 21, p 639. 2013 Adult Sexual Dysfunction, Endocrinopathy, Fertility Problems )

A 56-year-old man has low libido and a normal physical exam. Morning serum testosterone is 365 ng/dl and prolactin is 48 ng/ml (normal < 20 ng/ml). The next step is: A) repeat prolactin assay. B) serum LH assay. C) pituitary MRI scan. D) testosterone replacement. E) bromocriptine.

C ( urine culture and empiric antibiotic therapy. The clinical presentation of abdominal pain in the area of a continent cutaneous diversion and explosive urinary leakage from the stoma (as opposed to dribbling leakage) is consistent with pouchitis. While asymptomatic bacteruria is common in patients who have undergone continent cutaneous diversion and does not usually require any treatment, symptomatic pouchitis should be treated. The explosive nature of the incontinence is related to hypercontractility of the bowel in the face of infection. Short courses of antibiotics (less than ten days) are not usually successful to clear the infection, presumably due the larger amounts of mucus and sediment seen in intestinal pouches. CT cystogram of the pouch would be useful if rupture of the pouch was suspected. However, clinical symptoms in this setting are not consistent with that diagnosis. Placement of an indwelling urinary catheter, while not harmful in this setting, would have no effect on the pouch infection. Surgical revision of the stoma is unwarranted when incontinence is likely due to infection.McKiernan JM, DeCastro GJ, Benson MC: Cutaneous continent urinary diversion, Wein AJ, Kavoussi LR, Novick AC, Partin AW, Peters CA (eds): CAMPBELL-WALSH UROLOGY, ed 10. Philadelphia, Elsevier Saunders, 2012, vol 3, chap 86, p 2450. 2013 Adult Urinary Diversion )

A 56-year-old woman three years after a radical cystectomy and continent cutaneous diversion has abdominal pain over the pouch and explosive leakage of urine despite regular catheterization. The next step is: A) placement of indwelling urinary catheter. B) pouchoscopy. C) urine culture and empiric antibiotic therapy. D) CT pouch-o-gram. E) surgical revision of the catheterizable stoma.

C ( retropubic midurethral sling. A repeat midurethral synthetic sling is an appropriate option for a patient with recurrent stress urinary incontinence. Cure rates for redo patients appear to be higher for the retropubic compared to the transobturator approach. This is likely due to a higher rate of intrinsic sphincter dysfunction in patients requiring repeat surgery. Urethral bulking agents would be less likely to lead to a long-term resolution of this patient's symptoms. Placement of an autologous fascial sling would also be a viable option for this patient. A retropubic bladder neck suspension is an inferior procedure to a retropubic transvaginal tape or to an autologous sling. Placement of an artificial urinary sphincter would not be indicated for this patient.Dmochowski RR, Padmanabhan P, Scarpero HM: Slings: Autologous, biologic, synthetic, and midurethral, Wein AJ, Kavoussi LR, Novick AC, Partin AW, Peters CA (eds): CAMPBELL-WALSH UROLOGY, ed 10. Philadelphia, Elsevier Saunders, 2012, vol 3, chap 73, p 2164. 2013 Adult Neurogenic Bladder, Voiding Dysfunction, Incontinence )

A 56-year-old woman with recurrent, symptomatic stress urinary incontinence desires treatment. She had a prior midurethral synthetic sling ten years prior that worked well until recently. Examination reveals loss of urine with cough and minimal urethral hypermobility. Urinalysis is normal. The next best step is: A) urethral bulking agent. B) transobturator transvaginal tape sling. C) retropubic midurethral sling. D) retropubic bladder neck suspension. E) artificial urinary sphincter.

C ( CIC. Although the majority of patients do not have difficulties voiding following injection of a bulking agent, when retention does occur, it should be treated with CIC using a small (10- 14 Fr) catheter. Larger catheters, indwelling catheters. or large urethral sounds will push the mucosal blebs apart or cause molding of the bulking agent around the catheter. Suprapubic cystotomy can be used if long term catheterization is needed, although this is very rare. Herschorn S: Injection therapy for urinary incontinence, Wein AJ, Kavoussi LR, Novick AC, Partin AW, Peters CA (eds): CAMPBELL-WALSH UROLOGY, ed 10. Philadelphia, Elsevier Saunders, 2012, vol 3, chap 74, p 2172. 2013 Adult Neurogenic Bladder, Voiding Dysfunction, Incontinence )

A 71-year-old woman has difficulty voiding two hours following injection of a transurethral bulking agent. Residual urine volume is 400 ml. The next step is: A) observation. B) alpha-blockers. C) CIC. D) indwelling urethral catheter. E) suprapubic cystostomy.

C ( Aa 0, C -7, Ap -3, stage 2. The POP-Q (pelvic organ prolapse quantification) is a classification system used to quantify the degree of pelvic organ prolapse (POP). Nine points are measured in relation to the hymenal ring, which is designated as 0 (zero). Any points located above the hymenal ring are given a negative value corresponding to the number of centimeters (cm) the point is above the hymen whereas points distal to the hymen are given positive values. Aa and Ap correspond to the point three cm up on the anterior and posterior walls respectively and will be -3 if there is no POP. Ba and Bp correspond to the distal-most aspect of the anterior and posterior walls respectively. The well-supported vault (C) should be at about -7 to -9. This patient has a cystocele and no posterior or apical prolapse. The distal-most portion of her anterior vagina is at 0 the level of the hymenal ring. Ap which corresponds to approximately the level of the bladder neck is -3. The vault is supported at -7. This is stage 2 which is defined as the most distal point of the POP being between +1 and -1 or within 1 cm of the hymen. Other points of measurements in POP-Q include the genital hiatus (GH) perineal body (PB) and total vaginal length (TVH). Kobashi KC: Evaluation of patients with urinary incontinence and pelvic prolapse Wein AJ Kavoussi LR Novick AC Partin AW Peters CA (eds): CAMPBELL-WALSH UROLOGY ed 10. Philadelphia Elsevier Saunders Elsevier Saunders 2012 vol 3 chap 64 pp 1900-1901. 2013 Adult Neurogenic Bladder, Voiding Dysfunction, Incontinence )

A 57-year-old woman reports a vaginal bulge ten years after hysterectomy. On examination, she is found to have anterior prolapse, with the most distal point noted at her hymenal ring. Her vault and posterior wall show no prolapse. The POP-Q points consistent with her exam are: A) Aa -1, C 0, Ap -3, stage 1. B) Aa -1, C 0, Ap -1, stage 1. C) Aa 0, C -7, Ap -3, stage 2. D) Aa 0, C -7, Ap -1, stage 2. E) Aa +1, C +2, Ap -3, stage 3.

A ( observation. Long-term use of NSAIDs such as ibuprofen may result in an acute interstitial nephritis (AIN). Clinically this presents as an acute renal failure with WBC casts. Proteinuria is common. Unlike other types of drug induced AIN; fever, and eosinophilia and eosinophiluria are uncommon. Lupus nephritis is characterized by red cell casts. Antibiotics are not indicated for AIN and culture will be negative. Immunosuppressants, such as steroids and cyclosporine, have no role in the therapy of AIN. Observation is appropriate as discontinuation of the offending agent, in this case, the NSAID, will likely result in resolution of the AIN. Renal biopsy is over aggressive in this patient unless he fails a course of observation. Goldfarb DA, Poggio ED: Etiology, pathogenesis, and management of renal failure, Wein AJ, Kavoussi LR, Novick AC, Partin AW, Peters CA (eds): CAMPBELL-WALSH UROLOGY, ed 10. Philadelphia, Elsevier Saunders, 2012, vol 2, chap 43, p 1196. 2013 Adult Fluid & Electrolyte,Transplantation, Hypertension, Vasc Disease, Nephrology )

A 58-year-old diabetic man with lupus recently discontinued a two week course of ibuprofen for a shoulder injury. Serum creatinine is elevated from a baseline of 0.9 to 2.6 mg/dl. Urinalysis reveals moderate proteinuria and WBC casts without eosinophiluria. Renal ultrasound demonstrates no hydronephrosis. The next step is: A) observation. B) urine culture and antibiotics. C) prednisone. D) cyclosporine. E) percutaneous renal biopsy.

A ( frequency-volume chart. According to the recently updated guidelines on management of BPH by the AUA Practice Guideline Committee, recommended components of the diagnostic algorithm for routine evaluation include history, assessment of LUTS with an AUA Symptom Score, physical examination including DRE and urinalysis. A frequency-volume chart should be obtained if nocturia is a predominant symptom. Urodynamics, cystoscopy and serum creatinine are not required as part of the initial evaluation. McVary KT, Roehrborn CG, Avins AL, et al: Update on AUA guideline on the management of benign prostatic hyperplasia. J UROL 2011;185:1795. 2013 Adult Neurogenic Bladder, Voiding Dysfunction, Incontinence )

A 58-year-old man has frequency and bothersome nocturia with an AUA Symptom Score of 16. History and physical exam are normal, Urinalysis is negative. The next step is: A) frequency-volume chart. B) serum creatinine. C) uroflowmetry. D) cystoscopy. E) pressure-flow study.

E ( repeat TRUS biopsy. The PCA3 urine test is designed to detect the mRNA signal from the DD3 gene which is specific to prostate cancer. This test has now been validated in multiple prospective studies the largest of which was the REDUCE clinical trial. In this study PCA3 scores were measured in 1,072 subjects with a negative prior TRUS biopsy and the results were more closely correlated to the results of the second biopsy than PSA alone or free to total PSA ratio. The higher the level of the PCA3 score the higher the risk of a positive biopsy and a score of under 35 is considered low risk. Therefore a repeat prostate biopsy is warranted in this patient. PCA3 is now FDA-approved for the assessment of patients with a prior negative prostate biopsy. PCA3 is not affected by infection, prostate volume, or the use of 5-alpha reductase inhibitors.Aubin SM, Reid J, Sarno MJ, et al: PCA3 molecular urine test for predicting repeat prostate biopsy outcome in populations at risk: Validation in the placebo arm of the dutasteride REDUCE trial. J UROL 2010;184;1947-1952. 2013 Adult Neoplasm )

A 59-year-old man undergoes TRUS biopsy with a PSA of 4.5 ng/ml and a normal DRE. The biopsy reveals BPH and nine months later his PSA has risen to 5 ng/ml and he undergoes a PCA3 urine test. The PCA3 score is 99 (low risk < 35). The next step is: A) 5-alpha reductase inhibitor. B) one month of ciprofloxacin and repeat PCA3. C) free to total PSA ratio. D) prostate MRI. E) repeat TRUS biopsy.

B ( oral ciprofloxacin. This patient underwent prosthetic joint replacement less than two years prior to a stone manipulation procedure - SWL. According to the AUA Antimicrobial Prophylaxis Best Policy Statement, the antimicrobial prophylaxis should be either single dose oral fluoroquinolone or intravenous ampicillin-gentamicin combination.Wolf JS Jr, Bennett CJ, Dmochowski RR, et al: Best practice policy statement on urological surgery antimicrobial prophylaxis. UROLOGICAL SURGERY ANTIMICROBIAL PROPHYLAXIS BEST PRACTICE STATEMENT. American Urological Association Education and Research, Inc, 2008. http://www.auanet.org/content/guidelines-and-quality-care/clinical-guidelines/main-reports/antibiotic_prophylaxis.pdf 2013 General Infection & Inflammatory Disease )

A 60-year-old man who underwent left total hip replacement one year ago is about to undergo SWL of a 1 cm right renal pelvic stone. Appropriate antimicrobial prophylaxis is: A) none. B) oral ciprofloxacin. C) oral trimethoprim-sulfamethoxazole. D) I.V. ceftriaxone. E) I.V. vancomycin.

A ( right gastroepiploic. An omental flap should be preferentially based on the right gastroepiploic artery. The pedicle is mobilized off the stomach from the left. This will result in a well-vascularized and sufficiently long flap based on the right gastroepiploic. The right gastroepiploic is a larger vessel than the left, and its origin is somewhat caudal as compared to the left allowing a shorter course into the deep pelvis.Rovner ES: Urinary tract fistulae, Wein AJ, Kavoussi LR, Novick AC, Partin AW, Peters CA (eds): CAMPBELL-WALSH UROLOGY, ed 10. Philadelphia, Elsevier Saunders, 2012, vol 3, chap 77, pp 2237-2241. 2013 Adult Congenital Anomalies, Embryology, Anatomy )

A vascular pedicle of the omentum is preferentially based on which artery when using an omental flap for repair of a vesicovaginal fistula: A) right gastroepiploic. B) left gastroepiploic. C) superior mesenteric. D) gastric. E) splenic.

A ( no further operative intervention. The unexpected finding of renal lymphoma at the time of renal cortical tumor surgery is rare. Ninety percent of these cases are not primary renal lymphoma but rather systemic lymphoma with renal manifestation. Non-Hodgkin's lymphomas are the most common subtype. Multifocal masses, bilaterality and regional lymphadenopathy are all more common in renal lymphoma than in renal cortical tumors. In this patient the presence of diffuse renal infiltration by lymphoma will make post-operative systemic therapy necessary. In the setting of compromised renal function every attempt should be made to spare the remaining nephron mass in preparation for systemic chemotherapy. Therefore further surgical intervention is not warranted and completion of the operation and subsequent postoperative discussion regarding systemic therapy is the most logical next step.Campbell SC, Lane BR: Malignant renal tumors, Wein AJ, Kavoussi LR, Novick AC, Partin AW, Peters CA (eds): CAMPBELL-WALSH UROLOGY, ed 10. Philadelphia, Elsevier Saunders, 2012, vol 2, chap 49, pp 1471-1472. 2013 Adult Neoplasm )

A 61-year-old man with a serum creatinine of 1.7 mg/dl has a 5 cm upper pole left renal mass. He undergoes left partial nephrectomy. After complete gross resection of the mass frozen section reveals lymphoma with diffuse infiltration of normal renal parenchyma by lymphoma at the margins. His cold ischemic time was 18 minutes. The next step is: A) no further operative intervention. B) re-excision of tumor bed. C) cryotherapy of margin. D) biopsy of contralateral kidney. E) radical nephrectomy.

C ( between the pons and sacral spinal cord. Multiple sclerosis may involve the central and/or peripheral nervous systems. Depending on the location, level and extent of demyelination, a variety of urodynamic patterns may result. Pelvic floor EMG activity in this individual is increased during voiding which suggests striated sphincter dyssynergia, a urodynamic finding that exists only with neurological lesions between the pons and the sacral spinal cord. Lesions at or distal to the sacral spinal cord would likely result in detrusor areflexia and lesions above the pons result in detrusor overactivity with synergistic activity of the proximal and distal sphincter mechanisms. Wein AJ, Dmochowski RR: Neuromuscular dysfunction of the lower urinary tract, Wein AJ, Kavoussi LR, Novick AC, Partin AW, Peters CA (eds): CAMPBELL-WALSH UROLOGY, ed 10. Philadelphia, Elsevier Saunders, 2012, vol 3, chap 65, p 1920. 2013 General Neurogenic Bladder, Voiding Dysfunction, Incontinence )

A 62-year-old woman with multiple sclerosis has persistent urinary urgency and frequency. Pressure flow urodynamics reveal detrusor overactivity during bladder filling that reproduces her symptoms as well as increased pelvic floor EMG activity during voiding. An MRI scan will most likely reveal evidence of demyelination: A) of the cerebral cortex. B) of the cerebellum. C) between the pons and sacral spinal cord. D) between the conus medullaris and the cauda equina. E) between the sacral spinal cord and the bladder.

E ( temsirolimus. Temsirolimus is an inhibitor of the mammalian target of rapamycin (mTOR) kinase. This is a component of intracellular signaling pathways involved in growth/proliferation of cells. The medication suppresses angiogenesis and is given as a weekly I.V. infusion. Patients with three or more poor risk factors respond better to mTOR inhibitors than other currently used medications. Risk factors include: serum LDH > 1.5 times upper limit of normal, hemoglobin below lower limit of normal, serum calcium level of more than 10 mg/dl, time from initial diagnosis of RCC to randomization of less than one year, Karnofsky performance of 60 or 70, or metastases in multiple organs. This patient is considered poor risk and patients who received this regimen were 27% more likely to survive when compared to patients receiving interferon alpha alone. This medication is FDA-approved for advanced RCC. In addition bevacizumab, sunitinib and sorafenib have not been proven to have a survival benefit in this high risk subset of patients. Interferon has not demonstrated a consistent survival advantage in the management of metastatic RCC.Hudes G, Carducci M, Tomczak P, et al: Temsirolimus, interferon alpha, or both for advanced renal-cell carcinoma. NEJM 2007;356:2271-2281. 2013 Adult Neoplasm )

A 63-year-old woman with metastatic clear cell RCC and a poor performance status has a serum calcium of 11 mg/dl and a hemoglobin of 8 g/dl. The most appropriate treatment is: A) interferon. B) bevacizumab. C) sunitinib. D) sorafenib. E) temsirolimus.

B ( finasteride. Finasteride is an effective option for the management of gross hematuria after TURP for BPH. None of the other listed treatments (e.g., tamsulosin and bicalutamide) have efficacy or have been evaluated in this setting. It is known that one of the early effects of finasteride is the intraprostatic suppression of vascular endothelial growth factor. Clinically, finasteride has been shown to effectively treat post-prostatectomy hematuria, especially in the presence of friable prostate tissue. If hematuria does not resolve with this therapy then evaluation of the upper urinary tract should be considered to rule it out as the source of bleeding. Prolonged antibiotics would only be indicated in the setting of UTI suspected to be of prostatic origin.McVary KT, Roehrborn CG, Avins AL, et al: Update on AUA guideline on the management of benign prostatic hyperplasia. J UROL 2011;185:1798.McNicholas TA, Kirby RS, Lepor H: Evaluation and nonsurgical management of benign prostatic hyperplasia, Wein AJ, Kavoussi LR, Novick AC, Partin AW, Peters CA (eds): CAMPBELL-WALSH UROLOGY, ed 10. Philadelphia, Elsevier Saunders, 2012, vol 3, chap 92, p 2638. 2013 Adult Neurogenic Bladder, Voiding Dysfunction, Incontinence )

A 64-year-old man had a TURP six months ago and has an AUA Symptom Score of 5. He has persistent gross hematuria requiring cystoscopy and clot evacuation. Friable prostatic tissue was noted during cystoscopy. He does not wish to undergo further surgical treatment. The next step is: A) tamsulosin. B) finasteride. C) tamsulosin and finasteride. D) bicalutamide. E) antibiotics for one month.

A ( right nephrectomy. The diagnostic findings on ultrasound and renal scan are sufficient to confirm multicystic dysplastic kidney. The absolute indications for removal are few, including hypertension, solid mass, or significant respiratory or gastrointestinal compromise. Aspiration is an option in a premature neonate or a toxic child, but at six weeks of age, nephrectomy is preferred. The lack of renal function eliminates a salvageable UPJ obstruction. Therefore cystoscopy and retrograde pyelogram is unnecessary, and pyeloplasty is not an option.Pope JC IV: Renal dysgenesis and cystic disease of the kidney, Wein AJ, Kavoussi LR, Novick AC, Partin AW, Peters CA (eds): CAMPBELL-WALSH UROLOGY, ed 10. Philadelphia, Elsevier Saunders, 2012, vol 4, chap 118, p 3182. 2013 Pediatric Obstructive Uropathy, Laparoscopy/Robotics )

A six-week-old boy has a large, palpable, right abdominal mass. Ultrasonography demonstrates a fluid-filled mass with multiple cystic lucencies. A renal scan shows no uptake of the right kidney and a normal left kidney. He has moderate respiratory distress and an elevated right hemidiaphragm. The most appropriate therapy is: A) right nephrectomy. B) percutaneous aspiration of cysts. C) cystoscopy and right retrograde pyelogram. D) right pyeloplasty. E) observation.

C ( dilutional hyponatremia. The TUR syndrome consists of mental confusion, nausea, vomiting, hypertension, bradycardia and visual disturbance. It is secondary to a dilutional hyponatremia from free water overload. Up to 20 ml/minute are absorbed during a resection. The risk increases with the size of the prostate and the length of the resection. Patients become symptomatic when the serum sodium concentration reaches 125 mEq/l. Ammonium intoxication has been suggested when glycine has been utilized as an irrigant. Alternative irrigants such as sorbitol, glycine and mannitol are isoosmolar and non-hemolytic, whereas water is a cheap irrigant but is hypoosmolar. Sorbitol is an inert sugar and would not lead to hyperglycemia. Sepsis in this scenario would usually be manifested with fever, hypotension and tachycardia. Hypovolemia would result in tachycardia and hypotension.Fitzpatrick JM: Minimally invasive and endoscopic management of benign prostatic hyperplasia, Wein AJ, Kavoussi LR, Novick AC, Partin AW, Peters CA (eds): CAMPBELL-WALSH UROLOGY, ed 10. Philadelphia, Elsevier Saunders, 2012, vol 3, chap 93, pp 2683-2684. 2013 Adult Fluid & Electrolyte,Transplantation, Hypertension, Vasc Disease, Nephrology )

A 64-year-old man undergoes a TURP using glycine irrigation. Vital signs are stable throughout the procedure but at the end of the procedure he is confused and nauseated. The most likely problem is: A) sepsis. B) hyperglycemia. C) dilutional hyponatremia. D) hemolysis. E) hypovolemia.

A ( a mitotic rate > 5/hpf, absence of clear cytoplasm, and sinusoidal invasion. Distinguishing between adrenal adenoma and carcinoma can be difficult, and large-sized adrenal lesions do not always biologically behave as carcinomas. The Weiss criteria, which includes: high mitotic rate (> 5/hpf), atypical mitoses, venous invasion, high nuclear grade (Fuhrman 3-4), absence of cells with clear cytoplasm (< 25% of cells), a diffuse growth pattern (more than one third of tumor), necrosis, sinusoidal invasion, and capsular invasion are often used to distinguish between benign and malignant potential. Three or more of these features are needed for the diagnosis of carcinoma.Kutikov A, Crispen PL, Uzzo RG: Pathophysiology, evaluation, and medical management of adrenal disorders, Wein AJ, Kavoussi LR, Novick AC, Partin AW, Peters CA (eds): CAMPBELL-WALSH UROLOGY, ed 10. Philadelphia, Elsevier Saunders, 2012, vol 2, chap 57, pp 1715-1719. 2013 General Physiology, Immunology, & Adrenal )

A 64-year-old woman undergoes left laparoscopic adrenalectomy for a non-functional 5.5 cm adrenal mass. Three pathologic criteria suggestive of the diagnosis of carcinoma include: A) a mitotic rate > 5/hpf, absence of clear cytoplasm, and sinusoidal invasion. B) atypical mitoses, lack of necrosis, and venous invasion. C) focused growth pattern, high nuclear grade, and most cells have clear cytoplasm. D) capsular exclusion, sinusoidal exclusion, and atypical mitoses. E) mitotic rate < 5/hpf, diffuse growth pattern, and capsular invasion.

C ( 12-core TRUS-guided prostate biopsy. Active surveillance is an appropriate management strategy for selected men with localized prostate cancer. Patients who elect this approach should have favorable risk tumors, usually consisting of low-volume Gleason sum 6 or less disease. Active surveillance protocols are characterized by close monitoring of PSA kinetics and serial biopsy. Klotz et al from Toronto have suggested that PSAs be measured quarterly for the first two years after diagnosis and then twice annually after that, assuming the PSA is stable. A PSA doubling time of less than three years is considered to be an indication for intervention. They have recommended a 10-12 core biopsy at one year after original diagnosis and then every three to five years after that until age 80. If any Gleason sum 7 or higher disease is noted, this is considered to be an indication for intervention. In this case, the patient is one year removed from his initial diagnosis and a follow-up biopsy is indicated. Reassessment of PSA in three or six months is inappropriate if repeat biopsy is not performed. Saturation biopsy is not indicated in this setting. MRI can be useful in the follow-up for patients, but is not a critical element of active surveillance protocols. Klotz L: Active surveillance for prostate cancer: Patient selection and management. AUA UPDATE SERIES 2008 vol 27, lesson 33, p 326. 2013 Adult Neoplasm )

A 66-year-old man has a PSA of 4.1 ng/ml and is diagnosed with a Gleason sum 6 prostate cancer involving less than 5% of a single core from a 12-core biopsy. He elects active surveillance. One year later, his PSA is 4.5 ng/ml. The next step is: A) recheck PSA in three months. B) recheck PSA in six months. C) 12-core TRUS-guided prostate biopsy. D) 24-core TRUS-guided saturation biopsy. E) MRI.

D ( T2aNxM1. According to the 2010 TNM AJCC staging, renal cancers greater than 7 cm and less than or equal to 10 cm are now categorized as pT2a. Lesions greater than 10 cm are pT2b. Adrenal gland involvement depends on whether there is contiguous involvement (T4) or non-contiguous involvement (M1). The M1 designation is true even if the adrenal gland is on the ipsilateral side as the nephrectomy. When there are no pathologic lymph nodes available, the pathologic staging is designated as NX.Edge SB, Byrd D, Compton C, Fritz A: AJCC Staging Manual, ed 7. New York, Springer-Verlag, 2010, chap 43, pp 479-490. 2013 Adult Neoplasm )

A 66-year-old man undergoes a radical nephrectomy with adrenalectomy for an 8 cm upper pole RCC within the kidney. There is a focus of non-contiguous, metastatic RCC in the adrenal gland. No lymph nodes were removed. According to the 2010 TNM AJCC classification, pathologic stage is: A) T1bN0Mx. B) T1bNxM1. C) T2aNxM0. D) T2aNxM1. E) T4NxM0.

C ( circumferential excision including excision of anterior vaginal wall. In female urethral cancers, treatment recommendations are primarily dependent on tumor location and clinical stage. Local excision may be sufficient for the relatively uncommon small, superficial, distal urethral tumors, and can result in excellent functional results. For more proximal and advanced urethral tumors, a more aggressive approach is warranted. Small, exophytic, superficial tumors arising from the urethral meatus or anterior urethra (as in this case) may be surgically treated with circumferential excision of the distal urethra including a portion of the anterior vaginal wall. The distal third of the urethra may be excised while still maintaining urinary continence. Tumors in the distal urethra tend to be low stage, and cure rates of 70% to 90% have been achieved with local excision alone. 5-FU cream has been utilized in the treatment of carcinoma-in situ of the penis, but does not have a defined role in female urethral cancers. Although XRT has been effectively used for female urethral cancers, the addition of prophylactic lymphadenectomy in this choice makes it incorrect. Recommendations for performing groin dissection have been made only for patients who present with positive inguinal or pelvic lymphadenopathy without distant metastasis, or patients who develop adenopathy during surveillance. Anterior pelvic exenteration is employed for patients with proximal urethral cancers often as part of a multimodal approach including chemotherapy and XRT.Sharp DS, Angermeier KW: Surgery of penile and urethral carcinoma, Wein AJ, Kavoussi LR, Novick AC, Partin AW, Peters CA (eds): CAMPBELL-WALSH UROLOGY, ed 10. Philadelphia, Elsevier Saunders, 2012, vol 1, chap 35, p 934. 2013 Adult Neoplasm )

A 66-year-old woman has a polypoid mass at her urethral meatus. Office biopsy demonstrates a non-invasive high-grade squamous cell carcinoma. Radiologic staging reveals no evidence of regional or distant disease. The next step is: A) topical 5-FU cream. B) laser fulguration. C) circumferential excision including excision of anterior vaginal wall. D) XRT with ilioinguinal lymphadenectomy. E) anterior pelvic exenteration with pelvic lymphadenectomy.

E ( left radical nephroureterectomy. Standard therapy for patients with upper tract urothelial carcinoma involving the proximal ureter is nephroureterectomy. Endoscopic treatment of patients with upper tract urothelial carcinoma is generally recommended in those patients with a solitary kidney, bilateral disease, renal dysfunction, or significant intercurrent illness that precludes a major abdominal procedure. Endoscopic management may also be appropriate in selected patients with small, low-grade lesions in the presence of a normal contralateral kidney. However, most series suggest that recurrence is likely even with frequent reinspection and that progression to invasive disease occurs in a significant number of patients, depending on the stage and grade of the initial tumor. Open ureteral resection risks tumor spillage as well as recurrence. This patient has a normal contralateral right kidney. A recent large, multi-institutional examination of nephrouterectomy and ipsilateral lymph node dissection did not reveal any survival advantage for patients treated with node dissection with earlier stage upper tract malignancy as is the case with this patient. However, in patients with T2-T4 primary tumors, pathologic N0 patients did have a longer cancer-specific survival than pathologic Nx patients.Roscigno M, Shariat SF, Margulis V, et al: Impact of lymph node dissection on cancer specific survival in patients with upper tract urothelial carcinoma treated with radical nephroureterectomy. J UROL 2009;181:2482-2489.Sagalowsky AI, Jarrett TW, Flanigan RC: Urothelial tumors of the upper urinary tract and ureter, Wein AJ, Kavoussi LR, Novick AC, Partin AW, Peters CA (eds): CAMPBELL-WALSH UROLOGY, ed 10. Philadelphia, Elsevier Saunders, 2012, vol 2, chap 53, p 1516. 2013 Adult Neoplasm )

A 68-year-old diabetic man has a 1 cm left proximal ureteral non-invasive, high grade urothelial carcinoma. His serum creatinine is 0.9 mg/dl. Abdominal CT scan is otherwise normal. The next step is: A) ureteroscopic tumor ablation, stent placement, and intravesical BCG instillation. B) percutaneous resection and antegrade BCG. C) left partial ureterectomy. D) left ureterectomy with Boari flap. E) left radical nephroureterectomy.

A ( observation and support. This patient has metastatic calciphylaxis of the vessels (dry gangrene) in his digits and glans. This is seen in patients with ESRD and is associated with a high mortality rate. Most of these patients will die of their renal disease within six months. The characteristic lesions show microscopic calcification in the arterioles and capillaries which leads to dry necrosis. Antibiotics are not helpful and biopsies can be harmful as these lesions will not heal well due to their poor blood supply. Removal of the prosthesis would only be indicated if there was subsequent erosion or penile infection. This patient should continue with observation, support and wound care as needed.Jacobson HA: Penile calciphylaxis. UROL 2002;60,344.Frehally J: Bone and mineral metabolism in CKD: Clinical manifestations of renal osteodystrophy, in COMPREHENSIVE CLINICAL NEPHROLOGY, ed 3. Philadelphia, Mosby Elsevier, 2007, chap 74, pp 906-909. 2013 Adult Sexual Dysfunction, Endocrinopathy, Fertility Problems )

A 68-year-old man with ESRD has been on dialysis for ten years. He has a malleable penile prosthesis and his caregivers notice a firm, black, necrotic lesion on his glans. He has similar lesions on his fingers and toes. The best treatment is: A) observation and support. B) oral antibiotics for six weeks. C) I.V. antibiotics for six weeks. D) biopsy of lesion. E) removal of prosthesis.

E ( oxybutynin. BCG cystitis is a common side effect of BCG therapy. A possible bacterial infection should also always be considered. Quinolone antibiotics are not indicated with a negative urine culture and may have a negative effect on BCG therapy as it is partially tuberculocidal. Similarly, antitubercular medications may abrogate the effectiveness of BCG and add the risk of hepatotoxicity. A decrease in dose intensity may hamper therapeutic efficacy. The patient's symptoms are mild to moderate and are best treated symptomatically with an anti-spasmodic agent during the course of therapy.Jones JS, Larchian WA: Non-muscle-invasive bladder cancer (Ta, T1, and CIS), Wein AJ, Kavoussi LR, Novick AC, Partin AW, Peters CA (eds): CAMPBELL-WALSH UROLOGY, ed 10. Philadelphia, Elsevier Saunders, 2012, vol 3, chap 81, p 2345.van der Meijden AD PM, Klingeren BV, Steerenberg PA: The possible influence of antibiotics on results of bacillus Calmette-Guerin intravesical therapy for superficial bladder cancer. J UROL 1992;147:596-600. 2013 Adult Neoplasm )

A 68-year-old woman has nocturia x3, persistent suprapubic pain, urgency and daytime frequency after her third BCG instillation for recurrent stage Ta bladder cancer. A urinalysis reveals 5-10 RBC and 10-20 WBC/hpf. A urine culture is negative. The next step is: A) space remaining treatments two weeks apart. B) ciprofloxacin. C) decrease weekly intravesical dose of BCG by 50 percent. D) isoniazid therapy with BCG treatments. E) oxybutynin.

E ( neoadjuvant cisplatin, ifosfamide, and paclitaxel. This patient has a T2 (invasion into the corpus spongiosum or cavernosum) N3 (palpable fixed inguinal lymph nodes or nodal mas, either bilateral or unilateral) disease. In patients with unresectable primary tumors or bulky regional lymph node metastases, neoadjuvant treatment with a cisplatin-containing regimen is the most effective treatment modality and may allow curative resection. A phase 2 study using four courses of neoadjuvant paclitaxel, ifosfamide and cisplatin chemotherapy for TxN2-3 disease followed by bilateral inguinal lymph node dissections, and unilateral or bilateral pelvic lymph node dissections revealed excellent response with an objective response rate of 55% and complete pathologic response rate of 10%, toxicity was acceptable with no treatment-related deaths. This treatment is superior to single agent chemotherapy and has less toxicity than the previous multi-agent chemotherapeutic regimen of cisplatinum, bleomycin, and methotrexate. The optimal chemotherapy regimen however has yet to be determined. In this patient with bilateral bulky fixed nodes not responding to antibiotics, a needle biopsy of the lymph nodes could be considered for pathologic diagnosis. However, neither pelvic lymph node biopsy, sentinel inguinal lymph node biopsy, nor bilateral pelvic inguinal and inguinal lymph node dissection would be curative and would predispose the patient to non-healing surgical incision sites. Similarly, XRT to the inguinal nodes would not be curative for this extensive disease.Trabulsi EJ, Hoffman-Censits J: Chemotherapy for penile and urethral carcinoma. UROL CLIN N AM 2010;37:467-374.Pagliaro LC, Williams DL, Daliani D, et al: Neoadjuvant paclitaxel, ifosfamide, and cisplatin chemotherapy for metastatic penile cancer: A phase II study. J CLIN ONC 2010;28:3851-3857. 2013 Adult Neoplasm )

A 71-year-old healthy, uncircumcised man has a 4 cm penile tumor and undergoes partial penectomy. Pathology reveals high grade squamous cell carcinoma invading the corpora cavernosum with negative surgical margins. After four weeks of antibiotic therapy, staging evaluation reveals bilateral bulky fixed, inguinal adenopathy and bilateral pelvic adenopathy. The next step is: A) pelvic lymph node biopsy. B) sentinel inguinal lymph node biopsy. C) XRT to inguinal nodes. D) bilateral pelvic and inguinal lymph node dissection. E) neoadjuvant cisplatin, ifosfamide, and paclitaxel.

C ( random bladder biopsies. The urinary FISH test identifies intranuclear chromosomal abnormalities that have been associated with bladder cancer. Specifically it detects aneuploidy for chromosome 3,7 and 17 and homozygous loss of chromosome 9p21. This test is currently FDA approved for the evaluation of microscopic hematuria and bladder cancer. In 2007 Yoder and colleagues reported that 35/56 (62.5%) patients with prior urothelial carcinoma who had a normal evaluation by cystoscopy and a positive FISH subsequently were detected to have recurrent disease. The appropriate workup of this patient population remains highly variable, however random bladder biopsies are considered the standard of care for patients with a positive urine cytology and negative cystoscopy and should be regarded as a minimum evaluation in this high risk patient. The yield of upper tract endoscopy and cytology in the setting of a normal CT urogram is low and should be discouraged as an initial diagnostic maneuver. Because the patient has a history of bladder cancer, the most likely site of recurrence is in the bladder. Repeat FISH will not add anything to the evaluation, whether positive or negative. Wood DP: Urothelial tumors of the bladder, Wein AJ, Kavoussi LR, Novick AC, Partin AW, Peters CA (eds): CAMPBELL-WALSH UROLOGY, ed 10. Philadelphia, Elsevier Saunders, 2012, vol 3, chap 80, pp 2327-2328.Yoder BJ, Skacel M, Hedgepeth R, et al: Reflex UroVysion testing of bladder cancer surveillance patients with equivocal or negative urine cytology: A prospective study with focus on the natural history of anticipatory positive findings. AM J CLIN PATHOL 2007;127:295-301. 2013 Adult Neoplasm )

A 71-year-old man previously treated for CIS of the bladder has a positive fluorescence in-situ hybridization (FISH) urine test. Cytology, CT urogram, and cystoscopy are normal. The next step is: A) observation. B) repeat FISH test. C) random bladder biopsies. D) bilateral upper tract cytology. E) bilateral ureteroscopy.

D ( instillation of 5% formalin. Formaldehyde is a 37% solution of formaldehyde gas dissolved in water and should not be used intravesically. Formalin solution is made up of 1-10% formaldehyde diluted with normal saline and has been given in bladder instillations to control hemorrhage from advanced bladder tumors or radiation cystitis. Formalin solution is exceedingly irritating to the bladder and, thus, requires general or regional anesthesia. Because a 10% formalin solution may cause fibrosis and obstruction of the ureteral orifices, formalin instillation should begin with a 1% solution and be repeated with a 5% and then a 10% solution, if necessary. Many people begin with a 5% solution if other measures (i.e., silver nitrate and 1% alum) have failed. A cystogram should be performed before instillation to rule out vesicoureteral reflux. If reflux is present, Fogarty catheters should be passed up both ureters, and the patient should be tilted into the head-up position to protect the upper tracts from the toxic effects of formalin. Selective internal iliac arterial embolization is more invasive and should be reserved for patients that fail formalin instillation.Smit SG, Heyns CF: Management of radiation cystitis. NAT REV UROL 2010;7:206-214. 2013 Adult Neoplasm )

A 75-year-old man has severe bleeding from radiation cystitis requiring transfusion. Cystogram reveals no reflux. Previous therapeutic measures have failed including fulguration, clot evacuation, and irrigations with silver nitrate and 1% alum. The next step in management is: A) ileal loop urinary diversion. B) instillation of 10% formalin. C) instillation of 5% formaldehyde. D) instillation of 5% formalin. E) internal iliac artery embolization.

B ( vaginal pessary. Complete uterine prolapse (procidentia) can cause bilateral ureteral obstruction. Correction of the prolapse causes relief of the ureteral obstruction. In this case, a vaginal pessary to prevent prolapse is the best initial management to allow improvement in renal function and uninterrupted anticoagulant treatment of her phlebitis. Later hysterectomy may be indicated. Neither anterior vesicourethropexy nor anterior vaginal repair will correct uterine prolapse. Cystoscopic manipulation of the upper tracts through an infected bladder will, all too often, lead to pyelonephritis. All patients with advanced uterine prolapse should have upper tract imaging by ultrasound or pyelography. Lesser degrees of uterine procidentia carry little risk of hydronephrosis (2% in one study).Payne CK: Conservative management of urinary incontinence: Behavioral and pelvic floor therapy, urethral and pelvic devices, Wein AJ, Kavoussi LR, Novick AC, Partin AW, Peters CA (eds): CAMPBELL-WALSH UROLOGY, ed 10. Philadelphia, Elsevier Saunders, 2012, vol 3, chap 69, pp 2017-2018. 2013 Adult Obstructive Uropathy, Laparoscopy/Robotics )

A 75-year-old woman with lower limb phlebitis has marked bilateral hydroureteronephrosis on a CT scan performed to screen for an intra-abdominal malignancy. She has had marked uterine prolapse (procidentia) for several years. Stress incontinence of urine is controlled by wearing a pad changed once daily. Residual urine is 100 ml. Serum creatinine is 3.2 mg/dl. Urine culture grows Klebsiella 105 col/ml. The next step in management should be antibiotic therapy and: A) hysterectomy. B) vaginal pessary. C) anterior vaginal repair. D) anterior vesicourethropexy. E) bilateral ureteral stents.

E ( sipuleucel-T. This patient has asymptomatic castrate-resistant prostate cancer. The potential next steps are docetaxel chemotherapy or immune therapy with sipuleucel-T. Both are FDA-approved in this clinical situation. Sipuleucel-T is an active cellular immunotherapy that is a type of therapeutic cancer vaccine. It consists of autologous peripheral blood mononuclear cells with antigen presenting cells that have been activated ex vivo with a recombinant fusion protein that consists of prostatic acid phosphatase that is fused to granulocyte-macrophage colony-stimulating factor (an immune-cell activator).In men with asymptomatic or minimally symptomatic castrate-resistant prostate cancer, a 4.1 month median overall survival benefit was demonstrated compared to placebo. This patient has asymptomatic castrate-resistant prostate cancer with soft-tissue disease. Sunitinib is a tyrosine kinase inhibitor and although FDA-approved for kidney cancer, it has not been approved by the FDA for men with prostate cancer. Zoledronic acid is an I.V. bisphosphonate indicated for patients with metastatic castrate-resistant prostate cancer with bony sites of disease. It has been demonstrated to decrease pain as well as skeletal-related events. No survival advantage has been demonstrated. Although it has been studied for men on long term androgen deprivation therapy, this patient has been on therapy for only six months. Denosumab is a RANK-ligand inhibitor and also decreases skeletal-related events and helps to prevent skeletal-related events associated with osteoporosis. Cabazitaxel is FDA-approved for patients with metastatic castrate-resistant prostate cancer who have failed docetaxel. This patient has not had any chemotherapy. Although not listed, docetaxel is FDA-approved agent for men with castrate-resistant prostate cancer and may be an appropriate next treatment.Kantoff PW, Higano CS, Shore, ND, et al: Sipuleucel-T immunotherapy for castration-resistant prostate cancer. NEJM 2010;363:411-422. 2013 Adult Neoplasm )

A 76-year-old asymptomatic man has castrate-resistant prostate cancer. Bone scan is normal but CT scan of the pelvis demonstrates two new, enlarged pelvic lymph nodes. The next treatment is: A) sunitinib. B) zoledronic acid. C) denosumab. D) cabazitaxel. E) sipuleucel-T.

B ( placement of a catheter into the ileal stoma. Leakage and fistula from urinary diversion occur in 2 to 9% of patients. However, 20 to 60% of these fistulae close spontaneously. Conservative management can be safely attempted assuming the patient is not septic and that adequate drainage is maintained. Leakage could be from the ureteral ileal anastomosis or from the butt end of the conduit. Bilateral ureteral stents are already in place, which should address any concerns about a ureteral ileal anastomotic leak. Therefore, the best initial therapeutic maneuver in this patient is placement of a catheter into the ileal loop to facilitate drainage. While hyperalimentation is important in malnourished patients and should also be initiated, this would not address the immediate issue of the leak. If the stomal catheter failed to decrease the fistulous output, bilateral percutaneous nephrostomy tubes could be placed to divert the urinary stream. If this failed, surgical intervention would be required to address the problem.Skinner EC, Skinner DG, Stein JP: Orthotopic urinary diversion, Wein AJ, Kavoussi LR, Novick AC, Partin AW, Peters CA (eds): CAMPBELL-WALSH UROLOGY, ed 10. Philadelphia, Elsevier Saunders, 2012, vol 3, chap 87, p 2502. 2013 Adult Trauma & Fistulae )

A 78-year-old malnourished woman with a history of prior pelvic radiation for cervical cancer undergoes a radical cystectomy and ileal loop diversion with bilateral ureteral stents for urothelial cancer. At the time of surgery, the bowel shows signs of radiation changes. Four days post-operatively, her urine output decreases with a marked increase in output from her abdominal drain. The next step is: A) parenteral hyperalimentation. B) placement of a catheter into the ileal stoma. C) bilateral percutaneous nephrostomy tube placement. D) revision of the ureteroileal anastomoses. E) excision of the ileal loop and replacement with a transverse colon conduit.

A ( reassurance. Excessive urinary frequency in children is occasionally seen. The diagnosis is made by noting that the patient is continent of urine day and night and the urinalysis is normal. The key to the diagnosis is that the frequency does not persist at night. In this child, his urinary frequency falls under the category of a nervous habit, and may be associated with emotional stress. Urinary frequency generally goes away over time averaging three to six months. It can return in some patients but eventually resolves. Antimuscarinics seldom help these patients whose bladders are normal. Invasive testing such as VCUG and cystoscopy are not indicated since the findings are almost always normal. In the presence of a normal urinalysis, glucose tolerance testing is not indicated.MacLellan DL, Bauer SB: Neuropathic dysfunction of the lower urinary tract, Wein AJ, Kavoussi LR, Novick AC, Partin AW, Peters CA (eds): CAMPBELL-WALSH UROLOGY, ed 10. Philadelphia, Elsevier Saunders, 2012, vol 4, chap 128, p 3431. 2013 Pediatric Neurogenic Bladder, Voiding Dysfunction, Incontinence )

A five-year-old boy has developed frequent daytime urination, voiding at least every 30 minutes. He has no nocturia. Since toilet training at age two, he has been continent day and night. His urinalysis is normal. The most appropriate management is: A) reassurance. B) VCUG. C) antimuscarinics. D) glucose tolerance test. E) cystoscopy.

E ( resection of urethral plate and first stage buccal mucosa graft to urethral bed. Patients that have had numerous previous attempts at hypospadias repair often have a scarred and poorly vascularized urethral bed. In these situations, the best chance for success is resection of the scarred bed with a two-stage buccal mucosa urethroplasty. The minimal degree of residual ventral curvature will likely not be functionally significant and will improve with resection of the fibrotic urethral plate. Single-stage tubed buccal grafts have a higher degree a failure than a two-stage technique.Snodgrass W, Elmore J: Initial experience with staged buccal graft (Bracka) hypospadias reoperations. J UROL 2004;172:1720-1724.Snodgrass WT: Hypospadias, Wein AJ, Kavoussi LR, Novick AC, Partin AW, Peters CA (eds): CAMPBELL-WALSH UROLOGY, ed 10. Philadelphia, Elsevier Saunders, 2012, vol 4, chap 130, pp 3530-3533. 2013 Pediatric Congenital Anomalies, Embryology, Anatomy )

A five-year-old boy has had two prior failed attempts to repair his penoscrotal hypospadias. His urethral meatus is widely patent and is located at the penoscrotal junction. The distal urethral plate is fibrotic and scarred, and is associated with a 15 degree ventral curvature. He has minimal excess preputial and penile shaft skin. The next step is: A) incised plate urethroplasty. B) buccal mucosa onlay graft and corporal body grafting. C) buccal mucosa onlay graft and dorsal corporal plication. D) resection of urethral plate and tubed buccal mucosa graft urethroplasty. E) resection of urethral plate and first stage buccal mucosa graft to urethral bed.

E ( CT urogram. This boy has suffered a rapid deceleration injury and this mechanism of injury warrants evaluation with imaging. A pedicle injury or complete avulsion of the UPJ are potential injuries. These can both occur without hematuria. Hence, observation would be inappropriate. An abdominal tap for blood would not diagnose a renal injury. An ultrasound would likely not be diagnostic. A cystogram in a trauma patient is only indicated in the presence of gross hematuria or pelvic fracture. CT scan is diagnostic and more likely to identify other potential abdominal injuries.Husmann DA: Pediatric genitourinary trauma, Wein AJ, Kavoussi LR, Novick AC, Partin AW, Peters CA (eds): CAMPBELL-WALSH UROLOGY, ed 10. Philadelphia, Elsevier Saunders, 2012, vol 4, chap 138, p 3732. 2013 Pediatric Trauma & Fistulae )

A four-year-old boy fell from a second story window. On examination, his vital signs are stable, but he has right flank and upper quadrant abdominal tenderness and fullness. He does not have peritoneal signs. Urinalysis is normal. The next step is: A) observation. B) abdominal paracentesis. C) abdominal and renal ultrasound. D) cystogram. E) CT urogram.

E ( left nephroureterectomy. This patient has developed upper tract deterioration following cystectomy and ileal conduit diversion. This has been reported in some series to occur in over 50% of patients with long-term followup. The renogram in this instance demonstrates no obstruction to the right renal unit with hydronephrosis likely the result of chronic reflux. The renogram also demonstrates no significant function of the left renal unit. Because there is no reflux into the left system it cannot be monitored as to the possible development of upper tract urothelial carcinoma. In this setting, nephroureterectomy is recommended. Looposcopy will not add to the evaluation as it will not provide access to the left system. Bilateral percutaneous nephrostomy is not indicated because there is no evidence of obstruction of the right side. Similarly, there is no evidence of stomal stenosis. Revision of left ureteroileal anastomosis should not be undertaken for a non-functioning kidney. Another option would be left nephrostomy tube placement, antegrade studies, and selective cytology to further risk stratify the patient prior to making a final decision. Dahl DM, McDougal WS: Use of intestinal segments in urinary diversion, Wein AJ, Kavoussi LR, Novick AC, Partin AW, Peters CA (eds): CAMPBELL-WALSH UROLOGY, ed 10. Philadelphia, Elsevier Saunders, 2012, vol 3, chap 85, p 2411. 2013 Adult Urinary Diversion )

A healthy 66-year-old woman has a loopogram as shown seven years following cystectomy and ileal conduit for bladder cancer. CT scan demonstrates bilateral hydronephrosis and no evidence of recurrent disease. Chest x-ray and urine cytology are normal. Serum creatinine is 1.8 mg/dl. A renogram is also shown. The next step is: A) looposcopy. B) bilateral percutaneous nephrostomy. C) stomal revision. D) revision of left ureteroileal anastomosis. E) left nephroureterectomy.

C ( transverse vaginal septum. The MRI scan shows evidence of a distended upper vagina and presence of a uterus which is most consistent with either a transverse vaginal septum or distal vaginal agenesis. Transverse vaginal septa are believed to arise from a failure in fusion or canalization (or both) of the urogenital sinus and Müllerian ducts. Many of the patients present at puberty with primary amenorrhea and a distended upper vagina. A complete transverse vaginal septum may be located at various levels in the vagina, but there is a higher frequency in the middle and upper third of the vagina. Transperineal, transrectal, and abdominal ultrasonography and MRI scan may be beneficial in establishing the diagnosis and determining the location and thickness of a transverse vaginal septum. Vaginal atresia occurs when the urogenital sinus fails to contribute to formation of the lower (distal) portion of the vagina. Mayer-Rokitansky syndrome is characterized by either partial or complete absence of the vagina and coexisting uterine abnormalities, with the uterus either partially or completely absent. In Mayer-Rokitansky syndrome, the fallopian tubes and ovaries are present but may be either normal or hypoplastic. Two types exist, type I that involves only the Müllerian structures (vagina and uterus) and type II that will involve concurrent abnormalities of either the cardiac, renal or otologic systems. Androgen insensitivity syndrome is characterized by the absence of a uterus, salpinx, and upper 2/3 of the vagina, these structures regress under the active influence of MIF secreted from the testes. CAH infant will manifest an enlarged clitoris and genital ambiguity, with variable lengths of a urogenital sinus present dependent upon the degree of androgen secretion from the adrenal glands. An imperforate hymen should demonstrate a visible bulging membrane at the vaginal introitus. Rink RC, Kaefer M: Surgical management of disorders of sexual differentiation, cloacal malformation, and other abnormalities of the genitalia in girls, Wein AJ, Kavoussi LR, Novick AC, Partin AW, Peters CA (eds): CAMPBELL-WALSH UROLOGY, ed 10. Philadelphia, Elsevier Saunders, 2012, vol 4, chap 134, pp 3631-3632. 2013 Pediatric Congenital Anomalies, Embryology, Anatomy )

A healthy one-month-old girl has lower abdominal distention. An ultrasound demonstrates a cystic mass behind the bladder. Follow up MRI scan is shown. Physical exam reveals normal appearing genitalia with a single opening in the urethral position with no evidence of a vaginal opening. These findings are most consistent with: A) Mayer-Rokitansky syndrome. B) androgen insensitivity syndrome. C) transverse vaginal septum. D) imperforate hymen. E) CAH.

C ( I.V. hydration and alkalinization. This patient likely has rhabdomyolysis. The diagnosis is made by measurement of serum creatine phosphokinase (CPK) or a spot test for urine myoglobin. Risk factors for this at the time of laparoscopic surgery include a BMI of 25 or greater, male gender, prolonged operative time, full-table flexion and prolonged use of the kidney rest. Prevention is essential and includes minimizing the use of the kidney rest and intra-operative hypotension. Brownish-red urine suggestive of the hematuria may be noted in patients with rhabdomyolysis, however microscopic assessment of the urine will not show RBCs. Treatment at this time should be hydration and alkalinization to minimize the risk of renal failure. Renal function should be monitored. The need for transient dialysis occurs in a minority of patients so nephrology consultation may not be necessary. Radiographs of the hip are not indicated as it is unlikely that an injury to the bony pelvis would occur during a nephrectomy leading to these symptoms.Eichel L, Clayman RV: Fundamentals of laparoscopic and robotic urologic surgery, Wein AJ, Kavoussi LR, Novick AC, Partin AW, Peters CA (eds): CAMPBELL-WALSH UROLOGY, ed 10. Philadelphia, Elsevier Saunders, 2012, vol 1, chap 9, p 249. 2013 Adult Obstructive Uropathy, Laparoscopy/Robotics )

A man with a BMI of 35 (normal 18.5-24.9) complains of left flank and hip pain after undergoing laparoscopic right nephrectomy in the lateral position. The surgery was challenging and required use of the kidney rest for an extended period of time. Postoperatively, he appears to have brownish-red urine. Urine dip stick is 3+ for blood but no RBC os seen microscopically. The next step is: A) observation. B) nephrology consultation. C) I.V. hydration and alkalinization. D) radiographic imaging of the hip. E) initiation of patient-controlled anesthesia.

C ( a tooth abscess. Zoledronic acid and other bisphosphonates have become an important part of the management of patients with prostate cancer bone metastasis. These compounds reduce bone resorption by inhibiting osteoclastic activity and proliferation. In patients with progressive hormone refractory bone metastatic prostate cancer, zoledronic acid has been shown to reduce the incidence of skeletal events in a randomized prospective trial. Adverse events include fatigue, myalgias, fever, anemia and elevations in serum creatinine. Osteonecrosis of the mandibular bone is a severe complication of bisphosphonates usually associated with patients undergoing dental work or who have poor dentition or chronic dental disease. The bisphosphonates should be immediately discontinued in the setting of osteonecrosis or expected invasive dental procedures.Antonarakis ES, Carducci MA, Eisenberger MA: Treatment of castration-resistant prostate cancer, Wein AJ, Kavoussi LR, Novick AC, Partin AW, Peters CA (eds): CAMPBELL-WALSH UROLOGY, ed 10. Philadelphia, Elsevier Saunders, 2012, vol 3, chap 110, p 2954. 2013 Adult Neoplasm )

A man with castrate-resistant prostate cancer and bone metastases is on leuprolide acetate injections and intravenous zoledronic acid injections. He is asymptomatic. Zoledronic acid injections must be stopped if the patient develops: A) fever. B) fatigue. C) a tooth abscess. D) a pathologic fracture. E) severe osteoporosis.

A ( observation. Hydronephrosis is often persistent after resection of a PUV, and does not necessarily suggest failure of therapy. The hydronephrosis may also be due to persistent left-sided VUR immediately following successful valve resection. Both hydronephrosis and VUR may resolve in approximately 50% of boys following valve resection. This boy should be given time along with prophylactic antibiotics and re-evaluation in 12-18 months. In the absence of persistent urinary infection, anti-reflux surgery or removal of the left system and bladder diverticulum are not indicated, and might result in loss of system compliance. Assessment of renal function using either a MAG-3 with Lasix or DMSA renal scan would be necessary before considering removal of any kidney. Urinary diversion at the bladder or supravesical level is not indicated in this boy.Casale AJ: Posterior urethral valves, Wein AJ, Kavoussi LR, Novick AC, Partin AW, Peters CA (eds): CAMPBELL-WALSH UROLOGY, ed 10. Philadelphia, Elsevier Saunders, 2012, vol 4, chap 126, p 3401. 2013 Pediatric Obstructive Uropathy, Laparoscopy/Robotics )

A newborn boy with bilateral hydronephrosis has PUV, left sided grade 5 VUR, and a large bladder diverticulum on VCUG. After successful resection of valves, left hydronephrosis persists. The next step is prophylactic antibiotics and: A) observation. B) vesicostomy. C) left cutaneous ureterostomy. D) left anti-reflux surgery. E) left nephroureterectomy and bladder diverticulectomy.

A ( observation. In children who sustain blunt abdominal trauma, CT scan can define injuries to the abdominal viscera quite well. Conservative treatment in the hemodynamically stable patient with follow-up CT scan, if needed, will result in reducing the rate of abdominal exploration in the patient with stable visceral injuries to less than 10%. Cystoscopy and ureteral stent placement in this patient with a grade 3 renal laceration is not indicated, unless clinical symptoms, such as a prolonged ileus or sepsis should develop. Renal and hepatic arteriography would be indicated if there is decreasing hematocrit with increasing size of hematoma, and may allow for selective embolization of the bleeding vessel(s), potentially eliminating the need for exploration. Surgical exploration with repair of lacerations is not indicated in this stable patient and in some cases may result in disruption of hematoma, bleeding and unnecessary nephrectomy.Husmann DA: Pediatric genitourinary trauma, Wein AJ, Kavoussi LR, Novick AC, Partin AW, Peters CA (eds): CAMPBELL-WALSH UROLOGY, ed 10. Philadelphia, Elsevier Saunders, 2012, vol 4, chap 138, p 3739. 2013 Pediatric Trauma & Fistulae )

A nine-year-old girl is struck in the abdomen and right flank by an automobile. She is alert and her vital signs are stable. She has moderate guarding in her right upper quadrant and decreased bowel sounds. Her hematocrit is 33%, and she has blood-tinged urine. A CT scan demonstrates a 4 cm hepatic laceration and hematoma and a 3 cm laceration of the right kidney with minimal contrast extravasation. The next step is: A) observation. B) cystoscopy and ureteral stent placement. C) renal and hepatic arteriography. D) percutaneous drain placement. E) abdominal exploration.

C ( cystogram. Forceful expression of the over-distended bladder in the neonate, cystoscopy, inguinal hernia repair, and umbilical vessel catheterization are the most common causes of iatrogenic injuries to a child's bladder and may on rare occasions in association with VUR result in forniceal rupture. Although the Credé maneuver is used less commonly, it can be employed in infants who do not empty their bladders completely. However, the Credé voiding is contraindicated in children with a reactive external sphincter and should not be initiated in children with spinal bifida until they have been assessed with urodynamics. The Credé maneuver stimulates a reflux response in the external sphincter that increases urethral resistance, thereby increasing the pressure needed to expel urine from the bladder. Bladder rupture has occurred and VCUG will be diagnostic in most cases. Fluid bolus is unnecessary because there is no reason to suspect volume depletion and may delay diagnosis. Although plain film of the abdomen and CT scan without delay imaging may be suggestive of ascites, they are not diagnostic. CT cystogram is acceptable but results in excessive radiation exposure. Laparotomy is not indicated until a diagnosis is made.MacLellan DL, Bauer SB: Neuropathic dysfunction of the lower urinary tract, Wein AJ, Kavoussi LR, Novick AC, Partin AW, Peters CA (eds): CAMPBELL-WALSH UROLOGY, ed 10. Philadelphia, Elsevier Saunders, 2012, vol 4, chap 128, p 3439. 2013 Pediatric Trauma & Fistulae )

A one-day-old girl with spina bifida undergoes back closure and one week later has placement of a ventriculoperitoneal shunt. Following catheter removal, she voids with a moderate residual. Credé maneuver is started. Eight hours later, she develops abdominal distension and has minimal urine output. The bladder is catheterized for 3 cc. The next step is: A) fluid bolus. B) plain film of the abdomen. C) cystogram. D) abdominal/pelvic CT scan. E) laparotomy.

D ( hypocalcemia. This patient has developed acute renal failure (ARF). Children with a history of chronic renal insufficiency secondary to renal dysplasia are at significant risk for ARF after any major surgical procedure. Most cases are reversible. Patients with ARF can develop hyperkalemia, metabolic acidosis, hyperphosphatemia (from decreased renal excretion), and hyponatremia. The hypocalcemia that develops is due to several causes including hyperphosphatemia, decreased intestinal calcium reabsorption, and Vitamin D deficiency. The most important acute reason for the hypocalcemia is the hyperphosphatemia. When correcting the metabolic acidosis, it is important to monitor the ionized calcium level, since this can drop precipitously. Signs and symptoms of hypocalcemia include cramping, tetany, and prolonged QT interval on EKG.Coplevitch L, Kaplan B, Meyers K: Acute renal failure, in Docimo SG, Canning DA, Khoury AE (eds): CLINICAL PEDIATRIC UROLOGY, ed 5. London, Informa Healthcare, 2007, pp 363-365.Goldfarb DA, Poggio ED: Etiology, pathogenesis, and management of renal failure, Wein AJ, Kavoussi LR, Novick AC, Partin AW, Peters CA (eds): CAMPBELL-WALSH UROLOGY, ed 10. Philadelphia, Elsevier Saunders, 2012, vol 2, chap 43, pp 1203-1208. 2013 Pediatric Fluid & Electrolyte,Transplantation, Hypertension, Vasc Disease, Nephrology )

A one-year-old boy with prune belly syndrome has chronic renal insufficiency. He undergoes abdominoplasty and bilateral orchidopexy and develops oliguria post operatively that does not respond to fluid boluses. Serum electrolytes reveal elevated BUN and creatinine, metabolic acidosis, and hyperkalemia. Sodium polystyrene enemas are given and I.V. sodium bicarbonate is administered. The patient develops abdominal cramping and muscle spasms. This is most likely due to: A) hypokalemia. B) hypophosphatemia. C) hypermagnesemia. D) hypocalcemia. E) hyponatremia.

A ( observation. Intervention is not required for this patient at this time. Removal of the entire midurethral sling is not required for a small exposure of mesh. With partial sling excision, continence is maintained in the majority of patients; therefore, replacement of another sling would not be indicated. Removal of the entire midurethral sling is challenging, unnecessary and would likely lead to recurrent stress urinary incontinence. Transvaginal estrogen is thought to promote vaginal healing and is preferred over oral estrogen therapy due to other potential systemic effects. Extrusions that are larger or symptomatic should be treated. When a midurethral sling is eroded into or involves the urinary tract it should be treated with removal. Dmochowski RR, Padmanabhan P, Scarpero HM: Slings: Autologous, biologic, synthetic, and midurethral, Wein AJ, Kavoussi LR, Novick AC, Partin AW, Peters CA (eds): CAMPBELL- WALSH UROLOGY, ed 10. Philadelphia, Elsevier Saunders, 2012, vol 3, chap 73, p 2148. 2013 Adult Neurogenic Bladder, Voiding Dysfunction, Incontinence )

A pelvic examination of a 75-year-old woman two years after prior midurethral synthetic sling reveals extrusion of a small amount of mesh along her anterior vaginal wall. She is continent and denies any other urinary or vaginal complaints. She is not sexually active. Urinalysis is normal. The next step is: A) observation. B) removal of extruded mesh. C) removal of entire midurethral sling. D) oral estrogen hormone replacement. E) removal of exposed mesh and simultaneous sling replacement.

A ( consult neurologist for alternative seizure treatment. Both ketogenic diet and topiramate (Topamax®) can cause calcium phosphate stones. This child with small non-obstructing kidney stones, discovered after a single episode of painless gross hematuria, does not require surgical intervention at this time. Seeking alternative seizure pharmacotherapy by consulting a neurologist is the best next step. The stone is likely calcium-based, and thus medical therapy aimed at uric acid (raise the pH) or cystine (oral penicillamine) stones are not appropriate.Ferrandino MN, Pietrow PK, Preminger GM: Evaluation and medical management of urinary lithiasis, Wein AJ, Kavoussi LR, Novick AC, Partin AW, Peters CA (eds): CAMPBELL-WALSH UROLOGY, ed 10. Philadelphia, Elsevier Saunders, 2012, vol 2, chap 46, p 1320. 2013 Pediatric Calculous Disease )

A seven-year-old boy with a seizure disorder is managed with a ketogenic diet and topiramate, has one episode of painless gross hematuria. Renal ultrasound demonstrates 3 mm stones in the lower pole of both kidneys without hydronephrosis. The next step is: A) consult neurologist for alternative seizure treatment. B) urinary alkalinization. C) oral penicillamine. D) SWL. E) ureteroscopic laser lithotripsy.

D ( voiding diary. Daytime urinary incontinence occurs in 5% of seven-year-old children. In most children, the underlying problem is infrequent voiding. Timed voiding programs alone will be successful in the majority of children but require several months to be effective. This child has not had enough time to determine if the program will be effective. Changing to another medication or proceeding with urodynamic evaluation at this time is premature. Urethral dilation is not indicated for the treatment of daytime urinary incontinence. Some authors have suggested that an occult tethered cord is responsible for persistent daytime urinary incontinence. However, imaging of the spine should be reserved for those children with significant abnormalities on neurologic exam or urodynamic evaluation. MacLellan DL, Bauer SB: Neuropathic dysfunction of the lower urinary tract, Wein AJ, Kavoussi LR, Novick AC, Partin AW, Peters CA (eds): CAMPBELL-WALSH UROLOGY, ed 10. Philadelphia, Elsevier Saunders, 2012, vol 4, chap 128, p 3431. 2013 Pediatric Neurogenic Bladder, Voiding Dysfunction, Incontinence )

A seven-year-old girl has urgency, frequency, and daytime urinary incontinence. She is wet every day, requiring clothing changes twice daily. Physical exam and urinalysis are normal. She is placed on oxybutynin 5 mg bid and timed voiding. She returns two weeks later and reports no change in the daytime urinary incontinence. The next step is: A) MRI scan of the lumbosacral spine. B) add imipramine. C) videourodynamics. D) voiding diary. E) urethral dilation.

A ( hypercalciuria. Renal calculi occur in very low birth weight pre-term infants with a history of severe ventilatory problems and bronchopulmonary dysplasia. Many of these infants require long-term treatment with diuretic agents to control heart failure. The diuretic agent used most often is furosemide, which increases the rate of urinary calcium excretion up to ten times normal. Chronic hypercalciuria from furosemide therapy has been shown to result in nephrocalcinosis and calculus formation. Loss of calcium from chronic administration of furosemide may lead to secondary hyperparathyroidism and bone changes. Treatment includes switching from furosemide to thiazides. Other etiologies of stone formation do not occur with increased frequency in premature infants requiring diuretic therapyPalmer LS, Trachtman H: Renal functional development and diseases in children, Wein AJ, Kavoussi LR, Novick AC, Partin AW, Peters CA (eds): CAMPBELL-WALSH UROLOGY, ed 10. Philadelphia, Elsevier Saunders, 2012, vol 4, chap 112, p 3062. 2013 Pediatric Calculous Disease )

A six-week-old boy was born at 27 weeks gestation. His postnatal course has been complicated by respiratory distress, bronchopulmonary dysplasia, and a patent ductus arteriosus. He has required long-term diuretic therapy. A KUB reveals calcifications in the mid and upper abdominal regions consistent with bilateral renal calculi. The most likely mechanism for the formation of the stones is: A) hypercalciuria. B) hyperuricosuria. C) obstructive uropathy. D) Type I RTA. E) Type II RTA.

A ( observation. This boy has typical signs and symptoms of viral cystitis. Adenovirus is the most common virus although viral cultures are infrequently done in this setting. Supportive therapy is the mainstay of management. An ultrasound should be done to rule-out other serious causes of hematuria. Bladder wall thickening is to be expected in the acute phases of a viral infection. Symptoms usually resolve within two to four weeks. Antimuscarinics can be used when the urgency and frequency is more severe. Ribavirin can be considered in highly symptomatic patients, especially in those that are immunosuppressed. There is no indication for antibiotics. If his symptoms do not resolve after a few weeks, then one can get a VCUG or perform cystoscopy to rule out PUV or other pathology. However, doing this in the acute phase is premature since the clinical suspicion for valves is low. If he had valves, one would have expected symptoms prior to this time. The clinical suspicion for a stone is also low given the ultrasound findings. Thus, getting a CT scan is not indicated especially in light of the unnecessary exposure to radiation which is a significant concern in children.Shortliffe LMD: Infection and inflammation of the pediatric genitourinary tract, Wein AJ, Kavoussi LR, Novick AC, Partin AW, Peters CA (eds): CAMPBELL-WALSH UROLOGY, ed 10. Philadelphia, Elsevier Saunders, 2012, vol 4, chap 116, p 3117. 2013 Pediatric Infection & Inflammatory Disease )

A six-year-old boy has the sudden onset of gross hematuria, urgency, and frequency. Urinalysis shows 5-10 WBC/hpf and gross blood. Urine culture is negative. Ultrasound shows diffuse bladder wall thickening but no hydronephrosis or renal mass. The next step is: A) observation. B) antibiotics. C) VCUG. D) non contrast CT scan. E) cystoscopy.

D ( I.V. ketorolac. For post-operative bladder spasms refractory to the conventional pain management using opioids and antimuscarinics, intravenous ketorolac (NSAID) administered at 0.25 to 0.5 mg/kg every six hours is effective in reducing bladder spasms following bladder surgery. However, significant adverse effects have been reported including renal failure, prolonged bleeding and hypersensitivity reactions. It should be avoided in patients with renal insufficiency, NSAID sensitivity, persistent post-operative bleeding, and dehydration. Other treatments such as alpha blocker, benzodiazepine, rectal acetaminophen and caudal block do not provide any additional benefit. Estrada CR Jr, Ferrari LR: Core principles of perioperative management in children, Wein AJ, Kavoussi LR, Novick AC, Partin AW, Peters CA (eds): CAMPBELL-WALSH UROLOGY, ed 10. Philadelphia, Elsevier Saunders, 2012, vol 4, chap 119, p 3206. 2013 Pediatric Neurogenic Bladder, Voiding Dysfunction, Incontinence )

A six-year-old girl undergoes bilateral ureteral reimplantation. On the first post-operative day, she has severe bladder spasms refractory to parenteral opioids and oral antimuscarinic agents. Her urine is clear, and the creatinine is 0.4 mg/dl. The next step is: A) oral alpha blocker. B) oral benzodiazepine. C) rectal acetaminophen. D) I.V. ketorolac. E) caudal block.

B ( antibiotic suppression. Nephrogenic adenoma is a rare benign metaplastic response of urothelium to tissue injury. Most nephrogenic adenomas occur after an inciting event such as surgery, trauma, infection, and immunosuppression or in response to calculi. The adenomas will develop months to years after a precipitating event and will usually occur within the bladder but may occur on any urothelial surface including transposed bladder mucosal grafts. The main presentation is hematuria and irritative voiding symptoms, but patients may also present with obstructive symptoms or be diagnosed incidentally. The endoscopic appearance is that of a papillary exophytic lesion resembling a low-grade urothelial carcinoma. On histological evaluation, nephrogenic adenoma appear as subepithelial tubular structures similar to Henle's loops. Although at one time these lesions were considered to be premalignant in nature, recent studies have been unable to establish a relationship between nephrogenic adenomas and the subsequent development of malignancy. Even though there is no evidence of malignant potential, transurethral resection is recommended together with long-term antibiotic prophylaxis for at least one year after resection. Prolonged antibiotic therapy is suggested due to the frequent finding of UTI as an associated or causative factor. It is controversial regarding whether patients with nephrogenic adenomas should undergo surveillance cystoscopy. Although there is high incidence of recurrence (30-40%) it is feared that repeated cystoscopic evaluations could further traumatize the bladder urothelium leading to an increased incidence of recurrence. Since nephrogenic adenomas are now known to be a benign condition most authorities recommend cystoscopy, only if recurrent gross hematuria and/or irritative or obstructive voiding symptoms develop.Frimberger DC, Kropp BP: Bladder anomalies in children, Wein AJ, Kavoussi LR, Novick AC, Partin AW, Peters CA (eds): CAMPBELL-WALSH UROLOGY, ed 10. Philadelphia, Elsevier Saunders, 2012, vol 4, chap 125, p 3387.Garcia-Penit J, Orsola A, Parada R, et al: Synchronous nephrogenic adenoma in the bladder and neourethra (bladder mucosa) in a boy. BR J UROL INT 1999;84:169-170.Peeker R, Aldenborg F, Fall M: Nephrogenic adenoma: A study with special reference to clinical presentation. BR J UROL 1997;80(4):539-542. 2013 Pediatric Neoplasm )

A ten-year-old girl with a history of bilateral high grade VUR was treated with bilateral cross-trigonal ureteral reimplantation surgery at two years of age. She is evaluated for persistent gross hematuria following treatment of a UTI. Cystoscopy and bladder mass resection demonstrates nephrogenic adenomas. The next step is: A) reassurance. B) antibiotic suppression. C) intravesical BCG. D) laser ablation. E) partial cystectomy.

A ( renal vein thrombosis. This is a large infant, which may be associated with maternal diabetes. That history and the abdominal mass make the diagnosis of renal vein thrombosis most likely. The thrombocytopenia is characteristic. Renal artery thrombosis is usually associated with an indwelling umbilical artery catheter, and can result in a mass and hematuria. Congenital mesoblastic nephroma is the most common solid renal mass in an infant but is not usually associated with hematuria or thrombocytopenia. Henoch-Schonlein purpura is a systemic vasculitis that commonly presents at four to six years of age. The common features are palpable purpuric rash, abdominal pain with gastrointestinal bleeding, and arthritis. Hemolytic uremic syndrome is defined by the triad of microangiopathic hemolytic anemia, thrombocytopenia and acute renal failure that typically occurs after enterocolitis with E. coli (O157:H7). Lee RS, Borer JG: Perinatal urology, Wein AJ, Kavoussi LR, Novick AC, Partin AW, Peters CA (eds): CAMPBELL-WALSH UROLOGY, ed 10. Philadelphia, Elsevier Saunders, 2012, vol 4, chap 114, p 3060. 2013 Pediatric Fluid & Electrolyte,Transplantation, Hypertension, Vasc Disease, Nephrology )

A two-day-old, 4.5 kg girl has gross hematuria. A right flank mass is palpable. CBC shows thrombocytopenia. The most likely diagnosis is: A) renal vein thrombosis. B) renal artery thrombosis. C) Henoch-Schönlein purpura. D) hemolytic uremic syndrome. E) congenital mesoblastic nephroma.

D ( mediated by angiotensin II. Aldosterone levels are primarily regulated by angiotensin II through the renin-angiotensin-aldosterone system and directly by serum potassium levels. Increased sodium decreases aldosterone. Aldosterone is produced in the zona glomerulosa and not under direct control of ACTH. Renal hypoperfusion will increase aldosterone production. Kutikov A, Crispen PL, Uzzo RG: Pathophysiology, evaluation, and medical management of adrenal disorders, Wein AJ, Kavoussi LR, Novick AC, Partin AW, Peters CA (eds): CAMPBELL-WALSH UROLOGY, ed 10. Philadelphia, Elsevier Saunders, 2012, vol 2, chap 57, pp 1697-1698. 2013 General Physiology, Immunology, & Adrenal )

Aldosterone production is primarily: A) a function of the zona fasciculata. B) increased by atrial natriuretic factor. C) decreased in renal hypoperfusion. D) mediated by angiotensin II. E) increased with sodium loading.

B ( urethral catheter drainage. This boy is developing progressive renal insufficiency due to multiple factors including increased urine output from tubular injury and bladder decompensation. In this patient with an acute elevation in creatinine and hypertension, temporary decompression of the bladder to assess potential recovery of renal function should be performed. Videourodynamics to assess bladder function would be the next step in guiding appropriate therapy. Treatment options include CIC with nighttime bladder drainage, with or without antimuscarinics. Alpha-blocker therapy promotes relaxation of the internal urethral sphincter, but has not been consistently successful in these patients. Desmopressin may be indicated in the future following nephrology consultation.Casale AJ: Posterior urethral valves, Wein AJ, Kavoussi LR, Novick AC, Partin AW, Peters CA (eds): CAMPBELL-WALSH UROLOGY, ed 10. Philadelphia, Elsevier Saunders, 2012, vol 4, chap 126, p 3402.Glassberg K, Horowitz M: Urethral valve and other anomalies of the urethra in, King LR, Belman AB, Kramer SA (eds): CLINICAL PEDIATRIC UROLOGY, ed 4. London, Martin Dunitz, 2002, chap 28, pp 899-945. 2013 Pediatric Obstructive Uropathy, Laparoscopy/Robotics )

An 11-year-old boy with a history of a PUV develops hypertension and incontinence. Over the past six months, his creatinine has increased from 1.2 to 2.8 mg/dl and his BUN from 28 to 45 mg/dl. Ultrasound shows increased hydroureteronephrosis and a large PVR. The next step is: A) initiate transplant evaluation. B) urethral catheter drainage. C) alpha-blocker therapy. D) antimuscarinics. E) desmopressin.

E ( in an area of previous bladder neck surgery. The best treatment for a patient with low bladder compliance, small capacity, and sphincteric incontinence is the combined use of bladder augmentation and increased bladder outlet resistance. When an artificial urinary sphincter is used in conjunction with augmentation of the bladder, the timing of the two procedures does not appear to affect the outcome. More important factors are good bowel preparation, intravenous antibiotics, sterility of the urine, and meticulous surgical technique to avoid entering the previously augmented bladder during sphincter implantation which may predispose to infection and sphincter erosion. Patients who have undergone prior incontinence procedures are also at increased risk for sphincter erosion.Wessells H, Peterson AC: Surgical procedures for sphincteric incontinence in the male: The artificial genitourinary sphincter and perineal sling procedures, Wein AJ, Kavoussi LR, Novick AC, Partin AW, Peters CA (eds): CAMPBELL-WALSH UROLOGY, ed 10. Philadelphia, Elsevier Saunders, 2012, vol 3, chap 79, pp 2302-2304. 2013 Pediatric Neurogenic Bladder, Voiding Dysfunction, Incontinence )

An 11-year-old boy with spina bifida is scheduled for bladder augmentation and implantation of an artificial urinary sphincter. Infection or erosion are more common if sphincter implantation is performed: A) prior to bladder augmentation. B) following bladder augmentation. C) simultaneous with augmentation. D) with sigmoid enterocystoplasty. E) in an area of previous bladder neck surgery.

B ( PVR. Transient urinary incontinence occurs in almost one third of ambulatory elderly patients. LUTS in the elderly may be secondary to a number of medical conditions, including diabetes, immobility, congestive heart disease, etc. Antimuscarinic agents may cause or worsen urinary incontinence in elderly patients with poor detrusor contractility. This may present with new or worsened incontinence due to overflow after the initiation of an antimuscarinic agent and can be diagnosed with the non-invasive measurement of a PVR. Urine culture is not indicated in the setting of a normal urinalysis. There is no need at this point to proceed to uroflowmetry, urodynamics or cystoscopy but these may be useful in further evaluation.Resnick NM, Stasa DT, Yalla SV: Geriatric incontinence and voiding dysfunction, Wein AJ, Kavoussi LR, Novick AC, Partin AW, Peters CA (eds): CAMPBELL-WALSH UROLOGY, ed 10. Philadelphia, Elsevier Saunders, 2012, vol 3, chap 76, p 2205. 2013 Adult Core Competencies, Geriatric, Radiation Safety and Ultrasound )

An 86-year-old man with nocturia times three, daytime frequency, urinary urgency, and occasional incontinence is treated with tolterodine. His incontinence worsens. Urinalysis is normal. The next step is: A) urine culture. B) PVR. C) uroflowmetry. D) videourodynamics. E) cystoscopy.

A ( vomiting. Congestive heart failure is associated with fluid retention. Therefore, a patient with heart failure would not be dehydrated. Renal disease, Addison's disease, and diuretic excess cause hyponatremia by increased urinary excretion of sodium. Of the choices listed, only vomiting would result in hypovolemia, hyponatremia, and a very low concentration of sodium in the urine. Patients who experience excessive vomiting lose fluid and salt. Therefore, they become dehydrated and their kidneys reabsorb almost all of the filtered sodium. Thus, the urinary sodium concentration is low.Palmer BF: Hyponatremia, in Rakel RE, Bope ET (eds): RAKEL: CONN'S CURRENT THERAPY, ed 58. Philadelphia: Elsevier Saunders, 2006, p 720.Shoskes DA, McMahon AW: Renal physiology and pathophysiology, Wein AJ, Kavoussi LR, Novick AC, Partin AW, Peters CA (eds): CAMPBELL-WALSH UROLOGY, ed 10. Philadelphia, Elsevier Saunders, 2012, vol 2, chap 38, pp 1038-1040. 2013 General Fluid & Electrolyte,Transplantation, Hypertension, Vasc Disease, Nephrology )

An acutely ill 50-year-old man is in the emergency room. He is unable to give a history. Physical exam reveals severe dehydration. Serum Na is 125 mEq/l, and urinary Na is 8 mEq/l (> 20 mEq/l normal spot urine). The most likely cause of his hyponatremia is: A) vomiting. B) diuretic excess. C) renal disease. D) congestive heart failure. E) Addison's disease.

A ( proximally on the mesonephric duct. The ureteric bud arises off of the mesonephric duct. The segment distal to the ureteric bud is called the common excretory duct. The point of origin of the ureteric bud is the ureteral orifice. If the ureteral bud arises more distally than normal, the ureteral orifice enters the bladder earlier than usual and migrates cranially and laterally and will likely be associated with reflux. If the bud arises more proximally on the duct the orifice ends up medial and caudal. A very proximal origin leads to a persistent position on the mesonephric duct and termination outside the bladder. In the male this is either in the epididymis, vas deferens, seminal vesicles, and prostate. In the female, the mesonephric duct becomes the epoophoron, oophoron, and Gartner's duct. An ectopic ureter draining into these structures ruptures into the fallopian tube, uterus, upper vagina or vestibule. Peters CA, Schlussel RN, Mendelsohn C: Ectopic ureter, ureterocele, and ureteral anomalies, Wein AJ, Kavoussi LR, Novick AC, Partin AW, Peters CA (eds): CAMPBELL-WALSH UROLOGY, ed 10. Philadelphia, Elsevier Saunders, 2012, vol 4, chap 121, pp 3238-3239. 2013 General Congenital Anomalies, Embryology, Anatomy )

An ectopic ureteral insertion into the vagina is the result of the ureteral bud arising or interacting: A) proximally on the mesonephric duct. B) distally on the mesonephric duct. C) on the common excretory duct. D) with ectopic metanephric blastema. E) on the paramesonephric duct.

A ( observation. Up to 25% of children with multicystic dysplastic kidneys will have contralateral vesicoureteral reflux. The scan suggests pyelonephritis in the left upper pole. There is no need to remove the right multicystic dysplastic kidney at this time. Infection in a multicystic dysplastic kidney is extremely rare and not seen in this scenario. In this age child, there is still a good chance the reflux will resolve spontaneously. Prophylactic antibiotics and observation is the best treatment. In the absence of breakthrough infections, a circumcision is not needed. Khoury AE, Bagli DJ: Vesicoureteral reflux, Wein AJ, Kavoussi LR, Novick AC, Partin AW, Peters CA (eds): CAMPBELL-WALSH UROLOGY, ed 10. Philadelphia, Elsevier Saunders, 2012, vol 4, chap 122, pp 3286, 3288-3290. 2013 Pediatric Infection & Inflammatory Disease )

An eight-month-old uncircumcised boy is treated for a febrile UTI. Ultrasound shows a right multicystic dysplastic kidney and a normal left kidney. VCUG shows left grade 3 VUR. DMSA scan shows non-function of the right kidney and a left upper pole cortical defect. The next step is prophylactic antibiotics and: A) observation. B) circumcision. C) left antireflux surgery. D) right nephrectomy. E) left antireflux surgery and right nephrectomy.

A ( urodynamics. This patient should undergo urodynamic evaluation to assess the etiology of the incontinence. MRI scan of the spine would be indicated if the urodynamic study shows new abnormal findings, or new onset of lower extremity weakness or other alterations on neurologic exam. Implementing any additional treatment at this time without identifying the cause for the incontinence would be premature. Yeung CK, Sihoe JDY: Non-neuropathic dysfunction of the lower urinary tract in children, Wein AJ, Kavoussi LR, Novick AC, Partin AW, Peters CA (eds): CAMPBELL-WALSH UROLOGY, ed 10. Philadelphia, Elsevier Saunders, 2012, vol 4, chap 127, p 3411. 2013 Pediatric Neurogenic Bladder, Voiding Dysfunction, Incontinence )

An eight-year-old boy was treated for lipomyelomeningocele at birth. He is on CIC every four hours and oxybutynin 5 mg BID. He develops new incontinence. The next step is: A) urodynamics. B) MRI scan of spine. C) increase CIC frequency. D) increase antimuscarinics. E) start imipramine.

B ( autosomal dominant polycystic kidneys. Ten percent of patients with autosomal dominant polycystic kidney disease have berry aneurysms, and subarachnoid hemorrhage can be a lethal consequence. The high incidence of hypertension with autosomal polycystic kidney disease may contribute to the frequency of bleeding from the berry aneurysm. There is no association between renal aneurysms and berry aneurysms. Multicystic dysplastic kidneys have a slight association with hypertension but no extrarenal manifestations otherwise. Horseshoe kidney has a higher incidence of UPJ obstruction and Wilms' tumor, but is not associated with berry aneurysms. Autosomal recessive polycystic kidney disease is associated with liver failure, but has no known association with berry aneurysms. Pope JC IV: Renal dysgenesis and cystic disease of the kidney, Wein AJ, Kavoussi LR, Novick AC, Partin AW, Peters CA (eds): CAMPBELL-WALSH UROLOGY, ed 10. Philadelphia, Elsevier Saunders, 2012, vol 4, chap 118, pp 3170-3176. 2013 General Fluid & Electrolyte,Transplantation, Hypertension, Vasc Disease, Nephrology )

Berry aneurysms of the circle of Willis are most frequently associated with: A) renal aneurysms. B) autosomal dominant polycystic kidneys. C) autosomal recessive polycystic kidneys. D) horseshoe kidney. E) multicystic dysplastic kidneys.

D ( secondary syphilis. Condyloma lata are a cutaneous manifestation of secondary syphilis. They appear as flesh colored or hypopigmented, macerated papules or plaques. They most commonly involve genital and anal areas. The lesions are typically smooth and moist. Condyloma lata resemble condyloma acuminata but are distinguished by their smooth, flat and moist appearance. Condyloma lata have not been associated specifically with cervical carcinoma, gonococcal urethritis, herpes simplex, or HIV.Deshpande DJ, Nayak CS, Mishra SN, et al: Verrucous condyloma lata mimicking condyloma acuminata: An unusual presentation. INDIAN J SEX TRANSM DIS 2009;30:100-102. 2013 Adult Infection & Inflammatory Disease )

Condyloma lata are associated with: A) cervical carcinoma. B) gonococcal urethritis. C) herpes simplex virus. D) secondary syphilis. E) AIDS-related complex.

A ( penicillin. The findings described are characteristic of Clostridium perfringens wound infection. Clostridial infections should be considered in any patient with a wound infection, especially if there has been an injury to the colon. The organism is an anaerobe with a positive gram stain and a club shape. Clinically, the patient appears toxic and a bronze discoloration of the involved skin is characteristic. Crepitus may be absent. Empirical therapy must cover clostridial infection. As opposed to clostridium difficile, in cases of suspected C. perfringens myonecrosis or anaerobic cellulitis and necrotizing polymicrobial infection, two drug combination treatment is recommended. I.V. penicillin (2 to 3 million units every 3 hours or 3 to 4 million units every 4 hours) or ampicillin (2 g every 4 hours), plus I.V. clindamycin (0.6 g every 6 to 8 hours) or metronidazole (1 g loading dose followed by 0.5 g every 6 hours), provides coverage of the anaerobic organisms likely to be involved. Vancomycin, cefazolin and tetracycline do not provide effective coverage of anaerobic gram positive species. Fungal wound infection is not suspected with the clinical picture and thus fluconazole is not appropriate.Pasternak MS, Swartz MN: Cellulitis, necrotizing fasciitis, and subcutaneous tissue infections, in Mandell GL, et al: MANDELL, DOUGLAS, AND BENNETT'S PRINCIPLES AND PRACTICE OF INFECTIOUS DISEASES, ed 7. Philadelphia, Churchill Livingstone Elsevier, 2010, pp 1289-1312. 2013 Adult Infection & Inflammatory Disease )

During the course of a radical cystectomy, a rectal injury is primarily repaired. Three days postoperatively, the patient becomes septic. Physical examination reveals a tender surgical wound. The adjacent skin is edematous and has a bronze discoloration. Gram strain of the wound aspirate reveals gram-positive, club-shaped organisms. The next steps are surgical drainage, I.V. clindamycin and: A) penicillin. B) vancomycin. C) fluconazole. D) cefazolin. E) tetracycline.

B ( ureteral stent. A small stone fragment pushed through the wall of the ureter, if completely outside the wall and uninfected, is rarely a clinical problem and no intervention, such as retroperitoneoscopy or basket extraction, needs to be directed towards it. The ureteral perforation through which this fragment passed is managed by ureteral stenting; observation without a stent would lead to extravasation and a higher risk of stricture. Percutaneous nephrostomy and drain placement are not necessary if a stent can be inserted. Johnson DB, Pearle MS: Complications of ureteroscopy. UROL CLIN AMER 2004; 31:157-171. 2013 Adult Calculous Disease )

During ureteroscopic lithotripsy of an impacted 8 mm calcium oxalate stone in the proximal ureter, the ureter is cleared, but a 3 mm fragment is detected on fluoroscopy 1 cm lateral to the ureter. The next step is: A) observation. B) ureteral stent. C) basket extraction. D) percutaneous nephrostomy. E) retroperinoscopy.

B ( cheese. Low urinary pH predisposes to uric acid and calcium oxalate stones. Dietary acid load correlates with increased risk for stone formation. While cheese has one of the highest potential renal acid load (PRAL), milk and yogurt are less acidic and convey only a slight PRAL. The only foods with a net negative PRAL (alkaline load) are fruits and vegetables and should be encouraged in patients with stone disease. While dairy intake is encouraged to decrease the risk of enteric hyperoxaluria, cheese should be de-emphasized as the primary source. Penniston K: Role of diet in stone prevention, in Pearle MS, Nakada S (eds): UROLITHIASIS: MEDICAL AND SURGICAL MANAGEMENT. London, Informa Healthcare, 2009, chap 4, p 42. 2013 General Calculous Disease )

Food with high potential renal acid load (PRAL) includes: A) milk. B) cheese. C) yogurt. D) fruit. E) vegetables.

E ( hypogastric. Blood supply to the rectum arises proximally from the superior rectal (hemorrhoidal) artery, inferior mesenteric artery, and distally from the middle (hemorrhoidal) and inferior rectal arteries. When the inferior mesenteric artery is ligated, blood supply to the rectum is maintained by the middle hemorrhoidal artery which is a branch of the posterior division of the hypogastric artery and the inferior rectal artery which is a branch of the internal pudendal artery.Sheinfeld J, Bosl GJ: Surgery of testicular tumors, Wein AJ, Kavoussi LR, Novick AC, Partin AW, Peters CA (eds): CAMPBELL-WALSH UROLOGY, ed 10. Philadelphia, Elsevier Saunders, 2012, vol 1, chap 32, pp 876-880. 2013 Adult Congenital Anomalies, Embryology, Anatomy )

If the inferior mesenteric artery is ligated, the artery that maintains blood supply to the rectum is: A) superior mesenteric. B) ileocolic. C) middle sacral. D) external iliac. E) hypogastric.

B ( dorsally. The clitoral neural anatomy in the masculinized female patients with CAH is similar to that of the normal male or female phallus. At the mid-portion of the enlarged clitoral shaft, the nerves are found dorsally. This is the area which must be preserved for possible future genital sensation after the feminizing genitoplasty.Rink RC, Kaefer M: Surgical management of disorders of sexual differentiation, cloacal malformation, and other abnormalities of the genitalia in girls, Wein AJ, Kavoussi LR, Novick AC, Partin AW, Peters CA (eds): CAMPBELL-WALSH UROLOGY, ed 10. Philadelphia, Elsevier Saunders, 2012, vol 4, chap 134, p 3652. 2013 General Congenital Anomalies, Embryology, Anatomy )

In 46 XX female patients with CAH, the clitoral nerves at the mid-portion of the enlarged phallic shaft are found: A) ventrally. B) dorsally. C) laterally. D) between the urethra and vagina. E) between the shafts of the corpora cavernosa.

C ( micropapillary variant. Patients with histologic variants of urothelial carcinoma including squamous differentiation and small cell component actually appear to respond better to neoadjuvant chemotherapy. Similarly patients with a p53 mutation and lymphovascular invasion are considered higher risk patients with urothelial cancer and are recommended to have neoadjuvant chemotherapy. Micropapillary variant of urothelial carcinoma is the one variant that does not appear to respond to chemotherapy and requires immediate cystectomy.Scosyrev E, Ely BW, Messing EM, et al: Do mixed histological features affect survival benefit from neoadjuvant platinum-based combination chemotherapy in patients with locally advanced bladder cancer? A secondary analysis of Southwest Oncology Group-Directed Intergroup Study (S8710). BJU INT 2011;108:693-699.Kamat AM, Dinney CP, Gee JR, et al: Micropapillary bladder cancer: A review of the University of Texas M. D. Anderson Cancer Center experience with 100 consecutive patients. CANCER 2007;110:62-67. 2013 Adult Neoplasm )

In a patient with muscle invasive urothelial carcinoma, the pathologic characteristic that predicts a poor response to neoadjuvant chemotherapy is: A) squamous differentiation. B) lymphovascular invasion. C) micropapillary variant. D) p53 mutation. E) small cell component.

D ( decrease response rate. In a meta-analysis of four randomized trials in 286 patients, Galsky and colleagues have concluded that the substitution of carboplatin for cisplatin resulted in a statistically significant (three fold decrease) in the probability of achieving a complete response and a significant decrease in the overall response rate. No significant effect on survival could be analyzed and in general the renal safety profile is improved with the use of carboplatin. The duration of therapy is not affected by the substitution of carboplatin.Galsky MD, Chen GJ, Oh WK, et al: Comparative effectiveness of cisplatin-based and carboplatin-based chemotherapy for treatment of advanced urothelial carcinoma. ANN ONCOL 2012;23:406-410. 2013 Adult Neoplasm )

In the management of advanced bladder cancer the substitution of carboplatin for cisplatin in a multidrug regimen has been shown to: A) not affect outcome. B) increase renal toxicity. C) improve survival. D) decrease response rate. E) increase duration of therapy.

E ( produces interleukins. An integral part of the immune response involves the activation of T cells by dendritic or antigen presenting cells. This interaction occurs via the T cell receptor in the context of MHC (major histocompatibility complex) class II molecules. B cells, unlike T cells, can be directly stimulated by antigen, which then allows B cell differentiation into antibody producing plasma cells. TNF (tumor necrosis factor) and other cytokines are produced by activated T cells and augment the cellular and humoral immune response. An FDA-approved autologous dendritic cell therapy, sipuleucel-T, is currently available for the treatment of advanced prostate cancer.Dahm P, Vieweg J: Evolving immunotherapeutic strategies for the treatment of prostate and renal carcinomas. AUA UPDATE SERIES 2004, vol 23, lesson 4.Flechner SM, Finke JH, Fairchild RL: Basic principles of immunology, Wein AJ, Kavoussi LR, Novick AC, Partin AW, Peters CA (eds): CAMPBELL-WALSH UROLOGY, ed 10. Philadelphia, Elsevier Saunders, 2012, vol 1, chap 17, pp 504-505. 2013 General Physiology, Immunology, & Adrenal )

In the management of advanced urologic malignancies, stimulation of T cells by dendritic cells: A) is restricted to the Class I MHC (major histocompatibility complex). B) causes T cell differentiation into plasma cells. C) is suppressed by TNF (tumor necrosis factor). D) is augmented by blockade of the T cell receptor. E) produces interleukins.

E ( bladder wall viscoelasticity. The main determinants of compliance are the elastic and viscoelastic properties of the bladder. When these are destroyed as in fibrosis of the bladder, poor compliance results. Neurogenic influences may be operative in late stages of filling. Bladder smooth muscle maintains a steady level of contractility and tone that is dependent on the activity in the autonomic nerves, circulating hormones, local metabolites, locally secreted agents such as nitric oxide, and temperature.Wein AJ, Dmochowski RR: Neuromuscular dysfunction of the lower urinary tract, Wein AJ, Kavoussi LR, Novick AC, Partin AW, Peters CA (eds): CAMPBELL-WALSH UROLOGY, ed 10. Philadelphia, Elsevier Saunders, 2012, vol 3, chap 65, pp 1909-1912. 2013 General Physiology, Immunology, & Adrenal )

Normal bladder compliance during physiologic filling is primarily due to: A) autonomic nerve activity. B) circulating hormones. C) cholinergic receptor activity. D) local secretion of nitric oxide. E) bladder wall viscoelasticity.

C ( transitional and peripheral. The various zones of the prostate are not always easily distinguished on ultrasound. In glands with large adenoma, the transitional zone can often be distinguished from the anterior zone and the peripheral zone because of its more heterogeneous appearance. In addition, some patients will develop calcifications along the surgical capsule between the transitional zone and the peripheral zone. These calcifications are known as corpora amylacea and can be used on ultrasound to define the boundaries of these two zones.Trabulsi EJ, Halpern EJ, Gomella LG: Ultrasonography and biopsy of the prostate, Wein AJ, Kavoussi LR, Novick AC, Partin AW, Peters CA (eds): CAMPBELL-WALSH UROLOGY, ed 10. Philadelphia, Elsevier Saunders, 2012, vol 3, chap 97, p 2735. 2013 Adult Core Competencies, Geriatric, Radiation Safety and Ultrasound )

On prostate ultrasound, calcifications within the prostate known as corpora amylacea can be visualized between which zones: A) transitional and anterior. B) central and peripheral. C) transitional and peripheral. D) central and transitional. E) central and anterior.

E ( retinal angiomas. Patients with von-Hippel Lindau disease may have hemangioblastomas of the cerebellum, renal cell carcinomas, and cystadenomas of the epididymis. The diagnosis, however, can often be made most easily with inspection of the retina with identification of angiomas. Renal angiomyolipomas are commonly seen in tuberous sclerosis complex. Thyroid carcinoma can be seen more commonly in patients with multiple endocrine neoplasia syndrome. Cafe-au-lait spots are pathognomonic of neurofibromatosis.Neumann HP, Berger DP, Sigmund G, Blum U, et al: Pheochromocytomas, multiple endocrine neoplasia type 2, and von Hippel-Lindau disease. NEJM 1993;329(21):1531-1538.Campbell SC, Lane BR: Malignant renal tumors, Wein AJ, Kavoussi LR, Novick AC, Partin AW, Peters CA (eds): CAMPBELL-WALSH UROLOGY, ed 10. Philadelphia, Elsevier Saunders, 2012, vol 2, chap 49, p 1423. 2013 Adult Neoplasm )

Patients with von-Hippel Lindau disease most frequently have: A) renal angiomyolipoma. B) cafe-au-lait spots. C) glioblastomas. D) thyroid carcinoma. E) retinal angiomas.

B ( citrate. Performance sports drinks may increase urinary citrate and pH; lending a protective effect against urinary lithogenicity. However, these drinks have a high fructose and total carbohydrate content, so they should not be recommended as the primary means of hydration for stone formers. Though the sodium content may be high in these drinks, they do not lead to hypernatruria. Sports drinks have no effect on urinary calcium, oxalate, and uric acid. Goodman JW, Asplin JR, Goldfarb DS: Effect of two sports drinks on urinary lithogenicity. UROL RES 2009;37:41-46. 2013 Adult Calculous Disease )

Performance sports drinks may increase urinary: A) sodium. B) citrate. C) calcium. D) uric acid. E) oxalate.

A ( vaginismus. Sexual dysfunction in women is complicated in that many of the disorders coexist and isolated disorders are uncommon. Definitions frequently change or are not universally agreed upon. In the Revised Definitions for Female Sexual Dysfunction from the Second International Consensus of Sexual Medicine vaginismus is the persistent difficulty to allow entry of an object into the vagina despite the desire of the woman to participate. Dyspareunia on the other hand is persistent or recurrent pain with attempted penile-vaginal intercourse. Persistent sexual arousal disorder, sexual aversion disorder and objective arousal disorder are not primarily associated with the difficulty to insert an object into the vagina. Genital arousal disorder consists of complaints of impaired genital sexual arousal, which may include minimal vulvar swelling or vaginal lubrication from any type of sexual stimulation and reduced sexual sensations from caressing genitalia. However, subjective sexual excitement still occurs with nongenital sexual stimuli. In persistent sexual arousal disorder, the patient has spontaneous, intrusive, and unwanted genital arousal in the absence of sexual interest and desire. Arousal is unrelieved by orgasms and the feelings of arousal persist for hours or days. Extreme anxiety or disgust at the anticipation of or attempt at any sexual activity is sexual aversion disorder.Moore CK: Female sexual function and dysfunction, Wein AJ, Kavoussi LR, Novick AC, Partin AW, Peters CA (eds): CAMPBELL-WALSH UROLOGY, ed 10. Philadelphia, Elsevier Saunders, 2012, vol 1, chap 30, p 826. 2013 Adult Sexual Dysfunction, Endocrinopathy, Fertility Problems )

Persistent or recurrent difficulty to allow vaginal entry of a penis, finger, or other object, despite the woman's desire to participate is consistent with the diagnosis of: A) vaginismus. B) persistent sexual arousal disorder. C) dyspareunia. D) sexual aversion disorder. E) genital arousal disorder.

E ( the presence of oligohydramnios and renal cortical cysts. The most reliable indicators of postnatal renal function are the presence or absence of oligohydramnios and renal cortical cysts. Severe oligohydramnios and the presence of renal cortical cysts predict a poor prognosis for postnatal renal function. The anterior-posterior (AP) diameter of the renal pelvis is somewhat predictive of the likelihood of postnatal surgical intervention, but not predictive of postnatal renal function. An enlarged, thick-walled bladder with a dilated proximal urethra indicates bladder outlet obstruction but does not indicate the outcome of renal function. Similar to the AP diameter, the degree of parenchymal thinning is helpful in appreciating the severity of blockage and the likelihood for the need to intervene. Increased renal parenchymal echogenicity may, at times, be associated with impaired renal development and function, but in the absence of cortical cysts is unreliable.Peters CA, Chevalier RL: Congenital urinary obstruction: Pathophysiology and clinical evaluation, Wein AJ, Kavoussi LR, Novick AC, Partin AW, Peters CA (eds): CAMPBELL-WALSH UROLOGY, ed 10. Philadelphia, Elsevier Saunders, 2012, vol 4, chap 113, p 3040.Lee RS, Borer JG: Perinatal urology, Wein AJ, Kavoussi LR, Novick AC, Partin AW, Peters CA (eds): CAMPBELL-WALSH UROLOGY, ed 10. Philadelphia, Elsevier Saunders, 2012, vol 4, chap 114, p 3048. 2013 Pediatric Obstructive Uropathy, Laparoscopy/Robotics )

Prenatal ultrasound in a 22-week male fetus shows bilateral hydroureteronephrosis. The most important parameter(s) predictive of postnatal renal outcome is(are): A) the anterior-posterior diameter of the renal pelvis. B) renal parenchymal echogenicity. C) the degree of parenchymal thinning. D) the presence of a thick walled bladder and a dilated proximal urethra. E) the presence of oligohydramnios and renal cortical cysts.

1. Oliguria associated with acute tubular necrosis is characterized by which urinary findings: A.. t sodium t urea t osmolality. B.. t sodium t urea t osmolality. C. t sodium t urea t osmolality. D.. t sodium t urea t osmolality. E.. t sodium t urea t osmolality.

Question #1 ANSWER=A Most major causes of acute renal failure can be differentiated by urinalysis and urinary chemistries. With tubular cell injury the kidney is no longer able to reabsorb filtered salt and water (under normal conditions approximately 99% of filtered NaCl and water are reabsorbed and 50-80% of all filtered urea is excreted). In acute tubular necrosis renal tubular function is injured resulting in loss of filtered water which causes a decrease in urinary osmolality. In addition failure to resorb filtered sodium and failure to excrete urea (other functions of the renal tubular cell) will result in an increased urinary sodium and decreased urinary urea. Paller MS: Pathophysiology of acute renal failure in Greenberg A (ed): PRIMERON KIDNEY DISEASES. San Diego Academic Press 1994 pp 126-133. Goldfarb DA Poggio ED Demirjian S: Etiology pathogenesis and management of renal failure in Wein AJ Kavoussi LR Partin AW Peters CA (eds): CAMPBELL-WALSH UROLOGY ed 11. Philadelphia Elsevier 2015 vol 2 chap 46 pp 1048-1049.

10. A 42-year-old man with azoospermia and primary infertility has a FSH of 15 mlU/L small volume testes and an otherwise normal physical examination. The factor that most reliability predicts his ability to have a biologic child is: A.. vasography. B.. serum FSH. C. wife's fertility. D.. testicular volume. E.. testicular biopsy.

Question #10 ANSWER=C The presence of small volume testes with an elevated FSH suggests the presence of nonobstructive azoospermia. Most men with non-obstructive azoospermia will have sperm retrievable from the testes that can be used in conjunction with in vitro fertilization for the wife. The most important characteristic to determine eligibility for treatment will be the wife's age and fertility. Screening for obstruction with vasography is not of value. Testicular biopsy may be useful as an indicator for success with intracytoplasmic sperm injection (ICSI) and sperm harvest. With an elevated FSH diagnostic biopsy is not indicated. Carpi A Sabanegh E Mechanick J: Controversies in the management of nonobstructive azoospermia. FERT STERIL 2009;91 :963-970. Epub 2009 Mar 21. Niederberger CS: Male infertility in Wein AJ Kavoussi LR Partin AW Peters CA (eds): CAMPBELLWALSH UROLOGY ed 11. Philadelphia Elsevier 2015 vol 1 chap 24 p 573.

11. A 34-year-old woman is hypertensive. Laboratory studies reveal a serum sodium of 149 mEq/L potassium 2.9 mEq/L and CO2 28 mEq/L. Plasma renin activity is suppressed. A CT scan reveals an enlarged left adrenal gland but no distinct mass. The next step is: 4 A.. B .. C. D .. E .. spironolactone. nifedipine. MRI scan of adrenal. serum aldosterone:renin ratio. adrenal vein aldosterone sampling. Copyright 2017 by The American Urological Association.

Question #11 ANSWER=E This woman has hypertension due to primary hyperaldosteronism. The CT scan suggests hyperplasia of the left adrenal gland. In order to differentiate hyperplasia from an adenoma adrenal vein sampling for aldosterone will show elevated levels on the left and suppressed levels on the right if an adenoma is present. MRI scan will not differentiate between an adenoma and hyperplasia. A serum aldosterone:renin ratio will not lateralize the lesion. If adrenal vein sampling does not lateralize then medical therapy with spironolactone is indicated rather than nifedipine which is not potassium sparing. If an adenoma is present surgical removal is the best treatment. Kutikov A Crispen PL Uzzo RG: Pathophysiology evaluation and medical management of adrenal disorders in Wein AJ Kavoussi LR Partin AW Peters CA (eds): CAMPBELL-WALSH UROLOGY ed 11. Philadelphia Elsevier 2015 vol 2 chap 65 pp 1541-1544. 4 Copyright 2017 by The American Urological Association. ??

2. The most important factor responsible for the frequent recurrence of UTls in an otherwise healthy young woman is: A.. adhesive fimbriae of uropathogens. B.. specific receptors on urothelial cells. C. presence of pathogenic coliforms in stool. D.. feminine hygiene practices. E.. method of contraception.

Question #2 ANSWER=B Properties of uropathogens sexual activity feminine hygiene practices and the use of an IUD and/or spermicide may increase the frequency of UTls in predisposed women; however they are not the most important etiologic factors. Many women who have uropathogenic bacteria present in their bowel use various contraceptive and hygiene methods and are sexually active without developing infections. E. coli must colonize the peri-urethral area before an uncomplicated infection can occur. Coliform organisms are recovered only rarely from the region of the vaginal vestibule and external urethra in otherwise healthy women who do not have recurrent UTls. It is postulated by most researchers that host factors rather than specific pathogenicity of the micro-organisms are the prime determinants of colonization. E. coli tend to adhere more to vaginal and buccal epithelial cells obtained from women with recurrent infection than to controls. This explains why certain women are prone to frequent recurrent infections. It would also explain why women with asymptomatic bacteruria are more prone to recurrent infection with marriage and pregnancy and would account for UTls associated with intercourse various contraceptive methods etc. in highly susceptible women. Schaeffer AJ Matulewicz RS Klumpp DJ: Infections of the urinary tract in Wein AJ Kavoussi LR Partin AW Peters CA (eds): CAMPBELL-WALSH UROLOGY ed 11. Philadelphia Elsevier 2015 vol 1 chap 12 pp 241-248.

3. The antimicrobial agent that can be used at the usual dosage in an azotemic patient is: A.. nitrofurantoin. B.. sulfamethoxazole. C. doxycycline. D.. trimethoprim. E.. fluconazole.

Question #3 ANSWER=C All the antibiotics listed including most tetracyclines except doxycycline are excreted primarily in the urine and their blood levels increase in the presence of renal insufficiency. Doxycycline is excreted mainly in the feces and does not require consideration for a dosage reduction in an azotemic patient. Gilbert B Robbins P Livornese LL: Use of antibacterial agents in renal failure. MED CUN AM 2011;95:677-702.

4. A 23-year-old woman suffers a complex pelvic fracture in an MVC. A cystogram reveals limited extraperitoneal extravasation of contrast at the bladder neck. The bladder is compressed by a pelvic hematoma and an anterior vaginal laceration is also present. No other injuries are noted and she is hemodynamically stable. Treatment should be: A.. urethral catheter drainage. B.. suprapubic cystostomy. C. urethral catheter placement and repair of vaginal lacerations. D.. bladder repair and vaginal packing. E.. repair of vaginal and bladder lacerations.

Question #4 ANSWER=E Urethral and bladder neck injuries in women are rare but potentially devastating in their effects on long-term continence and bladder function. The urethra is short mobile and protected by the pubis in women. Female urethral and bladder neck injuries occur in 4.6% to 6% of women suffering pelvic fractures. The typical presentation includes gross hematuria or blood at the introitus. Despite blood in the vaginal vault over 40% of female bladder neck and urethral injuries are missed in the emergency department and only half will be detected on CT cystogram. As a result one must have a high index of suspicion and low threshold for performing a vaginal examination in females with pelvic fractures. Female bladder neck injuries should undergo immediate repair with primary closure of any vaginal lacerations to prevent fistula formation. Longitudinal tears of the female bladder neck have been associated with higher rates of incontinence. Such injuries should be repaired immediately to preserve the functional integrity of the bladder neck. In one recent series despite operative repair 16% of women developed vesicovaginal fistulas 43% had moderate or severe lower urinary tract systems and 38% had sexual dysfunction. Black P Miller E Porter JR Wessells H: Urethral and bladder neck injury associated with pelvic fracture in 25 female patients. J UROL 2006;175(6):2140-2144. Morey AM Brandes S Dugi Ill DD et al: UROTRAUMA: AUA GUIDELINE. American Urological Association Education and Research Inc 2014. http://www.auanet.org/education/guidelines/urotrauma.cfm

5. A two-year-old boy with normal penile development is explored for non-palpable testes through bilateral groin incisions. On each side the vas deferens and spermatic vessels end blindly at the internal ring. The next step is: A.. observation. B.. CT scan of abdomen. C. serum inhibin B and abdominal ultrasound. D.. FSH LH testosterone level and stimulate with hCG. E.. diagnostic laparoscopy. Copyright 2017 by The American Urological Association. 1

Question #5 ANSWER=A In a young patient with absent testes and normal penile development testosterone stimulation was present at 16 weeks gestation. Loss of testicular function before this time leads to inadequate virilization. The finding of a blind-ending vas deferens and vessels is adequate to define the pathology and further exploration in this case is unnecessary. Chromosomal study of such cases is usually unnecessary as they carry none of the stigmata of intersexuality and will have a normal (46 XY) karyotype. At age of puberty such anorchid patients will have elevated gonadotropin and require testosterone therapy. Barthold JS Hagerty JA: Etiology diagnosis and management of the undescended testis in Wein AJ Kavoussi LR Partin AW Peters CA (eds): CAMPBELL-WALSH UROLOGY ed 11. Philadelphia Elsevier 2015 vol 4 chap 148 p 3439. Kolon TF Herndon CDA Baker LA et al: EVALUATION AND TREATMEN T OF CYP TORCHIDISM: AUA GUIDELINE. American Urological Association Education and Research Inc 2014. http://www.auanet.org/education/guidelines/cryptorchidism.cfm

6. A 55-year-old woman who had a sacral neuromodulation implant placed four years 2 a ' dining efficacy despite several reprogramming sessions. A plain film Xray is shown. The next step is: A.. B .. C. D .. E .. revise revise e electrodes deeper. e lead more laterally ini 53. revise e IP.:irl in <-LL. revise e lead with ' · 'et. remove IPG and lead initiate 200 units onabotulinumtoxinA injections. Copyright 2017 by The American Urological Association. Copyright 2017 by The American Urological Association. 3

Question #6 ANSWER=D This plain film AP view shows the lead lateral in the S3 foramen. The lateral view shows it too deeply placed and this puts her at risk for deep stimulation of S2 roots causing leg and other lower extremity untoward stimulation. The use of the curved stylet would allow placement of the lead into S3 in a more medial to lateral configuration thereby allowing maximal contact of 2 Copyright 2017 by The American Urological Association. electrodes to the nerve. This is due to the nerve following a medial to lateral course. Revising leads to place deeper may create stimulation of leg and other untoward effects as mentioned above. S4 stimulation has not been shown to create better efficacy than S3. Lateral lead placement would not allow best contact with the nerve. It would be premature to remove the system and start onabotulinumtoxinA injections. If ultimately utilized the dose onabotulinumtoxinA used for OAB is 100 units. Jacobs SA Lane FL Osann KE Noblett KL: Randomized prospective crossover study of lnterStim lead wire placement with curved versus straight stylet. NEUROUROL URODY N 2014;33:488-492. Vasavada SP Rackley RR: Electrical stimulation and neuromodulation in storage and emptying failure in Wein AJ Kavoussi LR Partin AW Peters CA (eds): CAMPBELL-WALSH UROLOGY ed 11. Philadelphia Elsevier 2015 vol 3 chap 81 p 1899.

7. A five-day-old boy has vomiting and dehydration. His serum CO2 is 12 mEq/L K+ 5.5 mEq/L and creatinine 2.2 mg/dl. A VCUG demonstrates PUV and bilateral grade 4 VUR. The next step is: A.. percutaneous cystostomy. B.. percutaneous nephrostomies. C. valve ablation. D.. urethral catheter drainage. E.. cutaneous vesicostomy.

Question #7 ANSWER=D The management of the infant with a PUV depends on the severity of the obstruction and the degree of any renal dysplasia present. The main problems arise in management of the infant with severe obstruction and compromised renal function with dehydration acidosis and sepsis. Initially a small infant feeding tube placed transurethrally can provide bladder drainage. Once stabilized valve ablation can be undertaken. Vesicostomy is reserved for infants who cannot undergo primary valve ablation because of the inadequate size of their urethra or for very small unstable infants. If initial bladder level drainage does not result in satisfactory clinical improvement temporary supravesical diversion may be considered; however the vast majority of these patients will be found to have renal dysplasia not ureterovesical obstruction as the etiology of the persistently elevated creatinine. Shukla AR: Posterior urethral valves and urethral anomalies in Wein AJ Kavoussi LR Partin AW Peters CA (eds): CAMPBELL-WALSH UROLOGY ed 11. Philadelphia Elsevier 2015 vol 4 chap 141 pp 3258-3259.

8. A 40-year-old man suffers a gunshot to the abdomen with left ureteral transection at the L3 vertebral level and a ureteroureterostomy is performed. Post-operatively he is not able to flex his thigh. These deficits are due to injury to the: A.. femoral nerve. B.. ilioinguinal nerve. C. genitofemoral nerve. D.. lateral femoral cutaneous nerve. E.. obturator nerve.

Question #8 ANSWER=A The femoral nerve arises from the second third and fourth lumbar spinal segments. It appears at the lateral edge of the psoas muscle and descends into the thigh. It supplies a number of muscles including the quadriceps femoris complex articularis genu sartorius pectineus and iliopsoas. llioinguinal genitofemoral and lateral femoral cutaneous nerves are sensory nerves. The obturator nerve would be responsible for adduction of his leg. Berry M Bannister LH Standring SM: Nervous system in Bannister LH Berry MM Collins P Dyson M Dussek JE Ferguson MWJ (eds): GRAY'S ANATOMY ed 38. New York Churchill Livingstone 1995 chap 8 pp 1280-1282. Palmer DA Moinzadeh A: Surgical radiographic and endoscopic anatomy of the retroperitoneum in Wein AJ Kavoussi LR Partin AW Peters CA (eds): CAMPBELL-WALSH UROLOGY ed 11. Philadelphia Elsevier 2015 vol 1 chap 33 p 765. Copyright 2017 by The American Urological Association. 3

9. A two-month-old uncircumcised boy with a sacral dimple undergoes evaluation of a febrile UTI. Ultrasound shows bilateral hydroureteronephrosis and a conus medullaris at the mid-aspect of L4. VCUG shows bilateral grade 4 reflux and a normal urethra. The next step is: A.. CMG. B.. cystoscopy. C. MAG-3 renal scan. D.. circumcision. E.. vesicostomy.

Question #9 ANSWER=A This infant has a compromised urinary tract and a neurogenic cause must be considered. The conus normally ends above L3 and spinal ultrasound is a convenient and accurate method of screening in the neonatal period. Given his low con us a CMG would be important to see if filling curve and storage pressure are abnormal with abnormal urodynamic findings substantiating the presence of a clinically significant tethered cord. Circumcision is not mandatory. Vesicostomy at this point is premature and cystoscopy is not necessary. The hydronephrosis in this case is related to the bladder dysfunction and a MAG-3 scan is unnecessary. Maclellan DL Bauer SB: Neuromuscular dysfunction of the lower urinary tract in children in Wein AJ Kavoussi LR Partin AW Peters CA (eds): CAMPBELL-WALSH UROLOGY ed 11. Philadelphia Elsevier 2015 vol 4 chap 142 pp 3285-3286.

A ( on average 50 times greater than that from an anterior-posterior abdominal x-ray. There are an estimated 60-70 million CT scans performed in the USA, perhaps with 33% being unnecessarily performed. CT scans generate ionizing radiation with resulting DNA damage that could result in the induction of cancer. The cancer risk of CT scans is higher in the pediatric population. Furthermore, the digestive organs are more sensitive to radiation injury than the brain. Newer CT scans have automatic exposure-control option which will decrease the radiation exposure. An abdominal x-ray results in a dose of 0.25 mSv to the stomach whereas a single CT scan of the abdomen can result in a radiation dose 50 times or greater to the stomach.Brenner DJ, Hall EJ: Computed tomography: An increasing source of radiation exposure. NEJM 2007;357:2277-2284. 2013 Adult Core Competencies, Geriatric, Radiation Safety and Ultrasound )

Radiation exposure from a single abdominal CT scan is: A) on average 50 times greater than that from an anterior-posterior abdominal x-ray. B) is less harmful to the digestive organs compared to the brain. C) results in less cancer risk in younger patients. D) increased with automatic exposure-control option. E) the result of non-ionizing radiation.

C ( acetohydroxamic acid. Acetohydroxamic acid (AHA, Lithostat®) has been demonstrated in randomized clinical trials to decrease the rate of stone growth in patients with struvite calculi. It is important to note that AHA did not impact the rate of stone recurrence. Urinary acidification has been tested in vitro utilizing L-methionine, but has not been studied in humans. Though recurrence rates are higher (62%) in patients with infected urine compared to sterile urine (12%), the use of suppressive antibiotics to decrease stone growth has not been studied in a clinical trial. A low protein diet, although effective in feline studies, has not been shown to make a difference in human studies. Citric acid glucono-delta-lactone magnesium carbonate (Renacidin®) has been utilized for dissolution therapy of residual fragments.Park S: Pathophysiology and management of infection stones. in Pearle MS, Nakada S (eds): UROLITHIASIS: MEDICAL & SURGICAL MANAGEMENT. London, Informa Healthcare, 2009, chap 10, pp 129-130. 2013 Adult Calculous Disease )

Randomized clinical trials have demonstrated decreased rates of struvite stone growth with the use of: A) urinary acidification. B) suppressive antibiotics. C) acetohydroxamic acid. D) a low protein diet. E) citric acid glucono-delta-lactone magnesium carbonate.

D ( occurs by an active transport mechanism. About two-thirds of the glomerular ultrafiltrate is reabsorbed in the proximal tubule with little change in the osmolality or sodium concentration of the unreabsorbed fraction. In other words, fluid reabsorption in the proximal tubule is nearly isosmotic and is coupled to the active transport of sodium. Since chloride and bicarbonate are the primary anions in the extracellular fluid, most of the filtered sodium is reabsorbed with these anions. Because of the high water permeability of the proximal tubule, sodium transport occurs against a minimal concentration gradient. Aldosterone regulates sodium-potassium exchange in the collecting duct.Shoskes DA, McMahon AW: Renal physiology and pathophysiology, Wein AJ, Kavoussi LR, Novick AC, Partin AW, Peters CA (eds): CAMPBELL-WALSH UROLOGY, ed 10. Philadelphia, Elsevier Saunders, 2012, vol 2, chap 38, pp 1032-1033. 2013 Adult Physiology, Immunology, & Adrenal )

Sodium reabsorption in the proximal tubule: A) results in a hypotonic tubular fluid. B) occurs against a steep concentration gradient. C) is accompanied by bicarbonate excretion. D) occurs by an active transport mechanism. E) is regulated by aldosterone.

C ( ritonavir. High fat meals can inhibit the absorption of sildenafil and vardenafil but do not affect the absorption of tadalafil or the metabolism of PDE5 inhibitors. Ketoconazole, itraconazole, and protease inhibitors such as ritonavir can impair the metabolic breakdown of PDE5 inhibitors by blocking the CYP3A4 pathway. These agents may increase blood levels of inhibitors, requiring a PDE5 dose reduction. Agents such as rifampin may induce CYP3A4, enhancing the breakdown of inhibitors and requiring higher PDE5 doses. Warfarin and doxazosin have no effect on the metabolism of PED5 inhibitors. Doxazosin may exacerbate hypotensive changes with PED5 inhibitors.Lue TF, Broderick GA: Evaluation of nonsurgical management of erectile dysfunction and premature ejaculation, in Wein AJ, Kavoussi LR, Novick AC, Partin AW, Peters CA (eds): CAMPBELL-WALSH UROLOGY, ed 10. Philadelphia, Elsevier Saunders, 2012, vol 1, chap 22, p 750.Burnett AL: Evaluation and management of erectile dysfunction, Wein AJ, Kavoussi LR, Novick AC, Partin AW, Peters CA (eds): CAMPBELL-WALSH UROLOGY, ed 10. Philadelphia, Elsevier Saunders, 2012, vol 1, chap 24, p 742. 2013 Adult Sexual Dysfunction, Endocrinopathy, Fertility Problems )

The metabolism of sildenafil may be inhibited by: A) a fatty meal. B) rifampin. C) ritonavir. D) warfarin. E) doxazosin.

E ( leaving a catheter through superficial portion of stoma nightly. The usual first line approach for the problem of cutaneous stenosis of a Monti or Mitrofanoff channel is to leave a catheter through the stoma for some period of time. The concern with leaving a full-time indwelling catheter to stent the stoma for several days is that it may plug with mucous and introduce bacteria along its surface into the bladder. Mickelson et. al. reported use of the L-stent a catheter with a knot tied just 1-2 inches from the tip inserting it up to the knot at night with taping to hold it in place. This serves to stent only the cutaneous stoma and not enter the bladder itself. Their success with a modest period of nighttime stenting was excellent. None of the other approaches listed have shown particular impact on this problem.Mickelson JJ Yerkes EB Meyer T et al: L stent for stomal stenosis in catheterizable channels. J UROL 2009;182:1786-1791. 2013 Pediatric Urinary Diversion )

The need for operative revision of stomal stenosis in a continent, catheterizeable channel is reduced by: A) increasing size of catheter. B) increasing frequency of catheterization. C) steroid injection of stoma. D) use of a pre-lubricated catheter. E) leaving a catheter through superficial portion of stoma nightly.

D ( a direct toxic effect on renal tubular cells. Cisplatin nephrotoxicity is due to a direct toxic effect of the drug on renal tubular cells. Azotemia and dehydration are predisposing conditions which increase the risk of this complication. Cisplatin is not precipitated in the renal tubules nor does it affect glomerular hemodynamics. Goldfarb DA, Poggio ED: Etiology, pathogenesis, and management of renal failure, Wein AJ, Kavoussi LR, Novick AC, Partin AW, Peters CA (eds): CAMPBELL-WALSH UROLOGY, ed 10. Philadelphia, Elsevier Saunders, 2012, vol 2, chap 43, pp 1197-1198. 2013 General Physiology, Immunology, & Adrenal )

The nephrotoxic effect of cisplatin is due to: A) efferent arteriolar constriction. B) afferent arteriolar constriction. C) pre-existing plasma volume contraction. D) a direct toxic effect on renal tubular cells. E) renal tubular obstruction from drug precipitation.

A ( levator and prostatic. The prostate is covered with three distinct and separate fascial layers: Denonvilliers' fascia, the prostatic fascia, and the levator fascia. Denonvilliers' fascia is a filmy, delicate layer of connective tissue located between the anterior walls of the rectum and prostate. The neurovascular bundle on the prostate contain the cavernosal nerves and are located between the layers of the levator fascia and prostatic fascia.Schaeffer EM, Partin AW, Walsh PC: Radical retropubic and perineal prostatectomy, Wein AJ, Kavoussi LR, Novick AC, Partin AW, Peters CA (eds): CAMPBELL-WALSH UROLOGY, ed 10. Philadelphia, Elsevier Saunders, 2012, vol 3, chap 102, p 2801. 2013 Adult Neoplasm )

The neurovascular bundles on the prostate travel between the following two layers of fascia: A) levator and prostatic. B) Denonvilliers' and levator. C) Denonvilliers' and prostatic. D) lateral pelvic and prostatic. E) lateral pelvic and levator.

B ( hyperthyroidism. Sexual dysfunction and infertility can on rare occasions be caused by thyroid dysfunction. Hyperthyroidism/thyrotoxicosis can elevate the serum levels of sex hormone binding globulin (SHBG) and subsequently lower circulating free testosterone. Hypothyroidism can have the opposite effect (lower SHBG) as can insulin administration, glucocorticoid excess, hepatic disease and nephrotic syndrome. Estrogen can increase SHBG production and progestins can lower it.Sabanegh E, Agarwal A: Male infertility, Wein AJ, Kavoussi LR, Novick AC, Partin AW, Peters CA (eds): CAMPBELL-WALSH UROLOGY, ed 10. Philadelphia, Elsevier Saunders, 2012, vol 1, chap 21, p 627. 2013 Adult Physiology, Immunology, & Adrenal )

The production of sex hormone binding globulin (SHBG) is increased by: A) hypothyroidism. B) hyperthyroidism. C) insulin. D) nephrotic syndrome. E) progestins.

D ( early ambulation. The AUA Best Practices Policy on DVT prophylaxis stated that early ambulation is recommended for the vast majority of men undergoing TURP. Those men who are at increased risk for DVT (such as previous DVTs, malignancy, immobility, paresis, etc.) may benefit from pneumatic compressive stockings, subcutaneous low dose unfractionated heparin, or low molecular weight heparin (LMWH). However, the use of LMWH is contraindicated in a patient who receives spinal or epidural anesthesia as this is a FDA black box warning due to risk of spinal hematoma. Aspirin and other antiplatelet drugs, while highly effective at reducing vascular events associated with atherosclerotic disease, are not recommended for DVT prophylaxis in surgical patients. There is no indication for obtaining preoperative LE-duplex studies in an otherwise healthy male.Forrest JB, Clemens JQ, Finamore P, et al: Best practice policy statement for the prevention of deep vein thrombosis in patients undergoing urologic surgery. PREVENTION OF DVT AFTER UROLOGIC SURGERY BEST PRACTICE STATEMENT. American Urological Association Education and Research, Inc, 2008. http://www.auanet.org/content/guidelines-and-quality-care/clinical-guidelines/main-reports/dvt.pdf 2013 Adult Core Competencies, Geriatric, Radiation Safety and Ultrasound )

The recommended method to prevent postoperative DVT in an otherwise healthy man undergoing TURP under spinal anesthesia is: A) subcutaneous low dose unfractionated heparin. B) low molecular weight heparin. C) aspirin. D) early ambulation. E) obtain preoperative lower extremity duplex studies.

A ( A The pneumoperitoneum used in laparoscopy will have an effect on the cardiovascular, renal and respiratory systems. With pressures = 20 mmHg (most commonly used pressure for laparoscopy is 15 mmHg), heart rate, mean arterial pressures and systemic vascular resistance are all increased. Alterations in venous return and cardiac output are variable and are dependent on the hydration of the patient. In the hypovolemic or euvolemic state both venous return and cardiac output are decreased due to compression of the vena cava, however if the patient is hypervolemic, (fluid overloaded) the vena cave will resist the increase in the intraabdominal pressure and both venous return and cardiac output increased. The GFR is decreased due to compression of the renal vein and renal parenchyma by the elevated intraabdominal pressure.Eichel L, Clayman RV: Fundamentals of laparoscopic and robotic urologic surgery, Wein AJ, Kavoussi LR, Novick AC, Partin AW, Peters CA (eds): CAMPBELL-WALSH UROLOGY, ed 10. Philadelphia, Elsevier Saunders, 2012, vol 1, chap 9, p 231. 2013 Adult Obstructive Uropathy, Laparoscopy/Robotics )

With a pneumoperitoneum of 15 mmHg the following changes would be expected: A) A B) B C) C D) D E) E

D ( subdiaphragmatic and mediastinal radiation therapy. In a multivariate analysis, combined subdiaphragmatic and mediastinal radiation has a Hazard's Ratio (HR) of 3.7 for secondary malignancy and/or cardiovascular disease; chemotherapy also increased the risk (HR 1.9), but there is no difference between the chemotherapy regimens. Smoking increased risk 1.7 fold. There is no increased risk of secondary malignancy with surgery. Surveillance imaging is a risk factor for developing secondary malignancy, but the magnitude of the risk is much lower than radiation therapy.Haugnes HS, Wethal T, Aass N, et al: Cardiovascular risk factors and morbidity in long-term survivors of testicular cancer: A 20-year follow-up study. J CLIN ONCOL 2010;28:4649-4657.Abouassaly R, Fossa SD, Giwercman A, et al: Sequale of treatment of testis cancer. EUR UROL 2011;60:516-526.Fung C, Vaughn DJ: Complication associated with chemotherapy in testicular cancer. NATURE REVIEWS UROL 2011;8:213-222. 2013 Adult Neoplasm )

The risk factor associated with the highest chance for developing a secondary malignancy and/or cardiovascular disease in patients treated for testis cancer is: A) surveillance imaging. B) smoking. C) BEP chemotherapy. D) subdiaphragmatic and mediastinal radiation therapy. E) cisplatin, vinblastine, bleomycin (PVB) chemotherapy.

"D: ""accurately estimate, underestimate."" was the correct answer. There is an increased awareness of the radiation risks of computed tomography. In view of this, ultrasound is an attractive alternative to measure stone size. The clinician should be aware of the limitations on ultrasound imaging of urolithiasis. Ultrasound will correlate approximately two thirds of the time with the stone size determined on CT scan. Specifically, the ultrasound will overestimate the size of one third of the stones smaller than 10 mm, and underestimating the size of one third of stones, > 10 mm. KUB underestimates > 90% stones, > 10 mm due in part to its inability to measure in three dimensions. Viprakasit DP, Sawyer MD, Herrell SD, et al: Limitations of ultrasonography in the evaluation of urolithiasis: A correlation with computed tomography. J ENDOUROL 2012;26:209. Parsons JK, Lancini V, Shetye K, et al: Urinary stone size: Comparison of abdominal plain radiography and noncontrast CT measurements. J ENDOUROL 2003;17:725. Westesson K, Monga M: Asymptomatic renal calculi: Incidence and management. AUA UPDATE SERIES, 2012, lesson 36. "

Ultrasound and KUB, ____ and ____ stone size respectively: A. overestimate, overestimate. B. overestimate, underestimate. C. underestimate, overestimate. D. accurately estimate, underestimate. E. underestimate, accurately estimate."

D ( the degree of hydronephrosis varies with serial ultrasounds. On antenatal ultrasound, if the fetus has VUR, then the degree of fetal bladder fullness at the time of any particular study will determine how much volume is being refluxed and, thus, how much hydronephrosis is noted. Since bladder fullness differs between studies, the varying degrees of hydronephrosis is suggestive of VUR. A fuller bladder should cause an increase in hydronephrosis. Upper pole hydroureteronephrosis suggests an obstructive etiology; bladder wall thickness and persistent bladder fullness suggests bladder outlet obstruction or several other possibilities. A two-vessel cord is unrelated to the risk of reflux.Lee RS, Borer JG: Perinatal urology, Wein AJ, Kavoussi LR, Novick AC, Partin AW, Peters CA (eds): CAMPBELL-WALSH UROLOGY, ed 10. Philadelphia, Elsevier Saunders, 2012, vol 4, chap 114, p 3054. 2013 Pediatric Congenital Anomalies, Embryology, Anatomy )

Vesicoureteral reflux is suspected on antenatal ultrasound when: A) there is upper pole hydroureteronephrosis. B) the bladder is noted to be full on serial ultrasounds. C) there is bladder wall thickening. D) the degree of hydronephrosis varies with serial ultrasounds. E) there is a two-vessel umbilical cord.

C ( has a stone projecting over a rib. There is a greater need for supplemental anesthesia during SWL under sedation if the patient is female or young, has a history of anxiety, depression or prior SWL, or has a rib-projected calculus. Pain during SWL has been shown to be less with ureteral calculi when compared to renal calculi.Vergnolles M, Wallerand H, Gadrat F, et al: Predictive risk factors for pain during extracorporeal shockwave lithotripsy. J ENDO 2009;23:2021-2027. 2013 Adult Calculous Disease )

When performing SWL using sedation, one should anticipate a need for deeper anesthesia if the patient: A) is a male. B) is older than 50 years. C) has a stone projecting over a rib. D) has never had a prior SWL. E) has a ureteral calculus.

C ( spinal cord infarct at T4. Autonomic dysreflexia is a medical emergency caused by over stimulation of the sympathetic nervous system in individuals with spinal cord injuries at or above the 5th thoracic (T5) spinal cord levels, although patients with injuries between T6-10 maybe susceptible. Autonomic dysreflexia is classically stimulated by: An overfilled bladder, colonic distension (constipation), decubitus ulcer or silent orthopedic fracture. Patients exhibiting autonomic dysreflexia are symptomatic with complaints of a headache, flushing and diaphoresis (above the level of the spinal cord lesion), hypertension and bradycardia. Bradycardia occurs due to a reflex stimulated from stretch on the atrial ventricular node by the elevation in blood pressure. When seen in the office setting, the first step should be to empty the bladder and remove all noxious stimuli, i.e., cystoscope, urodynamic catheter, from the bladder. If the elevation in blood pressure does not respond, the patient should be treated with ½ to 1 inch of nitropaste to the chest wall. If rebound hypotension occurs the nitropaste maybe rapidly wiped off of the skin. Other options of pharmacologic therapy for autonomic dysreflexia in the office setting include oral or sublingual nifedipine. However, rebound hypotension can be problematic and difficult to deal with. Wein AJ, Dmochowski RR: Neuromuscular dysfunction of the lower urinary tract, Wein AJ, Kavoussi LR, Novick AC, Partin AW, Peters CA (eds): CAMPBELL-WALSH UROLOGY, ed 10. Philadelphia, Elsevier Saunders, 2012, vol 3, chap 65, p 1926. 2013 Adult Neurogenic Bladder, Voiding Dysfunction, Incontinence )

While performing a videourodynamic study in a patient with neurogenic bladder, the patient develops a severe headache. The neurological condition most likely to be associated with this event is: A) hemorrhagic stroke in globus pallidus. B) conus medullaris injury. C) spinal cord infarct at T4. D) astrocytoma displacing the reticulospinal tract at T8. E) multiple sclerosis.

100. A 75-year-old man with a prior history of a left radical nephrectomy develops intractable hypertension and has a right renal artery ostial stenosis of > 75%. His volume status and angiotensin II levels are best characterized respectively as: A.. euvolemic normal. B.. euvolemic elevated. C. hypervolemic suppressed. D.. hypervolemic normal. E.. hypervolemic elevated.

question #100 ANSWER=D This case exemplifies the classic one-kidney one-clip animal model of renovascular hypertension and is similar to the findings of the two-kidney two clip animal model of renovascular hypertension. In these models during the acute phase of obstruction there is an increase in renin release and activation of the renin-angiotensin-aldosterone system (RAAS) by the ischemic kidney(s) resulting in hypertension. With the absence of a contralateral kidney or if both kidneys are involved contralateral natriuresis by the unaffected kidney will not occur. Consequentially the stenotic kidney begins to conserve sodium and fluid resulting in volume expansion and an elevated renin (transitional phase). In the chronic phase the elevated blood pressure excess Copyright 2017 by The American Urological Association. 35 sodium retention and volume expansion all act as negative feedback mechanisms for suppression of renin release resulting in volume-expanded hypertension with normal reninangiotensin- 11 levels. These patients do not respond well to ACE inhibitors or angiotensin-11 antagonists unless concurrent sodium restriction is prescribed. In contrast the two-kidney oneclip model is characterized by unilateral release of renin from the ischemic kidney accompanied by contralateral suppression of renin from the normal kidney and natriuresis. Consequently there is sodium retention from the ischemic kidney and excretion from the contralateral kidney. This results in euvolemia and hypertension dependent upon angiotensin-11 vasoconstriction. Medical management of these patients is directed at the renin-angiotensin system (i.e. ACE inhibitors and angiotensin-11 antagonists) with or without sodium restriction based on the type of renal hypertension model. Gui mi FA Reiser IW Spitalewitz S: Renovascular hypertension and ischemic nephropathy in Wein AJ Kavoussi LR Partin AW Peters CA (eds): CAMPBELL-WALSH UROLOGY ed 11. Philadelphia Elsevier 2015 vol 2 chap 45 p 1029.

101. An 11-month-old boy has a right testicular mass. AFP is 50 IU/ml and hCG is normal. Abdominal-pelvic CT scan is normal. During inguinal exploration frozen section biopsy reveals teratoma. The next step is: A.. partial orchiectomy. B.. partial orchiectomy and serial scrotal ultrasounds. C. orchiectomy. D.. orchiectomy and adjuvant chemotherapy. E.. orchiectomy and RPLND.

question #101 ANSWER=A Pre-pubertal (in distinction to post-pubertal) mature teratoma of the testis have a benign clinical course. AFP levels are initially elevated in newborns and decline during the first year of life therefore in this patient the AFP elevation does not indicate yolk sac elements. Treatment is partial orchiectomy after confirmation of the frozen section. Orchiectomy and further treatment such as chemotherapy radiation or RPLND are reserved for malignant tumors such as yolk sac. Serial ultrasounds are not necessary in pre-pubertal patients with teratoma. F errer FA: Pediatric urologic oncology: Bladder and testis in Wein AJ Kavoussi LR Partin AW Peters CA (eds): CAMPBELL-WALSH UROLOGY ed 11. Philadelphia Elsevier 2015 vol 4 chap 156 pp 3593-3596.

102. A four-year-old girl is febrile with left upper pole hydroureteronephrosis and a debris-filled ectopic ureterocele. In addition to broad-spectrum I.V. antibiotics the next step is: A.. percutaneous nephrostomy. B.. cystoscopy and ureteral stent. C. cystoscopy and ureterocele incision. D.. open excision of ureterocele. E.. open excision of ureterocele and ipsilateral ureteral reimplant. Copyright 2017 by The American Urological Association. 35

question #102 ANSWER=C In a febrile but clinically stable patient endoscopic incision has the advantage of both draining the system and the possibility that it could be a definitive treatment. The best management is cystoscopic incision of the ureterocele in order to promptly and fully drain the infected urine from the ureterocele. Endoscopic incision of the ureterocele with subsequent decompression of the upper tract would obviate the need for stent placement. Percutaneous nephrostomy placement can be of benefit in a clinically labile patient where general anesthesia for endoscopic incision of the ureterocele would be hazardous. Definitive management may require an open approach including ureterocele excision and ureteral reimplantation. However that would not be recommended at present in this acute setting. Peters CA Mendelsohn C: Ectopic ureter ureterocele and ureteral anomalies in Wein AJ Kavoussi LR Partin AW Peters CA (eds): CAMPBELL-WALSH UROLOGY ed 11. Philadelphia Elsevier 2015 vol 4 chap 134 pp 3080-3086. 36 Copyright 2017 by The American Urological Association.

103. An 80-year-old woman has vague abdominal pain and no urine output for 12 hours. She underwent cystectomy and ileal conduit urinary diversion for urothelial carcinoma three years ago. Serum creatinine is normal and CT loopogram images are shown. The diagnosis is: A.. B .. C. D .. E .. stomal stenosis. intestinal stricture due to lymphoid depletion. parastomal hernia. stomal prolapse. intestinal volvulus. Copyright 2017 by The American Urological Association.

question #103 ANSWER=C She has small bowel herniation which is compressing her ileal loop resulting in outlet obstruction at the fascial level. Stomal stenosis can present similarly but without CT scan findings of a hernia. Lymphoid depletion does result in late-onset loop strictures but these are generally mid-loop. Stomal prolapse does not result in obstruction. Intestinal volvulus could result in obstruction of the loop but again the principal finding on the CT scan is the hernia. Dahl DM: Use of intestinal segments in urinary diversion Wein AJ Kavoussi LR Partin AW Peters CA (eds): CAMPBELL-WALSH UROLOGY ed 11. Philadelphia Elsevier Saunders 2015 vol 3 chap 97 pp 2290-2292.

104. A 45-year-old man arr putates his penis at the level of the proximal shaft. In addition to the repair of the urethra the following structures should be a nastomosed: A.. dorsal arteries deep dorsal ' 'orsal nerves. B.. cavernosal arteries and dorsal nerves. C cavernosal arteries dorsal arteries deep dorsal vein and dorsal nerves. D.. cavernosal arteries dorsal arteries and deep dorsal vein. E.. dorsal arteries and dorsal nerves.

question #104 ANSWER=A During microsurgical reconstruction of the amputated penis the urethra is reapproximated the two dorsal arteries the deep dorsal vein and as many nerve fascicles as possible are anastomosed. The cavernosal arteries are typically not reconstructed as they are difficult to access and the dorsal arteries provide adequate circulation. Jordan GH: Lower genitourinary tract trauma and male external genital trauma (nonpenetrating injuries penetrating injuries and avulsion injuries) Part II. AUA UPDATE SERIES 2000 vol 19 lesson 11. Morey AF Zhao LC: Genital and lower urinary tract trauma in Wein AJ Kavoussi LR Partin AW Peters CA (eds): CAMPBELL-WALSH UROLOGY ed 11. Philadelphia Elsevier 2015 vol 3 chap 101 pp 2381-2382.

105. A 68-year-old man undergoes a partial penectomy for a high-grade pT2 squamous cell carcinoma of the distal urethra. The margins are negative and the metastatic work-up is negative. The next step is: A.. observation. B.. adjuvant chemotherapy. C. bilateral superficial inguinal lymphadenectomy. D.. bilateral pelvic and inguinal lymphadenectomy. E.. total penectomy.

question #105 ANSWER=A This patient has a T2 squamous cell carcinoma of the distal urethra with negative margins. Assuming that his cystoscopy is normal and there are no other signs of disease the proper management is observation. Total penectomy is not advocated in the setting of negative margins. Currently there is no data that prophylactic inguinal node dissection provides any benefit. Adjuvant chemotherapy is not indicated for T2 disease. Radical penectomy with cystoprostatectomy is also not indicated with the finding of negative margins and no evidence of disease in the bladder. Sharp DS Angermeier KW: Tumors of the urethra in Wein AJ Kavoussi LR Partin AW Peters CA (eds): CAMPBELL-WALSH UROLOGY ed 11. Philadelphia Elsevier 2015 vol 1 chap 38 pp 881- 883.

106. A 76-year-old woman has hypertension type 2 diabetes and mild chronic renal insufficiency associated with proteinuria. The best reno-protective strategy includes initiation of: A.. amlodipine. B.. atenolol. C. clonidine. D.. lisinopril. E.. hydrochlorothiazide.

question #106 ANSWER=D Blood pressure control has been identified as one of several measures to help prevent progression of chronic kidney disease (CKD). Others include: lifestyle modification glycemic control reduction of proteinuria protein restriction lipid control correction of anemia Copyright 2017 by The American Urological Association. 37 correction of acidosis and maintenance of fluid balance. Angiotensin II is thought to be central to the progression of CKD via both hemodynamic and non-hemodynamic mechanisms. ACE inhibitors can reduce glomerular pressure as well as proteinuria which has a sentinel role in renal scarring. In addition ACE inhibitors appear to improve interstitial capillary pO2 levels thus decreasing renal sclerosis risk. All other therapies including calcium channel blockers (e.g. amlodipine) beta-blockers (e.g. atenolol) alpha-agonists (e.g. clonidine) and diuretics (e.g. hydrochlorothiazide) do not provide as much reno-protection as ACE inhibitors or angiotensin receptor blockers (e.g. losartan). Goldfarb DA Poggio ED Demirjian S: Etiology pathogenesis and management of renal failure in Wein AJ Kavoussi LR Partin AW Peters CA (eds): CAMPBELL-WALSH UROLOGY ed 11. Philadelphia Elsevier 2015 vol 2 chap 46 pp 1062-1063.

107. Clostridium difficile infections with severe dehydration and electrolyte imbalance without abdominal distension or ileus are best treated with: A.. oral metronidazole. B.. oral vancomycin. C. oral vancomycin and I.V. metronidazole. D.. I.V. metronidazole and rectal vancomycin. E.. I.V. vancomycin and rectal metronidazole.

question #107 ANSWER=C According to the American Society of Gastroenterology diarrhea with a positive test for clostridium difficile with significant volume loss and electrolyte abnormalities is defined as severe and/or complicated infection. I.V. fluid resuscitation electrolyte replacement and pharmacological DVT prophylaxis is recommended. In the absence of ileus or significant abdominal distention oral or enteral feeding should be given. CT scan of the abdomen and pelvis is recommended. Vancomycin orally or via the enteral tube at 125 mg PO four times daily plus metronidazole 500 mg I.V. three times daily is strongly recommended in the guidelines. Oral metronidazole is recommended for mild to moderate infection and considered under treatment for severe cases. Single agent treatment with vancomycin alone is also considered undertreatment. Rectal vancomycin and/or metronidazole is reserved for severe infection with ileus abdominal distention or toxic colon. Surawicz CM Brandt LJ Binion DG et al: Guidelines for diagnosis treatment and prevention of Clostridium difficile infections. AM GASTROENTEROL 2013;108:478-498.

108. A patient complains of stress urinary incontinence one year following radical prostatectomy. Physical evaluation confirms stress incontinence. However videourodynamic studies fail to document stress incontinence with Valsalva maneuvers at 150 ml 250 ml and end-fill capacity of 500 ml with the patient reaching Valsalva intra-abdominal pressures of > 60 cm H2O. The next step in the videourodynamic test is: 38 A.. B .. C. D .. E .. instill more volume. remove the urethral catheter and repeat the Valsalva maneuver. perform straining tapping and Crede maneuvers to attempt to elicit involuntary bladder contractions. increase Valsalva efforts to attempt to obtain pressures of > 100 cm H2O pressure. have patient void and check postvoid residual urine. Copyright 2017 by The American Urological Association.

question #108 ANSWER=B The determination of stress urinary incontinence (SUI) secondary to sphincter deficiency should be performed at bladder volumes of 200 ml or greater and require that the patient reaches Va Isa Iva intra-abdominal pressures of greater than or equal to 60 cm H2O. Failure to reach these intra-abdominal pressures or bladder volumes during the performance of a videourodynamic study are two of the main reasons for a false negative test for sphincteric incontinence. In the presence of a urodynamic study reaching adequate bladder volumes and adequate Valsalva pressures the next most common cause for a videourodynamic to fail to diagnose sphincteric incontinence is the presence of an indwelling catheter during the time of urodynamic study. An indwelling catheter may prevent demonstration of SUI at the time of testing by causing a physical obstruction of a coexisting bladder neck contraction or alternatively by causing spasm of the urethral sphincter related to catheterization. In these situations the urethral catheter should be removed while at maximum bladder capacity and a Valsalva maneuver performed. Leakage can be confirmed by the concurrent use of video portion of the study. If no leakage is found the patient should be requested to void and confirmation of complete bladder emptying noted to rule-out the possibility of coexisting impaired detrusor contractility and overflow incontinence. Alternatively a simple postvoid ultrasound residual could have been performed as part of the initial evaluation in a patient with post-prostatectomy urinary incontinence to rule- out this possibility. The use of straining tapping and Crede maneuvers are classically used in a patient with an underactive bladder in an attempt to elicit a detrusor contraction when the patient is unable to void on request. Valsalva intraabdominal pressures of 100 cm or higher are required to evaluate for hypermobility of the bladder neck in women complaining of stress urinary incontinence. Dector RM Harpster L: Pitfalls in leak point pressure determination. J UROL 1992;149:588-591. Maniam P Goldman HB: Removal of transurethral catheter during urodynamics may unmask stress urinary incontinence. J UROL 2002;167:2080-2082. Nitti VW Brucker BM: Urodynamic and video-urodynamic evaluation of the lower urinary tract in Wein AJ Kavoussi LR Partin AW Peters CA (eds): CAMPBELL-WALSH UROLOGY ed 11. Philadelphia Elsevier 2015 vol 3 chap 73 p 1727.

110. A 16-year-old girl with spina bifida and a ventriculoperitoneal shunt has abdominal pain fever nausea and vomiting. She has a prior augmentation cystoplasty Mitrofanoff channel and bladder neck ligation. A conventional cystogram is normal. A catheter is placed and I.V. antibiotics are initiated. The next step is: A.. observation. B.. abdominal ultrasound. C. CT cystogram. D.. endoscopy of the augmented bladder. E.. shunt series.

question #110 ANSWER=C Spontaneous bladder perforation following enteric bladder augmentation occurs in 8 to 10% of cases most commonly presenting with signs of peritonitis and acute abdomen. It can be lifethreatening especially in the neuropathic population as often their neurologic deficit causes atypical presenting symptoms and sepsis can occur rapidly. Late perforations most often occur in the bowel segment approximately 1 cm from the anastomotic line. A standard cystogram has a high false negative rate and a CT cystogram is recommended in patients with abdominal symptoms suggestive of bladder rupture. While most perforations are managed with laparotomy Copyright 2017 by The American Urological Association. 39 and primary closure conservative treatment with catheter drainage has been reported successful in very selected cases usually with contained late perforations in patients without VP shunts. Endoscopy is not diagnostic for perforation and can lead to worsening dissemination of infected urine in the abdomen. Shunt externalization is often required due to peritonitis and contamination of the abdominal shunt catheter but shunt series is not necessary. Adams MC Joseph DB Thomas JC: Urinary tract reconstruction in children in Wein AJ Kavoussi LR Partin AW Peters CA (eds): CAMPBELL-WALSH UROLOGY ed 11. Philadelphia Elsevier 2015 vol 4 chap 145 p 3354.

111. A 65-year-old woman has bothersome frequency urgency and nocturia one month after autologous fascial sling. Her postvoid residual is 300 ml and urinalysis is negative. The next step is: A.. alpha-blocker. B.. urethral dilation. C. CIC. D.. sling loosening. E.. sling incision.

question #111 ANSWER=( These symptoms are likely related to the sling being too tight leading to her de novo symptoms. The best option at this time would be to initiate CIC to allow for adequate emptying and minimization of symptoms. Alpha-blockers would not impact this anatomic obstruction. Urethral dilation is likely to be ineffective. Fascial slings tend to loosen up over time and with the use of CIC her residuals can be continually monitored for possible improvement. Sling loosening would not likely be effective. Sling incision should be considered if these symptoms do not improve with time. The optimal time for sling incision post-sling placement is not known though many authors advocate waiting three to four months after the sling has been placed. This is certainly different than how obstruction is managed after mid-urethral sling with many considering sling incision within the first month after sling placement. Dmochowski RR Osborn DJ Reynolds WS: Slings: Autologous biologic synthetic and midurethral in Wein AJ Kavoussi LR Partin AW Peters CA (eds): CAMPBELL-WALSH UROLOGY ed 11. Philadelphia Elsevier 2015 vol 3 chap 84 p 2005.

112. An icteric one-week-old boy has a right upper quadrant mass and scrotal ecchymosis. Ultrasound shows a complex right suprarenal mass with internal echoes. Labs demonstrate anemia and normal plasma catecholamine levels. The next step is: A.. repeat ultrasound in two weeks. B.. needle biopsy of the mass. C. ultrasound-guided percutaneous drainage. D.. bone marrow aspiration. E.. adrenalectomy.

question #112 ANSWER=A This patient has had an adrenal hemorrhage and displays the classic triad of mass anemia and jaundice (secondary to blood absorption from the retroperitoneum). Males can present with scrotal hemorrhage. Ultrasound is the best way to confirm the diagnosis and after excluding neuroblastoma is an ideal follow-up tool. CT scanning often requires heavy sedation or anesthesia in the very young child. Biopsy and surgery are not required. Spontaneous complete resolution will usually occur within months. Lee RS Borer JG: Perinatal urology in Wein AJ Kavoussi LR Partin AW Peters CA (eds): CAMPBELL-WALSH UROLOGY ed 11. Philadelphia Elsevier 2015 vol 4 chap 124 p 2886.

113. A 33-year-old man with erectile dysfunction has a testosterone of 179 ng/dl. He takes chronic opioids for back pain. His wife is pregnant with their first child and they would like to have more children. The next step is: A.. wean opioids. B.. testosterone gel. C. testosterone injection. D.. urethral alprostadil suppositories. E.. sex therapy.

question #113 ANSWER=A Roughly 74% of men who take chronic opioids will have low testosterone. Opioids have been shown to have an inhibitory effect on gonadotropin-releasing hormone secretion in the hypothalamus thus to decrease gonadotropin production and testosterone production. Weaning opioids may result in an increase in serum testosterone levels. Because this couple wants to have children in the future exogenous testosterone with gels and injections should not be used as this can impair semen parameters. The likely cause of this patient's erectile 40 Copyright 2017 by The American Urological Association. dysfunction (ED) is his low testosterone thus sex therapy will probably not solve his ED problem. Urethral alprostadil suppositories should not be used during unprotected intercourse with a pregnant partner as this may induce uterine contractions and should not be used in couples trying to achieve pregnancy. Burnett AL II: Evaluation and management of erectile dysfunction in Wein AJ Kavoussi LR Partin AW Peters CA (eds): CAMPBELL-WALSH UROLOGY ed 11. Philadelphia Elsevier 2015 vol 1 chap 27 pp 659-660. Daniell HW Lentz R Mazer NA: Open-label pilot study of testosterone patch therapy in men with opioid-induced androgen deficiency. J PAIN 2006;7:200. Kalyani RR Gavini S Dobs AS: Male hypogonadism in systemic disease. ENDOCRINOL METAB CUN N AM 2007;36:333.

115. A 28-year-old healthy woman has dysuria and frequency. Examination shows suprapubic tenderness and urinalysis reveals pyuria. Her pregnancy test is negative. The next step is: A.. urine culture. B.. single dose of trimethoprim/sulfamethoxazole. C. single dose of ciprofloxacin. D.. three day course of trimethoprim/sulfamethoxazole. E.. three day course of ciprofloxacin.

question #115 ANSWER=D This woman has an uncomplicated UTI. Urine culture is not necessary when symptoms are present she has no risk factors for a complicated UTI and urinalysis reveals pyuria and/or bacteriuria. Trimethoprim/sulfamethoxazole is the best empirical agent for uncomplicated UTI in women. Quinolones should be reserved for cases of recurrent UTI or a history of trimethoprim/sulfamethoxazole resistance. Three day therapy is the preferred treatment regimen for uncomplicated cystitis in women. Single dose therapy has a high rate of recurrence. Schaeffer AJ Matulewicz RS Klumpp DJ: Infections of the urinary tract in Wein AJ Kavoussi LR Partin AW Peters CA (eds): CAMPBELL-WALSH UROLOGY ed 11. Philadelphia Elsevier 2015 vol 1 chap 12 pp 266-268.

116. After two weeks of behavioral therapy and oxybutynin a 64-year-old woman continues to report frequency and urge incontinence. She has minimal side effects from the medication and her urinalysis is normal. The next step is: A.. continue present therapy. B.. stop oxybutynin start solifenacin. C. add mirabegron. D.. cystoscopy. E.. intravesical onabotulinumtoxinA injections.

question #116 ANSWER=A According to the AUA Guidelines on overactive bladder (OAB) a therapeutic trial of therapy is a minimum of four to eight weeks for pharmacologic therapy and eight to twelve weeks for behavioral therapy. This patient has only been treated for two weeks and further improvement Copyright 2017 by The American Urological Association. 41 may be seen with her present management plan. A change in oral therapy (solifenacin or mirabegron) should be considered only after she has failed an adequate trial length of her present therapy. Failure is defined as lack of improvement after four to eight weeks of pharmacologic therapy and/or the development of intolerable pharmacologic side effects. OnabotulinumtoxinA is indicated for patients that have failed oral pharmacologic therapy. In the routine evaluation of the OAB patient cystoscopy is not indicated unless the screening urinalysis is abnormal. Gormley EA Lightner DJ Burgio KL et al: DIAGNOSIS AND TREATMENT OF OVERACTIVE BLADDER (NON-NEUROGENIC) IN ADULTS: AUA/SUFU GUIDELINE. American Urological Association Education and Research Inc 2012IAmended 2014. http://www.auanet.org/education/guidelines/overactive-bladder.cfm

117. The best initial therapy for post-prostate biopsy sepsis is: A.. ciprofloxacin. B.. gentamicin. C. imipenem. D.. piperacillin/tazobactam. E.. trimethoprim/sulfamethoxazole.

question #117 ANSWER=C Recent studies on post-prostate biopsy sepsis have noted significant resistance to fluoroquinolones (90%) piperacillin (72%) trimethoprim/sulfamethoxazole (44%) and even gentamicin (22%). However minimal to no bacterial resistance is noted to the carbapenems (imipenem and meropenem) and amikacin. In addition the oral second (Cefuroxime Cefaclor) and third-generation cephalosporins (Cefixime Cefdinir) maintain a good bacterial sensitivity pattern. Gonzalez C M Averch T Boyd LA et al: AUA/SUNA WHI TE PAPER ON THE INCIDENCE PREVENTION AND TREATMENT OF COMPLICATIONS RELATED TO PROSTATE NEEDLE BIOPSY. American Urological Association Education and Research Inc 2012. http://www.aua net. org/com mon/pdf led ucation/cl in ica I-g u idance/ AUA-SU NA-PN B-Wh ite- Pa per. pdf

118. A six-year-old girl has low volume urinary incontinence and two documented afebrile UTls. Timed voiding and treatment of constipation have only helped modestly with incontinence and she had a third afebrile UTI. Uroflowmetry with EMG shows a voided volume of 180 ml with a low plateau-shaped curve and increased pelvic floor activity at the time of voiding. PVR is 50 ml. The next step is: 40 A.. B .. C. D .. E .. biofeedback. antimuscarinics. VCUG. CIC. onabotulinumtoxinA injection in the external sphincter. Copyright 2017 by The American Urological Association.

question #118 ANSWER=A The low plateau-shaped curve and elevated PVR are indicative of bladder outlet obstruction and the increased pelvic floor activity suggests uncoordinated detrusor-pelvic floor function during voiding. The absence of other neurologic signs or symptoms high volume incontinence or high PVR would make neurogenic bladder less likely. VCUG is not necessary in the absence of febrile U Tls to rule-out reflux although imaging of the outlet can show sphincteric dysfunction during voiding. Antimuscarinics are contraindicated in the setting of elevated PVR without CIC. The PVR volume is not of a sufficient volume to warrant CIC and CIC is unnecessarily invasive. Biofeedback has been shown to normalize micturition pattern to improve emptying incontinence and frequency of U TI. OnabotulinumtoxinA injection in the external sphincter can be used in refractory cases but is not approved for use in the sphincter or in children. Alphablocking agents have been shown to be effective but do not offer the potential of a long-term cure. Austin PF Vricella GJ: Functional disorders of the lower urinary tract in children in Wein AJ Kavoussi LR Partin AW Peters CA (eds): CAMPBELL-WALSH UROLOGY ed 11. Philadelphia Elsevier 2015 vol 4 chap 143 p 3307. 42 Copyright 2017 by The American Urological Association.

119. As compared to conventional laparoscopic radical nephrectomy hand-assisted nephrectomy is associated with: A.. prolonged ileus. B.. prolonged hospitalization. C. increased wound complications. D.. increased narcotic requirements. E.. increased port site metastases.

question #119 ANSWER=C Hand-assisted laparoscopic radical nephrectomy is comparable to conventional laparoscopic techniques by all measures of peri-operative and oncologic outcomes except for higher wound complications such as hernias and infections at the hand-port site. The published incidence of these complications with hand-assisted nephrectomy is approximately 4-9%. Schwartz MJ Rais-Bahrami S Kavoussi LR: Laparoscopic and robotic surgery of the kidney in Wein AJ Kavoussi LR Partin AW Peters CA (eds): CAMPBELL-WALSH UROLOGY ed 11. Philadelphia Elsevier 2015 vol 2 chap 61 p 1468. Ordon M Eichel L Landman J: Fundamentals of laparoscopic and robotic urologic surgery in Wein AJ Kavoussi LR Partin AW Peters CA (eds): CAMPBELL-WALSH UROLOGY ed 11. Philadelphia Elsevier 2015 vol 1 chap 10 p 223.

12. A 67-year-old man has bothersome LUTS six months after hip surgery despite tamsulosin treatment. His urinalysis shows 0-2 RBC/hpf and his PVR is 90 ml. DRE demonstrates 35 gm prostate. He undergoes urodynamics as shown. The next step is: A.. creatinine. B.. antimuscarinics. C scopy. D.. prostatic onabotulinumtoxinA. E.. TURP.

question #12 ANSWER=E This patient has urodynamically demonstrable bladder outlet obstruction. While antimuscarinics may help his irritative symptoms (e.g. frequency urgency urge incontinence) it will not address his primary obstructive problem and indeed may worsen his symptoms. Obtaining a serum creatinine is not useful or recommended in the BPH guidelines for work-up of LUTS. Cystoscopy may help assess prostatic size but would be unlikely to change management as his DRE shows a 35 gm prostate on exam. OnabotulinumtoxinA injections are not approved for nor do they have documented efficacy for the treatment of LUTS related to bladder outlet obstruction. In this patient TURP is the next step for the treatment of bladder outlet obstruction. Welliver C McVary KT: Minimally invasive and endoscopic management of benign prostatic hyperplasia in Wein AJ Kavoussi LR Partin AW Peters CA (eds): CAMPBELL-WALSH UROLOGY ed 11. Philadelphia Elsevier 2015 vol 3 chap 105 pp 2506-2509.

121. A recurrent calcium oxalate stone former has a urine calcium excretion of 180 mg/day (normal < 200 mg/day) and a uric acid excretion of 950 mg/day (normal < 800 mg/day). The next step is: A.. allopurinol. B.. chlorthalidone. C. hydrochlorothiazide. D.. potassium citrate. E.. triamterene.

question #121 ANSWER=A Clinicians should offer allopurinol to patients with recurrent calcium oxalate stones who have hyperuricosuria and normal urinary calcium. A prospective randomized controlled trial demonstrated that allopurinol reduced the risk of recurrent calcium oxalate stones in the setting of hyperuricosuria (urinary uric acid excretion> 800 mg/day) and normocalciuria. Whether the drug is effective in patients with hypercalciuria has not been established. Hyperuricemia is not a required criterion for allopurinol therapy. In addition to medication limiting non-dairy animal protein may maximize the efficacy of allopurinol. Chlorthalidone and hydrochlorothiazide would be indicated in the setting of hypercalciuria. Potassium citrate is indicated for hypocitraturia which is not present either. Triamterene although it is a potassium-sparing diuretic should be avoided as stones of this compound have been reported. Pearle MS Goldfarb DS Assimos DG et al: MEDICAL MANAGEMENT OF KIDNEY STONES: AUA GUIDELINE: AUA GUIDELINE. American Urological Association Education and Research Inc 2014. http://www.auanet.org/education/guidelines/management-kidney-stones.cfm

122. A 65-year-old man with metastatic clear cell RCC demonstrates progression after initial treatment with sunitinib therapy. His performance status is good. Level 1 evidence supports the use of: A.. bevacizumab. B.. sorafenib. C. pazopanib. D.. temsirolimus. E.. everolimus.

question #122 ANSWER=E Everolimus (RAD001) is an orally administered inhibitor of mTOR. In the RECORD 1 trial which compared everolimus to placebo in a phase 3 prospective randomized trial of patients who received previous targeted therapy progression-free survival was improved in the everolimustreated patients (4.0 vs. 1.9 months). As such treatment with everolimus is a category 1 recommendation after tyrosine kinase inhibitor therapy according to the NCCN Kidney Cancer panel. Axitinib is also a category 1 recommendation for this group of patients. A variety of other studies have evaluated the use of temsirolimus bevacizumab and sorafenib in the setting of failure after initial therapy and each of these agents have shown some effect. However given the limited outcomes these agents are given a category 2A recommendation in the recurrence. The NCCN guideline panel considers pazopanib a category 3 recommendation because no data exists for this drug in this setting. Motzer RJ Jonasch E Agarwal N et al: NCCN clinical practice guidelines in oncology: Kidney cancer (version 1.2015) http://www.nccn.org/professionals/physician_gls/pdf/kidney.pdf (accessed 10/21/2014) Srinivasan R Linehan WM: Treatment of advanced renal cell carcinoma in Wein AJ Kavoussi LR Partin AW Peters CA (eds): CAMPBELL-WALSH UROLOGY ed 11. Philadelphia Elsevier 2015 vol 2 chap 63 pp 1515-1516.

123. Fifty boys were equally randomized to urethral stent or no urethral stent following hypospadias repair. Ten boys in the stented group and four boys in the non-stented group reported bladder spasm pain. The best test to determine if there was a significant difference regarding bladder spasm relative to stent status is: A.. B .. C. D .. E .. Fisher's exact test. Chi-square test. analysis of variance (ANOVA). logistic regression. Pearson correlation coefficient.

question #123 ANSWER=A There are several appropriate analyses in the setting of a trial with a binary variable. Fisher's exact test should be used when the expected number of subjects in any subgroup is below five. Chi-square test is also appropriate when the dependent variable is measured as a binary variable and any of the expected sample size of subjects per subgroup is five or greater. ANOVA is used to compare more than two groups. Logistic regression would be the appropriate test if there was an additional independent variable. The Pearson correlation coefficient is the appropriate test when assessing the relationship between two variables. 44 Copyright 2017 by The American Urological Association. AUAUniversity Core Curriculum: Statistics. American Urological Association website. https://www.auanet.org/university/core_topic.cfm?core1D=122 Updated May 14 2014.

124. A 66-year-old man with advanced prostate cancer is starting abiraterone therapy. The addition of prednisone is necessary in order to: A.. decrease inflammatory response and pain. B.. reduce nausea and anorexia. C. minimize side effects of binding to CYP17. D.. inhibit the nuclear translocation of the androgen receptor. E.. decrease microtubule assembly.

question #124 ANSWER=C Abiraterone acetate is a specific potent and irreversible inhibitor of CYP17. CYP17 both facilitates the conversion of cholesterol to androstenedione and dehydroepiandrosterone (DHEA) made in the adrenal glands and also inhibits the conversion of secondary androgens into testosterone and dihydrotestosterone. CYP17 activity can be inhibited by both ketoconazole and abiraterone acetate; however ketoconazole competitively inhibits CYP 17. Its pharmacological effectiveness is therefore a reflection of the concentration of ketoconazole within the cell and are temporizing in nature. By contrast once abiraterone acetate binds to a molecule of CYP17 the CYP17 molecule is permanently disabled. Because abiraterone effectively shuts down the effects of CYP17 it inhibits the synthesis of androgens by both the testes and the adrenal. The completeness of this blockade will lead to loss of inhibitory feedback on the pituitary gland resulting in excess adrenal stimulation and a build-up in steroid precursors that will be diverted into alternative pathways that will eventually result in an increased production of aldosterone. The increase in mineralocorticoids leads to fluid retention and hypokalemia. To prevent this complication low dose prednisone is administered to enhance a negative feedback on the pituitary system. Although steroids can help reduce inflammation bone pain nausea and anorexia in any cancer patient steroids are not used for this indication during treatment with abiraterone. Enzalutamide not abiraterone is involved in the process of nuclear translocation of the androgen receptor as part of prostate cancer anti-tumor activity. Abiraterone is not involved in cellular microtubule assembly; this is the anti-tumor mechanism of docetaxel. Antonarakis ES Carducci MA Eisenberger MA: Treatment of castration-resistant prostate cancer in Wein AJ Kavoussi LR Partin AW Peters CA (eds): CAMPBELL-WALSH UROLOGY ed 11. Philadelphia Elsevier 2015 vol 3 chap 121 pp 2811-2812.

125. The human papillomavirus (HPV) vaccine: A.. is effective if the individual has been previously exposed to HPV. B.. should be administered before the onset of sexual activity. C. is FDA-approved for females only. D.. may cause a mild transient HPV-like outbreak. E.. is effective against all known HPV subtypes.

question #125 ANSWER=B Human papillomavirus (HPV ) is the most common sexually transmitted infection in the United States and worldwide. The highest prevalence of genital HPV is found in sexually active adolescents and young adults. HPV is associated with cervical and other cancers including: penile anal vulvar vaginal and oropharyngeal. The FDA-approved HPV vaccines have been shown to be very safe. This quadrivalent vaccine (Gardasil) contains no viral DNA and is bioengineered to contain virus-like particles produced from the major capsid protein of HPV types 16 18 6 and 11. There is no data to suggest that there are vaccine-specific adverse effects with the exception of rare anaphylaxis to the vaccine components. The HPV vaccines appear to be very effective but are beneficial only if an individual has not previously been exposed to HPV. Therefore it is recommended that the vaccine is administered before the onset of sexual activity. The duration of vaccine-induced antibodies is known to be at least five years in males and nine years in females. Clinical studies demonstrate a high degree of effectiveness in preventing genital lesions and intraepithelial neoplasia associated with HPV genotypes 6 11 16 and 18. The American Academy of Pediatrics recommends routine vaccination of both boys and girls who are 11-12 years of age. Copyright 2017 by The American Urological Association. 45 American Academy of Pediatrics Policy Statement HPV Vaccine Recommendations www.pediatrics.org/cgi/doi/10.1542/peds.2011-3865 Doi: 10.1542/peds.2011-3865 Pediatrics 2012 The American Congress of Obstetricians and Gynecologists Committee on Adolescent Health Care Opinion March 2014. http://www.acog.org/Resources-And-Publications/Committee-Opinions/Committee-on- Adolescent-Hea lth-Care/H u man-Papi I lomavi rus-Vacci nation

139. Two years after a low anterior resection with pelvic X-ray and chemotherapy for advanced colon cancer a 69-year-old man develops fecaluria and pneumaturia. Cystoscopy shows an irregular area in the posterior bladder wall. The next step is: A.. fulguration. B.. biopsy. C. partial cystectomy. D.. colon resection and bladder repair with omental interposition. E.. pelvic exenteration.

question #139 ANSWER=B This patient is at risk for a local recurrence and this may present as a fistula especially with a history of XRT. Biopsy is indicated rather than fulguration. He eventually may need resection and appropriate therapy but radical surgery is not indicated until a diagnosis is confirmed. Badlani GH De Ridder DJMK Mettu JR Rovner ES: Urinary tract fistulae in Wein AJ Kavoussi LR Partin AW Peters CA (eds): CAMPBELL-WALSH UROLOGY ed 11. Philadelphia Elsevier 2015 vol 3 chap 89 p 2103. 50 Copyright 2017 by The American Urological Association.

126. A 55-year-old man has lower extremity thrombophlebitis and is started on warfarin. Two weeks later he experiences abdominal pain and has a blood pressure of 84/50 mmHg. His hemoglobin is 13.5 gm/dl and serum potassium is 5.8 mEq/L. A CT scan demonstrates bilateral adrenal hemorrhage. The next step is I.V. fluids and administration of: A.. dexamethasone. B.. fresh frozen plasma. C. Kayexalate®. D.. fluorohydrocortisone. E.. Vitamin K.

question #126 ANSWER=A This patient has adrenal insufficiency secondary to bilateral adrenal hemorrhage. This can occur in anticoagulated patients typically during the first three weeks of therapy. The initial therapy should be administration of I.V. fluids and glucocorticoid therapy. Fresh frozen plasma is not acutely indicated with an adequate hemoglobin level. Kayexalate will help lower a high potassium but not improve the hypotension from adrenal steroid deficiency. Chronic but not acute adrenal insufficiency is treated with fluorohydrocortisone. Vitamin K will help restore clotting factors depleted by warfarin therapy but is not the initial therapy for this patient. Kutikov A Crispen PL Uzzo RG: Pathophysiology evaluation and medical management of adrenal disorders in Wein AJ Kavoussi LR Partin AW Peters CA (eds): CAMPBELL-WALSH UROLOGY ed 11. Philadelphia Elsevier 2015 vol 2 chap 65 pp 1553-1555.

127. A 28-year-old man with NSGCT has a serum AFP of 2 500 IU/ml small volume retroperitoneal nodes and liver metastases. The best choice of chemotherapy is: A.. three cycles of BEP. B.. four cycles of EP. C. four cycles of BEP. D.. four cycles of vinblastine ifosfamide and cisplatin (VIP). E.. high dose chemotherapy and bone marrow transplant.

question #127 ANSWER=( This patient has poor prognosis NSGCT because he has non-pulmonary visceral metastases. In that setting the standard first line regimen is BEP times four cycles. EP times four cycles or BEP times three cycles are both appropriate first-line options for good prognosis disease but not poor prognosis as is demonstrated here. The last two regimens VIP (Vinblastine lfosfamide and Cisplatin) and high dose chemotherapy with bone marrow rescue can be used for salvage therapy in patients who relapse or are refractory to first-line therapy but are not standard first line regimens. Stephenson AJ Gilligan TD: Neoplasms of the testis in Wein AJ Kavoussi LR Partin AW Peters CA (eds): CAMPBELL-WALSH UROLOGY ed 11. Philadelphia Elsevier 2015 vol 1 chap 34 p 802.

128. A 68-year-old man with obesity diabetes mellitus and dialysis-dependent renal failure has painful abdominal skin lesions and a 2 cm painful eschar on the scrotum. The next step is: 42 A.. B .. C. D .. E .. observation. corticosteroids. amphotericin B. biopsy. surgical debridement. Copyright 2017 by The American Urological Association.

question #128 ANSWER=A This patient has calciphylaxis an obliterative small vessel vasculopathy that causes skin necrosis and ulceration. Calciphylaxis predominantly affects patients with chronic kidney disease; obesity and diabetes mellitus appear to be additional risk factors. Skin debridement should be carried out very judiciously as the debrided areas tend to demonstrate poor wound healing and have little impact on the pain. Corticosteroids have not demonstrated efficacy in the management of calciphylaxis. Infections have not been implicated in the pathophysiology of calciphylaxis; therefore antibiotics and antifungals are not indicated. Biopsy is contraindicated in this individual as these lesions are easily identified visually. 46 Copyright 2017 by The American Urological Association. Nigwekar SU Kroshinsky D Nazarian RM et al: Calciphylaxis: Risk factors diagnosis and treatment. AM J KID DIS 2015;66:133-146. Cimmino CB Costabile RA: Biopsy is contraindicated in the management of penile calciphylaxis. J SEX MED 2014;11:2611-2617.

13. A 49-year-old man had a lesion of the glans penis and u xcisional biopsy. reveals squamous cell CIS with a positive margin. Physical examination reveals a well-healed scar and no inguinal adenopathy. The next step is: A.. podophyllin. B.. brachytherapy. C excisi ' OLIS scar. D.. partial penectomy. E.. total penectomy. Copyright 2017 by The American Urological Association. 5

question #13 ANSWER=( Given the positive margin this patient requires further therapy around the scar. This can include the excision of the scar laser therapy or topical therapy with either 5-FU or imiquimod. Podophyllin is used to treat genital warts and has no role in the treatment of carcinoma. Partial or complete penile amputation and radiation therapy are too aggressive for this patient with CIS. Pettaway CA Crook JM Pagliaro LC: Tumors of the penis in Wein AJ Kavoussi LR Partin AW Peters CA (eds): CAMPBELL-WALSH UROLOGY ed 11. Philadelphia Elsevier 2015 vol 1 chap 37 pp 855-856.

131. A 16-year-old boy with history of spina bifida and hydrocephalus status post back closure and ventricular-peritoneal shunt placement as a neonate has sudden onset of nausea vomiting and lower abdominal pain four months after bladder augmentation. A catheterized urine specimen shows 3-5 WBOhpf with moderate bacteria and a catheterized urine volume of 120 ml. His temperature 38.0° C and his vital signs are normal. Physical examination reveals diffuse abdominal tenderness. The next step is: A.. I.V. antibiotics. B.. abdominal and pelvic ultrasound. C. CT cystogram. D.. cystoscopy. E.. exploratory laparotomy.

question #131 ANSWER=C The differential diagnosis in this patient includes cystitis infected ventricular peritoneal (VP) shunt with secondary peritonitis peritonitis from an alternative source (e.g. appendicitis) or perforation of the augmented bladder. The chief diagnosis to rule-out in this situation is a rupture of the bladder augment and a CT cystogram should be obtained. If a CT scan documents intra-peritoneal fluid collection around the VP shunt and the absence of a bladder perforation strong consideration should be made for a VP shunt infection and appropriate neurosurgical evaluation is indicated. An abdominal and pelvic ultrasound may demonstrate intra-peritoneal fluid but could not differentiate an infected VP shunt with an associated fluid collection versus rupture of the augment. Cystoscopy places the individual at a higher risk of sepsis if a bladder rupture has occurred and is not as diagnostically accurate as a CT cystogram. Starting I.V. antibiotics may be appropriate but would not allow the physician to accurately diagnose the underlying etiology. Exploratory laparotomy is not diagnostically indicated at this time. Dahl DM: Use of intestinal segments in urinary diversion in Wein AJ Kavoussi LR Partin AW Peters CA (eds): CAMPBELL-WALSH UROLOGY ed 11. Philadelphia Elsevier 2015 vol 3 chap 97 p 2290.

132. A 14-year-old girl has recurrent uric acid stones. Her pediatrician has increased her fluid intake limited dietary animal protein and started allopurinol 50 mg/day. Urine pH is 5.5. The next step is: A.. increase allopurinol to 150 mg/day. B.. alpha-mercapotopropionylglycine. C. limit sodium intake to < 2.3 gm/day. D.. potassium citrate. E.. increase intake of fruits and vegetables.

question #132 ANSWER=D First line therapy in patients with uric acid stones is dietary restriction of animal proteins and alkalinization of urine to pH> 6 with potassium citrate. If this fails then increasing allopurinol to 100-300 mg/day will reduce the urinary uric acid. Alpha-mercaptopropionylglycine is used for treatment of cystine stones. Reducing sodium intake will reduce stone risk in patients with hypercalciuria. Increase in intake of fruits and vegetables does not have sufficient clinical evidence to reduce stone occurrence in patients with low urinary citrate. Morgan MSC Pearle MS: Medical management of kidney stones. AUA UPDATE SERIES 2015 vol 34 lesson 20 pp 188-193.

70. A nine-year-old girl with spina bifida has urinary incontinence. Urodynamics shows normal capacity with good compliance and a low Valsalva lPP. She has a fascial sl · ' ·s initially dry. Four months later she has recurrent incontinence. Ultrasound is normal. Videourodynamics demonstrate a bladder capacity of 250 ml a pressure-specific bladder capacity of 150 ml at 30 cm H2O a detrusor lPP of 60 cm of H2O and bilateral grade 1 reflux. The next step is: A.. prophylactic antibiotics. B.. oral antimuscarinic. C. endoscopic injection of bladder neck. D.. bladder augmentation. E.. bilateral ureteral reimplantation.

question #70 ANSWER=B Following bladder neck procedures in children with neurogenic sphincter incompetence the unmasking or development of detrusor hostility can be seen in a subset of children. This is manifested by a decrease in bladder compliance and/or increase in detrusor overactivity. In severe cases hydronephrosis and secondary reflux can develop. Thus the bladder and upper tracts must be monitored very carefully following bladder outlet procedures when augmentation is not performed concomitantly. When bladder hostility is recognized antimuscarinics should be instituted as first line management. However this will not be effective therapy in approximately one-third of patients. In this patient population serial urodynamic studies and onabotulinumtoxinA injections can be used for management or alternatively definitive treatment with a bladder augmentation can be pursued. In the presence of new onset of hydronephrosis and VUR antibiotic prophylaxis will not be adequate therapy. The outlet resistance is likely adequate for continence and the new incontinence is more likely due to worsening bladder dynamics. Bladder augmentation without a trial of antimuscarinics would not be the next step. Ureteral reimplantation is not indicated as the low grade VUR may resolve with bladder management. Adams MC Joseph DB Thomas JC: Urinary tract reconstruction in children in Wein AJ Kavoussi LR Partin AW Peters CA (eds): CAMPBELL-WALSH UROLOGY ed 11. Philadelphia Elsevier 2015 vol 4 chap 145 p 3336.

133. The event that initiates detumescence following a normal erection is: A.. a transient rise in intracorporal pressure. B.. a slow decrease in intracorporal pressure. C. a rapid decrease in intracorporal pressure. D.. cavernosal smooth muscle relaxation. E.. endothelial relaxation.

question #133 ANSWER=A In the study of animal models of erection and detumescence there are six to seven phases that occur. In the flaccidity or detumescence phase there are three phases. The initial event that occurs is cavernosal smooth muscle contraction (not relaxation which initiates an erection) that causes an initial rise in the intracorporal pressure. Thus endothelial relaxation does not initially occur. This is followed by a slow pressure decrease as the reopening of the venous channels occurs with resumption of the baseline arterial flow. The final phase is a rapid drop in intracorporal pressure leading to complete flaccidity. Lue TF: Physiology of penile erection and pathophysiology of erectile dysfunction in Wein AJ Kavoussi LR Partin AW Peters CA (eds): CAMPBELL-WALSH UROLOGY ed 11. Philadelphia Elsevier 2015 vol 1 chap 26 pp 623-629. 48 Copyright 2017 by The American Urological Association.

134. A six-year-old boy who had a right pyeloplasty in infancy for a UPJ obstruction now has right flank pain and vomiting. An ultrasound performed six months ago demonstrated minimal hydronephrosis. Current imaging shows moderate right hydronephrosis with a 7 mm calculus at the UPJ. The next step is: A.. tamsulosin. B.. SWL. C. ureteroscopic laser lithotripsy. D.. PCNL. E.. revision pyeloplasty and nephrolithotomy.

question #134 ANSWER=C Tamsulosin although not approved for use in children has shown efficacy as medical expulsive therapy in children but it is unlikely to be successful with a stone of this size particularly since it has not entered the ureter. SWL is used in children with stones up to 15 mm but has poor stone free rates in children with a history of urologic condition or reconstruction. Ureteroscopy and PCNL are preferred modalities in this situation but ureteroscopic management is less invasive with excellent stone free rates for stones< 15 mm. Revision pyeloplasty is not appropriate since there was minimal hydronephrosis on recent ultrasound suggesting no evidence of UPJ obstruction. Schneck FX Ost MC: Surgical management of pediatric stone disease in Wein AJ Kavoussi LR Partin AW Peters CA (eds): CAMPBELL-WALSH UROLOGY ed 11. Philadelphia Elsevier 2015 vol 4 chap 135 pp 3107-3112.

135. In a morbidly obese man with erectile dysfunction the serum androgen profile is most likely to show: A.. t total testosterone t estradiol and t serum hormone-binding globulin. B.. t total testosterone t estradiol and j serum hormone-binding globulin. C. t total testosterone j estradiol and t serum hormone-binding globulin. D.. j total testosterone t estradiol and j serum hormone-binding globulin. E.. j total testosterone j estradiol and t serum hormone-binding globulin.

question #135 ANSWER=C The majority of testosterone that circulates is primarily bound to serum hormone binding globulin (SHBG) with albumin and cortisol binding globulin (CBG) playing lesser roles. Only 1-3% of total testosterone circulates unbound (free). SHBG production in the liver and Sertoli cells are altered by obesity liver disease and nephrotic syndrome. Obese males have reduced SHBG and thus lower total testosterone while the free testosterone levels are generally unchanged. The excess aromatase activity in visceral fat in obese men translates into greater testosterone breakdown to estradiol which further lowers the total testosterone level and elevates the estradiol level. Parsons JK Hsieh TC: Integrated men's health: Androgen deficiency cardiovascular risk and metabolic syndrome in Wein AJ Kavoussi LR Partin AW Peters CA (eds): CAMPBELL-WALSH UROLOGY ed 11. Philadelphia Elsevier 2015 vol 1 chap 23 p 539.

136. After successful pneumoperitoneum is achieved with a Veress needle a 12 mm trocar is inserted above the umbilicus along the mid-line without laparoscopic assistance. Upon removal of the obturator brisk pulsatile blood is seen emanating from the trocar associated with abrupt tachycardia and hypotension to 85/60 mmHg. The next step is to fluid resuscitate call for a vascular surgeon and: A.. increase insufflation pressure. B.. close trocar valve maintain its position and perform exploratory laparotomy. C. remove trocar and perform exploratory laparotomy. D.. place additional trocar to assess injury. E.. place hand-assist port and repair injury laparoscopically.

question #136 ANSWER=B This is a scenario of a significant arterial injury due to blind placement of a large bore trocar. The most likely injury in this case is either to the aorta or common iliac arteries from blind passage of a trocar. As the patient is rapidly becoming unstable a vascular surgeon should be consulted but immediate action is required. Simply increased insufflation pressure would not address this major vascular injury. With the trocar presumably still within the injured vessel the trocar should be closed and kept in place if possible to allow for the exploratory laparotomy to be directed to the precise location of the injury. The trocar should not be completely removed as it may be providing some element of tamponade of the injured vessel. Withdrawal of the trocar from the vessel lumen in addition to the loss of pneumoperitoneum could result in rapid exsanguination; however if the trocar has already been withdrawn out of the injured vessel and a laparoscope placed attempts can be made to assess the degree and site of injury laparoscopically. Rapid conversion to exploratory laparotomy should be performed especially in the scenario of cardiovascular collapse. In such a case the trocar and laparoscope can be angulated along the anterior abdominal wall allowing for rapid cut down onto the trocar. As this particular patient is in rapid clinical decline adding an additional trocar or converting to a hand-assisted technique to attempt to repair the injury laparoscopically would be too time consuming and potentially Copyright 2017 by The American Urological Association. 49 dangerous. However in a more stable situation both may be reasonable approaches depending on the laparoscopic experience and skill level of the surgeon. Ordon M Eichel L Landman J: Fundamentals of laparoscopic and robotic urologic surgery in Wein AJ Kavoussi LR Partin AW Peters CA (eds): CAMPBELL-WALSH UROLOGY ed 11. Philadelphia Elsevier 2015 vol 1 chap 10 p 218.

137. A 62-year-old man had eight intralesional collagenase injections for a 60 degree dorsal penile curvature one year ago. He now has a 25 degree dorsal penile curvature and moderate erectile dysfunction unresponsive to PDE-5 inhibitor and wants further treatment. The next step is: 44 A.. B .. C. D .. E .. reassurance. eight additional intralesional collagenase injections. eight intralesional verapamil injections. penile plication. insertion of penile prosthesis.

question #137 ANSWER=E The patient is bothered by his disease and his condition is unlikely to improve; therefore reassurance would not be an appropriate option. Additional collagenase would be considered off-label use as only eight injections are currently indicated for treatment. In addition collagenase injections are only indicated for greater than 30 degrees of curvature. There is no data to suggest that switching to verapamil injections would offer a better response and verapamil injections are considered off-label use. A penile plication should be avoided in men with moderate ED. The best option is insertion of penile prosthesis. Nehra A Alterowitz R Culkin DJ et al: PEYRONIE'S DISEASE: AUA GUIDELINE. American Urological Association Education and Research Inc 2015; 194: pp 745-753. http://www.auanet.org/education/guidelines/peyronies-disease.cfm

138. Two weeks after undergoing a transobturator mid-urethral sling a 45-year-old female long distance runner complains of severe groin pain radiating to both inner thighs. Urinalysis is normal and PVR is 10 ml. Narcotics and anti-inflammatory medications are not helpful. The next step is: A.. observation. B.. refer to pain management. C. refer to physical therapy. D.. urethrolysis. E.. sling removal and retropubic sling.

question #138 ANSWER=A Groin/thigh pain after transobturator sling in women with thin athletic builds is not uncommon. Indeed some experts believe women fitting this description should preferentially undergo a retropubic mid-urethral sling rather than a transobturator sling. If a patient should develop groin/thigh pain following a transobturator sling conservative therapy with NSAIDs for analgesia and the passage of time should resolve the majority of symptoms. If however the pain persists after six to eight weeks consideration for referral to a pain clinic for trigger point injections and a physical therapy consultation may be of benefit. Sling urethrolysis and removal should be reserved for recalcitrant problems failing the aforementioned interventions. Dmochowski RR Osborn DJ Reynolds WS: Slings: Autologous biologic synthetic and midurethral in Wein AJ Kavoussi LR Partin AW Peters CA (eds): CAMPBELL-WALSH UROLOGY ed 11. Philadelphia Elsevier 2015 vol 3 chap 84 pp 2031-2033.

14. A ten-day-old infant boy is hospitalized for failure to thrive. After his umbilical stump fell off fluid has intermittently drained from the umbilicus. The umbilical fluid has a creatinine of 10 mg/dl and grows> 105 CFU/ml of E.coli. The next step should be antibiotics and: A.. observation. B.. urethral catheter drainage. C. VCUG. D.. cauterization of tract. E.. closure of fistula.

question #14 ANSWER=C The differential diagnosis of a wet umbilicus in the infant includes patent urachus omphalitis simple granulation of the healing stump patent vitelline or omphalomesenteric duct infected umbilical vessel and external urachal sinus. The finding of a urinary creatinine level in the fluid draining from the umbilical stump suggests a patent urachus. While probing the urachal tract may aid in diagnosis a VCUG should confirm the diagnosis and fully evaluate the lesion and any associated bladder outlet obstruction. Cauterization of the tract and closure of the fistula are not indicated until VCUG is performed to rule-out bladder outlet obstruction. Urethral catheter drainage will not definitively treat the patent urachus if obstruction is present. Frimberger D Kropp BP: Bladder anomalies in children in Wein AJ Kavoussi LR Partin AW Peters CA (eds): CAMPBELL-WALSH UROLOGY ed 11. Philadelphia Elsevier 2015 vol 4 chap 138 p 3176.

140. A 24-year-old man with a gunshot wound shattering the L-4 vertebral body achieves stable neurogenic bladder dysfunction nine months later. Pressure flow urodynamic studies will likely show: A.. detrusor overactivity sphincter dyssynergia. B.. detrusor overactivity normal sphincter EMG. C. detrusor areflexia sphincter dyssynergia. D.. detrusor areflexia normal sphincter EMG. E.. detrusor areflexia denervation potentials on EMG.

question #140 ANSWER=E An injury to the vertebral column at L-4 injures the cauda equina and depending on the extent of neural damage will produce a loss of motor and sensory fibers to the bladder pelvic floor and external sphincter. Detrusor sphincter dyssynergia is produced by suprasacral spinal cord lesions that interrupt the ascending and descending pathways between the sacral spinal cord and the center for reflex detrusor and urethral function in the brain stem. Reflex detrusor function requires sacral root and sacral cord integrity. While an areflexic bladder faces fixed internal sphincter activity that activity is normal and not truly dyssynergic. Since within the sacral and lumbar canal the nerve roots are intermingled a lesion that produces detrusor areflexia would be expected to have a similar effect on the external sphincter hence the denervation potentials. Wein AJ Dmochowski RR: Neuromuscular dysfunction of the lower urinary tract in Wein AJ Kavoussi LR Partin AW Peters CA (eds): CAMPBELL-WALSH UROLOGY ed 11. Philadelphia Elsevier 2015 vol 3 chap 75 pp 1773-1775.

141. A 42-year-old man has a three month history of a 60 degree dorsal penile curvature and significant penile pain with erections. The best treatment is: A.. ibuprofen. B.. Vitamin E. C. pentoxifylline. D.. penile stretching device. E.. intralesional collagenase injections.

question #141 ANSWER=A According to the 2015 AUA Guideline on Peyronie's NSAIDS are an appropriate treatment option for men presenting with penile pain in the active phase. Vitamin E is not recommended as a treatment option for men with Peyronie's disease. Pentoxifylline and penile stretching device are considered off-label use for the treatment of Peyronie's disease. Finally intralesional collagenase injections should only be offered during the stable phase of the disease. No oral agents such as Potaba other than NSAIDS are indicated for the treatment of Peyronie's disease. Nehra A Alterowitz R Culkin DJ et al: PEYRONIE'S DISEASE: AUA GUIDELINE. American Urological Association Education and Research Inc 2015;194:745-753. http://www.auanet.org/education/guidelines/peyronies-disease.cfm

142. While performing a right robotic partial nephrectomy and during an exchange of a robotic instrument by the bedside assistant the new instrument is inadvertently advanced into the liver. The most likely cause of this injury is due to: A.. B .. C. D .. E .. dislodgement of instrument faceplate. forceful insertion of the instrument. lack of visualization of instrument tip during advancement. accidental activation of instrument arm clutch button. defective instrument.

question #142 ANSWER=D Unlike conventional laparoscopy a robotic instrument must first be successfully engaged to the instrument faceplate before it can be advanced into the operative field. During routine exchange of a robotic instrument a safety mechanism is built into the system that allows for safe and automatic return of the instrument tip to 1 mm short of the final position of the prior instrument. In addition the precise trajectory of the previous instrument is saved allowing the new instrument to return to the same exact location by simply advancing the instrument once engaged by the faceplate. This feature is called the guided instrument exchange. Although all laparoscopic and robotic instruments should be inserted under laparoscopic view this safety feature in essence returns the new instrument to nearly the exact same location thus obviating the absolute need for the console surgeon to visualize the actual insertion and advancement of the new instrument tip. Taken together this guided instrument exchange safety feature prevents "past pointing" of the new instrument into surrounding vital anatomy whether by gentle or forceful insertion. However once the clutch button of the robotic arm is reset this safety mechanism is lost and the instrument requires manual introduction under laparoscopic view in coordination with the console surgeon. In the above scenario the bedside assistant likely activated the clutch button accidentally thus releasing the safety mechanism. Proper and Copyright 2017 by The American Urological Association. 51 successful engagement of the instrument to the faceplate would not be possible if the instrument was defective or the instrument faceplate was dislodged. AUA Urologic Robotic Surgery Course: Fundamentals of Robotic Surgery (https://www.auanet.org/education/modules/robotic-surgery/module1 .cfm)

143. A 60-year-old hypertensive man is well-controlled on a beta-blocker. CT angiography obtained for evaluation of a 3 cm infrarenal abdominal aortic aneurysm incidentally reveals a 50% ostial stenosis of the left renal artery. Doppler ultrasound of the renal arteries plasma renin and serum creatinine are normal. The next step is: A.. serial blood pressure and renal clearance evaluations. B.. repeat CT angiogram and Doppler ultrasound of the renal arteries in one year. C. split differential renal vein renin sampling. D.. percutaneous transluminal angioplasty. E.. percutaneous transluminal angioplasty with stent placement.

question #143 ANSWER=A Renal artery stenosis is estimated to be the etiology of elevations in blood pressure in< 1 % of the total hypertensive patient population and in 10-20% of patients with refractory hypertension. The radiographic presence of renal arterial stenosis alone is not however adequate justification to warrant a correction in a hypertensive patient. The stenotic lesion must be functionally significant in essence reduce blood flow in an amount sufficient to activate renin release. Classically arteriographic findings associated with an increase in renin secretion are renal arterial narrowing exceeding greater than 75 percent or greater than 50 percent renovascular stenosis with post-stenotic dilation. However the exact degree of renal artery stenosis that would justify revascularization is not known. In patients found to have a 50% or greater arterial stenosis duplex Doppler ultrasonographic scanning is recommended. This test is noninvasive relatively inexpensive can be used in patients with any level of renal function and is both sensitive (98%) and specific (98%) for the presence of renovascular induced hypertension. Currently this patient is asymptomatic with excellent blood pressure control and with a normal Doppler ultrasound study. Serial monitoring of blood pressure and renal function are sufficient for follow-up. In patients with a 75% narrowing 10% may progress to complete occlusion within two years and up to 60% will progress to stage 3 or higher renal failure within six years. Consideration for interval radiologic follow-up is usually added to the follow-up protocols in this asymptomatic patient population. Split differential renal vein renin measurements are now performed very infrequently because of their limited clinical utility and need for invasive catheterization. A renal vein renin ratio of greater than or equal to 1.5 (affected to non-affected side) is considered significant for the presence of renin-dependent hypertension; its clinical utility in being able to determine who will respond to renal revascularization is controversial. However it has been found to be useful for prognostic purposes in pediatric patients where there is an ipsilateral diminished renal function (less than 25%) and nephrectomy is a consideration for resolution of hypertension. Percutaneous transluminal balloon angioplasty+/stenting is not indicated in this patient who does not have renovascular hypertension. Randomized controlled trials comparing renal stent placement with balloon treatment alone have documented procedural superiority for primary stent placement. The need to reintervention in the percutaneous transluminal renal angioplasty group is approximately threefold higher (48%) compared with that of the stent group (14%). The reduced restenosis with stenting compared with angioplasty alone was not however associated with a significant difference in benefits in hypertension control or renal function. Gui mi FA Reiser IW Spitalewitz S: Renovascular hypertension and ischemic nephropathy in Wein AJ Kavoussi LR Partin AW Peters CA (eds): CAMPBELL-WALSH UROLOGY ed 11. Philadelphia Elsevier 2015 vol 2 chap 45 p 1028.

144. Angiotensin II maintains GFR during conditions of hypovolemia by causing: A.. afferent arteriolar vasodilation. B.. efferent arteriolar vasoconstriction. C. increased renal medullary blood flow. D.. renal artery dilation. E.. passive sodium absorption.

question #144 ANSWER=B The primary and direct mechanism by which angiotensin II (ATII) maintains GFR during hypoperfusion is by efferent arteriolar vasoconstriction. The effect of vasoconstriction is greater 52 Copyright 2017 by The American Urological Association. for the efferent than afferent arteriole. ATII causes a decrease in renal medullary blood flow rather than an increase. ATII may result in renal artery constriction during hypovolemia. ATII increases aldosterone production that affects the cortical collecting tubule activating sodium channels resulting in net sodium and water absorption. Shoskes DA McMahon AW: Renal physiology and pathophysiology in Wein AJ Kavoussi LR Partin AW Peters CA (eds): CAMPBELL-WALSH UROLOGY ed 11. Philadelphia Elsevier 2015 vol 2 chap 44 pp 1009-1010.

145. A 14-month-old boy has a urethrocutaneous fistula following a hypospadias repair. The most important step for successful fistula repair is: A.. intra-operative urethral calibration. B.. use of fine absorbable sutures. C. post-operative drip stent or catheter. D.. multi-layer closure. E.. vascularized interposition flap.

question #145 ANSWER=A Urethrocutaneous fistula is a complication of hypospadias repair. Although all of the choices listed may reduce fistula recurrence the most important step for a successful outcome of fistula repair is evaluation for and assurance of absence of urethral stricture distal to the fistula site. Repair therefore needs to include assessment for distal obstruction as well as excision of the fistula tract with closure of the urethral opening and vascularized flap coverage over the defect. Successful fistula repair requires healthy skin and subcutaneous tissue immediately surrounding the fistula site. In general urethral stenting does not impact outcome following a straight forward urethrocutaneous fistula repair and post-operative care requirement should be minimal. Snodgrass WT Bush NC: Hypospadias in Wein AJ Kavoussi LR Partin AW Peters CA (eds): CAMPBELL-WALSH UROLOGY ed 11. Philadelphia Elsevier 2015 vol 4 chap 147 p 3419.

146. A 52-year-old commercial airline pilot has asymptomatic microhematuria on a screening medical exam. CT scan demonstrates multiple bilateral renal calculi all less than 5 mm. The next step is cystoscopy and: A.. observation. B.. medical expulsive therapy. C. potassium citrate. D.. SWL. E.. ureteroscopy.

question #146 ANSWER=E The patient has a vocational indication for stone removal. The procedure most likely to render him stone-free in a single procedure is ureteroscopy. As the stones are in the kidney medical expulsive therapy is not indicated. The patient has not yet undergone metabolic work-up so potassium citrate at this point is premature. SWL will not permit a bilateral treatment in the same session and does not provide for immediate stone clearance so that the patient may resume his vocation. Pearle MS Goldfarb DS Assimos DG et al: MEDICAL MANAGEMENT OF KIDNEY S TONES: AUA GUIDELINE: AUA GUIDELINE. American Urological Association Education and Research Inc 2014. http://www.auanet.org/education/guidelines/management-kidney-stones.cfm

148. A 44-year-old man is scheduled to undergo renal transplantation. Pre-operatively he has a 1.2 cm left pelvic stone in his native kidney. The next step is: A.. SWL. B.. ureteroscopy with laser lithotripsy. C. PCNL. D.. left nephrectomy of native kidney. E.. proceed with renal transplantation.

question #148 ANSWER=E Asymptomatic renal stones in native kidneys do not require any pre-operative intervention. Thus this patient should proceed with renal transplantation and the stone does not need to be addressed with procedures such as SWL ureteroscopy PCNL and nephrectomy. Gritsch HA Blumberg JM: Renal transplantation in Wein AJ Kavoussi LR Partin AW Peters CA (eds): CAMPBELL-WALSH UROLOGY ed 11. Philadelphia Elsevier 2015 vol 2 chap 47 p 1069.

149. A 20-year-old healthy woman with no prior surgical history has lower abdominal pain and no urge to urinate for 36 hours. A catheter is placed with a return of 1300 ml. Neurologic evaluation is unremarkable. The urodynamic finding most likely to suggest the definitive diagnosis is: A.. impaired compliance. B.. low amplitude detrusor contractions. C. high voiding pressure low flow. D.. abnormal firing on electromyography. E.. detrusor external sphincter dyssynergia.

question #149 ANSWER=D Fowler's syndrome first described in 1985 refers to the development of urinary retention in young women in the absence of overt neurologic disease. The typical clinical history is that of a woman younger than 30 years who has found herself unable to void for a day or more. It is noteworthy that 50% of affected patients will be found to have polycystic ovaries in association with the urinary pathology. Patients classically will not complain of urinary urgency but invariably complain of increasing lower abdominal pain and discomfort. Clinical suspicion for Fowler's syndrome should be given if a young woman is found in urinary retention bladder capacity of over 1 L with no sensation of urinary urgency. On needle electrode electromyographic (EMG) examination of the external urethral sphincter abnormal EMG activity with complex repetitive discharges and decelerating bursts will be noted. This abnormal EMG electrical activity impairs external urinary sphincter relaxation. Simultaneous CMG studies reveal excellent bladder compliance associated with detrusor acontractility. Due to the absence of detrusor contractility true detrusor external sphincter dyssnergy is not seen. While the same EMG abnormality on occasion may be found in women with obstructed voiding it will not be associated with detrusor acontractility; rather the patient will be found to have high voiding pressures and low urinary flow rates. Wein AJ Dmochowski RR: Neuromuscular dysfunction of the lower urinary tract in Wein AJ Kavoussi LR Partin AW Peters CA (eds): CAMPBELL-WALSH UROLOGY ed 11. Philadelphia Elsevier 2015 vol 3 chap 75 p 1789.

15. A 76-year-old man with diabetes has hematuria. CT urogram shows a 5 mm filling defect in the distal right ureter. Ureteroscopic biopsy reveals a low grade urothelial carcinoma. The next step is: A.. nephroureterectomy. B.. ureteral stent and intravesical BCG. C. segmental resection and ureteroureterostomy. D.. ureteroscopic tumor ablation. E.. distal ureterectomy and reimplantation.

question #15 ANSWER=D In an older patient with medical problems ureteroscopic biopsy electro-resection and laser destruction have been utilized to successfully manage small low grade non-invasive ureteral Copyright 2017 by The American Urological Association. 5 tumors. This approach may avoid nephroureterectomy or partial ureteral resection. Although historically distal ureterectomy and reimplantation has been considered endoscopic management of solitary low-grade tumors has become the preferred treatment. Upper tract BCG may be effective for high-grade disease but delivery of the agent is least consistent when relying on reflux around a ureteral stent. Smith AK Matin SF Jarrett TW: Urothelial tumors of the upper urinary tract and ureter in Wein AJ Kavoussi LR Partin AW Peters CA (eds): CAMPBELL-WALSH UROLOGY ed 11. Philadelphia Elsevier 2015 vol 2 chap 58 pp 1388-1396.

150. A 64-year-old healthy man with back pain undergoes prostate biopsy for a PSA of 126 ng/ml. The biopsy reveals Gleason 8 (4+4) prostate cancer. Bone scan reveals multiple lesions in his lumbar spine ribs and right scapula. CT imaging reveals pelvic and retroperitoneal adenopathy. The next step is androgen deprivation therapy and: A.. ketoconazole. B.. sipuleucel-T. C. enzalutamide. D.. abiraterone. E.. docetaxel.

question #150 ANSWER=E The CHAARTED trial has shown clear survival benefits to ADT and docetaxel chemotherapy in the setting of high volume hormone-sensitive metastatic prostate cancer. In the pivotal trial Sweeney et al showed an HR of 0.61 (0.47-0.80) p=0.0003 with median overall survival 54 Copyright 2017 by The American Urological Association. advantage from 44.0 months to 57.6 months for the entire cohort. Specifically for the high volume disease the HR was 0.60 (0.45-0.80) p=0.0006 with median overall survival advantage from 32.2 months to 49.2 months. The STAMPEDE trial has shown similar results thus confirming the efficacy of cytotoxic chemotherapy with ADT in the hormone-sensitive metastatic setting. While ketoconazole could certainly aid in attaining castrate levels of testosterone this patient does not display impending pathologic fracture or cord compression and there is no clear indication for the addition of ketoconazole. Sipuleucel-T enzalutamide and abiraterone are all approved only for metastatic castration-resistant prostate cancer. Sweeney CJ Chen Yu-Hui Carducci M et al Chemohormonal therapy in metastatic hormonesensitive prostate cancer. NEJM 2015;373:737-746. Copyright 2017 by The American Urological Association. 55

16. A 72-year-old woman undergoes an abdominal hysterectomy. In the recovery room she is anuric for four hours despite several boluses of I.V. fluids. Her indwelling catheter is patent blood pressure is 100/50 mmHg and pulse is 100 8PM. Her estimated blood loss during the procedure was 1000 ml. The best explanation for her condition is: A.. acute tubular necrosis. B.. bilateral ureteral obstruction. C. prerenal azotemia. D.. hypovolemic shock. E.. bladder perforation.

question #16 ANSWER=B Hysterectomy accounts for over 50% of iatrogenic ureteral injuries and a high index of suspicion must be kept in this scenario. Anuria always implies complete ureteral obstruction until proven otherwise. The two most likely areas where the ureter can be occluded during hysterectomy are at the level of the broad ligaments and at the vaginal cuff and bladder trigone. Consequently the most likely finding in this patient would be a ureteral obstruction at the level of the vaginal cuff. While hypovolemic shock and low urine output are commonly seen after all types of abdominal operations the presence of anuria in this case suggests an obstructive etiology. Acute tubular necrosis does not normally occur in a precipitous fashion as in this case. Bladder perforation is unlikely if the catheter has been irrigated with good return. Santucci RA Chen ML: Upper urinary tract trauma in Wein AJ Kavoussi LR Partin AW Peters CA (eds): CAMPBELL-WALSH UROLOGY ed 11. Philadelphia Elsevier 2015 vol 2 chap 50 p 1158.

17. The boundaries of a standard inguinal lymph node dissection for the treatment of penile cancer should include: 6 A.. B .. C. D .. E .. inguinal ligament sartorius adductor longus. inguinal ligament sartorius fascia lata. inguinal ligament gracilis adductor longus. Cooper's ligament sartorius adductor longus. Cooper's ligament gracilis adductor brevis. Copyright 2017 by The American Urological Association.

question #17 ANSWER=A The limits of dissection for a standard inguinal lymph node dissection are the triangular area bounded by the inguinal ligament superiorly the sartorius muscle laterally and the adductor longus medially. Modified templates are frequently used for inguinal lymph node dissections. Angermeier KW Sotelo R Sharp DS: Inguinal node dissection in Wein AJ Kavoussi LR Partin AW Peters CA (eds): CAMPBELL-WALSH UROLOGY ed 11. Philadelphia Elsevier 2015 vol 1 chap 39 pp 894-896.

18. A 17-year-old boy with spina bifida has a two-week history of fever and vague abdominal pain. He has a prior bladder a on appendicovesicostomy bladder nee' nd bilateral cross-trigonal ureteral reimplantation. Renal ultrasound is shown. The next step is antibiotics and: A.. observation. B.. tamsulosin. C · scopy and ureteral stent. D.. percutaneous nephrostomy tube. E.. CT cystogram.

question #18 ANSWER=D This boy has a large proximal ureteral stone with acoustic shadowing and debris in the collecting system on ultrasound. With the size of the stone and duration of symptoms the stone is unlikely to pass spontaneously and will require surgical intervention. The previous reconstructive procedure (bladder neck sling cross trigonal reimplant and Mitrofanoff) makes bladder/ureteral access difficult and the small ureteral stent may become occluded from mucus in bladder from the bladder augmentation. The best way to remove the stone will likely be through the percutaneous approach and in the face of fever an initial drainage procedure with a nephrostomy tube will allow a period of antibiotic therapy and access for percutaneous removal. The patient is unlikely to have a secondary bladder perforation making CT cystogram not helpful. Schneck FX Ost MC: Surgical management of pediatric stone disease in Wein AJ Kavoussi LR Partin AW Peters CA (eds): CAMPBELL-WALSH UROLOGY ed 11. Philadelphia Elsevier 2015 vol 4 chap 135 pp 3112-3118.

19. 5-alpha-reductase deficiency is associated with: A.. poorly differentiated Wolffian structures. B.. presence of developed Mullerian structures. C gynecomastia. D.. elevated concentration of testosterone at puberty. E.. elevated dihydrotestosterone:testosterone ratio. Copyright 2017 by The American Urological Association. 7

question #19 ANSWER=D The defective conversion of testosterone to dihydrotestosterone due to 5-alpha-reductase deficiency produces a unique form of male disorder of sexual differentiation. At birth the Mullerian structures are absent (as Mullerian-inhibited substance is made appropriately by the testes) and testosterone-dependent Wolffian structures are well-differentiated. The genitalia are ambiguous to a variable degree. Gynecomastia can be seen in adults on 5-alpha-reductase inhibitors but is not seen in congenital 5-alpha-reductase deficiency. The 5-alpha-reductase enzyme defect is generally incomplete and at puberty the plasma concentration of dihydrotestosterone while low is detectable. Plasma testosterone and LH are elevated while the dihydrotestosterone:testosterone ratio is abnormally low. This is due to dihydrotestosterone being a major inhibitor of LH production via the gonadal-pituitary negative feedback loop. 5- alpha-reductase deficiency is inherited as an autosomal recessive trait and the enzymatic defect exhibits genetic heterogeneity. Diamond DA Yu RN: Disorders of sexual development: Etiology evaluation and medical management in Wein AJ Kavoussi LR Partin AW Peters CA (eds): CAMPBELL-WALSH UROLOGY ed 11. Philadelphia Elsevier 2015 vol 4 chap 150 p 3493.

20. A 32-year-old woman with a solitary kidney underwent urinary diversion with an ileal conduit as a child. She has stable moderate hydronephrosis but her serum creatinine has risen to 2.8 mg/dl. A loopogram shows no reflux and no residual urine. A diuretic renogram reveals delay in uptake of the radiopharmaceutical and poor response to diuretic with a T1/2 of 22 minutes. The next step is: A.. hydrate and repeat the renogram. B.. contrast CT scan. C. percutaneous nephrostomy tube. D.. renal biopsy. E.. revision of the ileal conduit.

question #20 ANSWER=( This woman most likely has chronic renal insufficiency and the renogram reflects this condition. Diseased kidneys may respond poorly to diuretic in the absence of obstruction. The only way to establish conclusively if an obstruction exists would be to place a nephrostomy tube. A pressure-flow study can then be performed and the serum creatinine observed. A renal biopsy if performed is likely to show focal segmental sclerosis and/or chronic pyelonephritis but this is not helpful in management. Non-contrast CT scan would be helpful to rule-out an obstructing stone; however a contrast CT scan is contraindicated due to poor renal function. It is unlikely that hydration would reverse any renal dysfunction unless the patient were very dehydrated and pre-renal which does not fit this scenario. Revision of the ileal conduit is not indicated until an obstruction has clearly been demonstrated. Weiss RM Martin DT: Physiology and pharmacology of the renal pelvis and ureter in Wein AJ Kavoussi LR Partin AW Peters CA (eds): CAMPBELL-WALSH UROLOGY ed 11. Philadelphia Elsevier 2015 vol 2 chap 43 pp 998-999. Dahl DM: Use of intestinal segments in urinary diversion in Wein AJ Kavoussi LR Partin AW Peters CA (eds): CAMPBELL-WALSH UROLOGY ed 11. Philadelphia Elsevier 2015 vol 3 chap 97 pp 2302-2305. Bishoff JT Rastinehad AR: Urinary tract imaging: Basic principles of computed tomography magnetic resonance imaging and plain film in Wein AJ Kavoussi LR Partin AW Peters CA (eds): CAMPBELL-WALSH UROLOGY ed 11. Philadelphia Elsevier 2015 vol 1 chap 2 pp 37-39. Copyright 2017 by The American Urological Association. 7

21. A 60-year-old man has a high grade T1 urothelial carcinoma of the bladder. He receives the fifth of six weekly instillations of intravesical BCG. Twelve hours later he has a temperature of 39.5° C difficulty breathing and hypotension. The most likely cause of this complication is: A.. reflux of BCG into the upper tracts. B.. acute UTI. C. traumatic catheterization. D.. more virulent strain of BCG. E.. impaired immunological state.

question #21 ANSWER=C The majority of patients tolerate BCG instillation well. In 2 602 patients treated with different strains of BCG high fever(> 39 degrees C) was noted in 2.9% of patients. Life-threatening BCG sepsis was noted in 0.4%. Fever > 39.5 degrees C that does not resolve within 12 hours despite antipyretic therapy is potentially dangerous. Since most cases of BCG sepsis are associated with I.V. absorption of BCG it is recommended that BCG not be given until at least one week after tumor resection. In the patients who died from BCG sepsis almost all cases had traumatic catheterization before instillation therapy or they were treated too early after TURBT or biopsy. Treatment should include isoniazid 300 mg rifampin 600 mg and ethambutol 1200 mg daily. After antituberculosis drugs are started corticosteroids may be given if the patient is toxic. Given the timing of the signs and symptoms in relation to the BCG instillation acute UTI is much less likely to be a cause of this patient's symptoms. Jones JS: Non-muscle-invasive bladder cancer (Ta T1 and CIS) in Wein AJ Kavoussi LR Partin AW Peters CA (eds): CAMPBELL-WALSH UROLOGY ed 11. Philadelphia Elsevier 2015 vol 3 chap 93 p 2215.

22. A 45-year-old man with a history of hypertension and significant tobacco use has erectile dysfunction one year following a crush injury to the pelvis. An arteriogram at the time of his injury revealed unilateral focal occlusion of the internal pudenda! artery. Treatment should be: A.. intracavernous vasoactive injections. B.. dorsal venous ligation. C. percutaneous angioplasty. D.. arterial revascularization. E.. penile prosthesis.

question #22 ANSWER=A Percutaneous or surgical revascularization of the internal pudenda! arteries is not indicated owing to the patient's age and associated risk factors for atherosclerotic vascular disease (e.g. hypertension and smoking). There is no indication for venous ligation. Owing to the vascular disease penile injections may not be successful but should be implemented prior to insertion of a penile prosthesis. Burnett AL II: Evaluation and management of erectile dysfunction in Wein AJ Kavoussi LR Partin AW Peters CA (eds): CAMPBELL-WALSH UROLOGY ed 11. Philadelphia Elsevier 2015 vol 1 chap 27 pp 664 667.

23. A 39-year-old man with a large left varicocele requests vasectomy reversal four years after vasectomy. At scrotal exploration he has rare non-motile sperm in the right vas deferens and an absence of sperm in clear fluid from the left vas deferens. The next step is: 8 A.. B .. C. D .. E .. bilateral vasovasostomy. left varicocelectomy and bilateral vasovasostomy. right vasovasostomy and left vasoepididymostomy. left testis biopsy and intra-operative wet prep evaluation. testicular sperm extraction. Copyright 2017 by The American Urological Association.

question #23 ANSWER=A With sperm in the vas and a patent abdominal vas deferens right vasovasostomy is indicated. For men with clear fluid in the vas deferens the prognosis for return of sperm to the ejaculate is excellent after vasovasostomy alone; therefore left vasovasostomy is also indicated. Epididymal exploration and intra-operative testis biopsy will not provide material information to affect treatment decisions. Varicocelectomy and vasovasostomy should not be performed simultaneously as venous outflow from the testis after varicocele repair is dependent primarily on the vasal vessels that are divided during vasectomy or vasovasostomy and testicular atrophy may result. Goldstein M: Surgical management of male infertility in Wein AJ Kavoussi LR Partin AW Peters CA (eds): CAMPBELL-WALSH UROLOGY ed 11. Philadelphia Elsevier 2015 vol 1 chap 25 p 590. 8 Copyright 2017 by The American Urological Association.

24. A 21-year-old man develops a large dorsal hematoma after a seemingly superficial stiletto knife wound to his penis at the dorsal penoscrotal junction. He is able to void normally after the injury and has no urethral bleeding or gross hematuria. The next step is: A.. pelvic MRI scan. B.. retrograde urethrography. C. urethroscopy. D.. antibiotics and wound closure. E.. exploration.

question #24 ANSWER=E Patients with tangential or superficial wounds clearly away from the urethra and that can void without urethral bleeding or hematuria do not require a retrograde urethrogram. However these patients should be explored except those with clearly superficial injuries. Patients with stab wounds usually can be expected to have preservation of potency. While most surgeons recommend retrograde urethrography in all patients with penetrating penile trauma experience in the literature suggests that few truly occult urethral injuries occur in these patients. In patients with low velocity injuries only those with blood at the meatus hematuria difficulty voiding or injury near the urethra may require retrograde urethrography. Most patients will require retrograde urethrography to rule-out urethral injury and many will need surgical exploration to rule-out and repair any corporal injury or other cause of bleeding. However select patients such as the one in this patient scenario do not require retrograde urethrography. Some patients with minimal wounds can be treated non-operatively. Pelvic MRI scan is not indicated for penetrating genital injuries but may be helpful in blunt genital trauma. Morey AF Zhao LC: Genital and lower urinary tract trauma in Wein AJ Kavoussi LR Partin AW Peters CA (eds): CAMPBELL-WALSH UROLOGY ed 11. Philadelphia Elsevier 2015 vol 3 chap 101 p 2379. Goldman HB Dmochowski RR Cox CE: Penetrating trauma to the penis: Functional results. J UROL 1996;155:551-553.

25. The initial response of the renal vasculature to complete ureteral obstruction is: A.. preglomerular vasodilatation. B.. postglomerular vasodilatation. C. afferent arteriolar constriction. D.. efferent arteriolar constriction. E.. renal artery vasoconstriction.

question #25 ANSWER=A The initial renal response to complete ureteral obstruction is to increase glomerular perfusion pressure. Postglomerular vasodilation without any change in the preglomerular vessels would result in lower glomerular perfusion pressures not higher. Likewise afferent arteriolar constriction and renal artery vasoconstriction would result in decreased glomerular perfusion pressure. Of the choices listed only preglomerular vasodilation and efferent arteriolar constriction lead to increased glomerular perfusion pressures. Preglomerular vasodilation is the first response in both unilateral and bilateral ureteral obstruction. Efferent arteriolar constriction does occur as a second phase in bilateral ureteral obstruction but does not occur in unilateral obstruction. Meldrum KK: Pathophysiology of urinary tract obstruction in Wein AJ Kavoussi LR Partin AW Peters CA (eds): CAMPBELL-WALSH UROLOGY ed 11. Philadelphia Elsevier 2015 vol 2 chap 48 pp 1094-1095.

26. A 54-year-old woman undergoes a continent cutaneous urinary diversion two years after pelvic radiation for cervical cancer. Four months later she has right lower quadrant pain and fecaluria. A pouchogram reveals contrast extending into the colon adjacent to the pouch. The next step is: A.. hyperalimentation. B.. bilateral nephrostomy drainage. C. pouch endoscopy and fulguration of fistula. D.. catheter drainage and low residue diet. E.. colonoscopy.

question #26 ANSWER=D Entero-pouch fistulas have been reported after ileal and right colon urinary diversion. The diagnosis should be suspected in patients who present with gastrointestinal symptoms and metabolic acidosis. These fistulas are most common after pelvic irradiation. Conservative therapy can be effective with low residue diet and continuous pouch drainage. Further diagnostic evaluation with colonoscopy or pouch endoscopy is of little value and biopsy or fulguration may enlarge the fistula. Bilateral nephrostomy drainage alone will not achieve maximal drainage of the pouch. Open surgical exploration may be required if this regimen fails. Hyperalimentation Copyright 2017 by The American Urological Association. 9 alone without catheter drainage is insufficient to resolve the fistula. Badlani GH De Ridder DJMK Mettu JR Rovner ES: Urinary tract fistulae in Wein AJ Kavoussi LR Partin AW Peters CA (eds): CAMPBELL-WALSH UROLOGY ed 11. Philadelphia Elsevier 2015 vol 3 chap 89 pp 2131-2132.

27. A 55-year-old man with bladder cancer undergoes a radical cystectomy. He is averse to an incontinent diversion. Intra-operative frozen-section reveals negative lymph nodes but invasive urothelial carcinoma at the prostatic apical margin. The next step is: A.. ileal neobladder. B.. ileal neobladder and adjuvant pelvic radiotherapy. C. ileal neobladder and adjuvant chemotherapy. D.. ileal conduit. E.. continent cutaneous urinary diversion.

question #27 ANSWER=E The presence of invasive urothelial carcinoma of the prostate carries a high risk of urethral recurrence and is a contraindication to orthotopic bladder replacement. All patients undergoing cystectomy should be counseled about the possibility that intra-operative findings might change the planned form of urinary diversion and all of the alternatives should be discussed prior to surgery. Of the choices listed the continent cutaneous urinary diversion is the best option for a patient who is strongly averse to an external appliance and has agreed to the concept of CIC. Skinner EC Daneshmand S: Orthotopic urinary diversion in Wein AJ Kavoussi LR Partin AW Peters CA (eds): CAMPBELL-WALSH UROLOGY ed 11. Philadelphia Elsevier 2015 vol 3 chap 99 pp 2347-2349.

28. The vertebral level at which the conus medullaris in the neonate terminates is: A.. L1. B.. L3. C. LS. D.. S1. E.. S3.

question #28 ANSWER=B The conus medullaris of the spinal cord terminates between the second and third lumbar vertebra in the newborn. In the adult the spinal cord usually terminates between the first and second lumbar vertebra. Understanding this relationship is critical to be able to diagnose a tethered cord. Cord tethering is often assumed to be present when the conus is below the L2 interspace with termination below L3 resulting in an absolute diagnosis. It is important to note that imaging features support rather than make the diagnosis. The clinical diagnosis of a tethered cord is based on the radiologic findings of tethering along with the clinical findings of "neurological and musculoskeletal signs and symptoms." Clinical findings that help support the diagnosis of a tethered cord are foot deformities leg weakness or pain gait abnormalities lower back pain scoliosis and fecal or urinary incontinence. From a urologic standpoint urinary incontinence or symptomatic voiding difficulties will be present in up to 50% of patients with a tethered cord and urodynamic abnormalities will be found in approximately 70% of patients. Maclellan DL Bauer SB: Neuromuscular dysfunction of the lower urinary tract in children in Wein AJ Kavoussi LR Partin AW Peters CA (eds): CAMPBELL-WALSH UROLOGY ed 11. Philadelphia Elsevier 2015 vol 4 chap 142 p 3285.

29. A 25-year-old man v. ith negative tumor markers underwent a left radical orchiectomy with path ' v · re and immature teratoma. Chest CT scan is negative. CT scan images of the abdomen and pelvis are shown. The next step is: 10 A.. B .. C. D .. E .. PET CT scan. percutaneous biopsy of the retroperitoneal mass. three cycles of EP. three cycles of BEP. RPLND.

question #29 ANSWER=D The patient has bulky retroperitoneal masses from NSGCT. Although the primary tumor was teratoma there was a component of immature teratoma which would classify the patient as a NSGCT. Despite teratoma only this patient should be treated like metastatic NSGCT and the correct treatment is three cycles of BEP. The patient has "good risk" NSGCT defined as primary testicular or retroperitoneal disease no pulmonary metastases and negative or low tumor markers. For "good risk" patients three cycles of BEP are equivalent to four cycles of EP. If the masses remain after chemotherapy which is likely in patients with teratoma he will require an 10 Copyright 2017 by The American Urological Association. extensive post-chemotherapy RPLND. A PET CT is not necessary as the patient needs chemotherapy regardless of the result. A biopsy is not necessary for the same reason. Stephenson AJ Gilligan TD: Neoplasms of the testis in Wein AJ Kavoussi LR Partin AW Peters CA (eds): CAMPBELL-WALSH UROLOGY ed 11. Philadelphia Elsevier 2015 vol 1 chap 34 pp 788- 789.

31. The prevalence of catheter-associated UTls can be reduced by: A.. prophylactic oral antibiotics. B.. routine meatal cleansing. C. antibiotic irrigation of the bladder. D.. maintenance of a closed drainage system. E.. hydrogen peroxide instillation into the drainage bag.

question #31 ANSWER=D The daily risk of acquisition of bacteriuria when an indwelling catheter in-situ is three to seven percent. The rate of bacterial acquisition is higher for women and older persons. Heath care surveys in the USA report that UTls are the fourth most common infection accounting for 13% of health care infections; two-thirds of UTls are directly related to the presence of an indwelling urinary catheter. Catheter-associated UTls result in increased morbidity and mortality among hospitalized patients. Factors proven to reduce catheter-associated UTls include: a closed drainage system early catheter removal and an aseptic insertion technique. Prophylactic antibiotics (systemic or topical) have not been shown to reduce the risk of CAUTI and indeed some studies have revealed their use increased the presence bacterial resistance and candiduria. Routine meatal cleansing intravesical antibiotic irrigation or hydrogen peroxide instillations into the drainage bag have not been demonstrated to reduce the frequency of catheterassociated infections. Hooton TM Bradley SF Cardenas DD et al: Diagnosis prevention and treatment of catheterassociated urinary tract infection in adults: 2009 International Clinical Practice Guidelines from the Infectious Disease Society of America. CUN INFECT DIS 2010;50:625-663. Averch TD Stoffel J Goldman HB et al: AUA White Paper on catheter-associated urinary tract infections: Definitions and significance in the urological patient. UROL PRACTICE 2015;2:321-328.

32. Transection of the dorsal nerve roots at S2-S4 results in: A.. urinary incontinence. B.. detrusor sphincter dyssynergia. C. loss of psychogenic erections. D.. anejaculation. E.. decreased penile sensation.

question #32 ANSWER=E The pudenda! nerve arises from the dorsal nerve roots at S2 S3 and S4. The pudenda! nerve provides innervation of the striated external sphincter; transection would cause sphincter weakness. It also gives arise to the dorsal penile nerve which provides somatic sensation to the penis. Interruption of the pudenda! nerve will cause decreased penile sensation but not affect psychogenic erections. This may be clinically applicable to those patients who undergo dorsal Copyright 2017 by The American Urological Association. 11 rhizotomy. The sympathetic chain arising from T10 to L2 is responsible for ejaculation and will not be impacted by transection of the sacral dorsal nerve roots. Vasavada SP Rackley RR: Electrical stimulation and neuromodulation in storage and emptying failure in Wein AJ Kavoussi LR Partin AW Peters CA (eds): CAMPBELL-WALSH UROLOGY ed 11. Philadelphia Elsevier 2015 vol 3 chap 81 p 1909.

34. A 35-year-old man with Hodgkin's disease has not voided for 18 hours. He is being treated with abdominal XRT and chemotherapy. The most likely cause of anuria is: 12 A.. B .. C. D .. E .. bilateral ureteral obstruction from retroperitoneal lymphoma. radiation enteritis with dehydration. acute tubular necrosis. renal tubular obstruction with uric acid crystals. acute radiation nephritis.

question #34 ANSWER=D Hyperuricemia can be seen during the initial treatment of acute leukemias and lymphomas in response to either chemotherapy or radiotherapy. The rapid destruction and cellular lysis of neoplastic cells results in a rapid rise in uric acid levels. Elevated urinary uric acid crystals will in the presence of acid urine precipitate within the distal convoluted tubules leading to intrarenal obstruction and renal failure. Prophylaxis (and treatment) is accomplished by a combination of alkalinization allopurinol and hydration. Goldfarb DA Poggio ED Demirjian S: Etiology pathogenesis and management of renal failure in Wein AJ Kavoussi LR Partin AW Peters CA (eds): CAMPBELL-WALSH UROLOGY ed 11. Philadelphia Elsevier 2015 vol 2 chap 46 pp 1045-1046.

35. A one-day-old boy has a history of severe prenatal bilat ureteronephrosis and oligohydramnios diagnosed at 19 weeks of gestation. Postnatal ultrasound confirms bile· ' oureteronephrosis and his VCUG is shown. The most common cause of neonatal mortality is: A.. urosepsis. B.. acute renal failure. C ' 1 nary hypoplasia. D.. urinary ascites. E.. congenital cardiac disease. Copyright 2017 by The American Urological Association. 13

question #35 ANSWER=C The VCUG demonstrates the presence of posterior urethral valves and vesicoureteral reflux. The majority of neonates with the coexisting findings of posterior urethral valves and oligohydramnios prior to 20 weeks gestation will be found to have pulmonary hypoplasia and neonatal respiratory distress. The presence of pulmonary hypoplasia in these infants still accounts for the majority of neonatal deaths in boys with posterior urethral valves. Urosepsis can occur but usually not in the neonatal period with early diagnosis and initiation of appropriate prophylactic antibiotics. Acute renal failure can present in the first week of life but can be managed with neonatal peritoneal dialysis to avoid immediate renal induced mortality. Urinary ascites is common with high-grade urethral obstruction but is usually protective for the kidneys and is almost always successfully managed with bladder drainage and broad spectrum antibiotics. Posterior urethral valves are not associated with lethal congenital cardiac disease. 12 Copyright 2017 by The American Urological Association. Shukla AR: Posterior urethral valves and urethral anomalies in Wein AJ Kavoussi LR Partin AW Peters CA (eds): CAMPBELL-WALSH UROLOGY ed 11. Philadelphia Elsevier 2015 vol 4 chap 141 p 3259.

36. The VHL tumor suppressor gene regulates the expression of: A.. basic fibroblastic growth factor. B.. epidermal growth factor receptor. C. c-Met proto-oncogene. D.. VEGF. E.. transforming growth factor beta.

question #36 ANSWER=D The wild type VHL tumor suppressor gene product suppresses the expression of VEGF a potent stimulator of angiogenesis through down-regulation of hypoxia-inducible factor 1 (HIF1). Mutation or loss of the VHL tumor suppressor gene leads to dysregulated expression of VEGF which contributes to the neovascularity associated with RCC. This pathway is of critical importance to practicing urologists as most recently developed tyrosine kinase inhibitors target the pathway directly or indirectly. All of the other listed genes are not directly regulated by HIF1 and therefore are not directly affected by VHL loss. Walther MM Enquist EG Jennings SB et al: Molecular genetics of renal cell carcinoma in Vogelzang NJ Scardino PT Shipley WU Coffey DS (eds): COMPREHENSIVE TEXTBOOK OF GENITOURINARY ONCOLOGY. Baltimore Williams & Wilkins 2000 chap 9 pp 116-128. Neumann HP Zbar B: Renal cysts renal cancer and von Hippel-Lindau disease. KID INTL 1997;51:16-26. Campbell SC Lane BR: Malignant renal tumors in Wein AJ Kavoussi LR Partin AW Peters CA (eds): CAMPBELL-WALSH UROLOGY ed 11. Philadelphia Elsevier 2015 vol 2 chap 57 pp 1321- 1324.

37. A 65-year-old man with rectal carcinoma treated by abdominal perinea! resection develops urinary incontinence two years later. His urinalysis is normal and PVR is 300 ml. Renal ultrasound demonstrates moderate bilateral hydronephrosis. The most likely urodynamic findings are: A.. detrusor overactivity with bladder outlet obstruction. B.. detrusor overactivity with detrusor external sphincter dyssynergia. C. detrusor areflexia with normal compliance. D.. detrusor areflexia with reduced compliance. E.. impaired bladder contractility with intrinsic sphincter deficiency.

question #37 ANSWER=D Permanent lower urinary tract dysfunction occurs in 15-20% of patients following radical pelvic surgery. The typical pattern is one of detrusor areflexia or hypocontractility in the presence of fixed residual striated sphincter tone. This fixed tone represents a functional obstruction that frequently results in decreased detrusor compliance. Although poor proximal sphincter function can also occur (intrinsic sphincter deficiency) this is often masked by prostate bulk in male patients. Wein AJ Dmochowski RR: Neuromuscular dysfunction of the lower urinary tract in Wein AJ Kavoussi LR Partin AW Peters CA (eds): CAMPBELL-WALSH UROLOGY ed 11. Philadelphia Elsevier 2015 vol 3 chap 75 pp 1781-1782.

39. A SO-year-old man is scheduled for a living-related renal transplant. He has a serum creatinine of 5.5 mg/dl and is not yet on dialysis. His non-contrast CT scan shows a 2 cm solid left renal mass. The next step is: A.. repeat CT scan with I.V. contrast. B.. radical nephrectomy and exclude patient from transplantation. C. simultaneous radical nephrectomy and renal transplantation. D.. radical nephrectomy transplant in two years if no recurrence. E.. partial nephrectomy transplant in two years if no recurrence.

question #39 ANSWER=( Incidentally discovered small asymptomatic renal tumors do not mandate a waiting period prior to transplantation. Repeating the CT scan with contrast risks further nephrotoxic injury with preexisting borderline renal function and will not change the management of the renal mass. Although partial nephrectomy may carry the advantage of preserving additional renal mass this is not applicable to this patient. The appropriate management in this setting is simultaneous nephrectomy and transplantation. Gritsch HA Blumberg JM: Renal transplantation in Wein AJ Kavoussi LR Partin AW Peters CA (eds): CAMPBELL-WALSH UROLOGY ed 11. Philadelphia Elsevier 2015 vol 2 chap 47 pp 1072- 1075.

40. Cranberry juice may help prevent UTI by reducing: 14 A.. B .. C. D .. E .. urine pH. urine osmolality. secretory lgA. bacterial adhesion. interleukin 6 {IL-6).

question #40 ANSWER=D A variety of host defense and bacterial virulence factors contribute to the pathogenesis of UTls. Host defenses include high urine osmolality low pH high urea efficient micturition and a number of urine inhibitors of bacterial adherence (e.g. Tamm-Horsfall protein lactoferrin oligosaccharides and mucopolysaccharides). Immune responses to UTls affect hormonal immunity (secretory lgA) as well as cytokine production (IL6 IL8). Cranberry juice contains substances that inhibit the adherence of uropathogenic bacteria to uroepithelial cells. Cranberry ingestion does not have a substantive effect on urine pH urine osmolality secretory lgA or interleukin levels. Schaeffer AJ Matulewicz RS Klumpp DJ: Infections of the urinary tract in Wein AJ Kavoussi LR Partin AW Peters CA (eds): CAMPBELL-WALSH UROLOGY ed 11. Philadelphia Elsevier 2015 vol 1 chap 12 p 274. Jepson RG Williams G Craig JC: Cranberries for preventing urinary tract infections. Cochrane Database of Systematic Reviews 2012 issue 10. Art. No.: CD001321. DOI: 10.1002/14651858.CD001321 .pubs. http://www.cochrane.org/CD001321/RENAL_cranberries-for-preventing-urinary-tract-infections 14 Copyright 2017 by The American Urological Association.

41. A neonate with a 3 cm phallus and non-palpable gonads can be confirmed to have at least one testicle by: A.. 46 XY karyotype. B.. elevated 17-hydroxyprogesterone. C. normal lH FSH levels. D.. normal Mullerian inhibiting substance concentration. E.. increased urinary ketosteroids.

question #41 ANSWER=D This neonate with a normal-sized phallus for a male(> 2.5 cm) could be a female with elevated testosterone due to congenital adrenal hyperplasia. Therefore an elevated testosterone does not equate with the presence of a testicle. Levels in newborns of LH FSH and testosterone can be normal or elevated with many intersex disorders and does not confirm the presence of a testicle. Elevated 17-hydroxyprogesterone and increased urinary ketosteroids would be findings of adrenal insufficiency but would not confirm the presence of a testicle. An hCG stimulation test with an increase in testosterone may be of value but the increase should be > 20 fold. In addition the infant may already have an excessively elevated testosterone level which could mask the findings of an hCG stimulation test. A 46 XY karyotype does not confirm the presence of testes. Mullerian-inhibiting substance is secreted by testicular Sertoli cells and is the one test which would diagnostically confirm the presence of at least one testicle. Kolon TF Herndon CDA Baker LA et al: EVALUATION AND TREATMENT OF CYPTORCHIDISM: AUA GUIDELINE. American Urological Association Education and Research Inc 2014. p 12-13. http://www.auanet.org/education/guidelines/cryptorchidism.cfm Barthold JS Hagerty JA: Etiology diagnosis and management of the undescended testis in Wein AJ Kavoussi LR Partin AW Peters CA (eds): CAMPBELL-WALSH UROLOGY ed 11. Philadelphia Elsevier 2015 vol 4 chap 148 pp 3439-3440.

42. A 78-year-old man had a radical cystectomy and ileal conduit for recurrent bladder cancer. Pathology showed stage pT3bN0M0 cancer. CT scan at one year was normal but at two years there was marked right hydroureteronephrosis with very thin residual renal parenchyma. loopogram shows a tight narrowing of the right distal ureter 2 cm above the ureteroileal junction. He is asymptomatic and serum creatinine is 1.6 mg/dl. The next step is: A.. observation. B.. retrograde balloon dilation of the ureter. C. percutaneous laser incision of the stricture. D.. open reimplantation of the ureter into the ileum. E.. right nephroureterectomy.

question #42 ANSWER=E This is an unusual site for a benign ureteroileal stricture and there is a high likelihood that this is the result of tumor recurrence in the ureter. Therefore observation is not a good option. Endoscopic management whether it be ureteroscopically or percutaneously is unlikely to work and does not establish the etiology of the obstruction. Since the kidney has little remaining parenchyma reimplantation makes little sense and therefore the best treatment is nephroureterectomy. Smith AK Matin SF Jarrett TW: Urothelial tumors of the upper urinary tract and ureter in Wein AJ Kavoussi LR Partin AW Peters CA (eds): CAMPBELL-WALSH UROLOGY ed 11. Philadelphia Elsevier 2015 vol 2 chap 58 pp 1376-1377.

43. In the process of spermatogenesis the final product of meiosis is the: A.. spermatogonia. B.. primary spermatocyte. C. secondary spermatocyte. D.. spermatid. E.. spermatozoa.

question #43 ANSWER=D Primary spermatocytes undergo one round of meiosis creating secondary spermatocytes which are 2N in DNA content and haploid. These subsequently undergo a second round of meiosis to form round spermatids which are 1 N in DNA and haploid which are the final products. The spermatids then eventually metamorphose into mature spermatozoa (spermiogenesis). Turek PJ: Male reproductive physiology in Wein AJ Kavoussi LR Partin AW Peters CA (eds): CAMPBELL-WALSH UROLOGY ed 11. Philadelphia Elsevier 2015 vol 1 chap 22 p 526. Copyright 2017 by The American Urological Association. 15

44. Complications associated with inguinal lymph node dissection for penile cancer are documented to occur most frequently in which of the following settings: A.. palliative indication. B.. prior chemotherapy. C. insulin-dependent diabetes. D.. congestive heart failure. E.. obesity.

question #44 ANSWER=A Complications of inguinal lymph node dissection can include debilitating lower extremity edema wound infection skin flap necrosis DVT hemorrhagic events and sepsis. The greatest risk factor for these complications is palliative indication primarily in patients with advanced disease with impending erosion into the vessels or through the skin. In the series from MD Anderson Cancer Center complication rates (minor and major combined) were 35% for a prophylactic dissection 36% for a therapeutic dissection and 67% for palliative indications. In addition most major complications occurred in the latter or "palliative" group. The reasons for the increased complication rate is presumably due to reduced lymphatic and venous drainage and compromised blood supply. Together these factors affect the viability of skin flaps and lymphatic flow and the majority of the complications are due to infectious causes. While diabetes heart disease and obesity are all important surgical considerations they have not been directly associated with complications related to inguinal node dissection. Similarly prior chemotherapy has not been associated directly with increased complications after inguinal node dissection. Pettaway CA Crook JM Pagliaro LC: Tumors of the penis in Wein AJ Kavoussi LR Partin AW Peters CA (eds): CAMPBELL-WALSH UROLOGY ed 11. Philadelphia Elsevier 2015 vol 1 chap 37 pp 859-860.

45. An 80-year-old man has urinary retention. He has bilateral pitting edema an elevated jugular venous pulse and a blood pressure of 200/120 mmHg. His creatinine is 4.0 mg/dl. The serum K+ and Na+ are normal. An ultrasound shows a very distended bladder and bilateral pelvicaliectasis. Three liters of urine is obtained from his bladder when he is catheterized. Urine output over the next two hours is 700 ml. The next step is: A.. serial creatinine measurement. B.. replace output ml per ml with D5 1/2 NS. C. monitor fluid intake and output every four hours. D.. monitor postural blood pressure for two hours. E.. spot check urine for osmolality sodium and potassium. Copyright 2017 by The American Urological Association. 15

question #45 ANSWER=E All patients with an output > 200 mUhr have post-obstructive diuresis and should be closely monitored. High risk patients with chronic obstruction edema congestive heart failure hypertension weight gain and azotemia are most likely to exhibit a post-obstructive diuresis after the release of obstruction. In the high risk patient a spot check urine for osmolality sodium and potassium will allow for the determination of the type of post-obstructive diuresis and will provide guidance for further management. High risk patients should have vital signs including postural blood pressure and output measured hourly. D5 1/2 NS is an appropriate replacement fluid in the patient with an elevated BUN and creatinine but generally replacement is given at half of the previous hour's urine output. Meldrum KK: Pathophysiology of urinary tract obstruction in Wein AJ Kavoussi LR Partin AW Peters CA (eds): CAMPBELL-WALSH UROLOGY ed 11. Philadelphia Elsevier 2015 vol 2 chap 48 p 1102.

46. A 15-year-old boy is involved in eed MVC. His vital signs are stable. 16 Urinalysis reveals 25-50 RBC/hpf. An abdominal CT scan with I.V. contrast is shown. The next step is: A.. B .. C. D .. E .. observation. delayed CT imaging. arteriography. retrograde pyelogram. immediate renal exploration. Copyright 2017 by The American Urological Association.

question #46 ANSWER=B Abdominal CT studies with contrast offer a fast and all-inclusive evaluation of the abdomen in cases of trauma. With the advent of rapid image acquisition the abdominal renal vasculature and the renal cortex will enhance. However delayed images of the abdomen will usually be needed to see contrast in the distal ureter. If the initial CT cuts reveal a severe renal fracture perinephric hematoma or perinephric fluid collection and especially if medial extravasation of contrast is found delayed films are usually necessary to assess for the presence of contrast in the distal ureter. The presence of a UPJ disruption should be considered when there is absence of contrast in the ipsilateral distal ureter on a delayed CT study. CT cuts to assess for this finding are ideally taken 10-20 minutes post-contrast infusion. If delayed CT cuts reveal no contrast in the distal ureter the next step is emergent surgery with a retrograde pyelogram to confirm the presence of the UP J injury and subsequently surgical repair. If the UP J and ureter are intact the patient may be managed in a non-operative fashion. Arteriography is not indicated as vital signs are stable. Matsumoto JS LeRoy AJ: Pediatric imaging in Belman AB King LR Kramer SA (eds): CLINICAL PEDIATRIC UROLOGY ed 4. Martin Duntz Florence KY 2002 pp 125-126. Husmann DA: Pediatric genitourinary trauma in Wein AJ Kavoussi LR Partin AW Peters CA (eds): CAMPBELL-WALSH UROLOGY ed 11. Philadelphia Elsevier 2015 vol 4 chap 154 p 3539.

47. Stent placement after uncomplicated ureteroscopic stone extraction for a 5 mm distal ureteral calculus: A.. is indicated if intracorporeal lithotripsy is performed. B.. improves stone-free rate. C increases post-procedure pain. D.. is indicated if balloon dilation was performed. E.. reduces the likelihood of ureteral strictures.

question #47 ANSWER=( Several randomized trials have revealed that ureteral stents are not required after uncomplicated ureteroscopic extraction of distal ureteral stones even after balloon dilation of the ureter or intracorporeal lithotripsy. Ureteral strictures are uncommon after ureteroscopy for distal stones whether or not a stent is inserted. Stents do not impact stone free rate but do increase post-procedure pain urinary symptoms and narcotic use. Matlaga BR Krambeck AE Lingeman JE: Surgical management of upper urinary tract calculi in Wein AJ Kavoussi LR Partin AW Peters CA (eds): CAMPBELL-WALSH UROLOGY ed 11. Philadelphia Elsevier 2015 vol 2 chap 54 p 1283.

76. The most common site of sympathetic nerve injury during a RPLND is the: A.. aortorenal ganglion adjacent to the renal hilum. B.. sympathetic chain inferior to the renal artery. C. aortic plexus posterolateral to the aorta. D.. inferior mesenteric plexus adjacent to the inferior mesenteric artery. E.. hypogastric plexus anterior to the aortic bifurcation.

question #76 ANSWER=E Nerve-sparing RPLND can be performed for stage 1 disease. Normally the post-ganglionic sympathetic fibers are identified below the renal vessels and are dissected out of the lymphatic tissue during a nerve-sparing RPLND. Regardless of the side of the dissection great care is taken during dissection over the aortic bifurcation as this is the site where the hypogastric plexus crosses anterior to the great vessels. This is the area where the sympathetic nerves are most vulnerable to injury and this is why the aortic bifurcation is not included in the modified RPLND template. Rice KR Cary CK Masterson TA Foster RS: Surgery of testicular tumors in Wein AJ Kavoussi LR Partin AW Peters CA (eds): CAMPBELL-WALSH UROLOGY ed 11. Philadelphia Elsevier 2015 vol 1 chap 35 p 821.

48. A 27-year-old woman is prescribed a ten day course of an oral quinolone for a pansensitive E. coli UTI. Four days later she develops a low-grade fever to 38° C and a skin rash. Urinalysis shows 1 + protein with WBC casts occasional eosinophils and 5- 10 RBOhpf. Urine gram stain is negative for bacteria. Serum creatinine is 1.8 mg/dl. The next step is discontinuation of quinolone antibiotics and: A.. observation. B.. change to ampicillin. C. change to cephalosporin. D.. oral antihistamines. E.. prednisone.

question #48 ANSWER=A The most likely diagnosis is acute interstitial nephritis. The best treatment is to discontinue the offending drug treat any related hypertension that may be present and limit protein intake. The vast majority of patients will have symptoms which spontaneously resolve. If symptoms persist renal biopsy may be necessary to confirm the diagnosis. Both ampicillin and cephalosporins may cause interstitial nephritis and could actually be harmful if used in this setting. Indeed adequate treatment for a U TI can be achieved in a single dose of medication and additional antibiotics are not indicated in this patient with a sterile gram stain at this time. The use of prednisone should be reserved following confirmation of the diagnosis with a renal biopsy. There is no role for the use of antihistamines in the treatment of interstitial nephritis. Goldfarb DA Poggio ED Demirjian S: Etiology pathogenesis and management of renal failure in Wein AJ Kavoussi LR Partin AW Peters CA (eds): CAMPBELL-WALSH UROLOGY ed 11. Philadelphia Elsevier 2015 vol 2 chap 46 pp 1043-1044.

49. A 28-year-old paraplegic man had a sphincterotomy seven years ago and wears a condom catheter. During an evaluation for renal insufficiency renal ultrasound reveals bilateral hydroureteronephrosis. The parameter or study most predictive of this complication is: A.. EMG. B.. CMG. C. Valsalva LPP. D.. detrusor LPP. E.. urethral pressure profilometry.

question #49 ANSWER=D Detrusor LPP is the most reliable urodynamic parameter to predict the risk of upper tract deterioration after sphincterotomy. A detrusor LPP of higher than 40 cm H2O indicates that the sphincterotomy has failed and may serve as a guide to determine whether a repeat sphincterotomy is necessary. Abnormal compliance which may be detected on CMG may also be a worrisome finding but there is much less established predictive value. To date there is no correlation of urethral function tests (urethral pressure profile Valsalva LPP EMG) to upper tract deterioration. Copyright 2017 by The American Urological Association. 17 Nitti VW Brucker BM: Urodynamic and video-urodynamic evaluation of the lower urinary tract in Wein AJ Kavoussi LR Partin AW Peters CA (eds): CAMPBELL-WALSH UROLOGY ed 11. Philadelphia Elsevier 2015 vol 3 chap 73 p 1727.

50. Stage 3 prolapse in the Pelvic Organ Prolapse Quantification (POPQ) system occurs when the most distal portion of the prolapse is: A.. 1 cm or less proximal or distal to the hymenal plane. B.. 1 cm or less proximal or distal to the introitus. C. > 1 cm distal to the hymen; entire vagina has not prolapsed. D.. > 1 cm distal to the introitus; entire vagina has not prolapsed. E.. associated with complete vaginal eversion.

question #50 ANSWER=C The International Continence Society has established a standardized system to quantify pelvic organ prolapse. This classification is known as the POPQ system an acronym for pelvic organ prolapse quantification. The system uses the hymenal ring as its central identification point. The hymen was chosen over the vaginal introitus because it can be more precisely located within the vaginal vault; all measurements are based from this location. This classification avoids use of the terms cystocele or rectocele recognizing that the actual organ prolapsing may be unable to be determined by a physical examination. The examination to determine POPQ stage is performed in a dorsal lithotomy position with the patient straining. The POPQ staging system has excellent inter-observer and intra-observer reliability and has become the standard for reporting outcomes following prolapse repair. The staging system is however not perfect and can be significantly affected by patient positioning with the degree of the prolapse being more severe if the patient is examined with the head of the table raised to 45 degrees or higher. In addition it fails to assess for unilateral or asymmetric defects. The POPQ staging system is defined as: Stage O - no prolapse Stage 1 - the most distal portion of the prolapse is more than 1 cm above the hymen Stage 2 - the most distal portion of the prolapse is +/-1 cm above or below the hymen Stage 3 - the most distal portion of the prolapse protrudes> 1 cm below the hymen and the total vagina has not prolapsed and Stage 4 - the entire vagina everts (i.e. complete prolapse). Kobashi KC: Evaluation and management of women with urinary incontinence and pelvic prolapse in Wein AJ Kavoussi LR Partin AW Peters CA (eds): CAMPBELL-WALSH UROLOGY ed 11. Philadelphia Elsevier 2015 vol 3 chap 71 pp 1698-1999.

51. A 64-year-old man undergoes a six core biopsy for a PSA of 5.6 ng/ml. Pathology is a Gleason 6 (3+3) prostate cancer in a single core involving less than 10% of the tissue. The other cores are normal. He prefers active surveillance. The next step is: 18 A.. B .. C. D .. E .. CT scan. initiate finasteride. repeat prostate biopsy with 12 or more cores. check PSA quarterly and repeat biopsy in one year. counsel patient that he is not appropriate for active surveillance.

question #51 ANSWER=C Active surveillance is a reasonable option for patients with low-risk prostate cancer. This patient's risk profile makes him a reasonable candidate for this approach. However a six-core biopsy is likely inadequate tissue sampling to truly identify indolent disease. Therefore initiation of active surveillance protocol with quarterly PSA and repeat biopsy in one year is premature and immediate systematic prostate rebiopsy is the next step. Additional imaging with either bone scan or pelvic CT scan is unnecessary in low-risk patients and would be inappropriate in this setting. There is no data supporting the use of finasteride in the management of prostate cancer. Carter HB Dall'Era MA: Active surveillance of prostate cancer in Wein AJ Kavoussi LR Partin AW Peters CA (eds): CAMPBELL-WALSH UROLOGY ed 11. Philadelphia Elsevier 2015 vol 3 chap 113 p 2634. '

52. Randall's plaques are composed of: A.. calcium oxalate. B.. calcium apatite. C. brushite. D.. hydroxyproline. E.. cholesterol.

question #52 ANSWER=B Large amounts of Randall's plaque are unique to idiopathic calcium oxalate stone formers and are invariably composed of calcium apatite crystals. Using papillary biopsies obtained during the 18 Copyright 2017 by The American Urological Association. time of PCNL Randall's plaque were found to initially form on the basement membrane of the thin limbs of the loops of Henle and grow by the continued deposition of calcium apatite and organic matrix. With growth the plaque will spread through the interstitium and eventually penetrate the urothelium exposing the plaque to urine where it will serve as an anchor for urinary solutes. Although Randall's plaque can be found in other stone formers Randall's plaque has been found to be a prerequisite for kidney stone formation in idiopathic calcium oxalate stone formers. Pearle MS Antonelli JA Lotan Y: Urinary lithiasis: Etiology epidemiology and pathogenesis in Wein AJ Kavoussi LR Partin AW Peters CA (eds): CAMPBELL-WALSH UROLOGY ed 11. Philadelphia Elsevier 2015 vol 2 chap 51 p 1176.

53. The C-arm fluoroscopic operational factor resulting in an increased radiation dose to both the patient and operating room personnel is: A.. increasing tube kilovoltage (kVp). B.. increasing tube current (mA). C. decreasing image intensifier to skin distance. D.. removing the image intensifier grid. E.. increasing the X-ray tube (source) to skin distance.

question #53 ANSWER=B The use of fluoroscopic imaging in urological surgery requires a basic knowledge of radiation protection principles so that the dose to the patient physician and ancillary staff can be minimized. It is important to remember that with an increase in patient size the dose rate will be greater and accumulate faster. In terms of manipulating the operational factors in fluoroscopy there is generally a trade-off in terms of image quality and radiation dose. Increasing the tube current results in greater image quality and increased dose to the patient and staff. Increasing the tube kilovoltage diminishes image quality (less contrast) but is usually associated with less radiation dose if the tube current is appropriately reduced. Decreasing the image intensifier to skin distance usually increases image quality depending on focal spot size and decreases the dose to the patient without significantly changing the dose to staff. Removing the grid decreases image quality as well as the radiation dose to patient and staff. Increasing the source to skin distance usually improves image quality and decreases the dose to the patient without significantly changing the dose to staff. Geise RA Morin RL: Radiation management in uroradiology in Pollack HM Mcclennan BL (eds): CLINICAL UROGRAPHY ed 2. Philadelphia WB Saunders Co 2000 vol 1 chap 3 pp 13-18. Bishoff JT Rastinehad AR: Urinary tract imaging: Basic principles of computed tomography magnetic resonance imaging and plain film in Wein AJ Kavoussi LR Partin AW Peters CA (eds): CAMPBELL-WALSH UROLOGY ed 11. Philadelphia Elsevier 2015 vol 1 chap 2 p 27.

54. Parathyroid hormone level is suppressed in a patient with: A.. obesity. B.. a recent renal transplant. C. renal calcium leak. D.. absorptive hypercalciuria. E.. renal insufficiency.

question #54 ANSWER=D Parathyroid levels are suppressed in patients with absorptive hypercalciuria as a result of transient elevation of serum calcium due to increased intestinal calcium absorption. The other conditions are associated with secondary elevation of PTH due to PTH-resistance (i.e. obesity African American) renal calcium loss and/or elevated serum phosphorus. Pearle MS Antonelli JA Lotan Y: Urinary lithiasis: Etiology epidemiology and pathogenesis in Wein AJ Kavoussi LR Partin AW Peters CA (eds): CAMPBELL-WALSH UROLOGY ed 11. Philadelphia Elsevier 2015 vol 2 chap 51 p 1183. Copyright 2017 by The American Urological Association. 19

55. Following TURBT for papillary urothelial carcinoma of the bladder peri-operative instillation of mitomycin C: A.. is unnecessary for small solitary low grade tumors. B.. is most effective in acidic urine. C. should be given within 24 hours of the resection. D.. should be followed by an induction course of intravesical therapy. E.. should be delayed for 24 hours if an extraperitoneal perforation occurs.

question #55 ANSWER=C A meta-analysis of seven randomized trials comprising nearly 1 500 patients with Ta-T1 bladder cancer with a median follow-up of 3.4 years demonstrated that one immediate post-TURBT instillation of intravesical mitomycin C resulted in a 40% reduction in tumor recurrence. Patients undergoing resection of single or multiple tumors benefited and benefit was not affected by tumor size. The timing of instillation however appears to be critical. In all studies documenting efficacy the instillation was given within the first 24 hours post-TURBT. One study has demonstrated that if the instillation is given 24 hours after tumor resection the risk of tumor recurrence increased two-fold. Peri-operative instillation is contraindicated in the setting of overt or suspected extra or intraperitoneal perforation or concurrent dilation of a urethral stricture or urethral injury as severe complications (e.g. chronic pain bladder necrosis necrosis of adjacent soft tissue and necrosis of either the corporal spongiosus or cavernosum) have all been reported in these settings. lntravesical mitomycin C is most effective in alkaline urine. Jones JS: Non-muscle-invasive bladder cancer (Ta T1 and CIS) in Wein AJ Kavoussi LR Partin AW Peters CA (eds): CAMPBELL-WALSH UROLOGY ed 11. Philadelphia Elsevier 2015 vol 3 chap 93 pp 2215-2216. Sylvester R Oosterlinck W van der Meijden A: A single immediate postoperative instillation of chemotherapy decreases the risk of recurrence in patients with stage Ta T1 bladder cancer: a meta-analysis of published results of randomized clinical trials. J UROL 2004;171:2186-2190.

56. The signal intensity of prostate cancer on T1 and T2 weighted MRI scan images is: A.. high T1 and high T2. B.. low T1 and high T2. C. high T1 and low T2. D.. low T1 and low T2. E.. intermediate T1 and high T2. Copyright 2017 by The American Urological Association. 19

question #56 ANSWER=D Prostate MRI scan especially with combined endorectal and phase-array coils is used in prostate cancer staging with up to 82% accuracy. The T1 and T2 weighted images are helpful in differentiating between post-biopsy hemorrhage which presents as a high T1 and a low T2 lesion and prostate cancer which presents as a low T1 and low T2 lesion. Barnes AS Tempany CMC: Image-guided minimally invasive therapy in Richie JP D'Amico (eds): UROLOGIC ONCOLOGY. Philadelphia Elsevier Saunders 2005 chap 7 p 115. Bishoff JT Rastinehad AR: Urinary tract imaging: Basic principles of computed tomography magnetic resonance imaging and plain film in Wein AJ Kavoussi LR Partin AW Peters CA (eds): CAMPBELL-WALSH UROLOGY ed 11. Philadelphia Elsevier 2015 vol 1 chap 2 p 56.

57. A 21-year-old woman undergoes resection of a cystic lung mass. ·' ology reveals lymphangioleiomyomatosis (LAM). Abdominal T1 MRI scan is shown. The most likely diagnosis is: 20 A.. B .. C. D .. E .. VHL. Birt-Hogg-Dube. tuberous sclerosis. hereditary papillary RCC. familial leiomyomatosis RCC.

question #57 ANSWER=C The scan reveals bilateral small renal masses. The largest of which is in the left kidney and on this T1-weighted image the high intensity of the lesion indicates fat within the tumor. This is consistent with an angiomyolipoma. This finding in conjunction with pulmonary lymphangioleiomyomatosis (LAM) is indicative of tuberous sclerosis complex (TSC). Familial leiomyomatosis would be associated with skin fibromas and renal carcinoma. VHL does not cause pulmonary LAM. Birt-Hogg-Dube is associated with chromophobe tumors and oncocytomas. It is also associated with lung cysts which can result in spontaneous pneumothorax but not LAM. Hereditary papillary RCC has no common pulmonary manifestations and does not cause ang iomyol i po mas. 20 Copyright 2017 by The American Urological Association. Pope JC IV : Renal dysgenesis and cystic disease of the kidney in Wein AJ Kavoussi LR Partin AW Peters CA (eds): CAMPBELL-WALSH UROLOGY ed 11. Philadelphia Elsevier 2015 vol 4 chap 131 pp 3023-3025.

58. A 23-year-old woman with cystic fibrosis takes nutritional supplements Vitamin C and antibiotic prophylaxis to prevent respiratory infections. She he aluria and recurrent calcium oxalate stones. The most likely cause of her stones is: A.. Vitamin C therapy. B.. reduction of intestinal Oxalobacter formigenes. C ' · ' calcium diet. D.. cystic fibrosis-associated ileal absorption disorder. E.. dietary glycine excess.

question #58 ANSWER=B Chronic antibiotic use may reduce normal levels of Oxalobacter formigenes in the intestine. This anaerobe metabolizes as much as 50% of ingested oxalate. High calcium diets are associated with decreased oxalate absorption. Few cystic fibrosis patients have ileal absorption disorders. Vitamin C and glycine while associated with oxalate metabolism are unlikely to increase urinary levels significantly. An emerging treatment for reduced intestinal Oxalobacter formigenes is probiotics. Pearle MS Antonelli JA Lotan Y: Urinary lithiasis: Etiology epidemiology and pathogenesis in Wein AJ Kavoussi LR Partin AW Peters CA (eds): CAMPBELL-WALSH UROLOGY ed 11. Philadelphia Elsevier 2015 vol 2 chap 51 p 1181.

59. A 45-year-old man with metastatic RCC involving the lung liver lymph nodes and bone undergoes a right radical nephrectomy. His pre-operative labs include a hemoglobin of 9 gm/dl and a calcium of 11.5 mg/dl. The treatment most likely to prolong overall survival is: A.. temsirolimus. B.. interferon alpha. C. bevacizumab. D.. sorafenib. E.. sunitinib.

question #59 ANSWER=A Temsirolimus is a specific inhibitor of the mammalian target of rapamycin (mT OR) kinase which is a component of intracellular signaling pathway involved in growth and proliferation of cells. Level 1 evidence in a recent study comparing temsirolimus to interferon alpha focused on patients with a poor prognosis (e.g. poor risk metastatic RCC patients). Poor risk metastatic RCC patients in this trial had to have at least three of the following poor risk features: LDH > 1.5 times upper level or normal Hgb below normal calcium> 10 mg/dL time from diagnosis of cancer> 12 months metastases spread to multiple organs and/or Karnofsky score= 60 or 70. This patient has multiple features that put him in a poorer risk group including multiple sites of metastasis anemia and hypercalcemia. In patients with poor risk features I.V. weekly infusion of temsirolimus when compared to interferon prolongs overall survival and is the first agent that has demonstrated an overall survival advantage in this category of patients. Sunitinib bevacizumab and sorafenib have not been shown to improve overall survival or prolong survival in patients with multiple poor risk features. Hudes G Carducci M Tomczak P et al: Temsirolimus interferon alfa or both for advanced renal cell carcinoma. NEJM 2007;356:2272-2281. Srinivasan R Linehan WM: Treatment of advanced renal cell carcinoma in Wein AJ Kavoussi LR Partin AW Peters CA (eds): CAMPBELL-WALSH UROLOGY ed 11. Philadelphia Elsevier 2015 vol 2 chap 63 pp 1514-1515.

60. A one-year-old uncircumcised boy with spina bifida is managed with CIC and oxybutynin. Ultrasound and VCUG are normal. He has recurrent asymptomatic episodes of cloudy urine. A recent urinalysis shows 10-20 WBOhpf and a urine culture grows 105 E. coli. The next step is: A.. observation. B.. treat with culture specific antibiotics and start prophylaxis. C. gentamicin bladder irrigations. D.. circumcision. E.. vesicostomy.

question #60 ANSWER=A Individuals with a neurogenic bladder that are being managed with CIC will have bacteriuria 40- 80% of the time. Only symptomatic infections (i.e. pain fever new onset of urinary incontinence or foul smelling cloudy urine lasting longer than three days) should be treated with antibiotics. T he presence of intermittent cloudy urine and/or mild pyuria is not enough to Copyright 2017 by The American Urological Association. 21 warrant antibiotic treatment. Overtreatment of asymptomatic bacteriuria in this patient population will lead to resistant organisms that are difficult to manage. The efficacy of prophylactic antibiotics in the setting of recurrent symptomatic infections in patients on CIC is not entirely clear. Gentamicin bladder irrigations have been shown to be effective in some patients with recurrent symptomatic infections. Changing CIC to sterile technique will likely be ineffective as well as unfeasible. Circumcision can reduce the risk of infection and should be considered if recurrent symptomatic UTls occur. Vesicostomy is not indicated for non-febrile UTls in this patient. Cooper CS Storm DW: Infection and inflammation of the pediatric genitourinary tract in Wein AJ Kavoussi LR Partin AW Peters CA (eds): CAMPBELL-WALSH UROLOGY ed 11. Philadelphia Elsevier 2015 vol 4 chap 127 p 2932. Wyndaele JJ: Complications of intermittent catheterization: Their prevention and treatment. SPINAL CORD 2002;40:536-541.

61. A 40-year-old woman has chronic renal insufficiency and a creatinine of 2.5 mg/dl. The medication that will increase the serum creatinine but not alter the true GFR is: A.. captopril. B.. cephalexin. C. trimethoprim. D.. spironolactone. E.. cisplatin.

question #61 ANSWER=( Endogenous creatinine and creatinine clearance is the most widely used surrogate for GFR. In the presence of normal renal function 90% of creatinine is filtered and 10% is secreted by the proximal tubules. As GFR declines tubular secretion may contribute up to 35% of all creatinine removal at levels of 40-80 mUmin. The commonly utilized antibiotic trimethoprim blocks the tubular secretion of creatinine. Since creatinine is produced at a steady state the serum creatinine will rise but the GFR does not change. Captopril and spironolactone might alter renal perfusion causing a change in the GFR that leads to an altered creatinine level. Cisplatin has a direct nephrotoxic effect but would not raise creatinine without altering GFR. Cephalexin could increase the serum creatinine through an acute drug-induced injury to the kidney such as interstitial nephritis. Goldfarb DA Poggio ED Demirjian S: Etiology pathogenesis and management of renal failure in Wein AJ Kavoussi LR Partin AW Peters CA (eds): CAMPBELL-WALSH UROLOGY ed 11. Philadelphia Elsevier 2015 vol 2 chap 46 p 1060.

62. The most important clinical or pathologic parameter associated with progression of non-muscle-invasive bladder cancer is: 22 A.. B .. C. D .. E .. tumor stage. early recurrence. location. presence of mutant p53. age.

question #62 ANSWER=A Several useful prognostic parameters exist for tumor progression in patients with Ta/Tis/T1 bladder cancer. The most important of these are tumor stage grade and presence of CIS. Tumor size and multiplicity are other factors that may predict progression. Early recurrence is not associated with progression of the disease except in patients with BCG treatment failure. The relationship between p53 status and tumor progression remains unclear. Age and tumor location do not impact progression risk. Jones JS: Non-muscle-invasive bladder cancer (Ta T1 and CIS) in Wein AJ Kavoussi LR Partin AW Peters CA (eds): CAMPBELL-WALSH UROLOGY ed 11. Philadelphia Elsevier 2015 vol 3 chap 93 p 2208. Millan-Rodriguez F Chechile-Toniolo G Salvador-Bayarri J et al: Multivariate analysis of the prognostic factors of primary superficial bladder cancer. J UROL 2000;163:73-78. 22 Copyright 2017 by The American Urological Association.

63. A three-year-old girl has recurrent febrile UTls in spite of prophylactic antibiotics. Her VCUG is shown. The next step is: A.. creatinine clearance. B.. DMSA scan. C. MR urogram. D.. urodynamics. E.. ureteral reimplantation.

question #63 ANSWER=B This child has massive grade 5 reflux into the right kidney. No information is available from the VCUG about the left kidney or the function of the right kidney. The abnormal calyces on the VCUG suggest that the function may be compromised. Urodynamics are often not interpretable in the face of high grade reflux (VUR). A DMSA renal scan is needed to determine function of the right kidney which will help determine whether reconstruction versus nephrectomy is suitable. Ureteral reimplantation is not indicated before assessment of renal function. An MR urogram will also give this information; however the MR urogram will most likely require either a general anesthetic or closely monitored sedation. Creatinine clearance may be abnormal but it will not be possible to tell which kidney has the deficit. Peters CA Skoog SJ Arant BS Jr et al: AUA GUIDELINE FOR MANAGEMENT AND SCREENING OF PRIMARYVESICOURETERAL REFLUX IN CHILDREN. American Urological Association Education and Research Inc 2010. p 5. http://www.auanet.org/education/guidelines/vesicoureteral-reflux-a.cfm

64. A 78-year-old man with dilated cardiomyopathy and obstructive pulmonary disease underwent percutaneous radiofrequency ablation of an enlarging 2.7 cm renal mass. Six months later MRI scan demonstrates persistent contrast-enhancement within the periphery of the tumor. The next step is: A.. MRI scan in six months. B.. renal mass biopsy. C. PET scan. D.. repeat percutaneous ablation. E.. partial nephrectomy.

question #64 ANSWER=D Percutaneous radiofrequency ablation of small renal masses has been offered in recent years as a less invasive method of treatment. Treatment efficacy is generally determined by follow-up CT or MRI scan evaluating for enhancement within the lesion. If enhancement is noted this is considered suggestive of residual or recurrent tumor. In most series between 5-20% of patients treated by radiofrequency ablation require re-treatment within the first year due to persistent enhancement. While long-term data is not available those patients undergoing a second ablative procedure appear to have a similar outcome to those treated effectively in the first ablation. While partial nephrectomy could be considered in this patient the frequency of persistent enhancement after the first ablation suggests that a second ablation is warranted prior to proceeding with more aggressive therapy. Additionally this is an older patient with multiple co-morbid conditions suggesting a less invasive approach is warranted prior to surgical intervention. Biopsy is not indicated in this patient given the fact that the previous ablation may obscure histologic interpretation and that the biopsy outcome would not likely influence the desire to complete treatment. PET scan has poor specificity and would not be informative in this case of localized renal mass. As the enhancement likely represents residual tumor it is not likely to abate with continued observation. Tracy CR Cadeddu JA: Nonsurgical focal therapy for renal tumors in Wein AJ Kavoussi LR Partin AW Peters CA (eds): CAMPBELL-WALSH UROLOGY ed 11. Philadelphia Elsevier 2015 vol 2 chap 62 pp 1489-1490.

65. A 52-year-old woman develops continuous leakage of clear fluid from her vagina six weeks following a laparoscopic transabdominal hysterectomy for benign disease. A CT urogram is unremarkable and cystoscopy reveals a subtrigonal 1 cm vesicovaginal fistula between the posterior wall of the bladder and the mid-vagina. The next step is: A.. placement of urethral catheter and repeat evaluation in six weeks. B.. cystoscopy and fulguration of the fistula. C. immediate transvaginal repair. D.. immediate transabdominal repair. E.. transabdominal repair in three months.

question #65 ANSWER=C This patient has developed an uncomplicated large diameter > 5 mm vesicovaginal fistula following abdominal hysterectomy. Because of its size it is very unlikely to close with prolonged Foley catheterization. Endoscopic treatment with fulguration is an option for management in small diameter fistula< 5 mm and is most successful if the fistula tract is long neck and tortuous. In this patient immediate surgical repair is indicated. The outcomes are not adversely affected by intervening at six weeks. Given the location of the fistula and the transabdominal approach used in the prior hysterectomy a transvaginal approach with interposition of labial or peritoneal Copyright 2017 by The American Urological Association. 23 flaps between the vesical and vaginal tissues at the time of repair would be optimal. Badlani GH De Ridder DJMK Mettu JR Rovner ES: Urinary tract fistulae in Wein AJ Kavoussi LR Partin AW Peters CA (eds): CAMPBELL-WALSH UROLOGY ed 11. Philadelphia Elsevier 2015 vol 3 chap 89 p 2109.

66. A 62-year-old man undergoes a TURBT for a lesion at the bladder dome. Final pathology reveals muscle-invasive small cell carcinoma. Metastatic work-up is negative. The next step is: A.. repeat TURBT. B.. neoadjuvant chemotherapy. C. XRT. D.. partial cystectomy. E.. radical cystoprostatectomy.

question #66 ANSWER=B Small cell carcinoma of the bladder is a relatively rare tumor that may arise in combination with urothelial carcinoma. It is usually biologically aggressive with early vascular and muscular invasion. These malignancies usually respond to but are not cured by chemotherapy regimens. Neither partial or initial radical cystectomy nor intravesical chemotherapy is appropriate in this setting. Radiation or extirpative surgery alone may result in cure rates of 5-20%. However neoadjuvant chemotherapy followed by surgery or radiation therapy results in cure rates of 40- 65%. Therefore the best treatment is chemotherapy followed by local treatment such as surgery or radiation if the patient does not progress. Guzzo TJ Vaughn DJ: Management of metastatic and invasive bladder cancer in Wein AJ Kavoussi LR Partin AW Peters CA (eds): CAMPBELL-WALSH UROLOGY ed 11. Philadelphia Elsevier 2015 vol 3 chap 94 p 2223.

67. A 45-year-old obese man has hypertension new onset diabetes and general weakness. Two 24-hour urine collections show elevated cortisol levels. The next step is: A.. low-dose dexamethasone test. B.. plasma corticotrophin (ACTH). C. high-dose dexamethasone test. D.. metyrapone test. E.. abdominal CT scan.

question #67 ANSWER=B Elevated urinary cortisol levels confirm the diagnosis of Cushing's syndrome but do not provide information about the etiology of the condition. The next step to determine the etiology is to measure a plasma corticotrophin or ACTH level. This will determine if the Cushing's is ACTHdependent or ACTH-independent. If ACTH levels are not elevated then the likely source is adrenal and an abdominal CT scan with attention to the adrenals is appropriate. However it is preferable and more efficient to determine if ACTH levels are elevated as the etiology of the Cushing's is unlikely to be of adrenal origin if ACTH is elevated. The high-dose dexamethasone suppression test and the meta pyrone tests are used in patients with ACTH-dependent Cushing's syndrome to determine if the source of excess ACTH secretion is pituitary or ectopic in nature and are only appropriate if serum corticotrophin levels are elevated. The high-dose dexamethasone suppression test and the metapyrone test have now largely been supplanted by direct measurements of ACTH in the venous plexus downstream from the pituitary gland (inferior petrosal sinus sampling). Kutikov A Crispen PL Uzzo RG: Pathophysiology evaluation and medical management of adrenal disorders in Wein AJ Kavoussi LR Partin AW Peters CA (eds): CAMPBELL-WALSH UROLOGY ed 11. Philadelphia Elsevier 2015 vol 2 chap 65 p 1537.

69. An asymptomatic 75-year-old man with multiple mediical co-morbidities has the incidental finding on CT scan as shown. Serum creatinine is 1.0 mg/dl. The appropriate management of the left kidney is: A.. observation. B.. ureteral placement with interval changes. C. percutaneous nephrostomy with interval changes. D.. laparoscopic pyeloplasty. E.. nephrectomy.

question #69 ANSWER=A The patient has bilateral UPJ obstructions incidentally noted on an abdominal CT scan with the radiographic evaluation revealing little to no renal tissue on the left side. In this asymptomatic patient with advance age and multiple medical co-morbidities observation is most appropriate. Indications for intervention include the development of symptoms calculi or infection. Since there is no renal tissue on the left UPJ repair whether laparoscopic or open will not improve renal function and is inappropriate. If the patient became symptomatic a nephrectomy could be considered. Nakada SY Best SL: Management of upper urinary tract obstruction in Wein AJ Kavoussi LR Partin AW Peters CA (eds): CAMPBELL-WALSH UROLOGY ed 11. Philadelphia Elsevier 2016 vol 2 chap 49 pp 1107-1108.

71. A 53-year-old man with a PSA of 2.7 ng/ml undergoes 12-core TRUS prostate needle biopsy. Pathology reveals focal high-grade PIN and atypical adenomatous hyperplasia (adenosis). The next step is: A.. examine multiple deeper tissue sections of current biopsy. B.. immediate repeat 12-core TRUS biopsy. C. immediate saturation biopsy. D.. repeat PSA in six months. E.. TRUS biopsy in six months.

question #71 ANSWER=D The management of high-grade PIN has changed. With the standard biopsy now including 10 to 12 cores it is no longer considered mandatory for patients to undergo immediate rebiopsy of Copyright 2017 by The American Urological Association. 25 their prostate. However in the setting of accompanying atypical small acinar proliferation (ASAP) immediate rebiopsy and/or additional examination of the original biopsy with deeper sections is usually recommended. In this case however the patient has atypical adenomatous hyperplasia (adenosis) which is felt to be a benign process and therefore does not require immediate rebiopsy. The patient therefore should be treated as if he has isolated high-grade PIN and should have serial PSA monitoring. If the PSA is increased in six months repeat biopsy can be considered. Zynger DL Yang XJ: High grade prostatic intraepithelial neoplasia and other atypical lesions of the prostate. AUA UPDATE SERIES 2008 vol 27 lesson 35 p 341. Epstein JI: Pathology of prostatic neoplasia in Wein AJ Kavoussi LR Partin AW Peters CA (eds): CAMPBELL-WALSH UROLOGY ed 11. Philadelphia Elsevier 2015 vol 3 chap 110 p 2593.

72. A morbidly obese 55-year-old woman undergoes Roux-En-Y bypass. In order to minimize stone risk the best treatment is: A.. aggressive hydration. B.. calcium carbonate. C. hydrochlorothiazide. D.. potassium citrate. E.. allopurinol.

question #72 ANSWER=B Bariatric surgery patients typically develop enteric hyperoxaluria which should be managed with calcium supplementation. Calcium will bind intestinal oxalate thereby reducing absorption of free oxalate and ultimately decreasing urinary oxalate. Increasing fluids will have little effect as these patients are chronically dehydrated and the other treatments do not address the problem. Lipkin ME Ferrandino MN Preminger GM: Evaluation and medical management of urinary lithiasis in Wein AJ Kavoussi LR Partin AW Peters CA (eds): CAMPBELL-WALSH UROLOGY ed 11. Philadelphia Elsevier 2015 vol 2 chap 52 p 1220.

73. A 61-year-old man with Parkinson's disease has urinary frequency urgency urinary incontinence and weak stream. Pressure-flow urodynamics reveal detrusor overactivity a sustained voiding detrusor pressure of 88 cm H2O and a maximum flow of 7 mUsec. Cystometric bladder capacity is 275 ml. PVR is 150 ml. The next step is: A.. antimuscarinic. B.. baclofen. C. alpha-blocker. D.. CIC. E.. laser vaporization of prostate.

question #73 ANSWER=C The urodynamic data documents involuntary detrusor contractions and bladder outlet obstruction most likely due to BPH. The most reasonable pharmacologic approach is to use an alpha-sympathetic blocking agent. Detrusor-external sphincter dyssynergia is not seen in Parkinson's obstruction. Thus baclofen which is intended to induce skeletal muscle relaxation is not indicated. Antimuscarinics may reduce involuntary detrusor contractions but may exacerbate emptying failure so should not be used until his emptying improves. TURP in Parkinson's patients carries with it a risk of urinary incontinence and should be utilized only in patients with definite bladder outlet obstruction due to BPH who have failed more conservative therapy. A trial of alpha-blocker is warranted prior to initiation of CIC or a TURP. Wein AJ Dmochowski RR: Neuromuscular dysfunction of the lower urinary tract in Wein AJ Kavoussi LR Partin AW Peters CA (eds): CAMPBELL-WALSH UROLOGY ed 11. Philadelphia Elsevier 2015 vol 3 chap 75 pp 1767-1768.

74. A 52-year-old woman is G4P4 and reports urinary incontinence with physical activities. On physical exam she is found to leak with cough. Aa point is at -3 and Ap point is at -3 while Ba and Bp points are at -2 on pelvic exam and she can contract her pelvic floor muscles during exam. She has no other medical problems and has not previously sought care for her incontinence. The next step is: A.. urodynamic testing. B.. pelvic floor muscle training. C. biofeedback. D.. mid-urethral sling and cystocele repair. E.. mid-urethral sling and rectocele repair.

question #74 ANSWER=B This patient is found to have stress urinary incontinence on exam. She has no prolapse on POP-Q testing. She has no other medical problems and there is no reason to suspect an abnormal flow or elevated residual (which might be suspected with prolapse or prior pelvic surgery); therefore further testing with urodynamics particularly prior to starting non-invasive therapy is 26 Copyright 2017 by The American Urological Association. unnecessary. Since she can contract her pelvic muscles on exam biofeedback may not be required and she should be started on a program of pelvic floor exercises. Mid-urethral sling may be offered to patients failing conservative management. Newman DK Burgio KL: Conservative management of urinary incontinence: Behavioral and pelvic floor therapy and urethral and pelvic devices in Wein AJ Kavoussi LR Partin AW Peters CA (eds): CAMPBELL-WALSH UROLOGY ed 11. Philadelphia Elsevier 2015 vol 3 chap 80 p 1883.

75. A 62-year-old man with bothersome LUTS has an AUA Symptom Score of 26 despite an adequate trial of an alpha-blocker and finasteride. DRE reveals a 40 gm benign prostate. His PSA six months ago was 2.3 ng/ml. Prior to laser prostatectomy the next step is: 26 A.. B .. C. D .. E .. urinalysis. repeat PSA. u roflowmetry. cystoscopy. pressure flow urodynamics.

question #75 ANSWER=A The only recommended test prior to surgery beyond those already mentioned is a urinalysis. A positive urinalysis may trigger other testing. PSA was normal within the last year and need not be repeated. Cystoscopy uroflowmetry and postvoid residual testing are all optional. Cystoscopy may be appropriate if the size of the prostate is in doubt particularly if it may be too large for endoscopic management. Uroflowmetry although not specific may be a reasonable indicator of bladder outlet obstruction. Pressure flow testing is the best assessment for outlet obstruction but is costly invasive and not recommended routinely unless the diagnosis is in doubt (for example younger men with small prostates and severe LUTS or if there is concern for neurogenic detrusor dysfunction). Welliver C McVary KT: Minimally invasive and endoscopic management of benign prostatic hyperplasia in Wein AJ Kavoussi LR Partin AW Peters CA (eds): CAMPBELL-WALSH UROLOGY ed 11. Philadelphia Elsevier 2015 vol 3 chap 105 p 2509.

78. A 52-year-old woman underwent a percutaneous needle biopsy of the right kidney. Two months later she has hypertension. Upon examination an abdominal bruit is heard in the right upper quadrant. A renal arteriogram demonstrates an arteriovenous fistula in the lower pole of the right kidney. She is asymptomatic with blood pressure well-controlled by medication and has a serum creatinine of 1.4 mg/dl. The next step is: A.. observation. B.. selective embolization. C. partial nephrectomy. D.. operative ligation. E.. nephrectomy.

question #78 ANSWER=A Most traumatic arterio-venous (AV) fistulas of the kidney such as those caused by percutaneous biopsy are asymptomatic small and will close spontaneously without intervention. Symptomatic fistulas can cause poorly controlled hypertension persistent hematuria or high-output heart failure. Symptomatic AV fistulas can be managed by embolization operative ligation of the feeding vessels or partial/complete nephrectomy depending upon their size and location. This patient's hypertension is well-controlled with medication and there is therefore no indication for intervention. Badlani GH De Ridder DJMK Mettu JR Rovner ES: Urinary tract fistulae in Wein AJ Kavoussi LR Partin AW Peters CA (eds): CAMPBELL-WALSH UROLOGY ed 11. Philadelphia Elsevier 2015 vol 3 chap 89 p 2137.

79. A 71-year-old man with a PSA of 14 ng/ml undergoes radical prostatectomy for cT2aNOMO Gleason score 8 (4+4) prostate cancer. Final pathology is Gleason 7 (4+3) with negative margins however seminal vesicle and bladder neck invasion is found. In this patient the feature associated with the highest risk of recurrence is: A.. PSA. B.. biopsy Gleason score. C. seminal vesicle invasion. D.. bladder neck invasion. E.. prostatectomy Gleason score.

question #79 ANSWER=( The presence of seminal vesicle invasion is the highest risk feature of this case scenario. Although bladder neck invasion is assigned to pathologic T4 status it has not been associated with any independent increased risk of recurrence following radical prostatectomy. The pre-operative PSA and final Gleason score in this case are categorized as intermediate risk in the D' Amico classification system. In cases of down grading from Gleason 8 to Gleason 7 on final pathologic analysis the final pathologic grade is most closely associated with risk of relapse. Yossepowitch 0 Sircar K Scardino PT et al: Bladder neck involvement in pathological stage pT4 radical prostatectomy specimens is not an independent prognostic factor. J UROL 2002;168:2011- 2015. Epstein JI: Pathology of prostatic neoplasia in Wein AJ Kavoussi LR Partin AW Peters CA (eds): CAMPBELL-WALSH UROLOGY ed 11. Philadelphia Elsevier 2015 vol 3 chap 110 p 2605.

80. A ten-year-old boy has nocturnal enuresis daytime dampness and two febrile UTls. Treatment with antibiotics fails to resolve his incontinence. An ultrasound reveals no hydronephrosis. KUB reveals no bladder stones voiding and postvoid films from a VCUG are shown. The next step is timed voiding and: A.. B .. C D .. E .. retrograde urethrogram. urodynamics. a-blockers. antimuscarinic therapy. endoscopic valve ablation.

question #80 ANSWER=B The postvoid films show missing sacral segments and abnormal lumbar vertebra. The cystogram shows a trabeculated bladder with no reflux. The voiding films show a dilated prostatic urethra as well as dilation of the bu I bar urethra. However there are no urethral valves so valve ablation is not needed. Given the bony abnormalities urodynamics and an MRI scan of the lumbar sacral spine to rule-out a coexisting tethered spinal cord are the next diagnostic steps. Starting medications are indicated only after a baseline urodynamic study is performed. The appearance of the urethra is secondary to detrusor sphincter dyssynergia. The patient does not have a bu I bar urethral stricture; therefore a retrograde urethrogram is not indicated. Maclellan DL Bauer SB: Neuromuscular dysfunction of the lower urinary tract in children in Wein AJ Kavoussi LR Partin AW Peters CA (eds): CAMPBELL-WALSH UROLOGY ed 11. Philadelphia Elsevier 2015 vol 4 chap 142 pp 3286-3289.

81. A 55-year-old man has a PSA of 1.7 ng/ml. His DRE reveals a suspicious prostatic nodule consistent with a cT3 prostate cancer. TRUS-guided prostatic biopsies reveal small cell carcinoma of the prostate. Metastatic evaluation is negative. The next step is: A.. systemic chemotherapy. B.. hormonal therapy. C. XRT. D.. neoadjuvant and concurrent hormonal therapy and XRT. E.. radical prostatectomy and bilateral pelvic lymph node dissection.

question #81 ANSWER=A Small cell carcinoma of the prostate is rare but associated with a high likelihood of metastatic disease at diagnosis and poor prognosis after treatment. Radical prostatectomy is not associated with good outcomes. Small cell carcinoma of the prostate does not secrete PSA significantly and is apparently androgen resistant. Systemic chemotherapy is thought to be the most effective strategy followed by or concurrent with radiation therapy. Chemotherapy agents used are similar to those used in patients with other small cell carcinoma (e.g. lung) with combinations of cisplatin and etoposide or paclitaxel or docetaxel and topotecan. Epstein JI: Pathology of prostatic neoplasia in Wein AJ Kavoussi LR Partin AW Peters CA (eds): CAMPBELL-WALSH UROLOGY ed 11. Philadelphia Elsevier 2015 vol 3 chap 110 p 2599.

82. The most appropriate peri-operative management of a patient undergoing adrenalectomy for Cushing's syndrome is: A.. hydration alpha-blockers and stress-dose steroids. B.. beta-blockers stress-dose steroids and careful glycemic control. C. potassium sparing diuretics and stress-dose steroids. D.. stress-dose steroids and careful glycemic control. E.. potassium sparing diuretics stress-dose steroids and careful glycemic control.

question #82 ANSWER=D Patients undergoing adrenalectomy for Cushing's syndrome have an excess of corticosteroids from an adrenal adenoma or carcinoma. These patients need stress-dose steroids and careful glycemic control as they often have obesity and diabetes. Alpha-blockers and hydration are indicated peri-operatively for patients with pheochromocytoma. Beta-blockers may also be necessary pre-operatively for patients with pheochromocytoma if they are tachycardic after alpha-blockade. Potassium-sparing diuretics are important for the peri-operative management of patients with hyperaldosteronism (Conn's disease) as they often have significant hypokalemia. Kutikov A Crispen PL Uzzo RG: Pathophysiology evaluation and medical management of adrenal disorders in Wein AJ Kavoussi LR Partin AW Peters CA (eds): CAMPBELL-WALSH UROLOGY ed 11. Philadelphia Elsevier 2015 vol 2 chap 65 p 1581.

83. A 48-year-old man with VHL has a 2 cm solid right adrenal mass in addition to multiple small (1-2 cm) bilateral renal masses. He is not hypertensive but serum catecholamines are slightly elevated and the adrenal mass is bright on T2-weighted MRI scan. The next step is alpha-blockade and: A.. observation. B.. percutaneous biopsy of the adrenal mass. C. partial adrenalectomy. D.. adrenalectomy. E.. adrenalectomy and partial nephrectomy.

question #83 ANSWER=C The main indications for partial adrenalectomy are solitary adrenal gland bilateral disease and patients with familial syndromes. Pheochromocytoma has been treated with partial adrenalectomy especially in patients with VHL familial pheochromocytoma or multiple endocrine neoplasia type 2a. Observation and biopsy are not appropriate for patients with pheochromocytoma. Open adrenalectomy and partial nephrectomy may be appropriate for patients with larger renal masses(> 3 cm). This patient will likely require multiple surgeries on both kidneys and adrenals so the maximum functioning renal and adrenal tissue should be preserved. Kutikov A Crispen PL Uzzo RG: Pathophysiology evaluation and medical management of adrenal disorders in Wein AJ Kavoussi LR Partin AW Peters CA (eds): CAMPBELL-WALSH UROLOGY ed 11. Philadelphia Elsevier 2015 vol 2 chap 65 p 1552. Copyright 2017 by The American Urological Association. 29

84. A healthy four-year-old boy has a one-month history of voiding every 15-20 minutes during the day. He is continent denies nocturia and has not had any UTls. He has normal daily bowel movements. Urinalysis is normal. The next step is: A.. reassurance. B.. VCUG. C. urodynamic study. D.. oxybutynin. E.. cystoscopy.

question #84 ANSWER=A This is a typical presentation of pediatric benign daytime urinary frequency syndrome. The etiology of this disorder is unclear but often follows a systemic illness. Treatment is with parental reassurance maintenance of a voiding diary and behavioral modification therapy to reward for progressive lengthening of the voiding interval. Resolution of symptoms will invariably occur within a few months. Antimuscarinic agents are rarely helpful. Imaging and urodynamic studies do not yield any significant findings. In the presence of a normal urinalysis cystoscopic evaluation is not indicated. Zoubek J Bloom DA Sedman AB: Extraordinary urinary frequency. PED 1990;85:1112-1114. Austin PF Vricella GJ: Functional disorders of the lower urinary tract in children in Wein AJ Kavoussi LR Partin AW Peters CA (eds): CAMPBELL-WALSH UROLOGY ed 11. Philadelphia Elsevier 2015 vol 4 chap 143 p 3310.

85. The most likely location for corporal perforation during penile prosthesis placement is: 30 A.. B .. C. D .. E .. dorsally. ventrally. at the septum. distally (near glans). proximally (near insertion).

question #85 ANSWER=C The corporal septum is the weakest portion of the corporal body. This is of particular importance during insertion of a penile prosthesis as the corporal septum may be perforated and an unrecognized cylinder cross over may occur after septal perforation. This complication may be avoided by placing the penis on traction aiming the tip of the dilator laterally and placing a spacer in the contralateral corpora while its mate is dilated. Thus the other locations described (dorsally ventrally distally and proximally) would be incorrect. Lue TF: Physiology of penile erection and pathophysiology of erectile dysfunction in Wein AJ Kavoussi LR Partin AW Peters CA (eds): CAMPBELL-WALSH UROLOGY ed 11. Philadelphia Elsevier 2015 vol 1 chap 26 p 614.

86. The abdomi raph in a 60-year-old man is shown. The condition that is most commonly associated with this finding is: A.. sarcoidosis. B.. diabetes mellitu. C. uncontrolled hypertension. D.. tuberculosis. E.. milk-alkali syndrome.

question #86 ANSWER=B The paired calcified tubular structures in the center of the pelvis are the vas deferens. Calcification of the vas deferens is virtually pathognomonic for diabetes mellitus. Although tuberculosis can on rare occasions have similar findings. This patient has also had a left iliac fossa renal transplant which contains a radio-opaque urinary stone. Adam A Dixon AK Gillard JH et al: Male genitourinary tract in Adam A Dixon AK Gillard JH et al (eds): GRAINGER AND ALLISON'S DIAGNOSTIC RADIOLOGY ed 6. London Churchill Livingstone 2014 chap 40 p 944.

87. A recurrent calcium oxalate stone former has an isolated finding of marked hypomagnesuria on metabolic evaluation. The next step is: A.. discontinuation of allopurinol. B.. evaluate for an underlying bowel disorder. C. oral magnesium supplementation. D.. increase intake of tofu and brown rice. E.. potassium citrate.

question #87 ANSWER=B The most common cause of hypomagnesuria is inflammatory bowel disease and an appropriate referral to a gastroenterologist to rule-out this disorder should be performed. The benefit of oral magnesium supplementation in the management of hypomagnesuric calcium nephrolithiasis has not been well-established. Though tofu and brown rice are rich in magnesium the impact of 30 Copyright 2017 by The American Urological Association. dietary intervention on hypomagnesuria has not been tested. Though increased fluid intake is an empiric measure to decrease the risk of stone formation it will not address this specific metabolic abnormality. Allopurinol does not impact urinary magnesium levels. Lipkin ME Ferrandino MN Preminger GM: Evaluation and medical management of urinary lithiasis in Wein AJ Kavoussi LR Partin AW Peters CA (eds): CAMPBELL-WALSH UROLOGY ed 11. Philadelphia Elsevier 2015 vol 2 chap 52 p 1228.

88. A newborn girl has a history of prenatal bilateral moderate hydronephrosis (anterioposterior pelvic diameter of 8 mm) without ureteral dilation. On day two of life ultrasound reveals no hydronephrosis. The next step is: A.. no further evaluation needed. B.. serum creatinine at one week. C. repeat ultrasound at six weeks of age. D.. VCUG. E.. radionuclide renal scan.

question #88 ANSWER=( This is a common clinical scenario. Newborns are relatively oliguric for the first 48 hours of life primarily due to neonatal decreased renal blood flow and low glomerular filtration rates of the immature neonatal kidney. The normal neonatal renal physiology is associated with diminished urine production and may lead to either an underestimation of the severity of hydronephrosis and/or to the presence of a II normal II renal ultrasound shortly following birth. For this reason it is always recommended that the child with mild to moderate antenatal hydronephrosis have a follow-up neonatal ultrasound two to six weeks following birth. There has been some debate about whether such patients also deserve a VCUG to rule-out vesicoureteral reflux. The 2010 AUA Guideline on reflux management however does not recommend a VCUG in patients with mild to moderate antenatal hydronephrosis unless the postnatal ultrasound reveals a dilated ureter or the presence of coexisting renal anomalies that would suggest the possibility of reflux. Serum creatinine is unlikely to be helpful in cases of mild hydronephrosis and renal scan is not indicated unless moderate or severe hydronephrosis is actually documented to be present via ultrasound. Lee RS Borer JG: Perinatal urology in Wein AJ Kavoussi LR Partin AW Peters CA (eds): CAMPBELL-WALSH UROLOGY ed 11. Philadelphia Elsevier 2015 vol 4 chap 124 pp 2888-2889.

89. A three-year-old boy with a history of daytime wetting and recurrent infections is found to have bilateral grade 3 VUR. The most important factor in predicting risk of breakthrough UTI is: A.. age. B.. intrarenal reflux. C. renal scarring. D.. circumcision status. E.. bladder and bowel dysfunction.

question #89 ANSWER=E The management of reflux has become extremely controversial. Although there is improvement in the evidence-based literature evaluating reflux there is still a lack of data to definitively establish the role of antibiotics and surgery in the management of low to moderate grade reflux. However the relationship between bladder and bowel dysfunction (BBD) and reflux is now widely recognized and accepted. When present it is the overriding factor that most affects the incidence of recurrent infections spontaneous resolution of reflux and successful surgical correction of reflux. When BBD is recognized to be present it needs to be treated aggressively given its effect on both the medical and surgical management of reflux. While all of the other listed factors may impact resolution rates of reflux (age) infection (circumcision status) and susceptibility to develop renal scarring (intrarenal reflux) none of them have the impact of BBD collectively. Peters CA Skoog SJ Arant BS Jr et al: AUA GUIDELINE FOR MANAGEMENT AND SCREENING OF PRIMARYVESICOURETERAL REFLUX IN CHILDREN. American Urological Association Education and Research Inc 201 O; 184. http://www.auanet.org/education/guidelines/vesicoureteral-reflux-a.cfm Khoury AE Bagli DJ: Vesicoureteral reflux in Wein AJ Kavoussi LR Partin AW Peters CA (eds): CAMPBELL-WALSH UROLOGY ed 11. Philadelphia Elsevier 2015 vol 4 chap 137 p 3134. Copyright 2017 by The American Urological Association. 31

90. A 55-year-old patient with a mechanical aortic valve and penicillin allergy is scheduled to undergo urodynamic testing. Urinalysis on the day of testing is unremarkable. The indicated antibiotic prophylaxis is: A.. none. B.. ciprofloxacin. C. vancomycin and gentamicin. D.. clindamycin and gentamicin. E. trimethoprim-sulfamethoxazole.

question #90 ANSWER=A The AUA Best Practice Statement on urologic surgery antimicrobial prophylaxis states that antimicrobial prophylaxis is not indicated prior to urodynamic testing unless specific risk factors exist. These risk factors include: advanced age anatomic abnormalities of the urinary tract poor nutritional status and immunodeficiency. Furthermore the American Heart Association does not recommend the use of antimicrobial prophylaxis prior to any urologic procedure solely for the prevention of infectious endocarditis. Therefore this patient does not require antimicrobial prophylaxis prior to undergoing urodynamic testing. Wolf JS Jr Bennett CJ Dmochowski RR et al: BEST PRACTICE POLICY STATEMENT ON UROLOGICAL SURGERY ANTIMICROBIAL PROPHYLAXIS. American Urological Association Education and Research Inc 2008. (Updated January 2014) http://www.auanet.org/education/guidelines/antimicrobial-prophylaxis.cfm

91. The most potent stimulator of aldosterone secretion is: 32 A.. B .. C. D .. E .. ACTH. angiotensin II. renin. potassium. sodium.

question #91 ANSWER=B The most potent stimulator of aldosterone secretion is angiotensin II. The juxtaglomerular apparatus is sensitive to renal perfusion. Decreased perfusion stimulates renin secretion which is converted in the lungs to angiotensin II and stimulates the secretion of aldosterone. Aldosterone secretion is also under the influence of both ACTH and potassium but they are secondary influences. Sodium has no direct influence other than through volume expansion and contraction. Kutikov A Crispen PL Uzzo RG: Pathophysiology evaluation and medical management of adrenal disorders in Wein AJ Kavoussi LR Partin AW Peters CA (eds): CAMPBELL-WALSH UROLOGY ed 11. Philadelphia Elsevier 2015 vol 2 chap 65 p 1530.

92. A 53-year-old woman has recurrent stress incontinence despite two previous midurethral slings and a urethral bulking injection. A videourodynamic study shows no detrusor overactivity with a maximal bladder capacity of 300 ml. Stress urinary incontinence is documented with a Valsalva LPP of 22 cm H20 at 200 ml and again at maximum capacity. The urethra has minimal mobility with straining. The next step is: A.. pelvic floor muscle exercises with biofeedback. B.. off-label imipramine. C. sacral neuromodulation. D.. retropubic mid-urethral polypropylene sling. E.. autologous pubovaginal sling.

question #92 ANSWER=E This patient presents with a complex case of recurrent stress urinary incontinence (SUI) in spite of previous treatment a fixed urethra and a low Valsalva LPP. The only reasonable option presented would be an autologous sling. Pelvic floor exercises would be unlikely to successfully address her symptoms. lmipramine can be considered for off-label utilization for mixed incontinence but is also unlikely to cure this degree of SUI. Sacral neuromodulation is primarily indicated for urinary urgency and/or urge incontinence and is not indicated for the treatment of stress incontinence. Of the two sling options presented the autologous pubovaginal sling is the better option for recurrent intrinsic sphincter deficiency particularly in the face of a fixed urethra. Dmochowski RR Osborn DJ Reynolds WS: Slings: Autologous biologic synthetic and midurethral in Wein AJ Kavoussi LR Partin AW Peters CA (eds): CAMPBELL-WALSH UROLOGY ed 11. Philadelphia Elsevier 2015 vol 3 chap 84 pp 2020-2021. 32 Copyright 2017 by The American Urological Association.

93. A 15-year-old boy has right flank pain after being struck in the back with a lacrosse stick during a match five hours ago. Vital signs are normal. Urinalysis reveals clear urine with 10-20 RBCs and hemoglobin is 15 g/dl. The next step is serial physical exams and: A.. repeat urinalysis. B.. abdominal ultrasound. C. single phase CT scan of abdomen. D.. triphasic CT scan of abdomen. E.. MRI scan.

question #93 ANSWER=B Radiographic assessment for possible genitourinary trauma is required for all symptomatic pediatric patients that have sustained blunt trauma with microscopic hematuria. A normal Focused Assessment with Sonography for Trauma (FAST) exam and serial physical exams for 24 hours will nearly rule-out all significant renal injuries and spare the patient radiation. CT scan with delayed views is recommended if the FAST exam is abnormal or gross hematuria is present. MRI scan is not indicated in the evaluation of trauma. Husmann DA: Pediatric genitourinary trauma in Wein AJ Kavoussi LR Partin AW Peters CA (eds): CAMPBELL-WALSH UROLOGY ed 11. Philadelphia Elsevier 2015 vol 4 chap 154 pp 3538- 3539.

94. Patients with type 2 (proximal) RTA do not form renal calculi because they differ from those with type 1 (distal) RTA in the renal handling of: A.. calcium. B.. citrate. C. bicarbonate. D.. phosphate. E.. sodium.

question #94 ANSWER=B Patients with distal or type 1 RTA have a basic defect in eliminating H+ from the distal tubule. Associated urinary changes include both an increase in calcium and a distinct decrease in citrate levels. This combination results in most type 1 patients forming renal calculi. In distinction type 2 RTA is caused by diminished bicarbonate reabsorption in the proximal tubule. While this defect also results in high urinary calcium levels for reasons that are not fully understood it does not lower urinary citrate and thus these patients do not commonly form renal calculi. Sodium and phosphate handling are altered in both forms of RTA but do not seem to have much impact on stone formation. In patients with type 1 RTA the administration of potassium citrate is the mainstay of attempting to minimize nephrocalcinosis and the formation of renal calculi. Lipkin ME Ferrandino MN Preminger GM: Evaluation and medical management of urinary lithiasis in Wein AJ Kavoussi LR Partin AW Peters CA (eds): CAMPBELL-WALSH UROLOGY ed 11. Philadelphia Elsevier 2015 vol 2 chap 52 pp 1211-1212.

98. A 48-year-old woman has an incidentally discovered 3 cm right adrenal mass on ultrasound. The next step is: A.. non-contrast CT scan. B.. contrast CT scan washout study. C. gadolinium-enhanced MRI scan washout study. D.. 18F-dopamine PET scan. E.. 131 I-MIBG radionucleotide imaging.

question #98 ANSWER=A Adrenal incidentalomas are unsuspected adrenal masses > 1 cm in diameter identified on imaging performed for seemingly unrelated causes. Ultrasound is a suboptimal imaging modality for detecting and fully characterizing adrenal lesions. Nevertheless many incidentalomas will be discovered on ultrasound imaging performed for unrelated reasons. An unenhanced CT scan is the first and perhaps single best and most easily interpreted test for intracellular lipid and can diagnose an adrenal adenoma in more than 70% of cases. Low attenuation (< 10 Hounsfield 34 Copyright 2017 by The American Urological Association. units (HU)) on unenhanced CT scan corresponds to high intracytoplasmic lipid content and is diagnostic for an adrenal adenoma. Ninety-eight percent (98%) of lesions with an attenuation of 10 HU or less on non-contrast CT scan are adrenal adenomas while less than 30% of adrenal adenomas are lipid-poor (also known as atypical adenomas) and have an attenuation of > 10 HU. If a lesion demonstrates an attenuation of >10 HU then additional radiological evaluation can be performed including CT washout study to help discriminate lipid-poor adenomas from other adrenal lesions. Gadolinium-enhanced MR washout studies do not exhibit the diagnostic strength of iodine-based CT washout studies and are not commonly employed. When MRI scan is used opposed phase chemical-shift MR imaging to evaluate for intracellular lipid content can help distinguish an adenoma from other adrenal lesions. Functional studies such as PET imaging and an MIBG scan (used to evaluate for pheochromocytomas) are not indicated in the initial evaluation of an adrenal incidentaloma. Kutikov A Crispen PL Uzzo RG: Pathophysiology evaluation and medical management of adrenal disorders in Wein AJ Kavoussi LR Partin AW Peters CA (eds): CAMPBELL-WALSH UROLOGY ed 11. Philadelphia Elsevier 2015 vol 2 chap 65 p 1557.

99. A ten-year-old boy has microscopic hematuria after treatment of a febrile upper respiratory infection. Repeat urinalyses two weeks and three months later are normal except for 5-8 RBOhpf. His serum creatinine is 0.6 mg/dl and a renal ultrasound is normal. A fasting spot urine calcium:creatinine ratio is elevated. The next step is: A.. cystoscopy. B.. C3 and ASO titers. C. non-contrast CT scan. D.. 24-hour urine calcium. E.. renal biopsy.

question #99 ANSWER=D Hematuria is one of the most common genitourinary abnormalities in children. An association with hypercalciuria and hematuria in children is well-documented. Calcium excretion exceeding 4 mg/kg is considered abnormal. In children the collection of 24-hour urine can be difficult. A spot urine calcium:creatinine ratio can be used for screening but hypercalciuria must be confirmed with a 24-hour urine calcium. A fasting level> 0.21 or a post-prandial level> 0.28 are abnormal. It is unclear how the hematuria is produced by the hypercalciuria. However these children are at risk for subsequent urolithiasis which has been reported in over ten percent of individuals. The imaging evaluation should include a renal ultrasound to exclude both calculi and structural abnormalities. Further imaging studies are not warranted. C3 and ASO titers are unwarranted as this child has no red cell casts or proteinuria. The yield of cystoscopy in this pediatric population is very low. Evaluation for renal inflammatory disease or parenchymal abnormality by biopsy or contrast imaging is very low in the absence of significant proteinuria. Norwood VF Peters CA: Disorders of renal functional development in children in Wein AJ Kavoussi LR Partin AW Peters CA (eds): CAMPBELL-WALSH UROLOGY ed 11. Philadelphia Elsevier 2015 vol 4 chap 123 pp 2852-2854. Garcia D Miller L Stapleton FB: Natural history of hematuria associated with hypercalciuria in childhood: AJDC 1991;145:1204-1207.


Ensembles d'études connexes

02.07 Review and Practice Exam for Geometry

View Set

Organizational Behavior Ch 1,2,3,4,5,6

View Set

UCONN Anthropology 1000 Final Exam

View Set